Download as pdf or txt
Download as pdf or txt
You are on page 1of 311

‫محطة البكالوريات التجريبية‬

‫ال ِقسم األو ْل ‪I :‬‬

‫مالحظة ‪ :‬هذه الباقة خاصة بتالميذ شعبتي رياضيات و تقني رياضي يمكن‬
‫لتالميذ شعبة علوم تجريبية االستفادة منها حسب المطلوب ‪،،‬‬

‫تحت شعار ‪:‬‬


‫اجلمهورية اجلزائرية الدميقراطية الشعبية‬
‫وزارة الرتبية الوطنية‬
‫دورة ماي ‪2023‬‬ ‫الشعبة‪ :‬تقين رايضي ‪ -‬رايضيات‬
‫املدة‪4 :‬سا و‪30‬د‬ ‫بكالوراي جترييب ف مادة العلوم الفيزايئية‬

‫على المترشح أن يختار أحد الموضوعين‪:‬‬


‫يحتوي الموضوع األول على ‪ 5‬صفحات (من الصفحة ‪ 01‬إلى الصفحة ‪)05‬‬
‫التمرين األول‪ 04( :‬نقاط)‬
‫مرت عدة سنوات على التفجيرات والتجارب النووية للمستعمر الفرنسي التي ما تزال‬
‫تخلف ضحايا في الجنوب الجزائري الشاسع من بين السكان الشباب الذين يعانون‬
‫من مشكلتين وهما‪ :‬تجاهل القوة االستعمارية القديمة تماما لهم وتأثيرات اإلشعاع‪.‬‬
‫في يوم االنفجار الموافق لتاريخ ‪ 13‬فيفري ‪ 1960‬سجل المستعمر الفرنسي دخوله‬
‫المدوي إلى نادي القوى النووية مخلفا وراءه بالحمودية نفايات نووية ملقاة فوق األرض التي ال زالت بعد نصف قرن تخلف‬

‫) ‪N (t‬‬ ‫ضحايا لها‪.‬‬


‫الشكل‪1.‬‬
‫‪N0‬‬ ‫أهم النظائر المؤثرة على الصحة بعد انفجار نووي‪.‬‬ ‫‪131‬‬
‫يعتبر نظير اليود ‪I‬‬
‫في حالة‬ ‫‪A‬‬
‫‪Z‬‬ ‫مشعة‪ ،‬ينتج عن تفككها نواة الكزينون ‪X e‬‬ ‫‪131‬‬
‫‪53‬‬‫‪ .1‬نواة اليود ‪I‬‬
‫‪.‬‬ ‫إثارة وفق نمط تفكك‬
‫‪−‬‬

‫‪ .1.1‬اكتب معادلة التفاعل النووي‪.‬‬


‫‪ .2.1‬عند رجوع النواة الناتجة المثارة إلى حالتها األساسية تبعث أشعة‬
‫كهرومغناطيسية شديدة النفاذية‪.‬‬
‫‪0,2‬‬ ‫اكتب معادلة تحوله النووي‪ ،‬مع تحديد نوع النشاط اإلشعاعي‪.‬‬ ‫‪-‬‬
‫و ‪ ، ZA X e‬ثم حدد أي‬ ‫‪ .2‬أحسب طاقة التماسك ‪ El‬لكل من النواتين ‪I‬‬
‫‪131‬‬
‫‪53‬‬
‫‪0‬‬ ‫‪4‬‬ ‫) ‪t ( jours‬‬ ‫النواتين أكثر استق اررا‪.‬‬
‫) ‪N (t‬‬
‫الممثل في الشكل‪.1.‬‬ ‫‪ ، 131‬نستعمل المنحنى البياني ) ‪= f ( t‬‬
‫‪ .3‬من أجل تحديد زمن نصف العمر لنواة ‪53 I‬‬
‫‪N0‬‬
‫‪ .1.3‬اكتب عبارة قانون التناقص االشعاعي ) ‪ N (t‬بداللة ‪  ، N 0‬و ‪. t‬‬
‫‪ .2.3‬عرف زمن نصف العمر ‪ ، t1/2‬ثم حدده بيانيا‪.‬‬
‫‪. A0‬‬ ‫‪ 131‬فقط‪ ،‬بحيث كان النشاط االشعاعي آنذاك ‪= 1,76  109 GBq‬‬
‫‪ .3.3‬نعتبر أن المنطقة تلوثت إشعاعيا باليود ‪53 I‬‬

‫حدد التاريخ الذي نعتبر فيه أن هذه المنطقة أصبحت غير ملوثة باليود المشع‪ .‬نعتبر أن منبعا غير فعال عندما يبلغ‬ ‫‪-‬‬
‫نشاطها اإلشعاعي ‪.1Bq‬‬
‫المعطيات‪1GBq = 109 Bq ، 1an = 365 jour :‬‬

‫) (‬
‫‪m 131‬‬ ‫‪131‬‬
‫(‬ ‫)‬ ‫‪1‬‬ ‫‪1‬‬
‫) (‬
‫‪53 I = 130,87760 u ; m 54 Xe = 130,87594 u ; m 0 n = 1,00866 u ; m 1 p = 1,00728 u‬‬ ‫) (‬
‫صفحة ‪ 1‬من ‪10‬‬
‫بكالوراي جترييب ف مادة العلوم الفيزايئية ‪ /‬الشعبة‪ :‬تقين رايضي ‪ -‬رايضيات ‪ /‬دورة ماي ‪2023‬‬
‫التمرين الثاني‪ 04( :‬نقاط)‬
‫𝟐𝑲 𝟏‬ ‫الشكل ‪2‬‬
‫تحتوي كثير من األجهزة االلكترونية مثل التلفزيونات‪ ،‬الهواتف‬
‫والساعات االلكترونية في تراكيبها على مكثفات‪.‬‬
‫𝑪‬
‫𝑬‬ ‫𝟐𝑹‬ ‫يهدف التمرين إلى دراسة شحن وتفريغ مكثفة‪.‬‬
‫𝟏𝑹‬ ‫نعتبر التركيب الكهربائي الممثل في الشكل‪ 2.‬والمتكون من‪:‬‬
‫مولد ذو توتر كهربائي ثابت قوته المحركة الكهربائية ‪. E‬‬ ‫‪-‬‬
‫مكثفة سعتها ‪ C = 10  F‬غير مشحونة‪.‬‬ ‫‪-‬‬
‫) ‪u R1 ( t‬‬ ‫الشكل ‪3‬‬ ‫ناقليين أوميين مقاومتهما ‪ R1‬و ‪. R2‬‬ ‫‪-‬‬
‫بادلة ‪. K‬‬ ‫‪-‬‬
‫في اللحظة ‪ t = 0‬نضع البادلة في الوضع )‪،(1‬‬
‫وعند اللحظة ‪ t1‬نغير وضع البادلة إلى الوضع )‪.(2‬‬
‫بواسطة جهار راسم االهتزاز ذو ذاكرة نتابع تطور‬
‫التوتر ‪ uR1‬بين طرفي الناقل األومي ‪ R1‬فنحصل‬
‫‪1‬‬
‫على المنحنيات الممثلة في الشكل‪.3.‬‬
‫‪0‬‬ ‫) ‪t ( ms‬‬
‫دراسة شحن المكثفة‪:‬‬ ‫‪-‬‬
‫‪ .1‬أعد رسم الدارة الكهربائية مع التمثيل بأسهم اتجاه‬
‫التيار الكهربائي ‪ i‬والتوترات ‪ uC‬و ‪. uR1‬‬
‫‪ .2‬بين كيف يمكن ربط راسم االهتزاز ذو ذاكرة‬
‫للحصول على التوتر ‪ uR1‬بين طرفي الناقل األومي ‪. R1‬‬
‫‪ .3‬بتطبيق قانون جمع التوترات‪ ،‬جد المعادلة التفاضلية التي يحققها التوتر ‪ uR1‬خالل عملية الشحن‪.‬‬

‫‪ .4‬باعتبار أن حل المعادلة التفاضلية يكتب على الشكل التالي‪u R1 ( t ) = A.e− .t + B :‬‬


‫حيث كل من ‪ B ، A  0‬و ‪ ‬ثوابت‪ ،‬حدد عبارة كل منها بداللة ثوابت الدارة الكهربائية‪.‬‬
‫‪ .5‬باالعتماد على الشكل‪ ،3.‬جد قيمتي ‪ E‬و ‪. R1‬‬
‫دراسة تفريغ المكثفة‪:‬‬ ‫‪-‬‬
‫المعادلة التفاضلية التي يحققها التوتر ‪ uC‬بين طرفي المكثفة خالل التفريغ هي‪+ uC = 0 :‬‬
‫‪duC‬‬
‫‪( R1 + R2 )  C ‬‬
‫‪dt‬‬
‫‪ .1‬حل هذه المعادلة يكتب على الشكل التالي‪uC ( t ) = A.e ( 1 ) :‬‬
‫‪− ' t −t‬‬

‫‪ .1.1‬جد عبارة ) ‪ uC ( t‬بداللة ‪  2‬و ‪ E‬محددا عبارة ‪ ، 2‬ثم استنتج عبارة )𝑡( ‪ 𝑢𝑅2‬بداللة ‪، E ، 2 ، t1‬‬
‫‪ R1‬و ‪. R2‬‬
‫‪ .2.1‬أحسب قيمة ‪. R2‬‬
‫‪ .2‬أحسب قيمة ' ‪ C‬سعة مكثفة ثانية الذي يجب تركيبها مع المكثفة ‪ C‬في الدارة السابقة للحصول على ثابت‬
‫الزمن ‪. ' = 3 2‬‬
‫صفحة ‪ 2‬من ‪10‬‬
‫بكالوراي جترييب ف مادة العلوم الفيزايئية ‪ /‬الشعبة‪ :‬تقين رايضي ‪ -‬رايضيات ‪ /‬دورة ماي ‪2023‬‬
‫التمرين الثالث‪ 06( :‬نقاط)‬
‫تعتبر رياضة التزلج من أفضل الرياضات الجبلية في فصل الشتاء‪ ،‬فهي تجمع بين‬
‫المغامرة وبناء اللياقة البدنية والرشاقة‪.‬‬
‫يهدف التمرين إلى دراسة حركة مركز عطالة متزلج ولوازمه على حلبة للتزلج‪.‬‬
‫ينزلق متزلج كتلته ‪ m = 80 kg‬والذي نعتبره كنقطة مادية‪ ،‬على مسار يتشكل من ثالثة أجزاء‪( :‬الشكل‪)4.‬‬
‫‪ -‬الجزء ‪ : AB‬قوس دائري جليدي االحتكاك فيه مهمل‪ ،‬نصف قطره ‪ r = 5 m‬ومركزه ‪ O‬بحيث ‪.  = 60‬‬
‫‪ -‬الجزء ‪ : BC‬مسار مستقيم أفقي مغطى بالثلج طوله ‪. l = 5 m‬‬
‫‪ -‬الجزء ‪ : CD‬منحدر مغطى بالثلج طوله ‪ L = 40 m‬يميل عن األفق بزاوية ‪. ‬‬
‫‪g = 9,8 m.s −2‬‬ ‫االحتكاكات مع الهواء مهملة‪.‬‬
‫‪ -‬المحاولة األولى‪:‬‬
‫‪O‬‬ ‫ينطلق المتزلج من النقطة ‪ A‬بدون سرعة ابتدائية‪ ،‬ليصل‬
‫الشكل ‪4‬‬
‫إلى الموضع ‪ ، B‬فيواصل الحركة على المسار األفقي ‪BC‬‬
‫المغطى بالثلج حتى تنعدم سرعته عند الموضع ‪، C‬أين‬
‫‪A‬‬
‫يالقي المنحدر المائل ‪. CD‬‬
‫‪B‬‬ ‫‪C‬‬ ‫سمحت دراسة حركة المتزلج على المسار ‪ BCD‬برسم‬
‫‪D‬‬ ‫تغيرات تسارع المتزلج ‪ a‬بداللة الزمن الممثل ببيان الشكل‪.5.‬‬

‫(‬
‫‪a m.s −1‬‬ ‫)‬ ‫الشكل ‪5‬‬ ‫‪ .1‬ما المقصود بنقطة مادية‪.‬‬
‫‪ .2‬بتطبيق مبدأ انحفاظ الطاقة للجملة (متزلج ولوازمه) بين الموضعين‬
‫‪ A‬و ‪ ، B‬بين أن‪vB = 2.g.r.(1 − cos  ) :‬‬

‫‪1‬‬ ‫‪ .3‬اعتمادا على الشكل‪ ،5.‬حدد قيمة تسارع المتزلج خالل كل مسار‬
‫‪ BC‬و ‪. CD‬‬
‫‪0‬‬ ‫)‪t (s‬‬
‫‪2‬‬ ‫‪ .4‬أحص‪ ،‬ثم مثل القوى المؤثرة على المتزلج خالل حركته على المسار‬
‫‪. BC‬‬
‫‪ .1.5 .5‬بتطبيق القانون الثاني لنيوتن على المتزلج خالل المسار ‪CD‬‬
‫‪f‬‬
‫‪aCD = g.sin  −‬‬ ‫‪ ،‬بين أن عبارة تسارعه يكتب من الشكل‪:‬‬
‫‪m‬‬
‫ثم استنتج عبارة تسارعه على المسار ‪. aBC‬‬
‫‪ .2.5‬أحسب شدة قوى االحتكاك ‪ ، f‬وقيمة زاوية المنحدر ‪. ‬‬
‫‪ .6‬يقطع المتزلج مسافة ‪ d ' = 5 m‬خالل حركته على المسار ‪ ، CD‬أحسب الزمن المستغرق لقطع هذه المسافة‬
‫وسرعته عندئذ‪.‬‬

‫صفحة ‪ 3‬من ‪10‬‬


‫بكالوراي جترييب ف مادة العلوم الفيزايئية ‪ /‬الشعبة‪ :‬تقين رايضي ‪ -‬رايضيات ‪ /‬دورة ماي ‪2023‬‬
‫‪ -‬المحاولة الثانية‪:‬‬
‫‪y‬‬
‫يعيد المتزلج محاولة أخرى لكن بتغيير ارتفاعه ليصل إلى الموضع ‪ C‬بسرعة‬
‫الشكل ‪6‬‬
‫‪. vC = 5 m.s −1‬‬
‫‪x‬‬ ‫يقفز المتزلج عند الموضع ‪ C‬بسرعة ‪ v C‬ليسقط على المستوي ‪ CD‬يميل‬
‫عن األفق بزاوية ‪( . ‬الشكل‪)6.‬‬
‫‪C‬‬

‫بتطبيق القانون الثاني لنيوتن على مركز عطالة المتزلج تحصلنا على‪:‬‬
‫‪D‬‬
‫‪vx = 5 m.s −1 ; v y = −9,8.t‬‬
‫‪ .1‬استخرج المعادالت الزمنية للموضع ) ‪ x ( t‬و ) ‪. y ( t‬‬
‫‪ .2‬أحسب ‪ tP‬زمن القفز‪.‬‬
‫‪ .3‬عين إحداثيتي نقطة السقوط ‪ ، P‬ثم استنتج المسافة ‪. CP‬‬
‫التمرين التجريبي‪ 06( :‬نقاط)‬
‫بيكاربونات أو هيدروجينوكربونات )‪ HCO3− (aq‬هو محلول منظم الدم‪ ،‬يعمل على‬
‫تنظيم قيمة الـ ‪ pH‬في الدورة الدموية‪ ،‬كما أنه يتواجد أيضا في المياه المعدنية والتي نستهلكها‬
‫بشكل دوري‪.‬‬
‫تلعب شوارد )‪ HCO3− (aq‬دور المحلول المتذبذب‪ ،‬عند تفاعله ينتج عنه شاردة الكربونات‬
‫) ‪ ، CO32− (aq‬أما في حاالت أخرى فينتج عنه ثنائي أكسيد الكربون المنحل في الماء حسب الصيغة الكيميائية‬
‫) ‪. ( CO2 , H 2O ) (aq‬‬
‫الجزء األول‪ :‬دراسة محلول مائي لهيدروجينو كربونات ) ‪HCO3− (aq‬‬ ‫‪-‬‬
‫(‬ ‫)‬
‫نتوفر على محلول مائي لهيدروجينوكربونات الصوديوم )‪ Na + (aq) + HCO3− (aq‬بتركيز المولي ‪ C0‬وحجمه‬

‫‪ ، V‬أعطى قياس ‪ pH‬المحلول القيمة ‪. 8,5‬‬


‫– تفاعل حمض – أساس‪.‬‬ ‫‪ -‬محلول متذبذب‪.‬‬ ‫‪ .1‬ما المقصود بـ‪:‬‬
‫‪ .2‬اكتب المعادلتين الموافقتين للتفاعلين الممكنين بين شاردة البيكاربونات )‪ HCO3− (aq‬والماء‪.‬‬
‫(‬ ‫)‬
‫‪ .3‬باالعتماد على عبارة ثابت الحموضة ‪ Ka‬للثنائية ‪ ، CO2 , H 2O / HCO3−‬جد العالقة التي تربط بين ‪، pH‬‬
‫‪ ‬‬ ‫وبين أنها تكتب على الشكل‪ :‬‬ ‫و‪‬‬
‫‪pH = pKa + log ‬‬ ‫‪‬‬ ‫‪pKa‬‬
‫‪ 100 −  ‬‬
‫‪ HCO −   100‬‬
‫‪ ، ( HCO3− ) = CO , H O‬حيث )‬
‫‪‬‬ ‫‪3‬‬
‫(‬
‫‪  HCO3−‬يمثل نسبة النوع القاعدي في‬
‫‪eq‬‬
‫‪+  HCO3− ‬‬
‫علما أن‬
‫‪2‬‬ ‫‪2‬‬ ‫‪eq ‬‬ ‫‪ eq‬‬
‫المحلول‪.‬‬

‫صفحة ‪ 4‬من ‪10‬‬


‫بكالوراي جترييب ف مادة العلوم الفيزايئية ‪ /‬الشعبة‪ :‬تقين رايضي ‪ -‬رايضيات ‪ /‬دورة ماي ‪2023‬‬
‫‪. log ‬‬ ‫‪‬‬ ‫‪ .4‬يمثل الشكل‪ 7.‬مخطط تغيرات الـ ‪ pH‬بداللة ‪‬‬
‫‪pH‬‬ ‫الشكل ‪7‬‬ ‫‪‬‬
‫‪ 100 −  ‬‬
‫(‬ ‫)‬
‫‪ .1.4‬حدد قيمة الـ ‪ pKa‬للثنائية ‪. CO2 , H 2O / HCO3−‬‬

‫(‬
‫‪ .2.4‬أحسب قيمة ‪  HCO3−‬من أجل ‪ ، pH = 8,5‬ثم استنتج‬ ‫)‬
‫النوع الكيميائي الغالب في المحلول‪.‬‬
‫الجزء الثاني‪ :‬معايرة قساوة الماء‬ ‫‪-‬‬
‫نعاير الحجم ‪ V1 = 20 mL‬من ماء معدني بواسطة محلول لحمض‬

‫(‬ ‫)‬
‫‪2‬‬
‫تركيزه المولي‬ ‫)‪H3O+ (aq) + Cl − (aq‬‬ ‫كلور الماء‬

‫‪ . C2 = 10 mol.L‬الشكل‪ 8.‬يمثل تطور ‪ pH‬المزيج بداللة ‪  ‬‬


‫‪0‬‬ ‫‪1‬‬ ‫‪−2‬‬ ‫‪−1‬‬
‫‪log ‬‬ ‫‪‬‬
‫‪ 100 −  ‬‬ ‫الحجم ‪ VA‬من الحمض المضاف‪.‬‬
‫‪ .1‬أكتب معادلة التفاعل الكيميائي الحادث بين شوارد هيدروجينوكربونات )‪ HCO3− (aq‬وشوارد األكسونيوم‬
‫‪ . H O‬علما أن الثنائيات الداخلية في التفاعل هي‪(CO2 , H2O / HCO3− ) ، ( H3O+ / H2O ) :‬‬ ‫‪3‬‬
‫‪+‬‬
‫) ‪(aq‬‬

‫‪ .2‬حدد إحداثيات نقطة التكافؤ ‪. E‬‬


‫‪pH‬‬ ‫الشكل ‪8‬‬ ‫‪ .3‬استنتج الكاشف الملون المناسب لهذه المعايرة‪ .‬برر إجابتك‪.‬‬
‫بشوارد‬ ‫‪C1‬‬ ‫المولي‬ ‫التركيز‬ ‫أحسب‬ ‫‪.1.4 .4‬‬
‫الهيدروجينوكربونات في الماء المعدني المستعمل‪.‬‬
‫‪ .2.4‬تشير البطاقة الملصقة على قارورة الماء المعدني‬
‫إلى وجود ‪ 430mg‬من شوارد الهيدروجينوكربونات في‬
‫لتر من الماء‪ .‬هل نتيجة المعايرة على توافق مع الداللة؟‬
‫‪2‬‬
‫‪" .5‬العيار األلكلمتري التام ‪" TAC‬‬
‫‪0‬‬ ‫‪4‬‬ ‫)‪Va (mL‬‬ ‫) ‪ ( le titre alcali métrique complet‬لمحلول هو حجم‬
‫كلور الماء ذي التركيز ‪ 2  10−2 mol.L−1‬المقدر بالـ ‪ ، mL‬الالزم لمعايرة ‪ 100 mL‬من محلول‪.‬‬
‫‪ -‬حدد ‪ TAC‬العيار األلكلمتري التام للماء المعدني‪.‬‬
‫المعطيات‪:‬‬
‫الكتلة المولية لشوارد الهيدروجينوكربونات‪M = 61 g .mol −1 :‬‬ ‫‪-‬‬
‫الفينول فتالين‬ ‫أزرق البروموتيمول‬ ‫أخضر البروموكريزول‬ ‫الكاشف الملون‬
‫‪8,2 −10,0‬‬ ‫‪6,0 − 7,6‬‬ ‫‪3,8 − 5, 4‬‬ ‫مجال تغير الـ ‪pH‬‬

‫انتهى الموضوع األول‪.‬‬

‫صفحة ‪ 5‬من ‪10‬‬


‫بكالوراي جترييب ف مادة العلوم الفيزايئية ‪ /‬الشعبة‪ :‬تقين رايضي ‪ -‬رايضيات ‪ /‬دورة ماي ‪2023‬‬
‫يحتوي الموضوع الثاني على ‪ 5‬صفحات (من الصفحة ‪ 06‬إلى الصفحة ‪)10‬‬
‫التمرين األول‪ 04( :‬نقاط)‬
‫تحتوي كثير من األجهزة مثل مكبرات الصوت‪ ،‬التلفزيونات‪ ،‬المحركات‪ ،‬المنوبات على‬
‫الوشائع‪ ،‬والتي تعتبر كمصدر لطاقة كهرومغناطيسية‪.‬‬
‫يهدف التمرين إلى دراسة تصرف ثنائي القطب ‪ RL‬اثناء فتح القاطعة‪.‬‬
‫نركب دارة كهربائية تحتوي على مولد لتوتر ثابت قوته المحركة الكهربائية ‪ ، E‬ناقل‬
‫أومي مقاومته ‪ ، R = 60 ‬وشيعة ذاتيتها ‪ L‬ومقاومتها الداخلية ‪، r‬قاطعة ‪ K‬وصمام ثنائي‪( .‬الشكل‪)1.‬‬
‫‪ .1‬نغلق القاطعة ‪ ، K‬فنحصل على نظام دائم حيث تكون شدة التيار ‪. i ( t ) = I 0‬‬
‫الشكل ‪1‬‬ ‫‪E‬‬
‫= ‪I0‬‬ ‫‪ -‬بتطبيق قانون جمع التوترات بين أن‪:‬‬
‫‪R+r‬‬
‫‪ .2‬عند اللحظة ‪ t = 0‬نفتح القاطعة ‪ K‬حتى ينعدم التيار‪.‬‬
‫‪ .1.2‬جد المعادلة التفاضلية التي تحققها شدة التيار الكهربائي ) ‪. i ( t‬‬
‫‪t‬‬
‫‪−‬‬
‫‪ .2.2‬تقبل المعادلة التفاضلية السابقة حال من الشكل ‪. i ( t ) = I 0 .e ‬‬
‫جد عبارة ‪ ‬بداللة ثوابت الدارة الكهربائية‪ ،‬وبين أنه متجانس مع‬ ‫‪-‬‬
‫الزمن‪.‬‬
‫‪t‬‬
‫‪−2‬‬
‫‪ .3‬أكتب عبارة الطاقة المغناطيسية ) ‪ Eb ( t‬المخزنة في الوشيعة وبين أنها تكتب على الشكل‪Eb ( t ) = E0 .e  :‬‬
‫حيث ‪ E0‬ثابت يطلب تعيينه‪.‬‬

‫) (‬
‫‪ .4‬في الشكلين ‪ 2‬و‪ ،3‬نعطي منحنى تغير الطاقة المخزنة ‪ Eb = f i 2‬وكذلك منحنى ) ‪Eb = g ( t‬‬

‫‪ .1.4‬أوجد بيانيا قيمة كل من شدة التيار ‪ I0‬وكذلك ‪. E0‬‬


‫‪ .2.4‬استنتج قيمة الذاتية ‪ L‬للوشيعة‪.‬‬
‫‪‬‬
‫= ' ‪ ، t‬واستنتج‬ ‫‪ .3.4‬بين أن المماس للمنحنى البياني (الشكل‪ )3.‬عند اللحظة ‪ t = 0‬يقطع محور األزمنة في اللحظة‬
‫‪2‬‬
‫قيمة ثابت الزمن ‪.‬‬
‫‪ .4.4‬جد قيمة كل من‪ :‬المقاومة الداخلية ‪ r‬والقوة المحركة الكهربائية ‪. E‬‬

‫(‬
‫‪Eb 10−3 J‬‬ ‫)‬ ‫الشكل ‪2‬‬ ‫(‬
‫‪Eb 10−3 J‬‬ ‫)‬ ‫الشكل ‪3‬‬

‫‪2‬‬ ‫‪2‬‬

‫‪0‬‬ ‫‪0,25‬‬
‫(‬
‫‪i 2 10−2 A2‬‬ ‫)‬ ‫‪0‬‬ ‫‪2‬‬
‫(‬
‫‪t 10−2 s‬‬ ‫)‬
‫صفحة ‪ 6‬من ‪10‬‬
‫بكالوراي جترييب ف مادة العلوم الفيزايئية ‪ /‬الشعبة‪ :‬تقين رايضي ‪ -‬رايضيات ‪ /‬دورة ماي ‪2023‬‬
‫التمرين الثاني‪ 04( :‬نقاط)‬
‫تكون الهيليوم انطالقا من الدوتيريوم ‪ 12 H‬والتريسيوم ‪( 13 H‬نظي ار الهيدروجين) هو‬
‫تفاعل اندماج نووي يحدث تلقائيا وباستمرار في قلب النجوم محر ار طاقة هائلة‪ .‬وقد‬
‫حاول اإلنسان إحداث هذا التفاعل في المخبر من أجل استغالل الطاقة المحررة‬
‫والتحكم في استعمالها عند الضرورة (مشروع ‪.) ITER‬‬
‫مفاعل‬ ‫‪2‬‬ ‫‪3‬‬ ‫‪A‬‬ ‫‪1‬‬ ‫ننمذج هذا التفاعل النووي بالمعادلة التالية‪:‬‬
‫‪1 H + 1 H → Z He + 0 n‬‬
‫‪ .1.1 .1‬عرف تفاعل االندماج النووي‪ ،‬ثم حدد ‪ A‬و ‪Z‬‬
‫الشكل ‪4‬‬
‫لنواة الهيلويوم‪.‬‬
‫‪ .2.1‬أحسب بوحدة ‪ MeV‬طاقة الربط لكل من‬
‫األنوية التالية‪ 12 H :‬و ‪. 13 H‬‬
‫‪ .3.1‬يمثل الشكل‪ 4.‬مخطط الطاقة للتفاعل السابق‪.‬‬
‫انقله على ورقة اإلجابة وأكمل الفراغات‪.‬‬ ‫‪-‬‬
‫استنتج طاقة الربط لنواة الهيليوم ‪. ZA He‬‬ ‫‪-‬‬
‫‪ .2‬يمثل الشكل‪ ،5.‬المخطط المبسط لمستويات الطاقة في‬
‫الشكل ‪5‬‬ ‫ذرة الهيدروجين‪.‬‬
‫‪ .1.2‬حدد طول الموجة ‪ ‬لإلشعاع الصادر عندما تنتقل ذرة‬
‫الهيدروجين من الحالة المثارة ‪ 3‬إلى الحالة األساسية‪.‬‬
‫‪ .2.2‬تستقبل ذرة الهيدروجين الموجودة في الحالة األساسية أحد‬
‫الفوتونات الثالثة التالية‪:‬‬
‫‪ -‬فوتون آت من إشعاع طول موجته ‪. a = 4004 nm‬‬
‫‪ -‬فوتون آت من إشعاع طول موجته ‪. b = 1279,6 nm‬‬
‫‪ -‬فوتون آت من إشعاع طول موجته ‪. c = 121,69 nm‬‬
‫‪ -‬حدد الفوتون الذي يمكن ان تمتصه ذرة الهيدروجين‪.‬‬
‫‪ .3.2‬استنتج الحالة التي تتواجد عليها ذرة الهيدروجين بعد امتصاص‬
‫الفوتون‪.‬‬
‫المعطيات‪:‬‬
‫(‬ ‫)‬ ‫) (‬ ‫) (‬
‫‪m ZAHe = 4,0015 u ; m 12 H = 2,01355 u ; m 13H = 3,01550 u ; m 01n = 1,00866 u‬‬ ‫) (‬
‫) (‬
‫‪m 11 p = 1,00728 u ; c = 3  108 m.s −1 ; h = 6,62  10−34 J .s‬‬

‫‪; 1eV = 1,6  10−19 J‬‬


‫‪MeV‬‬
‫‪1u = 931,5‬‬
‫‪2‬‬
‫‪c‬‬

‫صفحة ‪ 7‬من ‪10‬‬


‫بكالوراي جترييب ف مادة العلوم الفيزايئية ‪ /‬الشعبة‪ :‬تقين رايضي ‪ -‬رايضيات ‪ /‬دورة ماي ‪2023‬‬
‫التمرين الثالث‪ 06( :‬نقاط)‬
‫يعرف إيثانوات البنتيل أو عطر اإلجاص تحت اسم "أسيتات األميل"‬
‫يمكن الحصول عليه بمفاعلة حمض اإليثانويك مع الكحول األميلي المستخلص‬
‫‪O‬‬ ‫قديما من البطاطس الغنية بالنشا‪ .‬الصيغة نصف‬
‫المفصلة إليثانوات البنتيل هي‪:‬‬
‫الدراسة النظرية‪:‬‬
‫‪O‬‬
‫‪-‬‬
‫‪ .1‬سم الوظيفة الكيميائية الموجودة في هذا الجزئ‪.‬‬
‫‪ .2‬يمكن الحصول على إيثانوات البنتيل انطالقا من متفاعلين ‪ A‬و ‪. B‬‬
‫حدد صيغة كل من المتفاعلين ‪ A‬و ‪ B‬علما أن ‪ A‬هو عبارة عن حمض كربوكسيلي‪.‬‬ ‫‪-‬‬
‫‪ .3‬باستعمال الصيغ نصف المفصلة‪ ،‬أكتب معادلة التفاعل الكيميائي الحادث بين المتفاعل ‪ A‬و ‪. B‬‬
‫الدراسة الحركية‪:‬‬ ‫‪-‬‬
‫عند اللحظة ‪ ، t = 0‬نمزج ‪ 0,5mol‬من المتفاعل ‪ A‬و ‪ 0,5mol‬من المتفاعل ‪ ، B‬نضيف إلى المزيج كمية قليلة‬
‫) ‪n ( mmol‬‬ ‫الشكل ‪6‬‬ ‫من حمض الكبريت‪.‬‬
‫) ‪( 25 C‬‬ ‫يحفظ الوسط التفاعلي عند درجة ح اررة ثابتة‬
‫والحجم الكلي للوسط التفاعلي ‪. V = 83 mL‬‬
‫متابعة تطور كمية مادة المتفاعل ‪ A‬خالل الزمن‪ ،‬مكنتنا‬
‫من تحديد كمية مادة إيثانوات البنتيل المتشكل‪ ،‬تسمح‬
‫النتائج المتحصل عليها برسم المنحنى البياني الممثل‬
‫بداللة الزمن (الشكل‪.)6.‬‬ ‫لتطور كمية المادة ‪n‬‬
‫‪ .1‬حدد دور حمض الكبريت‪.‬‬
‫‪0,05‬‬ ‫‪ .2‬أنجز جدوال لتقدم التفاعل الكيميائي الحادث‪.‬‬
‫‪ .3‬أعط العالقة التي تربط بين كمية مادة إيثانوات‬
‫‪0‬‬ ‫‪10‬‬ ‫) ‪t ( min‬‬ ‫عند لحظة ‪. t‬‬ ‫البنتيل والتقدم ‪x‬‬
‫‪ .1.4 .4‬عرف السرعة الحجمية لتشكل إيثانوات البنتيل‪.‬‬
‫‪ .2.4‬أحسب قيمتها عند اللحظة ‪. t = 20min‬‬
‫‪ .5‬عرف زمن نصف التفاعل ‪ ، t1/2‬ثم حدده بيانيا‪.‬‬
‫‪ .6‬حدد التركيب المولي للمزيج عند اللحظة ‪. t = 60min‬‬
‫‪ .7‬أحسب مردود تفاعل األسترة‪ ،‬وثابت التوازن ‪. K‬‬
‫‪ .8‬في حالة االصطناع دون إضافة حمض الكبريت‪ ،‬بين كيف يتطور زمن نصف التفاعل بالنسبة للمحسوب في‬
‫السؤال السابق‪.‬‬

‫صفحة ‪ 8‬من ‪10‬‬


‫بكالوراي جترييب ف مادة العلوم الفيزايئية ‪ /‬الشعبة‪ :‬تقين رايضي ‪ -‬رايضيات ‪ /‬دورة ماي ‪2023‬‬
‫التمرين التجريبي‪ 06( :‬نقاط)‬
‫قام تلميذ من القسم النهائي بزيارة موقع الـ ‪ ( www.nasa.gov ) NASA‬إلجراء بحث حول طبيعة‬
‫القوى المعينة لألجسام الساقطة‪ ،‬فوجد مايلي‪:‬‬
‫في حالة األجسام ذات السرعات الصغيرة تعطى عبارة قوى االحتكاك بالعالقة‪. f 1 = k1.v :‬‬ ‫‪-‬‬
‫‪ -‬في حالة األجسام ذات السرعات الكبيرة تعطى عبارة قوى االحتكاك بالعالقة‪. f 2 = k2.v 2. j :‬‬
‫يهدف هذا التمرين إلى دراسة حركة السقوط الشاقولي لكرة الريشة ) ‪ ( Bad min ton‬في الهواء‪.‬‬
‫المعطيات‪:‬‬ ‫‪-‬‬
‫مميزات كرة الريشة‪ :‬كتلة ‪ m = 22 g‬حجم ‪VS‬‬ ‫‪-‬‬
‫قيمة الجاذبية األرضية‪g = 9,8 m.s −2 :‬‬ ‫‪-‬‬
‫معامل االحتكاك في حالة السرعات الصغيرة‪k1 = 0,023 kg.s −1 :‬‬ ‫‪-‬‬
‫معامل االحتكاك في حالة السرعات الكبيرة‪k2 = 2, 4 kg .m −1 :‬‬ ‫‪-‬‬
‫الكتلة الحجمية للهواء‪ air = 1, 2 kg .m −3 :‬‬ ‫‪-‬‬
‫الشكل‪7.‬‬ ‫الجزء األول‪:‬‬ ‫‪-‬‬
‫قام التلميذ بترك كرة الريشة ) ‪ ( S‬لتسقط دون سرعة ابتدائية شاقوليا نحو األسفل في‬
‫حقل الجاذبية المنتظم (الشكل‪ .)7.‬باستعمال برمجية مناسبة برسم منحنى السرعة ) ‪v ( t‬‬
‫بداللة الزمن الموضح في الشكل‪8.‬‬
‫‪ .1‬حدد مقصود العبارة‪ :‬حقل الجاذبية المنتظم‪.‬‬
‫‪ .2‬تخضع كرة الريشة خالل حركتها إلى ثالث قوى‪ :‬قوة الثقل ‪ ، P‬قوة االحتكاك‬
‫‪ ، f n‬دافعة أرخميدس ‪. ‬‬
‫‪.‬‬ ‫‪ .2.2‬أذكر مميزات دافعة أرخميدس‬

‫(‬
‫‪v m.s −1‬‬ ‫)‬ ‫الشكل ‪8‬‬
‫‪ .2.2‬مثل القوى المؤثرة على كرة الريشة خالل‬
‫حركتها‪.‬‬
‫‪ .3‬بتطبيق القانون الثاني لنيوتن على مركز عطالة‬
‫كرة الريشة في مرجع مناسب‪ ،‬وباعتبار شدة قوة‬
‫بالعبارة‬ ‫تعطى‬ ‫الهواء‬ ‫مع‬ ‫االحتكاك‬
‫‪ ، f n = kn .v n .k‬حيث ‪ kn‬معامل االحتكاك و‬
‫‪ n‬عدد طبيعي‪.‬‬
‫‪ .1.3‬أثبت أن المعادلة التفاضلية لتطور سرعة‬
‫مركز عطالة الكرة من الشكل‪:‬‬
‫‪0, 4‬‬
‫‪dv k n‬‬ ‫‪  .V ‬‬
‫‪+  v = g 1 − air S ‬‬
‫‪‬‬ ‫‪m ‬‬
‫)‪t (s‬‬
‫‪dt m‬‬
‫‪0‬‬ ‫‪0, 2‬‬

‫صفحة ‪ 9‬من ‪10‬‬


‫بكالوراي جترييب ف مادة العلوم الفيزايئية ‪ /‬الشعبة‪ :‬تقين رايضي ‪ -‬رايضيات ‪ /‬دورة ماي ‪2023‬‬
‫‪ .2.3‬جد عبارة كل من ‪ a0‬التسارع االبتدائي‪ vlim ،‬السرعة الحدية‪.‬‬
‫‪ .4‬سمحت برمجية مناسبة برسم منحنى السرعة ) ‪ v ( t‬بداللة الزمن الموضح في الشكل‪.8.‬‬
‫‪ .1.4‬أحسب قيمة التسارع االبتدائي ‪ ، a0‬واستنتج قيمة ‪ VS‬حجم كرة الريشة‪.‬‬
‫‪ .2.4‬حدد قيمة ‪ vlim‬السرعة الحدية‪ ،‬واستخرج قيمة ‪. n‬‬
‫الجزء الثاني‪:‬‬ ‫‪-‬‬
‫قام التلميذ بنزع الجزء العلوي لكرة الريشة‪ ،‬ثم تركها تسقط دون سرعة ابتدائية من ارتفاع ‪ h‬عن سطح األرض‪ ،‬وبعد‬
‫‪−1‬‬
‫الدراسة قام بحساب سرعة وصول الكرة إلى سطح األرض فوجدها ‪. v f = 5,88 m.s‬‬
‫‪ .1‬حدد تأثير نزع الجزء العلوي لكرة الريشة على الحركة‪.‬‬
‫‪ .2‬عرف السقوط الحر‪.‬‬
‫‪ .3‬بتطبيق القانون الثاني لنيوتن على مركز عطالة الكرة في مرجع سطحي أرضي‪ ،‬جد المعادلة التفاضلية للسرعة‪.‬‬
‫‪ .4‬استنتج المعادالت الزمنية للسرعة ) ‪ v ( t‬والموضع ) ‪. y ( t‬‬
‫‪ .5‬أحسب االرتفاع ‪ h‬الذي سقطت منه الكرة‪.‬‬

‫انتهى الموضوع الثاني‪.‬‬

‫صفحة ‪ 10‬من ‪10‬‬


‫األستاذ‪ :‬بوزيان زكرياء‬ ‫الشعب‪ :‬تقني رياضي ورياضي‬ ‫اإلجابة النموذجية بكالوريا تجريبي ‪2023‬‬

‫العالمة‬
‫عناصر اإلجابة‬
‫مجزأة مجموعة‬
‫الموضوع األول‬
‫التمرين األول‪ 04( :‬نقاط)‬
‫‪ .1.1 .1‬معادلة التفاعل النووي‪:‬‬
‫‪2x0,25‬‬
‫‪131‬‬ ‫* ‪A‬‬ ‫‪0‬‬
‫‪53 I → Z Xe + −1e‬‬
‫‪01‬‬
‫‪A = 131‬‬ ‫بتطبيق قانون االنحفاظ (صودي)‪Z = 54 :‬‬
‫‪131‬‬ ‫* ‪131‬‬ ‫‪0‬‬
‫‪53 I → 54 Xe + −1e‬‬

‫‪ .2.1‬معادلة التحول النووي مع تحديد نوع النشاط اإلشعاعي‪54 Xe → 54 Xe +  :‬‬


‫* ‪131‬‬ ‫‪131‬‬
‫‪2x0,25‬‬
‫‪ -‬نوع النشاط اإلشعاعي‪ :‬غاما ‪‬‬

‫(‬
‫) ( )‬
‫‪ ، El 131‬وتحديد النواة األكثر استق اررا‪:‬‬
‫‪ .2‬حساب طاقة التماسك ‪ El 53 I‬و ‪54 Xe‬‬
‫‪131‬‬

‫‪53 I ) = m.c = ( 53.mP + 78.mn − m ( 53 I ) )  931,5 = 1102,627 MeV‬‬


‫‪El ( 131‬‬
‫‪2x0,25‬‬ ‫‪2‬‬ ‫‪131‬‬

‫‪54 Xe ) = m.c = ( 54.mP + 77.mn − m ( 54 Xe ) )  931,5 = 1102,889 MeV‬‬


‫‪El ( 131‬‬
‫‪2x0,25‬‬ ‫‪2‬‬ ‫‪131‬‬

‫‪ -‬تحديد النواة األكثر استق اررا‪:‬‬


‫‪1,75‬‬

‫= ‪1‬‬
‫‪( ) = 1102,627 = 8,417 MeV / n‬‬
‫‪El 131‬‬
‫‪53 I‬‬
‫‪2x0,25‬‬ ‫‪A‬‬ ‫‪131‬‬

‫= ‪2‬‬
‫(‬
‫‪El 131‬‬
‫‪54 Xe‬‬ ‫‪) = 1102,889 = 8,419 MeV / n‬‬
‫‪A‬‬ ‫‪131‬‬
‫‪0,25‬‬ ‫‪ 131‬األكثر استق اررا‪.‬‬
‫بما أن ‪ ،  2   1‬إذن نواة ‪54 Xe‬‬

‫‪0,25‬‬ ‫‪ .1.3 .3‬عبارة قانون التناقص االشعاعي ) ‪N ( t ) = N0.e−.t : N ( t‬‬

‫‪ .2.3‬تعريف زمن نصف التفاعل ‪ t1/2‬وتحديد قيمته‪:‬‬


‫‪0,25‬‬ ‫‪N0‬‬
‫= ) ‪. N ( t1/2‬‬ ‫* تعريف ‪ :‬هو الزمن الالزم لتفكك نصف عدد األنوية المشعة االبتدائية‬
‫‪2‬‬
‫‪1,5‬‬ ‫* تحديد قيمة ‪: t1/2‬‬
‫‪N‬‬ ‫) ‪N ( t1/2‬‬
‫‪0,25‬‬ ‫→ ‪N ( t1/2 ) = 0‬‬ ‫‪= 0,5 → t1/2 = 8 jours‬‬
‫‪2‬‬ ‫‪N0‬‬
‫‪ .3.3‬تحديد تاريخ زوال االشعاع‪:‬‬
‫‪3x0,75‬‬ ‫‪ A( t ) ‬‬ ‫‪‬‬ ‫‪‬‬
‫‪A ( t ) = A0.e−.t → t = − 1/2  ln ‬‬
‫‪t‬‬ ‫‪8‬‬ ‫‪1‬‬
‫‪=−‬‬ ‫‪ ln ‬‬ ‫‪ = 484,88 jours‬‬
‫‪ln 2  A0 ‬‬ ‫‪ln 2  1,76  1018 ‬‬

‫صفحة ‪ 1‬من ‪12‬‬


‫األستاذ‪ :‬بوزيان زكرياء‬ ‫الشعب‪ :‬تقني رياضي ورياضي‬ ‫تابع لإلجابة النموذجية للبكالوريا التجريبي ‪2023‬‬

‫منه يمكن تحديد التاريخ بـ‪ 12 :‬جويلية ‪1961‬‬

‫التمرين الثاني‪ 04( :‬نقاط)‬


‫𝒊‬ ‫𝑲 𝟏‬ ‫‪ -‬دراسة شحن المكثفة‪:‬‬
‫‪ .1‬تمثيل اتجاه التيار والتوترات‪:‬‬
‫‪0,5‬‬ ‫‪2x0,25‬‬ ‫𝑪𝒖‬ ‫𝑪‬
‫𝑬‬
‫𝟏𝑹 𝒖‬ ‫𝟏𝑹‬

‫‪0,25‬‬ ‫‪0,25‬‬ ‫‪ .2‬ربط راسم االهتزاز من أجل معاينة التوتر ‪ : u R1‬الشكل أعاله‬

‫‪ .3‬المعادلة التفاضلية التي يحققها التوتر ‪: u R1‬‬


‫بتطبيق قانون جمع التوترات‪:‬‬
‫‪0,5‬‬ ‫‪2x0,25‬‬
‫‪du R1 duC‬‬ ‫‪duR1 i‬‬ ‫‪duR1 uR1‬‬
‫→ ‪u R1 + uC = E‬‬ ‫‪+‬‬ ‫→‪=0‬‬ ‫→‪+ =0‬‬ ‫‪+‬‬ ‫‪=0‬‬
‫‪dt‬‬ ‫‪dt‬‬ ‫‪dt‬‬ ‫‪C‬‬ ‫‪dt‬‬ ‫‪RC‬‬

‫‪ .4‬تحديد عبارة الثوابت ‪ B ، A‬و ‪: ‬‬


‫باشتقاق عبارة ‪ u R1‬وتعويضها في المعادلة التفاضلية‪ ،‬نجد‪:‬‬
‫‪Ae− t + B‬‬
‫‪− Ae− t +‬‬ ‫‪= 0 → − Ae− t +‬‬
‫‪A − t‬‬ ‫‪B‬‬
‫‪e‬‬ ‫‪+‬‬ ‫‪=0‬‬
‫‪RC‬‬ ‫‪RC‬‬ ‫‪RC‬‬
‫‪0,75‬‬ ‫‪3x0,25‬‬ ‫‪‬‬ ‫‪1‬‬
‫‪− t ‬‬ ‫‪1  B‬‬ ‫= ‪‬‬
‫‪→ Ae‬‬ ‫‪ − +‬‬ ‫‪+‬‬ ‫‪=0→‬‬ ‫‪RC‬‬
‫‪‬‬ ‫‪RC  RC‬‬ ‫‪ B = 0‬‬
‫من الشروط االبتدائية وبتطبيق قانون جمع التوترات‪ ،‬نجد‪u R1 ( 0 ) = E :‬‬

‫وعليه‪u R1 ( 0 ) = A.e− .0 + 0 = E → A = E :‬‬

‫‪ .5‬إيجاد قيمتي ‪ E‬و ‪: R1‬‬


‫‪0,25‬‬
‫* القوة المحركة الكهربائية ‪u R1 ( 0 ) = E = 4V : E‬‬
‫‪0,5‬‬ ‫* مقاومة الناقل األومي ‪: R1‬‬
‫‪‬‬ ‫‪10−3‬‬
‫‪0,25‬‬ ‫= ‪u R1 ( ) = 0,37  E = 1,48V →  = 1ms → R1‬‬ ‫=‬ ‫‪= 100 ‬‬
‫‪−6‬‬
‫‪C‬‬ ‫‪10  10‬‬

‫‪ -‬دراسة تفريغ المكثفة‪:‬‬


‫‪ .1.1 .1‬عبارة ) ‪ uC ( t‬و ) ‪: u R2 ( t‬‬

‫صفحة ‪ 2‬من ‪12‬‬


‫األستاذ‪ :‬بوزيان زكرياء‬ ‫الشعب‪ :‬تقني رياضي ورياضي‬ ‫تابع لإلجابة النموذجية للبكالوريا التجريبي ‪2023‬‬

‫* عبارة ) ‪: uC ( t‬‬
‫باشتقاق عبارة ‪ uC‬وتعويضها في المعادلة التفاضلية‪ ،‬نجد‪:‬‬
‫) ‪− '( t −t1‬‬ ‫‪‬‬ ‫‪‬‬
‫‪− '( t −t1 ) E.e‬‬
‫‪= 0 → Ee− t  − '+‬‬
‫‪1‬‬
‫‪− ' Ee‬‬ ‫‪+‬‬ ‫‪=0‬‬
‫‪( R1 + R2 ).C‬‬ ‫‪‬‬ ‫(‬ ‫‪R‬‬‫‪1‬‬ ‫‪+‬‬ ‫‪R‬‬ ‫‪2‬‬ ‫)‬ ‫‪.‬‬‫‪C‬‬ ‫‪‬‬
‫‪‬‬ ‫‪1‬‬
‫‪ ' = ( R + R ) .C‬‬
‫‪→‬‬ ‫‪1‬‬ ‫‪2‬‬
‫‪0,5‬‬
‫‪ = ( R + R ) .C‬‬
‫‪ 2‬‬ ‫‪1‬‬ ‫‪2‬‬
‫‪t −t1‬‬
‫‪−‬‬
‫‪2‬‬
‫‪uC ( t ) = E.e‬‬ ‫وعليه‪:‬‬
‫‪0,75‬‬ ‫* عبارة ) ‪: u R2 ( t‬‬
‫من قانون جمع التوترات‪:‬‬
‫) ‪u R2 ( t ) + u R1 ( t ) + uC ( t ) = 0 → u R2 ( t ) = −u R1 ( t ) − uC ( t‬‬
‫‪‬‬ ‫‪t −t‬‬ ‫‪‬‬ ‫‪t −t‬‬
‫‪− 1‬‬ ‫‪− 1‬‬
‫‪ E‬‬ ‫‪‬‬
‫‪→ u R2 ( t ) = − R1C. C − uC ( t ) → u R2 ( t ) = − R1C. − e  2‬‬ ‫‪2‬‬
‫‪du‬‬
‫‪‬‬ ‫‪−‬‬ ‫‪Ee‬‬
‫‪0,25‬‬ ‫‪dt‬‬ ‫‪ 2‬‬ ‫‪‬‬
‫‪‬‬ ‫‪‬‬
‫‪t −t1‬‬
‫‪−‬‬
‫‪ Ee  2‬‬
‫‪R2‬‬
‫‪u R2 ( t ) = −‬‬
‫‪R1 + R2‬‬

‫‪ .2.1‬حساب قيمة ‪: R2‬‬


‫‪0,25‬‬ ‫‪0,25‬‬ ‫‪u R2 ( t1 ) = −‬‬
‫‪R2‬‬
‫‪ E = −2 → R2 = 100 ‬‬
‫‪R1 + R2‬‬

‫‪ .2‬حساب قيمة ' ‪: C‬‬


‫إضافة مكثفة أخرى للدارة يكون على التفرع ألن ‪ ، '   2‬وعليه‪:‬‬
‫‪0,5‬‬ ‫‪0,5‬‬
‫‪ ' = ( R1 + R2 ).Ceq = 3( R1 + R2 ).C → Ceq = 3.C → C '+ C = 3.C‬‬
‫‪→ C ' = 2.C = 20  F‬‬

‫التمرين الثالث‪ 06( :‬نقاط)‬


‫‪ -‬المحاولة األولى‪:‬‬
‫‪0,25‬‬
‫‪0,25‬‬
‫‪ .1‬مقصود النقطة المادية ‪ :‬هي كل جسم ذو أبعاد مهملة أمام المرجع الذي يدرس بالنسبة إليه هذا‬
‫الجسم‪ ،‬وكتلة النقطة المادية هي كتلة هذا الجسم‪.‬‬
‫‪ .2‬إثبات عبارة ‪: vB‬‬
‫بتطبيق مبدأ انحفاظ الطاقة‪:‬‬

‫صفحة ‪ 3‬من ‪12‬‬


‫األستاذ‪ :‬بوزيان زكرياء‬ ‫الشعب‪ :‬تقني رياضي ورياضي‬ ‫تابع لإلجابة النموذجية للبكالوريا التجريبي ‪2023‬‬

‫‪0,5‬‬ ‫‪2x0,25‬‬ ‫‪0‬‬


‫) (‬ ‫‪1‬‬
‫) ‪Ec A + W P = EcB →  m  vB2 = m.g .h → vB = 2.g .r (1 − cos ‬‬
‫‪2‬‬
‫‪ .3‬تحديد قيمة تسارع المتزلج خالل مراحل الحركة‪:‬‬
‫‪0,5‬‬ ‫‪2x0,25‬‬
‫اعتمادا على الشكل‪aBC = −2,5 m.s −2 ; aCD = 2,4 m.s −2 :2332 .‬‬
‫‪ .4‬إحصاء وتمثيل القوى المؤثرة على مركز عطالة المتزلج‪:‬‬
‫‪ : P‬قوة الثقل‪.‬‬
‫‪01‬‬ ‫‪4x0,25‬‬
‫‪B‬‬
‫‪ : R N‬فعل السطح الناظمي‪.‬‬
‫‪C‬‬
‫‪ : f‬قوة االحتكاك (ألن ‪) aBC  0 m.s −2‬‬
‫‪ .1.5 .5‬إيجاد عبارة التسارع ‪ aCD‬واستنتاج ‪: aBC‬‬
‫*عبارة التسارع ‪: aCD‬‬
‫‪ -‬الجملة‪ :‬المتزلج‪.‬‬
‫‪ -‬المرجع‪ :‬سطحي أرضي نعتبره غاليليا‪.‬‬
‫‪D‬‬
‫‪ -‬بتطبيق القانون الثاني لنيوتن على مركز عطالة الجملة‪:‬‬
‫‪01‬‬ ‫‪4x0,25‬‬
‫‪ F ext = m.aCD → P + f + R N = m.aCD‬‬
‫بإسقاط العبارة الشعاعية على محور الحركة‪:‬‬
‫‪f‬‬
‫‪Px − f = m.aCD → m.g .sin  − f = m.aCD → aCD = g .sin  −‬‬
‫‪m‬‬
‫‪f‬‬
‫‪aBC = −‬‬ ‫*عبارة التسارع ‪ : aBC‬بما أن المستوي أفقي إذن ‪:‬‬
‫‪m‬‬
‫‪ .2.5‬حساب شدة قوى االحتكاك ‪ f‬وقيمة الزاوية ‪: ‬‬
‫* حساب شدة قوى االحتكاك ‪f = − m.aBC = −80  (−2, 4) = 192 N : f‬‬
‫‪0,5‬‬ ‫‪2x0,25‬‬
‫‪m.aCD + f 80  2,4 + 192‬‬
‫= ‪sin ‬‬ ‫=‬ ‫* حساب قيمة الزاوية ‪= 0,489 →  = 29,3 : ‬‬
‫‪m.g‬‬ ‫‪80  9,8‬‬
‫‪ .6‬حساب الزمن المستغرق لقطع المسافة ‪ d ' = 5 m‬وسرعته عندئذ‪:‬‬
‫* الزمن ' ‪: t‬‬
‫من عبارة التسارع ‪ aCD‬والشروط االبتدائية نتحصل على‪:‬‬
‫‪0,75‬‬ ‫‪3x0,25‬‬ ‫‪aCD = 2,4 m.s −2 → v = 2,4.t → x = 1,2.t 2‬‬
‫'‪d‬‬
‫='‪t‬‬ ‫وعليه‪= 2,04 s :‬‬
‫‪1,2‬‬
‫* السرعة ' ‪v ' = 2,4.t ' = 4,89 m.s−1 : v‬‬

‫‪ -‬المحاولة الثانية‪:‬‬
‫‪ .1‬استخراج المعادالت الزمنية للموضع ) ‪ x (t‬و ) ‪: y (t‬‬

‫صفحة ‪ 4‬من ‪12‬‬


‫األستاذ‪ :‬بوزيان زكرياء‬ ‫الشعب‪ :‬تقني رياضي ورياضي‬ ‫تابع لإلجابة النموذجية للبكالوريا التجريبي ‪2023‬‬

‫‪0,5‬‬ ‫‪2x0,25‬‬ ‫بالرجوع للدالة األصلية لكل من ‪ v x‬و ‪ ، v y‬والشروط االبتدائية‪x = 5.t ; y = −4,9.t 2 :‬‬

‫‪ .2‬حساب ‪ t P‬زمن القفز‪:‬‬


‫‪0,5‬‬ ‫‪2x0,25‬‬ ‫‪yp‬‬ ‫‪4,9.t p 2‬‬ ‫)‪tan(29,3‬‬
‫= ‪tan ‬‬ ‫=‬ ‫= ‪= 0,98.t p → t p‬‬ ‫‪= 0,57 s‬‬
‫‪xp‬‬ ‫‪5.t p‬‬ ‫‪0,98‬‬
‫‪ .3‬تعيين إحداثيتي نقطة السقوط ‪ ، P‬واستنتاج المسافة ‪: CP‬‬
‫‪0,5‬‬ ‫‪2x0,25‬‬ ‫*تعيين إحداثيتي نقطة السقوط ‪x = 2,85 m ; y = −1,59 m : P‬‬
‫*المسافة ‪CP = x p 2 + y p 2 = 3,26 m : CP‬‬
‫التمرين التجريبي‪ 06( :‬نقاط)‬
‫‪ -‬الجزء األول‪:‬‬
‫‪ .1‬المقصود بـمحلول متذبذب وتفاعل حمض ‪-‬أساس‪:‬‬
‫‪0,5‬‬ ‫‪ -‬المحلول المتذبذب‪ :‬هو كل محلول يسلوك سلوك الحمض واألساس‪.‬‬
‫‪0,25‬‬
‫‪0,25‬‬ ‫‪ -‬تفاعل حمض – أساس‪ :‬هو تفاعل يحدث فيه انتقال بروتون ‪ H +‬من حمض من الثنائية‬
‫(أساس‪/1‬حمض‪ )1‬إلى أساس الثنائية (أساس‪/2‬حمض‪.)2‬‬

‫‪ .2‬كتابة المعادلتين الموافقتين للتفاعلين الممكنين بين شاردة البيكاربونات ‪ HCO3−‬والماء‪:‬‬


‫‪0,5‬‬ ‫‪HCO3− + H 2O = CO32− + H 3O +‬‬
‫‪0,25‬‬
‫‪0,25‬‬ ‫‪HCO3− + H 2O = ( CO2 , H 2O ) + HO −‬‬
‫‪ .3‬العالقة التي تربط بين ‪ pKa ، pH‬و ‪: ‬‬
‫انطالقا من عبارة ثابت الحموضة ‪: Ka‬‬
‫‪ H O +   HCO − ‬‬ ‫‪  HCO −  ‬‬
‫‪→ pH = pKa + log  ‬‬
‫‪‬‬ ‫‪3‬‬ ‫‪‬‬ ‫‪‬‬ ‫‪3‬‬ ‫‪3 ‬‬
‫= ‪Ka‬‬
‫‪2x0,25‬‬ ‫‪CO2 , H 2O‬‬ ‫‪ CO , H O  ‬‬
‫‪‬‬ ‫‪2 2 ‬‬
‫‪‬‬ ‫‪‬‬
‫من جهة أخرى‪:‬‬
‫‪ HCO −   100‬‬ ‫‪‬‬ ‫‪CO2 , H 2O  +  HCO3−  100‬‬
‫‪01‬‬
‫(‬
‫= ‪ HCO3−‬‬ ‫‪‬‬
‫)‬ ‫‪3‬‬ ‫‪‬‬
‫‪−‬‬
‫‪CO2 , H 2O  +  HCO3 ‬‬
‫→‬
‫‪ HCO ‬‬‫‪−‬‬
‫‪‬‬ ‫=‪‬‬
‫‪‬‬
‫‪‬‬ ‫‪3‬‬
‫‪2x0,25‬‬
‫‪ HCO − ‬‬
‫→‬
‫→ ‪CO2 , H 2O  = 100 − 1 → CO2 , H 2O  = 100 − ‬‬ ‫‪‬‬ ‫‪3‬‬
‫=‬
‫‪‬‬
‫‪ HCO − ‬‬ ‫‪‬‬ ‫‪ HCO − ‬‬ ‫‪‬‬ ‫‪CO2 , H 2O  100 − ‬‬
‫‪‬‬ ‫‪3‬‬ ‫‪‬‬ ‫‪3‬‬
‫‪  ‬‬
‫‪pH = pKa + log ‬‬ ‫وعليه‪ ،‬تصبح العبارة‪ :‬‬
‫‪ 100 −  ‬‬

‫صفحة ‪ 5‬من ‪12‬‬


‫األستاذ‪ :‬بوزيان زكرياء‬ ‫الشعب‪ :‬تقني رياضي ورياضي‬ ‫تابع لإلجابة النموذجية للبكالوريا التجريبي ‪2023‬‬

‫‪ .1.4 .4‬تحديد قيمة الـ ‪: pKa‬‬


‫‪2x0,25‬‬ ‫‪  ‬‬
‫‪ pH = log ‬والعالقة السابقة نجد‪pKa = 6,6 :‬‬ ‫اعتمادا على البيان ‪ + 6,6‬‬
‫‪ 100 −  ‬‬
‫‪ .2.4‬استنتاج النوع الكيميائي الغالب‪:‬‬
‫من أجل ‪ ، pH = 8,5‬لدينا‪:‬‬
‫‪01‬‬ ‫‪  ‬‬ ‫‪  ‬‬ ‫‪‬‬
‫‪log ‬‬ ‫‪ + 6,6 = 8,5 → log ‬‬ ‫→ ‪ = 1,9‬‬ ‫‪= 101,9 = 79,432‬‬
‫‪0,25‬‬
‫‪ 100 −  ‬‬ ‫‪ 100 −  ‬‬ ‫‪100 − ‬‬

‫(‬
‫‪→  HCO3− = 98,75%‬‬‫)‬
‫‪0,25‬‬ ‫(‬ ‫)‬
‫بما أن ‪ ،  HCO3−  50%‬إذن ‪ HCO3−‬يمثل الصفة الغالبة‪.‬‬

‫‪ -‬الجزء الثاني‪:‬‬
‫‪0,25‬‬ ‫‪0,25‬‬
‫‪ .1‬معادلة تفاعل المعايرة‪HCO3− + H3O+ = ( CO2 , H 2O ) + H 2O :‬‬

‫‪0,5‬‬ ‫‪2x0,25‬‬ ‫‪ .2‬تحديد إحداثيات نقطة التكافؤ‪ :‬اعتمادا على طريقة المماسين )‪E ( 6,8 mL;5‬‬

‫‪ .3‬استنتاج الكاشف الملون المناسب‪ :‬أخضر البروموكريزول ألنه ‪ pH E‬ينتمي إلى مجال تغير الـ‬
‫‪0,5‬‬ ‫‪2x0,25‬‬
‫‪pH‬‬
‫‪ .1.4 .4‬حساب التركيز المولي ‪: C1‬‬
‫‪0,5‬‬ ‫‪2x0,25‬‬ ‫‪C A.VA, E‬‬
‫= ‪C1.V1 = C A.VA, E → C1‬‬ ‫عند نقطة التكافؤ‪= 6,8  10−3 mol.L−1 :‬‬
‫‪V1‬‬
‫‪2x0,25‬‬ ‫‪ .2.4‬التأكد من الداللة‪Cm. = C1.M = 6,8 10−3  61 = 0,41 g.L−1 :‬‬
‫‪0,75‬‬
‫‪0x25‬‬ ‫وعليه فالقيمة متوافقة في حدود أخطاء التجربة‪.‬‬
‫‪ .5‬حساب العيار األلكلمتري التام للماء المعدني‪:‬‬
‫‪0,5‬‬ ‫‪2x0,25‬‬ ‫‪V1 ' 6,8  10−3  100‬‬
‫‪TAC = C1 ‬‬ ‫=‬ ‫‪= 34 mL‬‬
‫' ‪C2‬‬ ‫‪2  10−2‬‬
‫الموضوع الثاني‬
‫التمرين األول‪ 04( :‬نقاط)‬
‫‪ .1‬إثبات عبارة ‪: I 0‬‬
‫‪0,5‬‬
‫بتطبيق قانون جمع التوترات‪:‬‬
‫‪0‬‬
‫‪dI‬‬ ‫‪E‬‬
‫‪2x0,25‬‬ ‫‪ub + uR = E → L. 0‬‬ ‫= ‪+ ( R + r ).I 0 = E → I 0‬‬
‫‪dt‬‬ ‫‪R+r‬‬

‫‪ .1.2 .2‬إيجاد المعادلة التفاضلية التي يحققها شدة التيار الكهربائي ) ‪: i (t‬‬

‫صفحة ‪ 6‬من ‪12‬‬


‫األستاذ‪ :‬بوزيان زكرياء‬ ‫الشعب‪ :‬تقني رياضي ورياضي‬ ‫تابع لإلجابة النموذجية للبكالوريا التجريبي ‪2023‬‬

‫بتطبيق قانون جمع التوترات‪:‬‬


‫‪di‬‬ ‫‪di R + r‬‬
‫‪2x0,25‬‬ ‫‪ub + u R = 0 → L. + ( R + r ) .i = 0 → +‬‬ ‫‪i = 0‬‬
‫‪dt‬‬ ‫‪dt‬‬ ‫‪L‬‬

‫‪ .2.2‬إيجاد عبارة ثابت الزمن ‪ ،‬وتبيان أنه متجانس مع الزمن‪:‬‬


‫* ثابت الزمن ‪:‬‬
‫‪0,25‬‬ ‫‪t‬‬
‫‪di I 0 −‬‬
‫‪= e‬‬ ‫باشتقاق عبارة ) ‪ ، i ( t‬نجد‪:‬‬
‫‪1,25‬‬ ‫‪dt ‬‬
‫‪di‬‬
‫في المعادلة التفاضلية السابقة‪ ،‬نجد‪:‬‬ ‫بتعويض عبارتي ) ‪ i ( t‬و‬
‫‪dt‬‬
‫‪t‬‬ ‫‪‬‬ ‫‪− ‬‬
‫‪t‬‬ ‫‪t‬‬
‫‪I 0 − R + r ‬‬ ‫‪− 1 R+r‬‬
‫‪e +‬‬ ‫‪. I0  e   = 0 → I0  e   −‬‬ ‫‪=0‬‬
‫‪0,25‬‬ ‫‪‬‬ ‫‪L ‬‬ ‫‪‬‬ ‫‪‬‬ ‫‪L ‬‬
‫‪‬‬ ‫‪‬‬
‫‪L‬‬
‫= ‪→‬‬
‫‪R+r‬‬

‫‪U .T‬‬
‫‪0,25‬‬ ‫‪ L‬‬
‫= = ‪ ‬‬ ‫* التحليل البعدي لـ ‪I = T :‬‬
‫‪ R‬‬ ‫‪U‬‬
‫‪I‬‬
‫‪ .3‬كتابة عبارة الطاقة المغناطيسية ) ‪: Eb (t‬‬
‫‪1‬‬
‫نعلم أن‪Eb (t ) =  L  i 2 (t ) :‬‬
‫‪2‬‬
‫‪0,25‬‬ ‫بتعويض ) ‪ i ( t‬في العبارة السابقة‪:‬‬
‫‪0,25‬‬
‫‪2‬‬ ‫‪2‬‬
‫‪‬‬ ‫‪−‬‬
‫‪t‬‬ ‫‪‬‬ ‫‪ −t ‬‬ ‫‪−2‬‬
‫‪t‬‬
‫‪Eb (t ) =  L   I 0  e ‬‬ ‫‪ → E (t ) =  L  I 2  e   → E (t ) = E .e ‬‬
‫‪1‬‬ ‫‪1‬‬
‫‪2‬‬ ‫‪‬‬ ‫‪‬‬ ‫‪b‬‬
‫‪2‬‬
‫‪0‬‬ ‫‪‬‬ ‫‪b‬‬ ‫‪0‬‬
‫‪‬‬ ‫‪‬‬ ‫‪‬‬ ‫‪‬‬
‫‪ .1.4 .4‬إيجاد قيمة شدة التيار األعظمي ‪ I 0‬و ‪: E0‬‬
‫‪0,25‬‬
‫*شدة التيار األعظمي ‪ : I 0‬اعتمادا على الشكل‪ ،2.‬نجد‪I 0 = 10−2 = 0,1 A :‬‬
‫‪0,25‬‬ ‫*الطاقة األعظمية ‪ : E0‬اعتمادا على الشكل‪ ،2.‬نجد‪E0 = 7,6 10−3 J :‬‬
‫‪ .2.4‬استنتاج قيمة ذاتية الوشيعة ‪: L‬‬
‫‪0,25‬‬ ‫‪1‬‬ ‫‪2E‬‬ ‫‪2  7,6  10−3‬‬
‫= ‪E0‬‬ ‫= ‪ L  I 02 → L = 0‬‬ ‫‪= 1,52 H‬‬
‫‪2‬‬ ‫‪2‬‬ ‫‪−2‬‬
‫‪I0‬‬ ‫‪10‬‬

‫‪02‬‬ ‫‪‬‬
‫‪ .3.4‬تبيان المماس عند اللحظة ‪ t = 0‬يقطع محور األزمنة في اللحظة = ' ‪ ، t‬واستنتاج قيمة ‪:‬‬
‫‪2‬‬

‫صفحة ‪ 7‬من ‪12‬‬


‫األستاذ‪ :‬بوزيان زكرياء‬ ‫الشعب‪ :‬تقني رياضي ورياضي‬ ‫تابع لإلجابة النموذجية للبكالوريا التجريبي ‪2023‬‬

‫‪0,25‬‬ ‫‪dEb‬‬ ‫‪2E‬‬


‫= ‪Eb‬‬ ‫العبارة الرياضية للماس عند ‪ t + E0 → Eb = − 0  t + E0 : t = 0‬‬
‫‪dt t =0‬‬ ‫‪‬‬
‫‪0,25‬‬ ‫‪2E‬‬ ‫‪‬‬
‫‪0,25‬‬ ‫‪Eb = − 0  t '+ E0 = 0 → t ' = →  = 2  10 −2 s‬‬
‫‪‬‬ ‫‪2‬‬
‫‪ .4.4‬إيجاد قيمة ‪ r‬والقوة المحركة الكهربائية ‪: E‬‬
‫‪0,25‬‬ ‫‪L‬‬ ‫‪L‬‬ ‫‪1,52‬‬
‫=‪‬‬ ‫=‪→r = −R‬‬ ‫*المقاومة الداخلية ‪− 60 = 16  : r‬‬
‫‪R+r‬‬ ‫‪‬‬ ‫‪2  10−2‬‬
‫‪E‬‬
‫‪0,25‬‬ ‫= ‪I0‬‬ ‫*القوة المحركة الكهربائية ‪→ E = ( R + r ) .I 0 = ( 60 + 16 ) .0,1 = 7,6V : E‬‬
‫‪R+r‬‬
‫التمرين الثاني‪ 04( :‬نقاط)‬
‫‪ .1.1 .5‬تعريف االندماج النووي وتحديد ‪ A‬و ‪: Z‬‬

‫‪0,25‬‬
‫*تعريف االندماج النووي‪ :‬هو تفاعل نووي مفتعل ناتج عن التحام نواتين خفيفتين لتشكيل نواة أثقل‬
‫‪0,75‬‬ ‫مع تحرير طاقة‪.‬‬
‫‪2‬‬ ‫‪3‬‬ ‫‪A‬‬ ‫‪1‬‬
‫‪1 H + 1 H → Z He + 0 n‬‬ ‫*تحديد ‪ A‬و ‪: Z‬‬
‫‪2x0,25‬‬
‫‪A=4‬‬ ‫بتطبيق قانوني االنحفاظ صودي‪Z = 2 :‬‬

‫‪ .2.1‬حساب طاقة الربط لكل من األنوية ‪ 12 H‬و ‪: 13H‬‬

‫‪0,75‬‬
‫‪3x0,25‬‬
‫) (‬ ‫(‬ ‫‪( ))  931,5 = 2,226 MeV‬‬
‫‪El 12 H = m.c 2 = 1.mP + 1.mn − m 12 H‬‬

‫‪El ( 13H ) = m.c 2 = (1.mP + 2.mn − m ( 13H ) )  931,5 = 8,476 MeV‬‬

‫‪ .3.1‬اكمال الفراغات واستنتاج طاقة الربط لنواة الهيليوم ‪: 24 He‬‬

‫‪01‬‬
‫‪2x0,25‬‬

‫‪2x0,25‬‬ ‫) (‬
‫‪El 24 He = −E2 = − ( 4666,96 − 4695,26 ) = 28,3 MeV‬‬

‫‪ .1.2 .6‬تحديد طول الموجة ‪: ‬‬

‫صفحة ‪ 8‬من ‪12‬‬


‫األستاذ‪ :‬بوزيان زكرياء‬ ‫الشعب‪ :‬تقني رياضي ورياضي‬ ‫تابع لإلجابة النموذجية للبكالوريا التجريبي ‪2023‬‬

‫‪0,5‬‬ ‫‪0,25‬‬ ‫‪E = E3 − E1 = h ‬‬


‫‪C‬‬
‫→‬
‫‪‬‬
‫‪6,62  10−34  3  108‬‬
‫‪= 1,026  10−7 m = 102,67 nm‬‬
‫‪h.C‬‬
‫‪0,25‬‬ ‫=‪‬‬ ‫=‬
‫‪E3 − E1  −13,6‬‬ ‫‪‬‬ ‫‪−19‬‬
‫‪ 2 − ( −13,6 )   1,6  10‬‬
‫‪ 3‬‬ ‫‪‬‬
‫‪ .2.2‬تحديد الفوتون الممتص‪:‬‬
‫‪ a = 4004 nm‬‬ ‫‪ Ea = 0,31eV‬‬ ‫‪Ea = −13,29 eV‬‬
‫‪‬‬ ‫‪‬‬ ‫‪‬‬
‫‪0,75‬‬ ‫‪3x0,25‬‬ ‫‪b = 1279,6 nm →  Eb = 0,97eV →  Eb = −12,63 eV‬‬
‫‪  = 121,69 nm  E = 10,20 eV‬‬ ‫‪ E = −3,4 eV‬‬
‫‪ c‬‬ ‫‪ c‬‬ ‫‪‬‬ ‫‪c‬‬
‫الفوتون الممتص هو ‪C‬‬

‫‪0,25‬‬ ‫‪0,25‬‬ ‫‪ .3.2‬استنتاج الحالة التي تتواجد عليها ذرة الهيدروجين‪n = 2 :‬‬

‫التمرين الثالث‪ 06( :‬نقاط)‬


‫‪0,25‬‬ ‫‪0,25‬‬ ‫‪ -‬الدراسة النظرية‪:‬‬
‫‪ .1‬تسمية الوظيفية ‪ :‬استرية ' ‪R − COO − R‬‬
‫‪ .2‬تحديد صيغة كل من المتفاعلين ‪ A‬و ‪: B‬‬
‫‪0,5‬‬ ‫‪2x0,25‬‬
‫‪( A) → CH3 − COOH‬‬ ‫‪( B ) → CH3 − CH 2 − CH 2 − CH 2 − CH 2OH‬‬
‫‪ .3‬كتابة معادلة التفاعل الكيميائي الحادث بين المتفاعل ‪ A‬و ‪: B‬‬
‫‪0,5‬‬ ‫‪0,5‬‬
‫‪CH 3 − COOH + CH 3 − CH 2 − CH 2 − CH 2 − CH 2OH → CH 3 − COO − CH 2 − CH 2 − CH 2 − CH 2 − CH 3 + H 2O‬‬

‫‪ -‬الدراسة الحركية‪:‬‬
‫‪0,25‬‬ ‫‪0,25‬‬
‫‪ .1‬تحديد دور حمض الكبريت‪ :‬تسريع التفاعل (وسيط)‬
‫‪ .2‬جدول تقدم التفاعل‪:‬‬
‫معادلة التفاعل‬ ‫‪A‬‬ ‫‪+‬‬ ‫‪B‬‬ ‫=‬ ‫‪E‬‬ ‫‪+‬‬ ‫‪H2O‬‬

‫الحالة‬ ‫التقدم‬ ‫)‪n(A‬‬ ‫)‪n(B‬‬ ‫)‪n(E‬‬ ‫)‪n(H2O‬‬


‫‪0,5‬‬ ‫‪0,5‬‬
‫ابتدائية‬ ‫‪0‬‬ ‫‪0,5‬‬ ‫‪0,5‬‬ ‫‪0‬‬ ‫‪0‬‬
‫انتقالية‬ ‫‪x‬‬ ‫‪0,5 − x‬‬ ‫‪0,5 − x‬‬ ‫‪x‬‬ ‫‪x‬‬
‫نهائية‬ ‫‪xf‬‬ ‫‪0,5 − x f‬‬ ‫‪0,5 − x f‬‬ ‫‪xf‬‬ ‫‪xf‬‬

‫‪0,25‬‬ ‫‪0,25‬‬ ‫‪ .3‬العالقة التي تربط بين ) ‪ nt ( E‬والتقدم ‪nt ( E ) = x : x‬‬

‫‪0,25‬‬ ‫‪ .1.4 .4‬تعريف السرعة الحجمية لتشكل إيثانوات البنتيل‪ :‬هي سرعة تشكل إيثانوات البنتيل في وحدة‬
‫‪0,25‬‬ ‫) ‪1 dn ( E‬‬
‫= ) ‪vvol ( E‬‬ ‫‪‬‬ ‫الحجوم‪.‬‬
‫‪0,75‬‬ ‫‪VT‬‬ ‫‪dt‬‬

‫‪ .2.4‬حساب قيمة السرعة الحجمية لتشكل إيثانوات البنتيل عند ‪: t = 20min‬‬

‫صفحة ‪ 9‬من ‪12‬‬


‫األستاذ‪ :‬بوزيان زكرياء‬ ‫الشعب‪ :‬تقني رياضي ورياضي‬ ‫تابع لإلجابة النموذجية للبكالوريا التجريبي ‪2023‬‬

‫‪0,25‬‬ ‫‪0,2975 − 0,22‬‬


‫‪= 4,66  10−2 mol.L−1.min −1‬‬
‫‪1‬‬
‫= ) ‪vvol ( E‬‬ ‫‪‬‬
‫‪83  10−3‬‬ ‫‪20 − 0‬‬
‫‪0,25‬‬ ‫‪ .5‬تعريف زمن نصف التفاعل‪ ،‬وتحديد قيمته بيانيا‪ :‬هو الزمن الالزم لبلوغ تقدم التفاعل نصف تقدمه‬
‫‪0,25‬‬ ‫‪xf‬‬
‫‪0,75‬‬ ‫= ) ‪x ( t1/2‬‬ ‫النهائي‪.‬‬
‫‪2‬‬
‫)‪n f (E‬‬
‫‪0,25‬‬ ‫= ) ‪n ( t1/2‬‬ ‫‪= 0,1675 mol → t1/2 = 6min‬‬
‫‪2‬‬
‫‪ .6‬تحديد التركيب المولي للمزيج عند ‪: t = 60min‬‬
‫‪01‬‬ ‫‪4x0,25‬‬ ‫‪n f ( E ) = n f ( H 2O ) = 0,335 mol‬‬
‫‪n f ( A) = n f ( B ) = 0,165 mol‬‬
‫‪ .7‬حساب مردود تفاعل األسترة ‪ r‬وثابت التوازن ‪: K‬‬
‫‪01‬‬ ‫‪4x0,25‬‬ ‫)‪nf (E‬‬ ‫‪n f ( E ) .n f ( H 2O ) 0,3352‬‬
‫=‪r‬‬ ‫‪ 100 = 67%‬‬ ‫;‬ ‫=‪K‬‬ ‫=‬ ‫‪= 4,1‬‬
‫)‪n0 ( A‬‬ ‫) ‪n f ( A ) .n f ( B‬‬ ‫‪0,1652‬‬
‫‪ .8‬تبيان تطور زمن نصف التفاعل في حالة غياب حمض الكبريت‪:‬‬
‫‪0,25‬‬ ‫‪0,25‬‬
‫في غياب حمض الكبريت يصبح التفاعل أبطئ‪ ،‬وعليه فقيمة زمن نصف التفاعل تصبح أكبر‪.‬‬
‫التمرين التجريبي‪ 06( :‬نقاط)‬
‫‪ -‬الجزء األول‪:‬‬
‫‪0,25‬‬
‫‪0,25‬‬ ‫‪ .1‬المقصود بحقل الجاذبية المنتظم‪ :‬منطقة من الفضاء يكون فيها لشعاع حقل الجاذبية ‪ g‬ثابت في‬
‫المنحى‪ ،‬االتجاه والشدة‪.‬‬
‫‪ .1.2 .2‬مميزات دافعة أرخميس ‪: ‬‬
‫‪ -‬المبدأ‪ :‬مركز عطالة الجسم‪.‬‬
‫‪0,5‬‬ ‫‪ -‬الحامل‪ :‬شاقولي‬
‫‪ -‬االتجاه‪ :‬نحو األعلى‬
‫‪ -‬الشدة‪ :‬تعطى بالعالقة ‪ =  air .VS .g‬‬
‫‪1,25‬‬
‫‪ .2.2‬تمثيل القوى المؤثرة على كرة الريشة خالل حركتها‪:‬‬

‫‪3x0,25‬‬

‫‪ .1.3 .3‬تبيان المعادلة التفاضلية‪:‬‬


‫المرجع‪ :‬سطحي أرضي نعتبره غاليليا‪.‬‬ ‫‪-‬‬

‫صفحة ‪ 10‬من ‪12‬‬


‫األستاذ‪ :‬بوزيان زكرياء‬ ‫الشعب‪ :‬تقني رياضي ورياضي‬ ‫تابع لإلجابة النموذجية للبكالوريا التجريبي ‪2023‬‬

‫‪0,25‬‬ ‫الجملة‪ :‬كرة الريشة‪.‬‬ ‫‪-‬‬


‫بتطبيق القانون الثاني لنيوتن على مركز عطالة الجملة‪:‬‬
‫‪0,25‬‬
‫‪ F ext = m.a → P + f +  = m.a‬‬
‫) (‬
‫بإسقاط العبارة الشعاعية على المحور ‪: Oy‬‬
‫‪1,75‬‬ ‫‪dv‬‬ ‫‪dv kn n‬‬ ‫‪  .V ‬‬
‫‪0,25‬‬ ‫‪m.g − kn .v n −  = m.‬‬ ‫→‬ ‫‪+  v = g 1 − air S ‬‬
‫‪dt‬‬ ‫‪dt m‬‬ ‫‪‬‬ ‫‪m ‬‬

‫‪ .2.3‬إيجاد عبارة ‪ a0‬و ‪: vlim‬‬


‫*عبارة التسارع االبتدائي ‪: a0‬‬
‫‪  .V ‬‬ ‫‪‬‬ ‫‪dv‬‬ ‫‪‬‬
‫‪2x0,25‬‬ ‫;‪  v = 0‬وعليه‪a0 = g 1 − air S  :‬‬ ‫عند ‪= a0  : t = 0‬‬
‫‪‬‬ ‫‪m ‬‬ ‫‪‬‬ ‫‪dt t =0‬‬ ‫‪‬‬
‫*عبارة السرعة الحدية ‪: vlim‬‬
‫‪2x0,25‬‬ ‫) ‪g ( m − air .VS‬‬ ‫‪‬‬ ‫‪dv‬‬ ‫‪‬‬
‫‪vlim = n‬‬ ‫في النظام الدائم‪  v = vlim ; = 0  :‬وعليه‪:‬‬
‫‪kn‬‬ ‫‪‬‬ ‫‪dt‬‬ ‫‪‬‬
‫‪ .1.4 .4‬حساب قيمة التسارع االبتدائي ‪ ، a0‬واستنتاج قيمة ‪ VS‬حجم كرة الريشة‪:‬‬
‫*التسارع االبتدائي ‪: a0‬‬
‫‪2,4 − 0‬‬
‫‪= 6 m.s −2‬‬
‫‪0,25‬‬
‫‪dv‬‬
‫= ‪a0‬‬ ‫=‬
‫‪dt t =0 0,4 − 0‬‬
‫*حساب حجم كرة الريشة ‪:VS‬‬
‫‪  .V‬‬ ‫‪‬‬ ‫‪m  a0 ‬‬ ‫‪−3 3‬‬
‫‪0,25‬‬ ‫‪a0 = g 1 − air S‬‬ ‫‪‬‬ ‫→‬ ‫‪V‬‬‫‪S‬‬ ‫=‬ ‫‪1 −  = 7,1  10 m‬‬
‫‪‬‬ ‫‪m‬‬ ‫‪‬‬ ‫‪air ‬‬ ‫‪g ‬‬

‫‪ .2.4‬تحديد قيمة السرعة الحدية ‪ ، vlim‬واستخراج قيمة ‪: n‬‬


‫‪01‬‬ ‫‪0,25‬‬ ‫*السرعة الحدية ‪vlim = 2,4 m.s −1 : vlim‬‬
‫) ‪g ( m −  air .VS‬‬ ‫‪vlim n‬‬
‫‪vlim = n‬‬ ‫= ‪→ kn‬‬
‫‪kn‬‬ ‫) ‪g ( m −  air .VS‬‬
‫‪‬‬ ‫) ‪g ( m −  air .VS‬‬
‫‪1‬‬‫‪k‬‬ ‫=‬ ‫‪= 7,65  10−4 kg.s −1‬‬
‫‪0,25‬‬ ‫‪‬‬ ‫‪vlim‬‬
‫‪‬‬
‫‪ k = g ( m −  air .VS ) = 0,023 kg.m −1‬‬
‫‪ 2‬‬ ‫‪vlim 2‬‬
‫‪‬‬
‫*قيمة ‪ : n‬بما أن ‪ k2 = 0,023 kg.m−1‬فإن ‪. n = 2‬‬

‫صفحة ‪ 11‬من ‪12‬‬


‫األستاذ‪ :‬بوزيان زكرياء‬ ‫الشعب‪ :‬تقني رياضي ورياضي‬ ‫تابع لإلجابة النموذجية للبكالوريا التجريبي ‪2023‬‬

‫‪ -‬الجزء الثاني‪:‬‬
‫‪ .1‬تحديد تأثير نزع الجزء العلوي لكرة الريشة على الحركة‪ :‬يهمل تأثير الهواء (دافعة أرخميدس‬
‫‪0,25‬‬
‫‪0,25‬‬ ‫واالحتكاك)‬

‫‪0,25‬‬ ‫‪0,25‬‬ ‫‪ .2‬تعريف السقوط الحر‪ :‬حركة جسم خاضع لقوة ثقله فقط‪.‬‬

‫‪ .3‬إيجاد المعادلة التفاضلية للسرعة‪:‬‬


‫المرجع‪ :‬سطحي أرضي نعتبره غاليليا‪.‬‬ ‫‪-‬‬
‫الجملة‪ :‬كرة الريشة‪.‬‬ ‫‪-‬‬
‫‪0,5‬‬ ‫‪0,5‬‬
‫بتطبيق القانون الثاني لنيوتن على مركز عطالة الجملة‪:‬‬
‫‪ F ext = m.a → P = m.a → a = g‬‬
‫‪dv‬‬
‫‪dt‬‬
‫) (‬
‫بإسقاط العبارة الشعاعية على المحور ‪= g : Oy‬‬

‫‪ .4‬استنتاج المعادالت الزمنية للسرعة ) ‪ v ( t‬والموضع ) ‪: z ( t‬‬


‫‪0,5‬‬ ‫‪2x0,25‬‬
‫‪v = g .t‬‬ ‫;‬ ‫=‪y‬‬
‫‪1 2‬‬
‫بمكاملة العبارة السابقة‪ ،‬وباالعتماد على الشروط االبتدائية‪gt :‬‬
‫‪2‬‬

‫‪ .5‬حساب االرتفاع ‪ h‬الذي سقطت منه الكرة‪:‬‬


‫‪0,25‬‬ ‫‪0,25‬‬
‫= ‪tf‬‬
‫‪vf‬‬
‫=‬
‫‪5,88‬‬ ‫‪1‬‬
‫‪= 0,6s → h =  9,8  0,62 = 1,764 m‬‬
‫‪g‬‬ ‫‪9,8‬‬ ‫‪2‬‬

‫صفحة ‪ 12‬من ‪12‬‬


‫الجمهورية الجزائرية الديمقراطية الشعبية‬
‫منصة نحن سندك التعليمية‬ ‫وزارة التربية الوطنية‪.‬‬
‫دورة مـاي ‪.2023‬‬ ‫امتحان البكالوريا التجريبي‪.‬‬
‫الشعبة‪ :‬رياضيات ‪ +‬تقني رياضي‪.‬‬
‫المدة‪4 :‬سا و‪30‬د‬ ‫اختبار في مادة‪ :‬العلوم الفيزيائية‬

‫عهى انمتزشح أن ٌختار أحذ انمىضىعٍه اَتٍٍه‪:‬‬


‫انمىضىع األول‬
‫ٌحتىي انمىضىع األول عهى ‪ 05‬صفحاث (مه انصفحت ‪ 01‬مه ‪ 10‬إنى انصفحت ‪ 05‬مه ‪)10‬‬
‫انتمزٌه األول‪ 04( :‬وقاط)‬
‫توجد المكثفات بكثرة في الدارات االلكترونية نظ ار ألىميتيا البالغة‪ ،‬من اجل التعرف عمى خصائص المكثفة نحقق‬
‫المكونة من مولد لمتــوتر مثالي قوتو المـحركة الكـيربائية ‪ ، E‬قاطعة ‪ ، k‬مكثفتان‬
‫ّ‬ ‫الدارة الكــيربائية المـمثّمة بالشكل(‪ )1‬و‬

‫معكوس‬
‫فارغتان سعتييما ‪C 1‬و ‪ ، C 2‬ناقالن أوميان مقاومتييما ‪ R1‬و ‪R 2‬‬
‫حيث مقاومتيما المكافئة ىي ‪. Req  5 k ‬‬
‫نصل الدارة إلى مدخمي راسم اىتزاز ذي ذاكرة‪ ،‬وعند المحظة ‪. t  0‬‬
‫نغمق القاطعة ‪ k‬فنـشاىد المنحنيين (‪ )1‬و(‪ )2‬كما في الشكل(‪.)2‬‬
‫‪ .1‬ارفق كل منحنى بالمدخل المناسب مع التبرير‪.‬‬
‫‪ّ .2‬بين ان المكثفتين تخزنان نفس الشحنة الكيربائية‪.‬‬
‫انشكم (‪)1‬‬ ‫‪ .3‬جد المعادلة التفاضمية التي يحققيا ‪ u C 1‬التوتر بين طرفي‬
‫المكثفة ذات السعة ‪. C 1‬‬
‫‪t‬‬
‫‪‬‬
‫و ‪.‬‬ ‫ان حل المعادلة التفاضمية السابقة من الشكل‪ ، uC1 (t )  A  (1  e ) :‬جد عبارة الثابتين ‪A‬‬
‫‪‬‬
‫‪ .4‬عمما ّ‬
‫‪ .5‬استنتج عبارتي ) ‪ uC 2 (t‬و ) ‪. i (t‬‬

‫‪ .6‬اعتمادا عمى المنحنيين جد ما يمي‪:‬‬

‫‪ .1.6‬القوة المحركة الكيربائية ‪ E‬لممولد‪.‬‬


‫‪ .2.6‬شدة التيار األعظمية ‪ I 0‬المار في الدارة ‪.‬‬
‫‪ .3.6‬ثابت الزمن المميز لمدارة ‪.‬‬
‫‪ .7‬جد قيمة كل من ‪ R1‬و ‪. R 2‬‬
‫‪C‬‬
‫ثم استنتج قيمة كل من ‪C 1‬و ‪.C 2‬‬
‫‪ّ ، C‬‬
‫ان ‪ 4‬‬
‫‪ّ .8‬بين ّ‬
‫‪2‬‬

‫‪1‬‬

‫المخزنة في كل مكثفة في‬


‫ّ‬ ‫‪ .9‬احسب الطاقة الكيربائية‬
‫انشكم (‪)2‬‬ ‫نياية عممية الشحن‪.‬‬

‫صفحة ‪ 1‬من ‪10‬‬


‫اختبار في مادة العلوم الفيزيائية ‪ /‬الشعبة‪ :‬رياضيات ‪ +‬تقني رياضي ‪ /‬بكالوريا تجريبي دورة ماي ‪2023‬‬

‫انتمزٌه انثاوً‪ 06( :‬وقاط)‬


‫‪B‬‬
‫ٌهذف هذا انتمزٌه إنى دراست انحزكت عهى مستى مائم و أفقً‪.‬‬
‫‪‬‬ ‫انمزحهت األونى‪ :‬دراست حزكت جمهت مٍكاوٍكٍت عهى مستى مائم‪.‬‬
‫‪A‬‬
‫تتحرك سيارة كتمتيا ‪ m  1800 kg‬عمى طريق مستقيم طولو ‪ AB  120 m‬يميل عن األفق بالزاوية ‪ ،   4‬حيث‬
‫‪‬‬ ‫‪‬‬
‫تخضع السيارة أثناء حركتيا إلى قوة محركة ‪ F1‬شدتيا ثابتة و موازية لمطريق‪ ،‬كما تخضع إلى قوة احتكاك ‪ f‬ثابتة في‬
‫الشدة و معاكسة لجية الحركة‪.‬‬
‫يمثل الشكل (‪ )3‬مخطط الحصيمة الطاقوية لمجممة (السيارة ‪ +‬األرض) بين الموضعين ‪ A‬و ‪. B‬‬
‫‪ .1‬مثل القوى الخارجية المطبقة عمى السيارة خالل الحركة‪.‬‬
‫‪C‬‬
‫‪ .2‬بتطبيق القانون الثاني لنيوتن عمى السيارة بين أن عبارة تسارعيا تكتب بالعالقة‬
‫‪180kJ‬‬ ‫‪D‬‬
‫‪F1  f‬‬
‫‪0‬‬
‫‪a‬‬ ‫التالية‪ g .sin   :‬‬
‫(السيارة ‪ +‬األرض)‬
‫‪m‬‬
‫‪ .3‬اعتمادا عمى مخطط الحصيمة الطاقوية لمجممة (السيارة ‪ +‬األرض)‪:‬‬
‫الشكل (‪)3‬‬
‫‪ .1.3‬حدد طبيعة حركة السيارة مع التعميل‪.‬‬
‫‪‬‬
‫‪.2.3‬جد شدة القوة المحركة لمسيارة ‪. F1‬‬
‫‪.3.3‬بين أن شدة قوة االحتكاك ىي ‪. f  269,5 N‬‬
‫‪.4.3‬ماذا يمثل المقدارين الفيزيائيين ‪ C‬و ‪ D‬في مخطط الحصيمة الطاقوية؟ احسب قيمتييما‪.‬‬

‫انمزحهت انثاوٍت‪ :‬دراست حزكت جمهت مٍكاوٍكٍت عهى مستى أفقً‪.‬‬


‫عند الوصول إلى الجزء األفقي (النقطة ‪ )O‬عند لحظة ‪ t  0‬نعتبرىا كمبدأ لقياس األزمنة ‪،‬‬
‫‪‬‬ ‫‪‬‬
‫تخضع الجممة في ىذا الجزء إلى تأثير قوة االحتكاك السابقة ‪ f‬و قوة ‪ F2‬تطبق عمى الجممة‬
‫شدتيا ثابتة و موازية لمطريق و في جية الحركة‪ ،‬تم الحصول عمى البيان الممثل في الشكل (‪.)4‬‬

‫‪‬‬
‫‪v m .s 1‬‬ ‫‪‬‬ ‫الشكل (‪)4‬‬ ‫‪ .1‬مثل القوى الخارجية المطبقة عمى مركز عطالة الجممة في ىذه المرحمة‪.‬‬
‫‪ .2‬بتطبيق مبدأ انحفاظ الطاقة عمى الجممة المدروسة بين أن عبارة تسارع‬
‫‪F2  f‬‬
‫‪a‬‬ ‫مركز عطالتيا تعطى بالعبارة التالية‪:‬‬
‫‪m‬‬
‫‪ .3‬جد من البيان قيمة تسارع مركز عطالة الجممة المدروسة‪ ،‬ثم استنتج شدة‬
‫‪10‬‬ ‫‪‬‬
‫القوة ‪. F2‬‬
‫‪ .4‬اكتب المعادلتين الزمنيتين ‪ v t ‬و ‪ x t ‬لسرعة وموضع مركز عطالة‬
‫‪0‬‬ ‫‪2,5‬‬ ‫‪t s ‬‬

‫الجممة‪.‬‬
‫‪.5‬يوجد بالسيارة نظام إنذار تمقائي خاص بضبط السرعة المحددة عمى الطريق‪ ،‬السرعة المحددة ىي‪. 90 km .h 1‬‬

‫صفحة ‪ 2‬من ‪10‬‬


‫اختبار في مادة العلوم الفيزيائية ‪ /‬الشعبة‪ :‬رياضيات ‪ +‬تقني رياضي ‪ /‬بكالوريا تجريبي دورة ماي ‪2023‬‬

‫‪ .1.5‬جد المحظة الزمنية ‪ t 1 ‬الموافقة الشتغال اإلنذار بطريقتين مختمفتين‪.‬‬


‫‪ .2.5‬احسب المسافة المقطوعة بين المحظتين ‪ t  0‬و ‪. t 1‬‬
‫تعطى‪. g  9,8 m .s 2 :‬‬
‫انتمزٌه انثانث‪ 04( :‬وقاط)‬
‫‪ 1896‬ظاىرة النشاط اإلشعاعي والتي نال عمى إثرىا جائزة نوبل لمفيزياء‪ ،‬ىذا‬ ‫اكتشف العالم ىنري بكريل سنة‬
‫االكتشاف كان لو أفاق بالغة من خالل تطبيقاتو التي من أىميا التأريخ ‪.‬‬
‫ٌهذف انتمزٌه إنى دراست ظاهزة انىشاط اإلشعاعً وانتأكذ مه عمز مخطىطت ‪.‬‬
‫الشكل (‪)5‬‬
‫‪ .1‬عرف ظاىرة النشاط اإلشعاعي ‪.‬‬
‫‪ .2‬الشكل (‪ )5‬يوضح جزء من مخطط ) ‪. (Z  N‬‬
‫صنف األنوية التالية حسب نمط تفككيا إلى ‪ ‬أو ‪ ‬أو أنوية مستقرة‪.‬‬
‫‪‬‬ ‫‪‬‬

‫‪. 8O ،‬‬ ‫‪B ،‬‬ ‫‪N ، 146C ، 105 B ، 126C‬‬


‫‪17‬‬ ‫‪11‬‬ ‫‪13‬‬
‫‪5‬‬ ‫‪7‬‬

‫‪ .3‬اكتب معادلة تفكك نواة ‪. 6 C‬‬


‫‪14‬‬

‫‪ .4‬تمكنت الجزائر سنة ‪ 2023‬من استعادة مخطوطة إسالمية نادرة استولت‬


‫عمييا السمطات االستعمارية الفرنسية بعد غارة ضد األمير عبد القادر‪.‬‬
‫لمتأكد من تاريخ المخطوطة قمنا بقياس نشاطيا اإلشعاعي فوجدناه‬
‫‪ ، 245 mBq‬وعند قياس نشاط عينة مصنوعة من نفس المادة حديثا وجد نشاطيا اإلشعاعي ‪. 256 mBq‬‬
‫) ‪ N (t‬بين أن عبارة النشاط اإلشعاعي لمعينة تكتب بالعالقة‪:‬‬ ‫‪ .1.4‬انطالقا من عبارة قانون التناقص اإلشعاعي‬
‫‪.‬‬ ‫‪ A (t )  A0e  t‬مع ذكر المدلول الفيزيائي لمثابتين ‪ A 0‬و ‪‬‬
‫‪ .2.4‬حدد عمر المخطوطة ثم استنتج إلى أي سنة تعود‪ ،‬عمما أن‪. t 1/2 (6 C )  5730 ans :‬‬
‫‪14‬‬

‫‪ .5‬يستعمل نظير األزوت ‪ 7 N‬لمتأكد من سالمة عضمة القمب‪ ،‬وذلك بحقن المريض بحقنتين منفصمتين من ىذا‬
‫‪13‬‬

‫النظير المشع ‪ .‬بإدخال المعطيات الموافقة في جياز الكمبيوتر تمكنا من رسم البيان الممثل في الشكل (‪.)6‬‬
‫‪ 7 N‬ثم احسب‬ ‫‪ .1.5‬حدد قيمة زمن نصف العمر لمنظير‬
‫‪13‬‬

‫قيمة ثابت النشاط اإلشعاعي ‪ ‬بوحدة ) ‪. (s‬‬


‫‪1‬‬

‫‪ .2.5‬عين عدد األنوية االبتدائية المشعة في الحقنة ثم اسنتنج‬


‫نشاطيا االبتدائي‪.‬‬
‫‪ .3.5‬تصبح الحقنة غير فعالة إذا تناقص نشاطيا بنسبة‬
‫‪ ، 95%‬جد المدة الزمنية الالزمة النتياء فعالية الحقنة‪.‬‬
‫‪ .4.5‬تم حقن المريض بالحقنة األولى عمى الساعة التاسعة‬
‫صباحا‪ ،‬حدد أقل توقيت يوافق حقن المريض بالحقنة الثانية‪.‬‬
‫الشكل (‪)6‬‬

‫صفحة ‪ 3‬من ‪10‬‬


‫اختبار في مادة العلوم الفيزيائية ‪ /‬الشعبة‪ :‬رياضيات ‪ +‬تقني رياضي ‪ /‬بكالوريا تجريبي دورة ماي ‪2023‬‬

‫انتمزٌه انتجزٌبً‪ 06( :‬وقاط)‬


‫النحاس والفضة من أقدم المعادن التي عرفيا اإلنسان‪ ،‬استخدميا بكثرة عمى نطاق واسع مثل صناعة األواني والحمي‬
‫والنقود‪ ...‬الخ‪.‬‬
‫ييدف التمرين إلى دراسة تفاعل النحاس مع نترات الفضة وكذلك دراسة العمود الكيربائي نحاس – فضة‪.‬‬
‫‪ -I‬نضع في كأس بيشر حجما 𝐿𝑚 ‪ 𝑉 = 100‬من محمول نترات الفضة (‪ )𝐴𝑔+ + 𝑁𝑂3−‬تركيزه المولي 𝑐‪ ،‬عند‬
‫الــمحــظة ‪ 𝑡 = 0‬ن ـغ ــمر صف ــيحة من النــحاس 𝑢𝐶 كتمـتيا 𝑔‪ ، 𝑚0 = 3,81‬فيحدث تــحول كيمــيائي تام وبــطيء‬
‫ينمذج بمعادلة التفاعل ‪𝐶𝑢(𝑠) + 2𝐴𝑔+(𝑎𝑞 ) = 𝐶𝑢2+ (𝑎𝑞 ) + 2𝐴𝑔(𝑠) :‬‬
‫باستعمال طريقة مناسبة تحصمنا عمى بيان تغيرات كتمة الفضمة )𝑔𝐴(𝑚 بداللة الزمن كما يوضحو الشكل (‪.)7‬‬

‫انشكـل (‪)7‬‬

‫‪ .1‬عرف تفاعل أكسدة ‪ -‬إرجاع ثم اكتب الثنائيتين الداخمتين في التفاعل‪.‬‬


‫‪ .2‬أنشئ جدوال لتقدم التفاعل الحادث‪.‬‬
‫‪ .3‬جد عبارة )𝑡( 𝑔𝐴𝑚 بداللة 𝑥 ثم استنتج قيمة 𝑓𝑥 ‪.‬‬
‫‪ .4‬جد المتفاعل المحد واستنتج قيمة التركيز المولي 𝑐 ‪.‬‬
‫𝑓𝑚‬
‫= ) ‪ 𝑚(𝑡1/2‬ثم عين قيمة ‪. 𝑡1/2‬‬ ‫‪ .5‬بين أن ‪:‬‬
‫‪2‬‬
‫‪ .6‬بين أن السرعة الحجمية الختفاء شوارد الفضة تعطى بالعالقة ‪𝑣𝑣𝑜𝑙 (𝐴𝑔+) = 9.26 × 10−2 𝑑𝑚/𝑑𝑡 :‬ثم‬
‫احسب قيمتيا عند المحظتين 𝑛𝑖𝑚‪ 𝑡 = 20‬و 𝑛𝑖𝑚‪ ، 𝑡 = 40‬دون استنتاجك‪.‬‬
‫‪ .7‬نعيد التجربة السابقة باستعمال نفس الكتمة من النحاس عمى شكل قطع مجزأة‪.‬‬
‫‪ .1.7‬اذكر تأثير ذلك عمى كل من ‪ :‬السرعة الحجمية الختفاء شوارد الفضة و زمن نصف التفاعل ‪.‬‬
‫‪ .2.7‬حدد العامل الحركي المدروس ثم ارسم كيفيا عمى نفس المعمم البيان المتحصل عميو في ىذه الحالة‪.‬‬
‫نصفين ‪:‬‬
‫ـ‬ ‫‪ -II‬يتشكل عمود كيربائي من‬
‫النصف األول عبارة عن صفيحة من النحاس 𝑢𝐶 مغمورة جزئيا في محمول نترات النحاس ) ‪(𝐶𝑢2+ + 2𝑁𝑂3−‬حجمو‬
‫النصف الثاني ـفىو عـبارة عن صفيحة من الفضة 𝑔𝐴 مغمورة‬
‫كيزه المولي ‪𝑐1 = 0.5𝑚𝑜𝑙. 𝐿−1‬أما ـ‬
‫𝐿𝑚 ‪𝑉1 = 100‬وتر ـ‬
‫جزئيا في محمول حجمو 𝐿𝑚 ‪𝑉2 = 100‬من نترات الفضة ) ‪(𝐴𝑔+ + 𝑁𝑂3−‬تركيزه المولي ‪.𝑐2 = 0.1𝑚𝑜𝑙. 𝐿−1‬‬

‫صفحة ‪ 4‬من ‪10‬‬


‫اختبار في مادة العلوم الفيزيائية ‪ /‬الشعبة‪ :‬رياضيات ‪ +‬تقني رياضي ‪ /‬بكالوريا تجريبي دورة ماي ‪2023‬‬

‫نصل المحمولين بورق ترشيح مبمّل بمحمول نترات البوتاسيوم ‪ ،‬ثم نربط أمبير متر بين الصفيحتين فيمر تيـار كىربائي من‬
‫شدتو𝐴𝑚 ‪.𝐼 = 2‬‬
‫صفيحة الفضة نحو صفيحة النحاس ّ‬
‫‪.1‬ارسم شكل العمود مبينا إشارتي القطبين وجية مرور اإللكترونات‪.‬‬
‫‪ .2‬أعط الرمز االصطالحي لمعمود المدروس‪.‬‬
‫‪ .3‬أنشئ جدوال لتقدم التفاعل الحادث‪ ،‬واحسب كسر التفاعل االبتدائي 𝑖𝑟𝑄‪.‬‬
‫‪.4‬عمما أن الجممة تتطور في االتجاه المباشر‪ ،‬اختر قيمة ثابت التوازن 𝐾 ليذا التفاعل من بين القيم التالية مبر ار‬
‫‪𝐾 = 2,1 × 1015‬‬ ‫‪،‬‬ ‫‪𝐾 = 50‬‬ ‫جوابك ‪، 𝐾 = 2,1 × 10−15 :‬‬
‫‪ .5‬احسب كمية الكيرباء المنتجة للعمود ومدة اشتغالو 𝑡∆ ‪.‬‬

‫انمعــطــٍاث ‪:‬‬
‫𝑙𝑜𝑚‪𝑀 𝐴𝑔 = 108 𝑔/𝑚𝑜𝑙 ،𝑀(𝐶𝑢) = 63.5𝑔/𝑚𝑜𝑙 ،1𝐹 = 96500 𝐶/‬‬

‫اوتهى انمىضىع األول‬

‫صفحة ‪ 5‬من ‪10‬‬


‫اختبار في مادة العلوم الفيزيائية ‪ /‬الشعبة‪ :‬تقني رياضي ‪ +‬رياضيات ‪ /‬بكالوريا تجريبي دورة ماي ‪2023‬‬

‫انمىضىع انثبوً‬
‫ٌحتىي انمىضىع انثبوً عهى ‪ 05‬صفحبث (مه انصفحت ‪ 06‬مه ‪ 10‬إنى انصفحت ‪ 10‬مه ‪)10‬‬
‫انتمرٌه األول‪ 06( :‬وقبط)‬
‫‪ 2:45‬من صباح‬ ‫>> ‪ .....‬إنو أخطر قرار في القرن العشرين ‪ .....‬في التوقيت‬
‫يوم ‪ 1945/08/06‬أقمعت القاذفة ‪ B - 29‬محممة بقنبمة نووية من قاعدة أمريكية‬
‫بالمحيط اليادي‪ ،‬و بعد ‪ 6‬ساعات ونصف بالتوقيت الياباني‪ ،‬ألقيت القنبمة النووية‬
‫من عمى ارتفاع ‪ 580m‬تقريبا فوق مركز مدينة ىيروشيما‪.<<...‬‬
‫نتائج االنفجار الثانوية‬ ‫نتائج االنفجار األولية‬ ‫مواصفات قنبمة ىيروشيما‬
‫‪ -‬حروق بالغة بسبب الح اررةالشديدة‪.‬‬ ‫‪ -‬تتكون كرة من الميب بقطر‬ ‫‪ -‬الطول‪. 3m :‬‬
‫‪ -‬فقدان البصر‪.‬‬ ‫‪15m‬خالل ‪ 0,1m s‬بدرجة‬ ‫‪ -‬القطر‪. 0,7 m :‬‬
‫‪ -‬األورام السرطانية بفعالإلشعاعات‪.‬‬ ‫ح اررة ‪ 300‬ألف درجة مئوية‪.‬‬ ‫‪ -‬الوزن‪ 4,4 :‬طن ‪.‬‬
‫‪ -‬سقوط مطر أسود غني باإلشعاعات‪.‬‬ ‫‪ -‬قتل في المحظة األولى حوالي‬ ‫‪A‬‬
‫‪ZU‬‬ ‫‪ -‬الوقود النووي‪ :‬اليورانيوم‬
‫‪ 80‬ألف شخص‪.‬‬ ‫‪ -‬الطاقة الناتجة‪ :‬تعادل ‪12,5‬‬
‫كيموطن من مادة ‪.TNT‬‬

‫ٌهدف انتمرٌه إنى دراست تفبعم االوشطبر انىىوي وانىشبط اإلشعبعً ألحد انىظبئر انمشعت انىبتجت عىه‪.‬‬
‫‪ .1 - I‬يحدث أثناء انفجار القنبمة النووية انشطار نووي‪ ،‬عرفو‪.‬‬
‫‪.2‬اشرح العبارتين‪ - :‬تتكىن كرة مه انههب بقطر ‪ 15m‬خالل ‪ 0,1m s‬بدرجت حرارة ‪ 300‬أنف درجت مئىٌت‪.‬‬
‫‪ -‬سقىط مطر أسىد غىً ببإلشعبعبث‪.‬‬
‫‪ .3‬الشكل (‪ )1‬المرفق يبين الحصيمة الكتمية ألحد تفاعالت االنشطار النووي الحادث‪ ،‬باالعتماد عميو‪:‬‬
‫) ‪m (u‬‬ ‫‪ .1.3‬حدد قيمة كل من‪ x , Z , A :‬و ‪. y‬‬
‫الشكل (‪)1‬‬
‫‪92 p 144 n‬‬
‫‪ .2.3‬استنتج معادلة تفاعل االنشطار الحادث‪.‬‬
‫‪237,924‬‬
‫‪. 139‬‬
‫‪ .3.3‬احسب طاقة الربط لكل من النواتين ‪ Z U‬و ‪y Xe‬‬
‫‪A‬‬

‫‪ .4.3‬جد قيمة الطاقة الناتجة عن انشطار نواة واحدة من اليورانيوم ‪ ZAU‬بـ‬


‫‪A‬‬
‫‪Z‬‬‫‪U  01n‬‬ ‫‪ MeV‬ثم بـ ‪ ( J‬يعطى ‪.) 1MeV  1,6 1013 J :‬‬
‫‪236,001‬‬
‫‪ .4‬كتمة اليورانيوم المنشطرة ىي ‪. m  700g‬‬
‫‪139‬‬
‫‪y‬‬ ‫‪Xe  38‬‬
‫‪94‬‬
‫‪Sr  x 01n‬‬
‫‪235,809‬‬ ‫‪ .1.4‬احسب الطاقة المحررة من انشطار ىذه الكتمة ‪. m‬‬
‫‪ .2.4‬تحقق بالحساب من العبارة »انطبقت انىبتجت‪ :‬تعبدل ‪12,5‬كٍهىطه مه مبدة‬
‫« ‪TNT‬عمما أن ‪1g‬من مادة ‪ TNT‬تحرر طاقة قدرىا ‪. 4,18kJ‬‬
‫نظير مشع‪ ،‬يتفكك تمقائيا مصد ار جسيما ‪.  ‬‬ ‫‪‬‬ ‫‪‬‬
‫‪ - II‬السترونتيوم ‪Sr‬‬
‫‪94‬‬

‫‪.1‬اكتب معادلة التفكك اإلشعاعي لمسترونتيوم ‪ 94‬إذا عممت أن ناتج التفكك ىو عنصر اإليتريوم ''‪. ZA Y‬‬

‫صفحة ‪ 6‬من ‪10‬‬


‫اختبار في مادة العلوم الفيزيائية ‪ /‬الشعبة‪ :‬رياضيات ‪ +‬تقني رياضي ‪ /‬بكالوريا تجريبي دورة ماي ‪2023‬‬

‫‪.2‬يمثل المنحنى المبين في الشكل ( ‪ N  f t  )2‬تغيرات عدد األنوية المشعة المتبقية لمسترونتيومبداللة الزمن لعينة‬
‫‪‬‬
‫‪N 1020‬‬ ‫‪‬‬ ‫انشكم (‪)2‬‬ ‫مشعة كتمتيا ‪. m 0‬‬
‫‪.1.2‬أعط عبارة قانون التناقص اإلشعاعي‪ ،‬ثم تأكد أنو حل لممعادلة‬
‫‪dN‬‬
‫‪. N  ‬‬ ‫التفاضمية التالية ‪:‬‬
‫‪dt‬‬
‫‪.2.2‬اعتمادا عمى منحنى الشكل (‪: )2‬‬
‫أ‪ -‬جد قيمة النشاط اإلشعاعي االبتدائي ‪ A 0‬لمعينة‪.‬‬
‫‪1‬‬
‫ب‪ -‬استنتج قيمة ثابت النشاط اإلشعاعي ‪ ‬لمسترونتيوم ‪ 94‬بطريقتين‪.‬‬
‫‪t s ‬‬
‫‪0‬‬ ‫‪40‬‬ ‫ج‪ -‬احسب كتمة العينة المشعة ‪. m 0‬‬
‫‪.3.2‬إن القياسات لعينة من السترونتيوم ‪ 94‬يجب أن ال تتجاوز مدة زمنية ‪ t m‬قدرىا ‪ 9‬دقائق‪ ،‬برر ىذا اإلجراء‪.‬‬
‫حدد تأثير ذلك عمى المقادير التالية ‪:‬‬
‫‪.4.2‬نضاعف كتمة العينة المشعة السابقة‪ّ ،‬‬
‫‪‬نمط التفكك ‪.‬‬ ‫‪ ‬المدة الزمنية لمقياس ‪. t m‬‬ ‫‪‬النشاط اإلشعاعي االبتدائي ‪ A 0‬لمعينة ‪.‬‬
‫‪E 94‬‬
‫‪‬‬
‫‪1tonne  103 kg ، N A  6,021023 mol 1 ، l 38 Sr  8,62‬‬
‫‪A‬‬
‫‪‬‬ ‫‪MeV‬‬
‫‪nucléon‬‬
‫المعطيات‪،1u  931,5 MeV / c 2 :‬‬
‫انتمرٌه انثبوً‪ 04( :‬وقبط)‬
‫نعتبر الدارة الممثمة بالشكل(‪ )3‬المقابل والتي تتكون من ‪:‬أمبير متر ‪، A‬‬
‫فولط متر ‪ ،V‬وشيعة مقاومتيا ‪ r‬و ذاتيتيا ‪ ، L  250mH‬مولد مثالي‬
‫لمتوتر قوتو المحركة الكيربائية ‪. E‬‬
‫‪ .1‬أعد رسم الدارة و ضع الرمزين ‪ A‬و ‪ V‬في المكان المناسب‪ ،‬ثم مثل‬
‫جية التيار و التوتر بين طرفي الوشيعة ‪. u b‬‬
‫الشكل (‪)3‬‬ ‫‪ .2‬في النظام الدائم يشير األمبير متر إلى القيمة ‪ ، I 0  410 mA‬والفولط متر‬
‫إلى القيمة ‪، u  5,95V‬استنتج القيمة ‪ r‬لمقاومة الوشيعة ‪.‬‬
‫‪ .3‬احسب قيمة الطاقة المخزنة في الوشيعة‪.‬‬
‫‪ .4‬جد المعادلة التفاضمية لتطور شدة التيار ‪ i‬المارة في الدارة‪.‬‬
‫‪1‬‬
‫‪ t‬‬
‫‪ ، i (t )    (1  e‬حيث ‪ ‬و‬ ‫‪‬‬
‫أن حل ىذه المعادلة ىو من الشكل‪) :‬‬
‫‪ .5‬عمما ّ‬
‫‪ ‬ثابتان يطمب إيجاد عبارتييما‪.‬‬
‫‪ .6‬نغير الوشيعة السابقة بوشيعة أخرى ذاتيتيا ‪ L 0‬ومقاومتيا ميممة ‪ ،‬ونمرر في‬
‫الوشيعة تيا ار كيربائيا تتغير شدتو ‪ i‬بداللة الزمن كما يبين الشكل (‪:)4‬‬
‫لما‪ 0  t  2,5ms :‬و ‪0  t  5ms‬‬
‫‪ .1.6‬اكتب العبارة المحظية لشدة التيار ّ‬
‫‪ .2.6‬استنتج عبارة التوتر ‪ u B‬بين طرفي الوشيعة ‪.‬‬

‫الشكل (‪)4‬‬
‫صفحة ‪ 7‬من ‪10‬‬
‫اختبار في مادة العلوم الفيزيائية ‪ /‬الشعبة‪ :‬رياضيات ‪ +‬تقني رياضي ‪ /‬بكالوريا تجريبي دورة ماي ‪2023‬‬

‫‪ .3.6‬م ن بين المنحنيات الممثمة في الشكل(‪ ،)5‬حدد المنحنى الموافق لمتوتر بين طرفي الوشيعة‪:‬‬

‫) ‪u b (mV‬‬ ‫) ‪u b (mV‬‬ ‫الشكل (‪)5‬‬ ‫) ‪u b (mV‬‬

‫‪ .4.6‬استنتج قيمة ذاتية الوشيعة ‪. L 0‬‬

‫انتمرٌه انثبنث‪ 04( :‬وقبط)‬


‫لقد جعمت االقمار االصطناعية من العالم قرية الصغيرة ‪ ،‬حيث تم احصاء حوالي ‪ 2063‬قمر اصطناعي تدور حول‬
‫األرض حتى أفريل ‪ ، 2019‬في ىذا التمرين سندرس حركة قمر اصطناعي ونتعرف عمى بعض خصائصو‪.‬‬
‫نعتبر األرض كروية الشكل نصف قطرىا ‪ RT‬وكتمتيا ‪ ، M T‬يدور حوليا قمر اصطناعي كتمتو ‪ m‬عمى ارتفاع ‪ h‬من‬
‫سطحيا بسرعة ‪v‬وفق مسار نعتبره دائريا‪.‬يعطى ثابت الجذب العام ‪.G  6,67  1011 SI  :‬‬
‫‪‬‬
‫‪ .1‬أعط العبارة الحرفية لشدة قوة التجاذب ‪ F T /s‬بين األرض والقمر االصطناعي بداللة‪ G ، h ، M T ، m :‬و ‪RT‬‬
‫ثم بين أن عبارة االرتفاع ‪ h‬يمكن أن‬
‫‪ .2‬استنتج العبارة الحرفية لمجاذبية األرضية ‪ g‬بداللة ‪ h ، M T ، RT :‬و ‪ّ ،G‬‬
‫‪1‬‬
‫‪ ، h  A‬حيث ‪ A‬و ‪ B‬ثابتين يطمب‬ ‫تكتب عمى الشكل‪ B :‬‬
‫‪g‬‬
‫تحديد عبارتيىما‪.‬‬
‫‪ .3‬بتطبيق القانون الثاني لنيوتن جد عبارة ‪ v‬سرعة القمر االصطناعي في‬
‫مداره بداللة ‪ h ، M T ، RT :‬و ‪.G‬‬
‫‪ .4‬باستعمال برنامج محاكاة تحصمنا عمى منحنى الشكل (‪ ،)6‬الذي يمثل‬
‫‪1‬‬
‫( ‪ ، h  f‬باالعتماد عميو‪:‬‬ ‫)‬
‫‪g‬‬
‫‪ .1.4‬أكتب العبارة البيانية لممنحنى‪.‬‬
‫الشكل (‪)6‬‬
‫‪ .3.4‬استنتج كتمة األرض ‪ M T‬و قيمة نصف قطر األرض ‪. RT‬‬
‫‪ .4.4‬جد قيمة تسارع الجاذبية ‪ g 0‬عمى سطح األرض‪.‬‬
‫‪ .5‬القمر االصطناعي ‪ Météosat 9‬ينتمي إلى سمسمة األقمار ‪ Météosat‬التي تعتبر األفضل في مجال األرصاد‬
‫الجوية والمناخ والبيانات البيئية‪ ،‬حيث قيمة الجاذبية األرضية فيمدارىذا القمرىي‪. g  0,22 SI  :‬‬
‫صفحة ‪ 8‬من ‪10‬‬
‫اختبار في مادة العلوم الفيزيائية ‪ /‬الشعبة‪ :‬رياضيات ‪ +‬تقني رياضي ‪ /‬بكالوريا تجريبي دورة ماي ‪2023‬‬

‫‪ .1.5‬جد بيانيا ارتفاع القمر االصطناعي ‪ Météosat 9‬عن سطح األرض‪ ،‬واستنتج سرعتو ‪ v‬في مداره ‪.‬‬
‫مستقر‪.‬‬
‫ا‬ ‫‪ .2.5‬احسب دور القمر االصطناعي ‪ ، Météosat 9‬واذكر إن كان قم ار جيو‬

‫انتمرٌه انتجرٌبً‪ 06( :‬وقبط)‬


‫في حصة اعمال مخبرية أحضر أستاذ مادة العموم الفيزيائية قارورة ماء جافيل وأراد أن يتأكد مع تالميذه من صحة‬
‫ممصق القارورة ‪،‬لكن قبل بداية التجربة قام األستاذ باإلجابة عمى بعض أسئمة التالميذ التي‬
‫ة‬ ‫دونة عمى‬
‫الكتابة ‪ 12°Chl‬الم ّ‬
‫تمحورت حول تعريف ماء جافيل وماذا تعني الداللة ) ‪ ، (°Chl‬فكانت اجابتو كما يمي‪:‬‬
‫‪ ‬ماء جافيل ىو محمول يحتوي عمى شوارد الكمور ) ‪ Cl - (aq‬وشوارد الييبوكموريت ) ‪ ، ClO - (aq‬يتم تحضيره بانحالل‬
‫تفاعل تام ننمذجو بالمعادلة الكيميائية‬ ‫غاز ثنائي الكمور في كمية وافرة من محمول ىيدروكسيد الصوديوم‪ ،‬وفق‬
‫التالية‪Cl 2 ( g ) + 2HO - (aq ) = ClO - (aq ) + Cl - (aq ) + H 2O (l ).......(1):‬‬
‫‪ ‬الدرجة الكمورومترية ) ‪ (°Chl‬لماء جافيل ىي حجم غاز ‪ Cl2‬مقاسا في الشرطين النظاميين) ‪(VM = 22, 4.L / mol‬‬
‫الذي يجب استعمالو لتحضير ‪ 1.L‬منمحمولو‪.‬‬
‫‪-I‬الدرجة الكمورومترية ) ‪(°Chl‬‬
‫بين أن الدرجة الكمورومترية تعطى بالعالقة التالية ‪.: °Chl = C 0 ×V M‬‬
‫‪ .1‬انطالقا من معادلة التفاعل (‪ّ )1‬‬
‫‪ .2‬احسب التركيز المولي ‪ c 0‬الموافق لمممصقة المشار الييا بالداللة " ‪. "12°Chl‬‬
‫‪ -II‬التأكد من الدرجة الكمورومترية‪:‬‬
‫حضراألستاذ مع تالميذه محموال ) ‪ (S‬حجمو‬
‫‪ّ c0‬‬ ‫‪ .1‬انطالقا من محمول ماء جافيل تجاري الذي تركيزه المولي‬
‫أضافلييكمية من يود‬
‫إ‬ ‫‪،‬تم أخذ حجم ‪ V1 = 20.mL‬من المحمول الممددو‬‫‪ V = 200 mL‬تركيزه المولي ‪ c‬م مدد ‪ 10‬مرات ّ‬
‫‪:‬‬ ‫البوتاسيوم)) ‪ (K + (aq ) + I - (aq‬بوفرة في وسط حمضي ليتشكل ثنائي اليود وفق تفاعل تام ينمذج بالمعادلة التالية‬
‫)‪ClO - (aq ) + 2I - (aq ) + 2H + (aq ) = I 2 (aq ) + Cl - (aq ) + H 2O ( l )......(2‬‬
‫‪ .1.1‬أنجز جدول تقدم التفاعل )‪ (2‬عمما أن شوارد اليود بوفرة‪.‬‬
‫ثم جد عبارة كمية مادة ثنائي اليود ‪ I 2‬في نياية التفاعل بداللة ‪ c 0‬و ‪.V 1‬‬
‫حدد المتفاعل المحد ‪ّ ،‬‬
‫‪ّ .2.1‬‬
‫‪ .2‬نعاير ثنائي اليود المتشكل في نياية التفاعل بمحمول ثيوكبريتات الصوديوم )) ‪ (2Na+ (aq ) + S 2O32- (aq‬تركيزه‬
‫نحصل عمى التكافؤ من أجل حجم ‪ V E = 21mL‬لثيوكبريتات الصوديوم ‪.‬‬
‫المولي ‪ c 2 = 0,1mol / L‬ف‬
‫‪ .1.2‬اكتب المعادلة االجمالية المنمذجة لتفاعل المعايرة ‪.‬يعطى )) ‪(I 2 (aq ) / I - (aq )) ,: (S 4O62- (aq ) / S 2O32- (aq‬‬
‫‪ .2.2‬أثبت صحة العالقة التالية‪، c 0V 1 = 5c 2V E :‬احسب التركيز المولي ‪. c 0‬‬
‫لممصق ‪.‬‬
‫ة‬ ‫‪ .3‬احسب الدرجة الكمورومترية ‪ °Chl‬ثم قارنيا مع ما كتب عمى ا‬
‫‪ –III‬دراسة التفكك الذاتي لماء جافيل‪:‬‬
‫تفكك الشاردة ‪ ClO -‬ىو تفاعل بطيء جدا وتام ينمذج بالمعادلة التالية‪2ClO - (aq ) = O 2 ( g ) + 2Cl - (aq ).....(3) :‬‬
‫لدراسة ىذا التفاعل أخذ التالميذ حجما ‪V = 100 mL‬من محمول ماء جافيل تركيزه ‪ c = 5´ 10- 2 moL ×L- 1‬وأضافوا إليو‬
‫كمية من شوارد الكوبالت ‪ Co 2+‬فحدث التفاعل المنمذج بالمعادلة (‪، )3‬متابعة حجم غاز ثنائياألكسجين المنطمق مكنتنا‬
‫من رسم المنحنى المقابل في الشكل (‪:)7‬‬

‫صفحة ‪ 9‬من ‪10‬‬


‫اختبار في مادة العلوم الفيزيائية ‪ /‬الشعبة‪ :‬رياضيات ‪ +‬تقني رياضي ‪ /‬بكالوريا تجريبي دورة ماي ‪2023‬‬

‫‪ .1‬ارسم التركيب التجريبي الذي استعممو التالميذ مع توضيع البيانات الالزمة‪.‬‬


‫‪ .2‬اشرح دور شوارد الكوبالت ‪ Co 2+‬في ىذا التفاعل‪.‬‬
‫‪ .3‬أنشئ جدول تقدم التفاعل واحسب قيمة التقدم النيائي‪.‬‬
‫وضح إن‬
‫‪ .4‬احسب حجم الغاز المنطمق في نياية التفاعل‪ّ ،‬‬
‫انتيى التفاعل عند المحظة ‪. t = 40min‬‬
‫يعطى‪.V m = 24 L ×mol :‬‬
‫‪-1‬‬

‫‪ .5‬جد بيانيا زمن نصف التفاعل ‪. t 1/2‬‬


‫ان السرعة الحجمية لمتفاعل تعطى بالعالقة‪:‬‬
‫‪ّ .6‬بين ّ‬
‫× ‪ ، v vol (t ) = 0, 42‬احسب قيمتيا عند المحظة‬
‫) ‪dV (O2‬‬
‫‪dt‬‬
‫ثم استنتج سرعة اختفاء شوارد ‪ ClO -‬عند نفس المحظة‪.‬‬ ‫االبتدائية ّ‬
‫‪ .7‬أعاد التالميذ التجربة السابقة باستعمال كمية اكبر من شوارد الكوبالت ‪، Co 2+‬اذكر كيف تتغير المقادير التالية‪:‬‬
‫‪ ‬السرعة الحجمية االبتدائية‪.‬‬
‫‪ ‬زمن نصف التفاعل‪.‬‬
‫‪ ‬حجم غاز ثنائي االكسجين المنطمق في نياية التفاعل‪.‬‬

‫اوتهى انمىضىع انثبوً‬

‫صفحة ‪ 10‬من ‪10‬‬


‫ يوافق‬1 ‫ ألن التوتر بين طرفي المكثفة يزداد خالل عملية الشحن وبالتالي المنحنى‬Y 1 ‫ يوافق المدخل‬2 ‫ المنحنى‬.1
. Y 2 ‫المدخل‬
‫ بما أن المكثفتان مربوطتين على التسلسل فإنها تتراكم على أحد لبوسيهما نفس عدد اإللكترونات وبالتالي تخزنان‬.2
. q (t )  C 1.uC1 (t )  C 2 .uC 2 (t ) : ‫نفس الشحنة الكهربائية أي‬
. u C 1 ‫ المعادلة التفاضلية لـ‬.3
dq q
Req  uC1   E :‫ و بالتالي‬u Req  uC1  uC 2  E :‫بتطبيق قانون جمع التوترات‬
dt C2
duC1  C 1  dC 1.uC1 C 1.uC1
R eqC 1  1   C1
u  E : ‫بالتالي‬ ‫و‬ R eq  u C1   E : ‫و بالتالي‬
dt  C 2  dt C 2

duC1 1  C1  E
 1  uC1  : ‫و منه‬
dt ReqC 1  C 2  R eqC 1
. ‫و‬ A ‫ إيجاد عبارة الثابتين‬.4
t

d (A  (1  e  )) 1  C1  
t
E
 1   A  (1  e ) 

dt ReqC 1  C 2  R eqC 1
C 2  C1 C  C1 : ‫و بالتالي‬
t t
A   E
e   A 2 Ae  
 R eqC 1C 2 R eqC 1C 2 R eqC 1
C 2  C1 : ‫ بالمطابقة نجد‬A e   C 2  C 1 A 
t
E C 2  C1 
t
: ‫و بالتالي‬
A E  Ae 
C2  R eqC 1C 2 R eqC 1 R eqC 1C 2
R eqC 1C 2 A 
t
C 2  C1 
t
 : ‫و منه‬ e 
 Ae  ‫و‬
A 
EC 2 : ‫ومنه‬
C 2  C1  R eqC 1C 2 C 2  C1
C 2 C 1

: ‫و منه‬
t
EC 2
uC1 (t )   (1  e
ReqC 1C 2
)
C 2  C1
: uC 2 (t ) ‫ استنتاج عبارة‬.5
C1
uC 2 (t )  uC (t ) : ‫ و بالتالي‬q (t )  C 1.uC1 (t )  C 2 .uC 2 (t ) : ‫لدينا‬
C2 1
t
 (1  e ) : ‫و منه‬
EC 1 
uC 2 (t )  
C 2  C1
: i (t ) ‫استنتاج عبارة‬
E t EC 2 C 2  C 1 t
duC1
i (t )  e : ‫ و منه‬i (t )  C 1  C1  e
R eq dt C 2  C 1 R eqC 1C 2
‫‪ .6‬اعتمادا على المنحنيين نجد ‪:‬‬
‫‪ .1.6‬القوة المحركة الكهربائية ‪ E‬للمولد‪.‬‬
‫بتطبيق قانون جمع التوترات في حالة النظام الدائم نجد ‪u Req max  uC1 max  uC 2 max  E :‬‬
‫و بالتالي ‪ 8  2  E :‬و منه ‪E  10V :‬‬
‫‪ .2.6‬شدة التيار األعظمية ‪ I 0‬المار في الدارة ‪.‬‬
‫‪E‬‬ ‫‪10‬‬
‫‪ I 0 ‬ومنه ‪I 0  2 103 A :‬‬ ‫‪‬‬
‫‪R eq 5000‬‬
‫‪ .3.6‬ثابت الزمن المميز للدارة ‪  9ms .‬‬
‫‪ .7‬إيجاد قيمة كل من ‪ R1‬و ‪. R 2‬‬
‫‪8‬‬ ‫‪8‬‬
‫‪R1 ‬‬ ‫‪‬‬ ‫من البيان ‪ 1‬عند اللحظة لدينا ‪ u R1 (0)  R1I 0  8V :‬وبالتالي ‪ 4000  :‬‬
‫‪I 0 2 103‬‬
‫لدينا ‪ Req  R1  R 2 :‬وبالتالي ‪ R 2  Req  R1  5000  4000 :‬ومنه ‪R 2  1000  :‬‬
‫‪C2‬‬
‫‪ ،‬واستنتاج قيمة كل من ‪ C 1‬و ‪.C 2‬‬ ‫‪ .8‬تبيان أنّ ‪ 4‬‬
‫‪C1‬‬
‫لدينا ‪ C 1.uC1 (t )  C 2 .uC 2 (t ) :‬في النظام الدائم يصبح لدينا ‪C 1.uC1 max  C 2 .uC 2 max :‬‬
‫‪C2‬‬ ‫‪C 2 uC1 max 8‬‬
‫‪4‬‬ ‫و منه ‪:‬‬ ‫‪‬‬ ‫‪‬‬ ‫و بالتالي ‪:‬‬
‫‪C1‬‬ ‫‪C 1 uC 2 max 2‬‬
‫‪‬‬ ‫‪9 103‬‬
‫لدينا ‪  Req .C eq :‬و بالتالي ‪:‬‬
‫‪6‬‬
‫‪C eq ‬و منه ‪C eq  1,8 10 F :‬‬ ‫‪‬‬
‫‪R eq‬‬ ‫‪5000‬‬
‫‪CC‬‬
‫و لدينا ‪C 2  4C 1 :‬‬ ‫‪C eq‬‬ ‫و بما أن المكثفتان مربوطتين على التسلسل فإن ‪ 1 2 :‬‬
‫‪C1  C 2‬‬
‫‪C eq‬‬‫‪1,8 106‬‬ ‫‪4C 1 C 1‬‬ ‫‪4C 12‬‬
‫‪C1 ‬‬ ‫‪‬‬ ‫‪C eq ‬و بالتالي ‪:‬‬ ‫‪‬‬ ‫و بالتالي ‪ 0,8C 1 :‬‬
‫‪0,8‬‬ ‫‪0,8‬‬ ‫‪C 1  4C 1 5C 1‬‬
‫و منه ‪ C 1  2, 25 106 F :‬و ‪C 2  4C 1  4  2,25 106‬و منه ‪C 2  9 106 F :‬‬
‫‪ .9‬الطاقة الكهربائية المخزّنة في كل مكثفة في نهاية عملية الشحن‪.‬‬
‫‪1‬‬ ‫‪1‬‬
‫‪Ec1max  7, 2 105 J‬‬ ‫و منه ‪:‬‬ ‫‪Ec1max  C 1.u c21 max  2, 25 106  82‬‬
‫‪2‬‬ ‫‪2‬‬
‫‪1‬‬ ‫‪1‬‬
‫‪Ec 2max  1,8 105 J‬‬ ‫و منه ‪:‬‬ ‫‪Ec 2max  C 2 .u c22 max  9 106  22‬‬
‫‪2‬‬ ‫‪2‬‬
‫المرحلة األولى‪ :‬دراسة حركة جملة ميكانيكية على مستو مائل‪.‬‬
‫‪x‬‬
‫‪F1‬‬ ‫‪R‬‬ ‫‪ .1‬تمثيل القوى الخارجية المطبقة على السيارة خالل الحركة‪.‬‬
‫‪0.25‬‬ ‫‪f‬‬
‫'‪x‬‬ ‫الجملة المدروسة‪ :‬سيارة‪ .‬مرجع الدراسة ‪ :‬سطحي أرضي نغتبره غاليليا‪.‬‬
‫‪P‬‬
‫باعتبار السيارة نقطة مادية ‪.‬‬
‫‪0.25‬‬ ‫‪ .2‬بتطبيق القانون الثاني لنيوتن على السيارة لدينا ‪ Fext  m .a :‬و بالتالي ‪F1  f  P  R  m .a :‬‬
‫‪F f‬‬
‫‪0.25‬‬ ‫‪a 1‬‬ ‫باإلسقاط على المحور ) ‪ (x ' x‬نجد ‪ F1  f  P .sin   m .a :‬و منه ‪ g .sin  :‬‬
‫‪m‬‬
‫‪ .1.3‬تحديد طبيعة الحركة‪.‬‬
‫بما أن الجملة المدروسة ( سيارة ‪ +‬أرض ) وهناك تغير في االرتفاع فإن العمود الممثل داخل الحصيلة خاص بتغير ‪0.25‬‬
‫‪0.25‬‬ ‫الطاقة الكامنة الثقالية فقط وبالتالي نستنتج أن ‪ Ec‬ال تتغير ومنه نستنتج أن ‪ v=cste‬ومنه الحركة مستقيمة منتظمة‪.‬‬
‫‪.2.3‬إيجاد شدة القوة المحركة للسيارة ‪. F1‬‬
‫‪0.25‬‬
‫‪W (F1 ) 180  103‬‬
‫‪0.25‬‬ ‫‪ F1 ‬و منه‪F1  1500N :‬‬ ‫‪‬‬ ‫من الحصيلة الطاقوية لدينا‪ W (F1 )  F1 .AB :‬وبالتالي‪:‬‬
‫‪AB‬‬ ‫‪120‬‬
‫‪ .3.3‬تبيان أن شدة قوة االحتكاك هي ‪. f  269,5 N‬‬
‫‪F1  f‬‬ ‫‪F f‬‬
‫‪0.5‬‬ ‫‪ a  1‬وبما أن الحركة مستقيمة منتظمة فإن‪ a  0 :‬وبالتالي ‪ g .sin   0 :‬‬ ‫لدينا ‪ g .sin  :‬‬
‫‪m‬‬ ‫‪m‬‬
‫‪0.25‬‬ ‫وبالتالي‪ f  F1  mg .sin   1500  1800  9,8sin 4 :‬ومنه‪f  269,5 N :‬‬
‫‪0.25‬‬ ‫‪ - .4.3‬المقدار ‪ C‬يمثل ‪.EppB‬‬
‫‪0.25‬‬ ‫‪ -‬المقدار ‪ D‬يمثل قيمة عمل قوة االحتكاك ) ‪. W (f‬‬
‫‪0.25‬‬
‫لدينا ‪ W (f )  f .AB  269,5  120 :‬ومنه ‪D  W (f )  32340J :‬‬

‫بتطبيق مبدأ انحفاظ الطاقة لدينا على الجملة ( سيارة ‪ +‬أرض ) بين الموضعين ‪ A‬و ‪ B‬لدينا ‪:‬‬
‫‪0.5‬‬ ‫‪ W (F1 )  W (f )  Epp B‬وبالتالي ‪ Epp B  180000  32340 :‬ومنه ‪Epp B  C  147660J :‬‬
‫المرحلة الثانية‪ :‬دراسة حركة جملة ميكانيكية على مستو أفقي‪.‬‬
‫‪R‬‬
‫‪0.25‬‬ ‫'‪x‬‬ ‫‪f‬‬ ‫‪F2‬‬ ‫‪x‬‬ ‫‪ .1‬تمثيل القوى الخارجية المطبقة على السيارة خالل الحركة‪.‬‬
‫‪F f‬‬
‫‪P‬‬ ‫‪a 2‬‬ ‫‪ .2‬إثبات أن عبارة التسارع تعطى بالعالقة ‪:‬‬
‫‪m‬‬
‫بتطبيق مبدأ انحفاظ الطاقة على الجملة ( سيارة ) نجد ‪:‬‬
‫‪0.25‬‬
‫‪F f‬‬ ‫‪1‬‬ ‫‪1‬‬ ‫‪1‬‬
‫‪ 2‬و‬ ‫‪ Ec 0 W (F2 )  W (f )  Ec‬و بالتالي ‪ mv 02  F2 .d  f .d  mv 2 :‬و بالتالي ‪d  (v 2  v 02 ) :‬‬
‫‪m‬‬ ‫‪2‬‬ ‫‪2‬‬ ‫‪2‬‬
‫‪F f‬‬ ‫‪F2  f‬‬ ‫‪1‬‬
‫‪0.25‬‬ ‫‪a 2‬‬ ‫و منه ‪:‬‬ ‫من محذوفية الزمن لدينا ‪ v 2  v 02  2a.d :‬و بالتالي ‪d  (2a.d ) :‬‬
‫‪m‬‬ ‫‪m‬‬ ‫‪2‬‬
‫‪ .3‬إيجاد بيانيا قيمة التسارع ‪:‬‬
‫‪v 25  15‬‬
‫‪0.25‬‬ ‫‪ a ‬و منه ‪a  1m / s 2 :‬‬ ‫‪‬‬ ‫‪ -‬التسارع يمثل معامل توجيه البيان و بالتالي ‪:‬‬
‫‪t‬‬ ‫‪10  0‬‬

‫‪ -‬استنتاج شدة القوة ‪. F2‬‬


‫‪F2  f‬‬
‫‪0.25‬‬ ‫‪F2  2069,5N‬‬ ‫و منه ‪:‬‬ ‫‪F2  a.m  f  11800  269,5‬‬ ‫‪ a ‬و بالتالي ‪:‬‬ ‫لدينا ‪:‬‬
‫‪m‬‬
‫‪F2  f‬‬ ‫‪dv‬‬
‫‪0.25‬‬ ‫‪v (t ) ‬‬ ‫‪ a ‬بالمكاملة نجد ‪ v (t )  a.t  v 0 :‬و منه ‪.t  v 0 :‬‬ ‫‪ .‬المعادلتين الزمنيتين ‪:‬‬
‫‪m‬‬ ‫‪dt‬‬
‫‪F f 2‬‬
‫‪0.25‬‬ ‫‪x (t )  2‬‬ ‫بالمكاملة نجد ‪.t  v 0 .t :‬‬
‫‪2m‬‬
‫‪ .1.5‬اللحظة الزمنية ‪ t 1 ‬الموافقة الشتغال اإلنذار‪.‬‬
‫‪90  1000‬‬
‫‪0.25‬‬ ‫‪ v 1 ‬باإلسقاط بيانيا نجد ‪. t1  10s :‬‬ ‫طريقة (‪ : )1‬لدينا ‪ 25m / s :‬‬
‫‪3600‬‬
‫‪v 1  v 0 25  15‬‬
‫‪ t 1 ‬و منه ‪t1  10s :‬‬ ‫‪‬‬ ‫طريقة (‪ : )2‬بالتعويض في المعادلة الزمنية للسرعة نجد ‪:‬‬
‫‪a‬‬ ‫‪1‬‬
‫‪ .2.5‬حساب المسافة المقطوعة بين اللحظتين ‪ t  0‬و ‪. t 1‬‬
‫‪0.25‬‬ ‫‪(15  25)  10‬‬
‫‪ d ‬و منه ‪d  200m :‬‬ ‫المسافة المقطوعة تمثل مساحة شبه المنحرف و بالتالي ‪:‬‬
‫‪2‬‬

‫‪ .1‬تعريف ظاهرة النشاط اإلشعاعي‪ :‬هي ظاهرة تلقائية وعشوائية وحتمية يحدث فيها تحول لألنوية المشعة فتنتج‬
‫أنوية أكثر استقرارا مع انبعاث جسيمات‪.‬‬
‫‪.2 .1‬على جزء من مخطط ) ‪ . (Z  N‬تصنيف األنوية التالية حسب نمط تفككها إلى ‪  ‬أو ‪  ‬أو أنوية مستقرة‪:‬‬
‫‪17‬‬ ‫‪11‬‬ ‫‪13‬‬ ‫‪14‬‬ ‫‪10‬‬ ‫‪12‬‬ ‫‪A‬‬
‫‪8‬‬
‫‪O‬‬ ‫‪5‬‬
‫‪B‬‬ ‫‪7‬‬
‫‪N‬‬ ‫‪6‬‬
‫‪C‬‬ ‫‪5‬‬
‫‪B‬‬ ‫‪6‬‬
‫‪C‬‬ ‫‪Z‬‬
‫‪X‬‬
‫‪Z‬‬

‫‪N A Z‬‬

‫) ‪(Z  N‬‬

‫‪‬‬ ‫‪‬‬
‫و ‪ 5 B‬و ‪ 5 B‬و ‪.8 O‬‬
‫‪17‬‬ ‫‪11‬‬ ‫‪10‬‬
‫‪ -‬األنوية المستقرة هي التي تقع في وادي االستقرار في المخطط ) ‪ (Z  N‬وهي ‪C :‬‬
‫‪12‬‬
‫‪6‬‬

‫‪ -‬األنوية غير المستقرة التي تقع فوق وادي االستقرار في المخطط ) ‪ (Z  N‬هي ‪ 7 N‬و تتفكك وفق النمط ‪.  ‬‬
‫‪13‬‬

‫‪ -‬األنوية غير المستقرة التي تقع تحت وادي االستقرار في المخطط ) ‪ (Z  N‬هي ‪ 6 C‬تتفكك وفق النمط ‪.  ‬‬
‫‪14‬‬
‫‪. 146C  147 N   ‬‬ ‫‪ .3‬معادلة تفكك نواة ‪: 6 C‬‬
‫‪14‬‬

‫‪ .1.4‬عبارة قانون التناقص اإلشعاعي هي‪. N t   N 0 e   t :‬‬


‫‪: A (t )  A0e‬‬
‫‪ t‬‬
‫تبيان أن عبارة النشاط اإلشعاعي للعينة تكتب بالعالقة‪:‬‬

‫‪. A t   A0 e‬‬ ‫‪t‬‬


‫‪ A t   ‬ولـما ‪ t  0‬نجد‪ A0   N 0 :‬أي‪:‬‬
‫‪dN t ‬‬
‫‪‬‬
‫‪d N 0 e t‬‬‫‪‬‬ ‫‪‬‬
‫لدينا‪  N 0 e   t :‬‬
‫‪dt‬‬ ‫‪dt‬‬
‫المدلول الفيزيائي للثابتين ‪ A 0‬و ‪: ‬‬
‫‪ : ‬ثابت التفكك ٍ( ثابت النشاط اإلشعاعي )‪.‬‬ ‫‪،‬‬ ‫‪ : A 0‬النشاط اإلشعاعي االبتدائي‬
‫‪ .2.4‬تحديد عمر المخطوطة‪:‬‬
‫‪ A (t ) ‬‬ ‫‪A t ‬‬
‫‪ln ‬‬ ‫‪‬‬ ‫‪‬‬ ‫‪‬‬ ‫‪‬‬ ‫‪t‬‬ ‫ومنه‪:‬‬ ‫لدينا‪ A t   A0e   t :‬ومنه‪ e   t :‬‬
‫‪ A0 ‬‬ ‫‪A0‬‬
‫‪1‬‬ ‫‪t1/2‬‬ ‫‪ln  2 ‬‬ ‫‪1‬‬ ‫‪ A ‬‬ ‫‪1‬‬ ‫‪ A (t ) ‬‬
‫‪.‬‬ ‫‪‬‬ ‫‪  ‬ومنه‪:‬‬ ‫‪ t    ln ‬أي‪ t   ln  0  :‬و نعلم أن‪:‬‬ ‫أي‪ :‬‬
‫‪‬‬ ‫‪ln  2 ‬‬ ‫‪t1/2‬‬ ‫‪‬‬ ‫‪ A (t ) ‬‬ ‫‪‬‬ ‫‪ A0 ‬‬
‫‪.‬‬ ‫‪t 1/ 2‬‬ ‫‪ A 0  5730‬‬ ‫‪ 256  10 3 ‬‬ ‫إذن‪:‬‬
‫‪t ‬‬ ‫‪ ln ‬‬ ‫‪‬‬ ‫‪ ln ‬‬ ‫‪3 ‬‬
‫‪ 363,1ans‬‬
‫‪ln  2 ‬‬ ‫‪ A (t )  0,693‬‬ ‫‪ 245  10 ‬‬
‫استنتاج إلى أي سنة تعود‪ :‬لدينا‪. 2023  363,1  1659,9ans :‬‬
‫‪.1.5‬تحديد قيمة زمن نصف العمر للنظير ‪: 7 N‬‬
‫‪13‬‬

‫‪N 0 2,8  10 11‬‬


‫‪ N t 1/2  ‬في‬ ‫‪‬‬ ‫زمن نصف العمر ‪ t 1/2‬يوافق فاصلة النقطة ذات الترتيبة ‪ 1,4  10 11 noys‬‬
‫‪2‬‬ ‫‪2‬‬
‫البيان ‪ N  f t ‬وباإلسقاط نقرأ‪. t1/2 10min  600s :‬‬
‫‪ln  2  0,693‬‬
‫‪. ‬‬ ‫حساب قيمة ثابت النشاط اإلشعاعي ‪ ‬بوحدة ) ‪ 1,155  10 3 s 1 : (s‬‬
‫‪1‬‬
‫‪‬‬
‫‪t1/2‬‬ ‫‪600‬‬
‫‪ .2.5‬عدد األنوية االبتدائية المشعة في الحقنة‪ :‬من البيان ‪ N  f t ‬و لـما ‪ t  0‬نقرأ‪. N 0  2,8 1011 noys :‬‬
‫اسنتناج نشاطها االبتدائي‪. A0   N 0  1,155  10 3  2,8 1011  3,234  108 Bq :‬‬
‫‪ .3.5‬حساب المدة الزمنية الالزمة إلنهاء فعالية الحقنة‪:‬‬
‫‪1‬‬ ‫‪ A (t ) ‬‬
‫‪ ، t    ln ‬ونعلم أن نشاط العينة تناقص بـ ‪ 95%‬أي‪ :‬تبقى ‪ 5%‬من نشاطها‬ ‫لدينا مما سبق‪ :‬‬
‫‪‬‬ ‫‪ 0 ‬‬
‫‪A‬‬
‫‪5‬‬
‫‪. A (t ) ‬‬ ‫االبتدائي ونكتب‪ A 0  0,05A 0 :‬‬
‫‪100‬‬
‫‪1‬‬
‫‪.t ‬‬ ‫أي‪ ln  0,05  2593,71s  43,23min :‬‬
‫‪1,155  10 3‬‬
‫‪ .4.5‬التوقيت الموافق لحقن المريض بالحقنة الثانية ‪:‬‬
‫‪t min  9 h 00min 00 s  43min 13s  9 h 43min13s‬‬
‫‪ .1‬تفاعل أكسدة ‪ -‬إرجاع ‪:‬هو تفاعل يحدث فيه انتقال اإللكترونات من مرجع ثنائية ) ‪ (Ox 1 / Réd 1‬إلى مؤكسد ثنائية‬
‫‪0.25‬‬
‫أخرى ) ‪. (Ox 2 / Réd 2‬‬
‫‪0.25‬‬ ‫الثائيتان الداخلتان في التفاعل ‪ (Cu 2 / Cu ) :‬و ) ‪. (Ag  / Ag‬‬
‫‪ .2‬جدول تقدم التفاعل‪:‬‬
‫معادلة التفاعل‬ ‫)𝑠(𝑔𝐴‪𝐶𝑢(𝑠) + 2𝐴𝑔+ (𝑎𝑞) = 𝐶𝑢2+ (𝑎𝑞) + 2‬‬
‫الحالة‬ ‫التقدم‬ ‫كمية المادة بـ ‪mol‬‬
‫االبتدائية‬ ‫‪x 0‬‬ ‫‪m0‬‬ ‫‪cV‬‬ ‫‪0‬‬ ‫‪0‬‬
‫‪0.25‬‬
‫‪M‬‬
‫االنتقالية‬ ‫) ‪x (t‬‬ ‫‪m0‬‬ ‫‪cV  2x‬‬ ‫‪x‬‬ ‫‪2x‬‬
‫‪x‬‬
‫‪M‬‬
‫النهائية‬ ‫‪xf‬‬ ‫‪m0‬‬ ‫‪cV  2x f‬‬ ‫‪xf‬‬ ‫‪2x f‬‬
‫‪xf‬‬
‫‪M‬‬

‫‪x‬‬ ‫‪m Ag t ‬‬ ‫‪3‬‬


‫‪m Ag t ‬‬ ‫‪m Ag t ‬‬
‫‪ n Ag t  ‬ومنه‪ 2 x :‬‬ ‫ونعلم أن‪:‬‬ ‫لدينا من جدول تقدم التفاعل‪n Ag t   2 x :‬‬
‫‪0.25‬‬ ‫‪M  Ag ‬‬ ‫‪M  Ag ‬‬
‫أي‪. m Ag t   2 x M  Ag  :‬‬
‫‪m f  Ag ‬‬
‫‪xf ‬‬ ‫ومنه‪:‬‬ ‫‪ x f‬نعلم أن‪m f  Ag   2 x f M  Ag  :‬‬
‫‪0.25‬‬ ‫‪2 M  Ag ‬‬
‫‪1,08‬‬
‫‪. xf ‬‬ ‫ومن البيان نقرأ‪ m f  Ag   1,08 g :‬أي‪ 5  10 3 mol :‬‬
‫‪2  108‬‬
‫‪4‬‬
‫‪m0‬‬ ‫‪3,81‬‬
‫‪n f Cu  ‬‬ ‫‪xf ‬‬ ‫لدينا من جدول تقدم التفاعل‪ 5  10 3  5,5  10 2 mol :‬‬
‫‪0.25‬‬ ‫‪M Cu ‬‬ ‫‪63,5‬‬
‫بما أن‪ nf Cu   0 :‬والتحول الكيميائي الحادث تام فإن شوارد الفضة ‪ Ag  aq ‬هي المتفاعل المحد ‪.‬‬
‫‪2x f 2  5  10 3‬‬
‫‪0.25‬‬ ‫‪.c ‬‬ ‫‪‬‬ ‫‪ c‬نعلم أن‪ cV  2x f  0 :‬ومنه‪ 0,1mol / L :‬‬
‫‪V‬‬ ‫‪100  10 3‬‬
‫‪mf‬‬
‫‪ t1/2‬لدينا مما سبق‪. m t   2 x M :‬‬ ‫‪m t1/2  ‬‬ ‫‪5‬‬
‫‪2‬‬
‫‪xf‬‬
‫‪x t1/2  ‬‬ ‫لما ‪ t  t1/2‬نجد‪ m t1/2   2 x t1/2  M :‬حيث‪:‬‬
‫‪0.25‬‬
‫‪2‬‬
‫‪m‬‬
‫ومنه‪ m t1/2   x f M :‬ونعلم أن‪x f  f :‬‬
‫‪2M‬‬
‫‪m‬‬ ‫‪1,08‬‬ ‫‪m‬‬
‫‪m t1/2   f ‬‬ ‫ومنه‪ m t1/2   f M :‬أي‪ 0,54 g :‬‬
‫‪2‬‬ ‫‪2‬‬ ‫‪2M‬‬
‫‪0.25‬‬
‫و باإلسقاط نقرأ‪. t1/2  11min :‬‬
‫‪‬‬ ‫‪‬‬
‫‪ .6‬تبيان أن السرعة الحجمية الختفاء شوارد الفضة تعطى بالعالقة ‪: v vol Ag   9,26  10 2 dm / dt :‬‬

‫‪1 d n Ag‬‬‫‪‬‬ ‫‪‬‬


‫‪‬‬
‫‪0.25‬‬ ‫‪n Ag‬‬ ‫‪‬‬ ‫‪‬‬
‫‪  cV‬‬ ‫‪ v vol Ag‬ولدينا‪ 2 x :‬‬ ‫‪‬‬ ‫‪‬‬
‫‪‬‬ ‫‪ ‬‬
‫‪V‬‬ ‫‪dt‬‬
‫نعلم أن‪:‬‬

‫‪1 d cV  2 x ‬‬


‫‪‬‬
‫‪v vol Ag   ‬‬
‫‪V‬‬
‫‪2 dx‬‬
‫‪dt‬‬
‫‪‬‬‫‪ v vol Ag ‬ومنه نجد‪:‬‬ ‫‪‬‬ ‫‪‬‬ ‫‪ ‬‬
‫‪V‬‬ ‫‪dt‬‬
‫ومنه‪:‬‬

‫‪ v vol‬نجد‪:‬‬ ‫‪‬‬ ‫‪‬‬


‫‪Ag   ‬‬
‫‪V‬‬
‫‪2 dx‬‬
‫‪dt‬‬
‫‪ x ‬وبالتعويض في‪:‬‬
‫‪2M‬‬
‫‪m‬‬
‫ولدينا مما سبق‪ m  2 x M :‬ومنه‪:‬‬

‫‪ m ‬‬
‫‪d‬‬
‫‪2 M ‬‬
‫‪‬‬
‫‪v vol Ag ‬‬ ‫‪‬‬ ‫‪2‬‬
‫‪ ‬‬
‫‪V‬‬
‫‪‬‬
‫‪dt‬‬
‫‪‬‬
‫‪2‬‬
‫‪2M V‬‬
‫‪‬‬
‫‪dm‬‬
‫‪dt‬‬
‫ومنه‪:‬‬

‫‪‬‬ ‫‪‬‬
‫‪0.25‬‬ ‫‪1‬‬ ‫‪dm‬‬ ‫‪1‬‬ ‫‪dm‬‬ ‫‪2 d m‬‬
‫‪v vol Ag  ‬‬ ‫‪‬‬ ‫‪‬‬ ‫‪‬‬ ‫‪‬‬ ‫‪9,26‬‬ ‫‪‬‬ ‫‪10‬‬ ‫‪‬‬ ‫أي‪:‬‬
‫‪MV‬‬ ‫‪dt‬‬ ‫‪108  100  10 3‬‬ ‫‪‬‬‫‪dt‬‬ ‫‪‬‬
‫‪dt‬‬
‫حساب قيمتها عند اللحظتين ‪ t  20min‬و ‪ ، t  40min‬مع تدوين االستنتاح‪:‬‬
‫‪ -‬عند اللحظة ‪: t  20min‬‬
‫‪1,08  0,414   1,8 10 3 mol / L .min‬‬
‫‪0.25‬‬
‫‪‬‬ ‫‪‬‬
‫‪v vol Ag   9,26  10 2 ‬‬
‫‪ 35  0 ‬‬
‫‪ -‬عند اللحظة ‪: t  40min‬‬
‫‪1,152  0,9   5,4 10 4 mol / L .min‬‬
‫‪‬‬
‫‪v vol Ag   9,26  10 2 ‬‬‫‪‬‬ ‫‪ 65  22 ‬‬
‫‪0.25‬‬ ‫نستنتج أن‪ :‬السرعة الحجمية الختفاء شوارد الفضة ‪ Ag  aq ‬تنقص بمرور الزمن‪.‬‬
‫‪ .7‬نعيد التجربة السابقة باستعمال نفس الكتلة من النحاس على شكل قطع مجزأة‪:‬‬
‫‪.1.7‬السرعة الحجمية الختفاء شوارد الفضة تزداد و زمن نصف التفاعل ينقص ألن استعمال معدن النحاس على شكل‬
‫‪0.25‬‬
‫قطع مجزأة يزيد من عدد التصادمات الفعالة بين المتفاعالت في وحدة الحجم وعليه زيادة السرعة الحجمية للتفاعل‬
‫ليبلغ التحول الكيميائي حالته النهائية في زمن أقل‪.‬‬
‫‪0.25‬‬ ‫‪ .2.7‬العامل الحركي المدروس هو سطح االتالمس‪.‬‬
‫‪ -‬رسم كيفيا على نفس المعلم البيان المتحصل عليه في هذه الحالة‪:‬‬
‫‪m Ag  g ‬‬

‫‪1‬‬
‫‪ 2‬‬ ‫‪.‬‬ ‫البيان ‪ 1‬حالة صفيحة نحاس مجزأة‬
‫البيان ‪  2 ‬حالة صفيحة من النحاس ‪.‬‬
‫‪0.25‬‬
‫‪.1 -II‬رسم شكل العمود مبينا إشارتي القطبين وجهة مرور اإللكترونات‪:‬‬
‫‪K‬‬ ‫‪I‬‬
‫‪A‬‬

‫‪K‬‬ ‫‪‬‬
‫‪ NO3‬‬ ‫‪‬‬ ‫‪.‬‬
‫جسر ملحي‬
‫‪‬‬ ‫‪‬‬
‫‪0.25‬‬
‫صفيحة الفضة ‪Ag  s ‬‬ ‫صفيحة النحاس ‪Cu  s ‬‬

‫‪ Ag‬‬ ‫‪‬‬


‫‪‬‬
‫محلول نترات الفضة ‪ NO3‬‬ ‫محلول نترات النحاس‬
‫‪Cu‬‬ ‫‪2‬‬
‫‪ 2 NO3‬‬ ‫‪‬‬
‫‪0.25‬‬ ‫‪ .2‬الرمز االصطالحي للعمود المدروس‪.    Cu /Cu 2 / / Ag  / Ag    :‬‬
‫‪.3‬جدول تقدم التفاعل الحادث‪:‬‬

‫(القطب السالب )تحدث عملية أكسدة‪. Cu 2 (aq) / Cu(s) : Cu (s)  Cu 2 (aq)  2 e :‬‬
‫‪‬‬

‫( القطب الموجب ) تحدث عملية إرجاع ‪Ag  (aq) / Ag (s):  Ag  (aq)  1e  Ag (s)   2 :‬‬
‫‪‬‬

‫‪‬‬ ‫‪‬‬
‫‪‬‬ ‫‪2‬‬
‫‪0.25‬‬ ‫معادلة التفاعل أكسدة إرجاع‪. Cu(s)  2 Ag (aq)  Cu (aq)  2 Ag (s) :‬‬

‫)‪Cu ( s)  2 Ag  (aq)  Cu 2 (aq)  2 Ag ( s‬‬


‫الحالة االبتدائية‬ ‫‪ncu‬‬ ‫‪c 2V 2‬‬ ‫‪c1V 1‬‬ ‫‪n Ag‬‬
‫‪0.25‬‬ ‫الحالة االنتقالية‬ ‫‪ncu  x‬‬ ‫‪c 2V 2  2x‬‬ ‫‪c1V 1  x‬‬ ‫‪n Ag  2x‬‬
‫الحالة النهائية‬ ‫‪ncu  x f‬‬ ‫‪c 2V 2  2x f‬‬ ‫‪c1V 1  x‬‬ ‫‪n Ag  2x f‬‬

‫‪Cu 2 ‬‬
‫‪‬‬ ‫‪ i  c1  0,5  50‬‬
‫‪0.25‬‬ ‫‪.Q ri ‬‬ ‫حساب كسر التفاعل االبتدائي ‪ :Q ri‬لدينا‪:‬‬
‫‪ 2‬‬
‫‪ Ag ‬‬
‫‪‬‬ ‫‪i‬‬
‫‪ 2  ‬‬
‫‪c‬‬
‫‪2‬‬
‫‪0,1‬‬
‫‪2‬‬

‫‪0.25‬‬ ‫‪ .4‬بما أن الجملة تتطور في االتجاه المباشر فإن‪ K  Q ri :‬وعليه‪. K  2,1 1015 :‬‬
‫‪ .5‬حساب كمية الكهرباء المنتجة للعمود‪:‬‬

‫‪3‬‬
‫‪0.25‬‬ ‫حيث‪ x f  5 10 mol :‬و ‪. Z  2‬‬ ‫نعلم أن ‪Q  Z .x f .F :‬‬

‫ومنه‪. Q  2  5 10 3  96500  965C :‬‬


‫استنتاج المدة الزمنية ‪ t‬الشتغال العمود‪:‬‬
‫‪965‬‬
‫‪. t ‬‬ ‫ومنه‪ 482500 s  134, 03 h :‬‬ ‫نعلم أن ‪Q  I .t :‬‬
‫‪0.25‬‬ ‫‪2 10 3‬‬
‫‪.1- I‬تعريف االنشطار النووي‪ :‬هو تحول نووي مفتعل يتم قذف نواة انشطارية ثقيلة بنيترون فتنتج نواتين أخف‬
‫وأكثر استقرارا وتحرير طاقة ونيترونات‪.‬‬
‫‪300‬‬ ‫‪0,1m s‬‬ ‫‪15m‬‬ ‫‪.2‬العبارة األولى‪:‬‬
‫الشرح‪ :‬تحول االنشطار النووي سريع يحرر طاقة كبيرة تظهر على شكل حرارة عالية‪.‬‬
‫العبارة الثانية‪:‬‬
‫الشرح‪ :‬تحول االنشطار النووي من سلبياته ينتج عنه نفايات نووية مشعة ملوثة للبيئة‪.‬‬
‫‪ .1.3‬قيمة الرقم الذري ‪ Z‬و العدد الكتلي ‪ A‬للنواة ‪: ZAU‬‬
‫ونكتب‪. Z  92 :‬‬ ‫‪A‬‬
‫‪ZU‬‬ ‫لدينا‪ :‬عدد البروتونات هو ‪ ، 92 p‬فهو يمثل الرقم الذري ‪ Z‬لنواة اليوارنيوم‬
‫ومنه‪. A 143  92  235 :‬‬ ‫حيث عدد النترونات‪N  144  1  143 :‬‬ ‫نعلم أن‪A  Z  N :‬‬
‫قيمة كل من ‪ x‬و ‪: y‬‬
‫‪x  236  233  3‬‬ ‫ومنه‪:‬‬ ‫‪235  1  139  94  x‬‬ ‫بتطبيق قانوني االنحفاظ لصودي نجد‪:‬‬
‫‪‬‬ ‫‪‬‬
‫‪ y  92  38  54‬‬ ‫‪92  y  38‬‬

‫‪ .2.3‬استنتاج معادلة تفاعل االنشطار النووي الحادث‪54 Xe  38 Sr  3 0 n :‬‬


‫‪ 01n  139‬‬
‫‪235‬‬ ‫‪94‬‬ ‫‪1‬‬
‫‪92U‬‬

‫‪: 139‬‬
‫‪.3 .3‬حساب طاقة الربط لكل من النواتين ‪ 92U‬و ‪54 Xe‬‬
‫‪235‬‬

‫‪El‬‬ ‫‪‬‬ ‫‪235‬‬


‫‪92U‬‬ ‫‪  m ‬‬ ‫‪235‬‬
‫‪92U‬‬ ‫‪.c‬‬ ‫‪2‬‬
‫‪ m‬‬ ‫‪‬‬ ‫‪235‬‬
‫‪92U‬‬ ‫‪  931,5  1,923  931,5  1791,27 MeV‬‬ ‫نعلم أن‪:‬‬ ‫‪‬‬

‫‪m‬‬ ‫‪‬‬
‫حيث من الحصيلة الكتلية نجد‪  237,624  236,001  1,923u :‬‬
‫‪235‬‬
‫‪92U‬‬

‫‪ ‬نعلم أن‪E  Xe   m  Xe .c  m  Xe   931,5 :‬‬


‫‪l‬‬
‫‪139‬‬
‫‪54‬‬
‫‪139‬‬
‫‪54‬‬
‫‪2‬‬ ‫‪139‬‬
‫‪54‬‬

‫و كذلك‪E  Sr   m  Sr .c  m  Sr   931,5 :‬‬


‫‪l‬‬
‫‪94‬‬
‫‪38‬‬
‫‪94‬‬
‫‪38‬‬
‫‪2‬‬ ‫‪94‬‬
‫‪38‬‬

‫ومنه‪E  Xe   E  Sr   m  Xe   m  Sr   931,5 :‬‬


‫‪139‬‬ ‫‪94‬‬ ‫‪139‬‬ ‫‪94‬‬
‫‪l‬‬ ‫‪‬‬ ‫‪54‬‬ ‫‪‬‬ ‫‪l‬‬ ‫‪38‬‬ ‫‪54‬‬ ‫‪38‬‬

‫أي‪E  Xe   m  Xe   m  Sr   931,5  E  Sr  :‬‬


‫‪139‬‬ ‫‪139‬‬ ‫‪94‬‬ ‫‪94‬‬
‫‪‬‬ ‫‪l‬‬ ‫‪54‬‬ ‫‪‬‬ ‫‪54‬‬ ‫‪38‬‬ ‫‪l‬‬ ‫‪38‬‬

‫من الحصيلة الكتلية نجد‪ m  Xe   m  Sr   235,809  237,924   2,115u :‬‬


‫‪139‬‬ ‫‪94‬‬
‫‪‬‬ ‫‪54‬‬ ‫‪‬‬ ‫‪38‬‬

‫حيث‪. E  Sr   8,62  94  810,28 MeV :‬‬ ‫‪l‬‬


‫‪94‬‬
‫‪38‬‬

‫إذن‪E  Xe   2,115  931,5  810,28  1159,84 MeV :‬‬


‫‪l‬‬
‫‪139‬‬
‫‪54‬‬

‫بـ ‪: MeV‬‬ ‫‪235‬‬


‫‪92U‬‬ ‫‪ .4.3‬قيمة الطاقة الناتجة ‪ E lib‬عن انشطار نواة واحدة من اليورانيوم‬
‫‪E lib  m .c 2  m  931,5  235,809  236,001  931,5  178,85 MeV‬‬
‫‪E lib  E l‬‬ ‫‪‬‬ ‫‪139‬‬
‫‪54 Xe‬‬ ‫‪E ‬‬ ‫‪l‬‬
‫‪94‬‬
‫‪38 Sr‬‬ ‫‪E ‬‬ ‫‪l‬‬
‫‪235‬‬
‫‪92U‬‬ ‫‪  1159,84  810,28  1791,27  178,85 MeV‬‬
‫قيمة الطاقة الناتجة ‪ E lib‬بوحدة الجول ‪ J‬هي‪. E lib  178,85 1,6 1013  2,86 1011 J :‬‬
‫‪N‬‬ ‫‪m‬‬
‫‪‬‬ ‫نعلم أن‪ E  N E lib :‬و من العالقة‬ ‫‪ .1.4‬حساب الطاقة المحررة من انشطار الكتلة ‪m  700g‬‬
‫‪NA M‬‬
‫‪m .N A‬‬
‫الموافقة للكتلة ‪m‬‬ ‫‪235‬‬
‫‪92U‬‬ ‫حيث ‪ N‬عدد أنوية‬ ‫‪N ‬‬ ‫نجد‪:‬‬
‫‪M‬‬
‫‪m .N A .E lib 700  6,02  1023  178,85‬‬
‫‪E‬‬ ‫‪‬‬ ‫ومنه‪ 3,2  1026 MeV  5,12  1013 J :‬‬
‫‪M‬‬ ‫‪235‬‬
‫‪ «TNT‬علما أن ‪1g‬من مادة‬ ‫‪12,5‬‬ ‫‪ .2.4‬التحقق بالحساب من العبارة »‬
‫‪ TNT‬تحرر طاقة قدرها ‪: 4,18kJ‬‬
‫‪5,12  1013  1‬‬ ‫‪‬‬
‫‪1 g  4,18  10 J‬‬
‫‪3‬‬
‫‪m‬‬ ‫‪ 12,25  109 g  12,25  106 kg‬‬ ‫ومنه‪:‬‬ ‫‪‬‬ ‫نعلم أن‪:‬‬
‫‪4,18  10‬‬ ‫‪m  g   5,12  10‬‬
‫‪3‬‬ ‫‪13‬‬
‫‪‬‬
‫أي‪ m  12,25 k toone :‬إذن‪ :‬يمكن اعتبار أن العبارة صحيحة ‪.‬‬ ‫حيث‪1 toone  103 kg :‬‬
‫‪94‬‬
‫‪38 Sr‬‬ ‫‪‬‬ ‫'‪A‬‬
‫‪Z 'Y‬‬ ‫‪ .1 - II‬معادلة التفكك اإلشعاعي للسترونتيوم ‪: 94‬لدينا‪ 10e :‬‬
‫‪A '  94‬‬
‫‪. 38‬‬
‫‪94‬‬
‫‪Sr  39‬‬
‫‪94‬‬
‫ونكتب‪Y    :‬‬ ‫‪‬‬ ‫وبتطبيق قانوني االنحفاظ لصودي نجد‪:‬‬
‫‪ Z '  38  1  39‬‬
‫‪.1.2‬عبارة قانون التناقص اإلشعاعي‪. N t   N 0 e   t :‬‬
‫‪dN t ‬‬
‫‪ :‬بتعويض العبارة ‪N t   N 0 e   t‬‬ ‫التأكد أنه حل للمعادلة التفاضلية ‪  N t   0‬‬
‫‪dt‬‬
‫‪.‬‬ ‫‪ N 0e‬‬ ‫‪t‬‬
‫‪‬‬
‫‪‬‬
‫‪d N 0 e t‬‬ ‫‪ N‬‬ ‫‪t‬‬
‫في المعادلة التفاضلية نجد‪  N 0 e   t  0 :‬‬
‫‪0e‬‬
‫‪dt‬‬
‫‪dN t ‬‬ ‫‪‬‬ ‫‪0  4   1020‬‬
‫‪A0  ‬‬ ‫‪‬‬ ‫‪ .2.2‬أ‪ .‬قيمة النشاط ‪ A 0‬للعينة‪ 3,7  1018 Bq :‬‬
‫‪dt t  0‬‬ ‫‪108  0 ‬‬
‫ب‪ .‬استنتاج قيمة ثابت النشاط اإلشعاعي ‪ ‬للسترونتيوم ‪ 94‬بطريقتين‪:‬‬
‫‪A0 3,7  1020‬‬
‫‪. ‬‬ ‫‪‬‬ ‫لدينا‪ A0   N 0 :‬ومنه‪ 9,25  10 3 s 1 :‬‬
‫‪N0‬‬ ‫‪4  10‬‬ ‫‪20‬‬

‫من البيان ‪ N  f t ‬نقرأ قيمة ثابت الزمن‪   108s :‬ومنه‪.   1  1  9, 26  10 3 s 1 :‬‬
‫‪‬‬ ‫‪108‬‬
‫‪. m 0  N 0M‬‬ ‫‪‬‬
‫‪4  1020  94‬‬
‫‪ 62, 46  10 3 g‬‬ ‫جـ ‪ .‬حساب كتلة العينة المشعة ‪ : m 0‬لدينا‪:‬‬
‫‪NA‬‬ ‫‪6,02  10‬‬ ‫‪23‬‬

‫‪ .3.2‬مدة القياسات ‪ t m‬ال تتجاوز ‪: 9min‬ألن أقصى مدة لتفكك العينة هي‪. 5  5 108  540 s  9min :‬‬

‫‪.4.2‬عند مضاعفة كتلة العينة المشعة السابقة فإن‪:‬‬


‫‪m 0' N A‬‬
‫'‪A0‬‬ ‫‪‬‬ ‫‪‬‬
‫'‪N 0‬‬ ‫النشاط اإلشعاعي االبتدائي ‪ A 0‬للعينة تتضاعف قيمته ألن‪ 2 A0 :‬‬ ‫‪‬‬
‫‪M‬‬
‫المدة الزمنية للقياس ‪ : t m‬تبقى ثابتة (ال تتغير ) ألن‪ :‬مدة التفكك تتعلق بنوع النواة المشعة ‪.‬‬ ‫‪‬‬

‫‪ ‬نمط التفكك‪ :‬اليتغير‪.‬‬


‫‪ .1‬إعادة رسم الدارة ‪:‬‬
‫‪ .2‬حسب قانون جمع التوترات ‪𝑢𝑏 = 𝐸 :‬‬
‫في النظام الدائم( الوشيعة تسلك سلوك ناقل أومي ) يكون ‪𝑟𝐼0 = 𝐸 :‬‬
‫و منه ‪. 𝑟 = 𝐸 ⁄𝐼0 :‬‬
‫ت‪.‬ع ‪𝑟 = 5,95⁄0,410 = 14.5 𝛺:‬‬
‫‪ .3‬حساب الطاقة المخزنة في الوشيعة ‪:‬‬
‫‪1‬‬
‫‪𝐸𝐿 = 𝐿𝐼0 2 = 0.5 × 250 × 10−3 × (0.410)2‬‬
‫‪2‬‬
‫𝐽 ‪𝐸𝐿 = 2.1 × 10−2‬‬
‫‪ .4‬المعادلة التفاضلية لشدة التيار )𝑡(𝑖 ‪:‬‬
‫𝑖𝑑‬
‫حسب قانون جمع التوترات ‪ 𝑢𝑏 = 𝐸 :‬و منه 𝐸 = 𝑖𝑟 ‪𝐿 +‬‬
‫𝑡𝑑‬
‫𝑟 𝑖𝑑‬ ‫𝐸‬
‫=𝑖 ‪+‬‬
‫‪𝑑𝑡 𝐿1‬‬ ‫𝐿‬ ‫‪1‬‬
‫𝑖𝑑‬ ‫𝑡𝛽‪𝛼 −‬‬ ‫𝑡 ‪−‬‬
‫‪ ،‬بالتعويض في المعادلة التفاضلية‬ ‫𝑒 =‬ ‫‪ -5‬باشتقاق العبارة ‪ 𝑖(𝑡) = 𝛼 ⋅ (1 − 𝑒 ):‬نجد ‪:‬‬
‫𝛽‬
‫𝑡𝑑‬ ‫𝛽‬
‫𝐿‬ ‫𝐸‬
‫=𝜏=𝛽‬ ‫= ‪𝛼 = 𝐼0‬‬ ‫نجد الثوابت ‪:‬‬
‫𝑟‬ ‫𝑟‬
‫‪1.6‬العبارة اللحظية لشدة التيار لما ‪:‬‬
‫البيان عبارة عن دالة خطية من الشكل 𝑡𝑎 = )𝑡(𝑖 أي 𝑡 ‪. 𝑖(𝑡) = 0.32 .‬‬ ‫أ‪:0 ≤ 𝑡 ≤ 2,5𝑚𝑠 .‬‬
‫ب‪:2,5𝑚𝑠 ≤ 𝑡 ≤ 5𝑚𝑠 .‬‬
‫𝐴 ‪𝐼0 = 0.8‬‬
‫‪ 2.6‬استنتاج عبارة التوتر 𝒃𝒖 ‪.‬‬
‫في النظام االنتقالي 𝑠𝑚‪: 0 ≤ 𝑡 ≤ 2,5‬‬
‫𝑖𝑑‬
‫‪𝑢𝑏 = 𝐿0‬‬ ‫‪= 0.32 𝐿0‬‬
‫𝑡𝑑‬
‫في النظام الدائم 𝑠𝑚‪: 2,5𝑚𝑠 ≤ 𝑡 ≤ 5‬‬
‫‪𝑑𝐼0‬‬
‫‪𝑢𝑏 = 𝐿0‬‬ ‫‪=0‬‬
‫𝑡𝑑‬
‫‪ 3.6‬من العبارتين السابقتين البيان الموافق للتوتر بين طرفي الوشيعة هو البيان 𝐶 ‪.‬‬

‫‪ 4.6‬استنتاج قيمة ذاتية الوشيعة ‪: 𝐿0‬‬


‫𝑖𝑑‬
‫‪𝑢𝑏 = 𝐿0‬‬ ‫𝑉 ‪= 0.32 𝐿0 = 25 × 5 × 10−3‬‬
‫𝑡𝑑‬
‫و منه ‪𝐿0 = 0,4 𝐻:‬‬
‫𝑚‪𝑀𝑇 .‬‬
‫𝐺 = 𝑆‪𝐹𝑇/‬‬ ‫‪ .1‬العبارة الحرفية لشدة قوة الجذب العام ‪:‬‬
‫‪(𝑅𝑇 +ℎ)2‬‬
‫‪ .2‬إثبات العبارة ‪:‬‬
‫𝑃 = 𝑆‪𝐹𝑇/‬‬
‫و منه ‪:‬‬
‫𝑚 ‪𝑀𝑇 .‬‬ ‫𝑇𝑀‬
‫𝐺‬ ‫=‬ ‫‪𝑚.‬‬ ‫𝑔‬ ‫‪‬‬‫𝑔‬ ‫=‬ ‫𝐺‬
‫‪(𝑅𝑇 + ℎ)2‬‬ ‫‪(𝑅𝑇 + ℎ)2‬‬
‫‪1‬‬ ‫𝑇𝑀𝐺‬
‫√ = )‪(𝑅𝑇 + ℎ‬فتكون العالقة النهائية ‪ℎ = √𝐺𝑀𝑇 × √ − 𝑅𝑇 :‬‬ ‫و عليه ‪:‬‬
‫𝑔‬ ‫𝑔‬

‫حيث ‪ 𝐴 = √𝐺𝑀𝑇 :‬و 𝑇𝑅‪𝐵 = −‬‬


‫‪ .3‬إيجاد عبارة السرعة ‪:‬‬
‫الجملة ‪ :‬قمر اصطناعي ‪.‬‬
‫المرجع ‪ :‬مرجع جيو مركزي ( مرجع مركزي أرضي ) نعتبره غاليليا ‪.‬‬
‫بتطبيق القانون الثاني لنيوتن ‪𝐹𝑒𝑥𝑡 = 𝑚. 𝑎:‬‬
‫⃗⃗⃗⃗⃗⃗⃗ ∑‬
‫فيكون ‪𝑇/𝑆 = 𝑚. 𝑎 :‬‬
‫⃗⃗⃗⃗⃗⃗⃗⃗‬
‫𝐹‬
‫باإلسقاط على المحور الناظمي ⃗𝑛 نجد ‪:‬‬
‫𝑛𝑎 ‪𝐹𝑇/𝑆 = 𝑚.‬‬
‫𝑚 ‪𝑀𝑇 .‬‬ ‫‪𝑣2‬‬
‫𝐺‬ ‫𝑚=‬
‫‪(𝑅𝑇 + ℎ)2‬‬ ‫)‪(𝑅𝑇 + ℎ‬‬
‫و منه عبارة السرعة ‪:‬‬

‫𝑇𝑀𝐺‬
‫√=𝑣‬
‫)‪(𝑅𝑇 + ℎ‬‬
‫‪ .1.4‬العبارة البيانية للمنحنى ‪:‬‬
‫‪1‬‬
‫المنحنى دالة تآلفية عبارته ‪. ℎ = 𝑎√ + 𝑏:‬‬
‫𝑔‬

‫‪ .2.4‬استنتاج كتلة األرض ‪:‬‬


‫بمطابقة العبارة النظرية مع العبارة البيانية نجد ‪ 𝑎 = √𝐺𝑀𝑇 :‬و 𝑇𝑅‪𝑏 = −‬‬
‫حساب معامل التوجيه ‪:‬‬
‫‪(6.4 − 0) × 106‬‬
‫=𝑎‬ ‫‪= 2 × 107‬‬
‫‪0,64 − 0.32‬‬
‫‪𝑎2‬‬ ‫‪(2×107 )2‬‬
‫ومنه ‪𝑀𝑇 = 5.99 × 1024 𝑘𝑔 :‬‬ ‫= 𝑇𝑀‬ ‫=‬ ‫ومنه ‪:‬‬
‫𝐺‬ ‫‪6.67×10−11‬‬
‫إذن‪𝑅𝑇 = 6.4 × 106 𝑚 = 6400 𝑘𝑚 :‬‬ ‫بيانيا 𝑚 ‪𝑏 = −6.4 × 106‬‬
‫‪ .3.4‬قيمة تسارع الجاذبية ‪:𝑔0‬‬
‫من العبارة النظرية نجد أن ‪ 𝑔0‬توافق قيمة ‪ ℎ = 0‬و منه يكون ‪:‬‬
‫‪1‬‬
‫√ × 𝑇𝑀𝐺√ = 𝑇𝑅‬
‫‪𝑔0‬‬

‫‪2‬‬ ‫𝑇𝑀𝐺‬ ‫‪𝐺𝑀𝑇 6.67 × 10−11 × 5.99 × 1024‬‬


‫𝑇𝑅‬ ‫=‬ ‫= ‪→ 𝑔0‬‬ ‫‪2‬‬ ‫=‬ ‫‪6‬‬ ‫‪2‬‬
‫‪= 9.75 𝑚⁄𝑠 2‬‬
‫‪𝑔0‬‬ ‫𝑇𝑅‬ ‫) ‪(6.4 × 10‬‬
‫‪ .1.5‬إيجاد ارتفاع القمر ‪:‬‬
‫‪1‬‬ ‫‪1‬‬
‫=‬ ‫نقوم بحساب القيمة ‪= 2.13‬‬
‫𝑔√‬ ‫‪√0.22‬‬
‫بإسقاط القيمة على البيان نجد ‪. ℎ = 36 × 10 𝑚:‬‬
‫‪6‬‬

‫استنتاج سرعته ‪ :‬من عبارة السرعة نجد ‪:‬‬

‫‪6.67 × 10−11 × 5.99 × 1024‬‬


‫√=𝑣‬ ‫𝑠‪= 3069 𝑚⁄‬‬
‫) ‪((6.4 + 36) × 106‬‬
‫‪ .2.5‬حساب دور القمر الصناعي 𝑇 ‪:‬‬
‫‪6‬‬
‫) ‪2𝜋(𝑅𝑇 + ℎ) 2 × 3.14 × ((6.4 + 36) × 10‬‬
‫=𝑇‬ ‫=‬ ‫‪= 86846 𝑠 ≈ 24 ℎ‬‬
‫𝑣‬ ‫‪3069‬‬
‫القمر جيو مستقر ألن ‪𝑇 ≈ 24 ℎ‬‬

‫‪: Chl‬‬ ‫‪ .1‬اثبات العبارة ‪c 0 V M‬‬


‫انحالل كلي لغاز الكلور معناه ‪ 𝑛𝐶𝑙2 − 𝑥𝑚𝑎𝑥 = 0:‬إذن ‪ 𝑛𝐶𝑙2 = 𝑥𝑚𝑎𝑥 :‬و منه 𝑚𝑉 × 𝑥𝑎𝑚𝑥 = ‪𝑉𝐶𝑙2‬‬
‫𝑥𝑎𝑚𝑥‬
‫= ‪ 𝐶𝑙𝑂− = 𝑐0‬اذن 𝑉 ‪𝑥𝑚𝑎𝑥 = 𝑐0 .‬‬ ‫من جهة أخرى لدينا ‪:‬‬
‫𝑉‬
‫و منه ‪ ( 𝑉𝐶𝑙2 = 𝑥𝑚𝑎𝑥 × 𝑉𝑚 = 𝑐0 × 𝑉 × 𝑉𝑚 :‬باعتبار الحجم الكلي 𝐿‪ ) 𝑉 = 1‬يكون ‪:‬‬
‫𝑚𝑉 × ‪𝑉𝐶𝑙2 = 𝑐ℎ𝑙 ° = 𝑐0‬‬
‫‪ .2‬حساب ‪: 𝑐0‬‬
‫‪𝑉𝐶𝑙2‬‬ ‫‪12‬‬
‫= ‪𝑐0‬‬ ‫=‬ ‫𝐿‪= 0.53 𝑚𝑜𝑙/‬‬
‫𝑚𝑉‬ ‫‪22.4‬‬
‫‪ 1.1‬جدول التقدم ‪:‬‬
‫م‪ .‬التفاعل‬ ‫)‪𝐶𝑙𝑂− (𝑎𝑞) + 2𝐼 − (𝑎𝑞) + 2𝐻+ (𝑎𝑞) = 𝐼2 (𝑎𝑞) + 𝐶𝑙 − (𝑎𝑞) + 𝐻2 𝑂(ℓ‬‬
‫الحالة‬ ‫التقدم‬ ‫كميات المادة )𝑙𝑜𝑚(‬
‫ح‪.‬ابتدائية‬ ‫‪0‬‬ ‫‪𝑛0‬‬ ‫بوفرة‬ ‫بوفــرة‬ ‫‪0‬‬ ‫‪0‬‬ ‫بوفرة‬
‫ح‪.‬انتقالية‬ ‫𝑥‬
‫𝑥 ‪𝑛0 −‬‬ ‫بوفرة‬ ‫بوفـــرة‬ ‫𝑥‬ ‫𝑥‬ ‫بوفرة‬

‫ح‪.‬نهائية‬ ‫𝑓𝑥 ‪𝑛0 −‬‬ ‫بوفرة‬ ‫بوفـــرة‬


‫𝑓𝑥‬ ‫𝑓𝑥‬ ‫𝑓𝑥‬ ‫بوفرة‬
‫‪ .2.1‬تحديد المتفاعل المحد ‪:‬‬
‫بما أن شوارد اليود ‪ 𝐼 −‬بوفرة و التفاعل تام فإن ‪ 𝐶𝑙𝑂−‬متفاعل محد ‪.‬‬
‫من جدول التقدم ‪𝑛(𝐼2 )𝑓 = 𝑥𝑓 :‬‬
‫إذن ‪𝑐0 × 𝑉 = 𝑥𝑓 :‬‬ ‫‪ 𝐶𝑙𝑂−‬متفاعل محد معناه ‪ 𝑛𝐶𝑙𝑂− − 𝑥𝑓 = 0:‬و منه ‪𝑛𝐶𝑙𝑂− = 𝑥𝑓 :‬‬
‫و عليه يكون ‪𝑛(𝐼2 )𝑓 = 𝑥𝑓 = 𝑐0 × 𝑉 :‬‬
‫‪ .1.2‬معادلة تفاعل المعايرة ‪:‬‬
‫‪𝐼2 + 2𝑒 − = 2𝐼 −‬‬
‫‪𝑆2 𝑂32− = 𝑆4 𝑂82− + 2𝑒 −‬‬
‫‪𝐼2 + 𝑆2 𝑂32− = 2𝐼 − + 𝑆4 𝑂82−‬‬
‫‪ .2.2‬إيجاد العالقة‪:‬‬
‫‪𝐶0‬‬ ‫𝐸𝑉 ‪𝐶2‬‬
‫= ‪. 𝑐1‬‬ ‫= ‪ 𝑐1 𝑉1‬و نعلم – من نص التمرين و عالقة التمديد – أن ‪:‬‬ ‫من عالقة التــكافــؤ‪:‬‬
‫‪10‬‬ ‫‪2‬‬
‫‪𝐶0 𝑉1‬‬ ‫𝐸𝑉 ‪𝐶2‬‬
‫إذن‪𝑐0 𝑉1 = 5𝑐2 𝑉𝐸 :‬‬ ‫=‬ ‫و منه تصبح العالقة‪:‬‬
‫‪10‬‬ ‫‪2‬‬
‫حساب ‪: 𝑐0‬‬
‫‪5𝑐2 𝑉𝐸 5 × 0.1 × 21‬‬
‫= ‪𝑐0‬‬ ‫=‬ ‫𝐿‪= 0.525 𝑚𝑜𝑙 ⁄‬‬
‫‪𝑉1‬‬ ‫‪20‬‬
‫حساب الدرجة الكلورومترية ‪:‬‬
‫‪𝑐ℎ𝑙 ° = 𝐶0 × 𝑉𝑚 = 0.525 × 22.4 = 11.8° ≈ 12°‬‬
‫إذن يوافق ما كتب على القارورة‪.‬‬
‫‪.‬‬
‫‪ .1‬رسم التركيب التجريبي المستعمل في هذه المتابعة مع إرفاقه بالبيانات المناسبة‪:‬‬

‫اسم العنصر‬ ‫الرقم‬


‫‪5‬‬
‫إرلينة ماير‬ ‫‪1‬‬

‫‪1‬‬ ‫‪4‬‬ ‫المزيج التفاعلي (ماء جافيل ‪) Co 2 +‬‬ ‫‪2‬‬

‫‪3‬‬ ‫حوض مائي‬ ‫‪3‬‬


‫‪2‬‬
‫مخبار مدرج‬ ‫‪4‬‬

‫‪O 2 ‬‬ ‫غاز‬ ‫‪5‬‬

‫‪ .2‬دور شوارد الكوبالت ‪ :‬بما أنها لم تظهر معادلة التفاعل فدورها هو وسيط يسرع التفاعل ‪.‬‬
‫‪ .3‬جدول تقدم التفاعل ‪:‬‬
‫معادلة التفاعل‬ ‫)𝑞𝑎( ‪2𝐶𝑙𝑂− (𝑎𝑞) = 𝑂2 (𝑔) + 2𝐶𝑙−‬‬
‫الحالة‬ ‫التقدم‬ ‫كميات المادة )𝑙𝑜𝑚(‬
‫ح إبتدائية‬ ‫‪0‬‬ ‫‪𝑛0‬‬ ‫‪0‬‬ ‫‪0‬‬
‫ح إنتقالية‬ ‫𝑥‬ ‫𝑥‪𝑛0 − 2‬‬ ‫𝑥‬ ‫𝑥‪2‬‬
‫ح نهائية‬ ‫𝑓𝑥‬ ‫𝑓𝑥‪𝑛0 − 2‬‬ ‫𝑓𝑥‬ ‫𝑓𝑥‪2‬‬
‫‪ 𝐶𝑙𝑂−‬متفاعل محد ( تفاعل تام ) و منه ‪:‬‬
‫𝑥𝑎𝑚𝑥‪𝑛𝐶𝑙𝑂− − 2𝑥𝑚𝑎𝑥 = 0 → 𝑛𝐶𝑙𝑂− = 2‬‬
‫‪𝑐 × 𝑉 5 × 10−2 × 100 × 10−3‬‬
‫= 𝑥𝑎𝑚𝑥‬ ‫=‬
‫‪2‬‬ ‫‪2‬‬
‫𝐿‪𝑥𝑚𝑎𝑥 = 2.5 × 10−3 𝑚𝑜𝑙 ⁄‬‬
‫‪ .4‬حجم غاز األكسجين المنطلق في نهاية التفاعل ‪:‬‬
‫‪𝑉𝑂2‬‬
‫→ 𝑥𝑎𝑚𝑥 = ‪𝑛𝑂2‬‬ ‫𝑥𝑎𝑚𝑥 =‬
‫𝑚𝑉‬
‫‪𝑉𝑂2‬‬ ‫𝑙𝑚 ‪= 𝑥𝑚𝑎𝑥 × 𝑉𝑚 = 2.5 × 10−3 × 24 = 0.06 𝐿 = 60‬‬
‫نالحظ أنه عند 𝑛𝑖𝑚 ‪ 𝑡 = 40‬حجم ‪ 𝑉𝑂2‬المنطلق بيانيا ‪𝑉𝑂2 (40 𝑚𝑖𝑛) = 47 𝑚𝑙:‬‬
‫إذن لم ينتهي التفاعل عند هذه اللحظة ‪.‬‬
‫‪ .5‬إيجاد زمن نصف التفاعل ‪: 𝑡1/2‬‬
‫‪𝑉𝑂2‬‬ ‫‪60‬‬
‫𝑓‬
‫= ) ‪𝑉𝑂2 (𝑡1‬‬ ‫=‬ ‫𝐿𝑚 ‪= 30‬‬
‫‪2‬‬ ‫‪2‬‬ ‫‪2‬‬
‫باإلسقاط على البيان نجد ‪𝑡1/2 = 16 𝑚𝑖𝑛:‬‬
‫𝑥𝑑 ‪1‬‬
‫= 𝑙𝑜𝑣𝑣‬ ‫‪ .6‬إثبات عبارة السرعة الحجمية ‪:‬‬
‫𝑡𝑑 𝑇𝑉‬
‫‪𝑉𝑂2‬‬
‫= )𝑡(𝑥 = )𝑡( ‪𝑛𝑂2‬‬ ‫لدينا من جدول التقدم‬
‫𝑚𝑉‬
‫𝑂𝑉‬
‫‪2‬‬
‫‪1‬‬ ‫‪𝑑𝑉𝑂2‬‬ ‫) 𝑚𝑉 (𝑑 ‪1‬‬
‫= 𝑙𝑜𝑣𝑣‬ ‫= 𝑙𝑜𝑣𝑣 و منه ‪:‬‬ ‫‪:‬‬ ‫نعوض في عبارة السرعة الحجمية فنجد‬
‫𝑡𝑑 𝑚𝑉‪𝑉𝑇 .‬‬ ‫𝑡𝑑 𝑇𝑉‬
‫‪𝑑𝑉𝑂2‬‬
‫× ‪𝑣𝑣𝑜𝑙 = 0.42‬‬ ‫و عليه ‪:‬‬
‫𝑡𝑑‬
‫حساب قيمتها عند اللحظة ‪: 𝑡 = 0‬‬
‫‪(25 − 0) × 10−3‬‬
‫𝑙𝑜𝑣𝑣‬ ‫× ‪= 0.42‬‬
‫)‪(10 − 0‬‬
‫‪−3‬‬
‫𝑙𝑜𝑣𝑣‬ ‫𝑛𝑖𝑚 ‪= 1.05 × 10 𝑚𝑜𝑙/𝐿.‬‬
‫سرعة اختفاء شوارد ‪: 𝐶𝑙𝑂−‬‬
‫‪𝑑𝑛𝐶𝑙𝑂−‬‬ ‫)𝑥‪𝑑(𝑛0 −2‬‬
‫‪𝑣𝐶𝑙𝑂− = −‬‬ ‫=‬
‫𝑡𝑑‬ ‫𝑡𝑑‬
‫𝑥𝑑‬
‫× ‪𝑣𝐶𝑙𝑂− = 2‬‬
‫𝑡𝑑‬
‫‪1‬‬ ‫𝑥𝑑 ‪1‬‬
‫‪𝑣 −‬‬ ‫× ×‪=2‬‬
‫𝑂𝑙𝐶 𝑉‬ ‫𝑡𝑑 𝑉‬
‫و منه ‪𝑣𝐶𝑙𝑂− = 2 × 𝑉 × 𝑣𝑣𝑜𝑙 = 2 × 0.1 × 1.05 × 10−3 :‬‬
‫𝐿‪𝑣𝐶𝑙𝑂− = 2.1 × 10−4 𝑚𝑜𝑙 ⁄‬‬
‫‪ .7‬كيف تتغير المقادير‪:‬‬
‫أ‪ .‬تتغير السرعة بتغير التراكيز االبتدائية للمتفاعالت ( يعتبر هذا العامل وسيط ) ‪.‬‬
‫ب‪ .‬زيادة سرعة التفاعل يؤدي الى تناقص قيمة زمن نصف التفاعل ‪. 𝑡1/2‬‬
‫ج‪ .‬ال تتغير كمية النواتج ( الحالة النهائية ) بزيادة سرعة التفاعل عن طريق زيادة التراكيز االبتدائية ‪.‬‬
‫اﻟﺠﻤﻬﻮرﻳﺔ اﻟﺠﺰاﺋﺮﻳﺔ اﻟﺪﻳﻤﻘﺮاﻃﻴﺔ اﻟﺸﻌﺒﻴﺔ‬
‫ﻣﺪﻳﺮﻳﺔ اﻟﺘﺮﺑﻴﺔ ﻟﻮﻻﻳﺔ ﻗﺴﻨﻄﻴﻨﺔ‬ ‫وزارة اﻟﺘﺮﺑﻴﺔ اﻟﻮﻃﻨﻴﺔ‬
‫ﻣﻘﺎﻃﻌﺔ ﻗﺴﻨﻄﻴﻨﺔ‪2‬‬ ‫اﻣﺘﺤﺎن اﻟﺒﻜﺎﻟﻮرﻳﺎ اﻟﺘﺠﺮﻳﺒﻲ‬
‫اﻟﺸﻌﺒﺔ‪ :‬رﻳﺎﺿﻴﺎت‪ ،‬ﺗﻘﻨﻲ رﻳﺎﺿﻲ‬ ‫دورة ﻣﺎي ‪2023‬‬
‫اﻟﻤﺪة ‪ 04 :‬ﺳﺎ و ‪ 30‬د‬ ‫اﺧﺘﺒﺎر ﻓﻲ ﻣﺎدة‪ :‬اﻟﻌﻠﻮم اﻟﻔﻴﺰﻳﺎﺋﻴﺔ‬
‫ﻋﻠﻰ اﻝﻤﺘرﺸﺢ أن ﻴﺨﺘﺎر أﺤد اﻝﻤوﻀوﻋﻴن اﻵﺘﻴﻴن‪:‬‬
‫اﻝﻤوﻀوع اﻷول‬
‫ﻴﺤﺘوي ﻋﻠﻰ )‪ (06‬ﺼﻔﺤﺎت )ﻤن اﻝﺼﻔﺤﺔ ‪ 1‬ﻤن ‪ 12‬إﻝﻰ اﻝﺼﻔﺤﺔ ‪ 6‬ﻤن ‪(12‬‬
‫اﻝﺠزء اﻷول‪ 14) :‬ﻨﻘطﺔ(‬
‫اﻝﺘﻤرﻴن اﻷول‪ 4) :‬ﻨﻘﺎط(‬
‫اﻜﺘﺸف ﻜوﻜب ﺒﻠوﺘو ) ‪ ( Pluton‬ﺴﻨﺔ ‪1930‬م و اﻋﺘُﺒر اﻝﺘﺎﺴﻊ ﻓﻲ اﻝﻤﺠﻤوﻋﺔ اﻝﺸﻤﺴﻴﺔ‪ ،‬و ﻓﻲ ﺴﻨﺔ ‪ 2005‬م اﻜﺘﺸف‬
‫ﺠﺴم ﺠدﻴد ﻤﻨﺠذب ﺤول اﻝﺸﻤس ُﺴﻤﻲ ) ‪ ( Eris‬ﻋﻠﻰ اﺴم إﻝﻬﺔ اﻝﺨﻼف ﻋﻨد اﻹﻏرﻴق‪ .‬إﻀﺎﻓﺔ ) ‪ ( Eris‬إﻝﻰ‬
‫ﻜواﻜب أﺨرى ﻤﺸﺎﺒﻬﺔ ﻜﺎن ﺒداﻴﺔ ﺨﻼف و ﺠدل ﺤﺎد ﺒﻴن اﻝﻔﻠﻜﻴﻴن ﺤول ﺘﻌرﻴف اﻝﻜوﻜب‪ ،‬و ﺨﻼل ﺘﺠﻤﻊ اﻹﺘﺤﺎد اﻝﻔﻠﻜﻲ‬
‫اﻝﻌﺎﻝﻤﻲ ) ‪ (UAI‬ﻓﻲ ﺒراغ ﺘﻘرر إﻨزال ) ‪ ( Pluton‬ﻤن رﺘﺒﺔ ﻜوﻜب إﻝﻰ ﺼف ﻜوﻴﻜب )‪ ( planète naine‬رﻓﻘﺔ‬
‫) ‪ ( Eris‬و ) ‪. (Cérès‬‬
‫ﻴﻬدف اﻝﺘﻤرﻴن إﻝﻰ ﺘﺤدﻴد ﺴﺒب إﻨزال )‪ (Pluton‬ﻤن رﺘﺒﺔ ﻜوﻜب إﻝﻰ ﺼف ﻜوﻴﻜب‪.‬‬
‫‪ .1‬ﻴدور إﻴرﻴس ﻓﻲ ﻤدار اﻫﻠﻴﻠﻴﺠﻲ ﺤول اﻝﺸﻤس ﺒدور ‪ TE‬ﻗدرﻩ ‪ 557‬ﺴﻨﺔ أرﻀﻴﺔ‪.‬‬
‫اﻝﻤﻌطﻴﺎت‪:‬‬
‫ اﻝدور اﻝﻤداري ﻝﻸرض‪. TT = 1,00 ans :‬‬
‫ اﻝدور اﻝﻤداري ﻝﺒﻠوﺘو‪. TP = 248 ans :‬‬
‫‪ .1.1‬اﻜﺘب ﻨص اﻝﻘﺎﻨون اﻝﺜﺎﻝث ﻝﻜﺒﻠر‪ ،‬اﻝﻤﺘﻌﻠق ﺒﺎﻝدور اﻝﻤداري ﻝﻜوﻜب ﺤول اﻝﺸﻤس‪ ،‬ﻓﻲ ﺤﺎﻝﺔ ﻤدار إﻫﻠﻴﻠﻴﺠﻲ‪.‬‬
‫‪ .2.1‬ﻫل ﻴﻘﻊ ﻤدار إﻴرﻴس أﺒﻌد أو أدﻨﻰ ﻤن ﻤدار ﺒﻠوﺘو؟ ﺒرر إﺠﺎﺒﺘك دون ﺤﺴﺎب‪.‬‬
‫‪ .2‬ﻓﻴﻤﺎ ﺒﻌد‪ ،‬اﻜﺘﺸف اﻝﻔﻠﻜﻴون أن إﻴرﻴس ﻴﻤﻠك ﻗﻤ ار طﺒﻴﻌﻴﺎ دﻴﺴﻨوﻤﻴﺎ‬
‫‪) Dysnomia‬اﺒﻨﺔ إﻴرﻴس(‪ .‬ﺜﻤﺎﻨﻴﺔ أﻴﺎم ﻤراﻗﺒﺔ ﻤن اﻷرض ﺴﻤﺤت‬
‫ﺒﺈﻨﺸﺎء ﻤدار دﻴﺴﻨوﻤﻴﺎ و اﻝﺤﺼول ﻋﻠﻰ ﺼورة اﻝﺸﻜل‪.1‬‬
‫ﻤﻌطﻴﺎت‪:‬‬
‫ ﻜﺘﻠﺔ ﺒﻠوﺘو‪. M P = 1,31×1022 kg :‬‬
‫ ﻨﺼف ﻗطر اﻝﻤدار اﻝداﺌري ﻝدﻴﺴﻨوﻤﻴﺎ‪. rD = 3,60 ×107 m :‬‬
‫ اﻝدور اﻝﻤداري ﻝدﻴﺴﻨوﻤﻴﺎ‪TD = 15,0 jours = 1,30 ×106 s :‬‬

‫ﺻﻔﺤﺔ ‪ 1‬ﻣﻦ ‪12‬‬


‫ﺑﻜﺎﻟﻮرﻳﺎ ﺗﺠﺮﻳﺒﻲ ‪2023‬‬ ‫ﺷﻌﺒﺔ‪ :‬رﻳﺎﺿﻴﺎت‪ ،‬ﺗﻘﻨﻲ رﻳﺎﺿﻲ‪.‬‬ ‫ﺗﺎﺑﻊ اﻟﻤﻮﺿﻮع ﻟﻤـﺎدة‪ :‬اﻟﻌﻠﻮم اﻟﻔﻴﺰﻳﺎﺋﻴﺔ‬
‫ ﺜﺎﺒت اﻝﺠذب اﻝﻌﺎم‪. G = 6,67 ×10−11 SI :‬‬
‫ﻨﻔرض أن ﺤرﻜﺔ دﻴﺴﻨوﻤﻴﺎ ﺤول إﻴرﻴس داﺌرﻴﺔ ﻤﻨﺘظﻤﺔ و ﻴؤﺨذ‪. π 2 = 10 :‬‬
‫‪ .1.2‬ﺤدد اﻝﻤرﺠﻊ اﻝذي ﻴﺴﻤﺢ ﺒدراﺴﺔ ﺤرﻜﺔ دﻴﺴﻨوﻤﻴﺎ ﺤول إﻴرﻴس‪ ،‬ﺴﻨﻌﺘﺒر ﻓﻴﻤﺎ ﻴﻠﻲ ﻫذا اﻝﻤرﺠﻊ ﻏﺎﻝﻴﻠﻴﺎ‪.‬‬
‫‬
‫الشكل ‪1‬‬
‫‪ .2.2‬اﻜﺘب ﻋﺒﺎرة ﺸﻌﺎع اﻝﺘﺴﺎرع ‪ a‬ﻝﻤرﻜز ﻋطﺎﻝﺔ دﻴﺴﻨوﻤﻴﺎ ﺒدﻻﻝﺔ‬
‫‬
‫‬ ‫‪ M E ، G‬و ‪ rD‬اﻝﻤﻌطﻴﺎت و ﺸﻌﺎع اﻝوﺤدة ‪ U ED‬اﻝﻤﻤﺜل ﻓﻲ اﻝﺸﻜل‪1.‬‬
‫‪u‬‬
‫‪ .3.2‬ﺤدد ﺤﺎﻤل و اﺘﺠﺎﻩ ﺸﻌﺎع اﻝﺘﺴﺎرع‪.‬‬
‫‪rD3‬‬
‫‪. TD = 2 π‬‬ ‫‪ .4.2‬ﺒﻴن أن ﻋﺒﺎرة اﻝدور اﻝﻤداري ﻝدﻴﺴﻨوﻤﻴﺎ ﻫﻲ‪:‬‬
‫‪GM E‬‬
‫ﻫل ﻗﺎﻨون ﻜﺒﻠر ﻤﺤﻘق؟ﻋﻠل‪.‬‬
‫‪ .5.2‬اﺴﺘﻨﺘﺞ ﻤن ﻋﺒﺎرة ‪ TD‬ﻋﺒﺎرة ‪ M E‬ﻜﺘﻠﺔ إﻴرﻴس‪ ،‬ﺜم اﺤﺴب ﻗﻴﻤﺘﻬﺎ‪.‬‬
‫‪M‬‬
‫‪ .6.2‬اﺤﺴب اﻝﻨﺴﺒﺔ ﺒﻴن ﻜﺘﻠﺘﻲ إﻴرﻴس و ﺒﻠوﺘو ‪ . E‬اﺸرح ﻝﻤﺎذا أدى اﻜﺘﺸﺎف إﻴرﻴس إﻝﻰ إﻋﺎدة اﻝﻨظر ﻓﻲ ﺘﺼﻨﻴف‬
‫‪MP‬‬
‫ﺒﻠوﺘو‪.‬‬
‫اﻝﺘﻤرﻴن اﻝﺜﺎﻨﻲ‪ 06) :‬ﻨﻘﺎط(‬
‫وﺠد أﺴﺘﺎذ اﻝﻌﻠوم اﻝﻔﻴزﻴﺎﺌﻴﺔ ﻤﻜﺜﻔﺔ ﺘﺤﻤل اﻝﻤﻌﻠوﻤﺔ اﻝﺘﺎﻝﻴﺔ‪ ، C = 1000 µF :‬وﻝﻠﺘﺄﻜد ﻤن ﺴﻌﺔ اﻝﻤﻜﺜﻔﺔ اﻝﺴﺎﺒﻘﺔ ﻗدم‬
‫ﻝﻠﺘﻼﻤﻴذ اﻝﻌﻨﺎﺼر و اﻝوﺴﺎﺌل اﻝﻜﻬرﺒﺎﺌﻴﺔ اﻝﺘﺎﻝﻴﺔ‪:‬‬
‫‪ -‬ﻤوﻝد ﺘوﺘر ﻗوﺘﻪ اﻝﻤﺤرﻜﺔ اﻝﻜﻬرﺒﺎﺌﻴﺔ ‪. E‬‬
‫‪ -‬ﻨﺎﻗل أوﻤﻲ ﻤﻘﺎوﻤﺘﻪ ‪. R = 20 k Ω‬‬
‫‪ -‬أﺴﻼك ﺘوﺼﻴل‪ ،‬ﻗﺎطﻌﺔ ﻜﻬرﺒﺎﺌﻴﺔ ‪. K‬‬
‫‪ -‬ﺠﻬﺎز اﻝﻔوﻝط ﻤﺘر اﻝرﻗﻤﻲ‪.‬‬
‫اﻝﺠزء اﻷول‪:‬‬
‫ﺒﻌد اﻝﺘﺄﻜد ﻤن أن اﻝﻤﻜﺜﻔﺔ ﻏﻴر ﻤﺸﺤوﻨﺔ‪ ،‬ﻗﺎم اﻝﺘﻼﻤﻴذ ﺒرﺒطﻬﺎ ﻤﻊ اﻝﻌﻨﺎﺼر اﻝﻜﻬرﺒﺎﺌﻴﺔ اﻝﺴﺎﺒﻘﺔ وﺤﻘﻘوا ﺒذﻝك دارة ﻜﻬرﺒﺎﺌﻴﺔ‪.‬‬
‫ﻓﻲ اﻝﻠﺤظﺔ ‪ ، t = 0‬ﻏﻠق أﺤد اﻝﺘﻼﻤﻴذ اﻝﻘﺎطﻌﺔ و ﺒﻘراءة ﺠﻴدة ﻋﻠﻰ ﺠﻬﺎز اﻝﻔوﻝط ﻤﺘر اﻝرﻗﻤﻲ ﺘم ﺘﺴﺠﻴل ﻗﻴم اﻝﺘوﺘر‬
‫اﻝﻜﻬرﺒﺎﺌﻲ ﺒﻴن طرﻓﻲ اﻝﻤﻜﺜﻔﺔ ) ‪ uC (t‬ﻓﻲ ﻝﺤظﺎت زﻤﻨﻴﺔ ﻤﻌﻴﻨﺔ‪ ،‬و دون اﻝﻨﺘﺎﺌﺞ ﻓﻲ اﻝﺠدول اﻝﺘﺎﻝﻲ‪:‬‬

‫)‪t( s‬‬ ‫‪0‬‬ ‫‪10‬‬ ‫‪20‬‬ ‫‪40‬‬ ‫‪60‬‬ ‫‪80‬‬ ‫‪100‬‬ ‫‪110‬‬ ‫‪120‬‬
‫) ‪uC (V‬‬ ‫‪0,00‬‬ ‫‪4,72‬‬ ‫‪7,56‬‬ ‫‪10,37‬‬ ‫‪11,40‬‬ ‫‪11,78‬‬ ‫‪11,92‬‬ ‫‪12‬‬ ‫‪12‬‬
‫‪ .1‬ارﺴم ﻤﺨطط اﻝدارة اﻝﻜﻬرﺒﺎﺌﻴﺔ اﻝﺘﻲ ﻗﺎم اﻝﺘﻼﻤﻴذ ﺒﺘﺤﻘﻴﻘﻬﺎ ﻤﻊ ﺘﻤﺜﻴل ﺠﻬﺔ اﻝﺘوﺘرات اﻝﻜﻬرﺒﺎﺌﻴﺔ ﺒﻴن طرﻓﻲ اﻝﻤوﻝد‬
‫و اﻝﻤﺴﺘﻘﺒﻼت وﺘﺤدﻴد ﺠﻬﺔ اﻝﺘﻴﺎر اﻝﻜﻬرﺒﺎﺌﻲ ‪. i‬‬
‫‪ .1.2‬ﻤﺎ ﻫﻲ اﻝظﺎﻫرة اﻝﻤدروﺴﺔ؟‪ .‬ﻓﺴرﻫﺎ ﻤﺠﻬرﻴﺎ‪.‬‬
‫‪ .2.2‬أوﺠد اﻝﻤﻌﺎدﻝﺔ اﻝﺘﻔﺎﻀﻠﻴﺔ اﻝﺘﻲ ﻴﺤﻘﻘﻬﺎ اﻝﺘوﺘر اﻝﻜﻬرﺒﺎﺌﻲ ) ‪ uC (t‬ﺒﻴن طرﻓﻲ اﻝﻤﻜﺜﻔﺔ ‪.‬‬

‫ﺻﻔﺤﺔ ‪ 2‬ﻣﻦ ‪12‬‬


‫ﺑﻜﺎﻟﻮرﻳﺎ ﺗﺠﺮﻳﺒﻲ ‪2023‬‬ ‫ﺷﻌﺒﺔ‪ :‬رﻳﺎﺿﻴﺎت‪ ،‬ﺗﻘﻨﻲ رﻳﺎﺿﻲ‪.‬‬ ‫ﺗﺎﺑﻊ اﻟﻤﻮﺿﻮع ﻟﻤـﺎدة‪ :‬اﻟﻌﻠﻮم اﻟﻔﻴﺰﻳﺎﺋﻴﺔ‬
‫‪ .3.2‬ﺤدد اﻝﻌﺒﺎرة اﻝﻠﺤظﻴﺔ ﻝﻠﺤل اﻝﺘﺤﻠﻴﻠﻲ ﻝﻠﻤﻌﺎدﻝﺔ اﻝﺘﻔﺎﻀﻠﻴﺔ ﻤن ﺒﻴن اﻝﻌﺒﺎرات اﻝﺘﺎﻝﻴﺔ‪:‬‬
‫‪t‬‬ ‫‪t‬‬ ‫‪t‬‬
‫‪−‬‬ ‫‪−‬‬ ‫‪−‬‬
‫‪uC = − E (1 + e‬‬ ‫)‪τ‬‬ ‫‪ uC = E (1 − e‬؛‬ ‫)‪τ‬‬ ‫‪ uC = − E ( e‬؛‬ ‫‪τ‬‬ ‫)‪− 1‬‬
‫ﺤﻴث‪ τ :‬ﺜﺎﺒت اﻝزﻤن‪.‬‬
‫‪ .4.2‬اﻜﺘب اﻝﻌﺒﺎرة اﻝﻠﺤظﻴﺔ ﻝﻠﺘوﺘر اﻝﻜﻬرﺒﺎﺌﻲ ) ‪ uR (t‬ﺒﻴن طرﻓﻲ اﻝﻨﺎﻗل اﻷوﻤﻲ‪.‬‬
‫‪ .2‬ارﺴم اﻝﻤﻨﺤﻨﻰ اﻝﺒﻴﺎﻨﻲ ) ‪ uC = f (t‬ﺒﺎﺴﺘﻌﻤﺎل ﺴﻠم رﺴم ﻤﻨﺎﺴب‪.‬‬
‫‪ .3‬اوﺠد ﻗﻴﻤﺘب ﻜل ﻤن‪ E :‬و ‪ τ‬ﺒﺎﻻﻋﺘﻤﺎد ﻋﻠﻰ اﻝﺒﻴﺎن‪.‬‬
‫‪ .4‬اﺴﺘﻨﺘﺞ ﺴﻌﺔ اﻝﻤﻜﺜﻔﺔ ‪ ، C‬و ﻫل ﺘواﻓق اﻝﻘﻴﻤﺔ اﻝﻤﻌطﺎة؟‬
‫‪ .1.5‬أﻜﺘب اﻝﻌﺒﺎرة اﻝﻠﺤظﻴﺔ ﻝﻠطﺎﻗﺔ اﻝﻤﺨزﻨﺔ ) ‪ EC (t‬ﻓﻲ اﻝﻤﻜﺜﻔﺔ‪.‬‬
‫‪ .2.5‬اﺤﺴب ﻗﻴﻤﺘﻬﺎ ﻓﻲ اﻝﻠﺤظﺘﻴن‪. t2 = 80 s ، t1 = 10 s :‬‬
‫) ‪uC (t‬‬ ‫اﻝﺠزء اﻝﺜﺎﻨﻲ‪:‬‬
‫) ‪uR ( t‬‬ ‫اﻋﺘﻤﺎدا ﻋﻠﻰ اﻝﻨﺘﺎﺌﺞ اﻝﺴﺎﺒﻘﺔ ﻗﺎم ﺘﻠﻤﻴذان ﺒرﺴم اﻝﻤﻨﺤﻨﻰ‬
‫) ‪u (t‬‬
‫‪ C‬ﻜﻤﺎ ﻤوﻀﻊ ﻓﻲ اﻝﺸﻜل‪.2‬‬ ‫اﻝﺒﻴﺎﻨﻲ‬
‫) ‪uR (t‬‬
‫‪1‬‬
‫) ‪u (t‬‬
‫‪ C‬ﺒدﻻﻝﺔ ‪ τ‬و ‪. t‬‬ ‫‪ِ .1‬ﺠد ﻋﺒﺎرة اﻝﻨﺴﺒﺔ‬
‫) ‪uR (t‬‬
‫‪ .2‬ﺤدد أي اﻝﻤﻨﺤﻨﻴﻴن )‪ (1‬أو )‪ (2‬ﺼﺤﻴﺢ ﻤﻊ اﻝﺘﻌﻠﻴل‪.‬‬
‫‪2‬‬
‫‪1‬‬ ‫‪ .1.3‬أوﺠد ﺒﻴﺎﻨﻴﺎ ﻗﻴﻤﺔ ﺜﺎﺒت اﻝزﻤن ‪ τ‬ﻤﻊ اﻝﺘﻌﻠﻴل‪.‬‬
‫‪ .2.3‬ﺘﺄﻜد ﻤن ﺴﻌﺔ اﻝﻤﻜﺜﻔﺔ ‪ C‬اﻝﺘﻲ ﺘﺤﺼل ﻋﻠﻴﻬﺎ ﺴﺎﺒﻘﺎ‪.‬‬
‫‪5‬‬ ‫الشكل ‪2‬‬ ‫)‪t ( s‬‬ ‫اﻝﺘﻤرﻴن اﻝﺜﺎﻝث‪ 06) :‬ﻨﻘﺎط(‬
‫ﺘﻌﺘﺒر اﻝزﻻزل ﻜوارث طﺒﻴﻌﻴﺔ ﻻ ﻴﻤﻜن اﻝﺘﻨﺒؤ ﺒﺘﺎرﻴﺦ ﺤدوﺜﻬﺎ ‪ ،‬ﻝﻜن ﻴﻤﻜن ﺘﺤدﻴد ﺘوارﻴﺦ اﻝﻬزات اﻝﺘﻲ وﻗﻌت ﺒﺎﻝﻤﻨطﻘﺔ‬
‫ﺨﻼل ﻗرون ﺴﺎﺒﻘﺔ و ذﻝك ﺒﺎﺴﺘﺨدام اﻝﻌﻨﺎﺼر اﻝﻤﺸﻌﺔ ‪ ،‬ﺒﺎﺴﺘطﺎﻋﺔ ﻫذﻩ اﻝﻌﻨﺎﺼر اﻝﺘﻌرﻴف ﺒﻌﻤر اﻝﻜون ‪ ،‬ﻋﻤر اﻷرض ‪،‬‬
‫اﻵﻝﻴﺎت اﻝﺠﻴوﻝوﺠﻴﺔ و ﺤﺘﻰ ﺘﺎرﻴﺦ اﻝﺒﺸرﻴﺔ‬
‫ﻨذﻜر ﻋﻠﻰ ﺴﺒﻴل اﻝﻤﺜﺎل زﻝزال وﻻﻴﺔ ﻜﻬرﻤﺎن ﻤرﻋش ﻤؤﺨ ار ﺒﺎﻝﺠﻨوب اﻝﺘرﻜﻲ‬
‫ﺤﻴث أﻜد اﻝﻤؤرﺨون أن اﻝﻤﻨطﻘﺔ ﺘﻌرﻀت ﻝزﻻزل ﻜﺒرى ﻋﻠﻰ ﻤر اﻝﻘرون‪.‬‬
‫‪ . I‬اﻝﻨﺸﺎط اﻹﺸﻌﺎﻋﻲ ﻝﻠﻜرﺒون‬
‫‪. 14‬‬
‫‪6 C ،‬‬ ‫‪6 C:‬‬
‫‪12‬‬
‫‪ .1‬اﻋط ﺘرﻜﻴب اﻝﻨواﺘﻴن اﻝﺘﺎﻝﻴﺘﻴن‬
‫‪ .2‬ﻤن ﺒﻴن اﻝﻨواﺘﻴن اﻝﺴﺎﺒﻘﺘﻴن ﺤدد اﻝﻨواة اﻝﻤﺸﻌﺔ و اﻝﻨواة اﻝﻤﺴﺘﻘرة ﻤﻊ‬
‫اﻝﺘﻌﻠﻴل ‪ ،‬ﻤﺒﻴﻨﺎ ﻨﻤط ﺘﻔﻜك اﻝﻨواة اﻝﻤﺸﻌﺔ ‪.‬‬
‫‪.‬‬ ‫‪16‬‬
‫‪8 O‬‬ ‫‪، 14‬‬
‫‪ .3‬اﻜﺘب ﻤﻌﺎدﻝﺔ اﻝﺘﻔﻜك اﻝﻨووي اﻝﺤﺎدث‪ .‬ﻴﻌطﻰ ‪7 N ، 5 B :‬‬
‫‪10‬‬

‫ﺻﻔﺤﺔ ‪ 3‬ﻣﻦ ‪12‬‬


‫ﺑﻜﺎﻟﻮرﻳﺎ ﺗﺠﺮﻳﺒﻲ ‪2023‬‬ ‫ﺷﻌﺒﺔ‪ :‬رﻳﺎﺿﻴﺎت‪ ،‬ﺗﻘﻨﻲ رﻳﺎﺿﻲ‪.‬‬ ‫ﺗﺎﺑﻊ اﻟﻤﻮﺿﻮع ﻟﻤـﺎدة‪ :‬اﻟﻌﻠﻮم اﻟﻔﻴﺰﻳﺎﺋﻴﺔ‬
‫‪ .4‬ﻨرﻤز ﺒـ ) ‪ N (t‬ﻝﻌدد اﻷﻨوﻴﺔ اﻝﻤﺘﺒﻘﻴﺔ ﻓﻲ اﻝﻠﺤظﺔ ‪ t‬و ‪ N 0‬ﻝﻌدد اﻷﻨوﻴﺔ ﻓﻲ اﻝﻠﺤظﺔ ‪ t = 0‬ﻝﻌﻴﻨﺔ ﻤﺸﻌﺔ ﻤن أﻨوﻴﺔ‬
‫اﻝﻜرﺒون‪:‬‬
‫‪ .1.4‬ﺠد اﻝﻤﻌﺎدﻝﺔ اﻝﺘﻔﺎﻀﻠﻴﺔ ﺒدﻻﻝﺔ ﻋدد اﻷﻨوﻴﺔ اﻝﻤﺘﺒﻘﻴﺔ ) ‪. N (t‬‬
‫‪ .2.4‬ﺒﻴن أن اﻝﻌﺒﺎرة ‪ N (t ) = N 0 e −λt :‬ﻫﻲ ﺤل ﻝﻠﻤﻌﺎدﻝﺔ اﻝﺘﻔﺎﻀﻠﻴﺔ اﻝﺴﺎﺒﻘﺔ ‪.‬‬
‫‪ .3.4‬ﻝﺘﻜن ﻜﺘﻠﺔ ﻋﻴﻨﺔ ﻤﺸﻌﺔ ﻝﻠﻜرﺒون ‪ ، m = 1 mg‬ﻨﺸﺎطﻬﺎ ‪ A‬ﻴﻘدر ﺒـ ‪ 9,89 × 109‬ﺘﻔﻜك ﻓﻲ اﻝدﻗﻴﻘﺔ ‪ ،‬ﺒﻴن أن‬
‫ﻋدد أﻓوﻗﺎدرو ﻴﻌطﻰ ﺒﺎﻝﻘﻴﻤﺔ اﻝﺘﻘرﻴﺒﻴﺔ ‪. N A = 6,02 × 1023 mol −1‬‬
‫ﻴﻌطﻰ ‪6 C ) = 5730 ans :‬‬
‫‪. t1/2 (14‬‬
‫‪ .4‬اذﻜر اﺴم اﻝوﺴﻴﻠﺔ اﻝﻤﺴﺘﻌﻤﻠﺔ ﻓﻲ ﻗﻴﺎس اﻝﻨﺸﺎط اﻻﺸﻌﺎﻋﻲ ‪. A‬‬
‫‪ . II‬ﺘﺄرﻴﺦ زﻝزال ﺴﺎن اﻨدرﻴﺎس ﺒﻜﺎﻝﻴﻔورﻨﻴﺎ‬
‫ﻓﻲ ﻋﺎم ‪ 1989‬م ﺒﺎﻝﻘرب ﻤن ﻓﺠوة ﺴﺎن اﻨدرﻴﺎس ﺒﻜﺎﻝﻴﻔورﻨﻴﺎ ﺘم اﺴﺘﺨراج ﻋﻴﻨﺎت ﻤﺘﺴﺎوﻴﺔ اﻝﻜﺘﻠﺔ ﻝﻨﺒﺎﺘﺎت ﻏﻤرت أﺜﻨﺎء‬
‫زﻻزل ﻗدﻴﻤﺔ‪ ،‬ﺘم ﻗﻴﺎس ﻨﺸﺎط ﻜل ﻤن اﻝﻌﻴﻨﺎت‪ ،‬ﻨﺸﺎط ﻋﻴﻨﺔ ﻤن ﻨﻔس اﻝﻨﺒﺎت اﻝﺤﻲ وﻨﻔس اﻝﻜﺘﻠﺔ ﻫو ) ‪A0 = 0, 255 ( SI‬‬
‫ﻨﻌﺘﺒر ﺒﺄن ﻫذا اﻝﻨﺸﺎط ﻨﺎﺘﺞ ﻓﻘط ﻋن وﺠود ‪6 C‬‬
‫‪. 14‬‬

‫النموذج‬ ‫‪1‬‬ ‫‪2‬‬ ‫‪3‬‬


‫نشاط النموذج )‪(SI‬‬ ‫‪0, 233‬‬ ‫‪0, 215‬‬ ‫‪0, 223‬‬

‫‪ .1‬ﻋرف اﻝﻨﺸﺎط اﻹﺸﻌﺎﻋﻲ ‪ A‬و اﻋط وﺤدﺘﻪ ﻓﻲ اﻝﻨظﺎم اﻝدوﻝﻲ‪.‬‬


‫‪ .2‬ﻗدر ﺴﻨﺔ وﻗوع اﻝزﻝزال اﻝﻤطﺎﺒق ﻝﻠﻌﻴﻨﺔ رﻗم ‪. 3‬‬
‫‪ .3‬ﻨﻘﺘرح ﻝﻠﻌﻴﻨﺘﻴن ‪ 1‬و ‪ 2‬اﻝﺴﻨﺘﻴن ‪ 581‬م و ‪ 1247‬م دون ﺘرﺘﻴب ‪ ،‬أﻨﺴب ﻝﻜل ﻋﻴﻨﺔ اﻝﺴﻨﺔ اﻝﺘﻲ ﺘواﻓﻘﻬﺎ‪ ،‬ﻋﻠل دون‬
‫ﺤﺴﺎب‪.‬‬
‫اﻝﺠزء اﻝﺜﺎﻨﻲ‪ 06) :‬ﻨﻘطﺔ(‬
‫اﻝﺘﻤرﻴن اﻝﺘﺠرﻴﺒﻲ‪ 06) :‬ﻨﻘﺎط(‬
‫اﻝﺠزءان اﻻول و اﻝﺜﺎﻨﻲ ﻤﺴﺘﻘﻼن ‪:‬‬
‫اﻝﺠزء اﻻول ‪:‬‬
‫اﻝﻤﻌطﻴﺎت‪ :‬ﺜﺎﺒت اﻝﻐﺎزات اﻝﻤﺜﺎﻝﻴﺔ‪M (CaCO3 ) = 100 g .mol −1 ، R = 8,31 SI :‬‬
‫ﻜرﺒوﻨﺎت اﻝﻜﺎﻝﺴﻴوم ‪ CaCO3‬ﺠﺴم ﺼﻠب أﺒﻴض ‪ ،‬ﻀﻌﻴف اﻻﻨﺤﻼل ﻓﻲ اﻝﻤﺎء‪ ،‬ﻴﺘﻔﺎﻋل ﻜﻠﻴﺎ ﻤﻊ اﻷﺤﻤﺎض اﻝﻘوﻴﺔ‪ ،‬إﻨﻪ‬
‫اﻝﻤﻜون اﻝرﺌﻴﺴﻲ ﻝﻠﺤﺠر اﻝﺠﻴري و اﻝرﺨﺎم و اﻝﻤرﺠﺎن و اﻝطﺒﺎﺸﻴر و ﻴدﺨل ﻓﻲ ﺘرﻜﻴب ﻤﻌﺠون اﻷﺴﻨﺎن و ﻴﺴﺘﻌﻤل أﻴﻀﺎ‬
‫ﻓﻲ ﺒﻌض اﻷدوﻴﺔ ﻝﺘﺨﻔﻴض اﻝﺤﻤوﻀﺔ ﻓﻲ اﻝﻤﻌدة‪.‬‬
‫‪ -‬ﻨﺄﺨذ ﻋﻴﻨﺔ ﻤن ﻜرﺒوﻨﺎت اﻝﻜﺎﻝﺴﻴوم ﻜﺘﻠﻬﺎ ‪ ، m0‬ﻨﻀﻌﻬﺎ داﺨل دورق و ﻨﻀﻴف ﻝﻬﺎ ﺤﺠﻤﺎ ﻗدرﻩ ‪ Va = 200 mL‬ﻤن‬
‫ﻤﺤﻠول ﺤﻤض ﻜﻠور اﻝﻬﻴدروﺠﻴن ))‪ ( H 3O + ( aq) + Cl − (aq‬ﺘرﻜﻴزﻩ اﻝﻤوﻝﻲ ‪ ca = 0,6 mol. L−1‬ﻝﺘﺸﻜﻴل ﻤزﻴﺞ‬
‫ﺴﺘﻴﻜﻴوﻤﺘري‪.‬‬

‫ﺻﻔﺤﺔ ‪ 4‬ﻣﻦ ‪12‬‬


‫ﺑﻜﺎﻟﻮرﻳﺎ ﺗﺠﺮﻳﺒﻲ ‪2023‬‬ ‫ﺷﻌﺒﺔ‪ :‬رﻳﺎﺿﻴﺎت‪ ،‬ﺗﻘﻨﻲ رﻳﺎﺿﻲ‪.‬‬ ‫ﺗﺎﺑﻊ اﻟﻤﻮﺿﻮع ﻟﻤـﺎدة‪ :‬اﻟﻌﻠﻮم اﻟﻔﻴﺰﻳﺎﺋﻴﺔ‬
‫‪ -‬ﻨﺼل اﻝدورق ﺒﻌد ﺴدﻩ ﺒﺈﺤﻜﺎم إﻝﻰ إﻨﺎء زﺠﺎﺠﻲ‬
‫ﺤﺠﻤﻪ ‪ V = 1L‬ﺒواﺴطﺔ أﻨﺒوب‪ ،‬ﻨزود اﻹﻨﺎء ﺒواﺴطﺔ‬
‫ﻤﻘﻴﺎس اﻝﻀﻐط ﺤﺴب اﻝﺸﻜل ‪. 3‬‬
‫ﻴﺒدأ اﻝﺘﻔﺎﻋل ﻓﻲ اﻝﻠﺤظﺔ ‪ t = 0‬و ﻴﺠري ﻓﻲ درﺠﺔ ﺤ اررة‬
‫اﻝﻤﺨﺒر ‪ θ1 = 25°C‬وﻓق اﻝﻤﻌﺎدﻝﺔ‪:‬‬
‫الشكل ‪3‬‬

‫)‪CaCO3 ( aq ) + 2 H 3O + ( aq ) = Ca 2 + ( aq ) + CO2 ( aq) + 3H 2O ( ℓ‬‬


‫‪dm mg‬‬ ‫ﺘﺤت ﻀﻐط ﻏﻴر ﻤرﺘﻔﻊ ﻴﻤﻜن اﻋﺘﺒﺎر ﻏﺎز ﺜﺎﻨﻲ أﻜﺴﻴد اﻝﻜرﺒون‬
‫) (‬
‫‪dt s‬‬ ‫اﻝﻤﻨطﻠق ﻤﺜﺎﻝﻴﺎ ‪.‬‬
‫‪80‬‬ ‫‪ .1‬اﻨﺸﺊ ﺠدول ﺘﻘدم اﻝﺘﻔﺎﻋل‪.‬‬
‫‪0‬‬
‫)‪t( s‬‬ ‫‪ .2‬ﻋرف اﻝﺘﻘدم اﻷﻋظﻤﻲ ﻝﻠﺘﻔﺎﻋل و اﺤﺴب ﻗﻴﻤﺘﻪ‪.‬‬
‫‪−5‬‬
‫‪ .3‬اﺤﺴب ﻗﻴﻤﺔ اﻝﻜﺘﻠﺔ ‪. m0‬‬
‫‪ .4‬ﺒواﺴطﺔ ﺒرﻨﺎﻤﺞ إﻋﻼم آﻝﻲ ﻤﻨﺎﺴب ﻤﺜﻠﻨﺎ اﻝﺘﻐﻴر اﻝﻠﺤظﻲ‬
‫‪dm‬‬
‫‪ ،‬اﻝﺸﻜل ‪. 4‬‬ ‫ﻝﻜﺘﻠﺔ ﻜرﺒوﻨﺎت اﻝﻜﺎﻝﺴﻴوم ﺒدﻻﻝﺔ اﻝزﻤن ) ‪= f (t‬‬
‫‪dt‬‬
‫الشكل ‪4‬‬
‫‪ .1.4‬ﻋرف اﻝﺴرﻋﺔ اﻝﺤﺠﻤﻴﺔ ﻝﻠﺘﻔﺎﻋل و ﻋﺒر ﻋﻨﻬﺎ ﺒدﻻﻝﺔ ‪، M‬‬
‫‪ Va ، m‬ﺤﻴث ‪ M‬اﻝﻜﺘﻠﺔ اﻝﻤوﻝﻴﺔ ﻝﻜرﺒوﻨﺎت اﻝﻜﺎﻝﺴﻴوم‪.‬‬
‫‪ .2.4‬أﺤﺴب اﻝﺴرﻋﺔ اﻝﺤﺠﻤﻴﺔ ﻝﻠﺘﻔﺎﻋل ﻓﻲ اﻝﻠﺤظﺔ ‪. t = 0‬‬
‫‪ .5‬أﻋدﻨﺎ ﻨﻔس اﻝﺘﺠرﺒﺔ ﻋﻨد درﺠﺔ ﺤ اررة ‪ θ2 = 40°C > θ1‬و ﺴﺠﻠﻨﺎ ﻗﻴم اﻝﻀﻐط ﻋﻠﻰ ﺠﻬﺎز ﻗﻴﺎس اﻝﻀﻐط ﻓﻲ اﻹﻨﺎء‬
‫ﻓﻲ ﻤﺨﺘﻠف اﻝﻠﺤظﺎت اﻝزﻤﻨﻴﺔ ﻓﺘﺤﺼﻠﻨﺎ ﻋﻠﻰ اﻝﺒﻴﺎن ) ‪ ، P = f (t‬اﻝﺸﻜل‪.5‬‬
‫)‪P(×104 Pa‬‬

‫‪10‬‬ ‫الشكل ‪5‬‬

‫‪5‬‬
‫‪20‬‬ ‫)‪t(s‬‬
‫‪ .1.5‬ﻋﺒر ﻋن اﻝﻀﻐط ‪ P‬ﺒدﻻﻝﺔ ‪. Pa ، V ، R ، x ، θ2‬‬
‫ﺤﻴث ‪ Pa‬ﻀﻐط اﻝﻬواء ﻓﻲ اﻻﻨﺎء اﻝزﺠﺎﺠﻲ و اﺴﺘﻨﺘﺞ ‪. Pa‬‬
‫‪ .2.5‬أﺤﺴب ﻜﻤﻴﺔ ﻤﺎدة ﻏﺎز ‪ CO2‬اﻝﻤﺘﺸﻜﻠﺔ ﻓﻲ ﻨﻬﺎﻴﺔ اﻝﺘﻔﺎﻋل ‪.‬‬
‫ﺻﻔﺤﺔ ‪ 5‬ﻣﻦ ‪12‬‬
‫ﺑﻜﺎﻟﻮرﻳﺎ ﺗﺠﺮﻳﺒﻲ ‪2023‬‬ ‫ﺷﻌﺒﺔ‪ :‬رﻳﺎﺿﻴﺎت‪ ،‬ﺗﻘﻨﻲ رﻳﺎﺿﻲ‪.‬‬ ‫ﺗﺎﺑﻊ اﻟﻤﻮﺿﻮع ﻟﻤـﺎدة‪ :‬اﻟﻌﻠﻮم اﻟﻔﻴﺰﻳﺎﺋﻴﺔ‬
‫) ‪V (Co2 ) dP(t‬‬
‫= ‪vvol‬‬ ‫‪ .3.5‬ﺒﻴن أن ﻋﺒﺎرة اﻝﺴرﻋﺔ اﻝﺤﺠﻤﻴﺔ ﻝﻠﺘﻔﺎﻋل ﻓﻲ ﻝﺤظﺔ ) ‪ (t‬ﺘﻌطﻰ ﺒﺎﻝﻌﺒﺎرة‪:‬‬
‫‪VRT‬‬ ‫‪dt‬‬
‫و اﺤﺴب ﻗﻴﻤﺘﻬﺎ ﻋﻨد ‪. t = 0‬‬
‫‪ .4.5‬ﻋرف زﻤن ﻨﺼف اﻝﺘﻔﺎﻋل ‪ t1/2‬و ﺤدد ﻗﻴﻤﺘﻪ ‪.‬‬
‫اﻝﺠزء اﻝﺜﺎﻨﻲ ‪:‬‬
‫ﺘﻌﺘﻤد اﻷﻋﻤدة ﻋﻠﻰ ﺘﺤوﻴل ﺠزء ﻤن اﻝطﺎﻗﺔ اﻝﻨﺎﺘﺠﺔ ﻋن ﺘﻔﺎﻋل أﻜﺴدة إرﺠﺎع اﻝﻰ طﺎﻗﺔ ﻜﻬرﺒﺎﺌﻴﺔ ﺘﺴﺘﻬﻠك ﻋﻨد اﻝﺤﺎﺠﺔ ‪.‬‬
‫ﻨﻘﺘرح ﻓﻲ ﻫذا اﻝﺠزء دراﺴﺔ ﻋﻤود‪ :‬أﻝﻤﻨﻴوم– ﻨﺤﺎس ‪:‬‬
‫ﻴﻨﻤذج اﻝﺘﺤول اﻝﻜﻴﻤﻴﺎﺌﻲ اﻝذي ﻴﺘﺤﻜم ﻓﻲ ﺘﺸﻐﻴل ﻫذا اﻝﻌﻤود ﺒﺎﻝﻤﻌﺎدﻝﺔ ‪:‬‬
‫)‪3Cu 2 + ( aq ) + 2 Al ( s ) = 3Cu ( aq ) + 2 Al 3+ ( aq‬‬
‫‪R‬‬ ‫‪K‬‬
‫ﻨﻨﺠز اﻝﻌﻤود ﻨﺤﺎس‪ -‬أﻝﻤﻨﻴوم ﺒوﺼل ﻨﺼﻔﻲ اﻝﻌﻤود ﺒواﺴطﺔ ﺠﺴر ﻤﻠﺤﻲ‬
‫‪A‬‬ ‫‪A‬‬
‫ﻝﻜﻠور اﻷﻤوﻨﻴوم )) ‪ ، ( NH 4+ (aq) + Cl − ( aq‬اﻝﻨﺼف اﻻول ﻴﺘﻜون ﻤن‬
‫جسر ملحي‬
‫‪Al‬‬ ‫‪Cu‬‬ ‫ﺼﻔﻴﺤﺔ ﻨﺤﺎس ﻤﻐﻤورة ﺠزﺌﻴﺎ ﻓﻲ ﻤﺤﻠول ﻤﺎﺌﻲ ﻝﻜﺒرﻴﺘﺎت اﻝﻨﺤﺎس اﻝﺜﻨﺎﺌﻲ‬
‫))‪ (Cu 2 + (aq) + SO4 2 − ( aq‬ﺘرﻜﻴزﻩ ‪ c0‬و ﺤﺠﻤﻪ ‪ ،V = 50ml‬وﻴﺘﻜون‬
‫اﻝﻨﺼف اﻝﺜﺎﻨﻲ ﻝﻠﻌﻤود ﻤن ﺼﻔﻴﺤﺔ اﻷﻝﻤﻴﻨﻴوم ﻤﻐﻤورة ﺠزﺌﻴﺎ ﻓﻲ ﻤﺤﻠول ﻤﺎﺌﻲ‬
‫الشكل ‪6‬‬ ‫ﻝﻜﻠور اﻷﻝﻤﻴﻨﻴوم ))‪ ( Al ( s ) + 3Cl − ( aq‬ﻝﻪ ﻨﻔس اﻝﺘرﻜﻴز اﻝﻤوﻝﻲ ‪c0‬‬
‫محلول كلور‬ ‫محلول كبريتات‬ ‫و ﻨﻔس اﻝﺤﺠم ‪ .V‬ﻨرﻜب ﺒﻴن ﻗطﺒﻲ اﻝﻌﻤود ﻨﺎﻗﻼ أوﻤﻴﺎ ‪ R‬و أﻤﺒﻴر ﻤﺘ ار‬
‫األلمنيوم‬ ‫النحاس الثنائي‬
‫و ﻗﺎطﻌﺔ ‪ K‬ﺤﺴب اﻝﺸﻜل‪.6‬‬
‫) ‪Cu 2 +  (×10−3 mol .L−1‬‬
‫‪‬‬ ‫‪‬‬ ‫ﻨﻐﻠق اﻝدارة ﻓﻲ اﻝﻠﺤظﺔ ‪ t = 0‬ﻓﻴﻤر ﻓﻴﻬﺎ ﺘﻴﺎر ﻜﻬرﺒﺎﺌﻲ ﺸدﺘﻪ ‪ I‬ﺜﺎﺒﺘﺔ‪.‬‬
‫ﻴﻤﺜل ﻤﻨﺤﻨﻰ اﻝﺸﻜل‪ 7‬ﺘﻐﻴرات اﻝﺘرﻜﻴز اﻝﻤوﻝﻲ ‪ Cu 2 + ( aq) ‬ﻝﺸوارد‬
‫الشكل ‪7‬‬
‫اﻝﻨﺤﺎس اﻝﺜﻨﺎﺌﻲ اﻝﻤوﺠودة ﻓﻲ اﻝﻨﺼف اﻻول ﻝﻠﻌﻤود ﺒدﻻﻝﺔ اﻝزﻤن ‪. t‬‬
‫‪ .1‬أﻋط اﻝرﻤز اﻻﺼطﻼﺤﻲ ﻝﻠﻌﻤود اﻝﻤدروس‪.‬‬
‫‪ .2‬أﻨﺸﺊ ﺠدوﻻ ﻝﺘﻘدم اﻝﺘﻔﺎﻋل‪.‬‬
‫‪ .3‬ﻋﺒر ﻋن اﻝﺘرﻜﻴز اﻝﻤوﻝﻲ ‪ Cu 2 + ( aq) ‬ﺒدﻻﻝﺔ اﻝزﻤن ‪ t‬و ‪ c0‬و ‪I‬‬
‫‪5‬‬
‫و ‪ V‬و ‪.F‬‬
‫‪2,5‬‬ ‫) ‪t (×102 s‬‬ ‫‪ .4‬اﺴﺘﻨﺘﺞ ﻗﻴﻤﺔ ﺸدة اﻝﺘﻴﺎر اﻝﻜﻬرﺒﺎﺌﻲ ‪ I‬ﻓﻲ اﻝدارة ‪ ،‬و اﻝﺘرﻜﻴز اﻝﻤوﻝﻲ‬
‫اﻻﺒﺘداﺌﻲ ‪c0‬‬
‫‪ .5‬ﻴﺴﺘﻬﻠك اﻝﻌﻤود ﻜﻠﻴﺎ ﻓﻲ ﻝﺤظﺔ ‪ ، tmax = 1000 s‬أوﺠد اﻝﺘﻐﻴر ﻓﻲ اﻝﻜﺘﻠﺔ ‪ ∆m‬ﻓﻲ ﻨﺼف ﻋﻤود اﻷﻝﻤﻨﻴوم ﺨﻼل ﻤدة‬
‫اﺸﺘﻐﺎل اﻝﻌﻤود‪ ،‬ﺜم أﺤﺴب ﻗﻴﻤﺘﻪ‪.‬‬
‫ﻴﻌطﻰ‪1F = 96500 SI :‬‬
‫اﻨﺘﻬﻰ اﻝﻤوﻀوع اﻷول‬

‫ﺻﻔﺤﺔ ‪ 6‬ﻣﻦ ‪12‬‬


‫ﺑﻜﺎﻟﻮرﻳﺎ ﺗﺠﺮﻳﺒﻲ ‪2023‬‬ ‫ﺷﻌﺒﺔ‪ :‬رﻳﺎﺿﻴﺎت‪ ،‬ﺗﻘﻨﻲ رﻳﺎﺿﻲ‪.‬‬ ‫ﺗﺎﺑﻊ اﻟﻤﻮﺿﻮع ﻟﻤـﺎدة‪ :‬اﻟﻌﻠﻮم اﻟﻔﻴﺰﻳﺎﺋﻴﺔ‬
‫اﻝﻤوﻀوع اﻝﺜﺎﻨﻲ‬
‫ﻴﺤﺘوي ﻋﻠﻰ )‪ (04‬ﺼﻔﺤﺎت )ﻤن اﻝﺼﻔﺤﺔ ‪7‬ﻤن ‪ 12‬إﻝﻰ اﻝﺼﻔﺤﺔ ‪ 12‬ﻤن ‪(12‬‬
‫اﻝﺠزء اﻷول‪ 13) :‬ﻨﻘطﺔ(‬
‫اﻝﺘﻤرﻴن اﻷول‪ 06) :‬ﻨﻘﺎط(‬
‫ﺘﻌﺘﺒر رﻴﺎﻀﺔ اﻝﻘﻔز ﺒواﺴطﺔ اﻝدراﺠﺎت اﻝﻨﺎرﻴﺔ ﻤن اﻝرﻴﺎﻀﺎت اﻝﻤﺸوﻗﺔ و اﻝﺨطﻴرة ﻓﻲ ﻨﻔس اﻝوﻗت ‪ ،‬ﻷﻨﻪ ﻴﺘم ﻓﻴﻬﺎ اﻝﻘﻔز‬
‫ﻋﻠﻰ ﺤواﺠز طﺒﻴﻌﻴﺔ ‪ .‬ﺴﻨدرس ﻓﻲ ﻫذا اﻝﺘﻤرﻴن ﺤرﻜﺔ اﻝﺠﻤﻠﺔ اﻝﻤﻴﻜﺎﻨﻴﻜﻴﺔ ) ‪ ) ( S‬دراج ‪ +‬دراﺠﺔ ( و اﻝﺘﻲ ﻨﻌﺘﺒرﻫﺎ ﺠﺴﻤﺎ‬
‫ﺼﻠﺒﺎ ﻜﺘﻠﺘﻪ ‪ ، m = 190 kg‬و ﻨﺘوﺼل إذا ﻜﺎﻨت اﻝﻘﻔزة ﻨﺎﺠﺤﺔ ؟ أم ﻻ؟ ﻝﺘﺨطﻲ اﻝﺤﺎﺠز ذو اﻝطول ‪. L = 13m‬‬
‫ﺘﺴﻴر اﻝﺠﻤﻠﺔ ﻋﻠﻰ اﻝﻤﺴﺎر اﻷﻤﻠس اﻷﻓﻘﻲ ) ‪ ( AB‬و ﺘواﺼل ﺤرﻜﺘﻬﺎ ﻋﻠﻰ اﻝﻤﺴﺘوي اﻝﺨﺸن اﻝﻤﺎﺌل ) ‪ ( BC‬ﻝﺘﻐﺎدرﻩ ﻋﻨد‬

‫‪z‬‬ ‫اﻝﻤوﻀﻊ ‪ C‬ﺤﺴب اﻝﺸﻜل‪.1‬‬

‫‬
‫‪vC‬‬ ‫)‪(E‬‬
‫‪C‬‬ ‫‪α‬‬
‫الشكل ‪1‬‬

‫‪d‬‬ ‫‪L‬‬ ‫‪h‬‬


‫‪α‬‬
‫‪A‬‬ ‫‪B‬‬ ‫‪O‬‬ ‫‪x‬‬

‫‬
‫‪ -1‬ﺘﻤر اﻝﺠﻤﻠﺔ ) ‪ ( S‬ﻓﻲ اﻝﻠﺤظﺔ ‪ t = 0‬ﻤن اﻝﻤوﻀﻊ ‪ A‬اﻝذي ﻨﻌﺘﺒرﻩ ﻤﺒدأ ﻝﻠﻔواﺼل و اﻷزﻤﻨﺔ ﺒﺴرﻋﺔ اﺒﺘداﺌﻴﺔ ‪v A‬‬
‫‬
‫ﺒﻔﻌل ﻗوة دﻓﻊ أﻓﻘﻴﺔ ﻝﻠﻤﺤرك ‪ F‬ﻤوازﻴﺔ ﻝﻠﻤﺴﺎر ) ‪ ( ABC‬و ﺜﺎﺒﺘﺔ ﻓﻲ اﻝﺸدة ‪.‬‬
‫ﻗﻤﻨﺎ ﺒﺘﺼوﻴر ﻓﻴدﻴو ﻝﺤرﻜﺔ اﻝﺠﻤﻠﺔ ﺜم ﻋﺎﻝﺠﻨﺎﻩ ﺒﺎﺴﺘﻌﻤﺎل ﺒرﻤﺠﻴﺔ ‪ Avistep 3‬ﻓﺘﺤﺼﻠﻨﺎ ﻋﻠﻰ ﻤﺨطط اﻝﺴرﻋﺔ ﺒدﻻﻝﺔ‬

‫) ‪v ( m.s −1‬‬ ‫اﻝزﻤن ﻋﻠﻰ اﻝﺠزﺌﻴن ) ‪ ( AB‬و ) ‪ ( BC‬ﻋﻠﻰ اﻝﺘرﺘﻴب ﻜﻤﺎ‬


‫ﻴوﻀﺤﻪ ﺒﻴﺎن اﻝﺸﻜل‪.2‬‬
‫الشكل ‪2‬‬

‫‪ -I‬دراﺴﺔ اﻝﺤرﻜﺔ ﻋﻠﻰ اﻝﻤﺴﺘوي اﻷﻓﻘﻲ )‪: (AB‬‬


‫ﻨﻬﻤل اﻻﺤﺘﻜﺎﻜﺎت و ﺘﺄﺜﻴر اﻝﻬواء‪.‬‬
‫ﺘﻌطﻰ‪. g = 10 m.s −2 :‬‬
‫‪ .1‬ﻤﺎﻫو اﻝﻤرﺠﻊ اﻝﻤﻨﺎﺴب ﻝدراﺴﺔ ﻫذﻩ اﻝﺤرﻜﺔ ؟ ﻋرﻓﻪ‪ .‬و ﻤﺎ‬
‫ﻫﻲ اﻝﻔرﻀﻴﺔ اﻝﻤﺘﻌﻠﻘﺔ ﺒﻬذا اﻝﻤرﺠﻊ و اﻝﺘﻲ ﺘﺴﻤﺢ ﺒﺘطﺒﻴق‬
‫اﻝﻘﺎﻨون اﻝﺜﺎﻨﻲ ﻝﻨﻴوﺘن؟‬
‫‪4‬‬
‫‪ .2‬ﺒﺘطﺒﻴق اﻝﻘﺎﻨون اﻝﺜﺎﻨﻲ ﻝﻨﻴوﺘن أوﺠد ﻋﺒﺎرة ﺘﺴﺎرع اﻝﺤرﻜﺔ‬
‫‪0‬‬ ‫‪1‬‬ ‫)‪t( s‬‬ ‫و اﺴﺘﻨﺘﺞ طﺒﻴﻌﺘﻬﺎ‪.‬‬

‫ﺻﻔﺤﺔ ‪ 7‬ﻣﻦ ‪12‬‬


‫ﺑﻜﺎﻟﻮرﻳﺎ ﺗﺠﺮﻳﺒﻲ ‪2023‬‬ ‫ﺷﻌﺒﺔ‪ :‬رﻳﺎﺿﻴﺎت‪ ،‬ﺗﻘﻨﻲ رﻳﺎﺿﻲ‪.‬‬ ‫ﺗﺎﺑﻊ اﻟﻤﻮﺿﻮع ﻟﻤـﺎدة‪ :‬اﻟﻌﻠﻮم اﻟﻔﻴﺰﻳﺎﺋﻴﺔ‬
‫‪ .3‬اﺴﺘﻨﺘﺞ ﺒﻴﺎﻨﻴﺎ ‪:‬‬
‫‬
‫‪ .1.3‬ﻗﻴﻤﺔ ﺘﺴﺎرع اﻝﺤرﻜﺔ وﺸدة ﻗوة دﻓﻊ اﻝﻤﺤرك ‪. F‬‬
‫‪ .2.3‬اﻝﻤﺴﺎﻓﺔ اﻝﻤﻘطوﻋﺔ ‪. AB‬‬
‫‪ -II‬دراﺴﺔ اﻝﺤرﻜﺔ ﻋﻠﻰ اﻝﻤﺴﺘوي اﻝﻤﺎﺌل ) ‪: ( BC‬‬
‫‬
‫ﻓﻲ ﻫذا اﻝﺠزء ﻤن اﻝﻤﺴﺎر‪ ،‬ﺘﺨﻀﻊ اﻝﺠﻤﻠﺔ ) ‪ ( S‬اﻝﻰ ﻗوة اﺤﺘﻜﺎك ‪ f‬ﻤوازﻴﺔ ﻝﻠﻤﺴﺎر و ﻤﻌﺎﻜﺴﺔ ﻝﺠﻬﺔ اﻝﺤرﻜﺔ‪.‬‬
‫‪ .1‬ﺒﺘطﺒﻴق اﻝﻘﺎﻨون اﻝﺜﺎﻨﻲ ﻝﻨﻴوﺘن ﻋﻠﻰ ﻤرﻜز ﻋطﺎﻝﺔ اﻝﺠﻤﻠﺔ‪ ،‬أﺜﺒت أن ‪:‬‬
‫‪dv‬‬ ‫‪F− f‬‬
‫‪= − g .sin α +‬‬
‫‪dt‬‬ ‫‪m‬‬
‫‪ .2‬اﻜﺘب اﻝﻤﻌﺎدﻝﺔ اﻝزﻤﻨﻴﺔ ﻝﻠﺴرﻋﺔ ) ‪. v (t‬‬
‫‪ .3‬اﻋﺘﻤﺎدا ﻋﻠﻰ ﻤﺨطط اﻝﺴرﻋﺔ ﺒدﻻﻝﺔ اﻝزﻤن أوﺠد ﻗﻴﻤﺔ ﺘﺴﺎرع ﺤرﻜﺔ ﻤرﻜز ﻋطﺎﻝﺔ اﻝﺠﻤﻠﺔ ) ‪. ( S‬‬
‫‪ .4‬اوﺠد ﺒﻴﺎﻨﻴﺎ اﻝﻤﺴﺎﻓﺔ اﻝﻤﻘطوﻋﺔ ‪. BC‬‬
‫‪ .5‬اﺴﺘﻨﺘﺞ ﺴرﻋﺔ وﺼول اﻝﺠﻤﻠﺔ اﻝﻰ اﻝﻤوﻀﻊ ‪. C‬‬
‫‪ -III‬دراﺴﺔ ﺤرﻜﺔ اﻝﺠﻤﻠﺔ ﺒﻌد ﻤﻐﺎدرﺘﻬﺎ اﻝﻤوﻀﻊ ‪: C‬‬
‫ﺘﻐﺎدر اﻝﺠﻤﻠﺔ اﻝﻤوﻀﻊ ‪ C‬ﻝﺘﺴﻘط ﻓﻲ ﻤﺠﺎل اﻝﺠﺎذﺒﻴﺔ اﻷرﻀﻴﺔ ) ﻨﻬﻤل ﺘﺄﺜﻴر اﻝﻬواء و داﻓﻌﺔ أرﺨﻤﻴدس( ‪.‬‬
‫‪ .1‬ﺒﺘطﺒﻴق اﻝﻘﺎﻨون اﻝﺜﺎﻨﻲ ﻝﻨﻴوﺘن ‪ :‬ادرس طﺒﻴﻌﺔ اﻝﺤرﻜﺔ ﻓﻲ اﻝﻤﻌﻠم ) ‪. (ox , oy‬‬
‫‪ .2‬أوﺠد اﻝﻤﻌﺎدﻝﺘﻴن اﻝزﻤﻨﻴﺘﻴن ﻝﻤرﻜﺒﺘﻲ اﻝﺴرﻋﺔ ‪ v x (t ) ، v z (t ) :‬ﺒدﻻﻝﺔ ‪. α‬‬
‫أﻋطت اﻝﻤﻌﺎدﻝﺘﻴن اﻝزﻤﻨﻴﺘﻴن ﻝﻠﻤوﻀﻊ‪:‬‬ ‫) ‪(S‬‬ ‫‪ .3‬اﻝدراﺴﺔ اﻝﺘﺠرﻴﺒﻴﺔ اﻝﺘﻲ ﺘﻤت ﻋﻠﻰ ﺤرﻜﺔ اﻝﺠﻤﻠﺔ‬
‫‪ x (t ) = 16, 2 t‬‬
‫‪‬‬
‫‪ z (t ) = − 5t + 8t + 17,5‬‬
‫‪2‬‬

‫‪ .1.3‬اوﺠد ﻤﻌﺎدﻝﺔ اﻝﻤﺴﺎر ) ‪. z = f ( x‬‬


‫‪ .2.3‬اﺤﺴب زاوﻴﺔ اﻝﻘذف ‪ α‬و اﻻرﺘﻔﺎع ‪. h‬‬
‫‬
‫‪ .3.3‬اﺴﺘﻨﺘﺞ ﺸدة ﻗوة اﻻﺤﺘﻜﺎك ‪. f‬‬
‫‪ -4‬ﻫل ﻴﻨﺠﺢ اﻝدراج ﺒﺎﻝﻘﻔزة أم ﻻ إذا ﻤر ﻓوق اﻝﺤﺎﺠز ﺒـ ‪ 0,5m‬؟ ﻋﻠﻤﺎ أن ‪. d = 11m :‬‬
‫اﻝﺘﻤرﻴن اﻝﺜﺎﻨﻲ‪ 04) :‬ﻨﻘﺎط(‬
‫ﺘﺤدث ﺘﻔﺎﻋﻼت اﻻﻨدﻤﺎج اﻝﻨووي داﺨل اﻝﺸﻤس ﻋﻨد درﺠﺔ ﺤ اررة ﺘﻘﺎرب ‪ 20‬ﻤﻠﻴون درﺠﺔ ﻤﺌوﻴﺔ و ﻴﺘﻜون ﻋﻨﻬﺎ اﻝﻬﻴﻠﻴوم‬
‫اﻨطﻼﻗﺎ ﻤن اﻝﻬﻴدروﺠﻴن ﺤﺴب ﺜﻼث ﻤراﺤل‪.‬‬
‫ﻴﻬدف اﻝﺘﻤرﻴن إﻝﻰ ﺘﺤدﻴد اﻝﻜﺘﻠﺔ اﻝﻀﺎﺌﻌﺔ ﻤن اﻝﺸﻤس ﺤﺎﻝﻴﺎ ﻤﻨذ ﻨﺸﺄﺘﻬﺎ‪.‬‬
‫اﻝﻤﻌطﻴﺎت‪:‬‬
‫‪، m( 42 He) = 4,0015 u ، m(11 H ) = 1,00728 u ، 1u = 931,5 MeV / c 2‬‬
‫‪) M S = 2 ×1030 kg ، 1ans = 365, 25Jours‬ﻜﺘﻠﺔ اﻝﺸﻤس( ‪c = 3 × 108 m.s −1 ،‬‬

‫ﺻﻔﺤﺔ ‪ 8‬ﻣﻦ ‪12‬‬


‫ﺑﻜﺎﻟﻮرﻳﺎ ﺗﺠﺮﻳﺒﻲ ‪2023‬‬ ‫ﺷﻌﺒﺔ‪ :‬رﻳﺎﺿﻴﺎت‪ ،‬ﺗﻘﻨﻲ رﻳﺎﺿﻲ‪.‬‬ ‫ﺗﺎﺑﻊ اﻟﻤﻮﺿﻮع ﻟﻤـﺎدة‪ :‬اﻟﻌﻠﻮم اﻟﻔﻴﺰﻳﺎﺋﻴﺔ‬
‫‪ .1‬اﻜﺘب ﻤﻌﺎدﻝﺔ اﻻﻨدﻤﺎج اﻝذي ﻴﺤدث ﻓﻲ ﻜل ﻤرﺤﻠﺔ ‪:‬‬
‫و ﺠﺴﻴﻤﺔ ‪. ZA X‬‬ ‫‪2‬‬
‫‪1H‬‬ ‫اﻝﻤرﺤﻠﺔ اﻻوﻝﻰ ‪ :‬إﻨدﻤﺎج ﻨواﺘﻲ اﻝﻬﻴدروﺠﻴن ‪ 11 H‬ﻴؤدي اﻝﻰ ﺘﻜون اﻝدوﺘﻴرﻴوم‬
‫ﻤﺎ طﺒﻴﻌﺔ ﻫذﻩ اﻝﺠﺴﻴﻤﺔ ؟‬
‫ﻴؤدي اﻝﻰ ﺘﻜون اﻝﻬﻴﻠﻴوم ‪ . 32 He‬ﻴراﻓق ﻫذا اﻝﺘﻔﺎﻋل اﻨﺒﻌﺎث اﺸﻌﺎع ‪γ‬‬ ‫‪2‬‬
‫‪1H‬‬ ‫و ﻨواة‬ ‫‪1‬‬
‫‪1H‬‬ ‫اﻝﻤرﺤﻠﺔ اﻝﺜﺎﻨﻴﺔ ‪ :‬اﻨدﻤﺎج ﻨواة‬
‫ﻜﻴف ﺘﻔﺴر اﻨﺒﻌﺎث ﻫذا اﻻﺸﻌﺎع ؟‬
‫اﻝﻤرﺤﻠﺔ اﻝﺜﺎﻝﺜﺔ ‪ :‬اﻨدﻤﺎج ﻨواﺘﻲ ‪ 32 He‬ﻴؤدي اﻝﻰ ﺘﻜون اﻝﻬﻴﻠﻴوم ‪ 42 He‬وﻨواﺘﻴن ‪ ZA Y‬ﻤﺘﻤﺎﺜﻠﻴن‪ .‬ﻤﺎ طﺒﻴﻌﺘﻬﻤﺎ؟‬
‫‪ .2‬اﺴﺘﻨﺘﺞ اﻝﻤﻌﺎدﻝﺔ اﻝﻨﻬﺎﺌﻴﺔ ﻝﺘﻔﺎﻋل اﻻﻨدﻤﺎج اﻝذي ﻴﺤدث داﺨل اﻝﺸﻤس ‪.‬‬
‫‪4 11 H → 42 He + 2 10 e + 2 γ‬‬ ‫‪ .3‬ﻴﺤدث ﺘﻔﺎﻋل اﻨدﻤﺎج ﻨووي آﺨر ﻓﻲ اﻝﺸﻤس وﻓق اﻝﻤﻌﺎدﻝﺔ‪:‬‬
‫‪ .1.3‬احسب بالجول ) ‪ ، ( J‬الطاقة المحررة عن تشكل نواة ھيليوم واحدة في الشمس‪.‬‬
‫‪ .2.3‬ﺘﻘدر اﻻﺴﺘطﺎﻋﺔ اﻻﺸﻌﺎﻋﻴﺔ ﻝﻠﺸﻤس ﺒـ ‪ . 3,9 . 1026W‬ﺒﻔرض أن اﻝطﺎﻗﺔ اﻝﺘﻲ ﺘﺤررﻫﺎ اﻝﺸﻤس ﻫﻲ ﻨﺘﻴﺠﺔ ﺘﻔﺎﻋـل‬
‫اﻻﻨدﻤﺎج اﻝﺴﺎﺒق ‪،‬أﺤﺴب اﻝﻨﻘص اﻝﺤﺎدث ﻓﻲ ﻜﺘﻠﺔ اﻝﺸﻤس ﺨﻼل ﻜل ﺜﺎﻨﻴﺔ ‪.‬‬
‫‪ .3.3‬ﻴﻘـدر ﻋﻤـر اﻝﺸــﻤس ﻤﻨـذ ﺒداﻴـﺔ ﺴــطوﻋﻬﺎ ﺒﺤـواﻝﻲ ‪ ، 4,6 . 109 ans‬ﺠــد اﻝﻨﺴـﺒﺔ اﻝﻤﺌوﻴـﺔ ‪ P %‬ﻝﻠﻜﺘﻠــﺔ اﻝﻀـﺎﺌﻌﺔ ﻤــن‬
‫اﻝﺸﻤس ﺤﺘﻰ اﻵن؟‬
‫‪E‬‬
‫‪EK‬‬ ‫اﻝﺘﻤرﻴن اﻝﺜﺎﻝث‪ 06) :‬ﻨﻘﺎط(‬
‫ﺘﺴﺘﻌﻤل اﻝوﺸﺎﺌﻊ‪ ،‬اﻝﻤﻜﺜﻔﺎت و اﻝﻨواﻗل اﻷوﻤﻴﺔ ﻓﻲ ﻜﺜﻴر ﻤن‬
‫اﻷﺠﻬزة اﻝﻜﻬرﺒﺎﺌﻴﺔ‪ ،‬و ﺘﺨﺘﻠف وظﺎﺌﻔﻬﺎ ﺤﺴب ﻜﻴﻔﻴﺔ رﺒطﻬﺎ‬
‫‪R1‬‬ ‫) ‪( L, r‬‬ ‫‪R2‬‬ ‫و ﻤﺠﺎﻻت اﺴﺘﻌﻤﺎﻻﺘﻬﺎ‪.‬‬
‫ﻴﻬدف اﻝﺘﻤرﻴن إﻝﻰ دراﺴﺔ اﻝدارة ‪. RL‬‬
‫الشكل ‪3‬‬ ‫ﻨرﻜب اﻝدارة اﻝﻜﻬرﺒﺎﺌﻴﺔ اﻝﻤوﻀﺤﺔ ﺒﺎﻝﺸﻜل‪ .3‬واﻝﻤؤﻝﻔﺔ ﻤن‪:‬‬
‫‪Y1‬‬ ‫‪Y2‬‬
‫)‪u(V‬‬ ‫‪ -‬ﻤوﻝد ﻝﻠﺘوﺘر اﻝﺜﺎﺒت ‪ E‬؛‬
‫‪ ( L, r‬؛‬ ‫)‬ ‫‪ -‬وﺸﻴﻌﺔ‬
‫الشكل ‪4‬‬
‫‪ -‬ﻨﺎﻗﻠﻴﻴن أوﻤﻴﻴن ﻤﻘﺎوﻤﺘﻴﻬﻤﺎ ‪ R1 = 100Ω‬و ‪ R2‬؛‬
‫‪ -‬ﻗﺎطﻌﺔ ‪ K‬؛‬
‫ﻨوﺼل اﻝدارة اﻝﻜﻬرﺒﺎﺌﻴﺔ ﺒراﺴم اﻫﺘزاز ذي ذاﻜرة ﻜﻤﺎ ﻫو‬
‫) ‪(a‬‬ ‫ﻤوﻀﺢ ﻓﻲ اﻝﺸﻜل ‪ . 3‬ﺜم ﻨﻐﻠق اﻝﻘﺎطﻌﺔ ‪ K‬ﻋﻨد اﻝﻠﺤظﺔ‬
‫‪ ، t = 0‬ﻓﻨﺸﺎﻫد ﻋﻠﻰ اﻝﺸﺎﺸﺔ اﻝﻤﻨﺤﻨﻴﻴن اﻝﺒﻴﺎﻨﻴن‬
‫) ‪ ( a‬و ) ‪ ( b‬اﻝﻤﺒﻴﻨﻴن ﻓﻲ اﻝﺸﻜل ‪. 4‬‬
‫)‪( b‬‬

‫‪1‬‬ ‫‪ .1‬ﻤﺎ ﻫﻲ اﻝظﺎﻫرة اﻝﺘﻲ ﺘﺤدث ﻓﻲ اﻝدارة؟‬


‫‪ .2‬ارﻓق ﻜل ﻤﻨﺤﻨﻰ ﺒﺎﻝﻤدﺨل اﻝﻤواﻓق ﻝﻪ ﻤﻊ اﻝﺘﻌﻠﻴل‪.‬‬
‫‪0‬‬ ‫‪1‬‬ ‫) ‪t (ms‬‬
‫ﺻﻔﺤﺔ ‪ 9‬ﻣﻦ ‪12‬‬
‫ﺑﻜﺎﻟﻮرﻳﺎ ﺗﺠﺮﻳﺒﻲ ‪2023‬‬ ‫ﺷﻌﺒﺔ‪ :‬رﻳﺎﺿﻴﺎت‪ ،‬ﺗﻘﻨﻲ رﻳﺎﺿﻲ‪.‬‬ ‫ﺗﺎﺑﻊ اﻟﻤﻮﺿﻮع ﻟﻤـﺎدة‪ :‬اﻟﻌﻠﻮم اﻟﻔﻴﺰﻳﺎﺋﻴﺔ‬
‫‪ .3‬اوﺠد اﻝﻤﻌﺎدﻝﺔ اﻝﺘﻔﺎﻀﻠﻴﺔ اﻝﺘﻲ ﻴﺤﻘﻘﻬﺎ اﻝﺘﻴﺎر ) ‪ i ( t‬اﻝﻤﺎر ﻓﻲ اﻝدارة‪.‬‬
‫‪t‬‬
‫‪−‬‬
‫‪ i (t ) = A + Be‬ﺤﻼ ﻝﻠﻤﻌﺎدﻝﺔ اﻝﺘﻔﺎﻀﻠﻴﺔ اﻝﺴﺎﺒﻘﺔ‪ ،‬أوﺠد ﻋﺒﺎرة ﻜل ﻤن‪ τ ، A :‬و ‪. B‬‬ ‫‪τ‬‬ ‫‪ .1.4‬ﺒﺎﻋﺘﺒﺎر اﻝﻌﺒﺎرة‬
‫‪ .2.4‬ﺒﻴن ﺒﺎﻝﺘﺤﻠﻴل اﻝﺒﻌدي و اﻻﻋﺘﻤﺎد ﻋﻠﻰ اﻝﻤﻌﺎدﻝﺔ اﻝﺘﻔﺎﻀﻠﻴﺔ أن ‪ τ‬ﻤﺘﺠﺎﻨس ﻤﻊ اﻝزﻤن‪.‬‬
‫‪ .3.4‬اﺴﺘﻨﺘﺞ اﻝﻌﺒﺎرة اﻝﻠﺤظﻴﺔ ﻝﻠﺘوﺘر ‪ U1‬ﺒﻴن طرﻓﻲ اﻝﻨﺎﻗل اﻷوﻤﻲ ) ‪ ، uR1 ( t‬ﺒدﻻﻝﺔ ‪. τ ، I 0 ، R1‬‬
‫)‪u(V‬‬ ‫‪ .4.4‬ﺒﻴن أن اﻝﺘوﺘر ‪ U 2‬ﻴﻜﺘب ﺒﺎﻝﺸﻜل‪:‬‬
‫‪t‬‬
‫‪−‬‬
‫الشكل ‪5‬‬ ‫‪U 2 = ( R2 + r ) I 0 + R1 I 0 e‬‬ ‫‪τ‬‬

‫‪ .5‬اﻋﺘﻤﺎدا ﻋﻠﻰ اﻝﺒﻴﺎن اﻝﺴﺎﺒق أوﺠد ﻗﻴم ﻜل ﻤن ‪ :‬اﻝﻘوة‬


‫اﻝﻤﺤرﻜﺔ اﻝﻜﻬرﺒﺎﺌﻴﺔ ‪ E‬ﻝﻠﻤوﻝد ‪ ،‬ذاﺘﻴﺔ اﻝوﺸﻴﻌﺔ ‪. L‬‬
‫‪c‬‬ ‫ﺒطرﻴﻘﺘﻴن‪ ،‬ﺸدة اﻝﺘﻴﺎر اﻷﻋظﻤﻴﺔ ‪. I 0‬‬
‫‪ .6‬ﻨﺤﺎﻓظ ﻋﻠﻰ اﻝﺘرﻜﻴب اﻝﺘﺠرﻴﺒﻲ اﻝﺴﺎﺒق وﻨﺴﺘﺒدل‬
‫اﻝوﺸﻴﻌﺔ اﻝﺴﺎﺒﻘﺔ ﺒوﺸﻴﻌﺔ ﺼرﻓﺔ ﻝﻬﺎ ﻨﻔس ذاﺘﻴﺔ اﻝوﺸﻴﻌﺔ‬
‫‪d‬‬
‫) ‪(d‬‬ ‫اﻝﺴﺎﺒﻘﺔ ﻓﻨﺤﺼل ﻋﻠﻰ اﻝﻤﻨﺤﻨﻴﻴن اﻝﺒﻴﺎﻨﻴن ) ‪ ( c‬و‬
‫‪1‬‬
‫اﻝﻤﺒﻴﻨﻴن ﻓﻲ اﻝﺸﻜل ‪ . 5‬أوﺠد ﻓﻲ ﻫذﻩ اﻝﺤﺎﻝﺔ ﻗﻴﻤﺔ ‪ R2‬ﺜم‬

‫‪0‬‬ ‫اﺴﺘﻨﺘﺞ ‪ r‬ﻤﻘﺎوﻤﺔ اﻝوﺸﻴﻌﺔ اﻝﺴﺎﺒﻘﺔ‪.‬‬


‫‪1‬‬ ‫) ‪t (ms‬‬
‫اﻝﺠزء اﻝﺜﺎﻨﻲ‪ 06) :‬ﻨﻘطﺔ(‬
‫اﻝﺘﻤرﻴن اﻝﺘﺠرﻴﺒﻲ‪ 06) :‬ﻨﻘﺎط(‬
‫‪ − I‬ﻨﺤﻀر ﻤﺤﻠوﻻ ) ‪ ( S1‬ﻝﺤﻤض ‪ AH‬ﺒﺘرﻜﻴز ﻤوﻝﻲ ‪ C1 = 10−2 mol. L−1‬وﺤﺠم ‪ V1 = 50mL‬ﺒﺘﻤدﻴد اﻝﺤﺠم ‪V0‬‬
‫ﻝﻤﺤﻠول ‪ S0‬ﺘرﻜﻴزﻩ اﻝﻤوﻝﻲ ‪. C0 = 5.10−2 mol. L−1‬‬
‫‪ .1.1‬اﺴﺘﻨﺘﺞ اﻝﺤﺠم اﻝﻼزم ‪ V0‬ﻝﻬذﻩ اﻝﻌﻤﻠﻴﺔ‪.‬‬
‫‪ .2.1‬ﺼف اﻝﺒروﺘوﻜول اﻝﺘﺠرﻴﺒﻲ اﻝذي ﻴﺴﻤﺢ ﺒﺘﺤﻀﻴر اﻝﻤﺤﻠول ‪ S 1‬ﺒﺎﺨﺘﻴﺎر اﻷدوات اﻝﻤﻨﺎﺴﺒﺔ ﻤن ﺒﻴن اﻝزﺠﺎﺠﻴﺎت‬
‫اﻝﻤﺒﻨﺔ ﻓﻲ اﻝﺸﻜل‪.6‬‬
‫‪ -‬ﻤﺎﺼﺔﻋﻴﺎرﻴﺔ‪ 5mL :‬و ‪10mL‬و ‪ 20mL‬؛‬
‫مخبار‬ ‫حوﺟلة‬ ‫‪ -‬ﺤوﺠﻠﺔ ﻋﻴﺎرﻴﺔ‪ 20mL :‬و ‪ 50mL‬و ‪ 100mL‬؛‬
‫مدرج‬ ‫عيارية‬ ‫ماصة‬
‫‪ -‬ﻤﺨﺒﺎر ﻤدرج‪ 50mL :‬و ‪ 100mL‬؛‬
‫الشكل ‪6‬‬ ‫‪ .2‬ﻨﻘﻴس ﺒواﺴطﺔ ﺠﻬﺎز ‪– pH‬ﻤﺘر‪ pH ،‬اﻝﻤﺤﻠول ‪ S 0‬ﻓﻨﺤﺼل ﻋﻠﻰ اﻝﻘﻴﻤﺔ ‪. pH 0 = 3,05‬‬
‫‪ .1.2‬ﺒﻴن أن اﻝﺤﻤض ‪ AH‬ﻀﻌﻴف‪.‬‬
‫‪1‬‬
‫‪ .2.2‬ﻤن أﺠل ﻤﺤﻠول ﻤﺎﺌﻲ ﻝﺤﻤض ﻀﻌﻴف ﻨﻌطﻲ اﻝﻌﻼﻗﺔ‪ . pH = ( pKa − log C ) :‬ﺘﺤﻘق أن ﻗﻴﻤﺔ ‪pH‬‬
‫‪2‬‬
‫اﻝﻤﺤﻠول ‪ S 1‬ﻫﻲ‪. pH1 = 3,4 :‬‬
‫‪ .3.2‬اﺴﺘﻨﺘﺞ ﻗﻴﻤﺔ ‪ pKa‬اﻝﺜﻨﺎﺌﻴﺔ ) ‪ ( AH / A−‬اﻝﻤواﻓﻘﺔ ﺜم ﺘﻌرف ﻋﻠﻰ اﻝﺤﻤض ‪ AH‬ﻤن ﺒﻴن اﻷﺤﻤﺎض اﻝﻤﻘﺘرﺤﺔ ﻓﻲ‬

‫ﺻﻔﺤﺔ ‪ 10‬ﻣﻦ ‪12‬‬


‫ﺑﻜﺎﻟﻮرﻳﺎ ﺗﺠﺮﻳﺒﻲ ‪2023‬‬ ‫ﺷﻌﺒﺔ‪ :‬رﻳﺎﺿﻴﺎت‪ ،‬ﺗﻘﻨﻲ رﻳﺎﺿﻲ‪.‬‬ ‫ﺗﺎﺑﻊ اﻟﻤﻮﺿﻮع ﻟﻤـﺎدة‪ :‬اﻟﻌﻠﻮم اﻟﻔﻴﺰﻳﺎﺋﻴﺔ‬
‫اﻝﺠدول اﻝﻤﻘﺎﺒل‪:‬‬
‫‪pK A‬‬ ‫الصيغة‬ ‫الحمض‬
‫‪4, 2‬‬ ‫‪C6 H 5 COOH‬‬ ‫حمض البنزويك‬
‫‪3,8‬‬ ‫‪HCOOH‬‬ ‫حمض الميثانويك‬
‫‪4,78‬‬ ‫‪CH 3COOH‬‬ ‫حمض االيثانويك‬

‫‪ .3‬ﻨذ ّﻜر ﺒﻌﺒﺎرة اﻝﻨﺴﺒﺔ اﻝﻨﻬﺎﺌﻴﺔ ﻝﺘﻘدم ﺘﻔﺎﻋل اﻨﺤﻼل ﺤﻤض ﻓﻲ اﻝﻤﺎء ﺒدﻻﻝﺔ ‪ pH‬اﻝﻤﺤﻠول و اﻝﺘرﻜﻴز اﻝﻤوﻝﻲ ﻝﻪ‪:‬‬
‫‪10− pH‬‬
‫= ‪ .τ f‬اﺤﺴب اﻝﻨﺴﺒﺘﻴن ‪ τ f0‬و ‪ τ f1‬ﻓﻲ ﻜل ﻤن اﻝﻤﺤﻠوﻝﻴن ‪ S0‬و ‪ S1‬ﻋﻠﻰ اﻝﺘواﻝﻲ‪ ،‬ﻤﺴﺘﻨﺘﺠﺎ أﺜر ﻋﻤﻠﻴﺔ‬
‫‪C‬‬
‫اﻝﺘﻤدﻴد ﻋﻠﻰ ﺘﻔﻜك اﻝﺤﻤض‪.‬‬
‫‪ − II‬ﻴﻨﺘﺞ ﻋن اﻝﺘﻔﺎﻋل اﻝﺤﺎدث ﺒﻴن اﻝﻤﻴﺜﺎﻨول ‪CH 3OH‬‬
‫وﺤﻤض اﻹﻴﺜﺎﻨوﻴك ‪ ، CH 3COOH‬ﻤرﻜب ﻋﻀوي ﻋﺒﺎرة‬
‫ﻋن ﺴﺎﺌل ﻗﺎﺒل ﻝﻼﺸﺘﻌﺎل ﻤﻊ راﺌﺤﺔ ﻝطﻴﻔﺔ ﻤﻤﻴزة‪ .‬ﻴﺴﺘﺨدم‬
‫اﻝﺴ ّﻤﻴﺔ ﻓﻲ اﻝﻤواد‬
‫ﻓﻲ ﺒﻌض اﻷﺤﻴﺎن ﻜﻤذﻴب ﻤﻨﺨﻔض ُ‬
‫اﻝﻼﺼﻘﺔ‪ ،‬اﻝدﻫﺎﻨﺎت‪ ،‬وﻤزﻴﻼت طﻼء اﻷظﺎﻓر‪....‬‬
‫اﻝﺘﺠرﺒﺔ‪:‬‬
‫ﻤﺠﻤوﻋﺔ أﻨﺎﺒﻴب اﺨﺘﺒﺎر ﻤزود ﻜل ﻤﻨﻬﺎ ﺒﻤﺒرد ﻫواﺌﻲ )ﻗﻨﺎة‬
‫ﻀﻴﻘﺔ( ﻤرﺘﺒﺔ داﺨل ﺤوض ﻤﺎﺌﻲ ﻓﻴﻪ ﻗطﻊ ﻤن اﻝﺠﻠﻴد‪.‬‬
‫أدﺨﻠﻨﺎ ﻓﻲ ﻜل أﻨﺒوب ‪ n1 = 0,05mol‬ﻤن ﺤﻤض اﻹﻴﺜﺎﻨوﻴك‬
‫و ‪ n2 = 0,05mol‬ﻤن اﻝﻤﻴﺜﺎﻨول‪.‬‬
‫ﻓﻲ اﻝﻠﺤظﺔ ‪ t = 0‬ﻨﻀﻊ ﻤﺠﻤوﻋﺔ اﻷﻨﺎﺒﻴب ﻓﻲ ﺤﻤﺎم ﻤﺎﺌﻲ درﺠﺔ ﺤ اررﺘﻪ ‪. 80°C‬‬
‫و ﻓﻲ ﻝﺤظﺎت زﻤﻨﻴﺔ ﻤﺨﺘﻠﻔﺔ ﻨﺄﺨذ ﻓﻲ ﻜل ﻤرة أﺤد اﻷﻨﺎﺒﻴب و ﻨﻀﻊ ﺴرﻴﻌﺎ ﻓﻲ اﻝﺤوض اﻝﺒﺎرد و ﻨﻌﺎﻴر اﻝﺤﻤض اﻝﻤﺘﺒﻘﻲ‬
‫ﺒواﺴطﺔ ﻤﺤﻠول ﻤﺎﺌﻲ ﻝﻠﺼود ﺘرﻜﻴزﻩ ‪ cb = 1,5mol.L−1‬ﻓﻲ وﺠود اﻝﻜﺎﺸف ﻓﻴﻨول ﻓﺘﺎﻝﻴﻴن‪.‬‬
‫ﺴم اﻝﻤرﻜب اﻝﻨﺎﺘﺞ‪.‬‬
‫‪ .1‬اﻜﺘب ﺒﺎﺴﺘﻌﻤﺎل اﻝﺼﻴﻎ ﻨﺼف اﻝﻤﻔﺼﻠﺔ ﻤﻌﺎدﻝﺔ اﻝﺘﻔﺎﻋل اﻝﻤدروس و ّ‬
‫‪ .2‬ﻤﺎ دور ﻜل ﻤن‪ :‬اﻝﻘﻨﺎة اﻝﻀﻴﻘﺔ ﻋﻠﻰ ﻜل أﻨﺒوب‪ ،‬اﻝﺤوض اﻝﺠﻠﻴدي واﻝﻔﻴﻨول ﻓﺘﺎﻝﻴﻴن؟‬
‫‪ .3‬أﻨﺸﺊ ﺠدول وﺼﻔﻲ ﻝﺘﻘدم اﻝﺘﻔﺎﻋل ﺒدﻻﻝﺔ ‪ n1‬و ‪ n2‬واﻝﺘﻘدم ‪. x‬‬
‫‪x‬‬
‫= ) ‪،τ ( %‬‬ ‫‪ .4‬ﺴﻤﺤت اﻝﻨﺘﺎﺌﺞ ﺒﻤﺘﺎﺒﻌﺔ ﺘﻐﻴرات ﻨﺴﺒﺔ اﻝﺘﻘدم ‪ τ‬ﺒدﻻﻝﺔ اﻝزﻤن ﻤﻌﺒ ار ﻋﻨﻪ ﺒﻨﺴﺒﺔ ﻤﺌوﻴﺔ أي‪.100 :‬‬
‫‪xmax‬‬
‫ﻓﻨﺤﺼل ﻋﻠﻰ اﻝﺒﻴﺎن اﻝﻤﻤﺜل ﻓﻲ اﻝﺸﻜل ‪. 7‬‬

‫ﺻﻔﺤﺔ ‪ 11‬ﻣﻦ ‪12‬‬


‫ﺑﻜﺎﻟﻮرﻳﺎ ﺗﺠﺮﻳﺒﻲ ‪2023‬‬ ‫ﺷﻌﺒﺔ‪ :‬رﻳﺎﺿﻴﺎت‪ ،‬ﺗﻘﻨﻲ رﻳﺎﺿﻲ‪.‬‬ ‫ﺗﺎﺑﻊ اﻟﻤﻮﺿﻮع ﻟﻤـﺎدة‪ :‬اﻟﻌﻠﻮم اﻟﻔﻴﺰﻳﺎﺋﻴﺔ‬
‫‪ .1.4‬ﺒﺎﺴﺘﻐﻼل اﻝﺒﻴﺎن اﻋط اﻝﻨﺴﺒﺔ اﻝﻨﻬﺎﺌﻴﺔ ﻝﺘﻘدم‬
‫‪τ%‬‬
‫اﻝﺘﻔﺎﻋل ‪τ f‬ﻋﻨد اﻝﺘوازن‪.‬‬
‫‪ .2.4‬اﺴﺘﻨﺘﺞ اﺤدى ﻤﻤﻴزات ﺘﻔﺎﻋل اﻷﺴﺘرة‪.‬‬
‫‪ .5‬ﻝﻴﻜن ‪ x1‬اﻝﺘﻘدم اﻝﻤﺴﺠل ﻝﺤﺎﻝﺔ اﻝﺠﻤﻠﺔ اﻝﻜﻴﻤﻴﺎﺌﻴﺔ‬
‫ﻋﻨد اﻝﻠﺤظﺔ ‪ ، 6 min‬أوﺠد ﻋﻨدﺌذ‪:‬‬
‫‪ .1.5‬اﻝﻨﺴﺒﺔ ‪ τ 1‬ﻝﺘﻘدم اﻝﺘﻔﺎﻋل ﻤﺴﺘﻨﺘﺠﺎ ﻗﻴﻤﺔ‬
‫اﻝﺘﻘدم ‪. x1‬‬
‫‪ .2.5‬اﻝﺘرﻜﻴب اﻝﻤوﻝﻲ ﻝﻠﺠﻤﻠﺔ اﻝﻜﻴﻤﻴﺎﺌﻴﺔاﻝﻤﺘﻔﺎﻋﻠﺔ‪.‬‬

‫الشكل ‪7‬‬
‫‪ .3.5‬ﺤﺠم اﻝﺼود ‪ Vb1‬اﻝﻤﻀﺎف ﻝﻤﻌﺎﻴرة اﻝﺤﻤض‬
‫‪10‬‬ ‫اﻝﻤﺘﺒﻘﻲ‪.‬‬
‫‪ .6‬ﻤن أﺠل ﺘﺤﺴﻴن اﻝﻨﺴﺒﺔ اﻝﻨﻬﺎﺌﻴﺔ ﻝﻠﺘﻘدم ‪τ f‬‬
‫‪0‬‬ ‫‪5‬‬ ‫)‪t (min‬‬ ‫ﻝﻬذا اﻝﺘﻔﺎﻋل ﻨﻘﺘرح اﻝﻘﻴﺎم ﺒﺎﻝﻌﻤﻠﻴﺎت‪:‬‬
‫‪ -‬إﻀﺎﻓﺔ وﺴﻴط ‪ -‬ﺤﻤض اﻝﻜﺒرﻴت اﻝﻤرﻜز ﻤﺜﻼ؛‬
‫‪ -‬ﺘﺤﻘﻴق ﺘﻘطﻴر ﻤﺠ أز ﻝﺤذف اﻝﻤﺎء اﻝﻤﺘﺸﻜل أﺜﻨﺎء اﻝﺘﻔﺎﻋل‪.‬‬
‫ﻤﻌﻠﻼ ﺠواﺒك‪ ،‬ﻤﺎ اﻻﻗﺘراح اﻷﻨﺴب ﻤن ﺒﻴن ﻫذﻴن اﻻﻗﺘراﺤﻴن؟‬

‫اﻨﺘﻬﻰ اﻝﻤوﻀوع اﻝﺜﺎﻨﻲ‬

‫ﺻﻔﺤﺔ ‪ 12‬ﻣﻦ ‪12‬‬


‫دورة‪ :‬ﻣﺎي ‪2023‬‬ ‫اﻹﺟﺎﺑﺔ اﻟﻨﻤﻮذﺟﻴﺔ ﻟﻤﻮﺿﻮع اﻣﺘﺤﺎن ﺷﻬﺎدة اﻟﺒﻜﺎﻟﻮﻳﺎ اﻟﺘﺠﺮﻳﺒﻴﺔ اﻟﻤﻮﺣﺪة‪ .‬ﻗﺴﻨﻄﻴﻨﺔ‪/‬ﻣﻘﺎﻃﻌﺔ‪2‬‬
‫اﻟﺸﻌﺒﺔ‪ :‬رﻳﺎﺿﻴﺎت‪ .‬ﺗﻘﻨﻲ رﻳﺎﺿﻲ‬ ‫اﺧﺘﺒﺎر ﻣﺎدة‪ :‬اﻟﻌﻠﻮم اﻟﻔﻴﺰﻳﺎﺋﻴﺔ‬

‫العالمة‬
‫عناصر اإلجابة الموضوع األول‬
‫مجموع‬ ‫مجزأة‬
‫اﻝﺠزء اﻷول‪ 13) :‬ﻨﻘطﺔ(‬
‫اﻝﺘﻤرﻴن اﻷول‪ 04) :‬ﻨﻘﺎط(‬
‫‪ .1.1‬ﻨص اﻝﻘﺎﻨون اﻝﺜﺎﻝث ﻝﻜﺒﻠر‪:‬‬
‫‪0,25‬‬ ‫‪0,25‬‬ ‫ﻤرﺒﻊ دور ﺤرﻜﺔ ﻜوﻜب ﻴﺘﻨﺎﺴب طردﻴﺎ ﻤﻊ ﻤﻜﻌب اﻝﺒﻌد اﻝﻤﺘوﺴط ﺒﻴن ﻤرﻜزي اﻝﻜوﻜب واﻝﺸﻤس‪.‬‬
‫‪ .2.1‬ﻤدار إﻴرﻴس‪ ،‬ﻴﻘﻊ أﺒﻌد أو أدﻨﻰ ﻤن ﻤدار ﺒﻠوﺘوك‪:‬‬
‫‪TE 2 TP2‬‬ ‫‪TE 2 rE3‬‬
‫‪0,25‬‬ ‫ﺤﺴب ﻗﺎﻨون ﻜﺒﻠر اﻝﺜﺎﻝث‪ 3 = 3 ⇒ 2 = 3 :‬و ﻜون أن‪ TE > TP :‬ﻴﻜون‪:‬‬
‫‪0,25‬‬ ‫‪rE‬‬ ‫‪rP‬‬ ‫‪TP rP‬‬
‫‪rE3‬‬
‫‪0,25‬‬ ‫‪ ، 3 > 1 ⇒ rE > rP‬و ﻤﻨﻪ ﻓﺈن ﻤدار إﻴرﻴس أﺒﻌد ﻤن ﻤدار ﺒﻠوﺘو‪.‬‬
‫‪rP‬‬
‫‪ .1.2‬اﻝﻤرﺠﻊ اﻝذي ﻴﺴﻤﺢ ﺒدراﺴﺔ ﺤرﻜﺔ دﻴﺴﻨوﻤﻴﺎ ﺤول إﻴرﻴس‪:‬‬
‫‪0,25‬‬ ‫‪0,25‬‬
‫ﻫو ﻤرﺠﻊ ﻴﻨطﺒق ﻋﻠﻰ ﻤرﻜز إﻴرﻴس )ﻤرﻜزي إﻴرﻴﺴﻲ(‬
‫‬ ‫‬
‫‪ .2.2‬ﻋﺒﺎرة ﺸﻌﺎع اﻝﺘﺴﺎرع ‪ a‬ﻝﻤرﻜز ﻋطﺎﻝﺔ دﻴﺴﻨوﻤﻴﺎ ﺒدﻻﻝﺔ اﻝﻤﻌطﻴﺎت و ﺸﻌﺎع اﻝوﺤدة ‪U ED‬‬
‫‪0,25‬‬
‫‬
‫‪ F‬‬ ‫ﺒﺘطﺒﻴق اﻝﻘﺎﻨون اﻝﺜﺎﻨﻲ ﻝﻨﻴوﺘن ﻋﻠﻰ اﻝﺠﻤﻠﺔ ﻗﻤر دﻴﺴﺘﻨوﻤﻴﺎ‪:‬‬
‫‪0,25‬‬ ‫‪u‬‬ ‫‬ ‫‬ ‫‬ ‫‬
‫‬ ‫‪∑ Fext = m.a G ⇒ F E / D = mD .a‬‬
‫‪n‬‬
‫‪0,25‬‬ ‫ ‪GM E .mD‬‬ ‫‬ ‫‬ ‫ ‪GM E‬‬
‫‪−‬‬ ‫‪u‬‬ ‫=‬ ‫‪m‬‬ ‫‪D‬‬
‫‪.‬‬‫‪a‬‬ ‫⇒‬ ‫‪a‬‬ ‫=‬ ‫‪−‬‬ ‫‪u‬‬
‫‪rD2‬‬ ‫‪rD2‬‬
‫‪ .3.2‬ﺘﺤدﻴد ﺤﺎﻤل و اﺘﺠﺎﻩ ﺸﻌﺎع اﻝﺘﺴﺎرع‪:‬‬
‫‪0,25‬‬ ‫‬
‫‪0,25‬‬ ‫‪ -‬اﻝﺤﺎﻤل ﻫو ﺤﺎﻤل اﻝﺸﻌﺎع ‪) uDE‬اﻝﻨﺎظم ﻋﻠﻰ اﻝﻤﻤﺎس ﻝﻠﻤدار(‬
‫‪0,25‬‬ ‫‪ -‬اﻝﺠﻬﺔ‪ :‬ﻨﺤو ﻤرﻜز اﻝﻤدار‪.‬‬

‫‪rD3‬‬
‫‪: TD = 2 π‬‬ ‫‪ .4.2‬إﺜﺒﺎت‬
‫‪GM E‬‬
‫‪v2‬‬ ‫‪2 π.rD‬‬
‫‪0,25‬‬ ‫= ‪ a = an‬و ﻤﻨﻪ‪:‬‬ ‫= ‪ ، TD‬و ﻜون أن اﻝﺤرﻜﺔ داﺌرﻴﺔ ﻤﻨﺘظﻤﺔ ﻴﻜون‪:‬‬ ‫ﻝدﻴﻨﺎ‪:‬‬
‫‪rD‬‬ ‫‪v‬‬
‫‪0,25‬‬
‫‪v 2 G. M E‬‬ ‫‪G. M E‬‬
‫‪ ،‬و ﺒﺎﻝﺘﻌوﻴض ﻓﻲ ﻋﺒﺎرة اﻝدور ‪ TD‬ﻨﺠد ﻓﻲ اﻝﻨﻬﺎﻴﺔ‪:‬‬ ‫=‬ ‫⇒‬ ‫‪v‬‬ ‫=‬
‫‪rD‬‬ ‫‪rD2‬‬ ‫‪rD‬‬
‫‪0,25‬‬
‫‪rD3‬‬
‫‪TD = 2π‬‬
‫‪G. M E‬‬

‫ﺻﻔﺤﺔ ‪ 1‬ﻣﻦ ‪21‬‬


‫دورة‪ :‬ﻣﺎي ‪2023‬‬ ‫اﻹﺟﺎﺑﺔ اﻟﻨﻤﻮذﺟﻴﺔ ﻟﻤﻮﺿﻮع اﻣﺘﺤﺎن ﺷﻬﺎدة اﻟﺒﻜﺎﻟﻮﻳﺎ اﻟﺘﺠﺮﻳﺒﻴﺔ اﻟﻤﻮﺣﺪة‪ .‬ﻗﺴﻨﻄﻴﻨﺔ‪/‬ﻣﻘﺎﻃﻌﺔ‪2‬‬
‫اﻟﺸﻌﺒﺔ‪ :‬رﻳﺎﺿﻴﺎت‪ .‬ﺗﻘﻨﻲ رﻳﺎﺿﻲ‬ ‫اﺧﺘﺒﺎر ﻣﺎدة‪ :‬اﻟﻌﻠﻮم اﻟﻔﻴﺰﻳﺎﺋﻴﺔ‬

‫▪ ﺘﺤﻘق ﻗﺎﻨون ﻜﺒﻠر ﻤﺤﻘق أم ﻻ ‪:‬‬


‫‪0,25‬‬
‫‪TD2‬‬ ‫‪4 π2 rD3 TD2‬‬ ‫‪4π2‬‬
‫‪0,25‬‬ ‫ﺜﺎﺒﺘﺔ و ﻤﻨﻪ ﻗﺎﻨون ﻜﺒﻠر‬ ‫اﻝﻨﺴﺒﺔ‬ ‫ﻴﻌﻨﻲ‬ ‫‪،‬‬ ‫‪T‬‬‫‪D‬‬
‫‪2‬‬
‫=‬ ‫⇒‬ ‫=‬ ‫ﻤﻤﺎ ﺴﺒق‪:‬‬
‫‪rD3‬‬ ‫‪G. M E‬‬ ‫‪rD3 G.M E‬‬
‫‪0,25‬‬
‫اﻝﺜﺎﻝث ﻤﺤﻘق‪.‬‬
‫‪ .5.2‬ﻋﺒﺎرة ‪ M E‬ﻜﺘﻠﺔ إﻴرﻴس و ﻗﻴﻤﺘﻬﺎ‪:‬‬
‫‪4 π2 rD3‬‬ ‫‪4 π2 rD3‬‬
‫‪0,25‬‬ ‫= ‪ ، TD2‬و ﻤﻨﻪ‪:‬‬ ‫= ‪⇒ME‬‬ ‫ﻤﻤﺎ ﺴﺒق‪:‬‬
‫‪0,25‬‬ ‫‪G. M E‬‬ ‫‪G.TD2‬‬
‫‪4 π2 .(3,6 × 107 )3‬‬
‫‪0,25‬‬ ‫= ‪ME‬‬ ‫‪−11‬‬
‫‪= 1,67 × 1022 kg‬‬
‫)‪6,67 × 10 (1,5 × 24 × 3600‬‬ ‫‪2‬‬

‫‪ME‬‬
‫‪:‬‬ ‫‪ .6.2‬ﺤﺴﺎب اﻝﻨﺴﺒﺔ ﺒﻴن ﻜﺘﻠﺘﻲ إﻴرﻴس و ﺒﻠوﺘو‬
‫‪MP‬‬
‫‪M E 1,67 × 1022‬‬
‫‪0,25‬‬ ‫‪ ،‬و ﻤﻨﻪ ﻜﺘﻠﺔ إﻴرﻴس أﻜﺒر ﺒﻘﻠﻴل ﻤن ﻜﺘﻠﺔ ﺒﻠوﺘو‪ ،‬ﻓﺈذا‬ ‫=‬ ‫ﻝدﻴﻨﺎ‪= 1, 27 :‬‬
‫‪0,25‬‬ ‫‪M P 1,31 × 1022‬‬
‫‪0,25‬‬ ‫ﻜﺎن إرﻴس ﻝم ﻴﺼﻨف ﻜوﻜﺒﺎ ﻓﺈن ﺒﻠوﺘو ﻻ ﻴﻤﻜن ﺘﺼﻨﻴﻔﻪ ﻜوﻜﺒﺎ‪.‬‬
‫‪E‬‬ ‫اﻝﺘﻤرﻴن اﻝﺜﺎﻨﻲ‪ 06) :‬ﻨﻘﺎط(‬
‫‪0,25‬‬ ‫‪I‬‬ ‫‪-‬‬
‫‪+‬‬ ‫اﻝﺠزء اﻷول‪:‬‬
‫‪0,25‬‬ ‫‪V‬‬ ‫‪ .1‬ﻤﺨطط اﻝدارة اﻝﻜﻬرﺒﺎﺌﻴﺔ ‪:‬‬
‫‪0,10‬‬
‫‪0,25‬‬

‫‪0.25‬‬ ‫‪uR‬‬ ‫‪uC‬‬

‫‪ .1.2‬اﻝظﺎﻫرة اﻝﻤدروﺴﺔ ﻤﻊ ﺘﻔﺴﻴرﻫﺎ ﻤﺠﻬرﻴﺎ‪:‬‬


‫ﺸﺤن ﻤﻜﺜﻔﺔ و ﻋﻠﻰ اﻝﻤﺴﺘوي اﻝﻤﺠﻬري ﺘﻨﺘﻘل اﻹﻝﻜﺘروﻨﺎت ﻤن اﻝﻠﺒوس اﻝﻤرﺘﺒط ﺒﺎﻝﻘطب اﻝﻤوﺠب‬
‫‪0,25‬‬ ‫‪0,25‬‬ ‫ﻝﻠﻤوﻝد‪ ،‬إﻝﻰ اﻝﻘطب اﻝﻤرﺘﺒط ﺒﺎﻝﻘطب اﻝﺴﺎﻝب ﻝﻠﻤوﻝد‪ ،‬و ﺒﺴﺒب اﻝﻌﺎزل ﺘﺘراﻜم اﻻﻝﻜﺘروﻨﺎت ﻓﻲ‬
‫اﻝﺜﺎﻨﻲ اﻝذي ﻴﺸﺤن ﺴﻠﺒﺎ ﻓﻲ ﺤﻴن ﻴﺸﺤن اﻝﻠﺒوس اﻵﺨر إﻴﺠﺎﺒﺎ‪.‬‬
‫‪ .2.2‬اﻝﻤﻌﺎدﻝﺔ اﻝﺘﻔﺎﻀﻠﻴﺔ اﻝﺘﻲ ﻴﺤﻘﻘﻬﺎ اﻝﺘوﺘر اﻝﻜﻬرﺒﺎﺌﻲ ﺒﻴن طرﻓﻲ اﻝﻤﻜﺜﻔﺔ ) ‪: uC (t‬‬
‫‪0,25‬‬ ‫‪du‬‬ ‫‪1‬‬ ‫‪E‬‬
‫‪uR + uC = E ⇒ C +‬‬ ‫= ‪uC‬‬ ‫ﺤﺴب ﻗﺎﻨون ﺠﻤﻊ اﻝﺘوﺘرات‪:‬‬
‫‪0,25‬‬ ‫‪dt RC‬‬ ‫‪RC‬‬
‫‪0,75‬‬ ‫‪ .3.2‬اﻝﻌﺒﺎرة اﻝﻠﺤظﻴﺔ ﻝﻠﺤل اﻝﺘﺤﻠﻴﻠﻲ ﻝﻠﻤﻌﺎدﻝﺔ اﻝﺘﻔﺎﻀﻠﻴﺔ‪:‬‬
‫اﻝﺘوﺘر ‪ uC‬ﻴزداد أﺜﻨﺎء اﻝﺸﺤن و ﻋﻠﻴﻪ اﻝﻤﻌﺎدﻝﺔ اﻝﻤواﻓﻘﺔ ﻝﺤل اﻝﻤﻌﺎدﻝﺔ‬
‫‪t‬‬
‫‪0,25‬‬ ‫‪−‬‬
‫اﻝﺘﻔﺎﻀﻠﻴﺔ‪uC = E (1 − e ) :‬‬
‫‪τ‬‬

‫ﺻﻔﺤﺔ ‪ 2‬ﻣﻦ ‪21‬‬


‫دورة‪ :‬ﻣﺎي ‪2023‬‬ ‫اﻹﺟﺎﺑﺔ اﻟﻨﻤﻮذﺟﻴﺔ ﻟﻤﻮﺿﻮع اﻣﺘﺤﺎن ﺷﻬﺎدة اﻟﺒﻜﺎﻟﻮﻳﺎ اﻟﺘﺠﺮﻳﺒﻴﺔ اﻟﻤﻮﺣﺪة‪ .‬ﻗﺴﻨﻄﻴﻨﺔ‪/‬ﻣﻘﺎﻃﻌﺔ‪2‬‬
‫اﻟﺸﻌﺒﺔ‪ :‬رﻳﺎﺿﻴﺎت‪ .‬ﺗﻘﻨﻲ رﻳﺎﺿﻲ‬ ‫اﺧﺘﺒﺎر ﻣﺎدة‪ :‬اﻟﻌﻠﻮم اﻟﻔﻴﺰﻳﺎﺋﻴﺔ‬

‫‪ .4.2‬اﻝﻌﺒﺎرة اﻝﻠﺤظﻴﺔ ﻝﻠﺘوﺘر اﻝﻜﻬرﺒﺎﺌﻲ ) ‪ uR (t‬ﺒﻴن طرﻓﻲ اﻝﻨﺎﻗل اﻷوﻤﻲ‪:‬‬


‫‪0,25‬‬ ‫‪0,25‬‬ ‫ﺤﺴب ﻗﺎﻨون ﺠﻤﻊ اﻝﺘوﺘرات‪:‬‬
‫‪uR + uC = E ⇒ uR = E − uC = E − E (1 − e − t / τ ) ⇒ uR = Ee − t / τ‬‬
‫‪ .2‬اﻝﻤﻨﺤﻨﻰ اﻝﺒﻴﺎﻨﻲ ) ‪: uC = f (t‬‬
‫)‪uC (V‬‬

‫‪0,50‬‬ ‫‪0,50‬‬

‫‪3‬‬

‫‪20‬‬ ‫)‪t( s‬‬


‫‪ .3‬ﻗﻴﻤﺔ ‪: E‬‬
‫‪0,25‬‬ ‫ﻤن اﻝﺒﻴﺎن‪uc ( ∞ ) = E = 12V :‬‬

‫‪0,25‬‬ ‫ﻗﻴﻤﺔ ‪: τ‬‬


‫‪0,25‬‬ ‫ﻝدﻴﻨﺎ‪ ، t = τ ⇒ uC = 0,63E = 0,63 × 12 = 7,56V :‬ﺒﺎﻹﺴﻘﺎط ﻤﻊ أﺨذ ﺴﻠم اﻝرﺴم‬
‫ﺒﻌﻴن اﻻﻋﺘﺒﺎر‪τ = 20 s :‬‬
‫‪ .4‬ﺴﻌﺔ اﻝﻤﻜﺜﻔﺔ ‪: C‬‬
‫‪0,25‬‬
‫‪0,25‬‬ ‫‪τ‬‬ ‫‪20‬‬
‫‪0,25‬‬ ‫= ‪ ، τ = RC ⇒ C‬و ﻫﻲ ﻤواﻓﻘﺔ ﻝﻠﻘﻴﻤﺔ اﻝﻤﺴﺠﻠﺔ‪.‬‬ ‫=‬ ‫ﻝدﻴﻨﺎ‪= 10−3 F :‬‬
‫‪R 20 ×10‬‬ ‫‪3‬‬

‫‪ .1.5‬اﻝﻌﺒﺎرة اﻝﻠﺤظﻴﺔ ﻝﻠطﺎﻗﺔ اﻝﻤﺨزﻨﺔ ) ‪ EC (t‬ﻓﻲ اﻝﻤﻜﺜﻔﺔ‪:‬‬


‫‪1‬‬
‫ﻝدﻴﻨﺎ‪ ، EC = CuC 2 :‬و ﺤﻴث‪ ، uC = E (1 − e − t / τ ) :‬ﻴﻜون‪:‬‬
‫‪0,25‬‬ ‫‪0,25‬‬ ‫‪2‬‬
‫‪1‬‬
‫‪EC = CE 2 (1 − e − t / τ )2‬‬
‫‪2‬‬
‫‪ .2.5‬ﻗﻴﻤﺔ طﺎﻗﺔ اﻝﻤﻜﺜﻔﺔ ﻋﻨد اﻝﻠﺤظﺘﻴن‪: t2 = 80 s ، t1 = 10 s :‬‬
‫‪10‬‬
‫‪1‬‬ ‫‪−‬‬
‫‪0,25‬‬ ‫‪▪ t = 10s ⇒ EC = × 10−3 (12)2 (1 − e 20 ) 2 = 0,011 J = 11 mJ‬‬
‫‪0,25‬‬ ‫‪2‬‬
‫‪80‬‬
‫‪1‬‬ ‫‪−‬‬
‫‪−3‬‬
‫‪0,25‬‬ ‫‪▪ t = 80 s ⇒ EC = × 10 (12) (1 − e 20 )2 = 0,069 J = 69 mJ‬‬
‫‪2‬‬
‫‪2‬‬

‫ﺻﻔﺤﺔ ‪ 3‬ﻣﻦ ‪21‬‬


‫دورة‪ :‬ﻣﺎي ‪2023‬‬ ‫اﻹﺟﺎﺑﺔ اﻟﻨﻤﻮذﺟﻴﺔ ﻟﻤﻮﺿﻮع اﻣﺘﺤﺎن ﺷﻬﺎدة اﻟﺒﻜﺎﻟﻮﻳﺎ اﻟﺘﺠﺮﻳﺒﻴﺔ اﻟﻤﻮﺣﺪة‪ .‬ﻗﺴﻨﻄﻴﻨﺔ‪/‬ﻣﻘﺎﻃﻌﺔ‪2‬‬
‫اﻟﺸﻌﺒﺔ‪ :‬رﻳﺎﺿﻴﺎت‪ .‬ﺗﻘﻨﻲ رﻳﺎﺿﻲ‬ ‫اﺧﺘﺒﺎر ﻣﺎدة‪ :‬اﻟﻌﻠﻮم اﻟﻔﻴﺰﻳﺎﺋﻴﺔ‬

‫اﻝﺠزء اﻝﺜﺎﻨﻲ‪:‬‬
‫‪0,25‬‬ ‫) ‪uC (t‬‬
‫ﺒدﻻﻝﺔ ‪ τ‬و ‪: t‬‬ ‫‪ .1‬ﻋﺒﺎرة اﻝﻨﺴﺒﺔ‬
‫‪0,50‬‬ ‫) ‪uR ( t‬‬
‫‪uC‬‬
‫‪0,25‬‬ ‫⇒ ‪uC = E (1 − e − t / τ ) , uR = Ee − t / τ‬‬ ‫‪= et / τ − 1‬‬
‫‪uR‬‬
‫‪ .2‬اﻝﻤﻨﺤﻨﻰ اﻝﺼﺤﻴﺢ‪:‬‬
‫‪0,25‬‬ ‫‪0,25‬‬ ‫‪uC‬‬
‫⇒ ‪ ، t = 0‬و ﻤﻨﻪ اﻝﻤﻨﺤﻨﻰ اﻝﺼﺤﻴﺢ ﻫو )‪. (2‬‬ ‫ﻝدﻴﻨﺎ‪= e0 − 1 = 0 :‬‬
‫‪uR‬‬
‫‪ .1.3‬ﻗﻴﻤﺔ ﺜﺎﺒت اﻝزﻤن ‪: τ‬‬
‫‪0,50‬‬ ‫‪0,25‬‬ ‫‪u‬‬
‫ﻝدﻴﻨﺎ‪ ، t = τ ⇒ C = e1 − 1 = 1,71 :‬و ﺒﺎﻹﺴﻘﺎط ﻨﺠد‪τ = 20 s :‬‬
‫‪uR‬‬
‫‪ .2.3‬اﻝﺘﺄﻜد ﻤن ﺴﻌﺔ اﻝﻤﻜﺜﻔﺔ ‪: C‬‬
‫‪0,25‬‬ ‫‪τ‬‬ ‫‪20‬‬
‫‪0,50‬‬ ‫= ‪ ، τ = RC ⇒ C‬و ﻫﻲ ﺘواﻓق اﻝﻨﺘﻴﺠﺔ‬ ‫=‬ ‫ﻝدﻴﻨﺎ‪= 10−3 F = 1000 µF :‬‬
‫‪0,25‬‬ ‫‪R 20 ×10‬‬ ‫‪3‬‬

‫اﻝﺴﺎﺒﻘﺔ‪.‬‬
‫اﻝﺘﻤرﻴن اﻝﺜﺎﻝث‪ 04) :‬ﻨﻘﺎط(‬
‫‪ . I‬اﻝﻨﺸﺎط اﻹﺸﻌﺎﻋﻲ ﻝﻠﻜرﺒون‬
‫‪0,25‬‬ ‫‪: 14‬‬
‫‪6 C ،‬‬
‫‪12‬‬
‫‪6‬‬ ‫‪ .1‬ﺘرﻜﻴب اﻝﻨواﺘﻴن اﻝﺘﺎﻝﻴﺘﻴن ‪C :‬‬
‫‪0,25‬‬ ‫‪ A = 12‬‬
‫‪⇒‬‬
‫‪ ،‬و ﻤﻨﻪ ﺘرﻜﻴب اﻝﻨواﺘﻴن ﻋﻠﻰ اﻝﻨﺤو اﻝﺘﺎﻝﻲ‪:‬‬ ‫‪12‬‬
‫‪6 C‬‬‫ﻝدﻴﻨﺎ ‪:‬‬
‫‪0,25‬‬ ‫‪Z = 6‬‬
‫‪ -‬ﻋدد اﻝﺒروﺘوﻨﺎت‪ ، Z = 6 :‬ﻋدد اﻝﻨﺘروﻨﺎت‪N = A − Z = 12 − 6 = 6 :‬‬
‫‪ .2‬ﺘﺤدﻴد اﻝﻨواة اﻝﻤﺸﻌﺔ و اﻝﻨواة اﻝﻤﺴﺘﻘرة ﻤﻊ اﻝﺘﻌﻠﻴل وﻨﻤط اﻝﺘﻔﻜك ﻝﻠﻨواة اﻝﻤﺸﻌﺔ‪:‬‬
‫‪0,25‬‬ ‫‪ -‬ﻓﻲ اﻝﻤﺠﺎل )‪ ، ( Z = 0, Z = 20‬ﺘﻘﻊ اﻷﻨوﻴﺔ اﻝﻤﺴﺘﻘرة ﻓﻲ اﻝﻤﺨطط ) ‪ ( N − Z‬ﻋﻠﻰ‬
‫اﻝﻤﻨﺼف ‪ ، N = Z‬ﻤﺎﻋدا ذﻝك ﻓﻬﻲ أﻨوﻴﺔ ﻏﻴر ﻤﺴﺘﻘرة )ﻤﺸﻌﺔ(‪.‬‬
‫‪ ، Z = 6; N = 6 ⇒ N = Z : 12‬و ﻋﻠﻴﻪ ﻫﻲ ﻨواة ﻤﺴﺘﻘرة‪.‬‬
‫‪ -‬ﺒﺎﻝﻨﺴﺒﺔ ﻝﻨواة اﻝﻜرﺒون ‪6 C‬‬

‫‪0,25‬‬ ‫‪0,25‬‬ ‫‪ ، Z = 6; N = 8 ⇒ N > Z : 14‬اﻝﻨواة ﻝﻬﺎ ﻓﺎﺌض ﻓﻲ اﻝﻨﺘروﻨﺎت و‬


‫‪ -‬ﺒﺎﻝﻨﺴﺒﺔ ﻝﻨواة اﻝﻜرﺒون ‪6 C‬‬

‫ﺒﺎﻝﺘﺎﻝﻲ ﺘﺘﻔﻜك وﻓق اﻝﻨﻤط ‪. β −‬‬


‫‪14‬‬
‫‪6 C‬‬ ‫‪⇒ ZA X +‬‬ ‫‪0‬‬
‫‪−1e‬‬ ‫‪ .3‬ﻤﻌﺎدﻝﺔ اﻝﺘﻔﻜك اﻝﻨووي اﻝﺤﺎدث‪:‬‬
‫‪0,25‬‬ ‫ﺤﺴب ﻗﺎﻨوﻨﻲ اﻻﻨﺤﻔﺎظ‪ ، 6 = Z − 1 ⇒ Z = 7 ، 14 = A + 0 ⇒ A = 14 :‬و ﻤﻨﻪ‪:‬‬
‫‪14‬‬
‫‪6 C‬‬ ‫⇒‬ ‫‪14‬‬
‫‪7 N‬‬ ‫‪+‬‬ ‫‪0‬‬
‫‪−1e‬‬

‫ﺻﻔﺤﺔ ‪ 4‬ﻣﻦ ‪21‬‬


‫دورة‪ :‬ﻣﺎي ‪2023‬‬ ‫اﻹﺟﺎﺑﺔ اﻟﻨﻤﻮذﺟﻴﺔ ﻟﻤﻮﺿﻮع اﻣﺘﺤﺎن ﺷﻬﺎدة اﻟﺒﻜﺎﻟﻮﻳﺎ اﻟﺘﺠﺮﻳﺒﻴﺔ اﻟﻤﻮﺣﺪة‪ .‬ﻗﺴﻨﻄﻴﻨﺔ‪/‬ﻣﻘﺎﻃﻌﺔ‪2‬‬
‫اﻟﺸﻌﺒﺔ‪ :‬رﻳﺎﺿﻴﺎت‪ .‬ﺗﻘﻨﻲ رﻳﺎﺿﻲ‬ ‫اﺧﺘﺒﺎر ﻣﺎدة‪ :‬اﻟﻌﻠﻮم اﻟﻔﻴﺰﻳﺎﺋﻴﺔ‬

‫‪ .1.4‬إﻴﺠﺎد اﻝﻤﻌﺎدﻝﺔ اﻝﺘﻔﺎﻀﻠﻴﺔ ﺒدﻻﻝﺔ ﻋدد اﻷﻨوﻴﺔ اﻝﻤﺘﺒﻘﻴﺔ ) ‪: N (t‬‬


‫) ‪dN (t‬‬ ‫) ‪dN (t‬‬
‫‪0,25‬‬ ‫‪0,25‬‬ ‫‪ ، −‬إذن‪:‬‬ ‫‪ ، A(t ) = λN (t ) ، A(t ) = −‬و ﻤﻨﻪ‪= λN (t ) :‬‬ ‫ﻝدﻴﻨﺎ‪:‬‬
‫‪dt‬‬ ‫‪dt‬‬
‫) ‪dN (t‬‬
‫‪+ λN ( t ) = 0‬‬
‫‪dt‬‬
‫‪ .2.4‬إﺜﺒﺎت أن اﻝﻌﺒﺎرة ‪ N (t ) = N 0 e −λt :‬ﻫﻲ ﺤل ﻝﻠﻤﻌﺎدﻝﺔ اﻝﺘﻔﺎﻀﻠﻴﺔ‪:‬‬
‫) ‪dN (t‬‬
‫⇒ ‪ ، N (t ) = N 0e −λt‬ﺒﺎﻝﺘﻌوﻴض ﻓﻲ اﻝﻤﻌﺎدﻝﺔ اﻝﺘﻔﺎﻀﻠﻴﺔ ﻨﺠد‪:‬‬ ‫ﻝدﻴﻨﺎ‪= −λN 0e −λt :‬‬
‫‪0,25‬‬ ‫‪0,25‬‬ ‫‪dt‬‬
‫‪−λt‬‬ ‫‪−λt‬‬
‫‪−λN 0 e + λN 0 e = 0 ⇒ 0 = 0‬‬
‫و ﻤﻨﻪ اﻝﺤل اﻝﻤﻌطﻰ ﻫو ﻓﻌﻼ ﺤل ﻝﻠﻤﻌﺎدﻝﺔ اﻝﺘﻔﺎﻀﻠﻴﺔ‪.‬‬
‫‪ .3.4‬إﺜﺒﺎت أن ﻋدد أﻓوﻗﺎدرو ﻴﻌطﻰ ﺒﺎﻝﻘﻴﻤﺔ اﻝﺘﻘرﻴﺒﻴﺔ ‪: N A = 6,02 × 1023 mol −1‬‬
‫‪0,25‬‬ ‫‪ln 2‬‬ ‫‪A.t‬‬
‫= ‪ ، A‬ﻤن ﺠﻬﺔ أﺨرى‪:‬‬ ‫ﻝدﻴﻨﺎ‪ ، A = λN :‬وﻤﻨﻪ‪N ⇒ N = 1/2 :‬‬
‫‪t1/2‬‬ ‫‪ln 2‬‬
‫‪N Am At1/2‬‬ ‫‪MAt1/2‬‬ ‫‪N‬‬ ‫‪m‬‬ ‫‪N .m‬‬
‫=‬ ‫= ‪⇒ NA‬‬ ‫‪ ،‬إذن‪:‬‬ ‫‪= ⇒N = A‬‬
‫‪0,75‬‬ ‫‪0,25‬‬ ‫‪M‬‬ ‫‪M‬‬ ‫‪ln 2.m‬‬ ‫‪NA M‬‬ ‫‪M‬‬
‫‪1‬‬
‫ﻝدﻴﻨﺎ‪ ، A = 9,89 × 109 min −1 = 9,89 × 109. = 1,65 × 108 Bq :‬و ﻤﻨﻪ‪:‬‬
‫‪60‬‬
‫‪0,25‬‬ ‫‪14 × 1,65 × 10 × 5730 × 365, 25 × 24 × 3600‬‬
‫‪8‬‬
‫= ‪NA‬‬ ‫‪−3‬‬
‫‪= 6,02 × 1023 mol −1‬‬
‫‪ln 2 × 10‬‬
‫‪0,25‬‬ ‫‪0,25‬‬ ‫‪ .4‬اﻝوﺴﻴﻠﺔ اﻝﻤﺴﺘﻌﻤﻠﺔ ﻓﻲ ﻗﻴﺎس اﻝﻨﺸﺎط اﻻﺸﻌﺎﻋﻲ ‪ A‬ﻫﻲ اﺴﺘﺨدام ﻋداد ﺠﻴﺠر‪.‬‬
‫‪ . II‬ﺘﺄرﻴﺦ زﻝزال ﺴﺎن اﻨدرﻴﺎس ﺒﻜﺎﻝﻴﻔورﻨﻴﺎ‬
‫‪0,25‬‬ ‫‪0,25‬‬ ‫‪ .1‬ﺘﻌرﻴف اﻝﻨﺸﺎط اﻹﺸﻌﺎﻋﻲ ‪ : A‬ﻫو ﻋدد اﻝﺘﻔﻜﻜﺎت ﻓﻲ وﺤدة اﻝزﻤن ) ‪ ، ( s‬وﺤدﺘﻪ ﻓﻲ اﻝﻨظﺎم‬
‫اﻝدوﻝﻲ ‪ :‬اﻝﺒﻜرﻴل )‪( Bq‬‬
‫‪ .2‬ﺘﺤدﻴد ﺴﻨﺔ وﻗوع اﻝزﻝزال اﻝﻤطﺎﺒق ﻝﻠﻌﻴﻨﺔ رﻗم ‪:3‬‬
‫‪A‬‬ ‫‪ A‬‬ ‫‪ln 2‬‬
‫‪ ،‬و ﻤﻨﻪ‪:‬‬ ‫‪= e −λt ⇒ ln   = −‬‬ ‫ﻝدﻴﻨﺎ‪ ، A = A0 e −λt :‬و ﻤﻨﻪ‪t :‬‬
‫‪A0‬‬ ‫‪ A0 ‬‬ ‫‪t1/2‬‬
‫‪0,25‬‬ ‫‪t1/2  A0 ‬‬
‫= ‪ ، t‬إذن‪:‬‬ ‫‪ln  ‬‬
‫‪ln 2  A ‬‬
‫‪0,50‬‬
‫‪5730  0, 255 ‬‬
‫=‪t‬‬ ‫‪ln‬‬ ‫‪= 1108, 480 ans‬‬
‫‪ln 2  0, 223 ‬‬
‫‪0,25‬‬
‫و ﻤﻨﻪ ﺴﻨﺔ وﻗوع اﻝزﻝزال ﻫﻲ‪ ، 1989 − 1108, 48 = 880 :‬أي وﻗﻊ اﻝزﻝزال ﺴﻨﺔ ‪ 880‬ﻤﻴﻼدي‪.‬‬

‫ﺻﻔﺤﺔ ‪ 5‬ﻣﻦ ‪21‬‬


‫دورة‪ :‬ﻣﺎي ‪2023‬‬ ‫اﻹﺟﺎﺑﺔ اﻟﻨﻤﻮذﺟﻴﺔ ﻟﻤﻮﺿﻮع اﻣﺘﺤﺎن ﺷﻬﺎدة اﻟﺒﻜﺎﻟﻮﻳﺎ اﻟﺘﺠﺮﻳﺒﻴﺔ اﻟﻤﻮﺣﺪة‪ .‬ﻗﺴﻨﻄﻴﻨﺔ‪/‬ﻣﻘﺎﻃﻌﺔ‪2‬‬
‫اﻟﺸﻌﺒﺔ‪ :‬رﻳﺎﺿﻴﺎت‪ .‬ﺗﻘﻨﻲ رﻳﺎﺿﻲ‬ ‫اﺧﺘﺒﺎر ﻣﺎدة‪ :‬اﻟﻌﻠﻮم اﻟﻔﻴﺰﻳﺎﺋﻴﺔ‬

‫‪ .3‬ﺘﻌﻴﻴن ﻝﻜل ﻋﻴﻨﺔ اﻝﺴﻨﺔ اﻝﺘﻲ ﺘواﻓﻘﻬﺎ‪:‬‬


‫‪t‬‬ ‫‪A ‬‬
‫‪0,25‬‬ ‫ﻤن اﻝﻌﻼﻗﺔ اﻝﺴﺎﺒﻘﺔ ‪ ، t = 1/2 ln  0 ‬ﻜﻠﻤﺎ ازداد ‪ A‬ﻨﻘص ﻋﻤر اﻝﻌﻴﻨﺔ و ﻜون أن‪:‬‬
‫‪ln 2  A ‬‬
‫‪0,50‬‬ ‫‪ A2 < A1‬ﻴﻜون‪:‬‬
‫‪0,25‬‬ ‫▪ اﻝﻌﻴﻨﺔ رﻗم )‪ (1‬ﺘواﻓق اﻝﺴﻨﺔ ‪. 581‬‬
‫▪ اﻝﻌﻴﻨﺔ رﻗم )‪ (2‬ﺘواﻓق اﻝﺴﻨﺔ ‪. 1247‬‬

‫اﻝﺠزء اﻝﺜﺎﻨﻲ‪ 06) :‬ﻨﻘطﺔ(‬


‫اﻝﺘﻤرﻴن اﻝﺘﺠرﻴﺒﻲ‪ 06) :‬ﻨﻘﺎط(‬
‫اﻝﺠزء اﻻول ‪:‬‬
‫‪ .1‬ﺠدول ﺘﻘدم اﻝﺘﻔﺎﻋل‪:‬‬

‫)‪CcCO3 ( s ) + 2 H 3O + (aq) = Ca +2 ( aq) + CO2 ( g ) + 3H 2O ( ℓ‬‬


‫اﻝﺘﻘدم اﻝﺤﺎﻝﺔ‬ ‫ﻜﻤﻴﺔ اﻝﻤﺎدة ‪mol‬‬

‫‪0,25‬‬ ‫‪0,25‬‬ ‫‪ x = 0 n (CaCO ) = m0 n ( H O + ) = c V‬اﺒﺘداﺌﻴﺔ‬ ‫‪0‬‬ ‫‪0‬‬


‫‪0‬‬ ‫‪3‬‬ ‫‪0‬‬ ‫‪3‬‬ ‫‪a a‬‬
‫‪M‬‬

‫بوفرة‬
‫‪ x‬اﻨﺘﻘﺎﻝﻴﺔ‬ ‫‪n0 (CaCO3 ) − x‬‬ ‫‪n0 ( H 3O + ) − 2 x‬‬ ‫‪x‬‬ ‫‪x‬‬
‫‪ xmax n0 (CaCO3 ) − xmax n0 ( H 3O + ) − 2 xmax‬ﻨﻬﺎﺌﻴﺔ‬ ‫‪x‬‬ ‫‪x‬‬

‫‪ .2‬ﺘﻌرﻴف اﻝﺘﻘدم اﻷﻋظﻤﻲ ﻝﻠﺘﻔﺎﻋل‪:‬‬


‫ﻫو اﻝﺘﻘدم اﻝذي ﻴﺒﻠﻐﻪ اﻝﺘﻔﺎﻋل ﻋﻨدﻤﺎ ﻴﺨﺘﻔﻲ اﻝﻤﺘﻔﺎﻋل اﻝﻤﺤد و ﻫو ﻗﻴﻤﺔ ﻨظرﻴﺔ ﻓﻘط‪.‬‬
‫ﻗﻴﻤﺘﻪ‪:‬‬

‫‪0,25‬‬ ‫‪0,25‬‬ ‫اﻝﻤزﻴﺞ اﻻﺒﺘداﺌﻲ ﺴﻴﺘوﻜﻴوﻤﺘري و ﻤن ﺠدول اﻝﺘﻘدم‪:‬‬


‫⇒ ‪n0 ( H 3O + ) = 2 xmax ⇒ caVa − 2 xmax = 0‬‬
‫‪cV‬‬ ‫‪0,6 × 0, 2‬‬
‫= ‪xmax = a a‬‬ ‫‪= 0,06 mol‬‬
‫‪2‬‬ ‫‪2‬‬
‫‪ .3‬ﻗﻴﻤﺔ اﻝﻜﺘﻠﺔ ‪: m0‬‬
‫اﻝﻤزﻴﺞ اﻻﺒﺘداﺌﻲ ﺴﺘوﻜﻴوﻤﺘري و ﻤن ﺠدول اﻝﺘﻘدم‪:‬‬
‫‪0,25‬‬ ‫‪0,25‬‬
‫‪m0‬‬
‫⇒ ‪n0 (CaCO3 ) − xmax = 0‬‬ ‫‪− xmax = 0 ⇒ m0 = M . xmax = 100 × 0,06 = 6 g‬‬
‫‪M‬‬

‫ﺻﻔﺤﺔ ‪ 6‬ﻣﻦ ‪21‬‬


‫دورة‪ :‬ﻣﺎي ‪2023‬‬ ‫اﻹﺟﺎﺑﺔ اﻟﻨﻤﻮذﺟﻴﺔ ﻟﻤﻮﺿﻮع اﻣﺘﺤﺎن ﺷﻬﺎدة اﻟﺒﻜﺎﻟﻮﻳﺎ اﻟﺘﺠﺮﻳﺒﻴﺔ اﻟﻤﻮﺣﺪة‪ .‬ﻗﺴﻨﻄﻴﻨﺔ‪/‬ﻣﻘﺎﻃﻌﺔ‪2‬‬
‫اﻟﺸﻌﺒﺔ‪ :‬رﻳﺎﺿﻴﺎت‪ .‬ﺗﻘﻨﻲ رﻳﺎﺿﻲ‬ ‫اﺧﺘﺒﺎر ﻣﺎدة‪ :‬اﻟﻌﻠﻮم اﻟﻔﻴﺰﻳﺎﺋﻴﺔ‬

‫‪ .1.4‬ﺘﻌرﻴف اﻝﺴرﻋﺔ اﻝﺤﺠﻤﻴﺔ ﻝﻠﺘﻔﺎﻋل‪:‬‬


‫‪1 dx‬‬
‫‪0,25‬‬ ‫= ‪vvol‬‬ ‫ﻫﻲ ﺴرﻋﺔ اﻝﺘﻔﺎﻋل ﻓﻲ وﺤدة اﻝﺤﺠم‪ ،‬ﻴﻌﺒر ﻋﻨﻬﺎ ﺒﺎﻝﻌﻼﻗﺔ‪:‬‬
‫‪V dt‬‬
‫ﻋﺒﺎرة اﻝﺴرﻋﺔ اﻝﺤﺠﻤﻴﺔ ﻝﻠﺘﻔﺎﻋل ﺒدﻻﻝﺔ ‪:Va ، m ، M‬‬
‫‪0,50‬‬ ‫ﻤن ﺠدول اﻝﺘﻘدم ‪:‬‬
‫‪m m0‬‬ ‫‪1‬‬
‫⇒ ‪n (CaCO3 ) = n0 (CaCO3 ) − x‬‬ ‫=‬ ‫)‪− x ⇒ x = ( m0 − m‬‬
‫‪M M‬‬ ‫‪M‬‬
‫ﺒﺎﻝﺘﻌوﻴض ﻓﻲ ﻋﺒﺎرة اﻝﺴرﻋﺔ اﻝﺤﺠﻤﻴﺔ‪:‬‬
‫‪0,25‬‬ ‫‪1 1‬‬ ‫‪1 dm‬‬
‫‪vvol = ( ( m0 − m) ⇒ vvol = −‬‬
‫‪V M‬‬ ‫‪VM dt‬‬
‫‪ .2.4‬ﺤﺴﺎب اﻝﺴرﻋﺔ اﻝﺤﺠﻤﻴﺔ ﻝﻠﺘﻔﺎﻋل ﻋﻨد اﻝﻠﺤظﺔ ‪: t = 0‬‬
‫‪dm‬‬
‫‪0,25‬‬ ‫‪0,25‬‬ ‫‪ ،‬و ﻤﻨﻪ‪:‬‬ ‫ﻤن اﻝﺒﻴﺎن ﻋﻨد اﻝﻠﺤظﺔ ‪= − 4,5 × 5 ×10−3 = − 2, 25 ×10−2 g.s −1 : t = 0‬‬
‫‪dt‬‬
‫‪1‬‬
‫‪vvol = −‬‬ ‫‪( − 2, 25 ×10−2 ) = 1,12 ×10−3 mol.L−1.s −1‬‬
‫‪0, 2 × 100‬‬
‫‪ .1.5‬ﻋﺒﺎرة اﻝﻀﻐط ‪ P‬ﺒدﻻﻝﺔ ‪: Pa ، V ، R ، x ، θ2‬‬
‫‪0,25‬‬ ‫ﻝدﻴﻨﺎ‪ ، P = P(CO2 ) + Pa :‬ﺒﺘطﺒﻴق ﻗﺎﻨون اﻝﻐﺎز اﻝﻤﺜﺎﻝﻲ‪، P( CO2 ) .V( CO2 ) = n( CO2 ) . RT :‬‬
‫‪0,50‬‬ ‫و ﻤن ﺠدول اﻝﺘﻘدم‪ ، n( CO2 ) = x :‬إذن‪:‬‬
‫‪0,25‬‬ ‫‪RTx‬‬ ‫‪RTx‬‬
‫= ) ‪P( CO2 ) .V( CO2 ) = x.RT ⇒ P( CO2‬‬ ‫=‪⇒ P‬‬ ‫‪+ Pa‬‬
‫) ‪V( CO2‬‬ ‫) ‪V( CO2‬‬
‫‪ .2.5‬ﺤﺴﺎب ﻜﻤﻴﺔ ﻏﺎز ‪ CO2‬اﻝﻤﺘﺸﻜﻠﺔ ﻓﻲ ﻨﻬﺎﻴﺔ اﻝﺘﻔﺎﻋل‪:‬‬
‫‪0,25‬‬ ‫ﻋﻨد ﻨﻬﺎﻴﺔ اﻝﺘﻔﺎﻋل‪n f (CO2 ) = xmax = 0,06 mol :‬‬
‫ﻴﻤﻜن أﻴﻀﺎ ﺘطﺒﻴق ﻗﺎﻨون اﻝﻐﺎز اﻝﻤﺜﺎﻝﻲ ﺤﻴث ﻨﺠد‪:‬‬
‫‪0,75‬‬ ‫) ‪( Pf − Pa )V (CO2‬‬ ‫‪(25 − 10)104 × 1 × 10−3‬‬
‫= ) ‪n (CO2‬‬ ‫=‬ ‫‪= 0,06 mol‬‬
‫‪RT‬‬ ‫)‪8,31 × (40 + 273‬‬
‫‪0,25‬‬
‫‪ .3.5‬ﺤﺴﺎب اﻝﺴرﻋﺔ اﻝﺤﺠﻤﻴﺔ ﻝﻠﺘﻔﺎﻋل ﻋﻨد ‪: t = 0‬‬
‫‪RTx‬‬ ‫) ‪( P − Pa )V( CO2‬‬ ‫‪1 dx‬‬
‫‪0,25‬‬ ‫‪ ،‬و ﻤﻨﻪ‪:‬‬ ‫= ‪= P − Pa ⇒ x‬‬ ‫= ‪ ، vvol‬و ﻤﻤﺎ ﺴﺒق‪:‬‬ ‫ﻝدﻴﻨﺎ‪:‬‬
‫‪V‬‬ ‫‪RT‬‬ ‫‪V dt‬‬
‫) ‪1 d ( P − Pa )V( CO2‬‬ ‫‪V( CO2 ) dP‬‬
‫= ‪vvol‬‬ ‫(‬ ‫= ‪) ⇒ vvol‬‬
‫‪V dt‬‬ ‫‪RT‬‬ ‫‪VRT dt‬‬

‫ﺻﻔﺤﺔ ‪ 7‬ﻣﻦ ‪21‬‬


‫دورة‪ :‬ﻣﺎي ‪2023‬‬ ‫اﻹﺟﺎﺑﺔ اﻟﻨﻤﻮذﺟﻴﺔ ﻟﻤﻮﺿﻮع اﻣﺘﺤﺎن ﺷﻬﺎدة اﻟﺒﻜﺎﻟﻮﻳﺎ اﻟﺘﺠﺮﻳﺒﻴﺔ اﻟﻤﻮﺣﺪة‪ .‬ﻗﺴﻨﻄﻴﻨﺔ‪/‬ﻣﻘﺎﻃﻌﺔ‪2‬‬
‫اﻟﺸﻌﺒﺔ‪ :‬رﻳﺎﺿﻴﺎت‪ .‬ﺗﻘﻨﻲ رﻳﺎﺿﻲ‬ ‫اﺧﺘﺒﺎر ﻣﺎدة‪ :‬اﻟﻌﻠﻮم اﻟﻔﻴﺰﻳﺎﺋﻴﺔ‬

‫ﻤن اﻝﺒﻴﺎن ﻋﻨد اﻝﻠﺤظﺔ ‪: t = 0‬‬


‫‪0,25‬‬ ‫‪10−3‬‬ ‫‪(25 − 10) × 104‬‬
‫= ‪vvol‬‬ ‫‪= 9,61×10−3 mol. L−1.s −1‬‬
‫‪0,50‬‬ ‫)‪0, 2 × 8,31× (40 + 273‬‬ ‫)‪(30 − 0‬‬
‫▪ ﺴﺒب اﺨﺘﻼف ﻗﻴﻤﺘﺔ اﻝﺴرﻋﺔ اﻝﺤﺠﻤﻴﺔ ﻤﻊ اﻝﻘﻴﻤﺔ اﻝﻤﺤﺴوﺒﺔ ﺴﺎﺒﻘﺎ‪ ،‬ﻴﻌود إﻝﻰ اﻻﺨﺘﻼف ﻓﻲ‬
‫‪0,25‬‬
‫درﺠﺔ اﻝﺤ اررة ﺤﻴث ﻜﻠﻤﺎ ﻜﺎﻨت درﺠﺔ اﻝﺤ اررة أﻜﺒر ﻜﺎﻨت اﻝﺴرﻋﺔ اﻝﺤﺠﻤﻴﺔ ﻝﻠﺘﻔﺎﻋل أﻜﺒر )درﺠﺔ‬
‫اﻝﺤ اررة ﻋﺎﻤل ﺤرﻜﻲ(‬
‫‪ .4.5‬ﺘﻌرﻴف زﻤن ﻨﺼف اﻝﺘﻔﺎﻋل ‪: t1/2‬‬
‫‪xf‬‬
‫‪0,25‬‬ ‫= ‪t = t1/2 ⇒ x1/2‬‬ ‫ﻫو اﻝزﻤن اﻝﻼزم ﻝﺒﻠوغ ﺘﻘدم اﻝﺘﻔﺎﻋل ﻨﺼف ﻗﻴﻤﺘﻪ اﻝﻨﻬﺎﺌﻴﺔ‪:‬‬
‫‪2‬‬
‫‪0,50‬‬ ‫ﻗﻴﻤﺔ ‪: t1/2‬‬
‫‪(1000 + 2500) × 102‬‬ ‫‪P + Pf‬‬
‫‪0,25‬‬ ‫= ‪، P1/2‬‬ ‫‪ ، P1/2 = a‬و ﻤﻨﻪ‪= 1750 hPa :‬‬ ‫ﺒﻴﺎﻨﻴﺎ‪:‬‬
‫‪2‬‬ ‫‪2‬‬
‫ﺒﺎﻹﺴﻘﺎط‪t1/2 = 180 s :‬‬
‫اﻝﺠزء اﻝﺜﺎﻨﻲ ‪:‬‬
‫‪ .1‬اﻝرﻤز اﻻﺼطﻼﺤﻲ ﻝﻠﻌﻤود اﻝﻤدروس‪:‬‬
‫‪0,25‬‬ ‫‪0,25‬‬
‫ﺤﺴب اﻝﻤﻌﺎدﻝﺔ ‪ Al‬ﺘﺄﻜﺴد )ﻗطب ﺴﺎﻝب( و ‪ Cu 2 +‬أرﺠﻊ )ﻗطب ﻤوﺠب( و ﻋﻠﻴﻪ‪:‬‬
‫) ‪( −) Al 3+ / Al / / Cu 2 + / Cu ( +‬‬
‫‪ .2‬ﺠدول ﻝﺘﻘدم اﻝﺘﻔﺎﻋل‪:‬‬

‫) ‪2 Al ( s‬‬ ‫‪+‬‬ ‫) ‪3Cu 2 + ( aq‬‬ ‫=‬ ‫‪2 Al 3+ ( aq) +‬‬ ‫) ‪3Cu ( s‬‬
‫اﻝﺘﻘدم اﻝﺤﺎﻝﺔ‬ ‫ﻜﻤﻴﺔ اﻝﻤﺎدة ‪mol‬‬
‫‪0,25‬‬ ‫‪0,25‬‬
‫‪ x = 0‬اﺒﺘداﺌﻴﺔ‬ ‫) ‪n0 ( Al‬‬ ‫) ‪n0 (Cu 2 +‬‬ ‫) ‪n0 ( Al 3+‬‬ ‫) ‪n0 (Cu‬‬
‫اﻨﺘﻘﺎﻝﻴﺔ‬ ‫‪x‬‬ ‫‪n0 ( Al ) − 2 x n0 (Cu 2 + ) − 3x n0 ( Al 3+ ) + 2 x n0 (Cu ) − 3x‬‬
‫‪ xmax n0 ( Al ) − 2 x f n0 (Cu 2 + ) − 3x f n0 ( Al 3+ ) + 2 x f n0 (Cu ) − 3x f‬ﻨﻬﺎﺌﻴﺔ‬

‫‪ .3‬ﻋﺒﺎرة ‪ Cu 2 + ( aq) ‬ﺒدﻻﻝﺔ اﻝزﻤن ‪ t‬و ‪ c0‬و ‪ I‬و ‪ V‬و ‪: F‬‬
‫‪0,25‬‬ ‫‪ 2+ ‬‬ ‫‪2+‬‬ ‫‪2+‬‬
‫ﻤن ﺠدول اﻝﺘﻘدم‪ ، n (Cu ) = n0 (Cu ) − 3x :‬و ﻤﻨﻪ‪Cu V = c0V − 3x :‬‬
‫‪0,50‬‬ ‫‪Tt Tt‬‬
‫= ‪Q = It = z. x.F ⇒ x‬‬ ‫=‬ ‫ﻤن ﺠﻬﺔ أﺨرى‪:‬‬
‫‪zF 6 F‬‬
‫‪0,25‬‬ ‫‪It‬‬ ‫‪It‬‬
‫‪Cu 2 + V = c0V − 3‬‬ ‫‪⇒ Cu 2+  = c0 −‬‬ ‫إذن‪:‬‬
‫‪‬‬ ‫‪‬‬ ‫‪6F‬‬ ‫‪2 FV‬‬

‫ﺻﻔﺤﺔ ‪ 8‬ﻣﻦ ‪21‬‬


‫دورة‪ :‬ﻣﺎي ‪2023‬‬ ‫اﻹﺟﺎﺑﺔ اﻟﻨﻤﻮذﺟﻴﺔ ﻟﻤﻮﺿﻮع اﻣﺘﺤﺎن ﺷﻬﺎدة اﻟﺒﻜﺎﻟﻮﻳﺎ اﻟﺘﺠﺮﻳﺒﻴﺔ اﻟﻤﻮﺣﺪة‪ .‬ﻗﺴﻨﻄﻴﻨﺔ‪/‬ﻣﻘﺎﻃﻌﺔ‪2‬‬
‫اﻟﺸﻌﺒﺔ‪ :‬رﻳﺎﺿﻴﺎت‪ .‬ﺗﻘﻨﻲ رﻳﺎﺿﻲ‬ ‫اﺧﺘﺒﺎر ﻣﺎدة‪ :‬اﻟﻌﻠﻮم اﻟﻔﻴﺰﻳﺎﺋﻴﺔ‬

‫‪ .4‬ﺸدة اﻝﺘﻴﺎر اﻝﻜﻬرﺒﺎﺌﻲ ‪ I‬ﻓﻲ اﻝدارة‪:‬‬


‫‪0,25‬‬ ‫‪ -‬ﺒﻴﺎﻨﻴﺎ اﻝﻤﻨﺤﻨﻰ ) ‪ Cu 2 +  = f (t‬ﻫو ﻤﺴﺘﻘﻴم ﻻ ﻴﺸﻤل اﻝﻤﺒدأ ﻤﻌﺎدﻝﺘﻪ ﻤن اﻝﺸﻜل‪:‬‬
‫‪ ، Cu 2 +  = at + b‬ﺤﻴث ‪:‬‬
‫‪ ، b = 0,02 ، a = −2 × 10−5‬و ﻤﻨﻪ‪. Cu 2 +  = −2 × 10−5 t + 0,02 :‬‬
‫‪0,75‬‬
‫‪0,25‬‬ ‫‪It‬‬
‫‪Cu 2 +  = −‬‬ ‫‪ -‬ﻨظرﻴﺎ و ﻤﻤﺎ ﺴﺒق‪+ c0 :‬‬
‫‪‬‬ ‫‪‬‬ ‫‪2 FV‬‬
‫‪I‬‬
‫‪−‬‬ ‫‪ -‬ﺒﺎﻝﻤطﺎﺒﻘﺔ‪= a ⇒ I = −2 FVa = 0,193 A :‬‬
‫‪2 FV‬‬
‫‪0,25‬‬ ‫اﻝﺘرﻜﻴز اﻝﻤوﻝﻲ اﻻﺒﺘداﺌﻲ ‪: c0‬‬
‫ﺒﺎﻝﻤطﺎﺒﻘﺔ أﻴﻀﺎ‪c0 = b = 0,02mol. L−1 :‬‬
‫‪ .5‬ﻋﺒﺎرة اﻝﺘﻐﻴر ﻓﻲ اﻝﻜﺘﻠﺔ ‪ ∆m‬ﺨﻼل ﻤدة اﺸﺘﻐﺎل اﻝﻌﻤود ﺒدﻻﻝﺔ ‪ t‬و ‪ c0‬و ‪ I‬و ‪ V‬و ‪F‬‬
‫) اﻝﻔﺎرداي (‪:‬‬
‫‪0,25‬‬ ‫ﻤن ﺠدول اﻝﺘﻘدم ﻜﻤﻴﺔ ﻤﺎدة اﻷﻝﻤﻨﻴوم اﻝﻤﺘﻔﺎﻋﻠﺔ ﻫﻲ‪ n ( Al ) = 2 x :‬و ﻜون أن‪:‬‬
‫‪∆m‬‬ ‫‪m‬‬
‫‪0,50‬‬ ‫= ) ‪ n ( Al‬ﻴﺼﺒﺢ‪= 2 x :‬‬
‫‪M‬‬ ‫‪M‬‬
‫‪∆m 2 It‬‬ ‫‪MIt‬‬ ‫‪It‬‬
‫‪.‬‬ ‫=‬ ‫= ‪⇒ ∆m‬‬ ‫= ‪ ، x‬ﻴﺼﺒﺢ‪:‬‬ ‫ﻤﻤﺎ ﺴﺒق وﺠدﻨﺎ‪:‬‬
‫‪0,25‬‬ ‫‪M 6F‬‬ ‫‪3F‬‬ ‫‪6F‬‬
‫ﻗﻴﻤﺘﻪ اﻝﺘﻐﻴر ﻓﻲ اﻝﻜﺘﻠﺔ ‪: ∆m‬‬
‫‪27 × 0,193 × 1000‬‬
‫= ‪∆m‬‬ ‫ﻤن ‪ ∆m‬اﻝﺴﺎﺒﻘﺔ‪= 0,018 g :‬‬
‫‪3 × 96500‬‬

‫ﺻﻔﺤﺔ ‪ 9‬ﻣﻦ ‪21‬‬


‫دورة‪ :‬ﻣﺎي ‪2023‬‬ ‫اﻹﺟﺎﺑﺔ اﻟﻨﻤﻮذﺟﻴﺔ ﻟﻤﻮﺿﻮع اﻣﺘﺤﺎن ﺷﻬﺎدة اﻟﺒﻜﺎﻟﻮﻳﺎ اﻟﺘﺠﺮﻳﺒﻴﺔ اﻟﻤﻮﺣﺪة‪ .‬ﻗﺴﻨﻄﻴﻨﺔ‪/‬ﻣﻘﺎﻃﻌﺔ‪2‬‬
‫اﻟﺸﻌﺒﺔ‪ :‬رﻳﺎﺿﻴﺎت‪ .‬ﺗﻘﻨﻲ رﻳﺎﺿﻲ‬ ‫اﺧﺘﺒﺎر ﻣﺎدة‪ :‬اﻟﻌﻠﻮم اﻟﻔﻴﺰﻳﺎﺋﻴﺔ‬

‫العالمة‬
‫عناصر اإلجابة الموضوع الثاني‬
‫مجموع‬ ‫مجزأة‬
‫اﻝﺠزء اﻷول‪ 13) :‬ﻨﻘطﺔ(‬
‫اﻝﺘﻤرﻴن اﻷول‪ 06) :‬ﻨﻘﺎط(‬
‫‪ -I‬دراﺴﺔ اﻝﺤرﻜﺔ ﻋﻠﻰ اﻝﻤﺴﺘوي اﻷﻓﻘﻲ ‪ :‬ﻨﻬﻤل اﻻﺤﺘﻜﺎﻜﺎت و ﺘﺄﺜﻴر اﻝﻬواء‪.‬‬
‫‪ .1‬اﻝﻤرﺠﻊ اﻝﻤﻨﺎﺴب ﻝدراﺴﺔ ﻫذﻩ اﻝﺤرﻜﺔ‪ :‬ﻫو اﻝﻤرﺠﻊ اﻝﺴطﺤﻲ اﻷرﻀﻲ‪.‬‬
‫‪0,25‬‬ ‫‪0,25‬‬
‫▪ ﺘﻌرﻴﻔﻪ‪ :‬ﻫو ﻨﻘطﺔ ﻤن ﺴطﺢ اﻷرض ‪ ،‬ﻴرﺘﺒط ﺒﻪ ﻤﻌﻠم ﻤﺤﺎورﻩ اﻝﺜﻼث ﻤﺘﻬﺠﺔ ﻨﺤو ﺜﻼث ﻨﺠوم‬
‫ﺜﺎﺒﺘﺔ ﻓﻲ اﻝﻔﻀﺎء‪.‬‬
‫▪ اﻝﻔرﻀﻴﺔ اﻝﻤﺘﻌﻠﻘﺔ ﺒﻬذا اﻝﻤرﺠﻊ و اﻝﺘﻲ ﺘﺴﻤﺢ ﺒﺘطﺒﻴق اﻝﻘﺎﻨون اﻝﺜﺎﻨﻲ ﻝﻨﻴوﺘن‪ :‬ﻫﻲ أن ﻴﻜون ﻏﺎﻝﻴﻠﻴﺎ‬
‫و ﻴﺘﺤﻘق ﻫذا ﻋﻨدﻤﺎ ﺘﻜون ﻤدة اﻝدراﺴﺔ أﻗل ﺒﻜﺜﻴر ﻤن ﻤدة دوران اﻷرض ﺤول ﻨﻔﺴﻬﺎ‪.‬‬
‫‬
‫‪R‬‬ ‫‪ .2‬إﻴﺠﺎد ﻋﺒﺎرة ﺘﺴﺎرع اﻝﺤرﻜﺔ‪:‬‬
‫‪0,25‬‬
‫‬ ‫ﺒﺘطﺒﻴق اﻝﻘﺎﻨون اﻝﺜﺎﻨﻲ ﻝﻨﻴوﺘن ﻋﻠﻰ اﻝﺠﻤﻠﺔ ‪ ، S‬ﻓﻲ ﻤرﺠﻊ‬
‫'‪x‬‬ ‫‪F‬‬ ‫‪x‬‬ ‫‬
‫ﺴطﺤﻲ أرﻀﻲ ﻨﻌﺘﺒرﻩ ﻏﺎﻝﻴﻠﻲ‪ ، ∑ Fext = ma G ،‬وﻤﻨﻪ‪:‬‬
‫‪A‬‬ ‫‪B‬‬ ‫  ‬ ‫‬
‫‪0,75‬‬ ‫‪0,25‬‬ ‫‬ ‫‪ ، F + P + R = ma G‬ﺒﺎﻹﺴﻘﺎط ﻋﻠﻰ اﻝﻤﺤور ) ‪ ، ( x ' x‬ﻨﺠد‪:‬‬
‫‪P‬‬
‫‪F‬‬
‫= ‪F = ma ⇒ a‬‬
‫‬ ‫‪m‬‬
‫‪0,25‬‬ ‫▪ طﺒﻴﻌﺔ اﻝﺤرﻜﺔ‪ F ، m :‬ﺜواﺒت و ﻤﻨﻪ ‪ a‬ﺜﺎﺒت و ﻜون أن اﻝﻤﺴﺎر ﻤﺴﺘﻘﻴم ﻓﺎﻝﺤرﻜﺔ ﻤﺴﺘﻘﻴﻤﺔ‬
‫ﻤﺘﻐﻴرة ﺒﺎﻨﺘظﺎم‪.‬‬
‫‪ .1.3‬ﻗﻴﻤﺔ ﺘﺴﺎرع اﻝﺤرﻜﺔ‪:‬‬
‫‪∆v (5,5 − 2) × 4‬‬
‫= ‪a1‬‬ ‫=‬ ‫ﻤن اﻝﺒﻴﺎن ﻓﻲ اﻝطور اﻷول ﻤن اﻝﺤرﻜﺔ‪= 4 m.s −2 :‬‬
‫‪0,25‬‬ ‫‪0,25‬‬ ‫‪∆t‬‬ ‫‪3,5 − 0‬‬
‫‬
‫▪ ﺸدة ﻗوة دﻓﻊ اﻝﻤﺤرك ‪: F‬‬
‫ﻤن ﻋﺒﺎرة اﻝﺘﺴﺎرع اﻝﺴﺎﺒﻘﺔ ﻝدﻴﻨﺎ‪ ، F = ma :‬وﻤﻨﻪ‪F = 190 × 4 = 760 N :‬‬
‫‪ .2.3‬اﻝﻤﺴﺎﻓﺔ اﻝﻤﻘطوﻋﺔ ‪: AB‬‬

‫‪0,25‬‬ ‫‪0,25‬‬ ‫ﺒﺎﺴﺘﻌﻤﺎل طرﻴﻘﺔ اﻝﻤﺴﺎﺤﺔ ﻓﻲ ﺤﺴﺎب اﻝﻤﺴﺎﻓﺔ ﻤن ﻤﺨطط اﻝﺴرﻋﺔ‪:‬‬


‫‪(22 + 8) × 3,5‬‬
‫= ‪. AB‬‬ ‫‪= 52,5 m‬‬
‫‪2‬‬

‫ﺻﻔﺤﺔ ‪ 10‬ﻣﻦ ‪21‬‬


‫دورة‪ :‬ﻣﺎي ‪2023‬‬ ‫اﻹﺟﺎﺑﺔ اﻟﻨﻤﻮذﺟﻴﺔ ﻟﻤﻮﺿﻮع اﻣﺘﺤﺎن ﺷﻬﺎدة اﻟﺒﻜﺎﻟﻮﻳﺎ اﻟﺘﺠﺮﻳﺒﻴﺔ اﻟﻤﻮﺣﺪة‪ .‬ﻗﺴﻨﻄﻴﻨﺔ‪/‬ﻣﻘﺎﻃﻌﺔ‪2‬‬
‫اﻟﺸﻌﺒﺔ‪ :‬رﻳﺎﺿﻴﺎت‪ .‬ﺗﻘﻨﻲ رﻳﺎﺿﻲ‬ ‫اﺧﺘﺒﺎر ﻣﺎدة‪ :‬اﻟﻌﻠﻮم اﻟﻔﻴﺰﻳﺎﺋﻴﺔ‬

‫‪ -II‬دراﺴﺔ اﻝﺤرﻜﺔ ﻋﻠﻰ اﻝﻤﺴﺘوي اﻝﻤﺎﺌل ) ‪: ( BC‬‬


‫‪0,25‬‬ ‫‪dv‬‬ ‫‪F− f‬‬
‫‪:‬‬ ‫‪= − g.sin α +‬‬ ‫‪ .1‬إﺜﺒﺎت أن ‪:‬‬
‫‪x‬‬ ‫‪dt‬‬ ‫‪m‬‬
‫‬ ‫‬ ‫ﺒﺘطﺒﻴق اﻝﻘﺎﻨون اﻝﺜﺎﻨﻲ ﻝﻨﻴوﺘن ﻋﻠﻰ اﻝﺠﻤﻠﺔ ‪ ، S‬ﻓﻲ ﻤرﺠﻊ‬
‫‪R‬‬ ‫‪F‬‬ ‫‬
‫‪C‬‬
‫ﺴطﺤﻲ أرﻀﻲ ﻨﻌﺘﺒرﻩ ﻏﺎﻝﻴﻠﻲ‪ ، ∑ Fext = ma G ،‬وﻤﻨﻪ‪:‬‬
‫‪0,75‬‬ ‫‪0,25‬‬ ‫‬ ‫   ‬ ‫‬
‫‪f‬‬ ‫‪ ، F + P + f + R = ma G‬ﺒﺎﻹﺴﻘﺎط ﻋﻠﻰ‬
‫‪α‬‬
‫'‪x‬‬ ‫‪α‬‬ ‫اﻝﻤﺤور ) ‪ ، ( x ' x‬ﻨﺠد‪، F − P sin α − f = ma :‬‬
‫‬
‫‪B‬‬
‫‪P‬‬ ‫وﻤﻨﻪ‪:‬‬
‫) ‪dv (t‬‬ ‫) ‪dv (t‬‬
‫‪0,25‬‬ ‫‪F − mg sin α − f = m‬‬ ‫‪⇒ − mg sin α + ( F − f ) = m‬‬ ‫⇒‬
‫‪dt‬‬ ‫‪dt‬‬
‫‪dv‬‬ ‫‪F− f‬‬
‫‪= − g sin α +‬‬
‫‪dt‬‬ ‫‪m‬‬
‫‪ .2‬اﻝﻤﻌﺎدﻝﺔ اﻝزﻤﻨﻴﺔ ﻝﻠﺴرﻋﺔ ) ‪: v (t‬‬
‫‪F− f‬‬ ‫‪dv‬‬ ‫‪F− f‬‬
‫‪0,25‬‬ ‫‪0,25‬‬ ‫‪v = ( − g sin α +‬‬ ‫‪ ، = − g sin α +‬وﻤﻨﻪ‪) t + v0 :‬‬ ‫ﻝدﻴﻨﺎ ﺴﺎﺒﻘﺎ‪:‬‬
‫‪m‬‬ ‫‪dt‬‬ ‫‪m‬‬
‫‪F− f‬‬
‫‪v = ( − g sin α +‬‬ ‫ﻤن اﻝﺸروك اﻻﺒﺘداﺌﻴﺔ‪ v0 = vB :‬و ﻤﻨﻪ ﻴﺼﺒﺢ‪) t + vB :‬‬
‫‪m‬‬
‫‪ .3‬ﻗﻴﻤﺔ ﺘﺴﺎرع ﺤرﻜﺔ ﻤرﻜز ﻋطﺎﻝﺔ اﻝﺠﻤﻠﺔ ) ‪: ( S‬‬
‫‪0,25‬‬ ‫‪0,25‬‬ ‫‪∆v (4,5 − 5,5) × 4‬‬
‫= ‪a2‬‬ ‫=‬ ‫ﻤن اﻝﺒﻴﺎن ﻓﻲ اﻝطور اﻝﺜﺎﻨﻲ ﻤن اﻝﺤرﻜﺔ‪= −2 m.s −2 :‬‬
‫‪∆t‬‬ ‫‪3,5 − 0‬‬
‫‪ .4‬اﻝﻤﺴﺎﻓﺔ اﻝﻤﻘطوﻋﺔ ‪: BC‬‬
‫‪0,25‬‬ ‫‪0,25‬‬ ‫ﻤن ﻤﺨطط اﻝﺴرﻋﺔ ) ‪ v (t‬ﺒﺎﺴﺘﻌﻤﺎل طرﻴﻘﺔ اﻝﻤﺴﺎﺤﺔ ﻨﺠد‪:‬‬
‫)‪(22 + 18) × (5,5 − 3,5‬‬
‫= ‪BC‬‬ ‫‪= 40 m‬‬
‫‪2‬‬
‫‪ .5‬ﺴرﻋﺔ وﺼول اﻝﺠﻤﻠﺔ اﻝﻰ اﻝﻤوﻀﻊ ‪: C‬‬
‫‪0,25‬‬ ‫‪0,25‬‬
‫ﻤن اﻝﺒﻴﺎن‪. vC = 4,5 × 4 = 18 m.s −1 :‬‬
‫‪ -III‬دراﺴﺔ ﺤرﻜﺔ اﻝﺠﻤﻠﺔ ﺒﻌد ﻤﻐﺎدرﺘﻬﺎ اﻝﻤوﻀﻊ ‪: C‬‬
‫‪0,25‬‬ ‫‪ .1‬دراﺴﺔ طﺒﻴﻌﺔ اﻝﺤرﻜﺔ ﻓﻲ اﻝﻤﻌﻠم ) ‪: (ox , oy‬‬
‫‪0,75‬‬ ‫‪0,25‬‬ ‫ﺒﺘطﺒﻴق اﻝﻘﺎﻨون اﻝﺜﺎﻨﻲ ﻝﻨﻴوﺘن ﻋﻠﻰ اﻝﺠﻤﻠﺔ ‪ ، S‬ﻓﻲ ﻤرﺠﻊ ﺴطﺤﻲ أرﻀﻲ ﻨﻌﺘﺒرﻩ ﻏﺎﻝﻴﻠﻲ‪،‬‬
‫‬ ‫‬ ‫‬
‫‪ ، ∑ Fext = ma G‬وﻤﻨﻪ‪ ، P = ma G :‬ﺒﺎﻹﺴﻘﺎط ﻋﻠﻰ اﻝﻤﺤورﻴن ) ‪: (oy ) ، (ox‬‬
‫‪0,25‬‬

‫ﺻﻔﺤﺔ ‪ 11‬ﻣﻦ ‪21‬‬


‫دورة‪ :‬ﻣﺎي ‪2023‬‬ ‫اﻹﺟﺎﺑﺔ اﻟﻨﻤﻮذﺟﻴﺔ ﻟﻤﻮﺿﻮع اﻣﺘﺤﺎن ﺷﻬﺎدة اﻟﺒﻜﺎﻟﻮﻳﺎ اﻟﺘﺠﺮﻳﺒﻴﺔ اﻟﻤﻮﺣﺪة‪ .‬ﻗﺴﻨﻄﻴﻨﺔ‪/‬ﻣﻘﺎﻃﻌﺔ‪2‬‬
‫اﻟﺸﻌﺒﺔ‪ :‬رﻳﺎﺿﻴﺎت‪ .‬ﺗﻘﻨﻲ رﻳﺎﺿﻲ‬ ‫اﺧﺘﺒﺎر ﻣﺎدة‪ :‬اﻟﻌﻠﻮم اﻟﻔﻴﺰﻳﺎﺋﻴﺔ‬

‫‪z‬‬
‫‪0 = ma x‬‬ ‫‪0 = ma x‬‬ ‫‪a x = 0‬‬
‫‪‬‬ ‫⇒‬ ‫‪‬‬ ‫⇒‬ ‫‪‬‬
‫‪0,25‬‬ ‫‬ ‫‪ − P = ma z‬‬ ‫‪ − mg = ma z‬‬ ‫‪a z = − g‬‬
‫‪vC‬‬
‫و ﻤﻨﻪ‪:‬‬
‫‪α‬‬
‫‪0,75‬‬ ‫‪0,25‬‬ ‫‬ ‫‪ -‬ﻤﺴﻘط اﻝﺤرﻜﺔ ﻋﻠﻰ اﻝﻤﺤور ) ‪ (ox‬ﻫﻲ ﺤرﻜﺔ‬
‫‪P‬‬
‫ﻤﺴﺘﻘﻴﻤﺔ ﻤﻨﺘظﻤﺔ‪.‬‬

‫‪0,25‬‬ ‫‪ -‬ﻤﺴﻘط اﻝﺤرﻜﺔ ﻋﻠﻰ اﻝﻤﺤور ) ‪ (oz‬ﻫﻲ ﺤرﻜﺔ ﻤﺴﺘﻘﻴﻤﺔ‬


‫‪O‬‬ ‫‪x‬‬ ‫ﻤﺘﻐﻴرة ﺒﺎﻨﺘظﺎم‪.‬‬
‫‪ .2‬اﻝﻤﻌﺎدﻝﺘﻴن اﻝزﻤﻨﻴﺘﻴن ﻝﻤرﻜﺒﺘﻲ اﻝﺴرﻋﺔ ‪ v x (t ) ، vz (t ) :‬ﺒدﻻﻝﺔ ‪: α‬‬
‫‪v0 x = vC cos α  x0 = 0‬‬
‫‪ ، ‬ﻨﻜﺘب اﻝﻤﻌﺎدﻻت اﻝزﻤﻨﻴﺔ ﺜم ﻨﺴﺘﻨﺘﺞ ﻤﻌﺎدﻝﺔ‬ ‫‪;‬‬ ‫اﻝﺸروط اﻻﺒﺘداﺌﻴﺔ‪:‬‬
‫‪v0 z = vC sin α  z0 = h‬‬
‫اﻝﻤﺴﺎر‪.‬‬
‫‪ dv x‬‬
‫‪ dt = 0‬‬
‫‪0,25‬‬
‫‪a x = 0‬‬ ‫‪v x = v0 x‬‬ ‫‪v x = vC cos α‬‬
‫‪‬‬ ‫⇒‬ ‫‪‬‬ ‫⇒‬ ‫‪‬‬ ‫⇒‬ ‫‪‬‬ ‫⇒‬
‫‪a‬‬
‫‪ z‬‬ ‫=‬ ‫‪−‬‬ ‫‪g‬‬ ‫‪dv‬‬
‫‪ z = −g‬‬ ‫‪v‬‬
‫‪ z‬‬ ‫=‬ ‫‪−‬‬ ‫‪gt‬‬ ‫‪+‬‬ ‫‪v‬‬‫‪0y‬‬ ‫‪v‬‬
‫‪ z‬‬ ‫=‬ ‫‪−‬‬ ‫‪gt‬‬ ‫‪+‬‬ ‫‪v‬‬‫‪C‬‬ ‫‪sin‬‬ ‫‪α‬‬
‫‪ dt‬‬
‫‪v x = 18cos α‬‬
‫‪0,25‬‬ ‫‪‬‬
‫‪vz = −10t + 18sin α‬‬
‫‪ .1.3‬أوﺠد ﻤﻌﺎدﻝﺔ اﻝﻤﺴﺎر ) ‪. z = f (t‬‬
‫‪x‬‬
‫= ‪ ، t‬ﺒﺎﻝﺘﻌوﻴض ﻓﻲ اﻝﻤﻌﺎدﻝﺔ ) ‪ z (t‬ﻨﺠد‪:‬‬ ‫ﻤن اﻝﻤﻌﺎدﻝﺔ ) ‪: x (t‬‬
‫‪16, 2‬‬
‫‪0,25‬‬ ‫‪0,25‬‬ ‫‪2‬‬
‫‪ x ‬‬ ‫‪ x ‬‬
‫‪z = −5 ‬‬ ‫‪‬‬ ‫‪+‬‬ ‫‪8‬‬ ‫‪‬‬ ‫‪‬‬ ‫‪+ 17,5 ⇒ z = −0,019 x 2 + 0, 49 x + 17,5‬‬
‫‪ 16, 20 ‬‬ ‫‪ 16, 2 ‬‬
‫‪ .2.3‬إﻴﺠﺎد زاوﻴﺔ اﻝﻘذف ‪α‬‬
‫‪ x (t ) =16,2 t‬‬ ‫‪v x (t ) =16,2‬‬ ‫‪v x = 18cos α‬‬
‫‪0,25‬‬ ‫‪‬‬ ‫‪⇒ ‬‬ ‫⇒‬ ‫‪‬‬
‫‪ z (t ) = − 5t + 8t + 17,5‬‬ ‫=‬ ‫‪−‬‬ ‫‪+‬‬ ‫‪vz = −10t + 18sin α‬‬
‫‪2‬‬
‫‪ z‬‬
‫‪v‬‬ ‫(‬ ‫‪t‬‬ ‫)‬ ‫‪10‬‬ ‫‪t‬‬ ‫‪8‬‬
‫‪0,50‬‬ ‫‪16, 2‬‬
‫= ‪18cos α = 16, 2 ⇒ cos α‬‬ ‫ﺒﺎﻝﻤطﺎﺒﻘﺔ‪= 0,9 ⇒ α = 26° :‬‬
‫‪18‬‬
‫‪0,25‬‬ ‫▪ اﻻرﺘﻔﺎع ‪: h‬‬
‫ﻤن اﻝﺸﻜل‪h = BC.sin α = 40 × sin 26 = 17,5 m :‬‬
‫‬
‫‪ .3.3‬ﺸدة ﻗوة اﻻﺤﺘﻜﺎك ‪: f‬‬
‫‪0,25‬‬ ‫‪0,25‬‬
‫‪ ، ma = − mg .sin α + F − f‬وﻤﻨﻪ‪:‬‬ ‫ﻤﻤﺎ ﺴﺒق ﻓﻲ ﻋﻠﻰ اﻝﻤﺴﺘوي اﻝﻤﺎﺌل ﻝدﻴﻨﺎ‪:‬‬

‫ﺻﻔﺤﺔ ‪ 12‬ﻣﻦ ‪21‬‬


‫دورة‪ :‬ﻣﺎي ‪2023‬‬ ‫اﻹﺟﺎﺑﺔ اﻟﻨﻤﻮذﺟﻴﺔ ﻟﻤﻮﺿﻮع اﻣﺘﺤﺎن ﺷﻬﺎدة اﻟﺒﻜﺎﻟﻮﻳﺎ اﻟﺘﺠﺮﻳﺒﻴﺔ اﻟﻤﻮﺣﺪة‪ .‬ﻗﺴﻨﻄﻴﻨﺔ‪/‬ﻣﻘﺎﻃﻌﺔ‪2‬‬
‫اﻟﺸﻌﺒﺔ‪ :‬رﻳﺎﺿﻴﺎت‪ .‬ﺗﻘﻨﻲ رﻳﺎﺿﻲ‬ ‫اﺧﺘﺒﺎر ﻣﺎدة‪ :‬اﻟﻌﻠﻮم اﻟﻔﻴﺰﻳﺎﺋﻴﺔ‬

‫) ‪f = F − m( g.sin α + a‬‬
‫‪0,25‬‬ ‫‪0,25‬‬
‫إذن‪f = 760 − 190(10sin 26 + ( −2)) = 307,1N :‬‬
‫‪ -4‬اﻝدراج ﻴﻨﺠﺢ ﺒﺎﻝﻘﻔزة أم ﻻ‪:‬‬
‫ﻨﻌوض ‪ ، x = d = 11 m‬ﻓﻲ اﻝﻤﻌﺎدﻝﺔ ) ‪: z (t‬‬
‫‪0,25‬‬ ‫‪0,25‬‬ ‫‪z = −0,019(11)2 + (0, 49 × 11) + 17,5 ≃ 20,6 m‬‬
‫ﻨﻼﺤظ أن‪ ، z > ( L + 0,5) ⇒ z > (13 + 0,5) ⇒ z > 13,5 :‬و ﻤﻨﻪ‪ :‬اﻝدراج ﻴﻨﺠﺢ ﺒﺎﻝﻘﻔزة ﻤن‬
‫ﻓوق اﻝﺤﺎﺠز ﺒـ ‪. 0,5m‬‬
‫اﻝﺘﻤرﻴن اﻝﺜﺎﻨﻲ‪ 05) :‬ﻨﻘﺎط(‬
‫‪ .1‬اﻜﺘب ﻤﻌﺎدﻝﺔ اﻻﻨدﻤﺎج اﻝذي ﻴﺤدث ﻓﻲ ﻜل ﻤرﺤﻠﺔ ‪:‬‬
‫‪0,25‬‬ ‫اﻝﻤرﺤﻠﺔ اﻻوﻝﻰ ‪:‬‬
‫‪1‬‬
‫‪1H‬‬ ‫‪+ 11 H → 12 H + ZA X‬‬
‫‪0,25‬‬ ‫‪A‬‬
‫‪Z‬‬ ‫ﺤﺴب ﻗﺎﻨوﻨﻲ اﻻﻨﺤﻔﺎظ‪ ، 1 + 1 = 1 + Z ⇒ Z = 1 ، 1 + 1 = 2 + A ⇒ A = 0 :‬و ﻤﻨﻪ‪X :‬‬
‫‪. 11 H + 11 H → 12 H +‬‬ ‫‪0‬‬
‫‪+1e‬‬ ‫ﻫو ‪) 0+1 e‬ﺒوزﻴﺘون(‪ ،‬وﻤﻌﺎدﻝﺔ اﻝﺘﻔﻜك ﺘﺼﺒﺢ‪:‬‬
‫اﻝﻤرﺤﻠﺔ اﻝﺜﺎﻨﻴﺔ ‪:‬‬
‫‪1 H + 1 H → 2 He‬‬
‫‪1‬‬ ‫‪2‬‬ ‫‪3‬‬ ‫*‬

‫‪2 He → 2 He + γ‬‬
‫‪1,00‬‬ ‫‪3‬‬ ‫*‬ ‫‪3‬‬

‫‪0,25‬‬
‫‪2‬‬
‫‪1H‬‬ ‫‪+ 12 H → 32 He + γ‬‬
‫ﺘﻔﺴﻴر اﻨﺒﻌﺎث اﻹﺸﻌﺎع ‪ : γ‬اﻝﻨواة اﻝﻨﺎﺘﺠﺔ ‪ 32 He‬ﻓﻲ ﺤﺎﻝﺔ ﻤﺜﺎرة‪ ،‬ﻤﻤﺎ ﻴﻌﻨﻲ ﺘﻤﻠك ﻓﺎﺌض ﻓﻲ اﻝطﺎﻗﺔ‬
‫‪ ،‬ﻓﺘﺼدر ﻫذا اﻝﻔﺎﺌض ﻓﻲ اﻝطﺎﻗﺔ ﻋن طرﻴق إﺼدار إﺸﻌﺎع ‪. γ‬‬
‫اﻝﻤرﺤﻠﺔ اﻝﺜﺎﻝﺜﺔ ‪:‬‬
‫‪3‬‬
‫‪2 He‬‬ ‫‪+ 32 He → 42 He + 2 ZA Y‬‬
‫‪0,25‬‬ ‫ﺤﺴب ﻗﺎﻨوﻨﻲ اﻻﻨﺤﻔﺎظ‪ ، 2 + 2 = 2 + 2 Z ⇒ Z = 1 ، 3 + 3 = 4 + 2 A ⇒ A = 1 :‬و ﻤﻨﻪ‪:‬‬
‫‪ ZA Y‬ﻫو ‪ 11 H‬و ﻤﻨﻪ اﻝﻤﻌﺎدﻝﺔ ﺘﺼﺒﺢ‪. 32 He + 32 He → 42 He + 2 11 H :‬‬
‫‪ .2‬اﻝﻤﻌﺎدﻝﺔ اﻝﻨﻬﺎﺌﻴﺔ ﻝﺘﻔﺎﻋل اﻻﻨدﻤﺎج اﻝذي ﻴﺤدث داﺨل اﻝﺸﻤس‪:‬‬
‫‪0,25‬‬ ‫‪1‬‬
‫→ ‪+ 11 H‬‬ ‫‪2‬‬
‫‪+‬‬ ‫‪0‬‬
‫‪1H‬‬ ‫‪1H‬‬ ‫‪+1e‬‬

‫‪0,50‬‬ ‫‪1‬‬
‫‪1H‬‬ ‫‪+‬‬ ‫‪2‬‬
‫‪1H‬‬ ‫‪→ 32 He + γ‬‬
‫‪0,25‬‬ ‫‪3‬‬
‫‪2 He‬‬ ‫‪+ 32 He → 42 He + 2 11H‬‬
‫‪1‬‬
‫‪1H‬‬ ‫‪+ 32 He → 42 He +‬‬ ‫‪0‬‬
‫‪+1e‬‬ ‫‪+γ‬‬

‫ﺻﻔﺤﺔ ‪ 13‬ﻣﻦ ‪21‬‬


‫دورة‪ :‬ﻣﺎي ‪2023‬‬ ‫اﻹﺟﺎﺑﺔ اﻟﻨﻤﻮذﺟﻴﺔ ﻟﻤﻮﺿﻮع اﻣﺘﺤﺎن ﺷﻬﺎدة اﻟﺒﻜﺎﻟﻮﻳﺎ اﻟﺘﺠﺮﻳﺒﻴﺔ اﻟﻤﻮﺣﺪة‪ .‬ﻗﺴﻨﻄﻴﻨﺔ‪/‬ﻣﻘﺎﻃﻌﺔ‪2‬‬
‫اﻟﺸﻌﺒﺔ‪ :‬رﻳﺎﺿﻴﺎت‪ .‬ﺗﻘﻨﻲ رﻳﺎﺿﻲ‬ ‫اﺧﺘﺒﺎر ﻣﺎدة‪ :‬اﻟﻌﻠﻮم اﻟﻔﻴﺰﻳﺎﺋﻴﺔ‬

‫‪ .1.3‬اﻝطﺎﻗﺔ اﻝﻤﺤررة ﻋن ﺘﺸﻜل ﻨواة ﻫﻴﻠﻴوم واﺤدة ﻓﻲ اﻝﺸﻤس‪:‬‬


‫‪0,25‬‬ ‫)‪Elib = 4m(11 H ) − m( 42 He) − 2m( 0−1 e‬‬
‫‪0,25‬‬
‫‪Elib = (4 × 1,00728) − 4,0015 − (2 × 0,00055)) × 931,5‬‬
‫‪0,25‬‬
‫‪Elib = 24,70 MeV = 4,10 × 10−2 J‬‬
‫‪ .2.3‬اﻝﻨﻘص اﻝﺤﺎدث ﻓﻲ اﻝﺸﻤس ﻜل ﺜﺎﻨﻴﺔ‪:‬‬
‫ﻨﺤﺴب اﻝطﺎﻗﺔ اﻝﺘﻲ ﺘﺤررﻫﺎ اﻝﺸﻤس ﺨﻼل ﺜﺎﻨﻴﺔ ﻓﻨﺠد‪:‬‬
‫‪0,25‬‬
‫‪E‬‬
‫=‪P‬‬ ‫‪→ E = P.∆t = 3,9 × 1026 × 1 = 3,9 × 1026 J‬‬
‫‪0,25‬‬ ‫‪∆t‬‬
‫‪0,25‬‬ ‫ﻫذﻩ اﻝطﺎﻗﺔ ﺘﻜﺎﻓﺊ ﺤﺴب ﻋﻼﻗﺔ اﻨﺸﺘﺎﻴن ) ‪ ( E = mc 2‬ﻨﻘص ﻜﺘﻠﺔ ﻓﻲ اﻝﺸﻤس ﻗدرﻩ‪:‬‬
‫‪E 3,9 × 1026‬‬
‫= ‪∆m = 2‬‬ ‫‪= 4,33 × 109 kg‬‬
‫‪c‬‬ ‫) ‪(3 × 10‬‬
‫‪8 2‬‬

‫‪ .3.3‬اﻝﻨﺴﺒﺔ اﻝﻤﺌوﻴﺔ ﻝﻠﻜﺘﻠﺔ اﻝﻀﺎﺌﻌﺔ ﻤن اﻝﺸﻤس ﺤﺎﻝﻴﺎ‪:‬‬


‫‪ -‬ﻨﺤﺴب اﻝﻜﺘﻠﺔ اﻝﻀﺎﺌﻌﺔ ﻓﻲ اﻝﺸﻤس ﻤﻨذ ﺴطوﻋﻬﺎ‪.‬‬
‫‪0,25‬‬
‫‪ -‬ﻓﻲ ﻜل ﺜﺎﻨﻴﺔ ﺘﻀﻴﻊ ﻤن اﻝﺸﻤس طﺎﻗﺔ ﻗدرﻫﺎ‪ 4,33 × 109 kg :‬و ﻋﻠﻴﻪ ﻤﻨذ ﺴطوع اﻝﺸﻤس‬
‫‪0,25‬‬ ‫ﺘﻀﻴﻊ ﻜﺘﻠﺔ ﻗدرﻫﺎ‪:‬‬
‫‪0,25‬‬ ‫‪∆m ' = 4,33 × 109 × 4,6 × 109 × 365, 25 × 24 × 3600 = 6, 29 × 1026 Kg‬‬
‫' ‪∆m‬‬ ‫‪6, 29 × 1026‬‬
‫=‪P‬‬ ‫= ‪× 100‬‬ ‫و منه نسبة الكتلة اﻝﻀﺎﺌﻌﺔ ﻫﻲ‪× 100 = 0,031% :‬‬
‫‪MS‬‬ ‫‪2 × 1030‬‬
‫اﻝﺘﻤرﻴن اﻝﺜﺎﻝث‪ 06) :‬ﻨﻘﺎط(‬
‫‪0,25‬‬ ‫‪0,25‬‬
‫‪ .1‬اﻝظﺎﻫرة اﻝﺘﻲ ﺘﺤدث ﻓﻲ اﻝدارة ﻫﻲ ظﺎﻫرة اﻝﺘﺤرﻴض اﻝذاﺘﻲ ﻝﻠوﺸﻴﻌﺔ‪.‬‬
‫‪ .2‬اﻝﻤدﺨل اﻝﻤواﻓق ﻝﻜل ﻤﻨﺤﻨﻰ‪:‬‬
‫‪0,25‬‬ ‫‪0,25‬‬ ‫ﺒﺎﻋﺘﺒﺎر ‪ U1‬ﻫو اﻝﺘوﺘر اﻝذي ﻴظﻬر ﻓﻲ اﻝﻤدﺨل ‪ Y1‬و ‪ U 2‬ﻫو اﻝﺘوﺘر اﻝذي ﻴظﻬر ﻋﻠﻰ اﻝﻤدﺨل‬
‫‪ Y2‬و ﺒﺎﻻﻋﺘﻤﺎد ﻋﻠﻰ ﺨﻀﺎﺌص ﺜﻨﺎﺌﻲ اﻝﻘطب ‪ RL‬ﻴﻜون‪:‬‬
‫ﻝدﻴﻨﺎ‪ ، t = 0 → i = 0 →U1 = uR1 = R1 .i = 0 :‬و ﻫذا ﻴﺘواﻓق ﻤﻊ اﻝﻤﻨﺤﻨﻰ ‪ ، b‬إذن‪ :‬اﻝﻤدﺨل‬
‫‪0,25‬‬ ‫‪0,25‬‬
‫‪ Y1‬ﻴواﻓق اﻝﻤﻨﺤﻨﻰ ‪ b‬و اﻝﻤدﺨل ‪ Y2‬ﻴواﻓق اﻝﻤﻨﺤﻨﻰ ‪. a‬‬
‫‪ .3‬اﻝﻤﻌﺎدﻝﺔ اﻝﺘﻔﺎﻀﻠﻴﺔ اﻝﺘﻲ ﻴﺤﻘﻘﻬﺎ اﻝﺘﻴﺎر ) ‪: i (t‬‬
‫ﺤﺴب ﻗﺎﻨون ﺠﻤﻊ اﻝﺘوﺘرات‪:‬‬
‫‪di‬‬
‫‪0,25‬‬ ‫‪0,25‬‬ ‫‪uR + uR (t ) + ub (t ) ⇒ R1 i + R2 i + L‬‬ ‫‪+ r .i‬‬
‫‪1‬‬ ‫‪2‬‬
‫‪dt‬‬
‫‪di‬‬ ‫‪di‬‬ ‫) ‪( R1 + R2 + r‬‬ ‫‪E‬‬
‫‪( R1 + R2 + r ). i + L‬‬ ‫‪= E⇒ +‬‬ ‫= ‪.i‬‬
‫‪dt‬‬ ‫‪dt‬‬ ‫‪L‬‬ ‫‪L‬‬
‫ﺻﻔﺤﺔ ‪ 14‬ﻣﻦ ‪21‬‬
‫دورة‪ :‬ﻣﺎي ‪2023‬‬ ‫اﻹﺟﺎﺑﺔ اﻟﻨﻤﻮذﺟﻴﺔ ﻟﻤﻮﺿﻮع اﻣﺘﺤﺎن ﺷﻬﺎدة اﻟﺒﻜﺎﻟﻮﻳﺎ اﻟﺘﺠﺮﻳﺒﻴﺔ اﻟﻤﻮﺣﺪة‪ .‬ﻗﺴﻨﻄﻴﻨﺔ‪/‬ﻣﻘﺎﻃﻌﺔ‪2‬‬
‫اﻟﺸﻌﺒﺔ‪ :‬رﻳﺎﺿﻴﺎت‪ .‬ﺗﻘﻨﻲ رﻳﺎﺿﻲ‬ ‫اﺧﺘﺒﺎر ﻣﺎدة‪ :‬اﻟﻌﻠﻮم اﻟﻔﻴﺰﻳﺎﺋﻴﺔ‬

‫‪ .1.4‬ﻋﺒﺎرات ﻜل ﻤن ‪ τ ، A‬و ‪: B‬‬


‫‪t‬‬
‫‪di‬‬ ‫‪B − τt‬‬ ‫‪−‬‬
‫ادﻴﻨﺎ ‪ ، i = A + Be ⇒ = − e :‬ﺒﺎﻝﺘﻌوﻴض ﻓﻲ اﻝﻤﻌﺎدﻝﺔ اﻝﺘﻔﺎﻀﻠﻴﺔ ﻨﺠد‪:‬‬ ‫‪τ‬‬

‫‪d‬‬ ‫‪τ‬‬
‫‪−t‬‬ ‫‪−t‬‬
‫) ‪− B τ ( R1 + R2 + r‬‬ ‫‪E‬‬
‫‪e +‬‬ ‫= ) ‪( A + Be τ‬‬
‫‪τ‬‬ ‫‪L‬‬ ‫‪L‬‬
‫‪−t‬‬
‫) ‪−1 ( R1 + R2 + r‬‬ ‫‪R + R2 + r‬‬ ‫‪E‬‬
‫‪0,50‬‬ ‫( ‪Be τ‬‬ ‫‪+‬‬ ‫‪)+( 1‬‬ ‫= ‪)A‬‬
‫‪τ‬‬ ‫‪L‬‬ ‫‪L‬‬ ‫‪L‬‬
‫ﻝﻜﻲ ﺘﺘﺤﻘق اﻝﻤﺴﺎواة ﻴﺠب أن ﻴﻜون‪:‬‬
‫) ‪−1 ( R1 + R2 + r‬‬ ‫‪1 R + R2 + r‬‬ ‫‪L‬‬
‫▪‬ ‫‪+‬‬ ‫‪=0 ⇒ = 1‬‬ ‫= ‪⇒τ‬‬ ‫‪.‬‬
‫‪τ‬‬ ‫‪L‬‬ ‫‪τ‬‬ ‫‪L‬‬ ‫‪R1 + R2 + r‬‬
‫‪0,25‬‬ ‫‪R + R2 + r‬‬ ‫‪E‬‬ ‫‪E‬‬
‫‪▪ ( 1‬‬ ‫= ‪)A = ⇒ A‬‬
‫‪L‬‬ ‫‪L‬‬ ‫‪R1 + R2 + r‬‬
‫‪−E‬‬
‫‪0,25‬‬ ‫= ‪t = 0 ⇒ i = 0A ⇒ A = -B ⇒ B‬‬ ‫ﻤن اﻝﺸروط اﻻﺒﺘداﺌﻴﺔ‪= I 0 :‬‬
‫‪R1 + R2 + r‬‬
‫‪ .2.4‬إﺜﺒﺎت أن ‪ τ‬ﻤﺘﺠﺎﻨس ﻤﻊ اﻝزﻤن‪:‬‬
‫‪di‬‬
‫‪ ، L‬ﺒﻘﺴﻤﺔ اﻝطرﻓﻴن ﻋﻠﻰ ‪: L‬‬ ‫ﻤﻤﺎ ﺴﺒق‪+ ( R1 + R2 + r ).i(t ) = E :‬‬
‫‪dt‬‬
‫‪L‬‬ ‫‪di‬‬ ‫‪E‬‬ ‫‪di‬‬
‫= ) ‪+ i (t‬‬ ‫‪⇒ τ + i (t ) = I 0‬‬
‫‪R1 + R2 + r dt‬‬ ‫‪R1 + R2 + r‬‬ ‫‪dt‬‬
‫‪di 1‬‬ ‫‪I‬‬
‫‪0,25‬‬ ‫ﺒﻘﺴﻤﺔ اﻝطرﻓﻴن ﻋﻠﻰ ‪ τ‬ﻨﺠد‪+ .i (t ) = 0 :‬‬
‫‪ ،‬وﻤﻨﻪ‪:‬‬
‫‪dt τ‬‬ ‫‪τ‬‬
‫‪0,50‬‬ ‫‪di I 0 1‬‬ ‫‪di 1‬‬ ‫‪di 1‬‬
‫⇒ ) ‪= − .i (t‬‬ ‫⇒ ) ‪= (i ( t ) − I 0‬‬ ‫⇒ ) ‪= (i ( t ) − I 0‬‬
‫‪dt τ τ‬‬ ‫‪dt τ‬‬ ‫‪dt τ‬‬
‫‪0,25‬‬ ‫‪[I ] = [I ] ⇒ 1 = 1 ⇒ τ = T‬‬
‫إذن ‪ τ‬ﻤﺘﺠﺎﻨس ﻤﻊ اﻝزﻤن‪.‬‬ ‫] [ ] [‬ ‫وﻤﻨﻪ‪:‬‬
‫] ‪[T ] [τ ] [T ] [τ‬‬
‫‪ .3.4‬ﻋﺒﺎرة ‪ U1‬ﺒدﻻﻝﺔ ‪: τ ، I 0 ، R1‬‬
‫) ‪U1 = U R1 (t ) = R1.i (t‬‬
‫‪0,25‬‬ ‫‪0,25‬‬
‫) ‪− ( R1 + R2 + r‬‬ ‫‪−t‬‬
‫‪E‬‬ ‫‪.t‬‬
‫‪U1 (t ) = R1.‬‬ ‫‪(1 − e L‬‬ ‫) ‪) ⇒ U R1 (t ) = R1. I 0 (1 − e τ‬‬
‫‪R1 + R2 + r‬‬
‫‪t‬‬
‫‪−‬‬
‫‪:U 2 = ( R2 + r ) I 0 + R1 I 0 e‬‬ ‫‪τ‬‬
‫‪ .4.4‬إﺜﺒﺎت أن‪:‬‬
‫‪0,25‬‬ ‫‪0,25‬‬
‫‪di‬‬
‫‪U 2 (t ) = U R 2 (t ) + U b (t ) ⇒ U 2 (t ) = R2 .i (t ) + L‬‬ ‫) ‪+ r i (t‬‬
‫‪dt‬‬

‫ﺻﻔﺤﺔ ‪ 15‬ﻣﻦ ‪21‬‬


‫دورة‪ :‬ﻣﺎي ‪2023‬‬ ‫اﻹﺟﺎﺑﺔ اﻟﻨﻤﻮذﺟﻴﺔ ﻟﻤﻮﺿﻮع اﻣﺘﺤﺎن ﺷﻬﺎدة اﻟﺒﻜﺎﻟﻮﻳﺎ اﻟﺘﺠﺮﻳﺒﻴﺔ اﻟﻤﻮﺣﺪة‪ .‬ﻗﺴﻨﻄﻴﻨﺔ‪/‬ﻣﻘﺎﻃﻌﺔ‪2‬‬
‫اﻟﺸﻌﺒﺔ‪ :‬رﻳﺎﺿﻴﺎت‪ .‬ﺗﻘﻨﻲ رﻳﺎﺿﻲ‬ ‫اﺧﺘﺒﺎر ﻣﺎدة‪ :‬اﻟﻌﻠﻮم اﻟﻔﻴﺰﻳﺎﺋﻴﺔ‬

‫‪t‬‬
‫‪−‬‬ ‫‪di I 0 − τt‬‬
‫⇒ ) ‪ ، i = I 0 (1 − e τ‬و ﻤﻨﻪ ﻴﺼﺒﺢ‪:‬‬ ‫ﻝدﻴﻨﺎ‪= e :‬‬
‫‪dt τ‬‬
‫‪−t‬‬ ‫‪−t‬‬
‫‪I0‬‬
‫‪0,25‬‬ ‫‪0,25‬‬ ‫‪U 2 (t ) = ( R2 + r ) I 0 (1 − e ) + L‬‬
‫‪τ‬‬
‫‪eτ‬‬
‫‪τ‬‬
‫‪−t‬‬
‫ﺒﻌد اﻝﺘﺒﺴﻴط ﻨﺠد‪U 2 (t ) = ( R2 + r ) I 0 + R1. I 0 e τ :‬‬
‫‪ .5‬ﻗﻴﻤﺔ ‪: E‬‬
‫ﺤﺴب ﻗﺎﻨون ﺠﻤﻊ اﻝﺘوﺘرات‪ ، E = U1 (t ) + U 2 (t ) :‬ﻋﻨد اﻝﻠﺤظﺔ ‪ t = 0‬ﻨﻜﺘب‪:‬‬
‫‪0,25‬‬ ‫‪0,25‬‬
‫)‪ ، E = U1 (t = 0) + U 2 (t = 0‬اﻋﺘﻤﺎدا ﻋﻠﻰ اﻝﻤﻨﺤﻨﻰ ‪ a‬اﻝﻤواﻓق ﻝـ ) ‪ U 2 (t‬و اﻝﻤﻨﺤﻨﻰ ‪b‬‬
‫اﻝﻤواﻓق ﻝـ ) ‪ U1 (t‬ﻋﻨد اﻝﻠﺤظﺔ ‪ t = 0‬ﻴﻜون‪E = 0 + 6 = 6V :‬‬

‫ﺻﻔﺤﺔ ‪ 16‬ﻣﻦ ‪21‬‬


‫دورة‪ :‬ﻣﺎي ‪2023‬‬ ‫اﻹﺟﺎﺑﺔ اﻟﻨﻤﻮذﺟﻴﺔ ﻟﻤﻮﺿﻮع اﻣﺘﺤﺎن ﺷﻬﺎدة اﻟﺒﻜﺎﻟﻮﻳﺎ اﻟﺘﺠﺮﻳﺒﻴﺔ اﻟﻤﻮﺣﺪة‪ .‬ﻗﺴﻨﻄﻴﻨﺔ‪/‬ﻣﻘﺎﻃﻌﺔ‪2‬‬
‫اﻟﺸﻌﺒﺔ‪ :‬رﻳﺎﺿﻴﺎت‪ .‬ﺗﻘﻨﻲ رﻳﺎﺿﻲ‬ ‫اﺧﺘﺒﺎر ﻣﺎدة‪ :‬اﻟﻌﻠﻮم اﻟﻔﻴﺰﻳﺎﺋﻴﺔ‬

‫• ﻗﻴﻤﺔ ذاﺘﻴﺔ اﻝوﺸﻴﻌﺔ ‪: L‬‬


‫▪ اﻝطرﻴﻘﺔ )‪: (1‬‬
‫‪ -‬ﻨﺤﺴب أوﻻ ﻗﻴﻤﺔ ‪ τ‬وﻤن اﻝﻤﻨﺤﻨﻰ ‪ b‬اﻝﻤواﻓق ﻝـ ) ‪ U1 (t‬ﻴﻜون‪:‬‬
‫‪0,25‬‬ ‫‪t = τ ⇒U1 ( τ) = 0,63U1max = 0,63 × 3 = 1,89 V‬‬
‫‪L‬‬
‫=‪τ‬‬ ‫ﺒﺎﻹﺴﻘﺎط ﻨﺠد‪ . τ = 1 ms :‬و ﻝدﻴﻨﺎ‪⇒ L = ( R1 + R2 + r ).τ ........(*) :‬‬
‫‪R1 + R2 + r‬‬
‫ﻤن اﻝﻤﻨﺤﻨﻰ ‪ a‬اﻝﻤواﻓق ﻝـ ) ‪ U 2 (t‬واﻝﻤﻨﺤﻨﻰ ‪ b‬اﻝﻤواﻓق ﻝـ ) ‪ U1 (t‬ﻓﻲ اﻝﻨظﺎم اﻝداﺌم ﻴﻜون ﻝدﻴﻨﺎ‪:‬‬
‫) ∞( ‪U1 ( ∞) = U 2 ( ∞ ) ⇒ uR ( ∞) = uR ( ∞) + ub‬‬
‫‪1‬‬ ‫‪2‬‬

‫‪di ‬‬
‫‪R1 . i ( ∞) = R2 . i ( ∞) + L‬‬ ‫)∞ ( ‪ + r. i‬‬
‫) ∞ (‪dt ‬‬
‫‪di ‬‬
‫و ﻤﻨﻪ ﻴﺼﺒﺢ‪:‬‬ ‫ﻓﻲ اﻝﻨظﺎم اﻝداﺌم أﻴن ﻴﻜون‪ = 0 ، i ( ∞) = 0 :‬‬
‫) ∞ (‪dt ‬‬
‫‪R1 . I 0 = R2 . I 0 + r. I 0 ⇒ R1 = R2 + r‬‬
‫‪0,25‬‬
‫ﺒﺎﻝﺘﻌوﻴض ﻓﻲ اﻝﻌﻼﻗﺔ )*( ﻨﺠد‪، L = ( R1 + R1 ).τ ⇒ L = 2 R1τ :‬وﻤﻨﻪ‪:‬‬
‫‪L = 2 ×10−3 ×100 = 0, 2H‬‬
‫‪1,00‬‬
‫اﻝطرﻴﻘﺔ اﻝﺜﺎﻨﻴﺔ‪:‬‬
‫) ‪di (t‬‬
‫‪ ، ub (t ) = L‬وﻤﻨﻪ‪:‬‬ ‫ﻝدﻴﻨﺎ‪+ r. i (t ) :‬‬
‫‪dt‬‬
‫) ‪di (t‬‬ ‫) ‪u ( t ) − r. i ( t‬‬
‫‪0,25‬‬ ‫‪L‬‬ ‫‪= ub (t ) − r. i (t ) ⇒ L = b‬‬
‫‪dt‬‬ ‫) ‪di (t‬‬
‫‪dt‬‬
‫‪ -‬ﻋﻨد اﻝﻠﺤظﺔ ‪ t = 0‬ﻨﻜﺘب‪:‬‬
‫)‪ub (t = 0) − r. i (t = 0‬‬
‫=‪L‬‬ ‫)**(‪......‬‬
‫‪di ‬‬
‫‪‬‬
‫‪dt t =0‬‬
‫ﺤﺴب ﻗﺎﻨون ﺠﻤﻊ اﻝﺘوﺘرات‪:‬‬
‫‪ub (t ) + uR1 (t ) + uR 2 (t ) = E‬‬
‫‪0,25‬‬
‫‪ub (t ) + R1 . i (t ) + R2 .i (t ) = E‬‬
‫ﻋﻨد اﻝﻠﺤظﺔ ‪ t = 0‬أﻴن ‪ i = 0‬ﻴﻜون‪ub (t = 0) = E = 6V :‬‬
‫) ‪dU1 (t‬‬ ‫) ‪di (t ) di (t ) 1 dU1 (t‬‬
‫⇒ ) ‪U1 (t ) = R1 .i (t‬‬ ‫‪= R1 .‬‬ ‫⇒‬ ‫=‬ ‫ﻝدﻴﻨﺎ‪:‬‬
‫‪dt‬‬ ‫‪dt‬‬ ‫‪dt‬‬ ‫‪R1 dt‬‬

‫ﺻﻔﺤﺔ ‪ 17‬ﻣﻦ ‪21‬‬


‫دورة‪ :‬ﻣﺎي ‪2023‬‬ ‫اﻹﺟﺎﺑﺔ اﻟﻨﻤﻮذﺟﻴﺔ ﻟﻤﻮﺿﻮع اﻣﺘﺤﺎن ﺷﻬﺎدة اﻟﺒﻜﺎﻟﻮﻳﺎ اﻟﺘﺠﺮﻳﺒﻴﺔ اﻟﻤﻮﺣﺪة‪ .‬ﻗﺴﻨﻄﻴﻨﺔ‪/‬ﻣﻘﺎﻃﻌﺔ‪2‬‬
‫اﻟﺸﻌﺒﺔ‪ :‬رﻳﺎﺿﻴﺎت‪ .‬ﺗﻘﻨﻲ رﻳﺎﺿﻲ‬ ‫اﺧﺘﺒﺎر ﻣﺎدة‪ :‬اﻟﻌﻠﻮم اﻟﻔﻴﺰﻳﺎﺋﻴﺔ‬

‫‪di ‬‬ ‫‪1 dU1 ‬‬


‫= ‪‬‬
‫‪ ،‬و ﻤن اﻝﻤﻨﺤﻨﻰ ‪ b‬اﻝﻤواﻓق ﻝﻠﺘوﺘر ) ‪: U 2 (t‬‬ ‫ﻋﻨد اﻝﻠﺤظﺔ ‪ t = 0‬ﻨﻜﺘب‪ :‬‬
‫‪dt ¨ t =0 R1 dt ¨ t =0‬‬
‫‪0,25‬‬ ‫‪dU1 ‬‬ ‫‪3‬‬ ‫‪di ‬‬ ‫‪1‬‬
‫‪‬‬ ‫= ‪= −3 = 3 ×103 = 30 SI ⇒ ‬‬ ‫‪× 3 ×103 = 30 SI‬‬
‫‪dt ¨ t =0‬‬ ‫‪10‬‬ ‫‪dt ¨ t =0 100‬‬
‫‪6−0‬‬
‫=‪L‬‬ ‫ﺒﺎﻝﺘﻌوﻴض ﻓﻲ )**( ﻨﺠد‪= 0, 2 H :‬‬
‫‪30‬‬
‫‪0,50‬‬ ‫• ﻗﻴﻤﺔ ‪: I 0‬‬
‫ﻝدﻴﻨﺎ‪ ، U1 (t ) = R1 . i (t ) :‬ﻓﻲ اﻝﻨظﺎم اﻝداﺌم أﻴن ‪ i = I 0‬ﻨﻜﺘب‪، U1 ( ∞) = R1 . I 0 :‬‬
‫)∞ ( ‪U‬‬
‫‪ ، I 0 = 1‬ﻤن اﻝﻤﻨﺤﻨﻰ ‪ b‬اﻝﻤواﻓق ﻝﻠﺘوﺘر ) ‪ U1 (t‬ﻝدﻴﻨﺎ‪، U1 (∞ ) = 3V :‬‬ ‫و ﻤﻨﻪ‪:‬‬
‫‪0,25‬‬ ‫‪R1‬‬
‫‪3‬‬
‫= ‪I0‬‬ ‫إذن‪= 3 ×10−2 A :‬‬
‫‪100‬‬

‫‪ .6‬ﻗﻴﻤﺔ ‪: R2‬‬
‫ﻨﺤﺴب أوﻻ ‪: I 0‬‬
‫ﻝدﻴﻨﺎ‪ ، U1' (t ) = R1 . i '(t ) :‬ﻓﻲ اﻝﻨظﺎم اﻝداﺌم أﻴن ‪ i = I 0‬ﻨﻜﺘب‪U1' ( ∞) = R1 . I 0' :‬‬
‫)∞ ( '‪U1‬‬
‫= ‪ ، I 0‬ﻤن اﻝﻤﻨﺤﻨﻰ ‪ d‬اﻝﻤواﻓق ﻝﻠﺘوﺘر ) ‪ ، U1 (t‬ﻝدﻴﻨﺎ‪، U1 ( ∞) = 3,3V :‬‬
‫'‬ ‫'‬ ‫'‬
‫و ﻤﻨﻪ‪:‬‬
‫‪R1‬‬
‫‪0,25‬‬ ‫‪3,3‬‬
‫= '‪I 0‬‬ ‫إذن ‪= 3,30 ×10−2 A :‬‬
‫‪100‬‬
‫ﻨﺤﺴب اﻵن ‪: R2‬‬
‫طرﻴﻘﺔ )‪: (1‬‬
‫‪0,50‬‬ ‫‪E‬‬ ‫‪E‬‬ ‫‪E‬‬ ‫‪6‬‬
‫= '‪I 0‬‬ ‫= ‪⇒ R1 + R2 = ' ⇒ R2 = ' − R1 ⇒ R2‬‬ ‫‪− 100 = 82 Ω‬‬
‫‪R1 + R2‬‬ ‫‪I0‬‬ ‫‪I0‬‬ ‫‪3,3 ×10−2‬‬
‫طرﻴﻘﺔ )‪: (2‬‬
‫‪di‬‬
‫‪U 2' (t ) = ub (t ) + uR 2 (t ) ⇒ U 2' (t ) = L‬‬ ‫) ‪(t ) + R2 . i (t‬‬
‫‪dt‬‬
‫)∞ ( ' ‪U‬‬ ‫‪di‬‬
‫' ‪U 2' (∞ ) = R2 . I 0' ⇒ R2 = 2‬‬ ‫نكتب‪:‬‬ ‫ﻓﻲ اﻝﻨظﺎم اﻝداﺌم أﻴن ' ‪، i = I 0‬‬
‫‪I0‬‬ ‫‪dt‬‬
‫ﻤن اﻝﻤﻨﺤﻨﻰ ‪ c‬اﻝﻤواﻓق ﻝﻠﺘوﺘر ) ‪ ، U 2' (t‬ﻝدﻴﻨﺎ‪، U 2' ( ∞ ) = 2,7 V :‬‬
‫‪2,7‬‬
‫‪0,25‬‬ ‫= ‪R2‬‬ ‫إذن‪= 82 Ω :‬‬
‫‪3,3 ×10−2‬‬

‫ﺻﻔﺤﺔ ‪ 18‬ﻣﻦ ‪21‬‬


‫دورة‪ :‬ﻣﺎي ‪2023‬‬ ‫اﻹﺟﺎﺑﺔ اﻟﻨﻤﻮذﺟﻴﺔ ﻟﻤﻮﺿﻮع اﻣﺘﺤﺎن ﺷﻬﺎدة اﻟﺒﻜﺎﻟﻮﻳﺎ اﻟﺘﺠﺮﻳﺒﻴﺔ اﻟﻤﻮﺣﺪة‪ .‬ﻗﺴﻨﻄﻴﻨﺔ‪/‬ﻣﻘﺎﻃﻌﺔ‪2‬‬
‫اﻟﺸﻌﺒﺔ‪ :‬رﻳﺎﺿﻴﺎت‪ .‬ﺗﻘﻨﻲ رﻳﺎﺿﻲ‬ ‫اﺧﺘﺒﺎر ﻣﺎدة‪ :‬اﻟﻌﻠﻮم اﻟﻔﻴﺰﻳﺎﺋﻴﺔ‬

‫• ﻗﻴﻤﺔ ‪: r‬‬
‫) ‪di (t‬‬
‫‪U 2' (t ) = ub (t ) + uR (t ) ⇒ U 2 (t ) = L.‬‬ ‫) ‪+ ( R2 + r ). i (t‬‬
‫‪2‬‬
‫‪dt‬‬
‫‪di‬‬
‫‪0,25‬‬ ‫‪0,25‬‬ ‫نكتب‪، U 2 (∞ ) = ( R2 + r ). I 0 :‬‬ ‫في النظام الدائم أين ‪= 0 ، i = I 0‬‬
‫‪dt‬‬
‫)∞ ( ‪U‬‬ ‫)∞( ‪U‬‬
‫‪ R2 + r = 2‬ومنه‪:‬‬ ‫‪⇒r= 2‬‬ ‫‪ ،‬ﻝدﻴﻨﺎ‪ U 2 (t ) :‬اﻝﻤواﻓق ﻝﻠﺘوﺘر ‪ ، a‬ﻤن اﻝﻤﻨﺤﻨﻰ ‪− R2‬‬
‫‪I0‬‬ ‫‪I0‬‬
‫‪3‬‬
‫= ‪ ، r‬إذن‪U 2 ( ∞ ) = 3V :‬‬ ‫‪− 82 = 18 Ω‬‬
‫‪3 ×10−2‬‬
‫اﻝﺠزء اﻝﺜﺎﻨﻲ‪ 06) :‬ﻨﻘطﺔ(‬
‫اﻝﺘﻤرﻴن اﻝﺘﺠرﻴﺒﻲ‪ 06) :‬ﻨﻘﺎط(‬
‫‪0,25‬‬ ‫‪0,25‬‬ ‫‪ .1.1- I‬اﺴﺘﻨﺘﺎج اﻝﺤﺠم اﻝﻼزم ‪:V 0‬‬
‫‪C 1V 1 10−2 × 50‬‬
‫‪C 1V 1 = C 0V 0‬‬ ‫= ‪⇒V 0‬‬ ‫=‬ ‫‪= 10mL‬‬ ‫ﻝدﻴﻨﺎ ﺸرط اﻝﺘﻤدﻴد‪:‬‬
‫‪C0‬‬ ‫‪5 × 10−2‬‬
‫‪ -2.1‬اﻝﺒروﺘوﻜول اﻝﺘﺠرﻴﺒﻲ‪:‬‬
‫‪ -‬ﻨﺴﻜب ﺤﺠﻤﺎ ‪ 15mL‬ﻤن اﻝﻤﺤﻠول ‪ S 0‬ﻓﻲ اﻝﻤﺨﺒﺎر اﻝﻤدرج ذو اﻝﺴﻌﺔ ‪ 50mL‬ﺜم ﺒواﺴطﺔ‬
‫‪0,25‬‬ ‫ﻤﺎﺼﺔ ﻋﻴﺎرﻴﺔ ﺴﻌﺘﻬﺎ ‪ 10mL‬ﻨﺄﺨذ اﻝﺤﺠم ‪ V 0‬و ﻨﻔرﻏﻪ ﻓﻲ ﺤوﺠﻠﺔ ﻋﻴﺎرﻴﺔ ﺴﻌﺘﻬﺎ ‪ 50mL‬ﺘﺤﺘوي‬
‫‪0,50‬‬ ‫اﺒﺘداء ﻜﻤﻴﺔ ﻤن اﻝﻤﺎء ) ‪. ( ≈ 20mL‬‬
‫‪ -‬ﻨﻜﻤل ﺒﺎﻝﻤﺎء اﻝﻤﻘطر إﻝﻰ ﺨط اﻝﻌﻴﺎر‪.‬‬
‫‪0,25‬‬
‫‪ -‬ﻨﺴد ﻓوﻫﺘﻬﺎ ﺒﺈﺤﻜﺎم ﺜم ﻨرج ﻗﻠﻴﻼ ﻝﻠﺘﺠﺎﻨس‪.‬‬
‫‪ .1.2‬ﺘﺒﻴﻴن أن ‪ AH‬ﺤﻤض ﻀﻌﻴف‪:‬‬
‫ﻨﺤﺴب ﻨﺴﺒﺔ اﻝﺘﻘدم اﻝﻨﻬﺎﺌﻲ ‪: τ f‬‬
‫‪ H 3O + ‬‬
‫‪0,25‬‬ ‫= ‪τf‬‬ ‫‪⇒ pH = 3,05 ⇒  H 3O +  = 10−3,05 = 8,9 ×10−4 mol .L−1‬‬
‫‪c0‬‬ ‫‪f‬‬

‫‪9,8 × 10−4‬‬
‫‪0,50‬‬ ‫= ‪τf‬‬ ‫‪≃ 0,02‬‬ ‫و ﻤﻨﻪ‪:‬‬
‫‪5 ×10−2‬‬
‫‪ ، τ f < 1‬وﻤﻨﻪ ﻨﺴﺘﻨﺘﺞ أن اﻨﺤﻼل اﻝﺤﻤض ‪ AH‬ﻏﻴر ﺘﺎم و ﺒﺎﻝﺘﺎﻝﻲ ﻫو ﺤﻤض ﻀﻌﻴف‪.‬‬
‫‪0,25‬‬
‫‪ .2.2‬اﻝﺘﺤﻘق ﻤن ﻗﻴﻤﺔ ‪: pH 1‬‬
‫‪1‬‬
‫‪pH = ( pKa − log C ) ⇒ 2 pH = pKa − log C ⇒ pKa = 2 pH + log C‬‬
‫‪2‬‬

‫ﺻﻔﺤﺔ ‪ 19‬ﻣﻦ ‪21‬‬


‫دورة‪ :‬ﻣﺎي ‪2023‬‬ ‫اﻹﺟﺎﺑﺔ اﻟﻨﻤﻮذﺟﻴﺔ ﻟﻤﻮﺿﻮع اﻣﺘﺤﺎن ﺷﻬﺎدة اﻟﺒﻜﺎﻟﻮﻳﺎ اﻟﺘﺠﺮﻳﺒﻴﺔ اﻟﻤﻮﺣﺪة‪ .‬ﻗﺴﻨﻄﻴﻨﺔ‪/‬ﻣﻘﺎﻃﻌﺔ‪2‬‬
‫اﻟﺸﻌﺒﺔ‪ :‬رﻳﺎﺿﻴﺎت‪ .‬ﺗﻘﻨﻲ رﻳﺎﺿﻲ‬ ‫اﺧﺘﺒﺎر ﻣﺎدة‪ :‬اﻟﻌﻠﻮم اﻟﻔﻴﺰﻳﺎﺋﻴﺔ‬

‫‪ 2 pH 0 + log C0 ‬‬


‫‪pKa = ‬‬ ‫‪ ⇒ 2 pH1 + log C1 = 2 pH 0 + log C0‬‬
‫‪0,25‬‬ ‫‪ 2 pH1 + log C1 ‬‬
‫‪1‬‬ ‫‪1‬‬ ‫‪C‬‬
‫‪pH1 = pH 0 + (log C0 − log C1 ) = pH 0 + log 0‬‬
‫‪0,50‬‬ ‫‪2‬‬ ‫‪2‬‬ ‫‪C1‬‬
‫‪0,25‬‬ ‫‪1‬‬ ‫‪5 × 10−2‬‬
‫‪pH1 = 3,05 + log‬‬ ‫‪≃ 3, 4‬‬
‫‪2‬‬ ‫‪10−2‬‬
‫‪ .3.2‬اﺴﺘﻨﺘﺎج ‪ pKa‬و اﻝﺘﻌرف ﻋﻠﻰ اﻝﺤﻤض‪:‬‬
‫‪0,25‬‬ ‫‪1‬‬
‫‪pH = ( pKa − log C ) ⇒ 2 pH = pKa − log C ⇒ pKa = 2 pH + log C‬‬
‫‪0,50‬‬ ‫‪2‬‬
‫‪0,25‬‬ ‫ﺒﺎﻝﺘﻌوﻴض ﻋﻠﻰ اﻝﻤﺤﻠول ‪ pKa = 2 pH 0 + log C0 : S 0‬وﻤﻨﻪ‪:‬‬
‫‪pKa = 2 × 3,05 + log 5 × 10−2 = 6,1 − 1, 30 ≃ 4,8‬‬
‫‪ .3‬ﺤﺴﺎب اﻝﻨﺴﺒﺘﻴن ‪ τ f‬و ‪: τ f‬‬
‫‪1‬‬ ‫‪0‬‬
‫‪− pH 0‬‬ ‫‪−3,05‬‬
‫‪10‬‬ ‫‪10‬‬
‫‪0,25‬‬ ‫= ‪τf‬‬ ‫=‬ ‫) ‪= 0, 0178 → (1, 78%‬‬
‫‪0‬‬
‫‪C0‬‬ ‫‪5 × 10−2‬‬
‫‪10− pH 1 10 −3,4‬‬
‫‪0,75‬‬ ‫‪0,25‬‬ ‫= ‪τf‬‬ ‫) ‪= −2 = 0, 0398 → ( 3, 98%‬‬
‫‪1‬‬
‫‪C1‬‬ ‫‪10‬‬
‫▪ أﺜر اﻝﺘﻤدﻴد ﻋﻠﻰ ﺘﻔﻜك اﻝﺤﻤض‪:‬‬
‫‪0,25‬‬
‫‪ τ f 〉τ f‬ﺘﻤدﻴد اﻝﻤﺤﻠول ﻴزﻴﺢ اﻝﺘوازن ﻓﻲ اﻻﺘﺠﺎﻩ اﻝﻤﺒﺎﺸر اﻝﻤواﻓق ﻝﺘﻔﻜك اﻝﺤﻤض ﻓﻲ اﻝﻤﺎء‪.‬‬ ‫‪1‬‬ ‫‪0‬‬

‫‪ .1.1. I‬ﻤﻌﺎدﻝﺔ اﻝﺘﻔﺎﻋل و ﺘﺴﻤﻴﺔ اﻝﻤرﻜب اﻝﻨﺎﺘﺞ‪:‬‬


‫‪0,25‬‬ ‫‪0,25‬‬ ‫) ‪CH 3COOH (l ) + CH 3OH ( l ) = H 2O(l ) + CH 3COOCH 3(l‬‬
‫اﻹﺴم‪ :‬إﻴﺜﺎﻨوات اﻝﻤﺜﻴل‪.‬‬
‫‪ .2‬دور ﻜل ﻤن ‪:‬‬
‫‪0,25‬‬ ‫* اﻝﻘﻨﺎة اﻝﻀﻴﻘﺔ‪ :‬اﻝﺤﻔﺎظ ﻋﻠﻰ ﻜﻤﻴﺎت اﻝﻤﺎدة اﻻﺒﺘداﺌﻴﺔ و ذﻝك ﺒﺘﻜﺜﻴف اﻷﺒﺨرة اﻝﻤﺘﺼﺎﻋدة و‬
‫إرﺠﺎﻋﻬﺎ ﻝﻠوﺴط اﻝﺘﻔﺎﻋﻠﻲ‪".‬اﻝﺘﺴﺨﻴن ﺒﺎﻻرﺘداد"‬
‫‪0,75‬‬ ‫‪0,25‬‬ ‫* اﻝﺤوض اﻝﺠﻠﻴدي‪ :‬ﺘﺜﺒﻴط اﻝﺘﻔﺎﻋل)ﺘوﻗﻴﻔﻪ(‪.‬‬

‫‪0,25‬‬ ‫* اﻝﻔﻴﻨول ﻓﺘﺎﻝﻴﻴن‪ :‬ﻜﺎﺸف ﻤﻠون ﻤﻨﺎﺴب ﻝﻠﻤﻌﺎﻴرة )ﺤﻤض ﻀﻌﻴف‪/‬أﺴﺎس ﻗوي(‪.‬‬

‫ﺻﻔﺤﺔ ‪ 20‬ﻣﻦ ‪21‬‬


‫دورة‪ :‬ﻣﺎي ‪2023‬‬ ‫اﻹﺟﺎﺑﺔ اﻟﻨﻤﻮذﺟﻴﺔ ﻟﻤﻮﺿﻮع اﻣﺘﺤﺎن ﺷﻬﺎدة اﻟﺒﻜﺎﻟﻮﻳﺎ اﻟﺘﺠﺮﻳﺒﻴﺔ اﻟﻤﻮﺣﺪة‪ .‬ﻗﺴﻨﻄﻴﻨﺔ‪/‬ﻣﻘﺎﻃﻌﺔ‪2‬‬
‫اﻟﺸﻌﺒﺔ‪ :‬رﻳﺎﺿﻴﺎت‪ .‬ﺗﻘﻨﻲ رﻳﺎﺿﻲ‬ ‫اﺧﺘﺒﺎر ﻣﺎدة‪ :‬اﻟﻌﻠﻮم اﻟﻔﻴﺰﻳﺎﺋﻴﺔ‬

‫‪ .3‬ﺠدول اﻝﺘﻘدم ﺒدﻻﻝﺔ ‪ n2 ; n1‬و ‪: x‬‬


‫) ‪CH 3COOH (l ) + CH 3OH ( l ) = H 2O(l ) + CH 3COOCH 3(l‬‬
‫اﻝﺤﺎﻝﺔ‬ ‫التقدم‬ ‫كمية المادة ) ‪(mol‬‬
‫‪0,25‬‬ ‫‪0,25‬‬ ‫‪x=0‬‬ ‫‪n1‬‬ ‫‪n2‬‬ ‫‪0‬‬ ‫‪0‬‬
‫اﺒﺘداﺌﻴﺔ‬
‫اﻨﺘﻘﺎﻝﻴﺔ‬ ‫‪x‬‬ ‫‪n1 − x‬‬ ‫‪n2 − x‬‬ ‫‪x‬‬ ‫‪x‬‬
‫ﻨﻬﺎﺌﻴﺔ‬ ‫‪xf‬‬ ‫‪n1 − xmax‬‬ ‫‪n2 − xmax‬‬ ‫‪x‬‬ ‫‪x‬‬

‫‪0,25‬‬ ‫‪0,25‬‬ ‫‪τ f = 67%‬‬ ‫‪ .1.4‬ﻗﻴﻤﺔ اﻝﻨﺴﺒﺔ اﻝﻨﻬﺎﻴﺔ ﻝﺘﻘدم اﻝﺘﻔﺎﻋل ‪ :τ f‬ﻤن اﻝﺒﻴﺎن‪:‬‬
‫‪0,25‬‬ ‫‪0,25‬‬ ‫‪ -‬ﺒطﻴﺊ) ‪( t f ≈ 40 min‬‬ ‫‪ .2.4‬ﻤﻴزﺘﺎاﻷﺴﺘرة‪ :‬ﻤﺤدود) ‪، ( τ f 〈100%‬‬
‫‪ .1.5‬ﺘﻌﻴﻴن ‪ τ 1‬ﻋﻨد اﻝﻠﺤظﺔ ‪: t1‬‬
‫‪0,25‬‬ ‫‪0,25‬‬
‫‪τ f 1 = 40% ⇒ τ f 1 = 0,40‬‬ ‫ﻤن اﻝﺒﻴﺎن و ﺒﺎﻹﺴﻘﺎط ﻨﺠد‪:‬‬
‫‪ .2.5‬اﻝﺘرﻜﻴب اﻝﻤوﻝﻲ ﻝﻠﺠﻤﻠﺔ اﻝﻤﺘﻔﺎﻋﻠﺔ ‪:‬‬
‫‪x1‬‬ ‫‪x‬‬
‫‪0,25‬‬ ‫= ‪τ1‬‬ ‫اﻝﺘﻘدم اﻝﻤﺴﺠل ﻋﻨدﺌذ ‪ x1‬ﺤﻴث‪= 1 ⇒ x1 = n.τ 1 :‬‬
‫‪xmax n‬‬
‫‪x1 = 0,05 × 0,40 = 2 × 10−2 mol‬‬
‫‪0,75‬‬
‫ﺒﺎﻝﺘﻌوﻴض‪:‬‬
‫‪• n H O = nCH COOCH = x1 = 2 × 10 mol‬‬
‫‪−2‬‬
‫‪0,25‬‬ ‫‪2‬‬ ‫‪3‬‬ ‫‪3‬‬

‫‪• nCH COOH = nCH OH = n − x1 = 3 × 10 mol‬‬


‫‪−2‬‬
‫‪0,25‬‬ ‫‪3‬‬ ‫‪3‬‬

‫‪ .3.5‬ﺤﺠم ﻤﺤﻠول اﻝﺼود ‪Vb1‬اﻝﻤﻀﺎف ﻋﻨدﺌذ‪:‬‬


‫‪nCH COOH‬‬ ‫‪= [CH 3COOH ]1 × V = Cb ×Vb1‬‬
‫‪3‬‬
‫ﻋﻨد اﻝﺘﻜﺎﻓؤ‪:‬‬
‫‪nCH COOH‬‬ ‫‪3 × 10−2‬‬
‫‪0,25‬‬ ‫= ‪Vb1‬‬ ‫‪3‬‬
‫=‬ ‫) ‪= 2 × 10−2 L ( 20mL‬‬
‫‪0,75‬‬ ‫‪Cb‬‬ ‫‪1,5‬‬ ‫▪ اﻻﻗﺘراح اﻷﻨﺴب ﻝﺘﺤﺴﻴن اﻝﻤردود‪:‬‬
‫‪0,25‬‬ ‫‪ -‬ﺘﺤﻘﻴق اﻝﺘﻘطﻴر اﻝﻤﺠ أز ﻝﺤذف اﻝﻤﺎء اﻝﻤﺘﺸﻜل ﻤﻤﺎ ﻴدﻓﻊ ﺒﺎﻝﺘوازن ﻝﻺزاﺤﺔ ﻓﻲ اﻻﺘﺠﺎﻩ اﻝﻤﺒﺎﺸر‬
‫ﻝﺘﻌوﻴض ﻜﻤﻴﺔ ﻤﺎدة اﻝﻤﺎء اﻝﻤﺤذوﻓﺔ‪.‬‬
‫‪0,25‬‬
‫أﻤﺎ ﺤﻤض اﻝﻜﺒرﻴت اﻝﻤرﻜز ﻓﻬو وﺴﻴط ﻴﺴرع اﻝﺘﻔﺎﻋل دون اﻝﺘﺄﺜﻴر ﻋﻠﻰ ﻤردودﻩ‪.‬‬

‫ﺻﻔﺤﺔ ‪ 21‬ﻣﻦ ‪21‬‬


‫الجمهورية الجزائرية الديموقراطية الشعبية‬
‫مديرية التربية لوالية النعامة‪.‬‬ ‫وزارة التربية الوطنية‪.‬‬
‫امتحان البكالوريا التجريبي دورة ماي ‪0202‬‬ ‫الشعبة‪ :‬تقني رياضي‪.‬‬
‫المدة‪20 :‬سا و‪30‬د‬ ‫ثانوية االخوة عزوزي المشرية‬ ‫اختبار في مادة‪ :‬العلوم الفيزيائية‬
‫على المترشح أن يختار أحد الموضوعين اآلتيين‪:‬‬
‫الموضوع األول‬
‫يحتوي الموضوع األول على ‪ 05‬صفحات (من الصفحة ‪ 10‬من ‪ 10‬إلى الصفحة ‪ 05‬من ‪)10‬‬

‫الجزء األول‪ 11(:‬نقطة)‬


‫التمرين األول‪ 11( :‬نقاط)‬
‫التجهيزات الحديثة تمكننا من تسجيل بيانات للسرعة والطاقة لبعض حركات االجسام الصلبة‪ ،‬والتي بواسطتها يتم تحديد طبيعة‬
‫الحركة ومعرفة بعض المقادير المميزة لها‪.‬‬
‫يهدف هذا التمرين الى دراسة حركة مركز عطالة 𝑮 للجسم)𝑺(‪.‬‬
‫يتكون مسار جسم متحرك)𝑆( كتلته ‪ 𝑚 = 200g‬من جزئين‪ ،‬الجزء األول يمثل خط الميل األعظم لمستو مائل بزاوية‬
‫‪ 𝛼 = 45°‬عن المستوي األفقي‪ ،‬وهو عبارة عن وسادة هوائية يمكن أن نلغي االحتكاكات على المستوي المائل بتشغيل‬
‫مضخة الوسادة الهوائية‪ .‬الجزء الثاني يمثل قوس من دائرة توجد في مستو شاقولي مركزه )𝑂( ونصف قطره 𝑚‪ 𝑟 = 1‬كما‬
‫هو موضح في الشكل )𝟏𝟎(‪ ،‬نهمل تأثير الهواء في كل التمرين ونجري التجربتين‪.‬‬
‫الجزء األول‪ :‬الحركة على المستوي المائل𝑩𝑨‪.‬‬
‫التجربة األولى‪ :‬نشغل المضخة الهوائية وندفع الجسم)𝑆( من النقطة 𝐴 بسرعة 𝐴⃗𝑣 موازية لخط الميل األعظم‪.‬‬
‫بواسطة تجهيز مناسب يمكن تحديد فواصل الجسم)𝑆( على المحور)𝑥 ‪ (𝑥 ′‬في لحظات زمنية مختلة‪.‬‬
‫التجربة الثانية‪ :‬نقوم بنفس التجربة السابقة‪ ،‬لكن بدون بتشغيل المضخة الهوائية‪ .‬نعتبر االحتكاك قوة ثابتة شدتها 𝑓‪.‬‬
‫بواسطة برنامج االعالم اآللي نمثل بيانيا مربع سرعة الجسم) ‪ (𝑣 2‬بداللة الفاصلة 𝑥 في كل تجربة‪ ،‬الممثل في الشكل)𝟐𝟎(‪.‬‬
‫‪ -1‬بتطبيق القانون الثاني لنيوتن على الجسم)𝑆( خالل التجربة األولى‪ ،‬جد عبارة تسارع الحركة 𝐺𝑎‪.‬‬
‫‪ -2‬بتطبيق مبدأ انحفاظ الطاقة للجملة (جسم)𝑆() بين الموضعين𝐴 و𝑀 (موضع كيفي) بين أن عبارة تسارع الحركة ‪𝑎𝐺′‬‬
‫خالل التجربة الثانية يكتب من الشكل‪:‬‬
‫𝑓‬
‫‪𝑎𝐺′ = g . 𝑠𝑖𝑛 𝛼 −‬‬
‫𝑚‬
‫‪ -3‬أكتب العالقة التي تربط بين ‪ 𝑣 2‬و 𝑥 في كل تجربة‪.‬‬
‫) 𝟐‪𝒗𝟐 (𝒎𝟐 . 𝒔−‬‬ ‫الشكل )𝟐𝟎(‬ ‫‪ -4‬انسب كل بيان للتجربة الموافقة له مع التعليل‪.‬‬
‫)𝟐(‬ ‫‪𝒙′‬‬ ‫𝑨‬ ‫)𝐒(‬
‫الشكل )𝟏𝟎(‬

‫𝑶‬
‫𝒎𝒄𝟕 ‪𝒉 = 𝟕𝟎,‬‬
‫)𝟏(‬ ‫𝜶‬
‫𝜶‬ ‫𝒓‬
‫𝟒‬
‫𝑩‬
‫𝑵‬
‫𝟎‬ ‫𝟐 ‪𝟎,‬‬ ‫)𝒎(𝒙‬ ‫𝒙‬

‫صفحة ‪ 1‬من ‪11‬‬


‫بكالوريا تجريبي دورة ماي ‪0102‬‬ ‫الشعبة‪ :‬تقني رياضي‬ ‫اختبار في مادة العلوم الفيزيائية‬ ‫‪‬‬

‫‪ -5‬باالعتماد على البيان جد‪ :‬السرعة االبتدائية 𝐴𝑣 ‪ -‬شدة تسارع الجاذبية األرضية ‪ – g‬شدة قوة االحتكاك 𝑓‪.‬‬
‫الجزء الثاني‪ :‬الحركة على المسار الدائري 𝑵𝑩‪ ( :‬التجربة المنجزة التجربة األولى )‪.‬‬
‫‪ -1‬بتطبيق مبدأ انحفاظ الطاقة للجملة (جسم)𝑆() بين الموضعين𝐵 و𝑁‪ ،‬أحسب سرعة الجسم في النقطة𝑁 التي تقع أسفل‬
‫المسار الدائري‪.‬‬
‫‪ -2‬أحسب شدة 𝑅 قوة تأثير الطريق على الجسم في النقطة𝑁‪.‬‬

‫التمرين الثاني‪ 11( :‬نقاط)‬


‫الوشيعة عبارة عن سلك طويل من النحاس ملفوف حول أسطوانة عازلة‪.‬‬

‫تحتوي كثير من األجهزة مثل مكبرات الصوت‪ ،‬المحركات و ُ‬


‫المنوبات على الوشائع‪.‬‬
‫يهدف هذا التمرين إلى اختبار سلوك وشيعة عندما تكون مزودة بنواة حديدية وبدونها‪.‬‬
‫في حصة األعمال المخبرية طلب األستاذ من فوج من التالميذ تحقيق التركيب التجريبي الموضح في الشكل)𝟑𝟎( والذي‬
‫الشكل (‪)13‬‬ ‫من العناصر الكهربائية التالية‪:‬‬
‫𝒌‬
‫𝟏𝒀‬ ‫مولد مثالي للتوتر الكهربائي قوة المحركة الكهربائية 𝐸‪.‬‬ ‫‪-‬‬
‫)𝒃(‬ ‫مقاومة قابلة للضبط 𝑅‪.‬‬ ‫‪-‬‬
‫𝑬‬ ‫وشيعة)𝑏( مزودة بنواة حديدية ذاتيتها𝐿 ومقاومتها ‪.𝑟 = 25Ω‬‬ ‫‪-‬‬
‫𝑹‬ ‫قاطعة 𝑘 مقاومتها مهملة‪.‬‬ ‫‪-‬‬
‫𝟐𝒀‬ ‫الجزء األول‪ :‬الوشيعة دون نواة حديدية‬
‫ضبط أحد التالميذ المقاومة على القيمة ‪ 𝑅0 = 100 Ω‬والذاتية على قيمة ‪ 𝐿0‬وعند اللحظة‪ 𝑡 = 0‬قام بغلق القاطعة𝑘‪ ،‬بواسطة‬
‫)𝑽(𝒖‬ ‫الشكل (‪)04‬‬ ‫راسم االهتزاز ذو ذاكرة تمكنا من مشاهدة المنحنيات البيانية‬
‫الموضحة في الشكل)𝟒𝟎(‪.‬‬
‫‪ -1‬سم الظاهرة التي تحدث عند غلق القاطعة‪.‬‬
‫‪ -2‬أنسب كل من المنحنيين )𝑎( و)𝑏(‪ ،‬المنحنى الممثل للتوترين‬
‫)𝑡( 𝑏𝑢 و )𝑡( 𝑅𝑢‪ −‬مع التعليل‪.‬‬
‫‪ -3‬أحد المنحنيين ال يؤكد الظاهرة المالحظة‪ ،‬ماذا يجب على الفوج القيام‬
‫)𝒂(‬
‫به لتصحيح الخطأ؟‬
‫𝟐‬
‫‪ -4‬بتطبيق قانون جمع التوترات بين أن المعادلة التفاضلية للتوتر 𝑏𝑢‬
‫𝟎‬
‫𝟖‬ ‫)𝒔 𝟒‪𝒕(× 𝟏𝟎−‬‬ ‫بين طرفي الوشيعة تكتب من الشكل‪:‬‬
‫𝑟 ‪𝑑𝑢𝑏 𝑅0 +‬‬ ‫𝐸 ‪𝑟.‬‬
‫‪+‬‬ ‫= 𝑏𝑢‬
‫𝑡𝑑‬ ‫𝐿‬ ‫𝐿‬
‫‪ -5‬أثبت أن‪ 𝑢𝑏 (𝑡) = 𝑟𝐼0 + 𝑅0 𝐼0 . 𝑒 −𝑡/𝜏 :‬حل للمعادلة التفاضلية‪.‬‬
‫‪ -6‬باالعتماد على المنحنيات البيانية الممثلة في الشكل)𝟒𝟎( استنتج قيم‪:‬‬
‫)𝒃(‬
‫𝐸 القوة المحركة الكهربائية‪.‬‬ ‫‪-‬‬
‫𝜏 ثابت الزمن‪.‬‬ ‫‪-‬‬

‫صفحة ‪ 0‬من ‪11‬‬


‫بكالوريا تجريبي دورة ماي ‪0102‬‬ ‫الشعبة‪ :‬تقني رياضي‬ ‫اختبار في مادة العلوم الفيزيائية‬ ‫‪‬‬

‫‪ 𝐿0‬ذاتية الوشيعة‪.‬‬ ‫‪-‬‬


‫‪ 𝐼0‬شدة التيار األعظمي‪.‬‬ ‫‪-‬‬
‫الجزء الثاني‪ :‬الوشيعة مزودة بنواة حديدية‪.‬‬
‫نزود الوشيعة بالنواة الحديدة وذلك بإدخال النواة الحديدية بين حلقات الوشيعة‪ ،‬نقوم بإجراء تجارب حسب الجدول التالي‪:‬‬
‫)𝑽( 𝒃𝒖‬ ‫الشكل (‪)05‬‬ ‫التجربة‬ ‫𝟏𝟎‬ ‫𝟐𝟎‬
‫)𝐻(𝐿‬ ‫‪𝐿 = 𝐿0‬‬ ‫‪𝐿 = 2𝐿0‬‬
‫)‪𝑅(Ω‬‬ ‫‪𝑅 = 2𝑅0‬‬ ‫‪𝑅 = 𝑅0‬‬

‫ثم تمثيل البيانات) ‪ (𝑎′ , 𝑏 ′‬الموافقة للتجربتين الممثلتين في الشكل)𝟓𝟎(‬


‫‪ -1‬ارفق كل تجربة بالبيان الموافق مع التعليل‪.‬‬
‫‪ -2‬ماذا يمكن القول عن استقرار التيار الكهربائي في الدارة في كل تجربة‪.‬‬
‫) ‪(𝒂′‬‬
‫𝟐‬
‫) ‪(𝒃′‬‬
‫𝟎‬ ‫)𝒔 𝟒‪𝒕(× 𝟏𝟎−‬‬
‫𝟔𝟏‬

‫التمرين الثالث‪ 10( :‬نقاط)‬


‫شهد العالم تطو ار كبي ار في مختلف المجاالت الصناعية والطبية وغيرها‪ ،‬وذلك بسبب التطور الكبير في فهم التفاعالت النووية‬
‫التلقائية والمفتعلة‪.‬‬
‫يهدف هذا التمرين الى دراسة تفاعل نووي تلقائي وتفاعل نووي مفتعل‪.‬‬
‫المشعة التي تتفكك إلى‬ ‫‪243‬‬
‫𝑚𝐶‪96‬‬ ‫عند اللحظة ‪ 𝑡 = 0‬تحتوي عينة من نواة الكوريوم‬ ‫‪.I‬‬
‫‪ ، 239‬على كتلة ‪.𝑚0‬‬
‫نواة البلوتونيوم 𝑢𝑃‪94‬‬

‫وحدد طبيعة الجسيم الصادر‪.‬‬ ‫‪243‬‬


‫𝑚𝐶‪96‬‬ ‫‪ -1‬أكتب معادلة التفكك الحادث لنواة‬
‫بداللة الزمن تعطى بالعالقة‪:‬‬ ‫‪243‬‬
‫𝑚𝐶‪96‬‬ ‫‪ -2‬باستخدام قانون التناقص االشعاعي‪ ،‬بين أن تغير كتلة 𝑚 من أنوية الكوريوم‬
‫𝑏 ‪ ، 𝑙𝑛(𝑚) = 𝑎. 𝑡 +‬مع تحديد عبارة كل من 𝑎 و𝑏 بداللة ‪ 𝑚0‬و 𝜆 ثابت التفكك‪.‬‬
‫)𝒎(𝒏𝒍‬ ‫‪ -3‬باستعمال برنامج االعالم اآللي تمكنا من رسم المنحنى البياني)𝑡(𝑓 = )𝑚(𝑛𝑙 الشكل (‪)16‬‬

‫𝟐𝟎𝟑 ‪𝟐,‬‬ ‫الممثل في الشكل)𝟔𝟎( حيث الكتلة مقاسة ب)‪ ،(g‬باالعتماد على البيان جد‪:‬‬
‫‪ -1-3‬قيمة الكتلة االبتدائية ‪.𝑚0‬‬
‫‪ -2-3‬قيمة ثابت التفكك االشعاعي 𝜆‪ ،‬ثم استنتج زمن نصف العمر ‪.𝑡1/2‬‬
‫𝟓𝟕 ‪𝟎,‬‬
‫‪ -3‬أحسب عند اللحظة ‪ 𝑡 = 3𝑡1/2‬عدد األنوية المتبقية في العينة‪.‬‬
‫𝟎‬ ‫)𝒔 𝟕𝟎𝟏 ×(𝒕‬
‫𝟐 ‪𝟓𝟑,‬‬
‫تستعمل عملية االنشطار النووي في إنتاج الطاقة الكهربائية في المفاعالت النووية كما تستعمل إلنتاج األسلحة النووية‬ ‫‪.II‬‬
‫إثر‬ ‫‪239‬‬
‫𝑢𝑃‪94‬‬ ‫الذي يعتبر عماد الوقود النووي‪ ،‬تنشطر نواة البلوتونيوم‬ ‫‪239‬‬
‫𝑢𝑃‪94‬‬ ‫من األنوية االنشطارية الهامة المستعملة‬
‫‪239‬‬
‫𝑢𝑃‪94‬‬ ‫⟶ 𝑛‪+ 10‬‬ ‫‪130‬‬
‫𝑏𝑆‪51‬‬ ‫قذفها بنترون وفق المعادلة التالية‪+ 107𝑧𝑇𝑐 + 𝑦 10𝑛 :‬‬

‫صفحة ‪ 2‬من ‪11‬‬


‫بكالوريا تجريبي دورة ماي ‪0102‬‬ ‫الشعبة‪ :‬تقني رياضي‬ ‫اختبار في مادة العلوم الفيزيائية‬ ‫‪‬‬

‫‪ -1‬استنتج قيمتي كل من 𝑧 و𝑦‪.‬‬


‫‪ -2‬احسب الطاقة المحررة من هذا االنشطار بوحدة )𝑉𝑒𝑀( و بوحدة )𝐽(‪.‬‬
‫و األنتيمون 𝑏𝑆‪51‬‬
‫‪ ، 130‬ثم استنج ايهما اكثر استقرار‪.‬‬ ‫‪ -3‬احسب طاقة الربط 𝑙𝐸 لنواتي التيكنيشيوم‬
‫‪107‬‬
‫𝑐𝑇𝑧‬

‫‪ -4‬لتكن عينة من البلوتونيوم ‪ 239‬عدد موالتها 𝑙𝑜𝑚‪ 𝑛 = 0,5‬أحسب الطاقة المحررة من انشطار العينة‪.‬‬
‫‪ -5‬غاز البوتان ) ‪ُ (𝐶4 𝐻10‬يوضع في قارورات الغاز‪ ،‬أحسب عدد موالت غاز البوتان) ‪ ،(𝐶4 𝐻10‬التي تعطي عند احتراقه‬
‫نفس الطاقة التي يعطيها انشطار𝑙𝑜𝑚‪ 0,5‬من البلوتونيوم ‪ ،239‬ثم استنتج عدد قارورات الغاز‪.‬‬

‫𝑙𝑜𝑚‪1𝑎𝑛𝑠 = 365,25𝑚𝑜𝑙−1 ، 𝑀(𝑃𝑢) = 239g/𝑚𝑜𝑙 ، 𝑀(𝐶𝑚) = 243g/‬‬ ‫المعطيات‪:‬‬


‫𝐽 ‪،1𝑀𝑒𝑉 = 1,6 × 10−13‬‬ ‫‪، 𝑁𝐴 = 6,023 × 1023 𝑚𝑜𝑙 −1‬‬ ‫‪، 1𝑢 = 931,5𝑀𝑒𝑉/𝑐 2‬‬
‫كتلة الغاز في القارورة ‪13𝐾g‬‬ ‫القدرة الح اررية لغاز البوتان ) ‪، 2800𝐾𝐽/𝑚𝑜𝑙 :(𝐶4 𝐻10‬‬
‫𝑙𝑜𝑚‪𝑀(𝐶4 𝐻10 ) = 58g/‬‬

‫الجسيم أو النواة‬
‫‪1‬‬ ‫‪1‬‬ ‫‪107‬‬ ‫‪130‬‬ ‫‪239‬‬
‫𝑃‪1‬‬ ‫𝑛‪0‬‬ ‫𝑐𝑇𝑧‬ ‫𝑏𝑆‪51‬‬ ‫𝑢𝑃‪94‬‬

‫‪1,0073‬‬ ‫‪1,00866‬‬ ‫‪106,8915‬‬ ‫‪129,8836‬‬ ‫‪239 , 0521‬‬ ‫)𝒖(الكتلة‬

‫الجزء الثاني‪ 10(:‬نقطة)‬


‫التمرين التجريبي‪ 10( :‬نقاط)‬ ‫‪-‬‬
‫في إحدى حصص األعمال المخبرية أقترح أستاذ العلوم الفيزيائية على تالميذ القسم النهائي تجربتين وذلك من أجل دراسة‬
‫تفاعل حمض اإليثانويك𝐻𝑂𝑂𝐶 ‪ 𝐶𝐻3‬مع شاردة هيدروجينوكربونات) ‪ ،(𝐻𝐶𝑂3−‬ثم معايرة حمض اإليثانويك‪.‬‬
‫يهدف هذا التمرين الى المتابعة الزمنية لتحول كيميائي عن طريق قياس الضغط وتحديد ثابت الحموضة 𝒂𝑲𝒑‬
‫للثنائية ) ‪.(𝑪𝑯𝟑 𝑪𝑶𝑶𝑯⁄𝑪𝑯𝟑 𝑪𝑶𝑶−‬‬
‫التجربة األولى‪:‬‬
‫من أجل دراسة التحول الكيميائي بين حمض اإليثانويك𝐻𝑂𝑂𝐶 ‪ 𝐶𝐻3‬مع شاردة هيدروجينوكربونات) ‪ ،(𝐻𝐶𝑂3−‬نضع في‬
‫دورق مفرغ من الهواء‪ ،‬حجما𝐿𝑚‪ 𝑉1 = 60‬من محلول حمض اإليثانويك )𝑞𝑎()𝐻𝑂𝑂𝐶 ‪ (𝐶𝐻3‬تركيزه المولي ‪ 𝑐1‬ثم‬
‫نضيف إليه حجما 𝐿𝑚‪ 𝑉2 = 20‬من محلول هيدروجينوكربونات الصوديوم )𝑞𝑎() ‪(𝑁𝑎+ , 𝐻𝐶𝑂3−‬تركيزه المولي ‪𝑐2‬‬
‫المحضر وذلك بإذابة كتلة قدرها‪ 𝑚 = 1,25g‬من كربونات الصوديوم الهيدروجينية )𝑠(𝑎𝑁 ‪ . 𝐻𝐶𝑂3‬عند إضافة محلول‬
‫هيدروجينوكربونات الصوديوم نغلق الدورق بإحكام ثم نقيس ضغط الغاز الناتج خالل فترات زمنية مختلفة‪.‬‬
‫ننمذج التحول الحادث بمعادلة التفاعل الكيميائي التالية‪:‬‬
‫‪−‬‬ ‫‪−‬‬
‫)𝑞𝑎(‪𝐶𝐻3 𝐶𝑂𝑂𝐻(𝑎𝑞) + 𝐻𝐶𝑂3‬‬ ‫)𝑞𝑎(𝑂𝑂𝐶 ‪= 𝐶𝐻3‬‬ ‫)𝑙(𝑂 ‪+ 𝐶𝑂2(g) + 𝐻2‬‬
‫‪ -1‬أحسب التركيز المولي ‪ 𝑐2‬لهيدروجينوكربونات الصوديوم )𝑞𝑎() ‪.(𝑁𝑎+ , 𝐻𝐶𝑂3−‬‬
‫‪ -2‬أنشئ جدول لتقدم التفاعل‪.‬‬
‫تمت معالجة النتائج بواسطة برمجية خاصة فتحصلنا على المنحنيين ) ‪ 𝑦 = g(𝑃𝐶𝑂2‬و )𝑡(𝑓 = ‪ 𝑃𝐶𝑂2‬الممثلين‬
‫في الشكلين)𝟕𝟎( و)𝟖𝟎( على الترتيب مع العلم أن‪𝑦 = [𝐶𝐻3 𝐶𝑂𝑂𝐻]𝑡 − [𝐶𝐻3 𝐶𝑂𝑂− ]𝑡 :‬‬

‫صفحة ‪ 1‬من ‪11‬‬


‫بكالوريا تجريبي دورة ماي ‪0102‬‬ ‫الشعبة‪ :‬تقني رياضي‬ ‫اختبار في مادة العلوم الفيزيائية‬ ‫‪‬‬

‫)𝑳‪𝒚(𝒎𝒐𝒍/‬‬ ‫الشكل (‪)17‬‬ ‫)𝒂𝑷 𝟑𝟎𝟏 ×( 𝟐𝑶𝑪𝑷‬ ‫الشكل(‪)18‬‬

‫𝟓𝟐 ‪𝟎,‬‬

‫𝟎‬ ‫)𝒂𝑷 𝟑𝟎𝟏 ×( 𝟐𝑶𝑪𝑷‬ ‫𝟎‬ ‫)𝒔(𝒕‬


‫𝟓‬ ‫𝟎𝟎𝟏‬

‫‪ -3‬بتطبيق قانون الغازات المثالية‪ ،‬جد عبارة التقدم 𝑥 بداللة ‪ 𝑃𝐶𝑂2‬ضغط الغاز‪ 𝑉𝐶𝑂2 ،‬حجم الغاز‪ 𝑇،‬درجة الح اررة و 𝑅‬
‫ثابت الغازات المثالية‪.‬‬
‫‪𝑉𝐶𝑂2‬‬
‫‪ -4‬أثبت أن عبارة 𝑦 في كل لحظة زمني تعطى بالعالقة التالية‪:‬‬
‫‪𝑐1 𝑉1‬‬
‫=𝑦‬ ‫‪−2‬‬ ‫𝑃‬
‫𝑇𝑉‬ ‫‪𝑉𝑇 .𝑅.𝑇 𝐶𝑂2‬‬
‫حيث ‪𝑉𝐶𝑂2‬حجم الغاز ويقدر بالـ ‪. 𝑚3‬‬
‫‪ -5‬اعتمادا على المنحنى البياني ) ‪ 𝑦 = 𝑔(𝑃𝐶𝑂2‬الممثل في الشكل)𝟕𝟎(‪ ،‬حدد حجم غاز ثنائي أكسيد الكربون ‪𝑉𝐶𝑂2‬‬
‫والتركيز المولي ‪.𝑐1‬‬
‫‪ -6‬المنحنى البياني الممثل في الشكل)𝟖𝟎( ينقصه سلم الرسم الخاص بضغط الغاز ‪.𝑃𝐶𝑂2‬‬
‫‪ -1-6‬حدد السلم الناقص في الرسم‪.‬‬
‫‪ -2-6‬حدد المتفاعل المحد‪ ،‬ثم استنتج قيمة التقدم األعظمي 𝑥𝑎𝑚𝑥‪.‬‬
‫‪𝑉𝐶𝑂2‬‬ ‫‪𝑑𝑃𝐶𝑂2‬‬
‫= 𝑙𝑜𝑉𝑣 ثم احسب قيمتها عند اللحظة 𝑠‪.𝑡 = 100‬‬ ‫‪.‬‬ ‫‪ -7‬بين أن عبارة السرعة الحجمية تعطى بالعالقة‪:‬‬
‫𝑇‪𝑉𝑇 .𝑅.‬‬ ‫𝑡𝑑‬
‫‪ -8‬حدد قيمة زمن نصف التفاعل ‪.𝑡1/2‬‬
‫𝐾‪𝑀(𝐻𝐶𝑂3 𝑁𝑎) = 84g/𝑚𝑜𝑙 ، 𝑅 = 8,314𝑃𝑎. 𝑚3 /𝑚𝑜𝑙. 𝐾 ، 𝑇 = 298‬‬ ‫المعطيات‪:‬‬
‫التجربة الثانية‪:‬‬
‫𝑯𝒑‬ ‫الشكل (‪)19‬‬
‫عند نهاية التجربة األولى‪ ،‬قام أحد التالميذ بأخذ حجم 𝐿𝑚‪،𝑉0 = 20‬‬
‫بواسطة ماصة عيارية من المحلول السابق‪ ،‬وقام بمعايرتها بواسطة‬
‫محلول هيدروكسيد الصوديوم )𝑞𝑎() ‪ (𝑁𝑎+ + 𝐻𝑂−‬تركيزه‬
‫المولي 𝐿‪ ،𝑐𝑏 = 5 × 10−1 𝑚𝑜𝑙/‬سمح جهاز )𝑂𝐴𝑋𝐸(‬
‫برسم المنحنى البياني الممثل لتغيرات𝐻𝑝 بداللة حجم‬
‫األساس المسكوب 𝑏𝑉 الممثل في الشكل)𝟗𝟎(‪.‬‬
‫‪ -1‬حدد إحداثيات نقطة التكافؤ) 𝐸𝐻𝑝 ‪.(𝑉𝑏𝐸 ,‬‬
‫𝟐‬ ‫‪ -2‬أحسب التركيز المولي 𝑎𝑐 للحمض‪.‬‬
‫‪ -3‬حدد قيمة ال𝑎𝐾𝑝 للثنائية ) ‪.(𝐶𝐻3 𝐶𝑂𝑂𝐻/𝐶𝐻3 𝐶𝑂𝑂−‬‬
‫𝟎‬ ‫)𝑳𝒎( 𝒃𝑽‬
‫𝟔‬

‫انتهى الموضوع األول‪.‬‬

‫صفحة ‪ 5‬من ‪11‬‬


‫بكالوريا تجريبي دورة ماي ‪0102‬‬ ‫الشعبة‪ :‬تقني رياضي‬ ‫اختبار في مادة العلوم الفيزيائية‬ ‫‪‬‬

‫الموضوع الثاني‬
‫يحتوي الموضوع الثاني على ‪ 10‬صفحات (من الصفحة ‪ 16‬من ‪ 01‬إلى الصفحة ‪ 01‬من ‪)01‬‬

‫الجزء األول‪ 11(:‬نقطة)‬


‫التمرين األول‪ 11( :‬نقاط)‬
‫للمكثفات دور أساسي في بعض األجهزة الكهربائية نتيجة ميزتها في تخزين الطاقة الكهربائية وإرجاعها عند الحاجة‪.‬‬
‫يهدف هذا التمرين إلى التحقق من البيانات المدونة على مكثفة‪.‬‬
‫مكثفة غير مشحونة تحمل البيانات التالية‪ ،(160𝜇𝐹 ,330𝑉):‬للتأكد من القيمة 𝐹𝜇‪ 𝐶 = 160‬لسعة هذه المكثفة نصلها‬
‫على التسلسل مع ناقل أومي مقاومته 𝛺𝐾‪ 𝑅 = 12,5‬ثم نشحنها بواسطة مولد قوته المحركة الكهربائية 𝐸‪.‬‬
‫بواسطة برنامج خاص نرسم المنحنى البياني الممثل في الشكل)𝟏𝟎( الذي يمثل تغيرات الطاقة المخزنة في المكثفة 𝐶𝐸 أثناء‬
‫الشحن بداللة الزمن‪.‬‬
‫‪ -1‬ارسم مخططا للدارة التي تسمح بشحن المكثفة‪.‬‬
‫‪ -2‬تعطى عبارة)𝑡( 𝑅𝑢 التوتر الكهربائي بين طرفي الناقل األومي‪𝑢𝑅 (𝑡) = 𝐸𝑒 −𝑡/𝜏 :‬‬
‫)𝑱 𝟔‪𝑬𝑪 (× 𝟏𝟎−‬‬ ‫الشكل (‪)10‬‬ ‫‪ -1-2‬بين أن الجداء 𝐶 ‪ 𝑅.‬متجانس مع الزمن‪.‬‬
‫‪ -2-2‬جد عبارة)𝑡( 𝐶𝑢 التوتر الكهربائي بين طرفي المكثفة‪.‬‬
‫‪ -3‬جد عبارة )𝑡( 𝐶𝐸 الطاقة المخزنة في المكثفة‪.‬‬
‫‪ -4‬باالعتماد على البيان الممثل في الشكل)𝟏𝟎( حدد‪:‬‬
‫‪ -1-4‬ثابت الزمن 𝜏‪.‬‬
‫‪ -2-4‬سعة المكثفة 𝐶‪ ،‬هل تتوافق النتيجة مع القيمة المسجلة‬
‫من طرف الصانع على المكثفة‪.‬‬
‫𝟒𝟐𝟑‬
‫‪ -3-4‬القوة المحركة الكهربائية 𝐸 للمولد‪.‬‬
‫‪0‬‬
‫𝟐‬ ‫)𝒔(𝒕‬ ‫‪ -5‬نقوم بإجراء تجربتين وذلك بتغيير المولد المستعمل‪.‬‬
‫‪ -1-5‬نستعمل مولد قوته المحركة الكهربائية 𝑉‪ 𝐸1 = 12‬هل تتغير قيمة ثابت الزمن 𝜏؟‬
‫‪ -2-5‬نستعمل مولد قوته المحركة الكهربائية 𝑉‪ ،𝐸2 > 330‬ماذا يحدث للمكثفة في هذه الحالة؟‬

‫التمرين األول‪ 11( :‬نقاط)‬


‫الجزء األول والجزء الثاني مستقلين‪:‬‬
‫الجزء األول‪:‬‬
‫تم العثور على جمجمتي رجلين من طرف علماء البناء أثناء إنجازهم ألشغال تتعلق بإنشاء ميناء‪ .‬هذا االكتشاف استدعى‬
‫ط ِرحت‬
‫تدخل علماء اآلثار الذين أكدوا أن الرجلين عاشا في أوروبا خالل الفترة الممتدة بين ‪ 30000‬سنة و ‪ 40000‬سنة‪ُ .‬‬
‫عدة أسئلة على الباحثين‪ ،‬هل عاشا في نفس الفترة؟ هل يتعلق األمر بجريمة قتل؟ خصوصا أن جمجمة (الرجل ‪ )1‬تجمل‬
‫آثا ار للضرب تؤكد ذلك‪.‬‬

‫صفحة ‪ 0‬من ‪11‬‬


‫بكالوريا تجريبي دورة ماي ‪0102‬‬ ‫الشعبة‪ :‬تقني رياضي‬ ‫اختبار في مادة العلوم الفيزيائية‬ ‫‪‬‬

‫الشكل (‪)12‬‬
‫𝑍‬
‫‪15‬‬ ‫‪16‬‬ ‫‪17‬‬
‫يهدف هذا الجزء الى دراسة التأريخ بواسطة الكربون المشع‪.‬‬
‫‪18‬‬
‫𝐵‪5‬‬ ‫𝐶‪6‬‬ ‫𝑁‪7‬‬ ‫𝑂‪8‬‬
‫يمثل الشكل)𝟐𝟎(جزء من مخطط )𝑍 ‪ (𝑁 −‬حيث تمثل المنطقة المظللة‬
‫‪14‬‬ ‫‪15‬‬ ‫‪16‬‬ ‫‪17‬‬
‫𝐵‪5‬‬ ‫𝐶‪6‬‬ ‫𝑁‪7‬‬ ‫𝑂‪8‬‬
‫وادي االستقرار الذي يشمل األنوية المستقرة‪.‬‬
‫‪13‬‬
‫𝐵‪5‬‬
‫‪14‬‬
‫𝐶‪6‬‬
‫‪15‬‬
‫𝑁‪7‬‬
‫‪16‬‬
‫𝑂‪8‬‬ ‫‪ -1‬عرف ما يلي‪ :‬النظائر‪ ،‬التفكك ‪.𝛽 −‬‬
‫‪12‬‬
‫𝐵‪5‬‬
‫‪13‬‬
‫𝐶‪6‬‬
‫‪14‬‬
‫𝑁‪7‬‬
‫‪15‬‬
‫𝑂‪8‬‬
‫مع تحديد النواة‬ ‫‪14‬‬
‫𝐶‪6‬‬ ‫‪ -2‬اعتمادا على الشكل)𝟐𝟎(‪ ،‬اكتب معادلة تفكك النواة‬
‫‪11‬‬ ‫‪12‬‬ ‫‪13‬‬ ‫‪14‬‬
‫الناتجة 𝑋𝑍𝐴 والجسيم الصادر‪.‬‬
‫𝐵‪5‬‬ ‫𝐶‪6‬‬ ‫𝑁‪7‬‬ ‫𝑂‪8‬‬
‫‪ -3‬إن نسبة الكربون ‪ 14‬في الكائن الحي ثابتة‪ ،‬وعند موت هذا الكائن تتناقص‬
‫‪10‬‬ ‫‪11‬‬ ‫‪12‬‬ ‫‪13‬‬
‫𝐵‪5‬‬ ‫𝑁‪7‬‬ ‫𝑂‪8‬‬
‫𝐶‪6‬‬
‫𝑁‬ ‫هذه النسبة‪ ،‬أعطى تحليل عظام الجمجمتين النتائج التالية‪:‬‬
‫𝟎𝑵‬
‫طبيعة العينة‬ ‫)𝒕(𝑵‬
‫جمجمة الرجل ‪"1‬اندير"‬ ‫‪60,98‬‬
‫جمجمة الرجل ‪" 2‬سبياند"‬ ‫‪53,48‬‬

‫الرجل ‪ :2‬سبياند‬ ‫الرجل‪ : 0‬اندير‬


‫‪ -1-3‬انطالقا من نتائج التحليل‪ ،‬حدد عمر عظام كل جمجمة اندير الرجل ‪ 1‬وجمجمة سبياند الرجل ‪.2‬‬
‫‪ -2-3‬هل تتطابق النتيجة مع األخبار المطبقة من طرف العلماء؟‬
‫‪ -3-3‬هل باستطاعتنا القول أن‪ " :‬سبياند هو الذي اغتال اندير"‪.‬‬
‫𝑠𝑛𝑎‪𝑡1/2 = 5730‬‬ ‫المعطيات‪ :‬نصف عمر الكربون ‪ 14‬هو‬
‫)𝑽𝒆(𝑬‬ ‫الجزء الثاني‪:‬‬
‫يمثل الشكل)𝟑𝟎( المخطط المبسط لمستويات الطاقة في ذرة الهيدروجين‪.‬‬
‫‪0‬‬ ‫∞=𝑛‬
‫‪ -1‬ما هو طول الموجة ‪ λ‬لإلشعاع الصادر عندما تنتقل ذرة الهيدروجين‬
‫‪−0,54‬‬ ‫‪𝑛=5‬‬
‫‪𝑛=4‬‬ ‫من الحالة المثارة ‪ 3‬إلى الحالة األساسية‪.‬‬
‫‪−0,85‬‬
‫‪−1,51‬‬ ‫‪𝑛=3‬‬ ‫‪ -2‬تستقبل ذرة الهيدروجين الموجودة في الحالة األساسية فوتون آت‬
‫من إشعاع‪ ،‬يعطى طول موجته 𝑚𝑛‪ ،λ𝑎 = 121,69‬هل يمكن‬
‫‪−3,4‬‬ ‫‪𝑛=2‬‬ ‫أن تمتصه ذرة الهيدروجين؟‬
‫‪−13,6‬‬ ‫‪𝑛=1‬‬ ‫‪ -3‬ماهي الحالة التي تتواجد عليها ذرة الهيدروجين بعد امتصاص الفوتون؟‬
‫الشكل )𝟑𝟎(‬
‫المعطيات‪:‬‬
‫‪8‬‬ ‫‪−34‬‬
‫𝑠‪𝐶 = 3 × 10 𝑚/‬‬ ‫‪، ℎ = 6,62 × 10‬‬ ‫𝑠 ‪𝐽.‬‬ ‫𝐽 ‪، 1𝑒𝑉 = 1,6.× 10−19‬‬

‫التمرين الثالث‪ 10( :‬نقاط)‬


‫حدائق األلعاب المائية تعتبر المالذ اآلمن لألطفال في ظل االرتفاع الشديد لدرجة الح اررة‪ ،‬كما تعد مثاال جيدا لتطبيق قوانين‬
‫نيوتن والطاقة‪.‬‬
‫يهدف هذا التمرين لدراسة حركة طفل ينزلق وفق مستوي مائل ثم قذيفة‪.‬‬

‫صفحة ‪ 7‬من ‪11‬‬


‫بكالوريا تجريبي دورة ماي ‪0102‬‬ ‫الشعبة‪ :‬تقني رياضي‬ ‫اختبار في مادة العلوم الفيزيائية‬ ‫‪‬‬

‫ينزلق طفل كتلته ‪ 𝑚 = 30 𝐾g‬من الموضع 𝐴 بدون سرعة ابتدائية ليصل إلى الموضع 𝑂 بسرعة ‪ 𝑣⃗0‬يصنع حاملها‬
‫الزاوية 𝛼 مع األفق‪ ،‬مستوى الماء في الحوض يقع على بعد𝐻 أسفل النقطة 𝑂‪ ،‬الشكل)𝟒𝟎(‪.‬‬

‫𝑨‬
‫𝒚‬
‫الشكل (‪)04‬‬

‫𝒉‬
‫𝟎⃗𝒗‬
‫⃗𝒋‬
‫𝜶‬ ‫𝒙‬
‫𝑯‬ ‫𝑩‬ ‫𝑶‬ ‫⃗𝒊‬
‫𝑷‬

‫حوض الماء‬

‫‪ ، 𝛼 = 30°‬تسارع الجاذبية األرضية ‪. g = 9,8 𝑚. 𝑠 −2‬‬ ‫𝑚 ‪، 𝐻 = 0,5‬‬ ‫المعطيات‪، ℎ = 3,6 𝑚 :‬‬


‫‪ ‬تتم دراسة حركة مركز عطالة الطفل في مرجع سطحي أرضي نعتبره غاليليا‪.‬‬
‫‪ ‬نعتبر قوى االحتكاك على المستوي المائل تكافئ قوة وحيدة ⃗𝑓 معاكسة لجهة الحركة وموازية للمسار شدتها ثابتة‪.‬‬
‫الجزء األول‪ :‬الحركة على المستوي المائل𝑩𝑨‪.‬‬
‫) 𝟐‪𝒂𝑮 (𝒎. 𝒔−‬‬ ‫الشكل (‪)05‬‬
‫‪ -1‬مثل القوى الخارجية المطبقة على مركز عطالة الطفل )𝐺(‪.‬‬
‫‪ -2‬بواسطة برنامج االعالم اآللي نمثل بيانيا تسارع 𝐺𝑎 مركز عطالة‬
‫الطفل )𝐺( بداللة الزمن 𝑡‪ ،‬الممثل في الشكل)𝟓𝟎(‪.‬‬
‫‪ -1-2‬باستغالل البيان استنتج طبيعة حركة مركز عطالة الطفل )𝐺(‪.‬‬
‫𝟏‬
‫‪ -3‬اكتب المعادلة الزمنية للسرعة)𝑡(𝑣 ‪ ،‬ثم استنتج قيمة 𝐵𝑣 سرعة‬
‫𝟎‬ ‫𝟓 ‪𝟎,‬‬
‫)𝒔(𝒕‬ ‫الطفل لحظة وصوله إلى الموضع 𝐵‪.‬‬
‫‪ -4‬أحسب طول المستوي المائل 𝐵𝐴‪.‬‬
‫‪ -5‬أنجز الحصيلة الطاقوية للجملة (طفل ‪ +‬أرض) بين الموضعين 𝐴 و 𝐵 باعتبار أن المستوي الذي يمر من 𝐵 هو‬
‫مرجع لدراسة الطاقة الكامنة الثقالية‪ ،‬ثم جد شدة قوة االحتكاك ⃗𝑓‪.‬‬
‫الجزء الثاني‪ :‬دراسة حركة القذيفة‪.‬‬
‫نعتبر أن سرعة الطفل عند الموضع 𝑂 هي‪.𝑣0 = 5,0 𝑚. 𝑠 −1 :‬‬
‫‪ -1‬أعط نص القانون الثاني لنيوتن‪.‬‬
‫‪ -2‬بتطبيق القانون الثاني لنيوتن على حركة الطفل اعتبا ار من الموضع 𝑂‪ ،‬في المعلم )⃗𝑗 ‪ (𝑂, 𝑖⃗,‬جد‪:‬‬
‫‪ -1-2‬المعادلتين الزمنيتين للموضع )𝑡(𝑥 و )𝑡(𝑦‪.‬‬
‫‪ -2-2‬معادلة المسار )𝑥(𝑓 = 𝑦‪.‬‬
‫‪ -3-2‬علما أن طول المسبح هو 𝑚‪ ،6‬بين أن الطفل يسقط داخل حوض الماء (المسبح)‪.‬‬

‫صفحة ‪ 8‬من ‪11‬‬


‫بكالوريا تجريبي دورة ماي ‪0102‬‬ ‫الشعبة‪ :‬تقني رياضي‬ ‫اختبار في مادة العلوم الفيزيائية‬ ‫‪‬‬

‫الجزء الثاني‪ 10(:‬نقطة)‬


‫التمرين التجريبي‪ 10( :‬نقاط)‬ ‫‪-‬‬
‫تستعمل بعض األحماض الكربوكسيلية كمواد حافظة لألغذية مثل األجبان والمشروبات والمعلبات‪...‬إلخ‪ ،‬كما تستعمل في‬
‫تحظير بعض العطور ومستحضرات التجميل وبعض األدوية‪.‬‬
‫‪ -‬الجداء الشاردي للماء ‪𝐾𝑒 = 10−14‬‬ ‫المعطيات‪ - :‬تمت القياسات عند درجة ح اررة ℃‪25‬‬
‫التجربة األولى‪ :‬دراسة تفاعل محلول حمض البروبانويك مع محلول هيدروكسيد الصوديوم‪.‬‬
‫أعطى قياس ال𝐻𝑝 لمحلول البروبانويك )𝑞𝑎(𝐻𝑂𝑂𝐶 ‪ 𝐶2 𝐻5‬تركيزه المولي 𝑎𝑐 القيمة ‪ ،𝑝𝐻 = 3,45‬عن طريق المعايرة‬
‫ال 𝐻𝑝 مترية نعاير حجما 𝐿𝑚‪ 𝑉𝑎 = 5‬من المحلول مائي السابق لحمض البروبانويك ‪ ،‬بواسطة محلول مائي لهيدروكسيد‬
‫الصوديوم )𝑞𝑎() ‪ (𝑁𝑎+ , 𝐻𝑂−‬تركيزه المولي 𝐿‪.𝑐𝑏 = 5,0 × 10−3 𝑚𝑜𝑙/‬‬
‫بداللة حجم األساس المسكوب 𝑏𝑉 الممثل في‬ ‫تسمح برمجية خاصة برسم المنحنى البياني الممثل لتغيرات‬
‫]𝐻𝑂𝑂𝐶 ‪[𝐶2 𝐻5‬‬
‫] ‪[𝐶2 𝐻5 𝐶𝑂𝑂 −‬‬
‫]𝑯𝑶𝑶𝑪 𝟓𝑯 𝟐𝑪[‬
‫الشكل (‪)06‬‬ ‫الشكل)𝟔𝟎(‪.‬‬
‫] ‪[𝑪𝟐 𝑯𝟓 𝑪𝑶𝑶−‬‬
‫‪ -1‬أكتب معادلة تفاعل المعايرة الحادث‪.‬‬
‫‪28,18‬‬
‫‪ -2‬بين أن قيمة 𝑎𝐾𝑝 ثابت الحموضة للثنائية‪:‬‬
‫‪𝑝𝐾𝑎 (𝐶2 𝐻5 𝐶𝑂𝑂𝐻⁄𝐶2 𝐻5 𝐶𝑂𝑂− ) = 4,9‬‬
‫‪ -3‬أحسب ثابت التوازن 𝐾 لهذا التفاعل‪ ،‬ماذا تستنتج؟‬
‫‪ -4‬اعتمادا على المنحى البياني الممثل في الشكل)𝟔𝟎(‪،‬‬
‫حدد حجم التكافؤ 𝐸𝑏𝑉‪.‬‬
‫)𝟏 ‪(𝟓,‬‬
‫‪5‬‬
‫‪ -5‬أحسب قيمة التركيز المولي 𝑎𝑐‪.‬‬

‫𝟎‬ ‫)𝑳𝒎( 𝒃𝑽‬


‫𝟐‬ ‫التجربة الثانية‪ :‬تصنيع االستر‬
‫نحضر مزيج يتكون من 𝑙𝑜𝑚 ‪ 𝑛0 = 0,02‬من حمض البروبانويك )𝑙(𝐻𝑂𝑂𝐶 ‪ 𝐶2 𝐻5‬و 𝑙𝑜𝑚 ‪ 𝑛0 = 0,02‬من اإليثانول‬
‫)𝑙(𝐻𝑂 ‪ 𝐶2 𝐻5‬في ظروف تجريبية معينة‪ ،‬نسخن الخليط باالرتداد بعد إضافة قطرات من حمض الكبريت المركز وبعض‬
‫حجر الخفاف‪.‬‬
‫‪ -1‬أكتب معادلة التفاعل المنمذجة لتفاعل األسترة‪ ،‬ثم أعط اسم األستر الناتج باالسم النظامي‪.‬‬
‫‪ -2‬اختر من بين التراكيب التجريبية )‪ (2) ،(1‬و )‪ (3‬التالية‪ ،‬التركيب المستعمل إلنجاز هذا التصنيع‪.‬‬

‫التركيب التجريبي )𝟑(‬ ‫التركيب التجريبي )𝟐(‬ ‫التركيب التجريبي )𝟏(‬


‫صفحة ‪ 9‬من ‪11‬‬
‫بكالوريا تجريبي دورة ماي ‪0102‬‬ ‫الشعبة‪ :‬تقني رياضي‬ ‫اختبار في مادة العلوم الفيزيائية‬ ‫‪‬‬

‫‪ -3‬التتبع الزمني لتطور كمية مادة الحمض المتبقية وكمية مادة الستر الناتج مكنتنا من رسم المنحنيين )𝑎( و )𝑏(‬
‫المبيننين في الشكل)𝟕𝟎(‪.‬‬
‫‪ -1-3‬حدد المنحنى البياني الممثل لتشكل االستر‪.‬‬
‫‪ -2-3‬أحسب المردود 𝑟 لتفاعل األسترة‪ ،‬ماذا تستنتج؟‬
‫‪ -3-3‬أنشئ جدوال لتقدم التفاعل‪.‬‬
‫‪ -4-3‬أحسب ثابت التوازن 𝐾 ‪.‬‬
‫‪ -4‬في نفس الظروف التجريبية السابقة‪ ،‬نعيد التجربة باستعمال من 𝑙𝑜𝑚 ‪ 𝑛0 = 0,02‬من حمض البروبانويك‬
‫)𝑙(𝐻𝑂𝑂𝐶 ‪ 𝐶2 𝐻5‬و 𝑙𝑜𝑚 ‪ 𝑛0 = 0,04‬من اإليثانول )𝑙(𝐻𝑂 ‪،𝐶2 𝐻5‬‬
‫‪ -1-4‬جد مردود التفاعل ‪ 𝑟 ′‬في هذه الحالة‪.‬‬
‫‪ -2-4‬بلمقارنة بين 𝑟 و ‪ ، 𝑟 ′‬ماذا تستنتج؟‬

‫)𝒍𝒐𝒎 𝟑‪𝒏(× 𝟏𝟎−‬‬ ‫الشكل (‪)07‬‬

‫)𝒂(‬

‫)𝒃(‬
‫𝟒‬

‫𝟎‬ ‫)𝒏𝒊𝒎(𝒕‬
‫𝟎𝟏‬

‫انتهى الموضوع الثاني‪.‬‬

‫صفحة ‪ 11‬من ‪11‬‬


‫األستاذ‪ :‬مكي يونس‬ ‫الشعبة‪ :‬تقني رياضي المؤسسة‪ :‬ثانوية االخوة عزوزي المشرية‬ ‫اإلجابة النموذجية‪ :‬للبكالوريا التجريبي‬

‫العالمة‬
‫عناصر اإلجابة الموضوع األول‬
‫مجموع‬ ‫مجزأة‬
‫التمرين األول (‪ 40‬نقاط)‪.‬‬ ‫‪-‬‬
‫الجزء األول‪:‬‬
‫‪ -1‬عبارة تسارع الحركة 𝑮𝒂‪:‬‬
‫الجملة‪ :‬الجسم )𝑆(‬ ‫‪-‬‬
‫‪0.25‬‬ ‫المرجع‪ :‬سطحي أرضي نعتبره غاليليا‪.‬‬ ‫‪-‬‬
‫القوى الخارجية المؤثرة‪𝑅⃗ ، 𝑃⃗ :‬‬ ‫‪-‬‬
‫𝟎𝟓 ‪𝟎.‬‬
‫بتطبيق القانون الثاني لنيوتن على مركز عطالة الجسم )𝑆(‪:‬‬ ‫‪-‬‬
‫𝐺𝑎 ‪∑ 𝐹𝑒𝑥𝑡 = 𝑚. 𝑎𝐺 ⇒ 𝑃⃗ + 𝑅⃗ = 𝑚.‬‬
‫بإسقاط العبارة الشعاعية على محور الحركة‪:‬‬
‫‪0.25‬‬ ‫𝛼 𝑛𝑖𝑠 ‪𝑃𝑥 = 𝑚. 𝑎𝐺 ⇒ 𝑚. g. 𝑠𝑖𝑛 𝛼 = 𝑚. 𝑎𝐺 ⇒ 𝑎𝐺 = g.‬‬

‫‪ -2‬إثبات عبارة 𝑮‪:𝒂′‬‬


‫بتطبيق مبدأ انحفاظ الطاقة على الجملة "الجسم )𝑆(" بين الموضع𝐴 وموضع كيفي‬
‫‪1‬‬
‫‪0.25‬‬ ‫𝐴𝐶𝐸‬ ‫⇒ ‪− |𝑊𝐴𝑀 (𝑓)| + 𝑊𝐴𝑀 (𝑃⃗) = 𝐸𝐶𝑀 ⇒ 𝐸𝐶𝐴 − 𝑓. 𝑥 + 𝑚. g. ℎ𝐴𝑀 = 𝑚𝑣 2‬‬
‫‪2‬‬
‫𝛼𝑛𝑖𝑠 ‪ℎ𝐴𝑀 = 𝑥.‬‬
‫‪1‬‬
‫𝟎𝟓 ‪𝟎.‬‬ ‫)∗( … ‪𝐸𝐶𝐴 − 𝑓. 𝑥 + 𝑚. g. 𝑥. 𝑠𝑖𝑛𝛼 = 𝑚𝑣 2‬‬
‫‪2‬‬
‫باشتقاق العبارة السابقة)∗( بالنسبة للزمن نجد‪:‬‬
‫‪0.25‬‬ ‫𝑥𝑑‬ ‫‪𝑑𝑥 1‬‬ ‫𝑣𝑑‬
‫‪−𝑓.‬‬ ‫‪+ 𝑚. g. 𝑠𝑖𝑛𝛼.‬‬ ‫‪= 𝑚. 2. 𝑣.‬‬ ‫⇒‬
‫𝑡𝑑‬ ‫‪𝑑𝑡 2‬‬ ‫𝑡𝑑‬
‫𝑓‬
‫‪−𝑓. 𝑣 + 𝑚. g. 𝑠𝑖𝑛𝛼. 𝑣 = 𝑚. 𝑣. 𝑎𝐺′ ⇒ 𝑎𝐺′ = g . 𝑠𝑖𝑛 𝛼 −‬‬
‫𝑚‬
‫مالحظة يمكن استعمال عالقة محذوفية الزمن‪.‬‬
‫‪ -3‬العالقة التي تربط بين 𝟐𝒗 و 𝒙 في كل تجربة‪:‬‬
‫حسب عالقة محذوفية الزمن‪𝑣 2 − 𝑣𝐴2 = 2. 𝑎. 𝑥 :‬‬
‫‪0.25‬‬ ‫𝑥 ‪𝑣 2 − 𝑣𝐴2 = 2. g. 𝑠𝑖𝑛 𝛼 .‬‬ ‫عبارة التجربة األولى‪:‬‬
‫𝟎𝟓 ‪𝟎.‬‬
‫عبارة التجربة الثانية‪:‬‬
‫𝑓‬
‫‪0.25‬‬ ‫𝑥 ‪𝑣 2 − 𝑣𝐴2 = 2 (g . 𝑠𝑖𝑛 𝛼 − ) .‬‬
‫𝑚‬

‫الصفحة ‪ 1‬من ‪11‬‬


‫األستاذ‪ :‬مكي يونس‬ ‫الشعبة‪ :‬تقني رياضي المؤسسة‪ :‬ثانوية االخوة عزوزي المشرية‬ ‫اإلجابة النموذجية‪ :‬للبكالوريا التجريبي‬

‫‪ -0‬انساب كل بيان للتجربة الموافقة له مع التعليل‪:‬‬


‫لدينا ‪𝑎𝐺 > 𝑎𝐺′ :‬‬
‫‪0.25‬‬ ‫المالحظ ان ميل المنحنى الثاني أكبر من ميل المنحنى األول ومنه‪:‬‬
‫𝟎𝟓 ‪𝟎.‬‬
‫البيان الثاني يوافق التجربة األولى (دون احتكاك)‪.‬‬
‫‪0.25‬‬
‫البيان األول يوافق التجربة الثانية (في وجود احاحتكاك)‪.‬‬

‫‪ -5‬اعتمادا على البيان إيجاد‪:‬‬


‫السرعة احابتدائية 𝐴𝑣‬
‫‪0.25‬‬ ‫معادلة البيان الثاني)‪ 𝑣 2 = 𝑎2 . 𝑥 + 4 :(2‬حيث ‪ 𝑎2‬يمثل ميل المنحنى‪.‬‬
‫بالمطابقة مع العالقة النظرية ‪ 𝑣 2 = 2. g. 𝑠𝑖𝑛 𝛼 . 𝑥 + 𝑣𝐴2‬نجد‪:‬‬
‫‪0.25‬‬ ‫𝑠‪𝑣𝐴2 = 4 ⇒ 𝑣𝐴 = 2𝑚/‬‬
‫شدة تسارع الجاذبية األرضية ‪g‬‬
‫بالمطابقة نجد‪2. g. 𝑠𝑖𝑛 𝛼 = 𝑎2 :‬‬
‫‪18 − 4‬‬
‫‪0.25‬‬ ‫= ‪𝑎2‬‬ ‫‪= 14‬‬ ‫‪14‬‬
‫{‬ ‫‪1−0‬‬ ‫=‪⇒g‬‬ ‫‪⇒ g ≈ 10𝑚/𝑠 2‬‬
‫‪2. g. 𝑠𝑖𝑛 𝛼 = 2. g. 𝑠𝑖𝑛45‬‬ ‫‪2.‬‬ ‫‪𝑠𝑖𝑛45‬‬
‫𝟎𝟎 ‪𝟏.‬‬
‫شدة قوة احاحتكاك 𝑓‪.‬‬
‫= ‪ 𝑎1‬يمثل ميل‬ ‫معادلة البيان احاول)‪ 𝑣 2 = 𝑎1 . 𝑥 + 4 :(1‬حيث ‪= 10𝑚/𝑠 2‬‬
‫‪14−4‬‬
‫‪1−0‬‬
‫المنحنى‪.‬‬
‫بالمطابقة مع العالقة النظرية ‪ 𝑣 2 = 2 (g . 𝑠𝑖𝑛 𝛼 − ) . 𝑥 + 𝑣𝐴2‬نجد‪:‬‬
‫𝑓‬
‫𝑚‬
‫𝑓‬ ‫‪𝑎1‬‬ ‫‪10‬‬
‫) ‪2 (g . 𝑠𝑖𝑛 𝛼 − ) = 𝑎1 ⇒ 𝑓 = 𝑚 (g . 𝑠𝑖𝑛 𝛼 − ) = 0,2 (10 . 𝑠𝑖𝑛 45 −‬‬
‫𝑚‬ ‫‪2‬‬ ‫‪2‬‬
‫‪0.25‬‬ ‫𝑁‪⇒ 𝑓 = 0,4‬‬

‫الجزء الثاني‪:‬‬
‫‪ -1‬حساب سرعة الجسم في النقطة𝑵‪:‬‬
‫نعتبر المستوي المرجعي لدراسة الطاقة الكامنة الثقالية هو المستوي األفقي الذي يشمل 𝑁‬
‫‪1‬‬ ‫‪1‬‬
‫‪0.25‬‬ ‫‪𝐸𝐶𝐵 + 𝐸𝑃𝑃𝐵 = 𝐸𝐶𝑁 ⇒ 𝑚. 𝑣𝐵2 + 𝑚. g. ℎ𝐵𝑁 = 𝑚. 𝑣𝑁2‬‬
‫𝟎𝟓 ‪𝟎.‬‬ ‫‪2‬‬ ‫‪2‬‬
‫)𝛼𝑠𝑜𝑐 ‪ℎ𝐵𝑁 = 𝑟(1 −‬‬
‫)𝛼𝑠𝑜𝑐 ‪𝑣𝑁 = 𝑣𝐵2 + 2. g. 𝑟(1 −‬‬
‫‪2‬‬

‫معادلة البيان الثاني)‪ 𝑣𝐵2 = 18𝑚2 /𝑠 2 :(2‬بالتعويض نجد‪:‬‬


‫‪0.25‬‬ ‫𝑠‪𝑣𝑁 = √18 + 2.10.1(1 − 𝑐𝑜𝑠45) ⇒ 𝑣𝑁 ≈ 4,9𝑚/‬‬

‫الصفحة ‪ 2‬من ‪11‬‬


‫األستاذ‪ :‬مكي يونس‬ ‫الشعبة‪ :‬تقني رياضي المؤسسة‪ :‬ثانوية االخوة عزوزي المشرية‬ ‫اإلجابة النموذجية‪ :‬للبكالوريا التجريبي‬

‫‪ -2‬شدة 𝑹 قوة تأثير الطريق على الجسم في النقطة𝑵‪:‬‬


‫الجملة‪ :‬الجسم )𝑆(‬ ‫‪-‬‬
‫المرجع‪ :‬سطحي أرضي نعتبره غاليليا‪.‬‬ ‫‪-‬‬

‫‪0.25‬‬ ‫القوى الخارجية المؤثرة‪𝑅⃗ ، 𝑃⃗ :‬‬ ‫‪-‬‬


‫𝟎𝟓 ‪𝟎.‬‬
‫بتطبيق القانون الثاني لنيوتن على مركز عطالة الجسم )𝑆(‪:‬‬ ‫‪-‬‬
‫𝑎 ‪∑ 𝐹𝑒𝑥𝑡 = 𝑚. 𝑎 ⇒ 𝑃⃗ + 𝑅⃗ = 𝑚.‬‬
‫بإسقاط العبارة الشعاعية على الناظم‪:‬‬
‫‪𝑣𝑁2‬‬ ‫‪4,92‬‬
‫‪𝑃 − 𝑅 = 𝑚. 𝑎𝑛 ⇒ 𝑅 = 𝑃 + 𝑚. 𝑎𝑛 = 𝑚. (g + ) ⇒ 𝑅 = 0,2. (10 +‬‬ ‫)‬
‫‪0.25‬‬ ‫𝑟‬ ‫‪1‬‬
‫𝑁‪⇒ 𝑅 = 6,8‬‬
‫التمرين الثاني (‪ 40‬نقاط)‪.‬‬ ‫‪-‬‬
‫الجزء األول‪:‬‬
‫𝟓𝟐 ‪𝟎.‬‬ ‫‪0.25‬‬ ‫‪ -1‬الظاهرة التي تحدث عند غلق القاطعة‪ :‬التحريض الكهرومغناطيسي مما يؤدي إلى تأخير نشأة‬
‫التيار الكهربائي في الدارة‪.‬‬
‫‪ -2‬انساب كل منحنى‪:‬‬
‫‪0.25‬‬ ‫المنحنى )𝑏( يمثل‪−𝑢𝑅 (𝑡):‬‬ ‫المنحنى )𝑎( يمثل‪𝑢𝑏 (𝑡) :‬‬
‫𝟎𝟓 ‪𝟎.‬‬ ‫التعليل‪ :‬عند غلق الدارة ينشأ تيار كهربائي مع مرور الزمن مما يؤدي إلى تزايد التوتر بين طرفي‬
‫‪0.25‬‬
‫الناقل األومي )𝑡( 𝑅𝑢‪ ،‬وحسب قانون جمع التوترات‪ ،‬التوتر بين طرفي الوشيعة يتناقص)𝑡( 𝑏𝑢 ‪.‬‬

‫‪ -3‬تحديد مشكلة الفوج‪:‬‬


‫عند غلق الدارة ينشأ تيار كهربائي مع مرور الزمن مما يؤدي إلى تزايد التوتر بين طرفي الناقل‬
‫األومي )𝑡( 𝑅𝑢‪ ،‬وحسب قانون جمع التوترات‪ ،‬التوتر بين طرفي الوشيعة يتناقص)𝑡( 𝑏𝑢‬
‫‪0.25‬‬
‫𝟓𝟐 ‪𝟎.‬‬ ‫حسب ربط راسم احاهتزاز ذو الذاكرة‪ ،‬المدخل ‪ 𝑦2‬مربوط من جهة الكمون المنخفض للمولد‪،‬‬
‫وعليه التوتر بين طرفي الناقل األومي يظهر على الشاشة معكوس‪ ،‬من أجل تصحيحه يجب‬
‫الضغط على الزر )𝑉𝑁𝐼( للمدخل ‪𝑦2‬‬

‫‪ -0‬المعادلة التفاضلية للتوتر 𝒃𝒖‪:‬‬


‫بتطبيق قانون جمع الوترات‬
‫𝑖𝑑‬
‫‪0.25‬‬ ‫‪𝑢𝑏 + 𝑢𝑅0 = 𝐸 ⇒ 𝐿.‬‬ ‫)∗( … 𝐸 = 𝑖) ‪+ (𝑟 + 𝑅0‬‬
‫𝑡𝑑‬
‫𝑏𝑢 ‪𝐸 −‬‬
‫‪𝑢𝑅0‬‬ ‫= 𝑖 ⇒ 𝑖 ‪= 𝐸 − 𝑢𝑏 = 𝑅.‬‬ ‫)‪… … (1‬‬
‫𝟎𝟓 ‪𝟎.‬‬ ‫𝑅‬
‫نشتق العبارة )‪ (1‬بالنسبة للزمن نجد‪= − . 𝑏 … … (1) :‬‬
‫𝑖𝑑‬ ‫𝑢𝑑 ‪1‬‬
‫𝑡𝑑‬ ‫𝑅‬ ‫𝑡𝑑‬
‫نعوض العبارة )‪ (1‬والعبارة )‪ (2‬في العبارة )∗(‪:‬‬
‫‪0.25‬‬ ‫𝑏𝑢𝑑 𝐿‬ ‫𝑏𝑢 ‪𝐸 −‬‬ ‫𝑟 ‪𝑑𝑢𝑏 𝑅0 +‬‬ ‫𝐸 ‪𝑟.‬‬
‫‪−‬‬ ‫‪.‬‬ ‫) ‪+ (𝑟 + 𝑅0‬‬ ‫⇒𝐸=‬ ‫‪+‬‬ ‫= 𝑏𝑢‬
‫𝑡𝑑 ‪𝑅0‬‬ ‫‪𝑅0‬‬ ‫𝑡𝑑‬ ‫𝐿‬ ‫𝐿‬

‫الصفحة ‪ 3‬من ‪11‬‬


‫األستاذ‪ :‬مكي يونس‬ ‫الشعبة‪ :‬تقني رياضي المؤسسة‪ :‬ثانوية االخوة عزوزي المشرية‬ ‫اإلجابة النموذجية‪ :‬للبكالوريا التجريبي‬

‫‪ -5‬إثبات أن‪ 𝒖𝒃 (𝒕) = 𝒓𝑰𝟎 + 𝑹𝟎 𝑰𝟎 . 𝒆−𝒕/𝝉 :‬حل للمعادلة التفاضلية‪:‬‬


‫𝑏𝑢𝑑‬ ‫‪𝑅0 𝐼0 −𝑡/‬‬
‫‪0.25‬‬ ‫‪=−‬‬ ‫𝜏 𝑒‪.‬‬
‫𝑡𝑑‬ ‫𝜏‬
‫بتعويض الحل والمشتقة في المعالة التفاضلية نجد‪:‬‬
‫𝟎𝟓 ‪𝟎.‬‬ ‫‪𝑅0 𝐼0 −𝑡/‬‬ ‫𝑟 ‪𝑅0 +‬‬ ‫𝐸 ‪𝑟.‬‬
‫‪−‬‬ ‫‪.𝑒 𝜏 +‬‬ ‫= ) 𝜏‪(𝑟𝐼0 + 𝑅0 𝐼0 . 𝑒 −𝑡/‬‬ ‫⇒‬
‫𝜏‬ ‫𝐿‬ ‫𝐿‬
‫𝑟 ‪𝑅0 +‬‬ ‫𝐸 ‪𝑟.‬‬
‫‪0.25‬‬ ‫= ‪𝑟𝐼0‬‬ ‫𝐸 = ‪⇒ (𝑅0 + 𝑟)𝐼0‬‬
‫𝐿‬ ‫𝐿‬
‫ومنه العبارة حل للمعادلة التفاضلية‪.‬‬
‫‪ -6‬اعتمادا على البيان إيجاد‪:‬‬
‫‪0.25‬‬ ‫𝐸 القوة المحركة الكهربائية‪ :‬من المنحنى )𝑎( ‪𝑢𝑏 (𝑡 = 0) = 𝐸 ⇒ 𝐸 = 10𝑉 :‬‬
‫𝟎𝟎 ‪𝟏.‬‬
‫𝜏 ثابت الزمن‪ :‬من المنحنى )𝑎( المماس عند اللحظة ‪ 𝑡 = 0‬يقطع الخط المقارب ‪ 𝑟𝐼0‬في‬
‫‪0.25‬‬ ‫نقطة فاصلتها 𝜏 و منه 𝑠 ‪𝜏 = 8 × 10−4‬‬
‫‪ 𝐿0‬ذاتية الوشيعة‪:‬‬
‫‪𝐿0‬‬
‫‪0.25‬‬ ‫=𝜏‬ ‫𝐻‪⇒ 𝐿0 = 𝜏(𝑅0 + 𝑟) = 8 × 10−4 (100 + 25) ⇒ 𝐿0 = 0,1‬‬
‫𝑟 ‪𝑅0 +‬‬
‫‪ 𝐼0‬شدة التيار األعظمي‬
‫‪2‬‬
‫‪0.25‬‬ ‫= ‪𝑢𝑏 (𝑡 = ∞) = 𝑟𝐼0 = 2 ⇒ 𝐼0‬‬ ‫𝐴‪⇒ 𝐼0 = 0,08‬‬
‫‪25‬‬
‫الجزء الثاني‪:‬‬
‫‪ -1‬ارفاق كل تجربة بالبيان الموافق مع التعليل‪:‬‬
‫التجربة الثانية‪:‬‬ ‫التجربة األولى‪:‬‬
‫‪2𝐿0‬‬ ‫‪𝐿0‬‬
‫= ‪𝜏2‬‬ ‫= ‪𝜏1‬‬
‫𝟎𝟓 ‪𝟎.‬‬ ‫‪0.25‬‬ ‫𝑟‪𝑅0 +‬‬ ‫𝑟‪2𝑅0 +‬‬
‫‪ 𝜏2 > 𝜏1‬وبالتالي‪:‬‬
‫التجربة األولى يمثلها المنحنى البياني ) ‪(𝑏 ′‬‬
‫‪0.25‬‬
‫التجربة الثاني يمثلها المنحنى البياني ) ‪(𝑎′‬‬
‫‪ -2‬المالحظة عن استقرار التيار الكهربائي في كل تجربة‪:‬‬
‫بالنسبة للتجربة األول‪ :‬المقاومة تضاعفت وبالتالي‬
‫𝐸‬
‫= ‪𝐼01‬‬ ‫‪< 𝐼0‬‬
‫{‬ ‫𝑟 ‪2𝑅0 +‬‬
‫𝜏 < ‪𝜏1‬‬
‫𝟎𝟓 ‪𝟎.‬‬ ‫‪0.25‬‬ ‫ومنه التيار الكهربائي يستغرق زمنا أقل من أجل بلوغ قيمته األعظمية‪.‬‬
‫بالنسبة للتجربة الثانية‪ :‬المقاومة تضاعفت وبالتالي‬
‫𝐸‬
‫= ‪𝐼02‬‬ ‫‪= 𝐼0‬‬
‫{‬ ‫𝑟 ‪𝑅0 +‬‬
‫𝜏‪𝜏2 = 2‬‬
‫‪0.25‬‬ ‫ومنه التيار الكهربائي يستغرق زمنا مضاعفا من أجل بلوغ قيمته األعظمية‪.‬‬

‫الصفحة ‪ 4‬من ‪11‬‬


‫األستاذ‪ :‬مكي يونس‬ ‫الشعبة‪ :‬تقني رياضي المؤسسة‪ :‬ثانوية االخوة عزوزي المشرية‬ ‫اإلجابة النموذجية‪ :‬للبكالوريا التجريبي‬

‫التمرين الثالث (‪ 46‬نقاط)‪.‬‬ ‫‪-‬‬


‫‪ -1‬معادلة التفكك الحادث‪:‬‬
‫‪243‬‬ ‫‪239‬‬ ‫𝐴‬
‫𝑚𝐶‪96‬‬ ‫→‬ ‫𝑢𝑃‪94‬‬ ‫‪+‬‬ ‫𝑋𝑍‬
‫من قانوني احانحفاظ لصودي‪:‬‬
‫𝟎𝟓 ‪𝟎.‬‬ ‫‪243 = 239 + 𝐴 ⇒ 𝐴 = 4‬‬
‫‪0.25‬‬ ‫{‬
‫‪96 = 94 + 𝑍 ⇒ 𝑍 = 2‬‬
‫‪243‬‬ ‫‪239‬‬ ‫‪4‬‬
‫𝑒𝐻‪96𝐶𝑚 → 94𝑃𝑢 + 2‬‬
‫‪0.25‬‬
‫طبيعة الجسيم الصادر‪ :‬نواة الهيليوم 𝑒𝐻‪2‬‬
‫‪4‬‬

‫‪ -2‬إثبات العبارة 𝒃 ‪:𝒍𝒏(𝒎) = 𝒂. 𝒕 +‬‬


‫‪0.25‬‬ ‫من قانون التناقص احاشعاعي 𝑡‪𝑁(𝑡) = 𝑁0 𝑒 −𝜆 .‬‬
‫)𝑡(𝑚‬ ‫‪𝑚0‬‬
‫𝟓𝟕 ‪𝟎.‬‬ ‫‪0.25‬‬ ‫= 𝐴𝑁 ‪.‬‬ ‫𝑡 ‪. 𝑁𝐴 𝑒 −𝜆 .𝑡 ⇒ 𝑚(𝑡) = 𝑚0 𝑒 −𝜆 .𝑡 ⇒ 𝑙𝑛(𝑚) = 𝑙𝑛(𝑚0 ) − 𝜆 .‬‬
‫𝑀‬ ‫𝑀‬
‫) ‪⇒ 𝑙𝑛(𝑚) = −𝜆 . 𝑡 + 𝑙𝑛(𝑚0‬‬
‫‪0.25‬‬
‫) ‪ 𝑏 = 𝑙𝑛(𝑚0‬و 𝜆‪𝑎 = −‬‬
‫‪ -3‬اعتمادا على البيان تحديد‪:‬‬
‫الكتلة احابتدائية ‪:𝑚0‬‬
‫معادلة البيان‪𝑙𝑛(𝑚) = 𝑎′ . 𝑡 + 2,302 :‬‬
‫بالمطابقة مع العالقة النظرية نجد‪:‬‬
‫‪0.25‬‬ ‫‪𝑙𝑛(𝑚0 ) = 2,302 ⇒ 𝑚0 = 𝑒 2,302 ⇒ 𝑚0 = 10g‬‬
‫ثابت التفكك احاشعاعي 𝜆‪ ،‬و استنتاج ‪𝑡1/2‬‬
‫‪2,302 − 1,5‬‬
‫= ‪𝑎′‬‬ ‫‪7‬‬
‫‪= −7,53 × 10−10 𝑠 −1‬‬
‫𝟓𝟐 ‪𝟏.‬‬ ‫‪(0 − 53,2 × 2). 10‬‬
‫بالمطابقة مع العالقة النظرية نجد‪:‬‬
‫‪0.25‬‬ ‫‪−𝜆 = 𝑎′ ⇒ 𝜆 = 7,53 × 10−10 𝑠 −1‬‬
‫‪𝑙𝑛2‬‬ ‫‪0,693‬‬
‫‪0.25‬‬ ‫‪𝑡1/2‬‬ ‫=‬ ‫=‬ ‫‪−10‬‬
‫𝑠𝑛𝑎‪= 9,20 × 108 𝑠 ⇒ 𝑡1/2 = 29,1‬‬
‫𝜆‬ ‫‪7,53 × 10‬‬
‫عدد األنوية المتبقية في العينة عند اللحظة ‪𝑡 = 3𝑡1/2‬‬
‫‪𝑁0‬‬ ‫‪𝑚0‬‬ ‫‪10‬‬
‫‪0.25‬‬ ‫= ‪𝑁(3𝑡1/2 ) = 3‬‬ ‫= 𝐴𝑁‬ ‫⇒ ‪. 6,023. 1023‬‬
‫‪2‬‬ ‫𝑀 ‪8.‬‬ ‫‪8.243‬‬
‫‪0.25‬‬ ‫𝑥𝑢𝑎𝑦𝑜𝑁 ‪𝑁(3𝑡1/2 ) = 3,1 × 1021‬‬
‫‪ -1‬استنتاج قيمتي كل من 𝒛 و𝒚‪:‬‬
‫من قانوني احانحفاظ لصودي‪:‬‬
‫𝟎𝟓 ‪𝟎.‬‬ ‫‪239 + 1 = 130 + 107 + 𝑦 ⇒ 𝑦 = 6‬‬
‫‪0.25‬‬ ‫{‬
‫‪0.25‬‬ ‫‪94 = 51 + 𝑍 ⇒ 𝑍 = 43‬‬
‫‪239‬‬ ‫‪1‬‬ ‫‪130‬‬ ‫‪107‬‬ ‫‪1‬‬
‫𝑛‪94𝑃𝑢 + 0𝑛 ⟶ 51𝑆𝑏 + 43𝑇𝑐 + 3 0‬‬

‫الصفحة ‪ 5‬من ‪11‬‬


‫األستاذ‪ :‬مكي يونس‬ ‫الشعبة‪ :‬تقني رياضي المؤسسة‪ :‬ثانوية االخوة عزوزي المشرية‬ ‫اإلجابة النموذجية‪ :‬للبكالوريا التجريبي‬

‫‪ -2‬حساب الطاقة المحررة من هذا االنشطار‪:‬‬


‫⇒ ‪∆𝐸 = 𝐸𝐿𝑖𝑏 = ∆𝑚. 𝑐 2‬‬
‫‪130‬‬
‫‪0.25‬‬ ‫𝑏𝑖𝐿𝐸‬ ‫‪= [𝑚( 239‬‬ ‫‪1‬‬ ‫‪107‬‬ ‫‪1‬‬
‫‪94𝑃𝑢 ) + 𝑚( 0𝑛) − 𝑚( 51𝑆𝑏) − 𝑚( 43𝑇𝑐 ) − 3𝑚( 0𝑛)] × 931,5‬‬
‫𝟓𝟕 ‪𝟎.‬‬ ‫‪𝐸𝐿𝑖𝑏 = (239 , 0521 − 129,8836 − 106,8915 − 2 × 1,00866) × 931,5‬‬
‫𝑉𝑒𝑀‪𝐸𝐿𝑖𝑏 = 241,89‬‬
‫‪0.25‬‬
‫𝐽 ‪𝐸𝐿𝑖𝑏 = 241,89 × 1,6 × 10−13 ⇒ 𝐸𝐿𝑖𝑏 = 3,87 × 10−11‬‬
‫‪0.25‬‬
‫‪ -3‬حساب طاقة الربط 𝒍𝑬‪:‬‬
‫‪0.25‬‬ ‫‪𝐸𝑙 = [𝑍𝑚𝑝 + (𝐴 − 𝑍)𝑚𝑛 − 𝑚( 𝐴𝑍𝑋)]. 𝑐 2‬‬
‫‪𝐸𝑙 ( 130‬‬
‫‪51𝑆𝑏 ) = (51 × 1,0073 + 79 × 1,00866 − 129,8836). 931,5‬‬

‫‪0.25‬‬ ‫‪𝐸𝑙 ( 130‬‬


‫𝑉𝑒𝑀‪51𝑆𝑏) = 1092,50‬‬
‫‪𝐸𝑙 ( 107‬‬
‫‪43𝑇𝑐 ) = (43 × 1,0073 + 64 × 1,00866 − 106,8915). 931,5‬‬
‫‪0.25‬‬ ‫‪𝐸𝑙 ( 107‬‬
‫𝟎𝟓 ‪𝟏.‬‬ ‫𝑉𝑒𝑀‪43𝑇𝑐 ) = 909,74‬‬

‫تحديد النواة األكثر استقرار‬


‫𝑙𝐸‬
‫=𝜉‬
‫𝐴‬
‫‪𝐸𝑙 130‬‬ ‫‪1092,50‬‬
‫‪0.25‬‬ ‫= )𝑏𝑆‪( 51‬‬ ‫𝑛𝑜‪= 8,40𝑀𝑒𝑉/𝑁𝑢𝑐𝑙é‬‬
‫𝐴‬ ‫‪130‬‬
‫‪𝐸𝑙 107‬‬ ‫‪909,74‬‬
‫‪0.25‬‬ ‫= )𝑐𝑇‪( 43‬‬ ‫𝑛𝑜‪= 8,50𝑀𝑒𝑉/𝑁𝑢𝑐𝑙é‬‬
‫𝐴‬ ‫‪107‬‬
‫رر هي‪43𝑇𝑐 :‬‬
‫النواة األكثر استق ا‬
‫‪107‬‬
‫‪0.25‬‬

‫‪ -0‬الطاقة المحررة من انشطار العينة 𝒍𝒐𝒎𝟓 ‪:𝒏 = 𝟎,‬‬


‫‪0.25‬‬ ‫𝑏𝑖𝐿𝐸 ‪𝐸𝑇 = 𝑁. 𝐸𝐿𝑖𝑏 = 𝑛. 𝑁𝐴 .‬‬
‫𝟎𝟓 ‪𝟎.‬‬ ‫‪𝐸𝑇 = 0,5 × 3,87 × 10−11 × 6,023. 1023‬‬
‫‪0.25‬‬ ‫𝐽 ‪𝐸𝑇 = 1,17 × 1013‬‬

‫‪ -5‬عدد قارورات غاز البوتان التي نفس الطاقة عند احتراقها‪:‬‬


‫𝐽 ‪1𝑚𝑜𝑙(𝐶4 𝐻10 ) ⟶ 2800 × 103‬‬ ‫‪1,17 × 1013‬‬
‫{‬ ‫= ‪⇒ 𝑛(𝐶4 𝐻10‬‬ ‫)‬ ‫⇒‬
‫𝐽 ‪𝑛(𝐶4 𝐻10 ) ⟶ 1,17 × 1013‬‬ ‫‪2800 × 103‬‬
‫‪0.25‬‬
‫𝑙𝑜𝑚 ‪𝑛(𝐶4 𝐻10 ) = 4,18 × 106‬‬
‫استنتاج عدد قارورات الغاز‬
‫𝟓𝟕 ‪𝟎.‬‬
‫) ‪𝑚(𝐶4 𝐻10 ) = 𝑀𝑛(𝐶4 𝐻10 ) × 𝑛(𝐶4 𝐻10‬‬
‫‪0.25‬‬ ‫‪𝑚(𝐶4 𝐻10 ) = 58 × 4,18 × 106 = 2,42 × 108 g‬‬
‫‪(𝐶4 𝐻10 ) ⟶ 13 × 103 g‬قارورة‪1‬‬ ‫‪2,42 × 108‬‬
‫{‬ ‫= ) ‪(𝐶4 𝐻10‬قارورة𝑥 ⇒‬ ‫⇒‬
‫‪(𝐶4 𝐻10 ) ⟶ 2,42 × 108 g‬قارورة𝑥‬ ‫‪13 × 103‬‬
‫‪0.25‬‬
‫قارورة ‪𝑥(𝐶4 𝐻10 ) = 186 × 102‬‬

‫الصفحة ‪ 1‬من ‪11‬‬


‫األستاذ‪ :‬مكي يونس‬ ‫الشعبة‪ :‬تقني رياضي المؤسسة‪ :‬ثانوية االخوة عزوزي المشرية‬ ‫اإلجابة النموذجية‪ :‬للبكالوريا التجريبي‬

‫التمرين التجريبي (‪ 46‬نقاط)‪.‬‬ ‫‪-‬‬


‫التجربة األولى‪:‬‬
‫𝟎𝟓 ‪𝟎.‬‬
‫‪ -1‬حساب التركيز المولي 𝟐𝒄‪:‬‬
‫‪2 × 0.25‬‬
‫𝑚‬ ‫𝑚‬ ‫‪1,25‬‬
‫= ‪𝑛 = 𝑐2 . 𝑉2‬‬ ‫= ‪⇒ 𝑐2‬‬ ‫=‬ ‫𝑙‪⇒ 𝑐2 = 0,74𝑚𝑜𝑙/‬‬
‫𝑀‬ ‫‪𝑀. 𝑉2 84 × 20 × 10−3‬‬
‫‪ -2‬جدول لتقدم التفاعل‪:‬‬
‫‪−‬‬ ‫‪−‬‬
‫معادلة التفاعل‬ ‫‪𝐶𝐻3 𝐶𝑂𝑂𝐻(𝑎𝑞) +‬‬ ‫)𝑞𝑎(‪𝐻𝐶𝑂3‬‬ ‫=‬ ‫)𝑞𝑎(𝑂𝑂𝐶 ‪𝐶𝐻3‬‬ ‫)𝑙(𝑂 ‪+ 𝐶𝑂2(g) + 𝐻2‬‬

‫الحالة‬ ‫التقدم‬ ‫كميات المادة بالمول)𝑙𝑜𝑚(‬


‫𝟓𝟐 ‪𝟎.‬‬
‫‪0.50‬‬
‫ابتدائية‬ ‫‪0‬‬ ‫‪𝑛1 = 𝑐1 . 𝑉1‬‬ ‫‪𝑛2 = 𝑐2 . 𝑉2‬‬ ‫‪0‬‬ ‫‪0‬‬ ‫بزيادة‬
‫وسطية‬ ‫𝑥‬ ‫𝑥 ‪𝑛1 −‬‬ ‫𝑥 ‪𝑛2 −‬‬ ‫𝑥‬ ‫𝑥‬ ‫بزيادة‬
‫𝑥𝑎𝑚𝑥 نهائية‬ ‫𝑥𝑎𝑚𝑥 ‪𝑛1 −‬‬ ‫𝑥𝑎𝑚𝑥 ‪𝑛2 −‬‬ ‫𝑥𝑎𝑚𝑥‬ ‫𝑥𝑎𝑚𝑥‬ ‫بزيادة‬

‫‪ -3‬عبارة التقدم𝒙 ‪:‬‬


‫من جدول التقدم وقانون الغازات المثالية‪:‬‬
‫𝑥 = ‪𝑛𝐶𝑂2‬‬ ‫‪𝑃𝐶𝑂2 . 𝑉𝐶𝑂2‬‬
‫‪𝟎. 𝟓𝟎 2 × 0.25‬‬ ‫= 𝑥 ⇒ 𝑇 ‪{𝑃 . 𝑉 = 𝑛 . 𝑅.‬‬
‫‪𝐶𝑂2 𝐶𝑂2‬‬ ‫‪𝐶𝑂2‬‬ ‫𝑇 ‪𝑅.‬‬

‫‪ -0‬اثبات عبارة 𝒚 ‪:‬‬


‫نعلم أن‪:‬‬
‫𝑡] ‪𝑦 = [𝐶𝐻3 𝐶𝑂𝑂𝐻]𝑡 − [𝐶𝐻3 𝐶𝑂𝑂−‬‬
‫من جدول التقدم‪:‬‬

‫𝑥 ‪𝑐1 . 𝑉1 −‬‬
‫𝟎𝟓 ‪𝟎.‬‬ ‫= 𝑡]𝐻𝑂𝑂𝐶 ‪[𝐶𝐻3‬‬
‫‪0.25‬‬ ‫𝑇𝑉‬ ‫𝑥 𝑥 ‪𝑐1 . 𝑉1 −‬‬ ‫‪𝑐1 . 𝑉1‬‬ ‫𝑥‬
‫𝑥‬ ‫⇒‬ ‫𝑦‬ ‫=‬ ‫‪−‬‬ ‫=‬ ‫‪−‬‬ ‫‪2‬‬
‫= 𝑡] ‪[𝐶𝐻3 𝐶𝑂𝑂−‬‬ ‫𝑇𝑉‬ ‫𝑇𝑉‬ ‫𝑇𝑉‬ ‫𝑇𝑉‬
‫{‬ ‫𝑇𝑉‬
‫بتعويض عبارة 𝑥 في العبارة األخيرة نجد‪:‬‬
‫‪𝑃𝐶𝑂2 . 𝑉𝐶𝑂2‬‬
‫‪0.25‬‬ ‫‪𝑐1 . 𝑉1‬‬ ‫𝑇 ‪𝑅.‬‬ ‫‪𝑐1 . 𝑉1‬‬ ‫‪𝑉𝐶𝑂2‬‬
‫=𝑦‬ ‫‪−2‬‬ ‫=𝑦 ⇒‬ ‫‪−2‬‬ ‫𝑃‬
‫𝑇𝑉‬ ‫𝑇𝑉‬ ‫𝑇𝑉‬ ‫‪𝑉𝑇 . 𝑅. 𝑇 𝐶𝑂2‬‬
‫‪ -5‬تحديد حجم غاز ثنائي أكسيد الكربون 𝟐𝑶𝑪𝑽 والتركيز المولي 𝟏𝒄‪:‬‬
‫𝑏 ‪𝑦 = 𝑎. 𝑃𝐶𝑂2 +‬‬ ‫معادلة البيان‪:‬‬
‫‪0,75 − 0,375‬‬
‫‪0.25‬‬ ‫=𝑎‬ ‫𝑎𝑝 ‪= −1,36 × 10−5 𝑚𝑜𝑙/𝐿.‬‬
‫{‬ ‫‪0 − 27,5 × 10−3‬‬
‫𝐿‪𝑏 = 0,75𝑚𝑜𝑙/‬‬

‫الصفحة ‪ 7‬من ‪11‬‬


‫األستاذ‪ :‬مكي يونس‬ ‫الشعبة‪ :‬تقني رياضي المؤسسة‪ :‬ثانوية االخوة عزوزي المشرية‬ ‫اإلجابة النموذجية‪ :‬للبكالوريا التجريبي‬

‫بالمطابقة مع العالقة البيانية مع عالقة السؤال ‪4‬‬


‫‪𝑉𝐶𝑂2‬‬ ‫‪𝑎. 𝑉𝑇 . 𝑅. 𝑇 −1,36 × 10−5 × 0,08 × 298 × 8,314‬‬
‫‪𝑎 = −2‬‬ ‫= ‪⇒ 𝑉𝐶𝑂2‬‬ ‫=‬
‫𝑇 ‪𝑉𝑇 . 𝑅.‬‬ ‫‪−2‬‬ ‫‪−2‬‬
‫𝟓𝟕 ‪𝟎.‬‬ ‫‪𝑐1 . 𝑉1‬‬ ‫‪𝑏. 𝑉𝑇 0,75 × 0,08‬‬
‫=𝑏‬ ‫= ‪⇒ 𝑐1‬‬ ‫=‬
‫{‬ ‫𝑇𝑉‬ ‫‪𝑉1‬‬ ‫‪0,06‬‬
‫‪0.25‬‬
‫‪𝑉𝐶𝑂2 = 1,35 × 10−3 𝑚3‬‬
‫‪0.25‬‬ ‫{‬
‫𝐿‪𝑐1 = 1𝑚𝑜𝑙/‬‬
‫‪ -1-6‬تحديد السلم الناقص في الرسم‪:‬‬
‫) ‪𝑃𝑓 (𝐶𝑂2‬‬ ‫‪27,5‬‬
‫‪0.25‬‬ ‫⟶ 𝑚𝑐‪1‬‬ ‫=‬ ‫𝑎𝑃 ‪= 10 × 103‬‬
‫عدد التدريجات‬ ‫‪2,75‬‬

‫‪ -2-6‬المتفاعل المحد‪:‬‬
‫نفرض أن 𝐻𝑂𝑂𝐶 ‪𝐶𝐻3‬هو المتفاعل المحد‬
‫𝟓𝟕 ‪𝟎.‬‬ ‫𝑙𝑜𝑚𝑚‪𝑐1 . 𝑉1 − 𝑥𝑚𝑎𝑥 = 0 ⇒ 𝑥𝑚𝑎𝑥 = 1 × 60 = 60‬‬
‫‪0.25‬‬ ‫نفرض أن ‪𝐻𝐶𝑂3−‬هو المتفاعل المحد‬
‫𝑙𝑜𝑚𝑚‪𝑐2 . 𝑉2 − 𝑥𝑚𝑎𝑥 = 0 ⇒ 𝑥𝑚𝑎𝑥 = 0,74 × 20 = 1,48‬‬
‫‪0.25‬‬ ‫ومنه المتفاعل المحد هو‪𝐻𝐶𝑂3− :‬‬
‫قيمة التقدم األعظمي‪𝑥𝑚𝑎𝑥 = 1,48𝑚𝑚𝑜𝑙 :‬‬
‫‪ -7‬إثبات عبارة السرعة الحجمية‪:‬‬
‫نعلم ان‪:‬‬
‫‪1‬‬ ‫𝑥𝑑‬
‫‪0.25‬‬ ‫= 𝑙𝑜𝑉𝑣‬ ‫‪.‬‬
‫𝑡𝑑 𝑇𝑉‬
‫‪𝑃𝐶𝑂2 .𝑉𝐶𝑂2‬‬
‫= 𝑥 بالنسبة للزمن‬ ‫نشتق العبارة‬
‫𝑇‪𝑅.‬‬
‫𝟓𝟕 ‪𝟎.‬‬ ‫‪𝑑𝑥 𝑉𝐶𝑂2 𝑑𝑃𝐶𝑂2‬‬
‫=‬ ‫‪.‬‬
‫‪0.25‬‬ ‫𝑡𝑑 𝑇 ‪𝑑𝑡 𝑅.‬‬
‫بتعويض العبارة الخيرة في عبارة السرعة الحجمية نجد‪:‬‬
‫‪1 𝑉𝐶𝑂2 𝑑𝑃𝐶𝑂2‬‬ ‫‪𝑉𝐶𝑂2 𝑑𝑃𝐶𝑂2‬‬
‫= 𝑙𝑜𝑉𝑣‬ ‫‪.‬‬ ‫=‬ ‫‪.‬‬
‫𝑡𝑑 𝑇 ‪𝑉𝑇 𝑅.‬‬ ‫𝑡𝑑 𝑇 ‪𝑉𝑇 𝑅.‬‬
‫حساب قيمتها عند اللحظة 𝒔𝟎𝟎𝟏 = 𝒕‬
‫‪1,35 × 10−3‬‬ ‫‪(20 − 10). 103‬‬
‫𝑡(‬
‫= )𝑠‪𝑣𝑉𝑜𝑙 = 100‬‬ ‫‪.‬‬ ‫=‬
‫‪80 × 10−3 × 8,314 × 298‬‬ ‫‪100 − 0‬‬
‫‪0.25‬‬ ‫𝑠 ‪𝑣𝑉𝑜𝑙 (𝑡 = 100𝑠) = 6,81 × 10−4 𝑚𝑜𝑙/𝐿.‬‬

‫‪ -8‬تحديد قيمة زمن نصف التفاعل 𝟐‪:𝒕𝟏/‬‬


‫‪0.25‬‬ ‫‪𝑃𝑓 27,5 × 103‬‬
‫𝟎𝟓 ‪𝟎.‬‬ ‫= = ) ‪𝑃(𝑡1/2‬‬ ‫𝑎𝑃 ‪= 13,75 × 103‬‬
‫‪2‬‬ ‫‪2‬‬
‫‪0.25‬‬ ‫𝑠‪𝑡1/2 = 60‬‬ ‫باإلسقاط على المنحنى نجد‪:‬‬

‫الصفحة ‪ 8‬من ‪11‬‬


‫األستاذ‪ :‬مكي يونس‬ ‫الشعبة‪ :‬تقني رياضي المؤسسة‪ :‬ثانوية االخوة عزوزي المشرية‬ ‫اإلجابة النموذجية‪ :‬للبكالوريا التجريبي‬

‫التجربة الثانية‪:‬‬
‫𝟎𝟓 ‪𝟎.‬‬
‫‪2 × 0.25‬‬ ‫‪ -1‬تحديد إحداثيات نقطة التكافؤ) 𝑬𝑯𝒑 ‪:(𝑽𝒃𝑬 ,‬‬
‫‪𝑉𝑏𝐸 = 22,5𝑚𝐿 ، 𝑝𝐻𝐸 = 9‬‬ ‫باستعمال المماسات نجد‪:‬‬
‫‪ -2‬حساب التركيز المولي 𝒂𝒄 للحمض‪:‬‬
‫عند نقطة التكافؤ يكون المزيج بنسب ستوكيومترية‪:‬‬
‫‪−1‬‬
‫𝟎𝟓 ‪𝟎.‬‬ ‫‪𝑐𝑏 . 𝑉𝑏𝐸 5 × 10 × 22,5‬‬ ‫‪−1‬‬
‫‪2 × 0.25‬‬ ‫𝐸𝑏𝑉 ‪𝑐𝑎 . 𝑉𝑎 = 𝑐𝑏 .‬‬ ‫= 𝑎𝑐 ⇒‬ ‫=‬ ‫𝐿‪⇒ 𝑐𝑎 = 5,6 × 10 𝑚𝑜𝑙/‬‬
‫𝑎𝑉‬ ‫‪20‬‬

‫‪ -3‬تحديد قيمة ال𝒂𝑲𝒑 ‪:‬‬


‫عند نقطة نصف التكافؤ ‪𝑝𝐻 = 𝑝𝐾𝑎 : 𝐸 ′‬‬
‫‪0.25‬‬ ‫‪𝑉𝑏𝐸 22,5‬‬
‫‪𝑉𝑏𝐸′‬‬ ‫=‬ ‫=‬ ‫𝐿𝑚‪= 11,25‬‬
‫‪2‬‬ ‫‪2‬‬
‫𝟎𝟓 ‪𝟎.‬‬ ‫‪0.25‬‬ ‫‪𝑝𝐻 = 𝑝𝐾𝑎 = 4,8‬‬ ‫باإلسقاط على المنحنى نجد‪:‬‬

‫الصفحة ‪ 9‬من ‪11‬‬


‫األستاذ‪ :‬مكي يونس‬ ‫الشعبة‪ :‬تقني رياضي المؤسسة‪ :‬ثانوية االخوة عزوزي المشرية‬ ‫اإلجابة النموذجية‪ :‬للبكالوريا التجريبي‬

‫العالمة‬
‫عناصر اإلجابة الموضوع الثاني‬
‫مجموع‬ ‫مجزأة‬
‫التمرين األول (‪ 40‬نقاط)‪.‬‬ ‫‪-‬‬
‫الجزء األول‪:‬‬
‫‪ -1‬مخططا للدارة التي تسمح بشحن المكثفة‪:‬‬

‫𝟓𝟐 ‪𝟎.‬‬ ‫‪0.25‬‬ ‫‪+‬‬


‫𝑪‬
‫𝑬‬

‫𝑹‬

‫‪ -1-2‬تبيان أن الجداء 𝑪 ‪ 𝑹.‬متجانس مع الزمن‪.‬‬


‫𝑢‬ ‫]𝑢[‬
‫= ]𝑅[ ⇒ = 𝑅 ⇒ 𝑖 ‪𝑢𝑅 = 𝑅.‬‬ ‫]𝑇[]𝑖[ ]𝑢[‬
‫‪2 × 0.25‬‬ ‫𝑖‬ ‫]𝑖[‬
‫= ]𝑅[ ‪⇔ [𝑐].‬‬ ‫]𝑇[ =‬
‫]𝑇[]𝑖[‬ ‫]𝑢[ ]𝑖[‬
‫= ]𝑐[ ⇒ 𝑡 ‪𝑞 = 𝐶. 𝑢𝐶 = 𝑖.‬‬
‫{‬ ‫]𝑢[‬
‫𝟎𝟎 ‪𝟏.‬‬ ‫إذن الجداء 𝑪𝑹 متجانس مع الزمن وحدته الثانية )𝒔(‬
‫‪ -2-2‬عبارة )𝒕( 𝑪𝒖‪:‬‬
‫‪0.25‬‬ ‫𝐸 = 𝑅𝑢 ‪𝑢𝐶 +‬‬ ‫من قانون جمع التوترات‪:‬‬
‫‪0.25‬‬
‫) 𝜏‪𝑢𝐶 = 𝐸 − 𝑢𝑅 = 𝐸 − 𝐸𝑒 −𝑡/𝜏 ⇒ 𝑢𝐶 (𝑡) = 𝐸(1 − 𝑒 −𝑡/‬‬

‫‪ -3‬عبارة )𝒕( 𝑪𝑬‪:‬‬


‫‪1‬‬
‫‪𝟎. 𝟓𝟎 2 × 0.25‬‬ ‫)𝑡( ‪𝐸𝐶 (𝑡) = 𝐶. 𝑢𝐶 2‬‬ ‫‪1‬‬ ‫‪2‬‬
‫{‬ ‫‪2‬‬ ‫) 𝜏‪⇒ 𝐸𝐶 (𝑡) = 𝐶. 𝐸 2 (1 − 𝑒 −𝑡/‬‬
‫‪2‬‬
‫) 𝜏‪𝑢𝐶 (𝑡) = 𝐸(1 − 𝑒 −𝑡/‬‬

‫‪ -1-0‬تحديد ثابت الزمن 𝝉‪:‬‬


‫‪0.25‬‬ ‫𝑥𝑎𝑚‪𝐸𝐶 (𝜏) = 0,4. 𝐸𝐶,‬‬ ‫𝐽 ‪= 0,4.324.5. 10 = 624. 10−6‬‬
‫‪−6‬‬

‫‪0.25‬‬ ‫𝑠‪𝜏 = 2‬‬ ‫باإلسقاط نجد‪:‬‬


‫‪ -2-0‬تحديد سعة المكثفة𝑪‪:‬‬
‫𝟓𝟕 ‪𝟏.‬‬
‫𝜏‬ ‫‪2‬‬
‫‪2 × 0.25‬‬ ‫= 𝐶 ⇒ 𝐶 ‪𝜏 = 𝑅.‬‬ ‫=‬ ‫𝐹𝜇‪= 1,6. 10−4 𝐹 ⇒ 𝐶 = 160‬‬
‫‪𝑅 12,5. 103‬‬

‫‪0.25‬‬ ‫نعم تتوافق القيمة المحسوبة مع القيمة المسجلة من طرف الصانع‪.‬‬

‫الصفحة ‪ 11‬من ‪11‬‬


‫األستاذ‪ :‬مكي يونس‬ ‫الشعبة‪ :‬تقني رياضي المؤسسة‪ :‬ثانوية االخوة عزوزي المشرية‬ ‫اإلجابة النموذجية‪ :‬للبكالوريا التجريبي‬

‫‪ -3-0‬تحديد لقوة المحركة الكهربائية 𝑬 للمولد‪:‬‬

‫‪1‬‬ ‫𝑥𝑎𝑚‪𝐸𝐶,‬‬ ‫‪324.5. 10−6‬‬


‫‪2 × 0.25‬‬ ‫𝑥𝑎𝑚‪𝐸𝐶,‬‬ ‫‪2‬‬ ‫√‬
‫‪= 𝐶. 𝐸 ⇒ 𝐸 = 2.‬‬ ‫√‬
‫‪= 2‬‬ ‫𝑉‪⇒ 𝐸 = 4,5‬‬
‫‪2‬‬ ‫𝐶‬ ‫‪160. 10−6‬‬

‫‪0.25‬‬ ‫‪ -1-5‬قيمة ثابت الزمن 𝝉‪ :‬حا تتغير عند استعمال مولد قوته المحركة الكهربائية 𝑉‪،𝐸1 = 12‬‬
‫𝟎𝟓 ‪𝟎.‬‬ ‫ألن 𝜏 حا تتعلق ب 𝐸‪.‬‬
‫‪0.25‬‬ ‫‪ -2-5‬عند استعمال مولد 𝑽𝟎𝟑𝟑 > 𝟐𝑬‪ ،‬المكثفة تنفجر ألنه أعلى توتر تحتمله 𝑉‪.330‬‬
‫التمرين الثاني (‪ 40‬نقاط)‪.‬‬ ‫‪-‬‬
‫الجزء األول‪:‬‬
‫‪ -1‬التعريفات‪:‬‬
‫𝟎𝟓 ‪𝟎.‬‬ ‫‪0.25‬‬ ‫‪ ‬النظائر‪ :‬هي ذرات تنتمي لنفس العنصر الكيميائي لها نفس الرقم الذري𝑍 وتختلف في عدد‬
‫النيوترونات 𝑁‪.‬‬
‫‪0.25‬‬ ‫‪ ‬التفكك ‪ :𝛽 −‬عبارة عن إلكترون 𝑒‪ ، −10‬ينتج عن تفكك أنوية لها فائض في النيترونات‪.‬‬
‫‪-0‬‬

‫𝑪𝟔𝟒𝟏 ‪:‬‬ ‫‪ -2‬معادلة تفكك النواة‬


‫‪14‬‬
‫𝐶‪6‬‬ ‫تقع فوق واد احاستقرار فإن نمط تفككها ‪ ، 𝛽 −‬وعليه‪⟶ 𝐴𝑍𝑋 + −10𝑒:‬‬ ‫𝟒𝟏‬
‫𝑪𝟔‬ ‫النواة‬
‫‪0.25‬‬
‫من قانوني احانحفاظ لصودي‪:‬‬
‫𝟓𝟕 ‪𝟎.‬‬
‫‪0.25‬‬ ‫𝐴 ‪14 = 𝐴 + 0 ⇒ 𝐴 = 14‬‬
‫{‬ ‫𝑁‪⇒ 𝑍𝑋 ≡ 147‬‬
‫‪6=𝑍−1⇒𝑍 =7‬‬
‫‪0.25‬‬ ‫‪14‬‬ ‫‪14‬‬ ‫‪0‬‬
‫𝑒‪6𝐶 ⟶ 7𝑁 + −1‬‬
‫‪ -1-3‬تحديد عمر عظام كل جمجمة‪:‬‬
‫‪ ‬جمجمة الرجل ‪"1‬اندير"‪:‬‬
‫من قانون التناقص احاشعاعي 𝑡‪𝑁(𝑡) = 𝑁0 𝑒 −𝜆 .‬‬
‫)𝑡(𝑁‬ ‫)𝑡(𝑁‬ ‫‪1‬‬ ‫‪𝑁0‬‬ ‫‪𝑡1/2‬‬ ‫‪𝑁0‬‬
‫‪0.25‬‬ ‫𝑛𝑙 ⇒ 𝑡‪= 𝑒 −𝜆 .‬‬ ‫𝑛𝑙 ‪= −𝜆 . 𝑡 ⇒ 𝑡 = .‬‬ ‫=‬ ‫𝑛𝑙 ‪.‬‬
‫‪𝑁0‬‬ ‫‪𝑁0‬‬ ‫𝜆‬ ‫‪𝑁(𝑡) 𝑙𝑛2‬‬ ‫)𝑡(𝑁‬
‫‪5730‬‬
‫‪0.25‬‬ ‫= ‪𝑡1‬‬ ‫𝑠𝑛𝑎‪𝑙𝑛60,98 ⇒ 𝑡1 = 33980‬‬
‫‪0,693‬‬
‫𝟓𝟐 ‪𝟏.‬‬ ‫‪ ‬جمجمة الرجل ‪" 2‬سبياند"‪:‬‬
‫‪5730‬‬
‫‪0.25‬‬ ‫= ‪𝑡2‬‬ ‫𝑠𝑛𝑎‪𝑙𝑛53,48 ⇒ 𝑡2 = 32895‬‬
‫‪0,693‬‬
‫‪ -2-3‬التعليق على النتائج‪:‬‬
‫‪0.25‬‬ ‫النتائج تتطابق مع األخبار المطبقة من طرف العلماء ألن عمر الجمجمتين يعود إلى ما قبل‬
‫‪ 03‬ألف سنة إلى ‪ 43‬ألف سنة‪.‬‬
‫‪ -3-3‬األمر ال يتعلق بجريمة قتل ألن‪ :‬الفارق الزمني بين الجمجمتين كبير‪.‬‬
‫‪0.25‬‬
‫𝑠𝑛𝑎‪𝑡1 − 𝑡2 = 32895 − 33980 = 1085‬‬
‫الجزء الثاني‪:‬‬
‫الصفحة ‪ 11‬من ‪11‬‬
‫األستاذ‪ :‬مكي يونس‬ ‫الشعبة‪ :‬تقني رياضي المؤسسة‪ :‬ثانوية االخوة عزوزي المشرية‬ ‫اإلجابة النموذجية‪ :‬للبكالوريا التجريبي‬

‫‪ -1‬طول الموجة𝛌 لإلشعاع الصادر‪:‬‬


‫𝟎𝟓 ‪𝟎.‬‬ ‫𝑐‬ ‫𝐶 ‪ℎ.‬‬ ‫‪6,62. 10−34 . 3. 108‬‬
‫‪Δ𝐸3⟶1‬‬ ‫= 𝜆 ⇒ ‪= 𝐸3 − 𝐸1 = ℎ.‬‬ ‫=‬
‫𝜆‬ ‫‪𝐸3 − 𝐸1 (−1,51 + 13,6). 1,6. 10−19‬‬
‫‪0.25‬‬ ‫𝑚 ‪𝜆 = 1,027. 10−7‬‬
‫‪0.25‬‬
‫‪ -2‬ذرة الهيدروجين يمكن أن تمتص طول الموجة 𝑚𝑛‪ λ𝑎 = 121,69‬ألن‪:‬‬
‫𝑐‬ ‫‪−34‬‬
‫‪3. 108‬‬
‫‪0.25‬‬ ‫𝑎⟶‪Δ𝐸1‬‬ ‫‪= ℎ. = 6,62. 10‬‬ ‫𝑉𝑒 ‪= 10,2‬‬
‫𝑎𝜆‬ ‫‪121,69. 10−9 . .1,6. 10−19‬‬
‫𝟓𝟕 ‪𝟎.‬‬
‫‪0.25‬‬ ‫𝑉𝑒 ‪Δ𝐸1⟶𝑎 = 𝐸𝑎 − 𝐸1 ⇒ 𝐸𝑎 = Δ𝐸1⟶𝑎 + 𝐸1 = 10,2 − 13,6 ⇒ 𝐸𝑎 = −3,4‬‬
‫بمقارنة النتيجة المتحصل عليها مع المخطط المبسط لمستويات الطاقة لذرة الهيدروجين‪ ،‬هي قيمة‬
‫‪0.25‬‬
‫موجودة‪ ،‬وبالتالي ذرة الهيدروجين امتصت الفوتون‪.‬‬

‫‪ -3‬الحالة التي تتواجد عليها ذرة الهيدروجين بعد امتصاص الفوتون‪:‬‬


‫𝟓𝟐 ‪𝟎.‬‬ ‫‪0.25‬‬
‫الحالة المثارة‪.‬‬

‫التمرين الثالث‪ 46( :‬نقاط)‬ ‫‪-‬‬


‫الجزء األول‪:‬‬
‫‪ -1‬تمثيل القوى الخارجية المطبقة مركز عطالة الطفل )𝑮(‪:‬‬
‫‪𝒙′‬‬
‫𝟓𝟕 ‪𝟎.‬‬ ‫𝒇⃗‬
‫𝑨‬ ‫𝑹⃗‬
‫⃗‬
‫‪3 × 0.25‬‬

‫𝑷⃗‬
‫⃗‬ ‫𝒙‬
‫𝑩‬

‫‪ -1-2‬طبيعة حركة مركز عطالة الطفل )𝑮(‪:‬‬


‫‪𝟎. 𝟓𝟎 2 × 0.25‬‬
‫من البيان‪ 𝑎𝐺 = 2 𝑚. 𝑠 −2 :‬أي أن ‪ 𝑎𝐺 > 0‬و ‪ ،𝑎𝐺 . 𝑣 > 0‬والمسار )𝐵𝐴( مستقيم فإن‪:‬‬
‫الحركة مستقيمة متسارعة بانتظام‪.‬‬
‫‪ -3‬المعادلة الزمنية للسرعة)𝒕(𝒗‪:‬‬
‫‪0.25‬‬ ‫)𝑡(𝑣𝑑‬
‫= )𝑡(𝑎‬ ‫𝐴𝑣 ‪⇒ 𝑣(𝑡) = 𝑎. 𝑡 +‬‬
‫𝑡𝑑‬
‫𝟓𝟕 ‪𝟎.‬‬ ‫من الشروط احابتدائية‪ 𝑣𝐴 = 0 𝑚. 𝑠 −1 :‬ومن البيان ‪𝑎𝐺 = 2 𝑚. 𝑠 −2‬‬
‫‪0.25‬‬ ‫𝑡 ‪𝑣(𝑡) = 2.‬‬

‫الصفحة ‪ 12‬من ‪11‬‬


‫األستاذ‪ :‬مكي يونس‬ ‫الشعبة‪ :‬تقني رياضي المؤسسة‪ :‬ثانوية االخوة عزوزي المشرية‬ ‫اإلجابة النموذجية‪ :‬للبكالوريا التجريبي‬

‫قيمة 𝑩𝒗 سرعة الطفل‪:‬‬


‫‪0.25‬‬ ‫من البيان‪𝑣𝐴 = 2 × 2,75 = 5,5𝑚. 𝑠 −1 ⇐ 𝑡𝐵 = 2,75 𝑠 :‬‬
‫‪ -0‬طول المستوي المائل 𝑩𝑨‪:‬‬
‫‪𝟎. 𝟓𝟎 2 × 0.25‬‬ ‫‪2‬‬ ‫‪2‬‬
‫‪𝑣𝐵2 − 𝑣𝐴2 (5,5)2 − 0‬‬
‫= 𝐵𝐴 ⇒ 𝐵𝐴 ‪𝑣𝐵 − 𝑣𝐴 = 2. 𝑎𝐺 .‬‬ ‫=‬ ‫𝑚‪⇒ 𝐴𝐵 = 7,56‬‬
‫𝐺𝑎 ‪2.‬‬ ‫‪2×2‬‬
‫‪ -5‬الحصيلة الطاقوية للجملة (طفل ‪ +‬أرض) بين الموضعين 𝑨 و 𝑩‪:‬‬

‫𝑨𝒑𝒑𝑬‬
‫𝑩𝑪𝑬‬
‫)𝑓(𝑊‬
‫‪2 × 0.25‬‬

‫𝑩𝒑𝒑𝑬‬
‫𝑨𝑪𝑬‬
‫)طفل ‪+‬أرض)‬
‫𝟎𝟎 ‪𝟏.‬‬
‫𝒇‪:‬‬
‫شدة قوة االحتكاك ⃗‬
‫بتطبيق مبدأ انحفاظ الطاقة‪:‬‬
‫‪0.25‬‬ ‫𝐵𝐶𝐸 = |)𝑓(𝑊| ‪𝐸𝐶𝐴 + 𝐸𝑃𝑃𝐴 − |𝑊(𝑓)| = 𝐸𝐶𝐵 + 𝐸𝑃𝑃𝐵 ⇒ 𝐸𝑃𝑃𝐴 −‬‬
‫‪𝑣𝐵2‬‬
‫‪1‬‬ ‫) ‪𝑚 (g. ℎ −‬‬
‫‪2‬‬ ‫‪2‬‬
‫= 𝑓 ⇒ 𝐵𝑣 ‪𝑚. g. ℎ − 𝑓. 𝐴𝐵 = 𝑚.‬‬
‫‪2‬‬ ‫𝐵𝐴‬
‫‪(5,5)2‬‬
‫‪30 (9,8 × 3,6 −‬‬
‫) ‪2‬‬
‫‪0.25‬‬ ‫=𝑓‬ ‫𝑁‪⇒ 𝑓 = 80‬‬
‫‪7,56‬‬

‫الجزء الثاني‪:‬‬
‫‪ -1‬نص القانون الثاني لنيوتن‪:‬‬
‫في مرجع غاليلي‪ ،‬المجموع الشعاعي للقوى الخارجية 𝑡𝑥𝑒𝐹 ∑ المطبقة على مركز عطالة جملة‬
‫𝟓𝟐 ‪𝟎.‬‬ ‫‪0.25‬‬
‫مادية يساوي في كل لحظة جداء كتلتها 𝑚 في شعاع تسارع مركز عطالتها ونكتب‪:‬‬
‫𝐺𝑎 ‪∑ 𝐹𝑒𝑥𝑡 = 𝑚.‬‬

‫‪ -1-2‬المعادلتين الزمنيتين للموضع )𝒕(𝒙 و )𝒕(𝒚‪:‬‬


‫الجملة‪ :‬الطفل‬ ‫‪-‬‬
‫المرجع‪ :‬سطحي أرضي نعتبره غاليليا‪.‬‬ ‫‪-‬‬
‫‪0.25‬‬
‫القوى الخارجية المؤثرة‪. 𝑃⃗ :‬‬ ‫‪-‬‬
‫بتطبيق القانون الثاني لنيوتن على مركز عطالة الجسم )𝑆(‪:‬‬ ‫‪-‬‬
‫𝑎 ‪∑ 𝐹𝑒𝑥𝑡 = 𝑚. 𝑎 ⇒ 𝑃⃗ = 𝑚.‬‬

‫الصفحة ‪ 13‬من ‪11‬‬


‫األستاذ‪ :‬مكي يونس‬ ‫الشعبة‪ :‬تقني رياضي المؤسسة‪ :‬ثانوية االخوة عزوزي المشرية‬ ‫اإلجابة النموذجية‪ :‬للبكالوريا التجريبي‬

‫بإسقاط العبارة الشعاعية على المحور )𝑥𝑜(‪:‬‬


‫𝑥𝑎 ‪0 = 𝑚.‬‬
‫بإسقاط العبارة الشعاعية على المحور )𝑦𝑜(‪:‬‬
‫‪−𝑃 = 𝑚. 𝑎𝑦 ⇒ 𝑎𝑦 = −g‬‬
‫‪0.25‬‬ ‫𝑥𝑣𝑑‬
‫= 𝑥𝑎‬ ‫𝛼𝑠𝑜𝑐 ‪= 0 ⇒ 𝑣𝑥 = 𝑐𝑡𝑒 = 𝑣0 .‬‬
‫𝑡𝑑‬
‫𝑦𝑣𝑑‬
‫= 𝑦𝑎‬ ‫𝛼𝑛𝑖𝑠 ‪= −g ⇒ 𝑣𝑦 = 𝑎𝑦 . 𝑡 + 𝑐1 = −g. t + 𝑣0 .‬‬
‫‪0.25‬‬ ‫𝑡𝑑‬
‫𝑡 × 𝛼𝑠𝑜𝑐 ‪𝑥(𝑡) = 𝑣0 . 𝑐𝑜𝑠𝛼 × 𝑡 + 𝑐2 ⇒ 𝑥(𝑡) = 𝑣0 .‬‬
‫{‬ ‫‪1‬‬
‫⇒ ‪𝑦(𝑡) = − g. 𝑡 2 × 𝑡 + 𝑣0 . 𝑠𝑖𝑛𝛼 × 𝑡 + 𝑐3‬‬
‫‪2‬‬
‫‪1‬‬
‫‪0.25‬‬ ‫𝑡 × 𝛼𝑛𝑖𝑠 ‪⇒ 𝑦(𝑡) = − g. 𝑡 2 + 𝑣0 .‬‬
‫‪2‬‬

‫‪ -2-2‬معادلة المسار )𝒙(𝒇 = 𝒚‪:‬‬


‫)𝑡(𝑥‬
‫= 𝑡 ⇒ 𝑡 × 𝛼𝑠𝑜𝑐 ‪𝑥(𝑡) = 𝑣0 .‬‬
‫𝟓𝟐 ‪𝟐.‬‬ ‫‪0.25‬‬ ‫𝛼𝑠𝑜𝑐 ‪𝑣0 .‬‬
‫‪1 2‬‬
‫{‬ ‫)𝑡(𝑦‬ ‫=‬ ‫‪−‬‬ ‫𝑡 × 𝛼𝑛𝑖𝑠 ‪g. t × 𝑡 + 𝑣0 .‬‬
‫‪2‬‬
‫‪2‬‬
‫‪1‬‬ ‫)𝑡(𝑥‬ ‫)𝑡(𝑥‬
‫( ‪⇒ 𝑦(𝑡) = − g.‬‬ ‫× 𝛼𝑛𝑖𝑠 ‪) + 𝑣0 .‬‬
‫‪2‬‬ ‫𝛼𝑠𝑜𝑐 ‪𝑣0 .‬‬ ‫𝛼𝑠𝑜𝑐 ‪𝑣0 .‬‬
‫‪g‬‬
‫‪𝑦(𝑡) = −‬‬ ‫‪2‬‬ ‫‪2‬‬
‫𝛼 𝑛𝑎𝑡 ‪. 𝑥2 + 𝑥.‬‬
‫‪0.25‬‬ ‫𝛼 𝑠𝑜𝑐 ‪2. 𝑣0 .‬‬

‫‪ -3-2‬تبيان أن الطفل يسقط داخل حوض الماء (المسبح)‪:‬‬


‫من أجل 𝑚‪ 𝑦 = −𝐻 = −0,5‬بالتعويض في معادلة المسار‪:‬‬
‫‪0.25‬‬ ‫‪9,8‬‬
‫‪−0,5 = −‬‬ ‫‪2‬‬ ‫‪2‬‬
‫‪. 𝑥 2 + 𝑥. 𝑡𝑎𝑛 30‬‬
‫‪2. 5 . 𝑐𝑜𝑠 30‬‬
‫‪2‬‬
‫‪−0,26. 𝑥 + 0,58. 𝑥 + 0,5 = 0‬‬
‫‪0.25‬‬ ‫مقبول 𝑚‪𝑥1 = 2,9‬‬
‫{ ⇒ ‪∆= 0,856‬‬
‫مرفوض 𝑚‪𝑥2 = −0,67‬‬

‫‪0.25‬‬ ‫الطفل يسقط داخل حوض الماء(المسبح) ألن 𝑚‪𝑥1 = 2,9𝑚 < 𝐿 = 6‬‬

‫الصفحة ‪ 14‬من ‪11‬‬


‫األستاذ‪ :‬مكي يونس‬ ‫الشعبة‪ :‬تقني رياضي المؤسسة‪ :‬ثانوية االخوة عزوزي المشرية‬ ‫اإلجابة النموذجية‪ :‬للبكالوريا التجريبي‬

‫التمرين التجريبي (‪ 46‬نقاط)‪.‬‬ ‫‪-‬‬


‫التجربة األولى‪:‬‬
‫‪ -1‬معادلة تفاعل المعايرة الحادث‪:‬‬
‫𝟎𝟓 ‪𝟎.‬‬ ‫‪−‬‬ ‫‪−‬‬
‫‪0.50‬‬ ‫‪𝐶2 𝐻5 𝐶𝑂𝑂𝐻(𝑎𝑞) +‬‬ ‫)𝑞𝑎(𝑂𝐻‬ ‫=‬ ‫)𝑞𝑎(𝑂𝑂𝐶 ‪𝐶2 𝐻5‬‬ ‫)𝑙(𝑂 ‪+ 𝐻2‬‬

‫‪ -2‬إثبات ان 𝟗 ‪:𝒑𝑲𝒂 = 𝟒,‬‬


‫من عالقة أندرسون‪:‬‬
‫𝟎𝟓 ‪𝟎.‬‬ ‫] ‪[𝐶2 𝐻5 𝐶𝑂𝑂−‬‬ ‫] ‪[𝐶2 𝐻5 𝐶𝑂𝑂−‬‬
‫‪0.25‬‬ ‫𝑔𝑜𝑙 ‪𝑝𝐻 = 𝑝𝐾𝑎 +‬‬ ‫𝑔𝑜𝑙 ‪⇒ 𝑝𝐾𝑎 = 𝑝𝐻 −‬‬
‫]𝐻𝑂𝑂𝐶 ‪[𝐶2 𝐻5‬‬ ‫]𝐻𝑂𝑂𝐶 ‪[𝐶2 𝐻5‬‬
‫]𝐻𝑂𝑂𝐶 ‪[𝐶2 𝐻5‬‬
‫‪0.25‬‬ ‫𝑔𝑜𝑙 ‪⇒ 𝑝𝐾𝑎 = 𝑝𝐻 +‬‬ ‫‪= 3,45 + 𝑙𝑜𝑔(28,18) = 4,90‬‬
‫] ‪[𝐶2 𝐻5 𝐶𝑂𝑂−‬‬
‫‪ -3‬حساب ثابت التوازن 𝑲 لهذا التفاعل‪:‬‬
‫𝑓] ‪[𝐶2 𝐻5 𝐶𝑂𝑂−‬‬ ‫𝑓] ‪[𝐻3 𝑂+ ]𝑓 . [𝐶2 𝐻5 𝐶𝑂𝑂−‬‬ ‫𝑎𝐾‬
‫‪0.25‬‬ ‫=𝐾‬ ‫=‬ ‫=‬ ‫⇒‬
‫𝑒𝐾 𝑓] ‪[𝐶2 𝐻5 𝐶𝑂𝑂𝐻 ]𝑓 . [𝑂𝐻 − ]𝑓 [𝐶2 𝐻5 𝐶𝑂𝑂𝐻 ]𝑓 . [𝑂𝐻 − ]𝑓 . [𝐻3 𝑂+‬‬
‫𝟓𝟕 ‪𝟎.‬‬ ‫‪10−𝑝𝐾𝑎 10−4,9‬‬
‫‪0.25‬‬ ‫‪𝐾 = −𝑝𝐾𝑒 = −14 = 1,26. 109‬‬
‫‪10‬‬ ‫‪10‬‬
‫‪0.25‬‬ ‫نستنتج‪ :‬أن تفاعل المعايرة تفاعل تام ألن ‪𝐾 > 104‬‬
‫‪ -0‬تحديد حجم التكافؤ 𝑬𝒃𝑽‪:‬‬
‫يكون 𝑎𝐾𝑝 = 𝐻𝑝 وهذا يحدث عند نقطة نصف التكافؤ‬ ‫من أجل النسبة ‪= 1‬‬
‫]𝐻𝑂𝑂𝐶 ‪[𝐶2 𝐻5‬‬
‫‪0.25‬‬
‫𝟎𝟓 ‪𝟎.‬‬ ‫] ‪[𝐶2 𝐻5 𝐶𝑂𝑂−‬‬
‫وعليه فإن 𝐿𝑚‪𝑉𝑏𝐸 = 10‬‬
‫‪0.25‬‬
‫‪ -5‬حساب التركيز المولي 𝒂𝒄‪:‬‬
‫‪0.25‬‬ ‫‪𝑐𝑏 . 𝑉𝑏𝐸 5. 10−3 . 10‬‬
‫= 𝑎𝑐 ⇒ 𝐸𝑏𝑉 ‪𝑐𝑎 . 𝑉𝑎 = 𝑐𝑏 .‬‬ ‫=‬ ‫⇒‬
‫𝟎𝟓 ‪𝟎.‬‬ ‫𝑎𝑉‬ ‫‪5‬‬
‫‪0.25‬‬ ‫𝐿‪𝑐𝑎 = 10−2 𝑚𝑜𝑙/‬‬

‫التجربة الثانية‪:‬‬
‫‪ -1‬معادلة التفاعل المنمذجة لتفاعل األسترة‪:‬‬
‫𝟎𝟓 ‪𝟎.‬‬ ‫‪0.25‬‬
‫)𝑙(𝐻𝑂 ‪𝐶2 𝐻5 𝐶𝑂𝑂𝐻(𝑙) + 𝐶2 𝐻5‬‬ ‫𝑂 ‪= 𝐶2 𝐻5 𝐶𝑂𝑂 − 𝐶2 𝐻5 (𝑙) + 𝐻2‬‬
‫‪0.25‬‬ ‫اسم احاستر الناتج‪ :‬بروبانوات اإليثيل‪.‬‬

‫‪ -2‬التركيب المستعمل إلنجاز هذا التصنيع‪:‬‬


‫𝟓𝟐 ‪𝟎.‬‬ ‫‪0.25‬‬ ‫التركيب التجريبي )‪(2‬‬

‫الصفحة ‪ 15‬من ‪11‬‬


‫األستاذ‪ :‬مكي يونس‬ ‫الشعبة‪ :‬تقني رياضي المؤسسة‪ :‬ثانوية االخوة عزوزي المشرية‬ ‫اإلجابة النموذجية‪ :‬للبكالوريا التجريبي‬

‫‪ -1-3‬تحديد المنحنى البياني الممثل لتشكل االستر‪:‬‬


‫المنحنى المناسب هو المنحنى )𝑎(‪.‬‬
‫‪0.25‬‬
‫‪ -2-3‬حساب 𝒓 مردود تفاعل األسترة‪:‬‬
‫‪0.25‬‬ ‫𝑓𝑥‬ ‫)𝐸( 𝑓𝑛‬ ‫‪13,33 × 10−3‬‬
‫=𝑟‬ ‫= ‪. 100‬‬ ‫= ‪. 100‬‬ ‫‪. 100 = 67%‬‬
‫𝑥𝑎𝑚𝑥‬ ‫)𝑒𝑑𝑖𝑐𝐴( ‪𝑛0‬‬ ‫‪0,02‬‬
‫‪0.25‬‬ ‫نستنتج‪ :‬أن صنف الكحول المستعمل أولي‪.‬‬
‫‪ -3-3‬جدول تقدم التفاعل‪:‬‬
‫معادلة التفاعل‬ ‫‪𝐶2 𝐻5 𝐶𝑂𝑂𝐻 +‬‬ ‫‪𝐶2 𝐻5 𝑂𝐻 = 𝐶5 𝐻10 𝑂2‬‬ ‫𝑂 ‪+ 𝐻2‬‬
‫𝟓𝟕 ‪𝟏.‬‬ ‫الحالة‬ ‫التقدم‬ ‫كميات المادة بالمول)𝑙𝑜𝑚(‬

‫‪2 × 0.25‬‬ ‫ابتدائية‬ ‫‪0‬‬ ‫‪𝑛0‬‬ ‫‪𝑛0‬‬ ‫‪0‬‬ ‫‪0‬‬


‫وسطية‬ ‫𝑥‬ ‫𝑥 ‪𝑛0 −‬‬ ‫𝑥 ‪𝑛0 −‬‬ ‫𝑥‬ ‫𝑥‬
‫نهائية‬ ‫𝑓𝑥‬ ‫𝑓𝑥 ‪𝑛0 −‬‬ ‫𝑓𝑥 ‪𝑛0 −‬‬ ‫𝑓𝑥‬ ‫𝑓𝑥‬

‫‪ -0-3‬حساب ثابت التوازن 𝑲 ‪:‬‬


‫] 𝑂 ‪[𝐶5 𝐻10 𝑂2 ]. [𝐻2‬‬ ‫)𝐸( 𝑓𝑛 ‪𝑛𝑓 (𝐸).‬‬
‫‪0.25‬‬ ‫=𝐾‬ ‫=‬
‫] 𝐻𝑂 ‪[𝐶2 𝐻5 𝐶𝑂𝑂𝐻 ]. [𝐶2 𝐻5‬‬ ‫‪2‬‬
‫))𝐸( 𝑓𝑛 ‪(𝑛0 (𝐴𝑐𝑖𝑑𝑒) −‬‬
‫‪𝑥𝑓2‬‬ ‫‪(13,33 × 10−3 )2‬‬
‫‪0.25‬‬ ‫=‬ ‫‪2 = (0,02 − 13,33 × 10−3 )2 ⇒ 𝐾 = 4‬‬
‫) 𝑓𝑥 ‪(𝑛0 −‬‬

‫‪ -1-0‬إيجاد مردود التفاعل ‪ 𝒓′‬في الحالة الثانية‪:‬‬


‫‪𝑥𝑓2‬‬
‫‪0.25‬‬ ‫=𝐾‬ ‫‪=4‬‬
‫) 𝑓𝑥 ‪(0,02 − 𝑥𝑓 )(0,04 −‬‬
‫‪3𝑥𝑓2 − 0,24. 𝑥𝑓 + 0,0032 = 0‬‬
‫مرفوض 𝑙𝑜𝑚 ‪𝑥𝑓1 = 0,063‬‬
‫𝟓𝟕 ‪𝟎.‬‬ ‫{ ⇒ ‪∆= 0,0192‬‬
‫مقبول 𝑙𝑜𝑚 ‪𝑥𝑓2 = 0,017‬‬
‫‪0.25‬‬ ‫‪𝑥𝑓2‬‬ ‫‪𝑥𝑓2‬‬ ‫‪0,017‬‬
‫= ‪𝑟′‬‬ ‫= ‪. 100‬‬ ‫= ‪. 100‬‬ ‫‪. 100 = 85%‬‬
‫𝑥𝑎𝑚𝑥‬ ‫)𝑒𝑑𝑖𝑐𝐴( ‪𝑛0‬‬ ‫‪0,02‬‬
‫‪ -2-0‬المقارنة بين𝒓 و ‪: 𝒓′‬‬
‫‪0.25‬‬
‫𝒓 > ‪ 𝒓′‬نستنتج أن استعمال مزيج غير متكافئ الموحات يساهم في رفع مردود التفاعل‪.‬‬

‫الصفحة ‪ 11‬من ‪11‬‬


‫امجل وريتتتتتتتتتتتتتة اجلزائريتتتتتتتتتتتتتة ادلميوانراطيتتتتتتتتتتتتتتتتة الشعبيتتتتتتتتتتتتتتتتتتتتتتتتتتة‬
‫السمنة ادلراسمية‪2023/2022 :‬‬ ‫و ار الرتبية الوطنية‬
‫الشعبة‪ 3 :‬ت ر‬ ‫مديرية الرتبية اجلزائر ‪ -‬غرب‬
‫املد ‪ 4 :‬سا و ‪ 30‬د‬ ‫اثنوية عبد امحليد دروا (مسروين ‪ )02‬أأو د فاي‬
‫امتحان باكلوراي جترييب س ماد العلوم الفزياياية‬
‫عىل املرتحش اختيار املوضعني الآتيني‪:‬‬
‫املوضوع الأول‬
‫حيتوي املوضوع ا ألول عىل ‪ 5‬صفحات (من الصفحة ‪ 1‬من ‪ 10‬اىل الصفحة ‪ 5‬من ‪)10‬‬
‫المترين ا ألول ‪:‬‬
‫يستتتتتتمتعت مإانتتتتتت ا ر )𝑒𝑖𝑟𝑡𝑢𝑛𝑖𝑚( لتقليتتتتتت استتتتتتمهالك الطاانتتتتتتة الة ر ايتتتتتتة س العتتتتتت رات الستتتتتتة ية و تتتتتتو تتتتتتا تتتتتتر‬
‫يستح لتحمك الآيل س اطفاء مصابيح السالمل وا ألروانة بعد مرور مد م ية انابةل للضبط مسمبقا‪.‬‬
‫هندف من خالل ذا المترين اىل دراسة مبد أأ اشمتغال مإان ا ر ‪.‬‬
‫ميث الشلك (‪ )1‬جزءا من تركيب مبسط ملإان ا ر مةون من ‪:‬‬
‫‪ -‬مودل مثايل للتوتر املسممتر انوته الة ر اية احملركة 𝐸‪.‬‬
‫‪ -‬اناطعة 𝐾 و ر 𝑃 يلعب دور اناطع التيار‪.‬‬
‫‪ -‬ان أأويم مقاومته 𝑅‪.‬‬
‫‪ -‬مةثفة سعها 𝐹𝜇 ‪.𝐶 = 200‬‬
‫‪ -‬مركبة الةرتونية متةن من اضاء املصباح 𝐿 ما دام التوتر بني طرس املةثفة أأصغر أأو يساوي توترا حداي 𝑆𝑢‪.‬‬
‫نقب أأن شد التيار الة ر املار س مدخ املركبة ا لةرتونية تبقى م عدمة س لك حلظة‪.‬‬
‫‪ −(I‬دراسة ثنا القطب 𝐶𝑅‬
‫عنتتتتد اللحظتتتتة ‪ 𝑡 = 0‬نغلتتتت القاطعتتتتة 𝐾 ونتتتترتك التتتتزر 𝑃 مفتوحتتتتا فتشتتتتحن املةثفتتتتة تتتتتدربيا بواستتتتطة املتتتتودل‪ .‬نعتتتتاين تطتتتتور التتتتتوتر‬
‫𝑐𝑢 بني طرس املةثفة سمتع ل جت زي معلومايت مالمئ‪.‬‬
‫[‪ ]1‬أأعد رمس اجلزء املدروس من ادلار و ّبني عليه أأس م التوترات و ة التيار الة ر املار س ادلار ‪.‬‬
‫[‪ ]2‬فرس جم راي ماذا حيدث عىل مسمتوى املةثفة‪.‬‬
‫𝑐𝑢𝑑‬
‫𝐶𝑅 ‪𝑢𝑐 +‬‬ ‫[‪ ]3‬بني أأن التوتر 𝑐𝑢 حيق املعادةل التفاضلية ‪= 𝐸 :‬‬
‫𝑡𝑑‬
‫𝑡‬
‫لك من 𝐴 و 𝜏 ليك تةون ادلاةل الزم ية ) 𝜏‪ 𝑢𝑐 (𝑡) = 𝐴 (1 − 𝑒 −‬حال للتعادةل التفاضلية السابقة‪.‬‬ ‫[‪ ]4‬جد عبار‬
‫[‪ ]5‬بني أأن الثاب 𝜏 متجانس مع الزمن‪.‬‬
‫الشلك (‪)1‬‬ ‫[‪ ]6‬ميث الشلك (‪ )1‬تغريات )𝑡( 𝑐𝑢‬
‫أأ ‪ -‬حدد بيانيا انمية لك من 𝐴 و 𝜏‪.‬‬
‫ب ‪ -‬اسمتنتج انمية املقاومة 𝑅‪.‬‬
‫‪ −(II‬حتديد مد اشمتغال املإان‬
‫املد الزم ية الال مة لوصول أأحد ساكن الع ر اىل ب بيته يه 𝑠 ‪.∆𝑡 = 80‬‬
‫[‪ ]1‬لتةن 𝑠𝑡 اللحظة اليت يأأخذ فهيا التوتر 𝑐𝑢 القمية احلدية ّبني أأ ّن‪:‬‬
‫𝑠𝑢‬
‫‪𝑡𝑠 = − 𝜏 ln (1 −‬‬ ‫)‬
‫𝐸‬
‫وصول ساكن الع ر اىل بيته‪.‬‬ ‫𝑉 ‪ّ 𝑢𝑠 = 15‬بني أأن املصباح 𝐿 ينطفئ انب‬ ‫[‪ ]2‬عل أأ ّن‬
‫[‪ ]3‬حتتتتدد القميتتتتة احلديتتتتة 𝑠𝑅 ملقاومتتتتة الناانتتتت ا ألويم التتتتيت ستتتتتح لستتتتاكن العتتتت ر لوصتتتتول اىل ب بيتتتتته انبتتتت انطفتتتتاء املصتتتتباح‬
‫(نعترب أأن انمي 𝐶 𝐸 و 𝑠𝑢 تتغري)‪.‬‬

‫الصفحة (‪ )1‬من (‪)10‬‬


‫المترين الثاين‬
‫دمتتتتتتتر ثتتتتتتتوران التتتتتتترب ن مونتتتتتتت بيليتتتتتتته )𝑒‪ (𝑀𝑜𝑛𝑡 𝑃𝑒𝑙é‬س عتتتتتتتام‬
‫‪ 1902‬مدينتتتتتتتتتة ستتتتتتتتتان بيتتتتتتتتتري )𝑒𝑟𝑟𝑒𝑖𝑃 ‪ (𝑆𝑎𝑖𝑛𝑡 −‬و حمافظتتتتتتتتتة‬
‫مارتينيتتتتت )𝑒𝑢𝑞𝑖𝑛𝑖𝑡𝑟𝑎𝑀(‪ .‬يطلتتتتت عليتتتتته أأيضتتتتتا امس " أأستتتتتو أأ رثتتتتتة‬
‫بر نيتتتتتة" متتتتتن القتتتتترن التتتتتت ‪ 20‬حيتتتتت أأدى اىل مقتتتتتت حتتتتتوايل ‪30000‬‬
‫نستتتتتة و نتتتت يالبيتتتتة تتتتذب الوفيتتتتات استتتتبب تتتتتدفقات امحلتتتتم الرب نيتتتتة‬
‫التتتتيت وانعتتتت س مدينتتتتة ستتتتان بيتتتتري والتتتتيت نتتتت س ذكل الوانتتتت تعتتتتد‬
‫يه أأكتتتتترب مدينتتتتتة س اجلزيتتتتتر ‪ .‬ولقتتتتتد دمتتتتترت ستتتتتان بيتتتتتري متامتتتتتا محلتتتتتم‬
‫الرب نية س غضون داناا من الثوران‪.‬‬
‫هيدف ذا المترين اىل اباد مقدار رسعة تدفقات امحلم الرب نية املنبعثة خالل ذا الانفجار الرب ين وحتديد أأانىص ارتفاع وصل اليه امحلم الرب نية‪.‬‬
‫س املرجتتتتتع الستتتتتطح ا ألريض املتتتتترتبط ملعتتتتت )𝑦 ‪ (𝑂, 𝑥,‬اذلي نعتتتتتتربب عطتتتتتايل متتتتتت دراستتتتتة حمتتتتتأ حركتتتتتة مركتتتتتز عطتتتتتاةل أأحتتتتتد‬
‫تدفقات امحلم الرب نية و اليت من أأج التبسميط نعترب ا نقطة مادية )𝑆( كتلها 𝑚‪.‬‬
‫تإخذ انمية شد اجلاذبية 𝐼 ‪ 𝑔 = 9,81 𝑆.‬و هتت س لك المترين تأأثري دافعة ارمخيدس وانو الاحتاكك مع ال واء‪.‬‬
‫‪ −(I‬وفقًتتتتا ألانتتتتوال شتتتتا د عيتتتتان تغتتتتادر امحلتتتتم الرب نيتتتتة فو تتتتة التتتترب ن ارسعتتتتـة ابتداايتتتتة ‪ 𝑣0‬يصتتتتنع حامل تتتتا اويتتتتة ‪ 𝛼 = 43°‬متتتتع‬
‫احملور ا ألفق ‪ .‬الشلك (‪ )2‬ميث تغريات الفاصةل 𝑥 و الفاصةل 𝑦 بد ةل الزمن‪.‬‬
‫[‪ ]1‬ارشح ملاذا ميةن اعتبار حركة مركز عطاةل أأحد تدفقات امحلم الرب نية سقوط ًا حرا؟‬
‫[‪ ]2‬بتطبيتتتت القتتتتانون الثتتتتاين لنيتتتتوتن جتتتتد العالانتتتتة بتتتتني شتتتتعاع ستتتتارع مركتتتتز عطتتتتاةل امجلتتتتةل املدروستتتتة 𝑎⃗⃗⃗⃗ وبتتتتني شتتتتعاع ستتتتارع‬
‫)𝑚(𝑦 ;)𝑚(𝑥‬ ‫اجلاذبية ا ألرضية 𝑔⃗⃗⃗⃗ مث أأعط عبار )𝑡( 𝑥𝑎 و )𝑡( 𝑦𝑎 س مع ادلراسة‪.‬‬
‫[‪ ]3‬اسمتنتج عباريت مركبيت الرسعة اللحظية )𝑡( 𝑥𝑣 و )𝑡( 𝑦𝑣‪.‬‬
‫[‪ ]4‬أأثب أأ ّن عباريت مركبيت شعاع املوضع تةتب لشلك‪:‬‬
‫)𝑡(𝑥‬ ‫𝑡 ‪𝑥(𝑡) = (𝑣0 . cos 𝛼).‬‬
‫‪1‬‬
‫𝑡 ‪𝑦(𝑡) = − . 𝑔. 𝑡 2 + (𝑣0 . sin 𝛼).‬‬
‫)𝑡(𝑦‬ ‫‪2‬‬
‫أأكتب معادةل املسار )𝑥(𝑓 = 𝑦‪.‬‬ ‫[‪]5‬‬
‫الشلك (‪)2‬‬
‫)𝑠(𝑡‬ ‫[‪ ]6‬عرف املدى مث حدد انميته بياني ًا‪.‬‬
‫[‪ ]7‬عرف اذلرو مث حدد انميها بياني ًا‪.‬‬
‫[‪ ]8‬جد الزمن اذلي اسمتغرانته امحلم الرب نية عندما للوصول اىل أأبعد نقطة انطعها‪.‬‬
‫[‪ ]9‬أأحسب ‪ 𝑣0‬الرسعة الابتدااية للحتم الرب نية س ذب احلاةل‪.‬‬
‫تتتوداي أأثنتتتتاء الثتتتتوران نعتتتتترب أأ ّن أأانصتتتتتى ارتفتتتتاع وصتتتتل اليتتتته امحلتتتتم الةبتتتتري ن ‪ ℎ‬حيتتتت‬
‫‪ −(II‬متتتتن أأجتتتت امحلتتتتم الرب نيتتتتة املقذوفتتتتة معت ً‬
‫𝑚 ‪ .ℎ = 400‬س تتتتتذب احلتتتتتاةل نعتتتتتترب أأ ّن شتتتتتعاع الرسعتتتتتـة الابتداايتتتتتة موجتتتتته شتتتتتاانوليا تتتتتو ا ألعتتتتتىل و نرمتتتتتز بتتتتتت ‪𝑣01‬‬
‫⃗⃗⃗⃗⃗⃗ و مستتتتتار‬
‫مركز عطاةل امجلةل املدروسة مسمتقمي‪.‬‬
‫[‪ ]1‬ستتتتتتمتعانة جتتتتتتا ت الستتتتتتإالني ]‪ [3‬و ]‪ [4‬متتتتتتن اجلتتتتتتزء ‪ I‬كيتتتتتت تصتتتتتتبح عبتتتتتتار لك متتتتتتن )𝑡( 𝑥𝑣 و )𝑡( 𝑦𝑣 و كتتتتتتذكل‬
‫عبار )𝑡(𝑥 و )𝑡(𝑦 س ذب احلاةل‪.‬‬
‫[‪ ]2‬متتتتن عبتتتتار )𝑡( 𝑦𝑣 جتتتتد العبتتتتار املوافقتتتتة للتتتتزمن 𝑡 اذلي ستتتتمتغرانه امجلتتتتةل للوصتتتتول انطالانتتتتا متتتتن نقطتتتتة املبتتتتد أأ 𝑂 اىل أأانصتتتتتى‬
‫ارتفاع ‪.ℎ‬‬
‫[‪ّ ]3‬بني أأ ّن عبار الرسعـة الابتدااية تةتب لشلك ‪ 𝑣01 = √2. 𝑔. ℎ :‬مث احسب انميها ملا 𝑚 ‪.ℎ = 400‬‬

‫الصفحة (‪ )2‬من (‪)10‬‬


‫المترين الثال‬
‫متقر وثالثتتتت ئة متتتتن نظتتتتائرب تقتتتتع س هنايتتتتة سلستتتتةل‬ ‫الرصتتتتا عنصتتتتتر كمييتتتتا رمتتتتزب 𝑏𝑃 وعتتتتددب ّ‬
‫اذلري ‪ 82‬كتتتت أأنّتتتته فلتتتتز (معتتتتدن) مستتتت ّ‬
‫‪. 210‬‬ ‫‪238‬‬
‫تفة العنارص الثقيةل املش ّعة مث اليورانيوم 𝑈‪ 92‬و البولونيوم 𝑜𝑃‪84‬‬

‫𝟖𝟑𝟐 و البولونيوم 𝒐𝑷𝒁𝑨 و التطبيقات املتة ة للك مهن ‪.‬‬ ‫ل‬


‫هيدف ذا ا مترين اىل دراسة النشاط الاشعاع للك من اليورانيوم 𝑼𝟐𝟗‬

‫املعطيات ‪:‬‬
‫‪ ‬كتةل الربوتون ‪𝑚𝑝 = 1,00728 𝑢 :‬‬ ‫‪𝑚( 238‬‬
‫اليورانيوم ‪92𝑈) = 238,00031 𝑢 : 238‬‬ ‫كتةل نوا‬ ‫‪‬‬
‫‪ ‬كتةل النوترون ‪𝑚𝑛 = 1,00866 𝑢 :‬‬ ‫‪𝑚( 206‬‬
‫‪82𝑃𝑏) = 205,92949 𝑢 : 206‬‬ ‫كتةل نوا الرصا‬ ‫‪‬‬
‫‪1 𝑢 = 931,5 𝑀𝑒𝑉. 𝑐 −2‬‬ ‫وحد الةتةل اذلرية ‪:‬‬ ‫‪‬‬ ‫‪𝑀( 238‬‬‫𝑙𝑜𝑚 ‪92𝑈) = 238 𝑔.‬‬
‫‪−1‬‬
‫الةتةل املولية ل ألورانيوم ‪: 238‬‬ ‫‪‬‬
‫𝑔𝐾 ‪1𝑢 = 1,67 × 10−27‬‬ ‫‪𝑀( 206‬‬‫𝑙𝑜𝑚 ‪82𝑃𝑏) = 206 𝑔.‬‬
‫‪−1‬‬
‫الةتةل املولية للرصا ‪: 206‬‬ ‫‪‬‬
‫‪𝑡1/2 ( 238‬‬
‫معر اليورانيوم ‪92𝑈) = 4,5. 10 𝑎𝑛𝑠 : 238‬‬
‫‪9‬‬
‫من نص‬ ‫‪‬‬
‫من نص معر البولونيوم ‪.𝑡1/2( 𝐴𝑍𝑃𝑜) = 138 𝑗𝑜𝑢𝑟𝑠 : 210‬‬ ‫‪‬‬
‫𝑙𝐸‬ ‫‪𝑀𝑒𝑉Τ‬‬
‫‪( 206‬‬
‫‪82𝑃𝑏) = 7,87‬‬ ‫‪𝑛𝑢𝑐𝑙é𝑜𝑛 : 206‬‬ ‫طاانة الربط للك نيلكيون لنسمبة لنوية الرصا‬ ‫‪‬‬
‫𝐴‬
‫اجلزء ا ألول‪:‬‬
‫لتتتتتار او تتبتتتتع تطتتتتور بعتتتت الظتتتتوا ر الطبيعيتتتتة يلجتتتتأأ العلتتتت ء اىل طرااتتتت وتق يتتتتات اتلفتتتتة تعمتتتتتد أأساستتتتا عتتتتىل انتتتتانون التنتتتتاان‬
‫ا شعاع ‪ .‬من بني ذب التق يات تق ية التار بواسطة اليورانيوم ‪ -‬الرصا ‪.‬‬
‫تتحتتتتتول نتتتتتوا اليورانيتتتتتوم ‪ 238‬اىل نتتتتتوا الرصتتتتتا ‪ 206‬عتتتتترب سلستتتتتةل متتاليتتتتتة متتتتتن اشتتتتتعاعات 𝛼 واشتتتتتعاعات ‪ .𝛽−‬نمنتتتتتذ تتتتتذب‬
‫التحو ت النووية ملعادةل احلصيةل‪:‬‬
‫‪238‬‬ ‫‪206‬‬
‫𝑈‪92‬‬ ‫→‬ ‫𝑏𝑃‪82‬‬ ‫𝑒𝐻‪+ 𝑥 −10𝑒 + 𝑦 42‬‬
‫𝟖𝟑𝟐‬
‫𝐈(‪ −‬دراسة نوا اليورانيوم ‪: 238‬‬
‫𝑼𝟐𝟗‬
‫[‪ ]1‬بتطبي انانوين الا فاظ حدد لك من العددين الصحيحني 𝑥 و 𝑦 املشار الهي س املعادةل احلصيةل‪.‬‬
‫[‪ ]2‬أأعط تركيب نوا اليورانيوم ‪.238‬‬
‫‪ 206‬أ‬
‫[‪ ]3‬احسب طاانة الربط لنسمبة لنوا اليورانيوم ‪ 238‬مث حتق أأن نوا 𝑏𝑃‪ 82‬أكرث اسمتقرارا من النوا 𝑈‪92‬‬
‫‪.238‬‬
‫𝐈𝐈(‪ −‬اتر خصر معدنية بواسطة اليورانيوم ‪ -‬الرصا ‪:‬‬
‫جند الرصا واليورانيوم بنسب اتلفة س الصخور املعدنية حسب اتر تةوهنا‪.‬‬
‫نعترب أأن تواجد الرصا س بع الصخور املعدنية ينتج فقط عن التفة التلقا ل ألورانيوم ‪ 238‬خالل الزمن و نةتب ‪:‬‬
‫)𝑡( 𝑏𝑃𝑁 ‪𝑁𝑢 (0) = 𝑁𝑢 (𝑡) +‬‬
‫)‪ 𝑁𝑢 (0‬عدد أأنوية اليورانيوم املشعة الابتدااية‪.‬‬ ‫حي ‪:‬‬
‫)𝑡( 𝑢𝑁 عدد أأنوية اليورانيوم املشعة املتبقية عند اللحظة 𝑡‪.‬‬
‫)𝑡( 𝑏𝑃𝑁 عدد أأنوية الرصا املوجود س العينة عند اللحظة 𝑡‪.‬‬
‫تتتتتتوفر عينتتتتة متتتتن خصتتتتر معدنيتتتتة حتتتتتتوي عنتتتتد حلظتتتتة تةوهنتتتتا التتتتيت نعترب تتتتا أأصتتتتال للتتتتتوار )‪ (𝑡 = 0‬عتتتتىل عتتتتدد )‪ 𝑁𝑢 (0‬متتتتن‬
‫أأنويتتتتتة اليورانيتتتتتوم ‪ .238‬حتتتتتتتوي تتتتتذب العينتتتتتة املعدنيتتتتتة عنتتتتتد حلظتتتتتة 𝑡 عتتتتتىل الةتتتتتتةل 𝑔 ‪ 𝑚𝑈 (𝑡) = 100‬متتتتتن اليورانيتتتتتوم ‪238‬‬
‫والةتةل 𝑔𝑚 ‪ 𝑚𝑃𝑏 (𝑡) = 1‬من الرصا ‪.206‬‬
‫[‪ ]1‬ما يه خصاا النشاط الاشعاع ؟‬
‫[‪ ]2‬أأثب أأن تعبري معر الصخر املعدنية و ‪:‬‬
‫‪𝑡1⁄‬‬ ‫‪𝑚𝑃𝑏 (𝑡) 𝑀( 238‬‬
‫)𝑈‪92‬‬
‫‪2‬‬
‫=𝑡‬ ‫‪𝑙𝑛 (1 +‬‬ ‫)‬
‫)‪ln(2‬‬ ‫‪𝑚𝑈 (𝑡) 𝑀( 206‬‬
‫)𝑏𝑃‪82‬‬
‫لسمنة‪.‬‬ ‫أأحسب 𝑡‬ ‫[‪]3‬‬

‫الصفحة (‪ )3‬من (‪)10‬‬


‫اجلزء الثاين‪:‬‬
‫البولونيتتتتتتوم فلتتتتتتز (معتتتتتتدن) در مت اكتشتتتتتتافه ستتتتتتمنة ‪ 1898‬متتتتتتن طتتتتتترف العتتتتتتامل بيتتتتتتري كتتتتتتوري )𝑒𝑖𝑟𝑢𝐶 𝑒𝑟𝑟𝑒𝑖𝑃( ‪ .‬رمتتتتتتزب 𝑜𝑃‬
‫ويستتتتمتيفدم س جمتتتتتال ا أليتتتتتاث النوويتتتتتة كتتتتت يتتتتتدخ البولونيتتتتتوم س جمتتتتال التصتتتتتوير الفوتتتتتتوغراس والطباعتتتتتة كتتتتت استتتتتمتيفدم س ا أل تتتتتز‬
‫التتتتيت تتتتتأأين ال تتتتواء متتتتن أأجتتتت م تتتتع تتتتترا الشتتتتحنات الة ر ايتتتتة‪ .‬يعتتتتترب البولونيتتتتوم 𝑜𝑃𝑍𝐴 النظتتتتري الوحيتتتتد املتواجتتتتد س الطبيعتتتتة ويتتتتإدي‬
‫‪.206‬‬‫𝑏𝑃‪82‬‬ ‫تفة نوا م ه اىل انبعاث دانيقة 𝛼 وتةون نوا الرصا‬
‫)𝑉𝑒𝑀 ‪𝐸 (× 105‬‬ ‫[‪ ]1‬اكتب معادةل تفة البولونيوم 𝑜𝑃𝑍𝐴 حمددا العددين 𝐴 و 𝑍 ‪.‬‬
‫[‪ ]2‬اعامتدا عىل اطط الطاانة املتث س الشلك (‪ )3‬احسب ‪:‬‬
‫𝑝 ‪126 𝑛 + 84‬‬
‫‪1,971820‬‬ ‫أأ ‪ -‬الطاانة احملرر 𝑏𝑖𝐿𝐸 لوحد 𝑉𝑒𝑀 خالل تفة نوا البولونيوم 𝑜𝑃𝑍𝐴‪.‬‬
‫ب ‪ -‬النق الةتيل 𝑚∆ لةيلو غرام لنوا البولونيوم‪.‬‬
‫[‪ ]3‬احسب بوحد ‪ 𝑠 −1‬اثب النشاط ا شعاع 𝜆 للبولونيوم‪.‬‬
‫𝐴‬
‫𝑜𝑃𝑍‬ ‫[‪ ]4‬النشتتتتتتتاط الابتتتتتتتتدا لعينتتتتتتتة متتتتتتتن أأنويتتتتتتتة البولونيتتتتتتتوم تتتتتتتو × ‪𝐴0 = 3,5‬‬
‫‪1,955372‬‬
‫𝑞𝐵 ‪ 1011‬حتتتتتدد بوحتتتتتد 𝑠𝑟𝑢𝑜𝐽 اللحظتتتتتة ‪ 𝑡1‬التتتتتيت يةتتتتتون فهيتتتتتا نشتتتتتاط تتتتتذب‬
‫‪206‬‬
‫𝑒𝐻‪𝑃𝑏 + 4‬‬
‫العينة و 𝑞𝐵 ‪.𝐴1 = 3,7 × 104‬‬
‫‪82‬‬ ‫‪2‬‬
‫‪1,955318‬‬
‫الشلك (‪)3‬‬
‫المترين التجرييب‬
‫اجلزء ا ألول‬
‫وضعنا س بيشتر جح 𝐿𝑚 ‪ 𝑉 = 250‬من متاد مط تر حتتتوي عتىل ثنتا اليتتود )𝑞𝑎(‪ 𝐼2‬برتكتزي ‪ 𝑐 = 3. 10−2 𝑚𝑜𝑙. 𝐿−1‬مث أأضتف ا عنتد درجتة‬
‫حرار اثبتة 𝐶‪ 𝑇 = 25°‬انطعة من معدن الةروم )𝑠(𝑟𝐶 كتلها 𝑔 ‪.𝑚 = 0,5‬‬
‫تعطى ‪𝑀(𝐶𝑟) = 52 𝑔. 𝑚𝑜𝑙 −1 ; 𝜆𝐼− = 7,68 𝑚𝑆. 𝑚2 . 𝑚𝑜𝑙 −1 ; 𝜆𝐶𝑟 3+ = 21,6 𝑚𝑆. 𝑚2 . 𝑚𝑜𝑙 −1 :‬‬
‫التحول الةمييا البط ء و التام احلادث بني ثنا اليود و الةروم يمنذ بتفاع كمييا معادلته ‪:‬‬
‫→ )𝑞𝑎( ‪2 𝐶𝑟(𝑠) + 3 𝐼2‬‬ ‫)𝑞𝑎( ‪2 𝐶𝑟 3+ (𝑎𝑞) + 6 𝐼 −‬‬
‫متابعتتتتة التحتتتتول عتتتتن طريتتتت انيتتتتاس الناانليتتتتة النوعيتتتتة ‪ σ‬للتتتتتزت التفتتتتاعيل س حلظتتتتات م يتتتتة اتلفتتتتة مة تنتتتتا متتتتن احلصتتتتول عتتتتىل جتتتتدول‬
‫القياسات التايل ‪:‬‬
‫)𝑛𝑖𝑚( 𝑡‬ ‫‪0‬‬ ‫‪1‬‬ ‫‪2‬‬ ‫‪4‬‬ ‫‪6‬‬ ‫‪8‬‬ ‫‪12‬‬ ‫‪14‬‬ ‫‪16‬‬
‫) ‪𝜎 (𝑆. 𝑚−1‬‬ ‫‪0‬‬ ‫‪0,426‬‬ ‫‪0,657‬‬ ‫‪0,829‬‬ ‫‪0,876‬‬ ‫‪0,888‬‬ ‫‪0,892‬‬ ‫‪0,892‬‬ ‫‪0,892‬‬
‫)𝑙𝑜𝑚𝑚( 𝑥‬

‫ارشح ملاذا ميةن متابعة ذا التحول عن طري انياس الناانلية النوعية؟‬ ‫[‪]1‬‬
‫أأكتب املعادلتني النصفيتني للأكسد و لالرجاع مث حدد الثناايتني ادلاخلتني س التفاع ‪.‬‬ ‫[‪]2‬‬
‫احسب مكية املاد الابتدااية للتتفاعلني‪.‬‬ ‫[‪]3‬‬
‫أأجنز جدو لتقدم التفاع احلادث‪.‬‬ ‫[‪]4‬‬
‫اكتب عبار الناانلية النوعية ‪ σ‬للتزت بد ةل التقدم 𝑥 مث أأمك اجلدول الساب و ارمس املنحىن )𝑡(𝑓 = 𝑥‪.‬‬ ‫[‪]5‬‬
‫عرف من نص التفاع ‪ 𝑡1/2‬مث عني انميته‪.‬‬ ‫[‪]6‬‬
‫عرف الرسعة احلجتية للتفاع و أأحسب انميها س اللحظتني ‪ 𝑡 = 0‬و 𝑛𝑖𝑚 ‪.𝑡 = 4‬‬ ‫[‪]7‬‬
‫فرس جم راي تطور الرسعة احلجتية للتفاع‬ ‫[‪]8‬‬

‫الصفحة (‪ )4‬من (‪)10‬‬


‫اجلزء الثاين‬
‫هيدف ذا اجلزء اىل دراسة معود رو‪ -‬کمييا ‪ :‬يتةون من‬
‫‪ ‬صفيحة من الةروم )𝑠(𝑟𝐶 مغتور س حملول ما لنرتات الةروم )𝑞𝑎() ‪.(𝐶𝑟 3+ + 3𝑁𝑂3−‬‬
‫‪ ‬صفيحة من الفضة )𝑠(𝑔𝐴 مغتور س حملول ما لنرتات الفضة )𝑞𝑎() ‪.(𝐴𝑔+ + 𝑁𝑂3−‬‬
‫‪ ‬جرس ملح يربط بني احمللولني‪.‬‬
‫املعطيات ‪ :‬الفاراداي ‪.𝐹 = 96500 𝐶. 𝑚𝑜𝑙−1‬‬

‫الشلك (‪)4‬‬
‫عرف الفاراداي‪.‬‬ ‫[‪]1‬‬
‫أأعط أأس ء مةو ت العتود املوافقة ل ألرانام املبي ة عىل الشلك (‪.)4‬‬ ‫[‪]2‬‬
‫ما و دور اجلرس ملح ‪.‬‬ ‫[‪]3‬‬
‫نتتتتربط انتتتت أأويم عتتتتىل التسلستتتت متتتتع أأمبريمتتتترت مث نتتتتربط ثنتتتتا القطتتتتب احملصتتتت عليتتتته بقطتتتتيب العتتتتتود الشتتتتلك (‪ .)4‬يشتتتتري‬ ‫[‪]4‬‬
‫ا ألمبريمتتتتترت اىل متتتتترور تيتتتتتار تتتتتر شتتتتتدته اثبتتتتتتة س ادلار 𝐴𝑚 ‪ 𝐼 = 60‬و نالحتتتتتد بعتتتتتد اشتتتتتمتغال العتتتتتتود ملتتتتتد م يتتتتتة‬
‫تأآلك صفيحة الةروم وترسب معدن الفضة عىل مسمتوى صفيحة الفضة‪.‬‬
‫أأ ‪ -‬عني أأانطاب العتود مع التعلي مث اعط رمزب الاصطاليح‪.‬‬
‫ب ‪ -‬اكتب معادةل التفاع احلاص عند لك مرسى واملعادةل احلصيةل أأثناء اشمتغال العتود‪.‬‬
‫بعد مد م ية 𝑡‪ Δ‬من اشمتغال العتود تةون انمية تقدم التفاع احلاص يه 𝑙𝑜𝑚‪.𝑥 = 1,21. 10−3‬‬ ‫[‪]5‬‬
‫أأ‪ -‬اجنز جدول تقدم التفاع احلاص ‪.‬‬
‫ب‪ -‬أأحسب انمية 𝑡‪.Δ‬‬

‫الصفحة (‪ )5‬من (‪)10‬‬


‫املوضوع الثاين‬
‫حيتوي املوضوع الثاين عىل ‪ 5‬صفحات (من الصفحة ‪ 6‬من ‪ 10‬اىل الصفحة ‪ 10‬من ‪)10‬‬
‫المترين ا ألول ‪:‬‬
‫س حصتتتتتة ل ألعتتتتت ل ا ربيتتتتتة أأحضتتتتتتر ا ألستتتتتمتاذ انتتتتت أأويم مقاومتتتتتته 𝑅 وشتتتتتميعة ذاتهيتتتتتا 𝐿 ومقاومهتتتتتا ادلاخليتتتتتة 𝑟 مث انتتتتتام بتفتتتتتوت‬
‫التالميذ اىل مجموعتني من أأج حتديد انمية لك من 𝑅 𝐿 و 𝑟‪ .‬الوساا املتوفر س ا ترب يه ‪:‬‬
‫‪ ‬أأسالك توصي ‪.‬‬ ‫‪ ‬اناطعة 𝐾‪.‬‬ ‫‪ ‬مودل للتوتر الثاب انوته احملركة 𝑉‪.𝐸 = 6‬‬
‫‪ ‬مةثفة فارية سعها 𝐹𝜇 ‪.𝐶 = 500‬‬ ‫‪ ‬فولط مرت رمق ‪.‬‬ ‫‪ ‬جت زي مدمع حلاسوب 𝑂𝐴𝑥𝐸‪.‬‬
‫‪ ‬وشميعة ذاتيها 𝐿 و مقاومها ادلاخلية 𝑟‬ ‫‪ ‬أأمبري مرت رمق ‪.‬‬ ‫‪ ‬حاسوب‪.‬‬

‫اانرتح ا ألسمتاذ عىل اجملتوعتني ما ييل ‪:‬‬


‫اجملتوعة ا ألوىل ‪ :‬اباد انمية مقاومة الناان ا ألويم ‪.‬‬
‫بعد تركيب ادلار املوحضة س الشلك (‪ )5‬و يل القاطعة عند اللحظة ‪.𝑡 = 0‬‬
‫[‪ ]1‬اانرتح طريقة جتريبية متة من تتبع تطور شد التيار )𝑡(𝑖 املار س ادلار بد ةل الزمن؟‬
‫[‪ ]2‬أأوجد املعادةل التفاضلية اليت حيقق ا )𝑡( 𝑐𝑢 بني طرس املةثفة ؟‬
‫الشلك (‪)5‬‬ ‫[‪ ]3‬اذا علت أأن العبار ) 𝑡𝛼 𝑒 ‪ 𝑢𝑐 (𝑡) = 𝐴(1 −‬ح للتعادةل التفاضلية جد عبار لك من 𝐴 و 𝛼‪.‬‬
‫)𝒕( 𝒄𝒖‬ ‫[‪ ]4‬أأكتب عبار )𝑡( 𝑅𝑢 مث اسمتنتج عبار )𝑡(𝑖‪.‬‬
‫)𝒕( 𝑹𝒖‬ ‫[‪ ]5‬ستتتتتتمتع ل التج تتتتتتزي متتتتتتدمع حلاستتتتتتوب 𝑂𝐴𝑥𝐸 و بواستتتتتتطة برجميتتتتتتة خاصتتتتتتة‬
‫الشلك (‪)6‬‬ ‫)𝑡( 𝑐𝑢‬
‫ف تحص عىل املنحىن املوحض س الشلك (‪.)6‬‬ ‫ندرس تغريات ‪:‬‬
‫)𝑡( 𝑅𝑢‬
‫)𝑡( 𝑐𝑢‬ ‫‪𝑡⁄‬‬
‫𝑒 ‪= −1 +‬‬ ‫𝜏‬ ‫أأ ‪ -‬أأثب أأ ّن ‪:‬‬
‫)𝑡( 𝑅𝑢‬
‫ب ‪ -‬استتتتمتنتج متتتتن البيتتتتان 𝜏 اثبتتتت التتتتزمن لثنتتتتا القطتتتتب املتتتتدروس مث حتقتتتت‬
‫أأ ّن ‪.𝑅 = 40Ω‬‬
‫𝟏‬
‫)𝒔𝒎(𝒕‬ ‫[‪ ]6‬أأحسب الطاانة ا زنة س املةثفة عند هناية معلية الشحن‪.‬‬
‫𝟎𝟏‬
‫اجملتوعة الثانية ‪ :‬اباد انمية لك من اذلاتية 𝐿 و املقاومة ادلاخلية 𝑟 للوشميعة‪.‬‬
‫بعتتتتتتد تركيتتتتتتب ادلار املوحضتتتتتتة س الشتتتتتتلك (‪ )7‬ويلتتتتتت القاطعتتتتتتة عنتتتتتتد اللحظتتتتتتة ‪𝑡 = 0‬‬
‫حتصتتتتتل اجملتوعتتتتتة عتتتتتىل البيتتتتتان املقابتتتتت املتثتتتتت لتغتتتتتريات التتتتتتوتر )𝑡( 𝑏𝑢 بتتتتتني طتتتتترس‬
‫الوشميعة بد ةل الزمن‪.‬‬
‫[‪ ]1‬س غيتتتتتاب التج تتتتتزي املتتتتتدمع حلاستتتتتوب متتتتتا تتتتتو اجل تتتتتا اذلي ميةتتتتتن استتتتتمتع‬
‫الشلك (‪)7‬‬
‫ملشتتتتتا د تغتتتتتريات التتتتتتوتر )𝑡( 𝑏𝑢 بتتتتتني طتتتتترس الوشتتتتتميعة بتتتتتد ةل التتتتتزمن ؟ بت ّ‬
‫تتتتني‬
‫)𝑽( 𝒃𝒖‬ ‫طريقة توصيهل س ادلار للحصول عىل م حىن الشلك (‪.)8‬‬
‫الشلك (‪)8‬‬ ‫[‪ ]2‬أأوجد املعادةل التفاضلية اليت حيقق ا التوتر )𝑡( 𝑏𝑢‪.‬‬
‫[‪ّ ]3‬بني أأن عبار التوتر بني طرس الوشميعة تةتب عىل الشلك ‪:‬‬
‫𝑡‬
‫)𝑟 ‪𝑢𝑏 (𝑡) = 𝐼0 (𝑅𝑒 − ⁄𝜏′ +‬‬
‫حي ‪ 𝐼0‬شد التيار س النظام ادلامئ و ‪ 𝜏′‬اثب الزمن يطلب تعيني عبار لك مهن ‪.‬‬
‫[‪ ]4‬أأوجد من البيان انمية اثب الزمن ‪.𝜏′‬‬
‫𝐿‬
‫𝟏‬ ‫= ‪. 𝑡′‬‬ ‫𝑅‬
‫[‪ ]5‬أأثب أأ ّن فاصةل نقطة تقاطع امل س عند اللحظة ‪ 𝑡 = 0‬مع حمور ا أل م ة يه‬
‫𝟎𝟐‬ ‫)𝒔𝒎( 𝒕‬ ‫[‪ ]6‬أأحسب انمية لك من اذلاتية 𝐿 و املقاومة ادلاخلية 𝑟 للوشميعة‪.‬‬

‫الصفحة (‪ )6‬من (‪)10‬‬


‫المترين الثاين ‪:‬‬
‫املعطيات ‪ :‬اثب اجلذب العام 𝐼𝑆 ‪.𝐺 = 6,67. 10−11‬‬
‫‪ −(I‬بيّن ادلراسات النظرية اليت أأجرا تا لك متن ‪ :‬كيبلتر يتالييل ونيتوتن اماكنيتة وضتع مقتر اصتطناع س متدار حتول ا ألر لةتن تذب ادلراستات‬
‫انتظرت حىت يوم ‪ 4‬أأكتوبر ‪ 1957‬لتتجسد س اطالق أأول مقتر اصتطناع ‪ Spoutnik‬متن طترف الاحتتاد الستوفيايت ليتتواىل بعتد ا ارستال الةثتري متن‬
‫ا ألمقار الاصطناعية من اتل البدلان نذكر مهنا ثالثة أأمقار مبينة مبعلوماهتا س اجلدول الآيت‪ .‬نعترب أأن حركة ذب ا ألمقار الاصطناعية حول مركز ا ألر تت‬
‫س مسار دائري‪.‬‬
‫القتر‬ ‫ادلور‬ ‫نص انطر مسار حركة القتر‬ ‫اثب كيبلر‬
‫)𝑠 ‪𝑇 (103‬‬ ‫)𝑚 ‪𝑟(106‬‬ ‫)𝐼𝑆 ‪𝑘(10−14‬‬
‫𝐴 ‪𝑆𝑝𝑜𝑡 −‬‬ ‫‪48‬‬ ‫‪28,5‬‬
‫𝐴 ‪𝐺𝑖𝑜𝑣𝑒 −‬‬ ‫‪54‬‬
‫𝑡𝑎𝑠 ‪𝐴𝑙𝑐𝑜𝑚 −‬‬ ‫‪42,2‬‬

‫[‪ ]1‬ما و مرجع دراسة ذب ا ألمقار عرفه و أأذكر الفرضية املتعلقة هبذا املرجع؟‬
‫[‪ ]2‬نقرتح ثالثة مسارات افرتاضية حلركة القتر حول ا ألر عني املسار اذلي يتعار مع القانون ا ألول لةيبلر عل ؟‬

‫املسار (‪)3‬‬ ‫املسار (‪)2‬‬ ‫املسار (‪)1‬‬


‫مث أأحسب رسعة القتر 𝐴 ‪.𝑆𝑝𝑜𝑡 −‬‬ ‫أأكتب عبار رسعة القتر 𝑣 بد ةل ‪ :‬ادلور 𝑇 و نص القطر‬
‫𝑟‬ ‫[‪]3‬‬
‫ذكّتر لقانون الثال لةيبلر مث وظفه مل ء اجلدول الساب ‪.‬‬ ‫[‪]4‬‬
‫أأحد ا ألمقار املذكور س اجلدول الساب و مقر جيومسمتقر عينه مع التعلي مث أأذكر الشتروط الثالثة اليت حيقق ا‪.‬‬ ‫[‪]5‬‬
‫أأحسب كتةل ا ألر 𝑇𝑀‪.‬‬ ‫[‪]6‬‬

‫‪ −(II‬تعمتتتتتد حمتتتتر ت التوجيتتتته ل ألمقتتتتار الاصتتتتطناعية واملعتتتتدات ا ألختتتترى عتتتتىل بطتتتتارايت نوويتتتتة طاانتتتتة متحتتتترر متتتتن جتتتتراء انبعتتتتاث‬
‫‪ 238‬اثب التفة 𝜆‪.‬‬ ‫جسماميت 𝛼 من أأنوية البلوتونيوم املشع 𝑢𝑃‪94‬‬

‫[‪ ]1‬اكتب معادةل التحول النووي املمنذجة لتفة نوا البلوتونيوم ‪ 238‬للحصول عىل نوا اليورانيوم 𝑈𝑍𝐴 ‪.‬‬
‫[‪ ]2‬بني أأن املعادةل التفاضلية اليت ختضع ل ا عدد ا ألنوية املتفةةة 𝑑𝑁 للبلوتونيوم ‪ 238‬يه من الشلك ‪:‬‬
‫𝑑𝑁𝑑‬
‫‪+ 𝜆𝑁𝑑 = 𝜆𝑁0‬‬
‫𝑡𝑑‬
‫الشلك (‪)9‬‬ ‫حي ‪ 𝑁0‬و عدد أأنوية البلوتونيوم الابتدااية س العينة املشعة‪.‬‬
‫[‪ ]3‬اذا ن ح ذب املعادةل التفاضلية من الشلك ‪𝑁𝑑 = 𝐴 + 𝐵𝑒 −𝛼𝑡 :‬‬
‫أأ ‪ -‬أأوجد عبار الثواب ‪ 𝐵 𝐴 :‬و 𝛼‪.‬‬
‫ب ‪ -‬ما املدلول الفزياي للك من 𝛼 و 𝐴‪.‬‬
‫𝑁𝑑‬
‫[‪ ]4‬نث ) 𝑑𝑁(𝑓 = 𝑑𝑡𝑑 فتحص عىل البيان املوحض س الشلك (‪.)9‬‬
‫أأ ‪ -‬سمتغالل البيان اسمتنتج انمييت الثابتني 𝜆 و ‪.𝑁0‬‬
‫ب ‪ -‬عرف من نص العتر ‪ 𝑡1/2‬للعينة املشعة واحسب انميته‪.‬‬

‫الصفحة (‪ )7‬من (‪)10‬‬


‫‪ . 238‬تقتتتتدم تتتتذب البطاريتتتتة ختتتتالل متتتتد‬ ‫[‪ ]5‬حتتتتتتوي بطاريتتتتة أأحتتتتد ا أ‬
‫لمقتتتتار الاصتتتتطناعية عتتتتىل كتتتتتةل 𝑔𝐾 ‪ 𝑚 = 1,2‬متتتتن 𝑢𝑃‪94‬‬

‫اشمتغال ا اسمتطاعة ر اية متوسطة مقدار ا 𝑊 ‪ 𝑃 = 888‬مبردود ‪.𝑟 = 60%‬‬


‫أأ ‪ -‬احسب الطاانة اللكية الناجتة عن التفة اللك للةتةل 𝑚‪.‬‬
‫ب ‪ -‬اسمتنتج مد اشمتغال البطارية‪.‬‬
‫يعطى ‪:‬‬
‫‪𝑚( 42𝐻𝑒) = 4,00150 𝑢 ; 𝑚( 𝐴𝑍𝑈) = 234,04095 𝑢 ; 𝑚( 238‬‬
‫𝑢 ‪92𝑃𝑢 ) = 238,04768‬‬
‫‪−13‬‬ ‫‪23‬‬ ‫‪−1‬‬
‫‪1𝑀𝑒𝑉 = 1,6 × 10‬‬ ‫𝑙𝑜𝑚 ‪𝐽 ; 𝑁𝐴 = 6,02 × 10‬‬ ‫‪; 1𝑢 = 931,5 𝑀𝑒𝑉. 𝑐 −2‬‬

‫المترين الثال ‪:‬‬


‫مالحظة ‪ :‬هنت تأأثري ال واء ولك الاحتاك ت‪.‬‬
‫يتتتترتك جستتتتم نقطتتتت )𝑆( دون رسعتتتتة ابتداايتتتتة متتتتن النقطتتتتة 𝐴 ليزنلتتتت وفتتتت ختتتتط امليتتتت ا ألعظتتتتم 𝐵𝐴 ملستتتتمتو مااتتتت يصتتتتنع متتتتع ا ألفتتتت‬
‫اوية ‪ .𝛼 = 30°‬املسافة 𝐿 = 𝐵𝐴‪.‬‬
‫يتصتتتتت 𝐵𝐴 مماستتتتتميا س النقطتتتتتة 𝐵 مبستتتتتر دائتتتتتري )𝐶𝐵( مركتتتتتزب 𝑂 ونصتتتتت انطتتتتترب 𝑟 ييتتتتت تةتتتتتون النقتتتتتاط 𝐴 𝐵 𝐶 و 𝑂‬
‫مضن نفس املسمتوى الشاانويل والنقطتان 𝐵 و 𝐶 عىل نفس املسمتوى ا ألفق ‪ .‬الشلك (‪.)10‬‬

‫الشلك (‪)10‬‬

‫يعطى ‪ :‬كتةل اجلسم ‪.𝑟 = 2𝑚 𝐿 = 5 𝑚 𝑔 = 10 𝑚. 𝑠 −2 𝑚 = 0,2 𝐾𝑔 :‬‬


‫[‪ ]1‬أأوجد عبار رسعة اجلسم )𝑆( عند مرورب لنقطة 𝐵 بد ةل 𝐿 𝑔 و 𝛼‪ .‬مث أأحسب انميها‪.‬‬
‫[‪ ]2‬حدد خصاا شعاع الرسعة للجسم )𝑆( س النقطة 𝐶‪.‬‬
‫[‪ ]3‬أأوجتتتتتد بتتتتتد ةل 𝑚 𝑔 و 𝛼 عبتتتتتار شتتتتتد القتتتتتو التتتتتيت تطبق تتتتتا الطريتتتتت عتتتتتىل اجلستتتتتم )𝑆( ختتتتتالل انز انتتتتته عتتتتتىل املستتتتتمتوى‬
‫املاا ‪ .‬أأحسب انميها‪.‬‬
‫لتةن 𝐼 أأخف نقطة من املسار ادلائري )𝐶𝐵(‪ .‬مير اجلسم )𝑆( لنقطة 𝐼 لرسعة ‪.𝑣𝐼 = 7,37 𝑚. 𝑠 −1‬‬
‫[‪ ]4‬أأحسب شد القو اليت تطبق ا الطري عىل اجلسم )𝑆( عند النقطة 𝐼‪.‬‬
‫عند وصول اجلسم )𝑆( اىل النقطة 𝐶 يغادر املسار )𝐶𝐵( ليقفز س ال واء‪.‬‬
‫𝐶( املعتتتتتتادةل ادلياكرتيتتتتتتة )𝑥(𝑓 = 𝑦 ملستتتتتتار اجلستتتتتتم )𝑆(‪ .‬نأأختتتتتتذ مبتتتتتتد أأ ا أل م تتتتتتة )‪(𝑡 = 0‬‬ ‫[‪ ]5‬أأوجتتتتتتد س املعتتتتتت ) 𝑦𝐶‬
‫⃗⃗⃗⃗ ‪⃗⃗⃗⃗𝑥 ,‬‬
‫حلظة مغادر اجلسم النقطة 𝐶‪.‬‬
‫[‪ ]6‬يسقط اجلسم )𝑆( عىل املسمتوى ا ألفق املار لنقطتني 𝐵 و 𝐶 س النقطة 𝑀‪ .‬أأحسب املسافة 𝑀𝐶‪.‬‬

‫الصفحة (‪ )8‬من (‪)10‬‬


‫المترين التجرييب ‪:‬‬
‫اجلزء ا ألول‬
‫‪𝑝𝐾𝑎 +‬‬ ‫ي انياس انو القاعد انطالانا من اثب القاعدية اذلي نرمز بتت 𝑏𝐾 نتاك عالانتة مبتارش بتني 𝑎𝐾𝑝 و 𝑏𝐾𝑝 اثبت القاعديتة اللوياريمتيتة حيت‬
‫)𝑞𝑎( ‪𝐴(𝑎𝑞) + 𝐻2 0(𝑙) = 𝐴𝐻 + (𝑎𝑞) + 𝑂𝐻 −‬‬ ‫‪ . 𝑝𝐾𝑏 = 14‬من أأج تفاع ا ألساس 𝐴 مع املاء املمنذ ملعادةل ‪:‬‬
‫تةتب عبار 𝑏𝐾 لشلك ‪:‬‬
‫] ‪[𝐴𝐻 +‬‬
‫] ‪𝐾𝑏 = [𝑂𝐻 −‬‬
‫]𝐴[‬
‫و تأأخذ انمية اجلداء‬ ‫عند 𝑪‪𝟐𝟓°‬‬ ‫أ‬
‫ذا اجلزء من المترين اىل اباد انمية 𝒃𝑲𝒑 للثنااية )𝟑𝑯𝑵‪ (𝑵𝑯𝟒 /‬و الـتأكد مهنا‪ .‬مجيع القياسات مت‬
‫‪+‬‬
‫هيدف‬
‫الشاردي لل ء 𝟒𝟏‪ .𝑲𝒆 = 𝟏𝟎−‬هنت مجيع الشوارد الناجتة عن الترشد اذلايت لل ء‪.‬‬
‫𝐻𝑝‬ ‫‪ −(I‬نأأخذ عد حمالي متن ا ألمونيتا ‪ 𝑁𝐻3‬برتاكتزي اتلفتة و نقتيس س لك متر‬
‫انمية 𝐻𝑝‪ .‬النتاجئ مة تنا من رمس املنحىن البياين )]‪. 𝑝𝐻 = 𝑓(𝑙𝑜𝑔[𝑁𝐻3‬‬
‫الشلك (‪)11‬‬ ‫𝟎𝟔 ‪𝟏𝟏,‬‬ ‫[‪ ]1‬بني أأ ّن‪:‬‬
‫‪1‬‬
‫= 𝐻𝑝‬ ‫)] ‪(2𝑝𝐾𝑒 − 𝑝𝐾𝑏 + 𝑙𝑜𝑔[𝑁𝐻3‬‬
‫‪2‬‬
‫𝑏𝐾 و اثب القاعدية للثنااية ) ‪.(𝑁𝐻4 + /𝑁𝐻3‬‬ ‫حي‬
‫𝟓𝟐 ‪𝟏𝟏,‬‬ ‫[‪ ]2‬أأكتتتتتب املعتتتتادةل البيانيتتتتة املوافقتتتتة املنحتتتتىن البيتتتتاين املتتتتوحض س‬
‫الشلك (‪.)11‬‬
‫[‪ ]3‬جد انمية 𝑏𝐾𝑝‪ .‬و اسمتنتج انمية 𝑎𝐾𝑝‪.‬‬
‫‪ −(II‬متتن أأج ت التأأكتتد متتن النتتتاجئ الستتابقة نأأختتذ حملتتول متتا ‪ 𝑆0‬ل ألستتاس‬
‫𝟕 ‪−𝟎,‬‬ ‫] ‪𝐿𝑜𝑔[𝑁𝐻3‬‬ ‫‪ 𝑁𝐻3‬ذي الرتكزي املويل اجمل ول ‪ 𝐶0‬و ‪.𝑝𝐻 = 11,20‬‬
‫نأأختتتتتذ جحتتتتت 𝐿𝑚 ‪ 𝑉𝑏 = 10‬متتتتتن احمللتتتتتول الستتتتتاب ونقتتتتتوم مبعايرتتتتتته‬
‫بواستتتتتتتتطة حملتتتتتتتتول تتتتتتتت تتتتتتتتور ال يتتتتتتتتدروجني ) ‪ (𝐻3𝑂+ + 𝐶𝑙−‬ذي الرتكتتتتتتتتزي املتتتتتتتتويل ‪ .𝑐𝑎 = 10−2𝑚𝑜𝑙. 𝐿−1‬النتتتتتتتتتاجئ‬
‫املتحص علهيا مة تنا من رمس البيان ) 𝐴𝑉(𝑓 = 𝐻𝑝 املوحض س الشلك (‪.)12‬‬
‫‪14‬‬
‫[‪ ]1‬أأكتب معادةل تفاع املعاير ‪.‬‬
‫𝑯𝒑‬ ‫الشلك (‪)12‬‬ ‫[‪ ]2‬عتتتترف نقطتتتتة التاكفتتتتإ مث حتتتتدد احتتتتداثياهتا و أأحستتتتب تتتتتركزي‬
‫‪12‬‬
‫املويل 𝑎𝑐‪.‬‬
‫‪10‬‬ ‫[‪ ]3‬متتتتن بتتتتني الةواشتتتت التاليتتتتة متتتتا تتتتو الاكشتتتت املناستتتتب‬
‫‪8‬‬ ‫ل ذب املعاير ؟ عل ‪.‬‬
‫‪6‬‬
‫جمال التغري اللوين للاكش‬ ‫امس الاكش اللوين‬
‫]‪[3,1 − 4,4‬‬ ‫ال يليانتني‬
‫‪4‬‬ ‫]‪[4,8 − 6,4‬‬ ‫أأ ر اللكورو فينول‬
‫‪2‬‬ ‫]‪[6 − 7,6‬‬ ‫أأ رق الربوموتميول‬
‫)𝑳𝒎( 𝑨𝑽‬ ‫أ‬ ‫‪𝑁𝐻4 +‬‬
‫‪0‬‬ ‫النتاجئ‬ ‫انارن‬ ‫‪.‬‬‫𝐾𝑝‬ ‫𝑏‬ ‫مية‬ ‫ان‬ ‫من‬ ‫كد‬ ‫تأ‬ ‫مث‬ ‫(‬ ‫[‪ ]4‬جد انمية 𝑎𝑘𝑝 للثنااية )‬
‫‪𝑁𝐻3‬‬
‫‪0‬‬ ‫‪5‬‬ ‫‪10‬‬ ‫‪15‬‬ ‫‪20‬‬ ‫‪25‬‬ ‫‪30‬‬ ‫‪35‬‬ ‫‪40‬‬ ‫‪45‬‬
‫مع تر احملسوبة سابقا س اجلزء 𝐼 من المترين‪.‬‬
‫اجلزء الثاين‬
‫‪ −(I‬مركبان كميياايان 𝐹 و ‪ 𝐹′‬عبار عن حولني هل نفس الصيغة اجملتةل 𝑂‪.𝐶3𝐻8‬‬
‫يتفاعتتتتتت 𝑙𝑜𝑚 ‪ 0,1‬متتتتتتن الةحتتتتتتول 𝐹 متتتتتتع 𝑙𝑜𝑚 ‪ 0,1‬متتتتتتن تتتتتت ا يثانويتتتتتت 𝐻𝑂𝑂𝐶 ‪ 𝐶𝐻3‬فينتتتتتتتج عنتتتتتتد التتتتتتتوا ن 𝑔 ‪6,97‬‬
‫من الاسرت ذي الةتةل املولية ‪104 𝑔. 𝑚𝑜𝑙 −1‬‬
‫[‪ ]1‬عرف تفاع ا ألسرت ‪.‬‬

‫الصفحة (‪ )9‬من (‪)10‬‬


‫احسب مكية ماد ا ألسرت الناجت مث اسمتنتج مردود تفاع ا ألسرت ‪.‬‬ ‫[‪]2‬‬
‫ما و صن الةحول 𝐹 املسمتعت ؟‬ ‫[‪]3‬‬
‫أأكتب الصيغة نص املفصةل للةحول 𝐹 و اعط ستيته النظامية‪.‬‬ ‫[‪]4‬‬
‫اكتب معادةل تفاع ا ألسرت احلادث‪.‬‬ ‫[‪]5‬‬

‫‪ −(II‬نتتتتتتز 𝑙𝑜𝑚 ‪ 0,5‬متتتتتتن تتتتتت ا يثانويتتتتتت 𝐻𝑂𝑂𝐶‪ 𝐶𝐻3‬متتتتتتع 𝑙𝑜𝑚 ‪ 0,5‬متتتتتتن حتتتتتتول ‪ 𝐹′‬و نظيتتتتتت بعتتتتتت القطتتتتتترات‬
‫متتتتن تتتت الةربيتتتت مث نستتتتخن املتتتتزت ونتتتتتابع تطتتتتور التفاعتتتت مبعتتتتاير امحلتتتت املتبقتتتت عنتتتتد لك ستتتتاعة ‪ .‬النتتتتتاجئ التجريبيتتتتة املتحصتتتت‬
‫علهيا دون س اجلدول التايل ‪:‬‬
‫)‪𝑡(ℎ‬‬ ‫‪0‬‬ ‫‪1‬‬ ‫‪2‬‬ ‫‪3‬‬ ‫‪4‬‬ ‫‪5‬‬ ‫‪6‬‬ ‫‪7‬‬ ‫‪8‬‬ ‫‪9‬‬ ‫‪10‬‬
‫)𝑙𝑜𝑚( ‪𝑛𝐹′‬‬ ‫‪0,50 0,38 0,31 0,27 0,24 0,22 0,21 0,20 0,20 0,20 0,20‬‬
‫𝑙𝑜𝑚( 𝑟𝑒𝑡𝑠𝑒𝑛‬

‫ا يثانوي و 𝑟𝑒𝑡𝑠𝑒𝑛 متث عدد مو ت ا ألسرت املتشلك‪.‬‬ ‫حي ‪ 𝑛𝐹′ :‬متث عدد مو ت‬
‫[‪ ]1‬أأمك اجلدول‪.‬‬
‫[‪ ]2‬ارمس املنحىن )𝑡(𝑓 = 𝑟𝑒𝑡𝑠𝑒𝑛 مث اسمتخر م ه خاصيتني من خصاا تفاع ا ألسرت ‪.‬‬
‫[‪ ]3‬احسب مردود تفاع ا ألسرت ‪.‬‬
‫[‪ ]4‬اسمتنتج صن الةحول ‪ 𝐹′‬املسمتعت ‪.‬‬
‫[‪ ]5‬اكتب الصيغة نص املفصةل ل ألسرت‪.‬‬

‫الصفحة (‪ )10‬من (‪)10‬‬


‫امجل وريتتتتتتتتتتتتتة اجلزائريتتتتتتتتتتتتتة ادلميوانراطيتتتتتتتتتتتتتتتتة الشعبيتتتتتتتتتتتتتتتتتتتتتتتتتتة‬
‫السمنة ادلراسمية‪2023/2022 :‬‬ ‫و ار الرتبية الوطنية‬
‫الشعبة‪ 3 :‬ت ر‬ ‫مديرية الرتبية اجلزائر – غرب‬
‫اثنوية عبد امحليد دروا (مسروين ‪ )02‬أأو د فاي‬
‫التصحيح المنوذيج ملوضوع امتحان باكلوراي جترييب س ماد العلوم الفزياياية‬

‫العالمة‬
‫عنارص ا جابة ( املوضوع ا ألول)‬
‫جمز أأ اجملتوع‬
‫المترين ا ألول ‪:‬‬
‫𝒊‬ ‫𝐼(‪ −‬دراسة ثنا القطب 𝐶𝑅 ‪:‬‬
‫𝑹𝒖‬ ‫[‪ ]1‬ادلار املدروسة (الشلك املقاب )‬
‫[‪ ]2‬التفسري اجمل ري لعتلية حشن املةثفة‪ :‬بعد يل القاطعة حتدث جهر ج عية لاللةرتو ت‬
‫𝑬‬
‫احلر من اللبوس ا ألول للتةثفة (املربوط لقطب املوجب للتودل) اىل اللبوس الثاين و‬
‫𝒄𝒖‬ ‫ألن الالةرتو ت سمتطيع اجتيا املاد العا ةل حيدث ترا لاللةرتو ت‪.‬‬
‫[‪ ]3‬املعادةل التفاضلية‪:‬‬
‫حسب انانون مجع التوترات ‪𝑢𝑐 + 𝑢𝑅 = 𝐸 :‬‬
‫𝑖 𝑅 = 𝑅𝑢‬
‫𝑐𝑢𝑑‬
‫{‬ ‫𝑞𝑑‬ ‫𝑑‬ ‫𝑐𝑢𝑑‬ ‫⇒‬ ‫𝐶𝑅 ‪𝑢𝑐 +‬‬ ‫𝐸=‬
‫=𝑖‬ ‫𝐶 = ) 𝑐𝑢𝐶( =‬ ‫𝑡𝑑‬
‫𝑡𝑑 𝑡𝑑‬ ‫𝑡𝑑‬
‫[‪ ]4‬اباد الثواب 𝐴 و 𝜏‪:‬‬
‫𝑡‬
‫‪𝑢𝑐 (𝑡) = 𝐴 (1‬‬ ‫) 𝜏‪− 𝑒 −‬‬ ‫𝑡‬ ‫𝑡‪𝐴 −‬‬
‫{‬ ‫⇒‬ ‫𝐶𝑅 ‪𝐴 (1 − 𝑒 −𝜏 ) +‬‬ ‫𝐸=𝜏 𝑒‬
‫𝑡‪𝑑𝑢𝑐 𝐴 −‬‬ ‫𝜏‬
‫𝑒 =‬ ‫𝜏‬
‫𝑡𝑑‬ ‫𝜏‬
‫𝑡‬ ‫𝑡 𝐴‬ ‫𝑡‪𝐴 −‬‬ ‫𝑡‬
‫⇒‬ ‫𝐸 = 𝜏‪𝐴 − 𝐴𝑒 𝜏 + 𝑅𝐶 𝑒 −‬‬
‫‪−‬‬
‫⇒‬ ‫𝐶𝑅 ‪𝐴 +‬‬ ‫𝜏‪𝑒 𝜏 = 𝐸 + −𝐴𝑒 −‬‬
‫𝜏‬ ‫𝜏‬
‫‪ A = E :‬و ‪.τ = RC‬‬ ‫ومه‬
‫‪ τ‬متجانس مع الزمن‪:‬‬ ‫[‪ ]5‬الرب ان أأ ّن الثاب‬
‫‪uR‬‬ ‫]‪[U‬‬
‫=‪R‬‬ ‫= ]‪[R‬‬
‫‪i‬‬ ‫]‪[I‬‬ ‫]‪[U] [I‬‬
‫⇒ ]‪[τ] = [R]. [C‬‬ ‫‪i‬‬ ‫⇒‬ ‫]‪[I‬‬ ‫⇒‬ ‫= ]‪[τ‬‬ ‫×‬ ‫⇒‬ ‫]‪[τ] = [T‬‬
‫=‪C‬‬ ‫= ]‪[C‬‬ ‫]‪[I] [U‬‬
‫‪duc‬‬ ‫]‪[U‬‬ ‫]‪[T‬‬
‫{‬ ‫‪dt‬‬ ‫{‬ ‫]‪[T‬‬
‫و م ه الثاب 𝜏 متجانس مع الزمن‪.‬‬
‫[‪ ]6‬أأ – من البيان ‪:‬‬
‫‪τ = 40 s‬‬ ‫‪A = E = 25 V‬‬
‫ب – انمية املقاومة 𝑅 ‪:‬‬
‫𝜏‬ ‫‪40‬‬
‫⇒ 𝐶𝑅 = ‪τ‬‬ ‫=𝑅‬ ‫=‬ ‫‪= 2. 105 Ω‬‬
‫‪𝐶 200 × 10−6‬‬
‫‪ −(II‬حتديد مد اشمتغال املإان‬
‫[‪ ]1‬عبار 𝑠𝑡‪:‬‬
‫𝑠𝑡‬ ‫)𝑡( 𝑠𝑢‬ ‫𝑠𝑡‬ ‫)𝑡( 𝑆𝑢‬
‫⇒ ) 𝜏 ‪𝑢𝑆 (𝑡) = 𝐸 (1 − 𝑒 −‬‬ ‫‪1−‬‬ ‫𝜏 ‪= 𝑒−‬‬ ‫⇒‬ ‫‪𝑡𝑆 = −𝜏. 𝑙𝑛 (1 −‬‬ ‫)‬
‫𝐸‬ ‫𝐸‬
‫𝑉 ‪: 𝑢𝑠 = 15‬‬ ‫[‪ ]2‬من أأج‬
‫‪15‬‬
‫‪𝑡𝑆 = −40. 𝑙𝑛 (1 −‬‬ ‫𝑠 ‪) = 36,65‬‬
‫‪25‬‬
‫وصول ساكن الع ر اىل بيته أل ّن 𝑠 ‪.𝑡𝑆 < 80‬‬ ‫املصباح 𝐿 ينطفئ انب‬
‫جمز أأ اجملتوع‬
‫[‪ ]3‬القمية احلدية 𝑠𝑅 ملقاومة الناان ا ألويم ‪:‬‬
‫)𝑡( 𝑆𝑢‬ ‫𝑆𝑡‪−‬‬ ‫𝑆𝑡‪−‬‬
‫‪𝑡𝑆 = −𝜏. 𝑙𝑛 (1 −‬‬ ‫⇒ )‬ ‫=𝜏‬ ‫⇒‬ ‫= 𝑆𝑅‬
‫𝐸‬ ‫)𝑡( 𝑆𝑢‬ ‫)𝑡( 𝑆𝑢‬
‫) 𝐸 ‪𝑙𝑛 (1 −‬‬ ‫‪𝐶. 𝑙𝑛 (1 −‬‬ ‫) 𝐸‬
‫‪𝑡𝑠 ≥ 80 𝑠 :‬‬ ‫من أأج‬
‫‪−80‬‬
‫= 𝑆𝑅‬ ‫‪= 4,4 × 105 Ω‬‬
‫‪15‬‬
‫) ‪200. 𝑙𝑛 (1 −‬‬
‫‪25‬‬
‫المترين الثاين ‪:‬‬
‫‪−(I‬‬
‫[‪ ]1‬ميةن اعتبار حركة مركز عطاةل أأحد تدفقات امحلم الرب نية سقوط ًا حرا ألهنا خاضعة فقط لقو ثقل ا (تأأثري دافعة ارمخيدس وانو‬
‫الاحتاكك مع ال واء همتةل)‪.‬‬
‫[‪ ]2‬عبار )𝑡( 𝑥𝑎 و )𝑡( 𝑦𝑎 س مع ادلراسة ‪:‬‬
‫امجلةل املدروسة ‪ :‬مركز عطاةل النقطة املادية )𝑆(‬
‫مرجع ادلراسة ‪ :‬السطح ا ألريض اذلي نعتربب عطايل‬
‫القوى املإثر ‪ :‬انو الثق ⃗⃗⃗‬
‫𝑃‬
‫بتطبي القانون الثاين لنيوتن ‪:‬‬
‫⃗⃗⃗⃗⃗⃗⃗‬
‫𝐹∑‬ ‫⃗⃗⃗⃗ ‪𝑒𝑥𝑡 = 𝑚.‬‬
‫𝑎 ‪⃗⃗⃗ = 𝑚.‬‬
‫𝑃 ⇒ 𝐺𝑎‬ ‫⇒ ⃗⃗⃗‬ ‫𝑔 ‪𝑚.‬‬
‫⃗⃗⃗ ‪⃗⃗⃗ = 𝑚.‬‬
‫𝑎‬ ‫⇒‬ ‫𝑔 = ⃗⃗⃗‬
‫𝑎‬ ‫⃗⃗⃗‬
‫)𝑦𝑂( جند‪:‬‬ ‫)‪(Ox‬‬ ‫سقاط وف حموري ادلراسة‬
‫‪𝑂𝑥 ∶ 𝑎𝑥 = 0‬‬
‫𝑔‪{𝑂𝑦 ∶ 𝑎 = −‬‬
‫𝑦‬
‫[‪ ]3‬مركبيت الرسعة اللحظية )𝑡( 𝑥𝑣 و )𝑡( 𝑦𝑣‪.‬‬
‫التاكم جراء‬
‫جراء التاكم ‪𝑂𝑥 ∶ 𝑎𝑥 = 0‬‬ ‫𝛼 ‪𝑂𝑥 ∶ 𝑣𝑥 (𝑡) = 𝑣0 cos‬‬
‫→ 𝑔‪{𝑂𝑦 ∶ 𝑎 = −‬‬ ‫{‬
‫𝑦‬ ‫𝛼 ‪𝑂𝑦 ∶ 𝑣𝑦 (𝑡) = −𝑔. 𝑡 + 𝑣0 sin‬‬
‫[‪ ]4‬عباريت مركبيت شعاع املوضع ‪:‬‬
‫𝛼 ‪𝑂𝑥 ∶ 𝑣𝑥 (𝑡) = 𝑣0 cos‬‬ ‫جراء التاكم‬ ‫𝑡 ‪𝑂𝑥 ∶ 𝑥(𝑡) = 𝑣0 cos 𝛼 .‬‬
‫{‬ ‫→‬ ‫{‬ ‫‪1‬‬
‫𝛼 ‪𝑂𝑦 ∶ 𝑣𝑦 (𝑡) = −𝑔. 𝑡 + 𝑣0 sin‬‬ ‫𝑡 ‪𝑂𝑦 ∶ 𝑦(𝑡) = − 𝑔. 𝑡 2 + 𝑣0 sin 𝛼 .‬‬
‫‪2‬‬
‫معادةل املسار )𝑥(𝑓 = 𝑦‬ ‫[‪]5‬‬
‫)‪x(t‬‬
‫⇒ ‪x(t) = v0 cos α . t‬‬ ‫=‪t‬‬
‫‪v0 cos α‬‬
‫‪1‬‬ ‫𝑔‬
‫𝑡 ‪𝑦(𝑡) = − 𝑔. 𝑡 2 + 𝑣0 sin 𝛼 .‬‬ ‫⇒‬ ‫‪𝑦=−‬‬ ‫𝑥 ‪𝑥 2 + tan 𝛼 .‬‬
‫‪2‬‬ ‫𝛼 ‪2𝑣0 cos2‬‬
‫‪2‬‬

‫املتتتتدى تتتتو أأبعتتتتد مستتتتافة أأفقيتتتتة يقطع تتتتا اجلستتتتم املقتتتتذوف أأو املستتتتافة ا ألفقيتتتتة بتتتتني موضتتتتع القتتتتذف وموضتتتتع‬ ‫[‪]6‬‬
‫السقوط‪.‬‬
‫من البيان جند ‪𝐿 = 1800 𝑚 :‬‬
‫اذلرو يه أأانىص ارتفاع تص اليه القذيفة‪.‬‬ ‫[‪]7‬‬
‫من البيان ‪𝑆 = 1000 𝑚 :‬‬
‫الزمن اذلي اسمتغرانته امحلم الرب نية عندما للوصول اىل أأبعد نقطة انطعها‪.‬‬ ‫[‪]8‬‬
‫من البيان ‪𝑡 = 22 𝑠 :‬‬
‫حساب ‪ 𝑣0‬الرسعة الابتدااية للحتم الرب نية ‪:‬‬ ‫[‪]9‬‬
‫)𝑡(𝑥‬ ‫‪1800‬‬
‫⇒ 𝑡 ‪𝑥(𝑡) = 𝑣0 𝑐𝑜𝑠 𝛼 .‬‬ ‫= ‪𝑣0‬‬ ‫=‬ ‫‪= 111,9 𝑚. 𝑠 −1‬‬
‫‪𝑡 𝑐𝑜𝑠 𝛼 22 × 𝑐𝑜𝑠 43‬‬
‫جمز أأ اجملتوع‬
‫𝐈𝐈(‪−‬‬
‫[‪ ]4‬من أأج شعاع الرسعـة الابتدااية موجه شاانوليا و ا ألعىل ‪α = 90° :‬‬
‫𝑚 ‪𝑥(𝑡) = 0‬‬
‫‪𝑣𝑥 (𝑡) = 0 𝑚. 𝑠 −1‬‬
‫⇒ ‪α = 90°‬‬ ‫{‬ ‫⇒ ‪α = 90°‬‬ ‫{‬ ‫‪1‬‬
‫‪𝑣𝑦 (𝑡) = −𝑔. 𝑡 + 𝑣01‬‬ ‫𝑡 ‪𝑦(𝑡) = − 𝑔. 𝑡 2 + 𝑣01 .‬‬
‫‪2‬‬
‫نقطة املبد أأ ‪ O‬اىل أأانصتى ارتفاع ‪.ℎ‬‬ ‫الزمن ‪ t‬اذلي سمتغرانه امجلةل للوصول انطالانا من‬ ‫[‪]5‬‬
‫‪ ℎ‬تنعدم مركبة الرسعة )𝑡( 𝑦𝑣‬ ‫عند أأانصتى ارتفاع‬
‫‪𝑣01‬‬
‫⇒ ‪𝑣𝑦 (𝑡) = 0‬‬ ‫‪−𝑔. 𝑡 + 𝑣01‬‬ ‫⇒ ‪=0‬‬ ‫=𝑡‬
‫𝑔‬
‫[‪ ]6‬عبار الرسعـة الابتدااية‪:‬‬
‫‪1‬‬
‫𝑡 ‪𝑦(𝑡) = − 𝑔. 𝑡 2 + 𝑣01 .‬‬ ‫‪1‬‬ ‫‪𝑣01 2‬‬ ‫‪𝑣01‬‬ ‫‪1 𝑣01 2‬‬
‫{‬ ‫‪2‬‬ ‫⇒‬ ‫‪ℎ‬‬ ‫=‬ ‫‪−‬‬ ‫‪𝑔.‬‬ ‫(‬ ‫)‬ ‫‪+‬‬ ‫𝑣‬ ‫‪.‬‬ ‫(‬ ‫)‬ ‫=‬
‫‪𝑣01‬‬ ‫‪2‬‬ ‫𝑔‬ ‫‪01‬‬
‫𝑔‬ ‫𝑔 ‪2‬‬
‫=𝑡‬
‫𝑔‬
‫‪⇒ 𝑣01 = √2. 𝑔. ℎ = √2 × 9,81 × 400 = 88,6 𝑚. 𝑠 −1‬‬
‫المترين الثال‬
‫اجلزء ا ألول‪:‬‬
‫‪ −(I‬دراسة نوا اليورانيوم ‪𝟗𝟐𝐔 : 238‬‬
‫𝟖𝟑𝟐‬

‫[‪ ]4‬بتطبي انانوين الا فاظ حدد لك من العددين الصحيحني 𝑥 و 𝑦 املشار الهي س املعادةل احلصيةل‪.‬‬
‫‪238‬‬ ‫‪206‬‬
‫→ ‪92U‬‬ ‫‪82Pb +‬‬ ‫‪x −10e + y 42He‬‬
‫⇒ ‪238 = 206 + 4y‬‬ ‫‪y=8‬‬ ‫‪ ‬ا فاظ العدد الةتيل ‪: A‬‬
‫‪92 = 82 − x + 2y‬‬ ‫⇒‬ ‫‪x=6‬‬ ‫‪ ‬ا فاظ العدد الشحين ‪: Z‬‬
‫‪238‬‬
‫→ ‪92U‬‬
‫‪206‬‬
‫‪82Pb +‬‬ ‫‪6 −10e + 8 42He‬‬ ‫ومه‪:‬‬
‫[‪ ]5‬تركيب نوا اليورانيوم ‪: 238‬‬
‫‪𝑁 = 𝐴 − 𝑍 = 238 − 92 = 146‬‬ ‫عدد النرتو ت ‪:‬‬ ‫‪‬‬ ‫‪ ‬عدد الربوتو ت ‪𝑍 = 92 :‬‬
‫[‪ ]6‬طاانة الربط لنسمبة لنوا اليورانيوم ‪: 238‬‬
‫‪𝐸𝑙 ( 238‬‬ ‫‪2‬‬ ‫‪238‬‬
‫𝑐 ‪92𝑈) = ∆𝑚. 𝑐 = [𝑍𝑚𝑝 + (𝐴 − 𝑍)𝑚𝑛 − 𝑚( 92𝑈)].‬‬
‫‪2‬‬
‫‪238‬‬
‫‪𝐸𝑙 ( 92𝑈) = [92 × 1,00728 + 146 × 1,00866 − 238,00031] × 931,5‬‬
‫‪𝐸𝑙 ( 238‬‬
‫𝑉𝑒𝑀 ‪92𝑈) = 1801,344015‬‬
‫‪238‬‬
‫𝑈‪92‬‬ ‫أل ّن ‪:‬‬‫‪206‬‬
‫𝑏𝑃‪82‬‬ ‫أأكرث اسمتقرارا من النوا‬ ‫نوا‬
‫‪𝐸𝑙 238‬‬ ‫‪1801,344015‬‬
‫= )𝑈‪( 92‬‬ ‫𝑛𝑜‪= 7,57 𝑀𝑒𝑉⁄𝑛𝑢𝑐𝑙é‬‬ ‫‪𝐸𝑙 238‬‬ ‫𝑙𝐸‬
‫{‬ ‫𝐴‬ ‫‪238‬‬ ‫⇒‬ ‫‪( 92𝑈) < ( 206‬‬‫)𝑏𝑃‬
‫‪𝐸𝑙 206‬‬ ‫𝐴‬ ‫‪𝐴 82‬‬
‫𝑛𝑜‪( 𝑃𝑏) = 7,87 𝑀𝑒𝑉⁄𝑛𝑢𝑐𝑙é‬‬
‫‪𝐴 82‬‬
‫𝐈𝐈(‪ −‬اتر خصر معدنية بواسطة اليورانيوم ‪ -‬الرصا ‪:‬‬
‫[‪ ]4‬خصاا النشاط الاشعاع ‪ :‬تلقا حمت عشوا و مسمتق عن العوام اخلارجية (مث الضغط و درجة احلرار )‪.‬‬
‫[‪ ]5‬تعبري معر الصخر املعدنية ‪:‬‬
‫𝑡𝜆‪𝑁 (𝑡) = 𝑁𝑢 (0)𝑒 −‬‬
‫𝑢 {‬ ‫𝑡𝜆‪⇒ 𝑁𝑢 (𝑡) = [𝑁𝑢 (𝑡) + 𝑁𝑃𝑏 (𝑡)]𝑒 −‬‬
‫)𝑡( 𝑏𝑃𝑁 ‪𝑁𝑢 (0) = 𝑁𝑢 (𝑡) +‬‬
‫)𝑡( 𝑢𝑁‬ ‫)𝑡( 𝑢𝑁‬
‫( = 𝑡𝜆‪⇒ 𝑒 −‬‬ ‫⇒ )‬ ‫( 𝑛𝑙 = 𝑡𝜆‪−‬‬ ‫)‬
‫𝑢𝑁‬ ‫)𝑡(‬ ‫𝑏𝑃𝑁 ‪+‬‬ ‫)𝑡(‬ ‫𝑢𝑁‬ ‫)𝑡(‬ ‫)𝑡( 𝑏𝑃𝑁 ‪+‬‬
‫)𝑡( 𝑢𝑚‬
‫‪1‬‬ ‫)𝑡( 𝑏𝑃𝑁 ‪𝑁𝑢 (𝑡) +‬‬ ‫‪𝑡1/2‬‬ ‫)𝑡( 𝑏𝑃𝑁‬ ‫‪𝑡1/2‬‬ ‫‪𝑀( 206‬‬
‫) 𝑏𝑃‪82‬‬
‫( 𝑛𝑙 = 𝑡 ⇒‬ ‫=)‬ ‫‪𝑙𝑛 (1 +‬‬ ‫=)‬ ‫‪𝑙𝑛 1 +‬‬
‫𝜆‬ ‫)𝑡( 𝑢𝑁‬ ‫)‪𝑙𝑛(2‬‬ ‫)𝑡( 𝑈𝑁‬ ‫)‪𝑙𝑛(2‬‬ ‫)𝑡( 𝑢𝑚‬
‫‪238‬‬
‫(‬ ‫(𝑀‬ ‫) )𝑈‪92‬‬
‫‪𝑡1⁄‬‬ ‫‪238‬‬
‫)𝑈‪𝑚𝑃𝑏 (𝑡) 𝑀( 92‬‬
‫‪2‬‬
‫=𝑡‬ ‫‪𝑙𝑛 (1 +‬‬ ‫)‬
‫)‪𝑙𝑛(2‬‬ ‫‪𝑚𝑈 (𝑡) 𝑀( 206‬‬
‫) 𝑏𝑃‪82‬‬
‫جمز أأ اجملتوع‬
‫[‪ ]6‬حساب 𝑡 لسمنة‪.‬‬
‫‪𝑡1⁄‬‬ ‫‪𝑚𝑃𝑏 (𝑡) 𝑀( 238‬‬
‫)𝑈‪92‬‬ ‫‪4,5. 109‬‬ ‫‪10−3 × 238‬‬
‫‪2‬‬
‫=𝑡‬ ‫‪𝑙𝑛 (1 +‬‬ ‫)‬ ‫=‬ ‫𝑛𝑙‬ ‫‪(1‬‬ ‫‪+‬‬ ‫𝑠𝑛𝑎 ‪) = 7,5 × 104‬‬
‫)‪𝑙𝑛(2‬‬ ‫‪𝑚𝑈 (𝑡) 𝑀( 206‬‬
‫‪82‬‬‫𝑏𝑃‬ ‫)‬ ‫𝑛𝑙‬ ‫)‪(2‬‬ ‫‪100‬‬ ‫×‬ ‫‪206‬‬
‫اجلزء الثاين‪:‬‬
‫𝐴‬
‫𝑜𝑃𝑍‬ ‫[‪ ]5‬معادةل تفة البولونيوم‬
‫‪A‬‬ ‫‪206‬‬
‫→ ‪ZPo‬‬ ‫‪82Pb‬‬ ‫‪+ 42He‬‬
‫⇒ ‪A = 206 + 4‬‬ ‫‪A = 210‬‬ ‫‪ ‬ا فاظ العدد الةتيل 𝐴 ‪:‬‬
‫‪Z = 82 + 2‬‬ ‫‪⇒ x = 84‬‬ ‫‪ ‬ا فاظ العدد الشحين 𝑍 ‪:‬‬
‫‪210‬‬
‫→ ‪84Po‬‬
‫‪206‬‬
‫‪82Pb + 2He‬‬
‫‪4‬‬ ‫ومه‪:‬‬
‫[‪ ]6‬أأ‪ -‬الطاانة احملرر 𝑏𝑖𝐿𝐸 لوحد 𝑉𝑒𝑀 خالل تفة نوا البولونيوم 𝑜𝑃𝑍𝐴‪.‬‬
‫‪ELib = (1,955372 − 1,955318) × 105 = 5,4 MeV‬‬
‫لةيلو غرام لنوا البولونيوم‪.‬‬ ‫ب ‪ -‬النق الةتيل‬
‫𝑚∆‬
‫‪𝐸𝑙 ( 210‬‬
‫)𝑜𝑃‪84‬‬ ‫‪= (1,971820 − 1,99372) × 10‬‬ ‫‪5‬‬
‫من ا طط ‪= 1644,8 𝑀𝑒𝑉 :‬‬
‫‪𝐸𝑙 ( 210‬‬
‫)𝑜𝑃‪84‬‬ ‫‪1644,8‬‬
‫‪𝐸𝑙 ( 210‬‬
‫𝑐 ‪84𝑃𝑜 ) = ∆𝑚.‬‬
‫‪2‬‬
‫= 𝑚∆ ⇒‬ ‫‪2‬‬
‫=‬ ‫𝑢 ‪= 1,76575‬‬
‫𝑐‬ ‫‪931,5‬‬
‫𝑢‪1‬‬ ‫→‬ ‫𝑔𝐾 ‪1,67 × 10−27‬‬
‫→ 𝑢 ‪1,76575‬‬ ‫𝑚∆‬
‫𝑔𝐾 ‪∆𝑚 = 1,76575 × 1,67 × 10−27 = 2,95 × 10−27‬‬
‫اثب النشاط ا شعاع 𝜆 للبولونيوم‪.‬‬ ‫[‪]7‬‬
‫)‪𝑙𝑛(2‬‬ ‫)‪𝑙𝑛(2‬‬
‫=𝜆‬ ‫=‬ ‫‪= 5,8 × 10−8 𝑠 −1‬‬
‫‪𝑡1/2‬‬ ‫‪138 × 24 × 60 × 60‬‬
‫النشتتتتتتاط الابتتتتتتتدا لعينتتتتتتة متتتتتتن أأنويتتتتتتة البولونيتتتتتتوم تتتتتتو 𝑞𝐵 ‪ 𝐴0 = 3,5 × 1011‬حتتتتتتدد بوحتتتتتتد 𝑠𝑟𝑢𝑜𝐽‬ ‫[‪]8‬‬
‫𝑞𝐵 ‪𝐴1 = 3,7 × 104‬‬ ‫اللحظة ‪ 𝑡1‬اليت يةون فهيا نشاط ذب العينة و‬
‫‪1‬‬ ‫‪𝐴1‬‬ ‫‪1‬‬ ‫‪3,7 × 104‬‬
‫⇒ ‪𝐴1 (𝑡1 ) = 𝐴0 𝑒 −𝜆𝑡1‬‬ ‫‪𝑡1 = − 𝑙𝑛 ( ) = −‬‬ ‫𝑛𝑙‬ ‫(‬ ‫)‬
‫𝜆‬ ‫‪𝐴0‬‬ ‫‪5,8 × 10−8‬‬ ‫‪3,5 × 1011‬‬
‫𝑠𝑟𝑢𝑜𝐽 ‪𝑡1 = 2,77 × 108 𝑠 = 3,2 × 103‬‬
‫المترين التجرييب‬
‫اجلزء ا ألول‬
‫[‪ ]1‬ميةن متابعة ذا التحول عن طري انياس الناانلية النوعية أل ّن املزت التفاعيل حيتوي عىل شوارد )𝑞𝑎( ‪ 𝐶𝑟 3+‬و شوارد )𝑞𝑎( ‪𝐼 −‬‬
‫تراكزي ا تتغري مع مرور الزمن‪.‬‬
‫[‪]2‬‬
‫‪𝐼2 + 2𝑒̅ → 2 𝐼 −‬‬ ‫‪ ‬املعادةل النصفية للأكسد‬
‫̅𝑒‪𝐶𝑟 → 𝐶𝑟 3+ + 3‬‬ ‫‪ ‬املعادةل النصفية لالرجاع‬
‫) ‪(𝐼2 Τ𝐼 −‬‬ ‫)𝑟𝐶‪; (𝐶𝑟 3+ Τ‬‬ ‫‪ ‬الثناايتني ادلاخلتني س التفاع‬
‫[‪ ]3‬مكية املاد الابتدااية للتتفاعلني‪.‬‬
‫‪ ‬مكية املاد الابتدااية لت )𝑠(𝑟𝐶‬ ‫‪ ‬مكية املاد الابتدااية لت )𝑞𝑎( ‪𝐼2‬‬
‫‪𝑚 0,5‬‬ ‫)‪𝑛0 (𝐼2 ) = 𝐶. 𝑉 = (3. 10−2 )(0,25‬‬
‫= )𝑟𝐶( ‪𝑛0‬‬ ‫=‬
‫‪𝑀 52‬‬ ‫𝑙𝑜𝑚 ‪𝑛0 (𝐼2 ) = 5. 10−3‬‬
‫𝑙𝑜𝑚 ‪𝑛0 (𝐶𝑟) = 9,6. 10−3‬‬
‫[‪ ]4‬جدول التقدم ‪:‬‬
‫معادةل التفاع‬ ‫→ )‪2 Cr(s) + 3 I2 (aq‬‬ ‫)‪2 Cr 3+ (aq) + 6 I − (aq‬‬
‫احلاةل‬ ‫التقدم‬ ‫)𝑙𝑜𝑚𝑚(‬ ‫مكية املاد‬
‫‪ 𝑥 = 0‬الابتدااية‬ ‫‪9,6‬‬ ‫‪5‬‬ ‫‪0‬‬ ‫‪0‬‬
‫)𝑡(𝑥 الانتقالية‬ ‫𝑥‪9,6 − 2‬‬ ‫𝑥‪5 − 3‬‬ ‫𝑥‪2‬‬ ‫𝑥‪6‬‬
‫الهنااية‬ ‫𝑓𝑥‬ ‫𝑓𝑥‪9,6 − 2‬‬ ‫𝑓𝑥‪5 − 3‬‬ ‫𝑓𝑥‪2‬‬ ‫𝑓𝑥‪6‬‬
‫جمز أأ اجملتوع‬
‫𝑥‬ ‫[‪ ]5‬عبار الناانلية النوعية ‪ σ‬للتزت بد ةل التقدم‬
‫𝑟𝐶[ ‪3+‬‬
‫] ‪3+‬‬ ‫‪−‬‬
‫] 𝐼[ ‪−‬‬
‫𝑟𝐶𝜆 = 𝜎‬ ‫𝐼𝜆 ‪+‬‬
‫𝑥‪2‬‬ ‫𝑥‪2‬‬ ‫𝑥‪6‬‬
‫= ] ‪[𝐶𝑟 3+‬‬ ‫⇒‬ ‫) ( ‪𝜎 = 𝜆𝐶𝑟3+ ( ) + 𝜆𝐼−‬‬
‫𝑉‬ ‫𝑉‬ ‫𝑉‬
‫𝑥‪6‬‬
‫= ] ‪[𝐼 −‬‬
‫{‬ ‫𝑉‬
‫‪2. 𝜆𝐶𝑟3+ + 6. 𝜆𝐼−‬‬
‫(=𝜎‬ ‫𝑥)‬
‫𝑉‬
‫اجلدول ‪:‬‬ ‫‪‬‬
‫)𝑛𝑖𝑚( 𝑡‬ ‫‪0‬‬ ‫‪1‬‬ ‫‪2‬‬ ‫‪4‬‬ ‫‪6‬‬ ‫‪8‬‬ ‫‪12‬‬ ‫‪14‬‬ ‫‪16‬‬
‫) ‪𝜎 (𝑆. 𝑚−1‬‬ ‫‪0‬‬ ‫‪0,426‬‬ ‫‪0,657‬‬ ‫‪0,829‬‬ ‫‪0,876‬‬ ‫‪0,888‬‬ ‫‪0,892‬‬ ‫‪0,892‬‬ ‫‪0,892‬‬
‫)𝑙𝑜𝑚𝑚( 𝑥‬ ‫‪0‬‬ ‫‪1,19‬‬ ‫‪1,84‬‬ ‫‪2,32‬‬ ‫‪2,45‬‬ ‫‪2,49‬‬ ‫‪2,50‬‬ ‫‪2,50‬‬ ‫‪2,50‬‬
‫البيان ‪:‬‬ ‫‪‬‬

‫)𝒍𝒐𝒎𝒎( 𝒙‬
‫‪3‬‬

‫‪2,5‬‬

‫‪2‬‬

‫‪1,5‬‬

‫‪1‬‬

‫‪0,5‬‬
‫)𝒏𝒊𝒎(𝒕‬
‫‪0‬‬
‫‪0‬‬ ‫‪2‬‬ ‫‪4‬‬ ‫‪6‬‬ ‫‪8‬‬ ‫‪10‬‬ ‫‪12‬‬ ‫‪14‬‬ ‫‪16‬‬ ‫‪18‬‬ ‫‪20‬‬

‫[‪ ]6‬من نص التفاع ‪ t1/2‬و الزمن الال م لبلوغ التفاع نص تقدمه الهنا ‪.‬‬
‫‪xf 2,5‬‬ ‫جند ‪:‬‬ ‫من أأج ‪:‬‬
‫= = ) ‪x(t1/2‬‬ ‫‪= 1,25 mol‬‬ ‫‪t = t1/2‬‬
‫‪2‬‬ ‫‪2‬‬
‫‪t1/2 = 1,1 min‬‬ ‫نعني ذب القمية عىل البيان مث نسقط عىل حمور الزمن جند ‪:‬‬
‫𝑥𝑑 ‪1‬‬
‫[‪ ]7‬الرسعة احلجتية للتفاع يه تغري التقدم 𝑥 لنسمبة للزمن 𝑡 س وحد احلجوم 𝑉 و نةتب ‪𝑣𝑉𝑜𝑙 = 𝑉 𝑑𝑡 :‬‬
‫‪ ‬عند 𝑛𝑖𝑚 ‪: 𝑡 = 4‬‬ ‫‪ ‬عند ‪: 𝑡 = 0‬‬
‫𝑥𝑑 ‪1‬‬ ‫𝑥𝑑 ‪1‬‬
‫= )𝑛𝑖𝑚 ‪𝑣𝑉𝑜𝑙 (𝑡 = 4‬‬ ‫|‬ ‫= )‪𝑣𝑉𝑜𝑙 (𝑡 = 0‬‬ ‫|‬
‫𝑛𝑖𝑚 ‪𝑉 𝑑𝑡 𝑡=4‬‬ ‫‪𝑉 𝑑𝑡 𝑡=0‬‬
‫‪1‬‬ ‫)‪(2,32 − 3‬‬ ‫‪1‬‬ ‫)‪(0 − 3‬‬
‫= )𝑛𝑖𝑚‪𝑣𝑉𝑜𝑙 (𝑡 = 4‬‬ ‫×‬ ‫= )‪𝑣𝑉𝑜𝑙 (𝑡 = 0‬‬ ‫×‬
‫‪0,25‬‬ ‫)‪(4 − 8‬‬ ‫)‪0,25 (0 − 2‬‬
‫‪𝑣𝑉𝑜𝑙 (𝑡 = 4𝑚𝑖𝑛) = 0,68 𝑚𝑚𝑜𝑙. 𝑚𝑖𝑛−1‬‬ ‫‪𝑣𝑉𝑜𝑙 (𝑡 = 0) = 6 𝑚𝑚𝑜𝑙. 𝑚𝑖𝑛−1‬‬
‫[‪ ]8‬تت اان الرسعة احلجتية للتفاع مع مرور الزمن اسبب ‪:‬‬
‫‪ ‬تناان تراكزي ا ألفراد املتفاعةل س املزت التفاعيل مبرور الزمن‬
‫‪ ‬تناان مي امل س (تفسري بياين)‬
‫اجلزء الثاين‬
‫[‪ ]6‬الفاراداي و مكية الشحنة الة ربية للك مول من ا لةرتو ت‪.‬‬
‫[‪ ]7‬مةو ت العتود ‪:‬‬
‫‪ - 2‬حملول نرتات الةروم )𝑞𝑎() ‪(𝐶𝑟 + 3𝑁𝑂3‬‬
‫‪3+‬‬ ‫‪−‬‬
‫‪ – 1‬صفيحة من الةروم )𝑠(𝑟𝐶‬
‫‪ - 3‬حملول نرتات الفضة )𝑞𝑎() ‪(𝐴𝑔+ + 𝑁𝑂3−‬‬ ‫‪ - 4‬اجلرس ملح‬
‫جمز أأ اجملتوع‬
‫[‪ ]8‬دور اجلرس ملح ‪ :‬حتقي التوصي الة ر (يل ادلار ) و ض ن التعادل الة ر‬
‫[‪ ]9‬أأ ‪ -‬أأانطاب العتود‬
‫‪ ‬تتتتتأآلك صتتتتفيحة الةتتتتروم يتتتتدل عتتتتىل حتتتتدوث معليتتتتة أأكستتتتد وم تتتته ( صتتتتفيحة الةتتتتروم حملتتتتول نتتتترتات‬
‫الةروم ) متث القطب السالب للعتود‪.‬‬
‫‪ ‬ترستتتب معتتتدن الفضتتتة عتتتىل مستتتمتوى صتتتفيحة الفضتتتة يتتتدل عتتتىل حتتتدوث معليتتتة ارجتتتاع وم تتته ( صتتتفيحة‬
‫الفضة حملول نرتات الفضة ) متث القطب املوجب للعتود‪.‬‬
‫‪+ 𝐶𝑟/𝐶𝑟 3+ // 𝐴𝑔+ /𝐴𝑔 −‬‬ ‫‪ ‬الرمز الاصطاليح للعتود ‪:‬‬
‫ب‪-‬‬
‫→ 𝑔𝐴‬ ‫‪+‬‬
‫̅𝑒 ‪𝐴𝑔 +‬‬ ‫أ‬
‫* املعادةل النصفية للكسد (القطب السالب) ‪:‬‬
‫→ ̅𝑒‪𝐶𝑟 3+ + 3‬‬ ‫𝑟𝐶‬ ‫* املعادةل النصفية لالرجاع (القطب املوجب) ‪:‬‬
‫→ 𝑟𝐶 ‪3𝐴𝑔 +‬‬ ‫‪3+‬‬ ‫‪+‬‬
‫𝑟𝐶‪𝐴𝑔 + 3‬‬ ‫* املعادةل ا ج لية شمتغال العتود ‪:‬‬
‫أأ‪ -‬اجنز جدول تقدم التفاع احلاص ‪.‬‬ ‫[‪]10‬‬
‫معادةل التفاع‬ ‫→ )𝑞𝑎( ‪3𝐴𝑔(𝑠) + 𝐶𝑟 3+‬‬ ‫)𝑠(𝑟𝐶‪𝐴𝑔+ (𝑎𝑞) + 3‬‬
‫احلاةل‬ ‫التقدم‬ ‫مكية املاد )𝑙𝑜𝑚(‬
‫الابتدااية‬ ‫‪𝑥=0‬‬ ‫) ‪𝑛0 (𝐶𝑟 3+‬‬ ‫) ‪𝑛0 (𝐴𝑔+‬‬
‫بتتوفتتتر‬

‫بتتتوفتتر‬
‫الانتقالية‬ ‫)𝑡(𝑥‬ ‫𝑥 ‪𝑛0 (𝐶𝑟 3+ ) −‬‬ ‫𝑥 ‪𝑛0 (𝐴𝑔+ ) −‬‬
‫الهنااية‬ ‫𝑓𝑥‬ ‫𝑓𝑥 ‪𝑛0 (𝐶𝑟 3+ ) − 𝑥𝑓 𝑛0 (𝐴𝑔+ ) −‬‬
‫ب‪ -‬انمية 𝑡‪.Δ‬‬
‫‪𝑍. 𝑥. 𝐹 3 × 1,21. 10−3 × 96500‬‬
‫𝐹 ‪𝑄 = 𝐼. ∆𝑡 = 𝑍. 𝑥.‬‬ ‫= 𝑡∆ ⇒‬ ‫=‬
‫𝐼‬ ‫‪60 × 10−3‬‬
‫‪3‬‬
‫𝑛𝑖𝑚‪∆𝑡 = 5,8 × 10 𝑠 = 1ℎ 37‬‬
‫عنارص ا جابة ( املوضوع الثاين)‬
‫جمز أأ اجملتوع‬
‫المترين ا ألول ‪:‬‬
‫اجملتوعة ا ألوىل ‪ :‬اباد انمية مقاومة الناان ا ألويم ‪.‬‬
‫[‪ ]1‬ميةتتتتن تتبتتتتع تطتتتتور شتتتتد التيتتتتار )𝑡(𝑖 املتتتتار س ادلار بتتتتد ةل التتتتزمن عتتتتن طريتتتت متابعتتتتة )𝑡( 𝑅𝑢 التتتتتوتر بتتتتني‬
‫)𝑡( 𝑢‬
‫طرس الناان ا ألويم حي ‪𝑖(𝑡) = 𝑅𝑅 :‬‬
‫[‪ ]2‬املعادةل التفاضلية اليت حيقق ا )𝑡( 𝑐𝑢‬
‫حسب انانون مجع التوترات ‪𝑢𝑐 + 𝑢𝑅 = 𝐸 :‬‬
‫𝑖 𝑅 = 𝑅𝑢‬
‫𝑐𝑢𝑑‬ ‫‪1‬‬ ‫𝐸‬
‫{‬ ‫𝑞𝑑‬ ‫𝑑‬ ‫𝑐𝑢𝑑‬ ‫⇒‬ ‫‪+‬‬ ‫= 𝑐𝑢‬
‫=𝑖‬ ‫𝐶 = ) 𝑐𝑢𝐶( =‬ ‫𝑡𝑑‬ ‫𝐶𝑅‬ ‫𝐶𝑅‬
‫𝑡𝑑 𝑡𝑑‬ ‫𝑡𝑑‬
‫[‪ ]3‬عبار لك من 𝐴 و 𝛼‪.‬‬
‫) 𝑡𝛼 𝑒 ‪𝑢𝑐 (𝑡) = 𝐴(1 −‬‬ ‫𝐴‬ ‫𝐸‬
‫𝑐𝑢𝑑 {‬ ‫𝑡𝛼‬
‫⇒‬ ‫‪𝐴𝛼𝑒 𝛼𝑡 +‬‬ ‫= ) 𝑡𝛼 𝑒 ‪(1 −‬‬
‫𝑒𝛼𝐴‪= −‬‬ ‫𝐶𝑅‬ ‫𝐶𝑅‬
‫𝑡𝑑‬
‫𝐴‬ ‫𝐸‬ ‫𝑡𝛼 𝐴‬
‫⇒‬ ‫= 𝑡𝛼 𝑒𝛼𝐴 ‪−‬‬ ‫‪+‬‬ ‫𝑒‬
‫𝐶𝑅‬ ‫𝐶𝑅 𝐶𝑅‬
‫‪ 𝐴 = 𝐸 :‬و 𝐶𝑅‪.𝛼 = − 1⁄‬‬ ‫ومه‬
‫[‪ ]4‬عبار )𝑡( 𝑅𝑢 و عبار )𝑡(𝑖‪.‬‬
‫‪ ‬عبار )𝑡(𝑖‬ ‫‪ ‬عبار )𝑡( 𝑅𝑢‬
‫𝑅𝑢‬ ‫𝑡‪𝐸 −‬‬ ‫⇒ 𝑐𝑢 ‪𝑢𝑅 = 𝐸 −‬‬ ‫𝑒 ‪𝑢𝑅 = 𝐸 − 𝐸 (1 −‬‬
‫‪−𝑡⁄‬‬
‫)𝜏‬
‫= )𝑡(𝑖‬ ‫⇒‬ ‫𝜏‪𝑖(𝑡) = 𝑒 ⁄‬‬
‫𝑅‬ ‫𝑅‬ ‫‪−𝑡⁄‬‬
‫𝑒𝐸 = 𝑅𝑢 ⇒‬ ‫𝜏‬
‫جمز أأ اجملتوع‬
‫[‪ ]5‬أأ ‪-‬‬
‫‪−𝑡⁄‬‬ ‫‪−𝑡⁄‬‬
‫𝑒 ‪𝑢𝑐 = 𝐸 (1 −‬‬ ‫)𝜏‬ ‫𝑒 ‪𝑢𝑐 (𝑡) 𝐸 (1 −‬‬ ‫)𝜏‬
‫‪1‬‬ ‫‪𝑡⁄‬‬
‫{‬ ‫⇒‬ ‫=‬ ‫𝑡‪−‬‬ ‫=‬ ‫𝑡‪−‬‬ ‫𝑒 ‪− 1 = −1 +‬‬ ‫𝜏‬
‫𝑡‪−‬‬
‫𝜏‪𝑢𝑅 = 𝐸𝑒 ⁄‬‬ ‫)𝑡( 𝑅𝑢‬ ‫𝜏‪𝐸𝑒 ⁄‬‬ ‫𝜏‪𝑒 ⁄‬‬
‫ب‪-‬‬
‫انمية 𝑅 ‪:‬‬ ‫‪‬‬ ‫اثب الزمن 𝜏 ‪:‬‬ ‫‪‬‬
‫𝜏‬ ‫‪20 × 10−3‬‬ ‫)𝜏( 𝑐𝑢‬ ‫𝜏‬
‫⇒ 𝐶𝑅 = 𝜏‬ ‫=𝑅‬ ‫=‬ ‫‪= −1 + 𝑒 Τ𝜏 = 1,7‬‬
‫‪𝐶 500 × 10‬‬ ‫‪−6‬‬ ‫)𝜏( 𝑅𝑢‬
‫𝛺‪𝑅 = 40‬‬ ‫نعتتتني تتتذب القميتتتة مث نستتتقط عتتتىل حمتتتور التتتزمن جنتتتد‪:‬‬
‫𝑠𝑚 ‪𝜏 = 20‬‬
‫[‪ ]6‬الطاانة ا زنة س املةثفة عند هناية معلية الشحن‪.‬‬
‫‪1‬‬ ‫‪1‬‬
‫= 𝑥𝑎𝑚 𝑐𝐸‬ ‫𝐽 ‪𝐶 𝐸 2 = × (500 × 10−6 ) × 62 = 4,5 × 10−6‬‬
‫‪2‬‬ ‫‪2‬‬

‫اجملتوعة الثانية ‪ :‬اباد انمية لك من اذلاتية 𝐿 و املقاومة ادلاخلية 𝑟 للوشميعة‪.‬‬


‫[‪]1‬‬
‫س غيتتتتاب التج تتتتزي املتتتتدمع حلاستتتتوب متتتتا تتتتو اجل تتتتا ميةتتتتن‬
‫استتتتتتتمتع ل رامس الا تتتتتتتذا ذو ذاكتتتتتتتر ملشتتتتتتتا د تغتتتتتتتريات‬
‫التوتر )𝑡( 𝑏𝑢 بني طرس الوشميعة بد ةل الزمن‪.‬‬
‫[‪ ]2‬املعادةل التفاضلية اليت حيقق ا التوتر )𝑡( 𝑏𝑢‪.‬‬
‫حسب انانون مجع التوترات ‪ 𝑢𝑏 + 𝑢𝑅 = 𝐸 :‬شمتقاق ‪:‬‬
‫𝑅𝑢𝑑 𝑏𝑢𝑑‬ ‫𝑏𝑢𝑑‬ ‫𝑖𝑑‬
‫‪+‬‬ ‫⇒ ‪=0‬‬ ‫‪+𝑅 = 0‬‬
‫𝑡𝑑 {‬ ‫𝑡𝑑‬
‫𝑖 𝑅 = 𝑅𝑢‬ ‫𝑡𝑑‬ ‫𝑡𝑑‬
‫𝑖𝑑‬ ‫𝑅 𝑏𝑢𝑑‬
‫⇒ 𝑖 𝑟 ‪𝑢𝑏 = 𝐿 +‬‬ ‫‪+ (𝑢𝑏 − 𝑟 𝑖) = 0‬‬
‫𝑡𝑑‬ ‫𝑡𝑑‬ ‫𝐿‬
‫𝑅 𝑏𝑢𝑑‬ ‫𝑟‬
‫⇒‬ ‫‪+ 𝑢𝑏 − 𝑅 𝑖 = 0‬‬
‫𝑡𝑑‬ ‫𝐿‬ ‫𝐿‬
‫𝑖 𝑅 = 𝑅𝑢‬ ‫𝑅 𝑏𝑢𝑑‬ ‫𝑟‬
‫{‬ ‫⇒‬ ‫‪+ 𝑢𝑏 − (𝐸 − 𝑢𝑏 ) = 0‬‬
‫𝑏𝑢 ‪𝑢𝑅 = 𝐸 −‬‬ ‫𝑡𝑑‬ ‫𝐿‬ ‫𝐿‬
‫)𝑟 ‪𝑑𝑢𝑏 (𝑅 +‬‬ ‫𝑟‬
‫⇒‬ ‫‪+‬‬ ‫𝐸 = 𝑏𝑢‬
‫𝑡𝑑‬ ‫𝐿‬ ‫𝐿‬
‫[‪ ]3‬تعيني عبار شد التيار ‪: 𝐼0‬‬
‫𝑡‬
‫‪𝑢𝑏 (𝑡) = 𝐼0‬‬ ‫‪(𝑅𝑒 − ⁄𝜏′‬‬
‫)𝑟 ‪+‬‬ ‫)𝑟 ‪(𝑅 +‬‬ ‫)𝑟 ‪(𝑅 +‬‬ ‫‪𝐼0 𝑅 −𝑡⁄‬‬ ‫𝑟‬
‫𝑡‪−‬‬
‫𝑢𝑑 {‬ ‫‪𝐼0 𝑅 −𝑡⁄‬‬ ‫⇒‬ ‫‪𝐼0 𝑅𝑒 ⁄𝜏′ +‬‬ ‫= ‪𝑟𝐼0‬‬ ‫𝐸 ‪𝑒 𝜏′ +‬‬
‫𝑏‬
‫‪=−‬‬ ‫‪𝑒 𝜏′‬‬ ‫𝐿‬ ‫𝐿‬ ‫𝜏‬ ‫𝐿‬
‫𝑡𝑑‬ ‫𝜏‬
‫𝐿‬
‫= ‪𝜏′‬‬
‫𝑟‪𝑅+‬‬
‫{ ⇒‬ ‫𝐸‬
‫= ‪𝐼0‬‬
‫)𝑟 ‪(𝑅 +‬‬
‫[‪ ]4‬من البيان ‪:‬‬
‫𝑠𝑚 ‪𝜏 ′ = 20‬‬
‫[‪ ]5‬نقطة تقاطع امل س عند اللحظة ‪ 𝑡 = 0‬مع حمور ا أل م ة‪.‬‬
‫معادةل امل س ‪𝑦 = 𝑓 ′ (𝑥0 ). (𝑥 − 𝑥0 ) + 𝑓(𝑥0 ) :‬‬
‫‪𝑥0 = 0‬‬
‫𝑏𝑢𝑑‬ ‫𝑅 ‪𝐼0‬‬ ‫𝑅 ‪𝐼0‬‬
‫( = ) ‪𝑓′ (𝑥0‬‬‫)‬ ‫‪=−‬‬ ‫⇒‬ ‫‪𝑦=−‬‬ ‫)𝑟 ‪𝑡 + 𝐼0 (𝑅 +‬‬
‫‪𝑑𝑡 𝑡=0‬‬ ‫‪𝜏′‬‬ ‫‪𝜏′‬‬
‫)𝑟 ‪{𝑓(𝑥0 ) = 𝑢𝑏 (𝑡 = 0) = 𝐼0 (𝑅 +‬‬
‫عند نقطة التقاطع ‪ 𝑡 = 𝑡′ :‬و ‪𝑦 = 0‬‬
‫𝑅 ‪𝐼0‬‬ ‫‪𝜏′‬‬ ‫𝐿‬
‫⇒‬ ‫⇒ )𝑟 ‪0 = − ′ 𝑡 ′ + 𝐼0 (𝑅 +‬‬ ‫)𝑟 ‪𝑡′ = (𝑅 +‬‬ ‫⇒‬ ‫= ‪𝑡′‬‬
‫𝜏‬ ‫𝑅‬ ‫𝑅‬
‫جمز أأ اجملتوع‬
‫[‪]6‬‬
‫𝑟‬ ‫املقاومة ادلاخلية للوشميعة‬ ‫‪‬‬ ‫𝐿‬ ‫‪ ‬اذلاتية‬
‫دلينا ‪:‬‬ ‫متتتتن البيتتتتان‪ :‬نقطتتتتتة تقتتتتاطع املتتتت س عنتتتتتد ‪𝑡 = 0‬‬

‫= ‪𝜏′‬‬
‫𝐿‬
‫⇒ 𝑠𝑚 ‪= 20‬‬ ‫=𝑟‬
‫𝐿‬
‫𝑅‪−‬‬
‫مع حمور ا أل م ة‬
‫𝑟‪𝑅+‬‬ ‫‪𝜏′‬‬ ‫𝐿‬
‫‪0,96‬‬ ‫= ‪𝑡′‬‬ ‫𝑅 × ‪= 24 𝑚𝑠 ⇒ 𝐿 = 𝑡 ′‬‬
‫⇒‬ ‫=𝑟‬ ‫‪− 40‬‬ ‫𝑅‬
‫‪20 × 10−3‬‬ ‫‪⇒ 𝐿 = 24 × 10−3 × 40‬‬
‫‪𝑟 =8Ω‬‬ ‫𝐻 ‪𝐿 = 0,96‬‬

‫المترين الثاين ‪:‬‬


‫‪−(I‬‬
‫[‪ ]1‬املرجع املناسب دلراسة حركة ذب ا ألمقار و املرجع السطح ا ألريض و و مرجع مزود مبع مبد أأب مركز ا ألر وحماورب‬
‫مو ة و ثالث جنوم نعترب ا ساكنة‪ .‬نفر أأ ّن ذا املرجع حيق مبد أأ العطاةل و نعتربب ساكن أأو يتحرك يركة مسمتقمية م تظتة‬
‫طيةل مد ادلراسة و و ما بعهل مرجعا ياليليا‪.‬‬

‫[‪ ]2‬حسب القانون ا ألول لةيبلر تدور ا ألمقار الصناعية حول ا ألر س مدارات ا ليليجية حتت ا ألر أأحد حمرانهيا‪.‬‬
‫وم ه املسار (‪ )3‬و اذلي يتعار مع القانون ا ألول لةيبلر ألن مركز ا ألر ينطب مع مركز املسار‬
‫[‪ ]3‬عبار رسعة القتر 𝑣 ‪:‬‬
‫𝑟𝜋‪2‬‬ ‫) ‪2𝜋(28,5 × 106‬‬
‫=𝑣‬ ‫⇒‬ ‫= 𝐴‪𝑣𝑆𝑝𝑜𝑡−‬‬ ‫‪= 3,7 × 103 𝑚. 𝑠 −1‬‬
‫𝑇‬ ‫‪48 × 103‬‬
‫[‪ ]4‬القانون الثال لةيبلر ‪ :‬يت اسب مربع دور حركة ا ألمقار الصناعية حول ا ألر طرد ًا مع مةعب البعد املتوسط بني ا ألر‬
‫‪𝑇2‬‬
‫‪𝑟3‬‬
‫𝑘=‬ ‫والقتر الصناع ‪ .‬و نةتب ‪:‬‬

‫اجلدول ‪:‬‬ ‫‪‬‬

‫القتر‬ ‫ادلور‬ ‫نص انطر مسار حركة القتر‬ ‫اثب كيبلر‬


‫‪6‬‬
‫)𝑠 ‪𝑇 (10‬‬ ‫‪3‬‬
‫)𝑚 ‪𝑟(10‬‬ ‫)𝐼𝑆 ‪𝑘(10−14‬‬
‫𝐴 ‪𝑆𝑝𝑜𝑡 −‬‬ ‫‪48‬‬ ‫‪28,5‬‬ ‫‪9,95‬‬
‫𝐴 ‪𝐺𝑖𝑜𝑣𝑒 −‬‬ ‫‪54‬‬ ‫‪30,8‬‬ ‫‪9,95‬‬
‫𝑡𝑎𝑠 ‪𝐴𝑙𝑐𝑜𝑚 −‬‬ ‫‪86,5‬‬ ‫‪42,2‬‬ ‫‪9,95‬‬

‫[‪ ]5‬القتر جيومسمتقر و القتر 𝑡𝑎𝑠 ‪ 𝐴𝑙𝑐𝑜𝑚 −‬أل ّن نفس دور حركة ا ألر حول نفس ا ‪:‬‬
‫‪𝑇 = 86,5 × 103 𝑠 = 24 ℎ‬‬
‫الشتتتتتروط الثالثتتتتة التتتتيت حيقق تتتتا ‪ :‬نفتتتتس دور حركتتتتة ا ألر حتتتتول نفستتتت ا يتتتتدور س نفتتتتس تتتتة دوران‬
‫ا ألر حول نفس ا يقع عىل مسمتوى خط الاسمتواء ل ألر ‪.‬‬
‫[‪ ]6‬كتةل ا ألر 𝑇𝑀 ‪:‬‬
‫‪4𝜋 2‬‬ ‫‪4𝜋 2‬‬ ‫‪4𝜋 2‬‬
‫=𝑘‬ ‫= 𝑇𝑀 ⇒‬ ‫=‬ ‫𝑔𝐾 ‪⇒ 𝑀𝑇 = 5,9 × 1024‬‬
‫𝑇𝑀 ‪𝐺.‬‬ ‫) ‪𝐺. 𝑘 (6,67 × 10−11 ). (9,95 × 10−14‬‬

‫‪−(II‬‬
‫[‪ ]1‬معادةل التحول النووي املمنذجة لتفة نوا البلوتونيوم ‪: 238‬‬
‫‪238‬‬ ‫‪A‬‬
‫𝑢𝑃‪94‬‬ ‫→‬ ‫𝑈‪Z‬‬ ‫‪+ 42He‬‬
‫⇒ ‪238 = A + 4‬‬ ‫‪𝐴 = 234‬‬ ‫‪ ‬ا فاظ العدد الةتيل ‪: A‬‬
‫‪94 = Z + 2‬‬ ‫⇒‬ ‫‪𝑍 = 92‬‬ ‫‪ ‬ا فاظ العدد الشحين ‪: Z‬‬
‫‪238‬‬
‫𝑢𝑃‪94‬‬ ‫→‬ ‫‪234‬‬
‫𝑈‪92‬‬ ‫𝑒𝐻‪+ 42‬‬ ‫ومه‪:‬‬
‫جمز أأ اجملتوع‬
‫[‪ ]2‬املعادةل التفاضلية اليت ختضع ل ا عدد ا ألنوية املتفةةة 𝑑𝑁 للبلوتونيوم ‪ 238‬يه من الشلك ‪:‬‬
‫)𝑡(𝑁𝑑‬
‫𝑡𝜆‪= −𝜆𝑁0 𝑒 −‬‬ ‫𝑑𝑁𝑑‬
‫𝑡𝜆‪𝑁(𝑡) = 𝑁0 𝑒 −‬‬ ‫𝑡𝑑‬
‫{‬ ‫{ ⇒‬ ‫‪⇒ −𝜆𝑁(𝑡) = −‬‬
‫𝑑𝑁 ‪𝑁(𝑡) = 𝑁0 −‬‬ ‫)𝑡(𝑁𝑑‬ ‫𝑑𝑁𝑑‬ ‫𝑡𝑑‬
‫‪=−‬‬
‫𝑡𝑑‬ ‫𝑡𝑑‬
‫𝑑𝑁𝑑‬
‫‪⇒ −𝜆(𝑁0 − 𝑁𝑑 ) = −‬‬
‫𝑡𝑑‬
‫𝑑𝑁𝑑‬
‫⇒‬ ‫‪+ 𝜆𝑁𝑑 = 𝜆𝑁0‬‬
‫𝑡𝑑‬
‫حي ‪ 𝑁0‬و عدد أأنوية البلوتونيوم الابتدااية س العينة املشعة‪.‬‬
‫[‪ ]3‬اذا ن ح ذب املعادةل التفاضلية من الشلك ‪𝑁𝑑 = 𝐴 + 𝐵𝑒 −𝛼𝑡 :‬‬
‫أأ ‪ -‬أأوجد عبار الثواب ‪ 𝐵 𝐴 :‬و 𝛼‪.‬‬
‫𝑡𝛼‪𝑁𝑑 = 𝐴 + 𝐵𝑒 −‬‬
‫𝑑𝑁𝑑{‬ ‫‪⇒ −𝛼𝐵𝑒 −𝛼𝑡 + 𝜆(𝐴 + 𝐵𝑒 −𝛼𝑡 ) = 𝜆𝑁0‬‬
‫𝑡𝛼‪= −𝛼𝐵𝑒 −‬‬
‫𝑡𝑑‬
‫‪𝐴 = 𝑁0‬‬
‫{ ⇒ 𝑡𝛼‪⇒ 𝜆𝐴 + 𝜆𝐵𝑒 −𝛼𝑡 = 𝜆𝑁0 + 𝛼𝐵𝑒 −‬‬
‫𝜆=𝛼‬
‫من الرشوط الابتدااية‬
‫‪𝑁𝑑 = 𝐴 + 𝐵 = 0‬‬ ‫⇒‬ ‫𝐴‪𝐵 = −‬‬ ‫⇒‬ ‫‪𝐵 = −𝑁0‬‬
‫ومه‬
‫𝑡𝜆‪−‬‬
‫𝑒 ‪𝑁𝑑 = 𝑁0 (1 −‬‬ ‫)‬
‫ب‪-‬‬
‫البلوتونيتتتتتوم الابتداايتتتتتة س 𝜆 اثب التفة او اثب النشاط الاشعاع ‪.‬‬ ‫‪ 𝑁0‬تتتتتو عتتتتتدد أأنويتتتتتة‬
‫العينة املشعة‬
‫[‪ ]4‬أأ ‪ -‬سمتغالل البيان اسمتنتج انمييت الثابتني 𝜆 و ‪.𝑁0‬‬
‫معادةل البيان ‪:‬‬
‫𝑑𝑁𝑑‬
‫) ‪= −(2,5 × 10−10 )𝑁𝑑 + (6 × 1010‬‬
‫𝑡𝑑‬
‫املعادةل النظرية ‪:‬‬
‫𝑑𝑁𝑑‬
‫‪= −𝜆𝑁𝑑 + 𝜆𝑁0‬‬
‫𝑡𝑑‬
‫ملطابقة جند ‪:‬‬
‫‪𝜆 = 2,5 × 10−10 𝑠 −1‬‬
‫‪10‬‬
‫‪6 × 10‬‬ ‫‪6 × 1010‬‬
‫= ‪𝑁0‬‬ ‫=‬ ‫𝑥𝑢𝑎𝑦𝑜𝑛 ‪= 2,4 × 10−20‬‬
‫𝜆‬ ‫‪2,5 × 10−10‬‬
‫ب ‪ -‬من نص العتر و الزمن الال م لتفة نص عدد ا ألنوية الابتدااية املشعة و بقاء النص ‪.‬‬
‫)‪ln(2‬‬ ‫)‪ln(2‬‬
‫= ‪𝑡1/2‬‬ ‫=‬ ‫𝑠𝑛𝑎 ‪= 2,77 × 109 𝑠 = 87,9‬‬
‫𝜆‬ ‫‪2,5 × 10−10‬‬
‫[‪ ]5‬أأ ‪ -‬احسب الطاانة اللكية الناجتة عن التفة اللك للةتةل 𝑚‪.‬‬
‫𝑚‬
‫= 𝑏𝑖𝐿𝐸 ‪𝐸𝑇𝑜𝑡 = 𝑁.‬‬ ‫‪𝑁 (𝑚( 238‬‬ ‫𝐴‬ ‫‪4‬‬
‫𝑐 ‪92𝑃𝑢 ) − 𝑚( 𝑍𝑈) − 𝑚( 2𝐻𝑒)) .‬‬
‫‪2‬‬
‫𝐴 𝑀‬
‫‪1,2 × 103‬‬
‫= 𝑡𝑜𝑇𝐸‬ ‫‪× 6,02 × 1023 × (238,04768 − 234,04095 − 4,00150) × 931,5‬‬
‫‪238‬‬
‫𝐽 ‪𝐸𝑇𝑜𝑡 = 1,48 × 1025 𝑀𝑒𝑉 = 2,37 × 1012‬‬
‫ب ‪ -‬اسمتنتج مد اشمتغال البطارية‪.‬‬
‫𝑒𝑙𝑒𝐸‬
‫=𝑃‬ ‫‪𝑟. 𝐸𝑇𝑜𝑡 0,6 × 2,37 × 1012‬‬
‫𝑡∆‬
‫𝑒𝑙𝑒𝐸‬ ‫⇒‬ ‫𝑡∆‬ ‫=‬ ‫=‬
‫𝑃‬ ‫‪888‬‬
‫=𝑟‬
‫{‬ ‫𝑡𝑜𝑇𝐸‬
‫𝑠𝑛𝑎 ‪⇒ ∆𝑡 = 1,6 × 109 𝑠 = 50,8‬‬
‫جمز أأ اجملتوع‬
‫المترين الثال‬
‫[‪ ]1‬عبار شد القو اليت تطبق ا الطري عىل اجلسم )𝑆( خالل انز انه عىل املسمتوى املاا ‪.‬‬
‫س املرجع السطح ا ألريض املعترب عطايل بتطبي القانون الثاين لنيوتن عىل امجلةل "اجلسم )𝑆(" خالل انز انه عىل‬
‫𝑎𝑚 = 𝑡𝑥𝑒𝐹⃗⃗⃗ ∑‬ ‫𝑎𝑚 = ⃗𝑅 ‪⃗⃗⃗ ⇒ 𝑃⃗ +‬‬ ‫⃗⃗⃗‬ ‫املسمتوى املاا ‪:‬‬
‫)‪(𝑂𝑥) ∶ 𝑃. sin 𝛼 = 𝑚𝑎𝑥 … (1‬‬
‫{‬ ‫سقاط وف حموري احلركة ‪:‬‬
‫)‪(𝑂𝑦) ∶ −𝑃. cos 𝛼 + 𝑅 = 0 … (2‬‬
‫𝛼 ‪𝑅 = 𝑚. 𝑔. cos‬‬ ‫من (‪ )2‬جند ‪:‬‬
‫)‪𝑅 = 0,2 × 10 × cos(30°‬‬
‫𝑁 ‪𝑅 = 1,73‬‬
‫[‪ ]2‬شد القو اليت تطبق ا الطري عىل اجلسم )𝑆( عند النقطة 𝐼‪.‬‬
‫س املرجتتتتع الستتتتطح ا ألريض املعتتتتترب عطتتتتايل بتطبيتتتت القتتتتانون الثتتتتاين لنيتتتتوتن عتتتتىل امجلتتتتةل "اجلستتتتم )𝑆("‬
‫𝑎𝑚 = 𝑡𝑥𝑒𝐹⃗⃗⃗ ∑‬‫‪⃗⃗⃗ ⇒ 𝑃⃗ +‬‬ ‫ختتتتتتتتتالل انز انتتتتتتتتته عتتتتتتتتتىل املستتتتتتتتتمتوى ادلائتتتتتتتتتري ‪:‬‬
‫⃗⃗⃗⃗‬
‫𝑎𝑚 = 𝐼𝑅‬
‫⃗⃗⃗‬
‫𝑁𝑎𝑚 = 𝐼𝑅 ‪(𝑁) ∶ −𝑃 +‬‬ ‫سقاط وف احملور الناظت ‪:‬‬
‫‪2‬‬
‫𝐼𝑣‬ ‫‪7,37²‬‬
‫( 𝑚 = 𝐼𝑅‬ ‫( ‪+ 𝑔) = 0,2‬‬ ‫)‪+ 10‬‬
‫𝑟‬ ‫‪2‬‬
‫𝑁 ‪𝑅𝐼 = 7,43‬‬
‫اجلسم النقطة 𝐶‪.‬‬ ‫)𝑥(𝑓 = 𝑦 ملسار اجلسم )𝑆(‪ .‬نأأخذ مبد أأ ا أل م ة )‪ (𝑡 = 0‬حلظة مغادر‬
‫[‪ ]3‬املعادةل ادلياكرتية‬
‫س املرجع السطح ا ألريض املعترب عطايل بتطبي القانون الثاين لنيوتن عىل امجلةل "اجلسم )𝑆(" خالل حركته ‪:‬‬
‫⃗⃗⃗⃗⃗⃗⃗‬
‫𝐹∑‬ ‫⃗⃗⃗⃗ ‪𝑒𝑥𝑡 = 𝑚.‬‬
‫𝑎 ‪⃗⃗⃗ = 𝑚.‬‬
‫𝑃 ⇒ 𝐺𝑎‬ ‫⇒ ⃗⃗⃗‬ ‫𝑔 ‪𝑚.‬‬
‫⃗⃗⃗ ‪⃗⃗⃗ = 𝑚.‬‬
‫𝑎‬ ‫⇒‬ ‫𝑔 = ⃗⃗⃗‬
‫𝑎‬ ‫⃗⃗⃗‬
‫)𝑦𝐶( جند‪:‬‬ ‫حموري ادلراسة )𝑥𝐶(‬ ‫سقاط وف‬
‫‪𝐶𝑥 ∶ 𝑎𝑥 = 0‬‬
‫𝑔‪{𝐶𝑦 ∶ 𝑎 = −‬‬
‫𝑦‬
‫مركبيت الرسعة اللحظية )𝑡( 𝑥𝑣 و )𝑡( 𝑦𝑣‪.‬‬
‫جراء التاكم ‪𝐶𝑥 ∶ 𝑎𝑥 = 0‬‬ ‫𝛼 ‪𝐶𝑥 ∶ 𝑣𝑥 (𝑡) = 𝑣𝐶 cos‬‬
‫→ 𝑔‪{𝐶𝑦 ∶ 𝑎 = −‬‬ ‫{‬
‫𝑦‬ ‫𝛼 ‪𝐶𝑦 ∶ 𝑣𝑦 (𝑡) = −𝑔. 𝑡 + 𝑣𝐶 sin‬‬
‫عباريت مركبيت شعاع املوضع ‪:‬‬
‫𝑡 ‪𝐶𝑥 ∶ 𝑥(𝑡) = 𝑣𝑐 cos 𝛼 .‬‬
‫𝛼 ‪𝐶𝑥 ∶ 𝑣𝑥 (𝑡) = 𝑣𝑐 cos‬‬ ‫جراء التاكم‬
‫{‬ ‫→‬ ‫{‬ ‫‪1‬‬
‫𝛼 ‪𝐶𝑦 ∶ 𝑣𝑦 (𝑡) = −𝑔. 𝑡 + 𝑣𝑐 sin‬‬ ‫𝑡 ‪𝐶𝑦 ∶ 𝑦(𝑡) = − 𝑔. 𝑡 2 + 𝑣𝑐 sin 𝛼 .‬‬
‫‪2‬‬
‫معادةل املسار )𝑥(𝑓 = 𝑦‬
‫)𝑡(𝑥‬ ‫𝑔‬
‫=𝑡‬ ‫⇒‬ ‫‪𝑦=−‬‬ ‫𝛼 ‪2 cos 2‬‬
‫𝑥 ‪𝑥 2 + tan 𝛼 .‬‬
‫𝛼 𝑠𝑜𝑐 𝑐𝑣‬ ‫‪2𝑣c‬‬
‫[‪ ]4‬املسافة 𝑀𝐶 ‪ :‬عند النقطة 𝑀 يةون ‪ 𝑦𝑀 = 0‬و م ه ‪:‬‬
‫𝑔‬ ‫‪2‬‬
‫‪𝑦𝑀 = −‬‬ ‫𝑥‬
‫𝑀 𝛼 ‪2 cos 2‬‬
‫⇒ 𝑀𝑥 ‪+ tan 𝛼 .‬‬ ‫𝑀𝑥 ‪0 = −0,133. 𝑥𝑀 2 + 0,577.‬‬
‫‪2𝑣c‬‬
‫𝑚 ‪𝑥𝑀 = 4,33‬‬
‫المترين التجرييب‬
‫اجلزء ا ألول‬
‫[‪]1‬‬
‫‪[𝐴𝐻 + ] [𝑂𝐻 − ]2‬‬ ‫‪[𝑂𝐻 − ]2‬‬
‫] ‪𝐾𝑏 = [𝑂𝐻 −‬‬ ‫=‬ ‫⇒‬ ‫( 𝑔𝑜𝐿‪𝑝𝐾𝑏 = −‬‬ ‫)‬
‫]𝐴[‬ ‫] ‪[𝑁𝐻3‬‬ ‫] ‪[𝑁𝐻3‬‬
‫⇒‬ ‫)] ‪𝑝𝐾𝑏 = −2. 𝐿𝑜𝑔[𝑂𝐻 − ] + 𝐿𝑜𝑔([𝑁𝐻3‬‬
‫𝑒𝑘‬
‫⇒‬ ‫𝑔𝑜𝐿 ‪𝑝𝐾𝑏 = −2.‬‬ ‫] ‪+ 𝐿𝑜𝑔[𝑁𝐻3‬‬
‫] ‪[𝐻3 𝑂 +‬‬
‫] ‪⇒ 𝑝𝐾𝑏 = −2. (𝐿𝑜𝑔𝑘𝑒 − 𝐿𝑜𝑔[𝐻3 𝑂+ ]) + 𝐿𝑜𝑔[𝑁𝐻3‬‬
‫] ‪⇒ 𝑝𝐾𝑏 = 2. 𝑝𝑘𝑒 − 2. 𝑝𝐻 + 𝐿𝑜𝑔[𝑁𝐻3‬‬
‫‪1‬‬
‫)] ‪⇒ 𝑝𝐻 = (2𝑝𝐾𝑒 − 𝑝𝐾𝑏 + 𝐿𝑜𝑔[𝑁𝐻3‬‬
‫‪2‬‬
‫جمز أأ اجملتوع‬
‫[‪ ]2‬املعادةل البيانية‬
‫‪1‬‬
‫= 𝐻𝑝‬ ‫‪𝐿𝑜𝑔[𝑁𝐻3 ] + 11,60‬‬
‫‪2‬‬
‫[‪ ]3‬انمية 𝑏𝐾𝑝‪ .‬و انمية 𝑎𝐾𝑝‪.‬‬
‫انمية 𝑎𝐾𝑝 ‪:‬‬ ‫‪‬‬ ‫‪ ‬انمية 𝑏𝐾𝑝 ‪ :‬ملطابقة بني العالانة البيانية و العالانة النظرية جند‬
‫𝑒𝐾𝑝 = 𝑏𝐾𝑝 ‪𝑝𝐾𝑎 +‬‬ ‫‪1‬‬
‫‪𝑝𝐾𝑒 −‬‬
‫‪𝑝𝐾 = 11,60‬‬
‫‪𝑝𝐾𝑎 = 𝑝𝐾𝑒 − 𝑝𝐾𝑏 = 14 − 4,8‬‬ ‫𝑏 ‪2‬‬
‫‪𝑝𝐾𝑎 = 9,2‬‬ ‫)‪𝑝𝐾𝑏 = 2(𝑝𝐾𝑒 − 11,60) = 2(14 − 11,60‬‬
‫‪𝑝𝐾𝑏 = 4,8‬‬
‫‪−(II‬‬
‫[‪ ]4‬معادةل تفاع املعاير‬
‫→ )𝑞𝑎( ‪𝑁𝐻3 (𝑎𝑞) + 𝐻3 𝑂+‬‬ ‫)𝑙(𝑂 ‪𝑁𝐻4 + (𝑎𝑞) + 𝐻2‬‬
‫[‪ ]5‬نقطة التاكفإ يه النقطة اليت يتساوى عند ا مكية ماد احمللول املعايَر مع مكية ماد احمللول املعاير‪.‬‬
‫احداثيات نقطة التاكفإ ‪(𝑉𝑏𝐸 = 25 𝑚𝐿 ; 𝑝𝐻𝐸 = 5,4) :‬‬
‫الرتكزي املويل 𝑎𝑐 ‪:‬‬
‫‪𝑐𝑎 𝑉𝑎𝐸 10−2 × 25‬‬
‫𝑏𝑉 𝑏𝑐 = 𝐸𝑎𝑉 𝑎𝑐‬ ‫⇒‬ ‫= 𝑏𝑐‬ ‫=‬ ‫‪= 2,5. 10−2 𝑚𝑜𝑙. 𝐿−1‬‬
‫𝑏𝑉‬ ‫‪10‬‬
‫[‪ ]6‬الاكش املناسب ل ذب املعاير و أأ ر اللكورو فينول أل ّن ‪:‬‬
‫]‪𝑝𝐻𝐸 = 5,4 ∈ [4,8 − 6,4‬‬
‫‪𝑁𝐻 +‬‬
‫[‪ ]7‬انمية‬
‫𝑎𝑘𝑝 للثنااية ) ‪( 𝑁𝐻4‬‬
‫‪3‬‬
‫‪𝑝𝐾𝑎 = 9,2‬‬ ‫من البيان عند نقطة نص التاكفإ جند ‪:‬‬
‫⇒ 𝑒𝐾𝑝 = 𝑏𝐾𝑝 ‪𝑝𝐾𝑎 +‬‬ ‫‪𝑝𝐾𝑏 = 𝑝𝐾𝑒 − 𝑝𝐾𝑎 = 14 − 9,2‬‬ ‫ومه‪:‬‬
‫‪𝑝𝐾𝑏 = 4,8‬‬
‫النتاجئ احملص علهيا املتوافقة‪.‬‬
‫اجلزء الثاين‬
‫كربوكسمييل و حول لينتج ا ألسرت و املاء‪.‬‬ ‫[‪ ]1‬تفاع ا ألسرت و تفاع كمييا حمدود بط ء و عةوس ي بني‬
‫[‪]2‬‬
‫مردود تفاع ا ألسرت‬ ‫‪‬‬ ‫‪ ‬مكية ماد ا ألسرت الناجت‬
‫𝑓𝑥‬ ‫‪0,067‬‬ ‫‪𝑚 6,97‬‬
‫=𝑟‬ ‫= ‪× 100‬‬ ‫‪× 100‬‬ ‫= 𝑟𝑒𝑡𝑠𝑒𝑛‬ ‫=‬ ‫𝑙𝑜𝑚 ‪= 0,067‬‬
‫𝑥𝑎𝑚𝑥‬ ‫‪0,1‬‬ ‫‪𝑀 104‬‬
‫‪𝑟 = 67%‬‬
‫[‪ ]3‬صن الةحول 𝐹 املسمتعت ‪ :‬حول أأويل أل ّن ‪.𝑟 = 67%‬‬
‫[‪ ]4‬الصيغة نص املفصةل للةحول 𝐹 ‪:‬‬
‫𝐻𝑂 ‪𝐶𝐻3 𝐶𝐻2 𝐶𝐻2‬‬
‫برو نول‬
‫[‪ ]5‬معادةل تفاع ا ألسرت احلادث ‪:‬‬
‫)𝑙(𝑂 ‪𝐶𝐻3 𝐶𝑂𝑂𝐻(𝑎𝑞) + 𝐶𝐻3 𝐶𝐻2 𝐶𝐻2 𝑂𝐻(𝑎𝑞) = 𝐶𝐻3 𝐶𝑂𝑂𝐶𝐻2 𝐶𝐻2 𝐶𝐻3 (𝑎𝑞) + 𝐻2‬‬
‫‪−(II‬‬
‫[‪ ]1‬اجلدول‪.‬‬
‫‪𝑛𝑒𝑠𝑡𝑒𝑟 = 𝑛0 − 𝑛𝐹′ = 0,5 − 𝑛𝐹′‬‬

‫)‪𝑡(ℎ‬‬ ‫‪0‬‬ ‫‪1‬‬ ‫‪2‬‬ ‫‪3‬‬ ‫‪4‬‬ ‫‪5‬‬ ‫‪6‬‬ ‫‪7‬‬ ‫‪8‬‬ ‫‪9‬‬ ‫‪10‬‬
‫)𝑙𝑜𝑚( ‪𝑛𝐹′‬‬ ‫‪0,50‬‬ ‫‪0,38‬‬ ‫‪0,31‬‬ ‫‪0,27‬‬ ‫‪0,24‬‬ ‫‪0,22‬‬ ‫‪0,21‬‬ ‫‪0,20‬‬ ‫‪0,20‬‬ ‫‪0,20‬‬ ‫‪0,20‬‬
‫𝑙𝑜𝑚( 𝑟𝑒𝑡𝑠𝑒𝑛‬ ‫‪0,00‬‬ ‫‪0,12‬‬ ‫‪0,19‬‬ ‫‪0,23‬‬ ‫‪0,26‬‬ ‫‪0,28‬‬ ‫‪0,29‬‬ ‫‪0,30‬‬ ‫‪0,30‬‬ ‫‪0,30‬‬ ‫‪0,30‬‬
‫جمز أأ اجملتوع‬
‫)𝑡(𝑓 = 𝑟𝑒𝑡𝑠𝑒𝑛‬ ‫[‪ ]2‬املنحىن‬
‫)𝒍𝒐𝒎( 𝒓𝒆𝒕𝒔𝒆𝒏‬
‫‪0,35‬‬

‫‪0,3‬‬

‫‪0,25‬‬

‫‪0,2‬‬

‫‪0,15‬‬

‫‪0,1‬‬

‫‪0,05‬‬
‫)𝒉(𝒕‬
‫‪0‬‬
‫‪0‬‬ ‫‪1‬‬ ‫‪2‬‬ ‫‪3‬‬ ‫‪4‬‬ ‫‪5‬‬ ‫‪6‬‬ ‫‪7‬‬ ‫‪8‬‬ ‫‪9‬‬ ‫‪10‬‬

‫خاصيتني من خصاا تفاع ا ألسرت ‪:‬‬


‫‪ ‬تفاع بط ء ألنه يسمتغرق عد ساعات‬
‫‪ ‬تفاع حمدود أل ّن ‪:‬‬
‫𝑥𝑎𝑚𝑥 < 𝑓𝑥‬
‫[‪ ]3‬مردود تفاع ا ألسرت ‪:‬‬
‫𝑓𝑥‬ ‫‪0,30‬‬
‫=𝑟‬ ‫= ‪× 100‬‬ ‫‪× 100 = 60%‬‬
‫𝑥𝑎𝑚𝑥‬ ‫‪0,50‬‬
‫حول اثنوي أل ّن ‪𝑟 = 60%‬‬ ‫[‪ ]4‬صن الةحول ‪ 𝐹′‬املسمتعت ‪: :‬‬
‫[‪ ]5‬اكتب الصيغة نص املفصةل ل ألسرت‪.‬‬
‫𝑂‬
‫𝐶 ‪𝐶𝐻3 −‬‬
‫‪𝑂 − 𝐶𝐻 − 𝐶𝐻3‬‬
‫|‬
‫‪𝐶𝐻3‬‬
0202

04 40

10
04
Volley  ball 

B
M0 O (Oxy)

y v0
M0

h0 h f  2,40m
hf h0  3,50 m
OB  2  OA  18m
O A M x
B
g  9,8 m.s 2
I
v0  22 m.s 1 M0
M
 Oxy  v a
h0 x v0

OM

14 1
0400

II
v0  v1

g 2
y x  tan   x  h0
2v02
v1 
04

14
83,5% 6 C

At 
ln 14
C
6 1.2
A0 14
7 N

0
1 t 104 ans 
 0,9677 Z  N 
At 
ln
A0

A  t   A0 e  .t
0‫الشكل‬ λ
 t1/2
14 t1/2
t1 2 A0
t ln
ln 2 A

14 0
0400
06

RL RC
RC 
R

E C

K
E
R  2 K
3
duC
dt
 104 V.s -1 
C
K
t 0 K
duC
uC
dt
uC
C  10 nF E
t 
r  60%
5
EG
0 1
4 uC  V  RL II

R2  20K  R1
r  20  L (b)
K
D
5 .t  0 K
t i
i t 
L R1
ub
t 0 K

14 0
0400
i  mA
i t 
di  t 
ub
dt

K R2
10
40
0 2,5
6 t  ms  06

.I
pH CH3  COOH  aq 
25℃

S1  S2  S3 


c  mmol.L1  10 1 0,1
pH 3,4 3,9 4,4
log  c 
S1 S2 V  200 mL

c  H 3O 

c pH CH 3  COOH / CH 3  COO  Ka


1
pH   pKa logc 
2
pH  f  logc 
pK a

14 0
0400
.II
0,2mol 0,2mol
C3 H 7OH

nacide  mmol 

40 K
0 10 t  min 
7

0,2mol n  mol 
r  80%  n  0,2 mol  n  mol 

14 5
0400

G  6,67  1011 SI

 h m
MT RT

G RT , M T , m , h
h 
1
B A h A B h
2

 1 
h f  2
 

14 0
0400
RT MT
h 106 m 

h   3,11km.s 1
T
5

0
2 
107 s 2 / m 2 
0 ,1 1
0 0,1
2 
 NH 
4  NO3 

V1  100 mL c1  0,1mol.L1

Cu 2
/ Cu 

V2  100 mL c2  0,1mol.L1

 Zn 2
/ Zn 

 COM 
U  1,10 V
F  9,65 104 C.mol 1 -
.I

COM V
R
Zn s  A Cu s 

 NH +
4  NO3 

R  50  .II
Zn2aq  Cu2aq 

t  5 h

14 7
0400

235
92 U

U + 01 n 
235
92
148
57 La + 85Z Br + x 01 n

A 235 148 85 1
Z X 92 U 57 La Z Br 0 n
m u  235,044 147,932 84,916 1,009

103
1u  kg  931,5MeV / c 2 c  3 108 m.s 1 1MeV  1,6 1013 J
NA
1année  365 jours N A  6,02 1023 mol 1

.I

z x
. m 3
Elib m  0,2 u

m  1g
r  30% P  1000 MW
r  30% P  1000 MW

.II
85
Z Br
t1 2  85 Kr   10,76 années t1 2  85 Br   2,90min 85
36 Kr
85
Z Br

14 8
0400
m  1g 85
Z Br
1% 85 4

NaHCO3

P  %

R  8,31 SI
M  NaHCO3   84 g.mol 1 25 0C
CH 3COOH V = 120 mL V0  1L
12g c  1,25 mol.L1

CH 3COOH  aq  + HCO3-  aq  = CH 3COO -  aq  + H 2O  l  + CO2  g 

PCO2  t 
PCO2  kPa 
P  %

CO2
x
PCO2  t  T VCO2 R
xmax
P  t1/2  1
t1/2 
Pmax 2
55 NaHCO3
t s
0
50
NaHCO3

14 9
0400

1L 10 g
20mL s s
ca  0,2 mol.L1  H3O  Cl    aq 

HCO3-  aq  + H 3O+  aq  = CO2  g  + 2H 2O  l 


pH
pH

acide / base

VaE
HCO3-
HCO3- pka

0
Va  mL 
2

P  %

14 14
04
I
a
 Px  0  m.ax  ax  0 
2  0, 25
 Fext  P  m.g  m.aG
1
P  
 Py  m g  m a y  a y   g 
. .

v
 vx  t   v0  dv  v0 x  v0 
v :a
2  0, 25  v y  t    g .t  v0  
  dt  v0 y  0 

 x  v0 .t 
2  0, 25 OM   :v
dOM
2
 y   1 g .t 2  h0  dt
 2 
0, 75
g 2
0, 25 y  x   x  h0 y (t) x(t) t
2v02
x  9m h 3
g 2 9,8
0, 25 0, 25 h  x  h0    92  3,5  2,78 m  2,40m
2
2v 0 2  22 2

x p  OM
 
y xp = 0 4
g 2.h0 2  2,40
0, 25 0, 25 0 x 2  h0  x p  v0 .
2 p
 22   15,4 m  18m
2v0 g 9,8

E f = Ei +  W Fext  vM 5
1 1
EcM = Ec0 +  W P   2
m.vM2 = m.v02 + m.g.h0
2
vM  v02  2.g.h 0  222  2  9,8  3,50  23,50 m.s 1

2  0, 25
0,50

15 1
II
 v1x  v0  v1 .cos      ax  0 
v1 
 v1 y  v1 .sin     a  
   ay  g 
 vx  t   v1 .cos     v0  dv
v :a
 v y  t    g .t  v1 .sin    
0, 25
  dt
0, 25  x  v1 .cos    .t 
0, 75
OM   :v
dOM
 y   1 g .t 2  v .sin    .t  h  dt
1 0
 2 
2
0, 25 1  x  x
y   g .   v1 .sin    .  h0
2  v1 .cos     v1 .cos   
g . 2
y x  tan    .x  h0
2.v02
h0  h f g .
tan   
1 g . 2
 xA hf   x A  tan    .x A  h0 
xA 2
2.v0 2 v02
3,5  2, 4 9,8
0, 25 tan      9  0, 031    tan 1 (0, 031)  1,8
9 2  22 2
0,50
0, 25 v0 22
v1    22, 01 m.s 1
cos   cos 1,8

15 2
44

0, 25 A (t )
(Bq )
0, 25 A Z
0, 25 A  0 Z  1 14
6 C  147 N  ZA r
6 C  7 N  1 e
14 14 0
N  A Z
1, 25    10 e
0, 25
Z  N  3.3
8 14
6C

14
. 7N

.
0, 25
.
0. ....... Z
7
.

15 3
2
0, 25 N (t )  N 0  e  t
A(t )
0, 25 dN  t  d ( N 0 e  t )
A(t )    A(t )     N0e t  A(t )  A0e  t
dt dt
λ
0,9677  0 A(t )
 a.t
1
a  1, 2096 ans ln
0,8  10  0
0, 25 4
A0
A(t )
ln  1,2096.t  1
A0
0, 25
A (t )
A (t )  A 0  e  t   e  t
2, 00 A0
0, 25

 ln e  t   ln
A (t ) A (t ) A(t )
ln  t   2 
A0 A0 A0
0, 25
  1,2096 104 ans 1
t 1/2 λ

N0 N 1
t  t1/2  N (t1/2 )   N (t1/2 )  N0e t1/2  0  N0e t1/2   e t1/2 ....(1)
2 2 2
1 ln 2
0, 25 t1/2  ln  t1/2  ln 2  t1/2 
2 
ln 2 ln 2
t 1/2    5730 an t 1/2
0, 25  1,2096  104

A(t )  A(t ) 
A(t )  A0e  t   e  t  ln    t
A0  A0 
 A0  1  A0  t1/2  A0 
0, 25 t  ln    t  ln  t  ln  
 A(t )    A(t )  ln 2  A(t ) 
5730  100 
t  t  1490, 67ans
ln 2  83,5 
ln
0, 75 0, 25
1968 1490,67 477ans
400  477  500
0, 25

15 4
46
RC I
i
uR uC  t 
2  0, 25 0 55
dq du
E uC uR  R.i  R.  RC c u R uC  uR  E
0,50 dt dt
duc
K R.C.  uc  E
dt

0, 25
duc 1 . E ........................
 uc  1
dt R.C R.C
duc
0, 25  a.uc  b.............. 2 
dt
du
 c
0, 25
a  dt  5 104 s 1 a
uc
1 1 1
0, 25 a  R.C      2 105 s 1
1,50 RC a 5 104
E
b  30 104 V.s 1
R.C

0, 25 E  3 105  2 105  6V
C

2 105
R.C  2 105 s C  C  108 F  10 nF
2 103
0, 25

t 
0, 25 1 1
Ec  cuC 2  Ec    cuC 2  
2 2
0,50 0, 25 uc    0,63E  3,78V
1
Ec    108  3,782  7,14 108 J
2

0, 25 E C max E C .E 2
r 100  E G  C max 100  100
EG r 2r
0,50 0, 25 108  62
EG  100  3 107 J
2  60

15 5
RL II

i t 

ub E  ub  uR1
0, 25
di
uR1  R1 .i ; ub  L  r.i
dt
0, 25 u R1
di  R1  r  E di
  .i  EL   R1  r  .i
dt  L  L dt
di
i  I0 0 R1
dt
E  R r E
R1   r 0 1  .I 0  L
0, 25 I0  L 
0, 25 6
R1   20  100  I 0  50 mA
0, 05
1,50 L
  2,5 103 s L
R1  r

0, 25 L   R1  r  .  0,3H
di
0
dt

0, 25 Ub  r.I 0  20  50 103  1V

i  0  I0
0, 25 0, 25

di
dt
di
ub  uR1  uR2  0  L   r  R1  R2  .i  0
dt
di  t 
0, 25
R1  R2  r
 .i  t 
dt L
0, 25
di  0  R1  R2  r
.I 0   100  2 10  20  50 103  353,3A.s 1
3

1, 0 dt L 0,3
0, 25 di
ub  0   L.  0   r.I 0  105 V
0, 25 dt

0, 25 0, 25

15 6
06

0, 75 3  0, 25 0
20
20 20 mL

0, 25 CH3COOH  aq   H 2O  l   CH3COO  aq   H3O  aq 

CH3COOH  H 2O  CH3COO  H3O


0,50

0, 25 x0 CV 0 0
x CV  x x x
x  xf CV  x f xf xf

 H 3O   CH 3COO  
 f  f
Ka  c pH Ka
CH3COOH  f
0,50 2  0, 25 CH3COOH  f  c   H3O   c
f
C2 H5COO    H 3O  
 f  f
2
 H 3O  
 f 102 pH
Ka   Ka  c  H 3O  
c c  
1
pH   pKa logc 
2
0,50 102 pH 102 pH
2  0, 25 Ka    log Ka   log  pKa   log102 pH  ( log c)
c c
1
pK a  2 pH  logc  pH   pK a  logc 
2

15 7
0,50

( )
pH
1, 00

0,50

1
log  c 
0 1

pH  a  logc   b
4, 4  2, 4
pH  0,5  logc   2, 4 b  2, 4 a  0,5
40
0, 25 0, 25
1
pH   pKa logc   pH  0,5.  logc   0,5 pKa
2
2, 4
0,5 pK a  2, 4  pK a   4,8
0,5

mmol
0, 25 0, 25
x0 n2 4 0 0
x n2  x 4 x x x
x  xf n2  x f 4  xf xf xf

15 8
xf
0, 25 n f  Acid   0, 2  x f  x f  0, 2  0,08  0,12 mol xmax  0, 2 mol f 
xmax
0,12
f   0, 6
0, 2
0, 25
r   f 100  r  0,6 100  60%

1,50
5.5
OH
0, 25
CH3 CH CH3
2  0, 25

CH3 COO CH CH3


0, 25
CH3

K
CH3COOC3 H5  f  H 2O f x 2f
0, 25 0, 25 K   2, 25
C3 H5OH  f CH3COOH  f  0, 2  x f 
2

r  80% ( )
xf
r 100  80  x f  0,8.xmax  x f  0,16 mol
xmax
0, 25
CH3COOC3 H5  f  H 2O f x 2f
K   2, 25
0,50 C3 H5OH  f CH3COOH  f  n  x f  . 0, 2  x f 
0,162 0,162
 2, 25  n   0,16  n  0, 44 mol
0, 25
 n  0,16  .  0, 2  0,16  2, 25   0, 2  0,16 

15 9
04

d1 S1 S2 d2

0, 25 0, 25

d1 d 2 v 1t v 2 t v 1 v 2 S1  S 2

0, 25 3
0, 25 S 1  S 2  d1  d 2 v 1  v 2
0,50

vs 4
s

FT S
T
o

0, 25 M T .m
x FT / S  G ... 1
 RT h
2

1, 75 5.4

0, 25
F ext  m.a  FT / S  m.a
Fn  m.an ...  2 
MT
an  G ...  3  2 1
 RT  h
2

0, 25
2
an  ...  4 
 RT h
0, 25
MT
 G ...  5  4  3
 RT  h 
1
h A B h 3.4
0, 25 2
MT 1
 2 G  h  G .M T . 2  RT ...  6   5
 RT  h  
2  0, 25
B   RT A  G.M T

15 14
RT MT
1
 h  . b
0, 25 2
  40,02 1013 m3.s 2 
 h  0  0 
  7
 1  0,16  107   0  40,02  10  0,16  10  b  b  6,4  10 m
13 6
b
0, 25  2 
 h 0 
1
1,50
h  40 , 02 1013. 2  6 , 4 106...  7 

G .M T  40 , 02  10  M T  6  1024 kg
13

2  0, 25  7  6
  RT  6 , 4  10  RT  6 , 4  10 m
6 6

T h 5.5
  3,11 103 m / s  h  34000km ,
2. .  RT  h 
0, 25
T   8, 2  104 s  24 h
0, 25

04
I
0, 25 0, 25

0, 25 0, 25
E  1,10 V
0, 25 0, 25

2  0, 25
Zn
0,50
Cu

0, 25 0, 25 
 Cu Cu 2 Zn2 Zn  
II
0, 25 0, 25 E 1,1
E  R.I  I    0, 022A  22 mA
R 50

Zn  Zn 2  2é Zn
0, 75 3  0, 25 Cu 2  2é  Cu Cu
Zn  s   Cu 2 aq  Cu  s   Zn 2 aq 

15 11
t
Zn  Cu 2  Cu  Zn2
2  0, 25
0,50 t 0 x0  cV
.  cV
.
t x  . x
cV  . x
cV
I .t 0, 022  5  3600
x   0, 00205 mol  2, 05 mmol
2  0, 25 y.F 2  9, 65 104

. x
cV x 0, 00205
1, 00 Cu 2    c   0,1   0, 0795 mol.L1
V V 0,1
2  0, 25
. x
cV x 0, 00205
 Zn2    c   0,1   0,1205 mol.L1
V V 0,1

00
-I
0, 25
0, 25

z x
0, 25 235  1  148  85  x  x  3
0, 25 92  0  57  z  0  z  35
0, 75
235
92 U  01n  148
57 La  35
85
Br  3. 01n
0, 25

m   mR   mP  m U   m  n    m  La   m  Br   3m  n  
0, 25
m  m U    m  La   m  Br   2m  n  
0,50
m  235,044  147,932  84,916  2 1,009  0,178 u
0, 25
E1

0, 25 0, 25 E1  m.c2  m  u   931,5  186,3 MeV


m  1g

m  235U  
0, 25 70 .
m  0, 7 g
100
m  235U 
0, 75 0, 25 N .N A  0, 7  6, 02 1023  1,8 1021 noyaux
M U
235
235

15 12
0, 25 E  N.E1  1,8 1021 186,3  3,34 1023 MeV=5,344 1010 J

Eel  P. t  109 1 365  3600  1,314 1015 J


1, 25 3  0, 25 Eel E
r  %  100  E U   el 100  4,38 1015 J
E U  r  %
2  0, 25
1 E U 
m 1année    8, 2 104 g  82 kg
E
-II

2  0, 25
0, 75
0, 25
85
35 Br  3685 Kr  Az r  A  0 ; Z  1 r  10e
 
85
35 Br  3685 Kr  10 e

dN  t 
2  0, 25 N  t   N0 .e  .t ; A t   
0,50 dt
dN  t 
A t       .N0 .e  .t   A0 .e  .t
dt

m  1g 85
Z Br

N 0  N  235U   1,8 1021 noyaux


2  0, 25
0,50
ln  2  ln  2 
A0  .N 0  N0   1,8 1021  7, 2 1018 Bq
t1 2 2,9  60

1%
2  0, 25 t .ln  2
0,50 
1 ln100 ln100
A0  A0 e  t  t1 2  10, 76   71,5 années
t1 2

100 ln2 ln2

15 13
m0 .
0, 25 0, 25 p  %  100 100 g
m

CH 3COOH  aq  + HCO3  aq  = CO2  g  + CH 3COO  aq  + H 2O  l 


- -

0 ,5 0 ,5
x0 n1 n2 0 0 0
x n1  x n2  x x x x
xf n1  x f n2  x f xf xf xf

PCO2  t  T VCO2 R x
0, 25 0, 25 PCO2 .V0 PV
.
n(CO2 )  x  x  n(CO2 ) 
R.T R.T
xmax
0, 25 PCO2 max .V0 341103 1103
xmax   xmax   xmax  0,138 mol
0 ,5 R.T 8,31 298
CH 3 COOH

0, 25 xmax  cV
.  xmax  1, 25  0,120  0,15mol  0,138 mol
-
HCO3 CH 3 COOH

P  t1/2  1

Pmax 2
P(t1/2 ).V0
P  t1/2  1
0, 25
x(t1/2 ) 1
1,00 0, 25   R .T  
xmax 2 Pmax .V0 Pmax 2
0, 25 R .T
Pmax 341
0, 25 P  t1/2    P  t1/2    170,5KPa  t1/2  0,8  50  40s t1 2
2 2
NaHCO3
0, 25 0, 25 m  NaHCO3   xmax .M  m  NaHCO3   0,138  84  11,6 g

15 14
NaHCO3
11, 6
0, 25 0, 25 p  %  100  p  %   96, 6%
12

0, 25 0, 25 HCO3

0,50 2  0, 25 H 3O / H 2O CO2 / HCO3 acide / base

0,50 2  0, 25

0, 25 VaE  5,8  2  11,6mL VaE



pH E = 3,8 < 7 HCO3
ca .VaE 0, 2 11, 6
cB   cB   0,116 mol.L1
VB 20
0,75 0, 25
1014
OH   cB

pH  9  OH   9  105 mol.L1

Va  0
10
HCO3- pka
VaE
0, 25 Va = = 5,8 mL  pKa = pH = 6,4
2
0, 25 n  cB .V  n  0,116 1  0,116mol
0, 25 m  n.M  0,116  84  9,744 g
0,75 NaHCO3
9, 744
0, 25 p  %  100  p  %   97, 44%
10
0, 25 0, 25

15 15
‫الجمهورية الجزائرية الديمقراطية الشعبية‬
‫االختبار التجريبي الموحد دورة ماي‪2023 :‬‬ ‫مديرة التربية لوالية البليدة‬
‫الثانويات ‪ :‬كل ثانويات مقاطعة البوينان‬ ‫الشعبة‪ :‬تقني رياضي ‪ -‬رياضيات‬

‫المدة ‪ 34:‬ساعة و‪ 03‬د‬ ‫االختبار التجريبي في مادة‪ :‬العلوم الفيزيائية‬


‫على المترشح أن يختار أحد الموضوعين اآلتيين‬
‫الموضوع األول‬
‫يحتوي املوضوعألاولعلى ‪ 06‬صفحات (من الصفحة ‪ 1‬من ‪10‬الى الصفحة ‪ 6‬من ‪)10‬‬
‫الجزء األول (‪َّ 34‬نقاط)‬
‫التمرين ألاول‪ 06 ( :‬نقاط)‬
‫يعترب محض كلور املاء) ‪ (𝐻3 𝑂+ + 𝐶𝑙 −‬أو ما يُعرف جتاريا بروح امللح من أكثر األمحاض استخداما خاصة يف تنظيف‬
‫اجملاري و أنابيب الصرف الصحي‪.‬‬
‫يهدف هذا التمرين اىل دراسة بعض التفاعالت الكيميائية هلذا احلمض‪.‬‬
‫‪ - I‬يف ايرلينة ماير نضع عند اللحظة ‪ t = 0‬وعند درجة حرارة ℃‪ θ = 25‬قطعة من الزنك ‪ Zn‬كتلتها ‪ m 0‬مع‬
‫حجم قدره 𝐿𝑚‪ 𝑉 = 100‬من حملول حلمض كلور املاء ) ‪ (𝐻3 𝑂+ + 𝐶𝑙 −‬تركيزه املولي ‪𝐶 = 5 × 10−2 𝑚𝑜𝑙. 𝐿−1‬‬
‫ِ‬
‫يعطى‪𝑴(𝒁𝒏) = 𝟔𝟓 , 𝟒𝒈/𝒎𝒐𝒍 :‬‬
‫التحول احلادث بطيء وتام‪ ،‬ينمذج باملعادلة ‪:‬‬
‫) ‪2H 3O + (aq ) + Zn(s ) = H 2 (g) + Zn 2+ (aq ) + 2 H 2O (l‬‬
‫‪ .1‬حدّد الثنائيتني ) ‪ ( ox / red‬املشاركتني يف هذا التفاعل ‪.‬‬
‫‪ .2‬اجنز جدول تقدم التفاعل‪.‬‬
‫‪ .3‬قمنا بقياس ‪ pH‬املزيج يف نهاية التفاعل فتحصلنا على القيمة ‪. 1,69‬‬
‫‪ .1.3‬احسب تركيز شوارد ‪ 𝐻3 𝑂+‬يف احلالة النهائية و استنتج كمية مادتها يف هذه احلالة‪.‬‬
‫‪ .2.3‬حدد املتفاعل احملد‪ ،‬ثم استنتج قيمة التقدم االعظمي ‪. x max‬‬
‫‪ .3.3‬حدّد كتلة الزّنك ‪. 𝑚0‬‬

‫الصفحة ‪ 1‬من ‪10‬‬


‫‪ -II‬املتابعة الزمنية هلذا التحول مكنتنا من رسم املنحنى‪ (  H 3O +  = f (t ) :‬الشكل‪)-4-‬‬
‫𝑳‪[𝑯𝟑 𝑶+] × 𝟏𝟎−𝟐 𝒎𝒐𝒍/‬‬ ‫‪ .1‬اكمل املنحنى البياني مع التعليل‪.‬‬
‫‪ .2‬جد بيانيا زمن نصف التفاعل ‪ ، t1/2‬موضّحا‬
‫كيفية ذلك‪.‬‬
‫‪ .3‬احسب السرعة احلجمية االبتدائية الختفاء‬
‫الشكل ‪-4-‬‬
‫شوارد ‪ ، H 3O +‬و استنتج السرعة احلجمية‬
‫للتفاعل األعظمية‪.‬‬
‫‪ .4‬نكرر التجربة يف درجة حرارة ℃‪. θ = 31‬‬
‫‪-‬ارسم على نفس الشكل املنحنى‬
‫𝟏‬
‫) ‪ ،  H 3O +  = g (t‬مع تفسري تأثري العامل‬
‫احلركي املسؤول عن تغري سرعة التفاعل‬
‫𝟎‬ ‫𝟐‬ ‫)𝒏𝒊𝒎(𝒕‬ ‫جمهريا‪.‬‬
‫‪ -III‬معايرة حملول النشادر بواسطة حملول محض كلور املاء ‪:‬‬
‫) ‪ NH 3 (aq‬تركيزه املولي ‪ C B‬بواسطة حملول‬ ‫نقوم مبعايرة حجما ‪ V B = 20 mL‬من حملول مائي ) ‪ ( S b‬للنشادر‬
‫محض كلور املاء املتبقي من التفاعل السابق( اجلزء ‪ )II‬ذي الرتكيز ‪، C A‬بواسطة املعايرة 𝐻𝑝 ‪ -‬مرتية حتصلنا على‬
‫املنحنى املمثل يف الشكل‪ - 5-‬تغريات ‪ pH‬املزيج بداللة حجم احمللول احلمضي املضاف ‪.V A‬‬
‫‪ .1‬اكتب معادلة تفاعل املعايرة ‪.‬‬
‫‪ .2‬ارسم الرتكيب التجرييب املستعمل مع ارفاقه بالبيانات‪.‬‬
‫𝑯𝒑‬
‫ثم احسب قيمة ‪. C B‬‬ ‫‪ .3‬جد احداثيي نقطة التكافؤ ‪ّ ،E‬‬
‫الشكل ‪-5-‬‬ ‫‪ .4‬جد بيانيا قيمة ثابت احلموضة ‪ pKa‬للثنائية ) ) ‪ ، ( NH 4 + (aq ) / NH 3 (aq‬و‬
‫استنتج قيمة 𝑎𝐾‪.‬‬
‫‪ .5‬احسب ثابت التوازن 𝐾 لتفاعل المعايرة ‪ ،‬ماذا تستنتج؟‬
‫‪ .6‬حدد الحجم ‪ V A‬من المحلول الحمضي الواجب اضافته لكي تتحقق العالقة ‪:‬‬
‫] ‪ [𝑁𝐻4+] = 15 [𝑁𝐻3‬في المزيج التفاعلي ‪.‬‬

‫‪2‬‬

‫‪0‬‬ ‫‪5‬‬ ‫)𝒍𝒎( 𝒂𝑽‬

‫ُ‬

‫الصفحة ‪ 2‬من ‪10‬‬


‫التمرين الثاني‪ 04( :‬نقاط)‬
‫ُﺴﻬﯿﻞ ﺴﺎت ‪ 2‬ﻗﻤر اﺼطﻨﺎﻋﻲ ﻗطري �ظﻬر ﺴﺎﻛﻨﺎ ﻟﻤﻼﺤظ ﻋﻠﻰ ﺴطﺢ اﻷرض‪�ُ ،‬ﺴﺘﻌﻤﻞ ﻓﻲ اﻻﺘﺼﺎﻻت‬
‫اﻟﻼﺴﻠﻛ�ﺔ ﻟﻠﺒث اﻹذاﻋﻲ واﻟﺘﻠﻔزي ﺒﺘﻘﻨ�ﺔ ﻋﺎﻟ�ﺔ اﻟﺠودة‪ّ �ُ .‬‬
‫ﺴﺘﻐﻞ ﻓﻲ ﺘﻐط�ﺔ وﻨﻘﻞ ُﻤ�ﺎر�ﺎت وأﺤداث كﺄس اﻟﻌﺎﻟم ‪2022‬‬ ‫ّ‬
‫ﻋﺒر اﻟﻘﻨوات اﻟﻔﻀﺎﺌ�ﺔ اﻟﻌﺎﻟﻤ�ﺔ‪ ،‬أُرﺴﻞ إﻟﻰ ﻤدارﻩ ﻓﻲ ‪ 15‬ﻨوﻓﻤﺒر‪.2018‬‬
‫ﯿﻬدف ﻫذا اﻟﺘﻤر�ن إﻟﻰ دراﺴﺔ ﺤركﺔ اﻟﻘﻤر اﻻﺼطﻨﺎﻋﻲ ُﺴﻬﯿﻞ ﺴﺎت ‪2‬‬
‫اﻟﻤﻤﯿزة ﻟﻪ‪.‬‬
‫ُ‬ ‫وﺘﺤدﯿد �ﻌض اﻟﻤﻘﺎدﯿر اﻟﻔﯿز�ﺎﺌ�ﺔ‬
‫ﻤﻌط�ﺎت‪ :‬ﻨﺼﻒ ﻗطر اﻷرض ‪RT  6400km‬‬
‫دور اﻷرض ﺤول ﻤﺤورﻫﺎ ‪TT  24h‬‬
‫ﺴﻬﯿﻞ ﺴﺎت‪2‬‬
‫‪ .I‬دراﺴﺔ ﺤركﺔ اﻟﻘﻤر اﻻﺼطﻨﺎﻋﻲ ُﺴﻬﯿﻞ ﺴﺎت ‪.2‬‬
‫ﻨﻌﺘﺒر ) ‪ ( S‬اﻟﻘﻤر اﻻﺼطﻨﺎﻋﻲ ُﺴﻬﯿﻞ ﺴﺎت ‪ ،2‬كﺘﻠﺘﻪ ‪ mS  5300kg‬ﯿدور ﺤول اﻷرض ﻓﻲ ﻤﺴﺎر داﺌري‬
‫‪‬‬
‫ﺟﮭﺔ اﻟﺪوران‬ ‫ﻨﺼﻒ ﻗطرﻩ ‪ ، r‬ﻋﻠﻰ ارﺘﻔﺎع ‪ h‬ﻤن ﺴطﺢ اﻷرض‪ ،‬ﺨﺎﻀﻊ ﻟﻘوة ﺠذب اﻷرض ‪ F T /S‬ﻓﻘط‪.‬‬
‫ِ‬
‫‪ (S ) ‬‬
‫ﺤدد اﻟﻤرﺠﻊ اﻟﻤﻨﺎﺴب ﻟدراﺴﺔ ﺤركﺔ ﻫذا اﻟﻘﻤر‪.‬‬ ‫‪ّ .1‬‬
‫‪‬‬
‫‪n‬‬
‫وﻤﺜّﻞ ﻋﻠ�ﻪ ﺸﻌﺎع اﻟﺴرﻋﺔ اﻟﻤدار�ﺔ ‪ v‬وﺸﻌﺎع ﻗوة ﺠذب اﻷرض ‪. F T /S‬‬ ‫‪ .2‬اﻨﻘﻞ )اﻟﺸكﻞ‪ِ (1‬‬
‫‪r‬‬ ‫‪‬‬ ‫‪‬‬
‫‪ .3‬اﻛﺘب اﻟﻌ�ﺎرة اﻟﺸﻌﺎﻋ�ﺔ ﻟﻠﻘوة ‪ F T /S‬ﺒدﻻﻟﺔ‪ r ، m S ، M T ، G :‬و ‪. n‬‬
‫‪‬‬
‫‪h‬‬ ‫)ﺤﯿث ‪ n‬ﺸﻌﺎع وﺤدة ﻨﺎظﻤﻲ‪ M T ،‬كﺘﻠﺔ اﻷرض‪ G ،‬ﺜﺎﺒت اﻟﺠذب اﻟﻌﺎم(‪.‬‬
‫‪RT‬‬ ‫‪ .4‬ﺒﺘطﺒﯿق اﻟﻘﺎﻨون اﻟﺜﺎﻨﻲ ﻟﻨﯿوﺘن ﻋﻠﻰ ﻤركز ﻋطﺎﻟﺔ ) ‪: ( S‬‬
‫اﻟﺸكﻞ‪1‬‬
‫‪2‬‬
‫ﻤﻤﯿزات ﺸﻌﺎع ﺘﺴﺎرع ﻤركز ﻋطﺎﻟﺔ اﻟﻘﻤر ) ‪ّ ( S‬‬
‫ﺜم اﺴﺘﻨﺘﺞ طﺒ�ﻌﺔ ﺤركﺘﻪ‪.‬‬ ‫‪ .1.4‬أﻋط ّ‬
‫‪FT‬‬ ‫‪/S‬‬ ‫)‪(10 N‬‬ ‫‪ .2.4‬اﻛﺘب ﻋ�ﺎرة ‪ v‬ﺒدﻻﻟﺔ ‪ M T ، G‬و ‪. r‬‬
‫‪ .3.4‬اﺴﺘﻨﺘﺞ ﻋ�ﺎرة اﻟدور ‪ TS‬ﻟﺤركﺔ ) ‪ ( S‬ﺒدﻻﻟﺔ اﻟﻤﻘﺎدﯿر‬
‫اﻟﻤذكورة ﻓﻲ اﻟﺴؤال )‪.(2.4‬‬
‫اﻟﻤﻤﯿزة ﻟﻠﻘﻤر ُﺴﻬﯿﻞ ﺴﺎت ‪.2‬‬
‫ّ‬ ‫‪ .II‬ﺘﺤدﯿد �ﻌض اﻟﻤﻘﺎدﯿر‬
‫ﺘﻤت ﻤﺤﺎﻛﺎة ﺤركﺘﻪ‬
‫ﻤﻤﯿزات اﻟﻘﻤر ) ‪ّ ( S‬‬
‫ﻟﻐرض ﺘﺤدﯿد ّ‬
‫ﺒواﺴطﺔ ﺒرﻤﺠ�ﺔ ﻤﻨﺎﺴ�ﺔ‪).‬اﻟﺸكﻞ‪� (2‬ﻤﺜّﻞ ﺒ�ﺎن ﺘﻐﯿرات ﺸدة‬
‫‪‬‬
‫ﻗوة ﺠذب اﻷرض ﻟﻠﻘﻤر اﻻﺼطﻨﺎﻋﻲ ‪ ، F T /S‬ﺒدﻻﻟﺔ ﻤﻘﻠوب‬
‫‪1‬‬
‫ﻤر�ﻊ ﻨﺼﻒ ﻗطر ﻤدارﻩ ‪.  2 ‬‬
‫‪1,96‬‬ ‫‪ r ‬‬

‫‪0‬‬ ‫‪� .1‬ﺎﺴﺘﻐﻼل اﻟﺒ�ﺎن اﻟﻤﻤﺜﱠﻞ ﻓﻲ)اﻟﺸكﻞ‪ (2‬اﻛﺘب ﻤﻌﺎدﻟﺘﻪ‬


‫اﻟﺸكﻞ‪2‬‬
‫‪0,93‬‬ ‫‪1‬‬ ‫‪16‬‬ ‫‪2‬‬
‫‪2 (10‬‬
‫اﻟر�ﺎﻀ�ﺔ ﺜم اﺴﺘﻨﺘﺞ ﻗ�ﻤﺔ اﻟﺜﺎﺒت ‪ K‬ﺤﯿث ) ‪. ( K = GM T‬‬
‫‪.m‬‬ ‫)‬
‫‪r‬‬

‫=‪ ، FT‬اﺴﺘﻨﺘﺞ ﻗ�ﻤﺔ اﻟﻤﻘﺎدﯿر اﻵﺘ�ﺔ‪:‬‬ ‫ﺸدة ﻗوة ﺠذب اﻷرض ﻟﻠﻘﻤر ) ‪ ( S‬ﻫﻲ ‪11,8 × 10 N‬‬‫أن ﻗ�ﻤﺔ ّ‬‫‪ .2‬إذا ﻋﻠﻤت ّ‬
‫‪2‬‬
‫‪S‬‬

‫‪ .1.2‬اﻻرﺘﻔﺎع ‪ h‬ﻋن ﺴطﺢ اﻷرض‪.‬‬


‫‪ .2.2‬اﻟﺴرﻋﺔ اﻟﻤدار�ﺔ ‪. v‬‬
‫‪ .3.2‬اﻟدور ‪. TS‬‬
‫‪ .3‬ﻫﻞ اﻟﻘﻤر ُﺴﻬﯿﻞ ﺴﺎت ‪ 2‬ﺠﯿوﻤﺴﺘﻘر؟ ّﺒرر إﺠﺎﺒﺘك‪.‬‬

‫الصفحة ‪ 3‬من ‪10‬‬


‫التمرين الثالث‪ 4( :‬نقاط)‬
‫تستعمؿ الطائرات المركحية في ايصاؿ المساعدات االنسانية الى المناطؽ المنككبة كالمعزكلة ‪ ،‬نريد دراسة حركة صندكؽ‬
‫كتمتو ‪ m‬يحكم عمى أغطية ك أفرشة ‪.‬تتحرؾ طائرة مركحية كفؽ مسار مستقيـ أفقي بسرعة ثابتة ك عند الحظة ‪ t  0‬تمر‬
‫الطائرة مف النقطة ‪ A‬حيث تترؾ الصندكؽ الذم منزؿ كفؽ مسار منحني ليسقط عمى سطح األرض في النقطة ‪ O‬التي نعتبرىا‬
‫‪‬‬
‫مبدأ لممعمـ )‪ (o,i,k‬أنظر الشكؿ (‪.)1‬تعطى‪g 10m / S 2 :‬‬

‫) ‪z (m‬‬

‫𝑨‬

‫الشكل(‪)1‬‬
‫‪‬‬
‫‪k‬‬
‫) ‪x (m‬‬
‫‪‬‬
‫‪O‬‬ ‫‪i‬‬
‫‪‬‬
‫معالجة التصكير المتعاقب لحركة مركز عطالة الصندكؽ ببرمجية مناسبة عمى جياز االعالـ األلي في المعمـ )‪(o,i,k‬‬
‫)𝒔‪𝒗𝒛 (𝒎/‬‬ ‫مكننا مف رسـ المنحنييف (أ) ك (ب) المكضحيف بالشكؿ (‪)2‬‬
‫)𝑚(𝑥‬

‫‪0‬‬
‫‪0,4‬‬ ‫)𝑠(𝑡‬
‫‪-4‬‬

‫(ب)‬ ‫(أ)‬

‫‪10‬‬

‫الشكل(‪)2‬‬
‫‪0‬‬ ‫‪0,4‬‬ ‫)𝑠(𝑡‬

‫‪ .1‬باالعتماد عمى البيانيف (أ) ك (ب) جد ما يمي ‪:‬‬


‫أ‪.‬زمف كصكؿ الصندكؽ الى سطح األرض ‪،‬مع تقديـ تفسير النحناء مسار الحركة ‪.‬‬
‫ب‪ .‬قيمة المركبة األفقية ‪v x‬لسرعة الصندكؽ ك باستعماؿ سمـ مناسب ارسـ البياف ) ‪. vx (t‬‬
‫ج‪ .‬احداثيات نقطة انطالؽ الصندكؽ ) ‪. A (x A , z A‬‬
‫‪‬‬
‫‪ .2‬ىؿ يمكف اعتبار المعمـ المرتبط بسطح األرض )‪ (o,i,k‬معمما عطاليا ك ماىي الفرضية المعتبرة في ذلؾ ‪.‬‬
‫‪‬‬
‫‪ .3‬بتطبيؽ القانكف الثاني لنيكتف عمى الصندكؽ في المعمـ )‪: (o,i,k‬‬
‫أ‪ .‬جد المعادلتيف الزمنيتيف لمحركة ) ‪ x (t‬ك ) ‪ . z (t‬ك استنتج طبيعة الحركة عمى كؿ محكر ‪.‬‬
‫ب‪ .‬بيف أف معادلة مسار الحركة تكتب مف الشكؿ ‪z (x )  8.103.x 2  0,8x .:‬‬
‫ج‪.‬حدد قيمة سرعة ارتطاـ الصندكؽ بسطح األرض عند النقطة ‪ O‬ك زاكية ميؿ شعاعيا ‪. α‬‬

‫الصفحة ‪ 4‬من ‪10‬‬


‫‪ -‬الجزء الثاني‪ 06(:‬نقاط)‬
‫‪ -‬التمرين التجريبي‪ 06( :‬نقاط)‬
‫لتحديد المقادير المميز ) ‪ ( L, r‬لكشيعة ك السعة ‪ C‬لمكثفة نحقؽ التركيب التجريبي المكضح بالشكؿ(‪ )1‬يتككف مف مكلد التكتر‬
‫الثابت قكتو المحركة الكيربائية ‪ E‬ك ثالث نكاقؿ أكمية ‪ R 3 , R 2 , R1  50Ω‬كصماـ ثنائي)‪ (diode‬ك بادلة‪.K‬‬
‫‪ .1‬عند المحظة ‪ t  0‬نضع البادلة في الكضع(‪.)1‬‬
‫) ‪du R 1 (R1  r‬‬ ‫‪R .E‬‬
‫‪‬‬ ‫أ‪ .‬بيف أف المعادلة التفاضمية لمتكتر الكيربائي بيف طرفي الناقؿ األكمي‪ R1‬ىي‪.u R 1  1 (1) :‬‬
‫‪dt‬‬ ‫‪L‬‬ ‫‪L‬‬
‫ب‪ .‬تحقؽ مف أف العبارة ‪ u R 1 (t )  AB .(1  e A ) :‬ىي حال لممعادلة)‪ (1‬حيث ‪ A‬ك ‪ B‬ثابتاف يطمب تعيينيما‪.‬‬
‫‪t‬‬

‫‪ .2‬بكاسطة جياز راسـ االىتزاز الميبطي ذك ذاكرة تمكنا مف مشاىدة البيانييف (أ) ك (ب) المكضحيف بالشكؿ (‪.)2‬‬
‫أ‪ .‬أنقؿ الدارة (‪ )1‬عمى كرقة االجابة مكضحا عمييا كيفية ربط جياز راسـ االىتزاز الميبطي لمشاىدة البيانيف(أ) ك (ب) ‪.‬‬
‫ب‪ .‬أنسب مع التعميؿ كؿ مف البيانيف (أ) ك (ب) لتطكر التكتر الكيربائي بيف طرفي العنصر الكيربائي المكافؽ‪.‬‬
‫ج‪ .‬استخرج قيمة ‪ E‬لممكلد ك شدة التيار الكيربائي األعظمية ‪ I 01‬ثـ أحسب قيمة مقاكمة الكشيعة ‪. r‬‬
‫‪E .1‬‬
‫‪ L ‬ثـ أحسب قيمتيا ‪.‬حيث ‪ 1‬ثابت الزمف ‪.‬‬ ‫د‪ .‬بيف أف ذاتية الكشيعة تعطى بالعبارة‬
‫‪I 01‬‬
‫)𝑉(𝑢‬

‫(أ)‬

‫(ب)‬

‫الشكل (‪)5‬‬
‫الشكل (‪)4‬‬
‫‪1‬‬

‫‪0‬‬ ‫‪20‬‬ ‫)𝒔‪𝒕(m‬‬

‫‪ .3‬أكتب عالقة ) ‪ u R 1 (t‬بداللة ‪ R1‬ك ) ‪. i 1 (t‬كاستنتج العبارة الحظية لشدة التيار الكيربائي ) ‪ i 1 (t‬المارة في الدارة (‪)1‬‬
‫‪‬‬
‫‪ .4‬استنتج العبارة المحظية لمطاقة المخزنة في الكشيعة ‪ .‬ك أحسب قيمتيا عند المحظة ‪. t  1‬‬
‫‪2‬‬
‫‪ .5‬نضع البادلة في الكضع (‪ )2‬عند لحظة ‪ t  0‬نعتبرىا كمبدأ لمزمف كنسجؿ تغيرات شدة التيار المارة في كؿ مف الدارة‬
‫(‪ )2‬ك (‪ )3‬بداللة الزمف‪ ،‬القياسات التجريبية المسجمة مكنتنا مف رسـ المنحنييف (ج) ك (د) المكضحيف بالشكؿ (‪. )3‬‬
‫‪120‬‬
‫‪12‬‬
‫‪100‬‬ ‫‪i mA ‬‬ ‫‪i mA ‬‬
‫‪10‬‬
‫‪80‬‬ ‫المنحنى(د)‬ ‫المنحنى(ج)‬
‫‪8‬‬
‫‪60‬‬
‫‪6‬‬
‫‪40‬‬
‫‪4‬‬
‫‪20‬‬
‫‪2‬‬
‫‪0‬‬ ‫)‪t (ms‬‬ ‫‪t s‬‬
‫‪0‬‬
‫‪0‬‬ ‫‪10‬‬ ‫‪20‬‬ ‫‪30‬‬ ‫‪40‬‬ ‫‪50‬‬
‫‪0‬‬ ‫‪0.005‬‬ ‫‪0.01‬‬ ‫‪0.015‬‬ ‫‪0.02‬‬ ‫‪0.025‬‬ ‫‪0.03‬‬
‫الشكل(‪)6‬‬

‫الصفحة ‪ 5‬من ‪10‬‬


‫أ‪ .‬ما دكر الصماـ الثنائي )‪. (diode‬‬
‫ب‪ .‬حدد مف بيف المنحنييف (ج) ك (د) ما ىك المنحنى المكافؽ لػمدارة (‪ ، )3‬مع التعميؿ ‪.‬‬
‫ج‪ .‬جد قيمة ثابت الزمف ‪𝜏3‬الخاص بالدارة (‪ )3‬ثـ أحسب قيمة مقاكمة الناقؿ األكمي ‪. R 3‬‬
‫د‪ .‬جد قيمة ثابت الزمف ‪  2‬الخاص بالدارة (‪ )2‬ثـ أحسب قيمة ‪ R 2‬ك ‪. C‬‬

‫انتهى املوضوع األول‬

‫الصفحة ‪ 6‬من ‪10‬‬


‫اﻟﻤوﻀوع اﻟﺜﺎﻨﻲ‬
‫�ﺤﺘوي اﻟﻤوﻀوع ﻋﻠﻰ )‪ (04‬ﺼﻔﺤﺎت )ﻤن اﻟﺼﻔﺤﺔ ‪ 07‬ﻤن ‪ 10‬إﻟﻰ اﻟﺼﻔﺤﺔ ‪ 10‬ﻤن ‪(10‬‬
‫اﻟﺠزء اﻷول‪ 14) :‬ﻨﻘطﺔ(‬
‫‪‬‬ ‫اﻟﺘﻤر�ن اﻷول‪ 06) :‬ﻨﻘﺎط(‬
‫ينتقؿ جسـ )‪ (s‬كتمتو ‪ m  0,2Kg‬عمى مسار ‪ ABC‬حيث يخضع أثناء حركتو عمى طكؿ ىذا المسار الى قكة احتكاؾ ‪f‬‬
‫ثابتة في الشدة ك حامميا مكازم لمسار الحركة كجيتيا معاكسة التجاه الحركة ‪ ،‬أنظر الشكؿ‪.2-‬‬
‫يتككف المسار ‪ ABC‬مف جزئييف ‪ :‬الجزء ‪ AB‬مستقيـ أفقي ك الجزء ‪ BC‬مستقيـ مائؿ عف األفؽ بزاكية𝛼 ‪.‬‬
‫يتحرؾ الجسـ )‪ (s‬عمى ثالث مراحؿ ‪:‬‬
‫‪‬‬
‫المرحمة األولى ‪ :‬يتحرؾ عمى الجزء ‪ AB‬بسرعة ثابتة تحت تأثير قكة محركة ‪ F‬شدتيا ثابتة كيصنع حامميا زاكية ‪  60o‬‬
‫مع المستكم األفقي ك تنعدـ عند كصكؿ الجسـ )‪ (S‬الى النقطة ‪. B‬‬
‫المرحمة الثانية ‪ :‬يصعد الجسـ )‪(s‬بعد ذلؾ كفؽ المستكم المائؿ ‪ BC‬حتى النقطة ‪( N‬حيث تنعدـ سرعتو)‬
‫المرحمة الثالثة ‪ :‬ينزؿ الجسـ )‪(s‬عمى المستكم المائؿ ‪ BC‬انطالقا مف النقطة ‪ N‬الى غاية كصكلو الى النقطة ‪. B‬‬
‫سمح لنا التصكير المتعاقب بعد معالجتو ببرنامج ال ػ ‪ AviStep‬مف رسـ منحنى السرعة ) ‪ v  f (t‬المكضح بالشكؿ‪1-‬‬
‫‪x‬‬
‫‪‬‬
‫‪C‬‬ ‫‪F‬‬
‫) ‪(S‬‬
‫‪N‬‬ ‫‪x‬‬ ‫‪‬‬
‫‪x‬‬ ‫‪x‬‬
‫‪‬‬
‫‪B‬‬ ‫‪A‬‬
‫الشكؿ‪1-‬‬
‫‪ .1‬اعتمادا عمى البياف المكضح في الشكل‪ : 2-‬حدد قيمة التسارع خالؿ كؿ مرحمة مف مراحؿ الحركة ‪ .‬ثـ حدد طبيعة‬
‫الحركة مع التعميؿ ‪.‬‬
‫‪ .2‬دراسة حركة الجسـ )‪ (s‬عمى المسار ‪ : AB‬أ‪ .‬مثؿ القكل المؤثرة عمى الجسـ )‪. (s‬‬
‫ب‪ .‬بتطبيؽ القانكف الثاني لنيكتف عمى الجممة (جسـ)‪ , )(s‬جد عالقة بيف ‪ F‬ك ‪ f‬ك ‪. ‬‬
‫‪ .3‬دراسة حركة الجسـ )‪ (s‬عمى المسار ‪ : BN‬أ‪ .‬مثؿ القكل المؤثرة عمى الجسـ )‪. (s‬‬
‫ب‪ .‬بتطبيؽ القانكف الثاني لنيكتف عمى الجممة (جسـ)‪ , )(s‬جد عالقة بيف تسارع الحركة ‪ a2‬ك ‪. f ,  , g , m‬‬
‫‪ .4‬دراسة حركة الجسـ )‪ (s‬عمى المسار ‪: NB‬أ‪ .‬مثؿ القكل المؤثرة عمى الجسـ )‪. (s‬‬
‫)𝒔‪𝒗(𝒎/‬‬
‫ب‪ .‬بتطبيؽ القانكف الثاني لنيكتف عمى‬
‫الجممة (جسـ)‪ , )(s‬جد عالقة بيف تسارع‬
‫الشكل‪2-‬‬
‫الحركة ‪ a3‬ك ‪. f ,  , g , m‬‬
‫‪ .5‬باستخداـ العالقات السابقة حدد ما يمي‪:‬‬
‫‪2,4‬‬ ‫أ‪ .‬زاكية الميؿ ‪ ‬لممستكم ‪. BC‬‬
‫‪‬‬
‫‪0‬‬ ‫ب‪ .‬شدة قكة االحتكاؾ ‪. f‬‬
‫‪0,25‬‬ ‫‪,‬‬ ‫)𝒔(𝒕‬ ‫‪‬‬
‫‪-2,4‬‬ ‫ج‪ .‬شدة القكة المحركة ‪. F‬‬
‫المعطيات‪g  10m / S 2 :‬‬

‫الصفحة ‪ 7‬من ‪10‬‬


‫التمرين الثاني‪ 44( :‬نقاط)‬
‫محموال تجاريا ) ‪ (S 0‬لغاز النشادر ‪ NH 3‬نسبة نقاوتو ‪ 28%‬وكثافتو‪. d  0,91‬‬
‫‪ .1‬بين أن تركيز المحمول التجاري ىو ‪.C 0 15mol / L‬‬
‫‪ .2‬انطالقا من المحمول التجاري حضرنا محموال ) ‪ (S‬حجمو ‪ 3L‬و تركيزه المولي ‪.C  0,01mol / L‬‬
‫*اذكر البروتوكول التجريبي لتحضير المحمول ) ‪ (S‬مع ذكر الزجاجيات المستعممة ‪.‬‬
‫‪ .3‬قياس الناقمية النوعية لممحمول ) ‪ (S‬أعطى القيمة ‪.  f 10,9mS / m‬‬
‫أ‪ .‬لماذا تمدد المحاليل قبل قياس الناقمية؟‪.‬‬
‫ب ‪ .‬أكتب معادلة انحالل النشادر ‪ NH 3‬في الماء و أنشئ جدول التقدم ليذا التفاعل ‪.‬‬
‫ج‪ .‬أكتب عبارة التركيز ‪  HO  f‬بداللة ‪ , HO , NH‬و ‪ .  f‬ثم أحسب قيمتو ‪.‬‬
‫‪‬‬
‫‪‬‬ ‫‪‬‬
‫‪4‬‬

‫د‪ .‬ثم استنتج قيمة ‪ pH‬المحمول ) ‪. (S‬‬


‫‪10 pH  pKe‬‬
‫‪ , f ‬ثم أحسب قيمتيا و ماذا تستنتج‪.‬‬ ‫ه‪ .‬بين أن نسبة التقدم النيائي ‪  f‬تعطى بالعالقة ‪:‬‬
‫‪C‬‬

‫‪ ، pKa  pH  log‬احسب قيمة‬


‫و‪ .‬أكتب عبارة ثابت الحموضة ‪ Ka‬لمثنائية ) ‪ (NH  / NH‬ثم بين أن ‪ NH 3 f‬‬
‫‪ NH 4 ‬‬
‫‪4‬‬ ‫‪3‬‬
‫‪f‬‬

‫ال ـ ـ ‪ pKa‬لمثنائية ) ‪. (NH 4 / NH 3‬‬


‫التمرين الثالث‪ 4( :‬نقاط)‬
‫تم اكتشاف عمود تتدخل فيه ثنائيات من نوع (معدن‪/‬شاردة معدن) في وقت كان فيه تطور التلغراف في حاجة ملحة‬

‫الشكل)𝟏𝟎(‬ ‫لمنابع التيار الكهربائي المستمر‪.‬‬


‫𝐷‬ ‫يهدف هذا التمرين إلى دراسة عمود نحاس‪-‬ألمنيوم‪.‬‬
‫𝐾‬ ‫‪A‬‬
‫ننجز العمود نحاس – ألمنيوم بوصل نصفي العمود بواسطة جسر ملحي‬
‫𝑙𝐴‬ ‫𝑢𝐶‬
‫جسرملحي‬ ‫لكلور األمونيوم )𝑞𝑎() ‪ ،(𝑁𝐻4+ , 𝐶𝑙 −‬يتكون النصف األول للعمود من‬
‫صفيحة من النحاس مغمور جزئيا في محلول لكبريتات النحاس الثنائي‬
‫)𝑞𝑎() ‪ ،(𝐶𝑢2+ , 𝑆𝑂42−‬تركيزه المولي ‪ 𝐶0‬وحجمه 𝐿𝑚‪𝑉0 = 50‬ويتكون‬
‫النصف الثاني للعمود من صفيحة األلمنيوم مغمورة جزئيا في محلول مائي‬
‫محلول كلور‬ ‫محلول كبريتات‬
‫االلمنيوم‬ ‫النحاس الثنائي‬

‫لكلور األلمنيوم )𝑞𝑎() ‪ (𝐴𝑙 3+ , 3𝐶𝑙 −‬له نفس التركيز المولي ‪ 𝐶0‬ونفس الحجم ‪.𝑉0‬‬
‫نركب بين قطبي العمود ناقال أوميا 𝐷 وأمبير متر𝐴 وقاطعة𝐾 كما يوضحه الشكل)𝟏𝟎(‪.‬‬
‫نغلق الدارة عند اللحظة‪ 𝑡 = 0‬فيمر فيها تيار كهربائي شدته𝐼 ثابتة‪.‬‬
‫يمثل المنحنى البياني الشكل)𝟐𝟎( تغيرات التركيز المولي لشوارد النحاس الثنائي الموجودة في النصف األول للعمود‬
‫بداللة الزمن‪.‬‬

‫الصفحة ‪ 8‬من ‪10‬‬


‫)𝑳‪𝑪𝒖𝟐+ (𝒎𝒎𝒐𝒍/‬‬ ‫الشكل)𝟐(‬

‫𝟎𝟏‬

‫𝟎‬ ‫)𝒔(𝒕‬
‫‪500‬‬
‫‪ -1‬أ‪-‬باالعتماد على معيار التطور التلقائي‪ ،‬حدد منحى تطور المجموعة الكيميائية المدروسة‪.‬‬
‫ب‪-‬حدد قطبي العمود وأعط الرمز االصطالحي للعمود المدروس‪.‬‬
‫ج‪ -‬أعد رسم العمود وبين عليه جهة التيار الكهربائي وجهة حامالت الشحن خارج العمود وداخله‪.‬‬
‫‪ -2‬أ‪-‬أنجز جدول تقدم التفاعل‪.‬‬
‫ب‪-‬أوجد عبارة ] ‪[𝐶𝑢2+‬عند اللحظة 𝑡بداللة𝐼‪ 𝑉0 ،𝐹،𝐶0 ،‬و 𝑡‬
‫ج‪ -‬استنتج قيمة شدة التيار𝐼 المار في الدارة‪.‬‬
‫‪ -3‬يستهلك العمود كليا عند اللحظة ‪ ،𝑡c‬أوجد بداللة𝐼 ‪ 𝑀، 𝑡c ،‬و𝐹 التغير 𝑚∆ لكتلة صفيحة االلمنيوم عندما يستهلك‬
‫العمود كليا‪ ،‬ثم أحسب 𝑚∆‪.‬‬
‫المعطيات‪𝑀(𝐴𝑙) = 27g/𝑚𝑜𝑙 1𝐹 = 96500𝐶/𝑚𝑜𝑙 :‬‬
‫‪2+‬‬
‫)𝑞𝑎(𝑢𝐶‪3‬‬ ‫‪3+‬‬
‫)𝑞𝑎(𝑙𝐴‪+ 2𝐴𝑙(𝑠) = 3𝐶𝑢(𝑠) + 2‬‬ ‫معادلة التفاعل بين شوارد النحاس ومعدن النحاس‬
‫ثابت التوازن المقرون بمعادلة التفاعل بين شوارد النحاس ومعدن األلمنيوم ‪𝐾 = 1020‬‬
‫‪ -‬الجزء الثاني‪ 06(:‬نقاط)‬
‫‪ -‬التمرين التجريبي‪ 06( :‬نقاط)‬
‫لمبمكتكنيكـ عدة نظائر منيا ما ىك قابؿ لمتفكؾ االشعاعي ك منيا ما ىك قابؿ لالنشطار النككم‬
‫كفؽ نمط التفكؾ‬ ‫لمبمكتكنيكـ𝑢𝑃‪94‬‬
‫‪238‬‬
‫‪ -I‬تعتمد بعض المحركات عمى بطارية نككية تكلد طاقة متحررة عف النشاط االشعاعي‬
‫‪ ‬معطيا نكاة اليكرانيكـ 𝑈 𝑍𝐴 ‪.‬‬
‫‪ .1‬عرؼ النظائر‪.‬‬
‫كفؽ النمط ‪ ، ‬مع تحديد كؿ مف ‪ A‬ك ‪. Z‬‬ ‫البمكتكنيكـ𝑢𝑃‪94‬‬
‫‪238‬‬
‫‪ .2‬أكتب معادلة التفكؾ النككم لنظير‬
‫كتحديد عدد األنكية االبتدائية ‪𝑁0‬المحتكاة في عينة منو قمنا في‬ ‫لمبمكتكنيكـ𝑢𝑃‪94‬‬
‫‪238‬‬
‫‪ -II‬مف أجؿ تعييف زمف نصؼ العمر‬
‫لحظات زمنية متتالية بقياس النشاط االشعاعي ‪ A‬ك كذا عدد أنكية اليكرانيكـ 𝑈𝑁 الناتجة ‪ ،‬نتائج القياسات التجريبية مكنتنا‬
‫مف رسـ المنحنى ) 𝑈𝑁(𝑓 = 𝐴 الكضح في الشكؿ(‪. )1‬‬

‫الصفحة ‪ 9‬من ‪10‬‬


‫)𝑠‪𝐴(1010 (𝑛𝑜𝑦𝑎𝑢𝑥/‬‬

‫‪ .1‬أكتب قانكف التناقص االشعاعي ك استنتج عبارة )𝑡(𝐴 بداللة‬


‫عدد أنكيةالبمكتكنيكـ المتبقية )𝑡(𝑁 ‪.‬‬
‫‪ .2‬أكتب العالقة بيف )𝑡(𝑁 ك 𝑈𝑁 ‪.‬‬
‫الشكل(‪)1‬‬ ‫‪ .3‬أكتب العالقة بيف )𝑡(𝐴 ك 𝑈𝑁 ك ثابت النشاط االشعاعي 𝜆 ‪.‬‬
‫‪ .4‬اعتمادا عمى منحنى ) 𝑈𝑁(𝑓 = 𝐴 ك عالقة السؤاؿ‪. 3‬‬
‫‪ -‬جد ما يمي ‪𝑁0 (𝑃𝑢 ) ، 𝑡1/2 ، 𝜆 ، A0 :‬‬
‫‪ -III‬في أحد المفاعالت النككية الذم يعتمد عمى االنشطار النككم ألحد‬
‫‪1‬‬
‫المتسمسؿ ك المغذل ذاتيا ك المكصكؼ‬ ‫‪A‬‬
‫نظائر البمكتكنيكـ ‪P‬‬
‫‪Z u‬‬

‫‪0‬‬ ‫‪1‬‬ ‫‪2,4‬‬ ‫)𝑦𝑜𝑛 ‪𝑁𝑈 (1020‬‬ ‫بمخطط الحصيمة الطاقكية المكضحة بالشكؿ‪. 2-‬‬

‫‪ .1‬عرؼ االنشطار النككم ك لماذا نقكؿ عنو أنو تفاعؿ تسمسمي مغذل ذاتيا‪.‬‬
‫‪ .2‬باالعتماد عمى مخطط الحصيمة الطاقكية ‪:‬‬
‫أ‪ .‬حدد تركيب نكاة البمكتكنيكـ الخاضعة لالنشطار النككم‪.‬‬

‫الشكل‪2-‬‬ ‫ب‪ .‬أكتب معادلة تفاعؿ االنشطار النككم لمبمكتكنيكـ ‪،‬‬


‫مع تحديدكؿ مف ‪ Z ‬ك ‪. x‬‬
‫‪ .3‬ماذا تمثؿ المقادير ‪. E 3 ، E 2 ، E 1‬‬
‫– أحسب قيمة ‪. E 2‬‬

‫‪.‬‬ ‫‪A‬‬
‫‪ .4‬استنتج طاقة ربط نكاة البمكتكنيكـ ‪P‬‬
‫‪Z u‬‬

‫‪ . ZA Pu ،‬مع التعميؿ‪.‬‬ ‫‪Mo ،‬‬ ‫‪ .5‬حدد النكاة األكثر استقرار مف بيف األنكية‪T :‬‬
‫‪102‬‬ ‫‪135‬‬
‫‪42‬‬ ‫‪Z e‬‬

‫بكحدة ‪ Mev‬ثـ بكحدة ‪. joule‬‬ ‫‪239‬‬


‫‪ .6‬أحسب قيمة الطاقة المحررة مف انشطار نكاة كاحدة مف البمكتكنيكـ ‪P‬‬
‫‪94 u‬‬

‫خالؿ مدة مف الزمف قدرىا ‪ t‬في انتاج الكيرباء‬ ‫‪A‬‬


‫‪ .7‬يستيمؾ ىذا المفاعؿ النككم كتمة قدرىا ‪ 97, 45Kg‬مف البمكتكنيكـ ‪P‬‬
‫‪Z u‬‬

‫باستطاعة قيمتيا ‪ Pe  900.MW‬ك بمردكد ‪. 30%‬‬


‫أ‪ .‬جد المدة الزمنية ‪ t‬الشتغاؿ المفاعؿ النككم ‪.‬‬
‫‪، N A  6,02 1023 mol 1‬‬ ‫‪، m  ZA Pu   239,00060u ،‬‬ ‫)‪m(10 n) 1,00866(u‬‬ ‫المعطيات‪:‬‬
‫‪1Mev  1,6 1013 j‬‬ ‫‪،‬‬ ‫‪1Mev  1,6 1013 j‬‬ ‫‪،‬‬ ‫‪1u  931,5Mev / C 2‬‬

‫‪1année  365,25 j‬‬ ‫‪، El‬‬ ‫‪( 135‬‬


‫‪ZT‬‬‫‪ e )  8,48Mev / nuc‬‬ ‫‪،‬‬ ‫‪42 Mo)  873,773Mev‬‬
‫‪E l ( 102‬‬
‫‪A‬‬

‫اﻨﺘﻬﻰ اﻟﻤوﻀوع اﻟﺜﺎﻨﻲ‬

‫الصفحة ‪ 10‬من ‪10‬‬


‫االجابت النموذجيت الختبار الباكالوريا التجريبي في مادة ‪ :‬العلوم الفيزيائيت ‪ /‬الشعبت ‪ :‬رياضياث ‪ ،‬تقني رياضي‬
‫الموضوع األول‬
‫التمرين ألاول‪ 06 ( :‬نقاط)‬
‫‪–I‬‬
‫‪ .1‬الثنائيتني ) ‪ ( ox / red‬املشاركيت هذا التفاعل‪ :‬لتحديدها نكتب املعادلتني النصفيتني لألكسدة واإلرجاع‪:‬‬
‫‪𝒁𝒏(𝒔) → 𝒁𝒏𝟐+‬‬‫‪(𝒂𝒒) + 𝟐é‬‬ ‫املعادلة النصفية لألكسدة‪:‬‬
‫𝟓‪𝟎. 2‬‬ ‫‪+‬‬
‫)𝒍(𝑶 𝟐𝑯𝟐 ‪𝟐𝑯𝟑 𝑶(𝒂𝒒) + 𝟐é → 𝑯𝟐(𝒈) +‬‬ ‫املعادلة النصفية لإلرجاع‪:‬‬
‫ومنه الثنائيتني )𝒅𝒆𝑹‪ (𝑶𝒙/‬الداخلتني يف التفاعل‪ (𝒁𝒏𝟐+ /𝒁𝒏) :‬و ) 𝟐𝑯‪.(𝑯𝟑 𝑶+ /‬‬
‫‪ .2‬متثيل جدول تقدم التفاعل‪:‬‬
‫𝟓‪𝟎. 2‬‬
‫معادلة التفاعل‬ ‫𝑯 = 𝒏𝒁 ‪𝟐𝑯 𝑶+ +‬‬ ‫‪+ 𝒁𝒏𝟐+‬‬ ‫𝑶 𝑯𝟐 ‪+‬‬
‫𝟑‬ ‫)𝒒𝒂(‬ ‫)𝒔(‬ ‫)𝒈(𝟐‬ ‫)𝒒𝒂(‬ ‫𝟐‬ ‫)𝒍(‬

‫حالة اجلملة‬ ‫التقدم‬ ‫كميات املادة باملول )‪(mol‬‬


‫حالة إبتدائية‬ ‫‪𝑚0‬‬
‫‪0‬‬ ‫𝑉𝐶 = ‪𝑛01‬‬ ‫‪𝑛02‬‬ ‫=‬ ‫‪0‬‬ ‫‪0‬‬ ‫بوفرة‬
‫)𝑛𝑍(𝑀‬
‫حالة إنتقالية‬ ‫)‪X(t‬‬ ‫)𝑡(𝑥‪𝑛01 − 2‬‬ ‫)𝑡(𝑥 ‪𝑛02 −‬‬ ‫)‪x(t‬‬ ‫)‪x(t‬‬ ‫بوفرة‬
‫حالة نهائية‬ ‫𝑥𝑎𝑚𝑥‬ ‫𝑥𝑎𝑚𝑥‪𝑛01 − 2‬‬ ‫𝑥𝑎𝑚𝑥 ‪𝑛02 −‬‬ ‫𝑥𝑎𝑚𝑥‬ ‫𝑥𝑎𝑚𝑥‬ ‫بوفرة‬

‫𝟓𝟐 ‪𝟎.‬‬
‫‪ .1 .3‬حساب تركيز شوارد‪ 𝑯𝟑 𝑶+‬يف احلالة النهائية‪[𝐻3 𝑂+ ]𝑓 = 10−𝑝𝐻𝑓 = 10−1,698 = 0,02𝑚𝑜𝑙/𝐿 :‬‬
‫استنتاج كمية مادة 𝑶 𝟑𝑯 يف هذه احلالة النهائية‪𝟎. 𝟐𝟓 :‬‬
‫‪+‬‬

‫𝑙𝑜𝑚 ‪𝑛𝑓 (𝐻3 𝑂+ ) = [𝐻3 𝑂+ ]𝑓 . 𝑉 = 10−𝑝𝐻𝑓 . 𝑉 = 0,02 × 0,1 = 2 × 10−3‬‬


‫‪ .2.3‬حتديد املتفاعل احملد‪ :‬مبا أن ‪ 𝑛𝑓 (𝐻3 𝑂+ ) ≠ 0‬فشوارد ‪ 𝐻3 𝑂+‬ليست متفاعل حمد‪ ،‬ومنه حتما قطعة الزنك 𝒏𝒁‬
‫𝟓𝟐 ‪𝟎.‬‬ ‫هي املتفاعل احملد‪.‬‬
‫𝟓𝟐 ‪𝟎.‬‬ ‫استنتاج قيمة التقدم االعظمي ‪: x max‬‬
‫‪𝑛0 (𝐻3‬‬ ‫𝑛‪𝑂+ )−‬‬ ‫𝑂 ‪𝑓 (𝐻3‬‬
‫)‪+‬‬
‫‪𝐶𝑉−2×10−3‬‬
‫= 𝑥𝑎𝑚𝑥 ⟹ 𝑥𝑎𝑚𝑥‪𝑛𝑓 (𝐻3 𝑂+ ) = 𝑛0 (𝐻3 𝑂+ ) − 2‬‬ ‫=‬
‫‪2‬‬ ‫‪2‬‬
‫‪5 × 10−3 − 2 × 10−3‬‬
‫=‬ ‫𝑙𝑜𝑚 ‪= 1,5 × 10−3‬‬
‫‪2‬‬
‫𝑙𝑜𝑚 ‪𝑥𝑚𝑎𝑥 = 1,5 × 10−3‬‬ ‫ومنه‪:‬‬
‫𝟓𝟐 ‪𝟎.‬‬
‫‪ .3.3‬إجياد الكتلة املتفاعلة من الزنك 𝟎𝒎‪ :‬مبا أن 𝒏𝒁 متفاعل حمد فإن ‪𝑛𝑓 (𝑍𝑛) = 0 ⟺ 𝑛02 − 𝑥𝑚𝑎𝑥 = 0 :‬‬
‫‪𝑚0‬‬
‫)𝑛𝑍(𝑀‬
‫𝑥𝑎𝑚𝑥 ‪−‬‬ ‫وبالتالي‪= 0 ⟹ 𝑚0 = 𝑥𝑚𝑎𝑥 . 𝑀(𝑍𝑛) = 1,5 × 10−3 × 64,5 = 0.09675𝑔 :‬‬
‫]‪+‬‬ ‫‪-II‬‬
‫𝑂‪3‬‬ ‫𝑙𝑜𝑚 ‪× 10−2‬‬
‫𝟓‬ ‫‪ .1‬إكمال املنحنى‪:‬‬
‫𝐿‪/‬‬ ‫𝟐 ‪𝟎.‬‬
‫‪+‬‬ ‫‪−2‬‬
‫التعليل‪ :‬ألن‪𝟎. 𝟐𝟓 [𝐻3 𝑂 ]𝑓 = 2 × 10 𝑚𝑜𝑙/𝐿 :‬‬
‫‪ – 2‬حتديد بيانيا زمن نصف التفاعل 𝟐‪: 𝐭 𝟏/‬‬
‫𝟓𝟐 ‪𝟎.‬‬ ‫𝑓] ‪[𝐻3 𝑂 + ]0 +[𝐻3 𝑂+‬‬ ‫‪5×10−2 +2×10−2‬‬
‫‪2‬‬ ‫= ‪[𝐻3 𝑂+ ]1/2‬‬ ‫=‬ ‫𝐿‪= 3.5 × 10−2 𝑚𝑜𝑙/‬‬
‫‪2‬‬ ‫‪2‬‬
‫‪1‬‬ ‫𝟓𝟐 ‪𝟎.‬‬ ‫باسقاط هذه القيمة على حنور األزمنة جند ‪𝑡1 = 1.4𝑚𝑖𝑛 :‬‬
‫‪2‬‬
‫‪0‬‬ ‫‪2‬‬ ‫𝒏𝒊𝒎(𝒕‬ ‫‪ .3‬حساب السرعة احلجمية اإلبتدائية إلختفاء شوارد ‪:𝐻3 𝑂+‬‬

‫الصفحة ‪1‬من ‪17‬‬


‫) ‪1 𝑑𝑛(𝐻3 𝑂+‬‬ ‫] ‪𝑑[𝐻3 𝑂+‬‬ ‫‪0− 5. 10‬‬
‫‪−2‬‬
‫‪−2‬‬ ‫𝟓𝟐 ‪𝟎.‬‬
‫‪𝑣𝐻3 𝑂+ (0) = −‬‬ ‫‪=−‬‬ ‫(=‬ ‫𝑛𝑖𝑚 ‪) =1,25.10 𝑚𝑜𝑙/𝐿.‬‬
‫𝑉‬ ‫𝑡𝑑‬ ‫𝑡𝑑‬ ‫‪4−0‬‬
‫استنتاج السرعة احلجمية للتفاعل‪:‬‬ ‫‪-‬‬
‫‪𝑣𝑣 𝐻3 𝑂 +‬‬
‫= ‪𝑣𝑣 0‬‬ ‫𝑛𝑖𝑚 ‪=0,00625 𝑚𝑜𝑙/𝐿.‬‬
‫‪2‬‬

‫𝟓𝟐 ‪𝟎.‬‬ ‫‪ .4‬رسم املنحنى‪ :‬الوصول للنظام الدائم (ينعدم البيان) يف زمن أقل من السابق‪.‬‬
‫العامل احلركي‪ :‬درجة احلرارة‬
‫تأثري العامل احلركي‪ :‬عند ارتفاع درجة احلرارة تزداد حركة اجلسيمات وبالتالي تزداد عدد التصادمات الفعالة ما‬
‫يؤدي لزيادة سرعة التفاعل‪𝟎. 𝟐𝟓 .‬‬
‫𝟓𝟐 ‪𝟎.‬‬ ‫‪ -III‬معايرة حملول النشادر بواسطة حملول محض كلور املاء ‪:‬‬
‫𝑂 ‪𝑁𝐻3 + 𝐻3 𝑂 = 𝑁𝐻4+ + 𝐻2‬‬
‫‪+‬‬
‫‪ .1‬معادلة تفاعل املعايرة ‪:‬‬
‫‪ .2‬الرتكيب التجرييب املستعمل يف تقنية املعايرة مرفق‬
‫بالبيانات‪:‬‬
‫𝟎‬ ‫‪.‬‬ ‫𝟓‬ ‫إكمال البيانات املرقمة ‪:‬‬
‫‪ .2‬حامل سحاحة‪.‬‬ ‫‪ .1‬سحاحة مدرجة ‪.‬‬
‫‪.3‬حملول معاير به ) 𝑙𝐶 ‪(𝐻3 𝑂 +‬‬
‫‪+‬‬ ‫‪−‬‬

‫‪ .5‬جهاز الـ 𝐻𝑝 مرت‪.‬‬ ‫‪ .4‬مسبار جهاز الـ 𝐻𝑝 مرت‪.‬‬


‫‪.7‬خمالط كهرومغناطيسي‪.‬‬ ‫‪.6‬حملول معايَر ) ‪. NH 3 (aq‬‬
‫‪.9‬قضيب مغناطيسي‪.‬‬ ‫‪.8‬بيشر‪.‬‬
‫‪ .3‬أحداثيات نقطة التكافؤ وحساب 𝐵𝐶‪:‬‬
‫‪ -‬احداثيات نقطة التكافؤ ’‘ ‪:‘E‬‬
‫𝟓𝟐 ‪E(PH𝐸 = 6, V𝑎𝐸 = 15𝑚𝐿) 𝟎.‬‬
‫) ‪𝑛𝐸 (𝐻3 𝑂+ ) = 𝑛(𝑁𝐻3‬‬ ‫عند نقطة التكافؤ يصبح املزيج ستوكيومرتي‪ :‬أي‪:‬‬ ‫‪ -‬حساب قيمة ‪: Cb‬‬
‫𝐿‪[𝐻3 𝑂+ ]𝑓 = 𝐶𝑎 = 2 × 10−2 𝑚𝑜𝑙/‬‬ ‫ونعلم أن‪:‬‬
‫𝟓𝟐 ‪𝟎.‬‬ ‫𝐸𝑎𝑉 𝑎𝐶‬ ‫‪2×10−2 ×15‬‬
‫𝐸𝑎𝑉 𝑎𝐶 = 𝐵𝑉 𝐵𝐶‬ ‫= 𝐵𝐶 ⟹‬ ‫=‬ ‫ومنه ‪= 0.015𝑚𝑜𝑙/𝐿 :‬‬
‫𝐵𝑉‬ ‫‪20‬‬
‫𝐿‪𝐶𝐵 = 0.015𝑚𝑜𝑙/‬‬
‫‪ .4‬تعيني قيمة ثابت احلموضة 𝒂 𝐊𝐏 للثنائية ))‪ ( NH4 +(aq) / NH3 (aq‬بيانيا‪:‬عند نقطة نصف التكافؤ واليت توافق‪:‬‬
‫‪V‬‬
‫𝐿𝑚‪ 2𝐵𝐸 = 7.5‬وعند إسقاطها بيانيا يكون‪𝟎. 𝟐𝟓 𝐏𝐊 𝒂 = 𝐏𝐇 =𝟗. 𝟐 :‬‬
‫‪ . 5‬حساب ثابت التوازن ‪K‬‬
‫‪+‬‬
‫𝑓] ‪[𝑁𝐻4‬‬ ‫‪1‬‬ ‫‪1‬‬ ‫‪𝟎.‬‬ ‫𝟓𝟐‬
‫=‪K‬‬ ‫‪+‬‬
‫=‬ ‫‪= −𝑃𝐾 = 10𝑃𝐾𝑎 = 109.2 = 1.58 × 109‬‬
‫𝑎 ‪[𝑁𝐻3 ]𝑓 . [𝐻3 𝑂 ]𝑓 𝐾𝑎 10‬‬
‫نالحظ أن‪ K = 1.58 × 109 > 104 :‬ومنه نستنتج أن تفاعل املعايرة تفاعل تام 𝟓𝟐 ‪𝟎.‬‬
‫‪ - .6‬حتديد احلجم 𝟏𝒂𝑽 من حملول محض كلوراملاء الذي جيب اضافته لكي تتحقق العالقة ‪𝟎. 2𝟓 :‬‬
‫] ‪[𝑁𝐻4 +‬‬
‫⟹ ] ‪[𝑁𝐻4 + ] = 15 [𝑁𝐻3‬‬ ‫] ‪[𝑁𝐻3‬‬
‫‪= 15‬‬ ‫] 𝟑𝑯𝑵[ 𝟓𝟏 = ] ‪ [𝑵𝑯𝟒 +‬يف املزيج التفاعلي ‪:‬‬
‫‪1‬‬ ‫] ‪[𝑁𝐻3‬‬
‫‪ PH = PK 𝑎 + log‬ومنه ‪PH = 9.2 + log( ) = 9.2 − 1.2 = 8 :‬‬ ‫ولدينا ‪:‬‬
‫‪15‬‬ ‫] ‪[𝑁𝐻4 +‬‬
‫𝐿𝑚‪𝑉𝑎1 = 14‬‬
‫‪ PH = 8‬باالسقاط جند ‪:‬‬

‫الصفحة ‪ 2‬من ‪17‬‬


‫العالمة‬
‫مجموع‬ ‫مجزأة‬ ‫حل التمرين الثاني‪ 74( :‬نقاط)‬

‫‪0,25‬‬ ‫‪0,25‬‬
‫‪ .3 /I‬المرجع المناسب لدراسة حركة هذا القمر‪ :‬مرجع جيو مركزي( مركزي أرضي)‪.‬‬

‫‪v‬‬ ‫‪ .2‬تمثيل شعاع السرعة المدارية ‪ v‬وشعاع قوة جذب األرض ‪: F T /S‬‬
‫) ‪(S‬‬
‫‪n‬‬
‫‪F T /S‬‬
‫‪0,50‬‬ ‫‪0,25×2‬‬
‫‪h‬‬
‫‪RT‬‬

‫بداللة‪ r ، m S ، M T ، G :‬و ‪: n‬‬ ‫‪ .3‬كتابة العبارة الشعاعية للقوة ‪F T /S‬‬


‫‪0,25‬‬ ‫‪0,25‬‬ ‫‪mS . M T‬‬
‫‪F T / S G .‬‬ ‫‪n‬‬
‫‪r2‬‬
‫مميزات شعاع تسارع مركز عطالة القمر ) ‪ ( S‬واستنتاج طبيعة الحركة‪:‬‬
‫‪ّ .1.1‬‬
‫‪0,62‬‬ ‫بتطبيق القانون الثاني لنيوتن في معلم عطالي ‪ F ext  mS aG‬‬
‫‪FT S‬‬ ‫‪M‬‬
‫‪F T S  mS . a G‬‬ ‫‪ aG ‬‬ ‫‪ G 2T .n‬‬
‫‪mS‬‬ ‫‪r‬‬
‫‪0,62‬‬ ‫‪ -‬مبدؤه مركز العطالة – حامله ناظمي – جهته نحو مركز األرض – شدته ثابتة‬
‫‪0,25‬‬ ‫‪ -‬طبيعة الحركة‪ :‬بما أن المسار دائري والتسارع مركزي(ناظمي) ثابت فالحركة دائرية منتظمة‪.‬‬
‫‪FT‬‬ ‫‪v 2 G MT‬‬
‫‪ .2.1‬عبارة ‪ v‬بداللة ‪ M T ، G‬و ‪: r‬‬
‫‪G MT‬‬
‫‪aG ‬‬ ‫‪‬‬ ‫‪‬‬ ‫‪v‬‬
‫‪S‬‬
‫‪1,25‬‬ ‫‪0,25‬‬ ‫‪mS‬‬ ‫‪r‬‬ ‫‪r‬‬ ‫‪2‬‬
‫‪r‬‬

‫‪2 r‬‬ ‫‪r3‬‬


‫‪0,25‬‬ ‫‪T ‬‬ ‫‪ T  2‬‬ ‫‪ .3.1‬عبارة الدور ‪: TS‬‬
‫‪v‬‬ ‫‪G MT‬‬

‫‪ .1 /II‬باستغالل البيان الممثَّل كتابة المعادلة الرياضية‪:‬‬


‫البيان خط مستقيم يمر من المبدأ معادلته من الشكل‪ 2,11016 . 2 :‬‬
‫‪1‬‬ ‫‪1‬‬
‫‪0,25‬‬ ‫‪FT‬‬ ‫‪S‬‬ ‫‪2‬‬
‫‪ A.‬‬
‫‪r‬‬ ‫‪r‬‬
‫حيث ‪ A‬معامل توجيه البيان العالقة النظرية‪FT S  K . m S . 2 :‬‬
‫‪0,50‬‬ ‫‪1‬‬
‫‪r‬‬
‫استنتاج قيمة الثابت ‪ K‬حيث ) ‪ . ( K GM T‬بالمطابقة‪ 39, 6 1013 SI :‬‬
‫‪A‬‬
‫‪0,25‬‬ ‫‪K‬‬
‫‪mS‬‬
‫‪ .1.2‬االرتفاع ‪ h‬عن سطح األرض‪h  r  RT :‬‬
‫‪0,25‬‬
‫‪1‬‬
‫‪2‬‬
‫بما أن‪ FT S 11,8 102 N :‬من البيان نجد‪ 5,58 1016 :‬‬
‫‪r‬‬
‫‪1‬‬ ‫‪1‬‬
‫‪0,75‬‬ ‫‪2‬‬
‫‪ 5,581016  r ‬‬ ‫‪ 4, 23.107 m  4, 23.104 km‬‬
‫‪r‬‬ ‫‪5,581016‬‬
‫‪h  4, 23.104 6, 4.103 3,59.104 km‬‬

‫الصفحة ‪ 3‬من ‪17‬‬


‫‪ .2.2‬السرعة المدارية ‪: v‬‬
‫‪0,25‬‬
‫‪G MT‬‬ ‫‪K‬‬ ‫‪39, 6 1013‬‬
‫‪v‬‬ ‫‪‬‬ ‫‪‬‬ ‫‪ 3060 m / s  3, 06km / s‬‬
‫‪r‬‬ ‫‪r‬‬ ‫‪4, 23 107‬‬
‫‪ .3.2‬الدور ‪: TS‬‬
‫‪0,25‬‬
‫‪2 r 2  4, 23 107‬‬
‫‪T‬‬ ‫‪‬‬ ‫‪ 86811, 76s  24 h‬‬
‫‪v‬‬ ‫‪3060‬‬
‫‪ .3‬نعم القمر ُسهيل سات ‪ 2‬جيو مستقر ألنه يحقق الشروط التالية‪:‬‬

‫‪0,50‬‬ ‫دوره يساوي دور األرض حول محورها ‪T S  24 h‬‬


‫‪0,50‬‬
‫من السياق يظهر ساكنا بالنسبة لمالحظ على سطح األرض فهو يدور في نفس جهة دوران‬
‫األرض ومساره يقع في مستوي خط االستواء‪.‬‬

‫العالمة‬ ‫حل التمرين ‪ 04( 3‬نقاط)‪:‬‬


‫‪.1‬تحديد بيانيا ما يمي ‪:‬‬
‫‪0.25‬‬ ‫‪t  5 0,4S  2S‬‬ ‫أ‪ .‬زمف كصكؿ الصندكؽ الى سطح األرض ‪.‬‬
‫‪0.25‬‬ ‫يتحرؾ الصندكؽ كفؽ مسار منحني ألف حامؿ القكة ك ال يتطابؽ مع حامؿ شعاع السرعة (متعامداف)‬
‫ُ‬ ‫ب‪ .‬قيمة المركبة األفقية 𝑥𝑣لسرعة الصندكؽ ك رسـ‬
‫)‪Vx(m/S‬‬
‫‪4‬‬
‫‪0.25‬‬ ‫الباف ) ‪. vx (t‬‬
‫ُ‬
‫‪0‬‬ ‫‪0.4‬‬ ‫)‪t(S‬‬
‫) ‪dx (t‬‬
‫‪v x ‬كمنو تحدد بيانيا مف منحنى‬ ‫لدينا‬
‫‪dt‬‬
‫) ‪ x (t‬بحساب ميؿ المماس ‪:‬‬
‫‪x (0  50)m‬‬
‫‪0.25‬‬ ‫‪v x  tan  ‬‬ ‫‪‬‬ ‫‪ 25m / S‬‬
‫‪t‬‬ ‫‪(2  0)S‬‬

‫رسـ البياف ‪vx (t ) :‬‬

‫الصفحة ‪ 4‬من ‪17‬‬


‫ج‪ .‬احداثيات نقطة انطالؽ الصندكؽ ) ‪. A (x A , z A‬‬
‫‪0.25‬‬ ‫مف المنحنى(أ)‪, x A  x  0   510  50m :‬‬
‫‪0.25‬‬ ‫‪20m / S 2S‬‬
‫‪zA ‬‬ ‫ك مف المنحنى(ب) ‪ z A‬تمثؿ المساحة‪ 20m :‬‬
‫‪2 ‬‬
‫‪ .2‬ىؿ يمكف اعتبار المعمـ المرتبط بسطح األرض )‪ (o,i,k‬معمما عطاليا ك ماىي الفرضية المعتبرة في ذلؾ‪.‬‬
‫‪‬‬
‫‪0.25‬‬ ‫نعـ يمكف اعتبار المعمـ )‪ (o,i,k‬المرتبط بسطح األرض في النقطة ‪( O‬معمـ سطحي أرضي ) عطاليا‬
‫ألف زمف دراسة الحركة (‪20‬ثانية) صغير جدا مقارنة بزمف دكر األرض حكؿ نفسيا (‪24‬ساعة)‪.‬‬
‫‪0.25‬‬ ‫كالفرضية ىي اعتبار حركة المرجع المرتبط بق المعمـ ساكف أك يتحرؾ حركة مستقيمة منتظمة‪.‬‬
‫‪‬‬
‫‪ .3‬بتطبيؽ القانكف الثاني لنيكتف عمى الصندكؽ في المعمـ )‪: (o,i,k‬‬
‫أ‪ .‬جد المعادلتيف الزمنيتيف لمحركة ) ‪ x (t‬ك ) ‪ . z (t‬ك استنتج طبيعة الحركة عمى كؿ محكر ‪.‬‬
‫‪‬‬
‫‪ - 1‬بتطبيؽ القانكف الثاني لنيكتف عمى الصندكؽ في المعمـ )‪: (o,i,k‬‬
‫‪0.25‬‬ ‫أ‪ /‬ايجاد المعادلتيف الزمنيتيف لمحركة )𝑡(𝑥 ك )𝑡(𝑧 ‪ .‬ك استنتاج طبيعة الحركة عمى كؿ محكر ‪.‬‬
‫‪‬‬ ‫‪‬‬ ‫‪‬‬ ‫‪‬‬
‫ك باإلسقاط عمى محكرم الحركة نجد‪:‬‬ ‫أم ‪P  m  a‬‬ ‫‪ F  ma‬‬
‫*باإلسقاط عمى المحكر ‪.  ox ‬‬
‫‪ 0  m  ax‬كمنو ‪ ax  0‬كلدينا‬
‫‪dvx‬‬
‫‪ax ‬‬ ‫‪0‬‬
‫‪0.25‬‬ ‫‪dt‬‬
‫‪ v x t  C te‬ت ع‪v x t   25 m / S :‬‬ ‫بالمكاممة بالنسبة لمزمف ‪ t‬نجد‬
‫‪0.25‬‬ ‫‪dx t ‬‬
‫‪v x t  ‬ك بالمكاممة بالنسبة لمزمف ‪ t‬نجد ‪x t   25.t  50 :‬‬‫كلدينا‬
‫‪dt‬‬
‫*باإلسقاط عمى المحكر)𝑧𝑜(‬
‫‪az  10m / S 2‬‬ ‫ك منو ‪ , az   g‬ت ع‬ ‫‪P  m.az‬‬ ‫لدينا‬
‫‪0.25‬‬ ‫) ‪dv z (t‬‬ ‫) ‪dv z (t‬‬
‫ك بالمكاممة بالنسبة لمزمف نجد ‪v z t   10.t :‬‬ ‫أم ‪ 10‬‬ ‫ك لدينا ‪ az‬‬
‫‪dt‬‬ ‫‪dt‬‬
‫‪dz t ‬‬ ‫‪dz t ‬‬
‫ك بالمكاممة بالنسبة لمزمف نجد ‪z t   5.t 2  20 :‬‬ ‫أم ‪ 10.t‬‬ ‫كلدينا ‪ v z‬‬
‫‪0.25‬‬ ‫‪dt‬‬ ‫‪dt‬‬
‫* استنتاج طبيعة الحركة عمى كؿ محكر‪:‬‬
‫عمى المحكر ‪ Ox ‬التسارع ‪ ax  0‬ك السرعة ‪ v x t   25m / S C‬كمنو الحركة منتظمة ‪.‬‬
‫‪te‬‬

‫‪0.25‬‬ ‫عمى المحكر ‪ az C  0 : Oz ‬ك ‪v z  0‬ك منو ‪ az .v z  0‬فالحركة متسارعة بانتظاـ ‪.‬‬
‫‪te‬‬

‫ب‪ .‬بيف أف معادلة مسار الحركة تكتب مف الشكؿ ‪z (x )  8.103.x 2  0,8x .:‬‬
‫‪50  x‬‬
‫‪ t ‬ك بالتعكيض في معادلة ‪ z t ‬نجد‬ ‫مف معادلة ‪ x t ‬لدينا‬
‫‪25‬‬
‫‪50  x 2‬‬
‫‪0.25‬‬ ‫أم ‪z  x   8.103.x 2  0,8x‬‬ ‫(‪z  x   5‬‬ ‫‪)  20‬‬
‫‪25‬‬
‫ج‪.‬حدد قيمة سرعة ارتطاـ الصندكؽ بسطح األرض عند النقطة ‪ O‬ك زاكية ميؿ شعاعيا ‪. α‬‬
‫عند لحظة االرتطاـ مع سطح األرض كمف البياف لدينا ‪v x  25m / S‬ك ‪v z  20m / S‬‬
‫‪0.25‬‬ ‫‪v  (25)2  (20)2‬‬ ‫ت ع ‪ 32m / S‬‬ ‫‪v  v x2  v z2‬‬ ‫ك منو‬
‫‪v‬‬ ‫‪25‬‬
‫ك منو ‪  38,7O‬‬ ‫‪tan   x ‬‬ ‫ك زاكية ميؿ شعاع السرعة ‪ 0,8‬‬
‫‪v z 20‬‬
‫الصفحة ‪ 5‬من ‪17‬‬
‫‪-‬‬
‫العالمة‬ ‫التمرين التجريبي‪ 06( :‬نقاط)‬ ‫‪-‬‬
‫‪ .1‬عند المحظة ‪ t  0‬نضع البادلة في الكضع(‪.)1‬‬
‫) ‪du R 1 (R1  r‬‬ ‫‪R .E‬‬
‫‪‬‬ ‫‪.u R 1  1‬‬ ‫أ‪ .‬بيف أف المعادلة التفاضمية لمتكتر الكيربائي ) ‪ u R1 (t‬ىي‪(1) :‬‬
‫‪dt‬‬ ‫‪L‬‬ ‫‪L‬‬
‫‪u‬‬ ‫) ‪di (t‬‬
‫‪0.25‬‬ ‫مف قانكف جمع التكترات ‪ ub  u R 1  E :‬ك لدينا ) ‪ u b t   L 1  ri 1 (t‬ك ‪i 1 t   R 1‬‬
‫‪R1‬‬ ‫‪dt‬‬
‫‪u‬‬
‫) ) ‪d ( R 1(t‬‬
‫‪0.25‬‬ ‫‪L‬‬ ‫بالتعكيض ‪, L di 1 (t )  ri t   u t   E‬كمنو ‪R 1  r u R 1 (t )  u t   E‬‬
‫‪R1‬‬ ‫‪1‬‬ ‫‪R1‬‬
‫‪dt‬‬ ‫‪R1‬‬ ‫‪dt‬‬
‫) ‪du R 1 (R1  r‬‬ ‫‪R .E‬‬
‫‪‬‬ ‫‪uR1  1‬‬ ‫كمنو )‪(1‬‬
‫‪dt‬‬ ‫‪L‬‬ ‫‪L‬‬
‫‪t‬‬
‫‪ u R 1 (t )  AB .(1  e‬ىي حال لممعادلة)‪ (1‬حيث ‪ A‬ك ‪ B‬ثابتاف يطمب تعيينيما‬ ‫‪A‬‬
‫ب‪ .‬تحقؽ مف أف العبارة ‪) :‬‬
‫‪0.25‬‬
‫‪t‬‬
‫‪du R 1‬‬ ‫‪‬‬
‫كبتعكيض كؿ مف عبارة الحؿ ك عبارة مشتؽ الحؿ في المعادلة التفاضمية نجد‪:‬‬ ‫لدينا ‪ Be‬‬
‫‪A‬‬
‫‪dt‬‬
‫‪ R1  r   A  R1  r‬‬ ‫‪ER ‬‬
‫‪t‬‬
‫‪0.25‬‬ ‫‪1 ‬‬ ‫‪A  Be  ‬‬ ‫‪AB  1   0‬‬
‫‪‬‬ ‫‪L‬‬ ‫‪‬‬ ‫‪ L‬‬ ‫‪L ‬‬
‫‪0.25‬‬
‫‪L‬‬ ‫‪R r‬‬
‫‪A‬‬ ‫‪ 1  1‬ك منو ‪ 1‬‬ ‫كمنو ‪A  0‬‬
‫‪R1  r‬‬ ‫‪L‬‬
‫‪0.25‬‬
‫‪ER1‬‬ ‫‪R1  r‬‬ ‫‪ER‬‬
‫‪B‬‬ ‫كمنو‬ ‫ك ‪AB  1  0‬‬
‫‪L‬‬ ‫‪L‬‬ ‫‪L‬‬
‫‪ .2‬بكاسطة جياز راسـ االىتزاز الميبطي ذك ذاكرة تمكنا مف مشاىدة البيانييف (أ) ك(ب) المكضحيف بالشكؿ (‪.)5‬‬
‫أ‪ .‬أنقؿ الدارة(‪)1‬عمى كرقة االجابة مكضحا عمييا كيفية‬
‫ربط جياز راسـ االىتزاز الميبطي لمشاىدة البيانيف(أ)‬
‫ك(ب) ‪.‬‬
‫‪0.5‬‬

‫الصفحة ‪ 6‬من ‪17‬‬


‫ب‪ .‬أنسب مع التعميؿ كؿ مف البيانيف (أ) ك (ب) لتطكر التكتر الكيربائي بيف طرفي العنصر الكيربائي المكافؽ‪.‬‬
‫‪0.25‬‬ ‫املحنى(أ)‪ً :‬ىافق جطىراث الخىجز بين طزفي املىلد (ًأخذ قيمت ثابخت ) ‪.‬‬
‫‪0.25‬‬ ‫املنحنى(ب)‪ً :‬ىافق جطىراث الخىجز بين طزفي الناقل ألاومي ‪ R1‬ألن شدة الخيار الكهزبائي جتزاًد نحى قيمت ثابخت‬
‫أعظميت و ‪ u R 1  R1i 1‬وبالخالي ‪ u R 1 t ‬جتزاًد نحى قيمت ثابخت أعظميت )‪. u R 1(max‬‬
‫ج‪ .‬استخرج قيمة ‪ E‬لممكلد ك شدة التيار الكيربائي األعظمية ‪ I 01‬ثـ أحسب قيمة مقاكمة الكشيعة ‪. r‬‬
‫‪0.25‬‬ ‫‪، E  6V‬‬ ‫* مف المنحنى(أ)‬
‫)‪u R 1(max‬‬‫‪5V‬‬
‫‪0.25‬‬ ‫‪، I 01 ‬‬ ‫‪‬‬ ‫* مف المنحنى(ب) ‪ 0,1A‬‬
‫‪R1‬‬ ‫‪50Ω‬‬
‫‪0.25‬‬ ‫‪r‬‬
‫‪E‬‬
‫‪ R1 ‬‬
‫‪6V‬‬
‫‪ I 01 ‬كمنو ‪ 50Ω  10Ω‬‬
‫‪E‬‬
‫* كلدينا‬
‫‪I 01‬‬ ‫‪0,1A‬‬ ‫‪R1  r‬‬
‫‪E .1‬‬
‫‪ L ‬ثـ أحسب قيمتيا ‪ .‬حيث ‪ 1‬ثابت الزمف ‪.‬‬ ‫د‪ .‬بيف أف ذاتية الكشيعة تعطى بالعبارة‬
‫‪I 01‬‬
‫‪0.25‬‬
‫‪ u R 1  1   0,63u R 1(max)  0,63  5V  3,15V‬و باإلسقاط على املنحنى(ب) نجد ‪. 1  10mS‬‬
‫‪0.25‬‬ ‫‪E 10  103 S  6V‬‬ ‫‪L‬‬
‫‪L  1  R1  r   1‬‬ ‫‪‬‬ ‫‪ 1 ‬كمنو ‪ 0,6H‬‬ ‫لدًنا‬
‫‪I 01‬‬ ‫‪0,1A‬‬ ‫‪R1  r‬‬
‫‪ .3‬أكتب عالقة ) ‪ u R 1 (t‬بداللة ‪ R1‬ك ) ‪. i 1 (t‬كاستنتج العبارة الحظية لشدة التيار الكيربائي ) ‪ i 1 (t‬المارة في الدارة‬
‫(‪)1‬‬
‫‪1‬‬
‫قانىن أوم‪ u R 1 t   R1  i 1 (t ) :‬ومنه ‪i 1 t   .u R 1 t ‬‬
‫‪0.25‬‬ ‫‪R1‬‬
‫‪E ‬‬ ‫‪t ‬‬ ‫‪1 ER1 ‬‬ ‫‪t ‬‬
‫‪, i 1 t  ‬‬ ‫‪‬‬ ‫‪1‬‬ ‫‪‬‬ ‫‪e‬‬ ‫‪1‬‬
‫‪‬‬ ‫كمنو‬ ‫‪i‬‬ ‫‪1 ‬‬
‫‪t‬‬ ‫‪‬‬ ‫‪.‬‬ ‫‪‬‬ ‫‪1‬‬ ‫‪‬‬ ‫‪e‬‬ ‫‪1‬‬
‫بالتعكيض نجد ‪‬‬
‫‪R1  r ‬‬ ‫‪‬‬ ‫‪R1 R1  r ‬‬ ‫‪‬‬
‫‪0.25‬‬
‫كمنو ‪i 1 t   I 01 1  e 1 ‬‬
‫‪t‬‬

‫‪‬‬ ‫‪‬‬
‫‪1‬‬
‫‪.t ‬‬ ‫‪ .4‬استنتج العبارة المحظية لمطاقة المخزنة في الكشيعة ‪ .‬ك أحسب قيمتيا عند المحظة‬
‫‪2‬‬
‫‪2‬‬
‫‪2 ‬‬ ‫‪τ1 ‬‬
‫‪1‬‬ ‫‪1‬‬ ‫‪t‬‬
‫‪E L t   .L .(i 1 t )  .L .I 01 1  e‬‬
‫‪2‬‬
‫لدينا ‪‬‬
‫‪0.25‬‬ ‫‪2‬‬ ‫‪2‬‬ ‫‪‬‬ ‫‪‬‬
‫‪2‬‬
‫‪‬‬ ‫‪1‬‬
‫‪‬‬
‫‪‬‬
‫‪ 1‬‬ ‫‪1‬‬ ‫‪2‬‬
‫‪‬‬ ‫‪2‬‬
‫‪1 ‬‬
‫‪E L    .0,6H .(0,1A ) 1  e‬‬ ‫ت ع ‪ 4,64.104 J‬‬
‫‪2 2‬‬ ‫‪‬‬ ‫‪‬‬
‫‪‬‬ ‫‪‬‬
‫‪ .5‬نضع البادلة في الكضع (‪ )2‬عند لحظة ‪ t  0‬نحصؿ عمى المنحنييف (ج) ك (د) المكضحيف بالشكؿ (‪)6‬‬
‫‪0.25‬‬ ‫‪.‬أ‪ .‬ما دكر الصماـ الثنائي )‪. (diode‬‬
‫هى جفادي خطز نشىء جىجز جحزيض ي مفزط بين طزفي الىشيعت عند الغياب املفاجئ للخيار الكهزبائي للمىلد ‪.‬‬

‫الصفحة ‪ 7‬من ‪17‬‬


‫ب‪ .‬حدد مف بيف المنحنييف (ج) ك (د) ما ىك المنحنى المكافؽ لػمدارة (‪ ، )3‬مع التعميؿ ‪.‬‬
‫‪0.25‬‬ ‫املنحنى (د)‪:‬هى الذي ًىافق الدارة(‪ )3‬ألن عند الغياب املفاجئ لخيار املىلد جىلد الىشيعت جيارا ًدعى بالخيار‬
‫جحزيض ي له نفس اججاه جيار املىلد و جدناقص شدجه جزيجا من قيمت أعظميت (حساوي شدة الخيار الغائب‬
‫للمىلد ) ‪ ( I 01  0,1A  100mA‬الى الصفز ‪.‬‬
‫ج‪ .‬جد قيمة ثابت الزمف ‪ 3‬الخاص بالدارة (‪ )3‬ثـ أحسب قيمة مقاكمة الناقؿ األكمي ‪. R 3‬‬
‫‪0.25‬‬ ‫ك باإلسقاط نجد ‪3  5mS‬‬ ‫من املنحنى (د) ‪i 3  3   0,37 100mA  37mA :‬‬

‫‪0.25‬‬ ‫‪R3 ‬‬


‫‪L‬‬
‫‪ R1  r ‬‬
‫‪0,6H‬‬
‫‪ 50Ω 10Ω  60Ω‬‬ ‫كمنو‬ ‫‪3 ‬‬
‫‪L‬‬
‫لدًنا‬
‫‪3‬‬ ‫‪5  103 S‬‬ ‫‪R1  R 3  r‬‬
‫د‪ .‬جد قيمة ثابت الزمف ‪ 2‬الخاص بالدارة (‪ )2‬ثـ أحسب قيمة ‪ R 2‬ك ‪. C‬‬
‫من املنحنى (ج) ‪ i 2  2   0,37 10mA  3,7mA :‬ك باإلسقاط نجد ‪2  0,003S  3mS‬‬
‫‪0.25‬‬
‫‪E‬‬ ‫‪6V‬‬ ‫‪E‬‬
‫‪R2 ‬‬ ‫‪‬‬ ‫‪ I 02 ‬كمنو ‪ 600Ω‬‬ ‫و لدًنا‬
‫‪I 02 0,01A‬‬ ‫‪R2‬‬
‫‪0.25‬‬ ‫‪2 3  103 S‬‬
‫‪C ‬‬ ‫‪‬‬ ‫كمنو ‪ 5.106 F  5F‬‬ ‫و لدًنا ‪2  R 2  C‬‬
‫‪R2‬‬ ‫‪600Ω‬‬

‫الصفحة ‪ 8‬من ‪17‬‬


‫عناصر اإلجابة ( الموضوع الثاني)‬

‫العالمة‬ ‫التمرين ‪ 06( 01‬نقاط)‪:‬‬


‫‪ .1‬تحديد قيمة التسارع خالل كل مرحمة من مراحل الحركة ‪ :‬التسارع ‪ a‬يمثل ميل منحنى السرعة ومنو‪:‬‬
‫‪v 4,8  4,8‬‬
‫‪0.25‬‬ ‫‪a1 ‬‬ ‫‪‬‬ ‫‪ 0m / S 2‬‬ ‫‪ .‬في المرحمة األولى(المسار ‪:) AB‬‬
‫‪t‬‬ ‫‪0,5  0‬‬
‫‪v‬‬ ‫‪0  4,8‬‬
‫‪0.25‬‬ ‫‪a2 ‬‬ ‫‪‬‬ ‫‪ .‬في المرحمة الثانية(المسار ‪ 6, 4m / S 2 : ) BN‬‬
‫)‪t (1, 25  0,5‬‬
‫‪v‬‬ ‫‪3,6  0‬‬
‫‪0.25‬‬ ‫‪a3 ‬‬ ‫‪‬‬ ‫‪ .‬في المرحمة الثالثة (المسار ‪ 3,6m / S 2 : ) NB‬‬
‫)‪t (2, 25  1, 25‬‬
‫تديد طبيعة الحركة مع التعميل ‪.‬‬
‫* ح‬
‫‪ .‬في المرحمة األولى(المسار ‪:) AB‬‬
‫‪0.25‬‬
‫‪.‬المسار مستقيم و السرعة ثابتة ‪v  0‬و التسارع معدوم ‪ a1  0‬ومنو الحركة مستقيمة منتظمة ‪.‬‬
‫‪ .‬في المرحمة الثانية(المسار ‪: ) BN‬‬
‫‪0.25‬‬ ‫‪.‬المسار مستقيم و السرعة موجبة ‪v  0‬و التسارع ‪ a2‬موجب )‪ (a2  0‬و ثابت ‪a2  C te‬‬
‫ومنو الجداء ‪ a2 v  0‬ومنو الحركة مستقيمة متسارعة بانتظام‬
‫‪ .‬في المرحمة الثالثة (المسار ‪: ) NB‬‬
‫‪0.25‬‬
‫‪.‬المسار مستقيم و السرعة سالبة ‪v  0‬و التسارع ‪ a3‬سالب )‪ (a3  0‬و ثابت ‪a3  C te‬‬
‫ومنو الجداء ‪ a3v  0‬ومنو الحركة مستقيمة متسارعة بانتظام‬
‫‪ .2‬دراسة حركة الجسم )‪ (s‬عمى المسار ‪: AB‬‬
‫أ‪ .‬تمثيل القوى المؤثرة عمى الجسم )‪. (s‬‬
‫‪0.5‬‬

‫ب‪ .‬بتطبيق القانون الثاني لنيوتن عمى الجممة (جسم )‪ , )(s‬ايجاد عالقة بين ‪ F‬و ‪ f‬و ‪. ‬‬
‫بتطبيق القانون الثاني لنيوتن في مرجع سطحي ارضي (يعتبر غاليميا) المزود بالمحور الموجو ) ‪: (x x‬‬
‫‪   ‬‬ ‫‪‬‬ ‫‪‬‬ ‫‪‬‬
‫‪0.25‬‬ ‫‪  F ext  m  a1‬أي ‪F  P  R  f  m a1 :‬‬
‫‪ F cos  f  m a1‬وبما أن ‪a1  0 :‬‬ ‫باإلسقاط عمى محور الدراسة الموجو ) ‪ (x x‬نجد‪:‬‬
‫‪0.25‬‬ ‫)‪F cos   f  0 ........(1‬‬ ‫فان ‪:‬‬

‫الصفحة ‪ 9‬من ‪17‬‬


‫‪ .3‬دراسة حركة الجسم )‪ (s‬عمى المسار ‪: BN‬‬
‫أ‪ .‬تمثيل القوى المؤثرة عمى الجسم )‪. (s‬‬
‫‪0.5‬‬

‫ب‪ .‬بتطبيق القانون الثاني لنيوتن عمى الجممة (جسم )‪ , )(s‬جد عالقة بين ‪ f ,  , g , m‬و التسارع ‪. a2‬‬
‫بتطبيق القانون الثاني لنيوتن في مرجع سطحي ارضي (يعتبر غاليميا) المزود بالمحور الموجو ) ‪: (x x‬‬
‫‪0.25‬‬ ‫‪  ‬‬ ‫‪‬‬ ‫‪‬‬ ‫‪‬‬
‫‪P  R  f  m a2‬‬ ‫‪  F ext  m  a 2‬أي ‪:‬‬
‫‪P  m.g‬‬ ‫‪ PSin  f  m a2‬وبما أن ‪:‬‬ ‫باإلسقاط عمى محور الدراسة الموجو ) ‪ (x x‬نجد‪:‬‬
‫‪0.25‬‬
‫)‪m.g .Sin  f  m.a2 ........(2‬‬ ‫فان ‪:‬‬
‫‪ .4‬دراسة حركة الجسم )‪ (s‬عمى المسار ‪: NB‬‬
‫أ‪ .‬تمثيل القوى المؤثرة عمى الجسم )‪. (s‬‬
‫‪0.5‬‬

‫ب‪ .‬بتطبيق القانون الثاني لنيوتن عمى الجممة (جسم)‪ , )(s‬جد عالقة بين ‪ f ,  , g , m‬و تسارع ‪. a3‬‬
‫بتطبيق القانون الثاني لنيوتن في مرجع سطحي ارضي (يعتبر غاليميا) المزود بالمحور الموجو ) ‪: (x x‬‬
‫‪  ‬‬ ‫‪‬‬ ‫‪‬‬ ‫‪‬‬
‫‪0.25‬‬ ‫‪  F ext  m  a 3‬أي ‪P  R  f  m a3 :‬‬
‫‪ PSin  f  m a3‬وبما أن ‪P  m.g :‬‬ ‫باإلسقاط عمى محور الدراسة الموجو ) ‪ (x x‬نجد‪:‬‬
‫‪0.25‬‬ ‫)‪m.g .Sin  f  m. a3 ........(3‬‬ ‫فان ‪:‬‬
‫‪ .5‬باستخدام العالقات السابقة تحديد ما يمي‪:‬‬
‫أ‪ .‬زاوية الميل ‪ ‬لممستوي ‪. BC‬‬
‫بجمع العالقتين (‪ )2‬و(‪ )3‬طرفا الى طرف نجد‪:‬‬
‫‪0.25‬‬ ‫أي‪2.g .Sin  a2  a3 :‬‬ ‫‪m.g .Sin  f‬‬ ‫‪ m.g .Sin  f  m.a2  m. a3‬‬

‫‪a2  a3‬‬ ‫)‪6, 4  (3,6‬‬


‫‪0.25‬‬ ‫‪ Sin   ‬ومنو ‪  300 :‬‬ ‫‪‬‬ ‫ومنو ‪ 0,5 :‬‬
‫‪2.g‬‬ ‫‪2.10‬‬
‫‪‬‬
‫ب‪ .‬شدة قوة االحتكاك ‪. f‬‬
‫بطرح العالقة (‪ )2‬من (‪ )3‬طرفا الى طرف نجد‪:‬‬
‫‪0.25‬‬ ‫‪2.f‬‬ ‫أي‪ m (a3  a2 ) :‬‬ ‫‪m.g.Sin  f  (m.g .Sin  f )  m. a3  m.a2‬‬

‫‪0.25‬‬ ‫‪f‬‬ ‫‪‬‬


‫))‪m (a3  a2 ) 0, 2(3,6  (6, 4‬‬
‫‪‬‬ ‫‪ 0, 28N‬‬ ‫ومنو‪:‬‬
‫‪2‬‬ ‫‪2‬‬

‫الصفحة ‪ 10‬من ‪17‬‬


‫‪‬‬
‫ج‪ .‬شدة القوة المحركة ‪: F‬‬
‫‪0.5‬‬ ‫‪F‬‬
‫‪f‬‬
‫‪‬‬
‫‪0, 28‬‬
‫نجد ‪ 0,56N‬‬ ‫من العالقة (‪: )1‬‬
‫‪cos  Cos 60‬‬
‫العالمة‬ ‫حل التمرين ‪ 04(2‬نقاط)‪:‬‬
‫‪ .1‬بين أن تركيز المحمول التجاري ىو ‪.C 0 15mol / L‬‬
‫‪10  P  d‬‬
‫‪, C0 ‬‬ ‫لدينا تركيز المحمول التجاري يعطى بالعالقة‬
‫‪M‬‬
‫‪0.25‬‬
‫و ‪M (NH 3 ) 14g / mol  31g / mol 17g / mol‬‬
‫‪10  28  0,91‬‬
‫‪C0 ‬‬ ‫‪ 14.98mol / L  15mol / L‬‬ ‫ت ع‪:‬‬
‫‪17‬‬
‫‪ .2‬حضرنا محموال ) ‪ (S‬حجمو ‪ 3L‬و تركيزه المولي ‪C  0,01mol / L‬انطالقا من المحمول التجاري ‪.‬‬
‫‪ ‬اذكر البروتوكول التجريبي لتحضير المحمول ) ‪ (S‬مع ذكر الزجاجيات المستعممة ‪.‬‬
‫*تنظيف الزجاجيات و الوسائل االزمة‪.‬‬
‫‪0, 01mol / L  3L‬‬
‫‪ V 0 ‬من‬ ‫*نأخذ بواسطة ماصة عيارية عينة حجميا ‪ 2.103 L  2mL‬‬
‫‪CV‬‬
‫‪‬‬
‫‪C0‬‬ ‫‪15mol / L‬‬
‫المحمول التجاري المركز‬
‫‪0.25‬‬
‫*نضع العينة في حوجمة عيارية سعتيا ‪ 3L‬بيا قميل من الماء المقطر مع الرج ‪.‬‬
‫* نضيف الماء المقطر حتى خط العيار مع الرج ايضا ‪.‬‬
‫قمي النوعية لممحمول ) ‪ (S‬أعطى القيمة ‪.  f 10,9mS / m‬‬
‫‪ .3‬قياس النا ة‬

‫‪0.25‬‬ ‫أ‪ .‬لماذا تمدد المحاليل قبل قياس الناقمية؟‪.‬‬


‫ألن قياس الناقمية يكون اكثر دقة في المحاليل المائية الشاردية الممددة (المخففة) ‪.‬‬
‫ب‪ .‬كتابة معادلة انحالل النشادر ‪ NH 3‬في الماء و أنشاء جدول التقدم ليذا التفاعل ‪.‬‬
‫‪NH 3‬‬ ‫) ‪(aq‬‬ ‫‪ H 2O(l )  NH 4‬‬ ‫) ‪(aq‬‬ ‫* معادلة التفاعل ‪ OH (aq ) :‬‬
‫* جدول التقدم ‪:‬‬
‫‪0.25‬‬
‫معادلة‬ ‫‪NH 3‬‬ ‫) ‪(aq‬‬ ‫‪‬‬ ‫‪H 2O(l ) ‬‬ ‫‪NH 4‬‬ ‫) ‪(aq‬‬ ‫) ‪ OH (aq‬‬
‫الحاالت‬ ‫التقدم‬ ‫كميات المادة بالمول ‪mol‬‬
‫ح‪ .‬ابتدائية‬ ‫‪0‬‬ ‫‪CV‬‬ ‫بوفرة‬ ‫‪0‬‬ ‫‪0‬‬
‫ح‪ .‬لحظية‬ ‫‪x‬‬ ‫‪CV  x‬‬ ‫بوفرة‬ ‫‪x‬‬ ‫‪x‬‬
‫ح‪ .‬نيائية‬ ‫‪x f  x max‬‬ ‫‪CV  x f‬‬ ‫بوفرة‬ ‫‪xf‬‬ ‫‪xf‬‬

‫ج ‪ .‬أكتب عبارة التركيز ‪  HO  f‬بداللة ‪ , HO , NH‬و ‪ .  f‬ثم أحسب قيمتو ‪.‬‬
‫‪‬‬
‫‪‬‬ ‫‪‬‬
‫‪4‬‬

‫لدينا عبارة الناقمية النوعية ‪ f  OH   f .OH  NH 4  f .NH :‬‬


‫‪‬‬ ‫‪‬‬
‫‪4‬‬

‫‪0.25‬‬ ‫‪ ,  NH 4   f‬ومنو ‪OH     NH 4 ‬‬


‫‪x‬‬
‫و‬ ‫و من جدول التقدم لدينا ‪OH    f :‬‬
‫‪x‬‬
‫‪f‬‬ ‫‪f‬‬ ‫‪f‬‬ ‫‪V‬‬ ‫‪f‬‬ ‫‪V‬‬
‫‪OH   ‬‬
‫‪f‬‬
‫‪‬‬
‫ومنو تصبح عبارة ‪  f  OH   f . OH   NH 4 : f‬ومنو نجد‬ ‫‪‬‬
‫‪0.25‬‬
‫‪f‬‬
‫‪‬‬
‫‪OH   NH ‬‬
‫‪4‬‬
‫‪‬‬
‫الصفحة ‪ 11‬من ‪17‬‬
10,9.10 3 S / m
. OH  f   0, 4mol / m 3  4.10 4 mol / L : ‫ت ع‬
19,9  7,35 .10 S .m / mol
3 2

0.25

. (S ) ‫ المحمول‬pH ‫ ثم استناتج قيمة‬.‫د‬


pH   log H 3O   :‫لدينا‬
f

, Ke   H 3O   . OH   ‫لدينا من عالقة الجداء الشاردي لمماء‬


f f
 pKe
10 Ke
0.5  H 3O    ‫ومنو‬ , 
 H O 
 f OH   ‫ومنو‬
 
OH  
f 3
f   f
14
10
 H 3O    4
 2,5.1011 mol / L :‫ت ع‬
f 4.10
pH  log(2,5.1011 ) 10,6 : ‫ت ع‬
10 pH  pKe
. ‫ ثم أحسب قيمتيا‬, f  : ‫ تعطى بالعالقة‬ f ‫ بين أن نسبة التقدم النيائي‬.‫ه‬
C
: ‫لدينا‬
xf
f 
x max

‫ ومنو‬OH   f  :‫ومن جدول التقدم لدينا‬


xf
x f  OH   V
f V
x max  CV :‫ومنو‬ CV  x max  0 :‫و بفرض أن التفاعل تام نجد‬
OH   OH   V
0.5 f  f
:‫ومنو‬ f  f
:‫ نجد‬f ‫وبالتعويض في عالقة‬
C CV
Ke 10 pKe
OH     pH
 10 pH pKe
: ‫ولدينا من الجداء الشاردي لمماء‬
f  H 3O 

10
f

10 pH  pKe
‫ وىو المطموب‬ f  :‫ األخيرة نجد‬ f ‫و بالتعويض في عبارة‬
C
1010,614
0.25 .‫ أساسضعيف‬NH 3 ‫ نستنتج أن‬ f   3,98.102  3,98%  4% :‫ ت ع‬ f ‫* حساب قيمة‬
0, 01

pKa  pH  log
 NH 3 f ‫( تبيان أن‬NH 4 / NH 3 ) ‫ لمثنائية‬Ka ‫ كتابة عبارة ثابت الحموضة‬.‫و‬
 NH 
4f

 NH 3 f . H 3O   f
. Ka  :‫لدينا‬
0.25  NH 4 
f

:‫ عمى طرفي المعادلة نجد‬log ‫و بإدخال الدالة‬


 NH 3 f ‫ أي‬log Ka  log  H 3O    log
 NH 3 f
 log  H 3O     log Ka  log
f  NH 4  f  NH 4 
f f

 log  H 3O   pH ‫و‬

,  log Ka  pKa ‫و بما ان‬
f

17 ‫ من‬12 ‫الصفحة‬
‫‪0.25‬‬
‫‪pKa  pH  log‬‬
‫‪ NH 3 f‬‬ ‫ومنو تصبح العالقة‬
‫‪ NH 4 ‬‬
‫‪f‬‬

‫* ثم حساب قيمة ال ـ ـ ‪ pKa‬لمثنائية ) ‪. (NH 4 / NH 3‬‬

‫‪, pKa  pH  log‬‬


‫‪ NH 3 f‬‬ ‫لدينا ‪:‬‬
‫‪ NH 4 ‬‬
‫‪f‬‬

‫ومنو ‪NH 4   4.104 mol / L‬‬ ‫و ‪NH 4   OH  ‬‬


‫‪f‬‬ ‫‪f‬‬ ‫‪f‬‬
‫‪0.5‬‬ ‫‪CV  x f‬‬ ‫‪x‬‬
‫‪ NH 3 f‬‬ ‫‪‬‬ ‫و ‪ C  f  C  NH 4 ‬‬
‫‪V‬‬ ‫‪V‬‬ ‫‪f‬‬

‫‪NH 3 f‬‬ ‫ومنو ‪ 0,01  4.104  9,6.103 mol / L :‬‬


‫‪9,6.103‬‬
‫‪. pKa  10,6  log‬‬ ‫‪ 9,2‬‬ ‫ت ع‪:‬‬
‫‪4.104‬‬

‫حل التمرين ‪ 04( 3‬نقاط)‪:‬‬

‫‪ -1‬أ‪-‬تحديد جهة تطور المجموعة الكيميائية المكونة للعمود‪:‬‬


‫𝑖‪[𝐴𝑙 3+ ]2‬‬ ‫‪𝐶02‬‬ ‫‪1‬‬ ‫‪1‬‬
‫‪0.25‬‬ ‫= 𝑖𝑟𝑄‬ ‫=‬ ‫=‬ ‫=‬ ‫‪= 20‬‬
‫𝑖‪[𝐶𝑢2+ ]3‬‬ ‫‪𝐶03 𝐶0 0,05‬‬
‫‪0.25‬‬ ‫𝐾 < 𝑖𝑟𝑄 تتطور المجموعة الكيميائية المكونة للعمود في االتجاه المباشر‪.‬‬
‫ب‪ -‬تحديد قطبي العمود‪:‬‬
‫‪2+‬‬
‫)𝑞𝑎(𝑢𝐶‬ ‫عند مسرى النحاس يحدث تفاعل إرجاع حسب المعادلة النصفية‪+ 2𝑒 − = 𝐶𝑢(𝑠) :‬‬
‫‪0.25‬‬
‫𝟓𝟕 ‪𝟏.‬‬
‫‪3+‬‬
‫)𝑞𝑎(𝑙𝐴 = )𝑠(𝑙𝐴‬ ‫عند مسرى األلمنيوم يحدث تفاعل أكسدة حسب المعادلة النصفية‪+ 3𝑒 − :‬‬
‫‪0.25‬‬ ‫إذن مسرى النحاس هو القطب الموجب ومسرى األلمنيوم هو القطب السالب‪.‬‬
‫‪0.25‬‬ ‫الرمز االصطالحي للعمود المدروس )‪(−) 𝐴𝑙/𝐴𝑙 3+ //𝐶𝑢2+ /𝐶𝑢 (+‬‬
‫𝐷‬ ‫𝐾‬
‫‪A‬‬ ‫ج‪ -‬رسم العمود وتبيان عليه جهة التيار‬
‫‪−‬‬
‫𝒆‬
‫‪𝐶𝑙 −‬‬ ‫‪𝑁𝐻4+‬‬
‫𝑰‬
‫الكهربائي وجهة حامالت الشحن‪.‬‬
‫𝑙𝐴‬ ‫𝑢𝐶‬
‫جسرملحي‬
‫‪2 × 0.25‬‬

‫محلول كلور‬ ‫محلول كبريتات‬


‫االلمنيوم‬ ‫النحاس الثنائي‬

‫الصفحة ‪ 13‬من ‪17‬‬


‫‪ -2‬أ‪-‬جدول تقدم التفاعل‪.‬‬
‫المعادلة‬
‫‪2+‬‬ ‫‪3+‬‬
‫)𝑞𝑎(𝑢𝐶‪3‬‬ ‫‪+‬‬ ‫)𝑠(𝑙𝐴‪2‬‬ ‫)𝑠(𝑢𝐶‪= 3‬‬ ‫‪+‬‬ ‫)𝑞𝑎(𝑙𝐴‪2‬‬
‫‪0.50‬‬ ‫‪𝐶0 . 𝑉0‬‬ ‫𝑙𝐴𝑛‬ ‫𝑢𝐶𝑛‬ ‫‪𝐶0 . 𝑉0‬‬
‫‪𝑡=0‬‬
‫𝑡‬ ‫𝑥‪𝐶0 . 𝑉0 − 3‬‬ ‫𝑥‪𝑛𝐴𝑙 − 2‬‬ ‫𝑥‪𝑛𝐶𝑢 + 3‬‬ ‫𝑥‪𝐶0 . 𝑉0 + 2‬‬

‫𝑓𝑡‬ ‫𝑥𝑎𝑚𝑥‪𝐶0 . 𝑉0 − 3‬‬ ‫𝑥𝑎𝑚𝑥‪𝑛𝐴𝑙 − 2‬‬ ‫𝑥𝑎𝑚𝑥‪𝑛𝐶𝑢 + 3‬‬ ‫𝑥𝑎𝑚𝑥‪𝐶0 . 𝑉0 + 2‬‬

‫ب‪-‬عبارة ] ‪[𝑪𝒖𝟐+‬‬
‫من جدول التقدم‪:‬‬
‫𝑥‪𝐶0 .𝑉0 −3‬‬ ‫𝑥‪3‬‬
‫‪0.25‬‬ ‫= ] ‪[𝐶𝑢2+‬‬ ‫‪= 𝐶0 −‬‬
‫‪𝑉0‬‬ ‫‪𝑉0‬‬

‫نعلم أن 𝑡‬
‫𝐼‬
‫= 𝑥 ⇒ 𝐹 ‪𝑞 = 𝐼. 𝑡 = 𝑧. 𝑥.‬‬
‫𝐹‪𝑧.‬‬
‫حيث𝑧 هو عدد اإللكترونات المتبادلة خالل كل تفاعل وهي ‪7‬‬
‫𝐼 ‪3‬‬ ‫𝐼‬
‫‪0.25‬‬ ‫‪[𝐶𝑢2+ ] = 𝐶0 − .‬‬ ‫‪𝑡 ⇒ [𝐶𝑢2+ ] = −‬‬ ‫‪𝑡 + 𝐶0‬‬
‫𝐹 ‪𝑉0 7.‬‬ ‫𝐹 ‪2𝑉0 .‬‬
‫ج‪ -‬قيمة شدة التيار𝑰 المار في الدارة‬
‫𝟓𝟕 ‪𝟏.‬‬
‫معادلة البيان‪= −2. 10−5 𝑚𝑚𝑜𝑙. 𝐿−1 . 𝑠 −1 ، [𝐶𝑢2+ ] = 𝑎. 𝑡 + 𝑏 :‬‬
‫‪0,05−0,04‬‬
‫‪0.25‬‬ ‫=𝑎‬
‫‪0−500‬‬
‫بالمطابقة مع العالقة السابقة نجد‪= 𝑎 ⇒ 𝐼 = −2. 𝑎𝑉0 . 𝐹 :‬‬
‫𝐼‬
‫‪0.25‬‬ ‫‪−‬‬
‫𝐹‪2𝑉0 .‬‬
‫‪0.25‬‬ ‫𝐴‪𝐼 = −2. (−2. 10−5 )50. 10−3 . 97500 = 0,193‬‬

‫‪ -3‬عبارة التغير𝒎∆‪:‬‬
‫من جدول التقدم‪= 2𝑥𝑚𝑎𝑥 ⇒ ∆𝑚 = 2. 𝑀𝑥𝑚𝑎𝑥 :‬‬
‫𝑚∆‬
‫‪0.25‬‬ ‫= 𝑙𝐴𝑛 ⇒ ‪𝑛𝐴𝑙 − 2𝑥𝑚𝑎𝑥 = 0‬‬
‫𝑀‬
‫𝟎𝟓 ‪𝟎.‬‬ ‫= 𝑥𝑎𝑚𝑥‬
‫𝐼‬
‫𝑡‬ ‫من جهة أخرى عند اللحظة ‪𝑡c‬‬
‫‪𝑧.𝐹 c‬‬
‫𝐼‬ ‫‪0,193‬‬
‫‪0.25‬‬ ‫‪∆𝑚 = 2. 𝑀.‬‬ ‫‪𝑡c ⇒ ∆𝑚 = 2.26.‬‬ ‫‪2500 = 0,045g‬‬
‫𝐹 ‪𝑧.‬‬ ‫‪7.97500‬‬
‫‪-‬‬
‫العالمة‬ ‫‪ -‬ح التمرين التجريبي‪ 06( :‬نقاط)‬
‫كفؽ‬ ‫‪238‬‬
‫‪ -I‬تعتمد بعض المحركات عمى بطارية نككية تكلد طاقة متحررة عف النشاط االشعاعي لمبمكتكنيكـ ‪P‬‬
‫‪94 u‬‬

‫نمط التفكؾ ‪ ‬معطيا نكاة اليكرانيكـ ‪. ZAU‬‬


‫‪0.25‬‬ ‫‪ .1‬عرؼ النظائر‪.‬‬
‫هي أنىييت لنفس العنصز لها نفس العدد الذري ‪ Z‬وجخخلف في العدد الكخلي ‪( A‬جخخلف في عدد نتروناتها ‪.)N‬‬
‫‪ 238‬كفؽ النمط ‪ ، ‬مع تحديد كؿ مف ‪ A‬ك ‪. Z‬‬ ‫‪ .2‬أكتب معادلة التفكؾ النككم لنظير البمكتكنيكـ ‪94 Pu‬‬

‫‪0.25‬‬ ‫‪P  ZAU  24He‬‬


‫‪238‬‬
‫‪94 u‬‬

‫بخطبيق قانىني الانحفاظلصىدي ‪:‬‬


‫‪ 238  A  4  A  234‬ك ‪94  A  2  Z  92‬‬
‫‪ 238‬كتحديد عدد األنكية االبتدائية ‪ N 0‬المحتكاة في عينة‬
‫‪ -II‬مف أجؿ تعييف زمف نصؼ العمر لمبمكتكنيكـ ‪94 Pu‬‬

‫‪ .1‬أكتب قانكف التناقص االشعاعي ك استنتج عبارة ) ‪ A (t‬بداللة عدد أنكيةالبمكتكنيكـ المتبقية) ‪. N (t‬‬
‫‪0.25‬‬ ‫) ‪dN (t‬‬
‫‪ A t  ‬كمنو ‪A t   .N 0 .e t‬‬ ‫لدينا ‪ N t   N 0 .e t‬ك‬
‫‪dt‬‬
‫كمنو ) ‪A t   .N (t‬‬

‫الصفحة ‪ 14‬من ‪17‬‬


‫‪ .2‬أكتب العالقة بيف ) ‪ N (t‬ك ‪. N U‬‬
‫‪0.25‬‬ ‫‪N t   N 0  N U‬‬
‫‪ .3‬أكتب العالقة بيف ) ‪ A (t‬ك ‪ N U‬ك ثابت النشاط االشعاعي ‪. ‬‬
‫‪0.25‬‬ ‫لدًنا ‪ N t     N 0  N U ‬ومنه ‪A t   N U  N 0 :‬‬
‫‪ .4‬اعتمادا عمى منحنى ) ‪ A  f (N U‬ك عالقة السؤاؿ‪ . 3‬جد ما يمي ‪N 0 ( Pu ) ، t 1/2 ،  ، A0 :‬‬
‫‪0.25‬‬ ‫معادلت البيان هي ‪ A t   a.N U  b :‬حيث ‪ a  tan  2,5.1010 S 1 :‬و ‪b  6.1010 noy .S 1‬‬
‫وباملطابقت بين معادلت البيان و العالقت النظزيت ألاخيرة نجد ‪:‬‬
‫‪ A0  N 0  b‬كمنو ‪A0  6.1010 Bq‬‬
‫‪0.25‬‬ ‫‪   a‬كمنو ‪  2,5.1010 S 1‬‬
‫‪ln 2‬‬
‫‪t1 ‬‬ ‫‪ 2,77.109 S  8,78ans‬‬
‫‪2‬‬ ‫‪‬‬
‫‪A0‬‬
‫‪ N 0  Pu  ‬أو من البيان ‪N 0  Pu   N Uf  2,4.1020 noy‬‬ ‫‪ 2,4.1020 noy‬‬
‫‪0.25‬‬ ‫‪‬‬

‫‪-III‬‬
‫‪ .1‬عرؼ االنشطار النككم ك لماذا نقكؿ عنو أنو تفاعؿ تسمسميمغذل ذاتيا‪.‬‬
‫مستقر) بنتركف بطيء لتنشطر الى نكاتيف أخؼ ك‬
‫ة‬ ‫‪ ‬ىك تفاعؿ نككم مفتعؿ يتـ فيو قذؼ نكاة ثقيمة (غير‬
‫‪0.25‬‬
‫أكثر استقرار ك تحرير طاقة معتبرة كنتركنات ذات طاقة حركية مكركسككبية ‪.‬‬

‫‪0.25‬‬
‫‪ ‬نقكؿ عنو أنو متسمسؿ كمغذل ذاتيا ألف النتركنات المتحررة تممؾ طاقة حركية ميكركسككبية (سرعة) ك‬
‫باصطداميا مع أنكيةالبمكتكنيكـ أخرل تحدث انشطارات أخرل كىكذا دكالؾ ‪.‬‬

‫‪ .2‬باالعتماد عمى مخطط الحصيمة الطاقكية ‪:‬‬


‫أ‪ .‬حدد تركيب نكاة البمكتكنيكـ الخاضعة لالنشطار النككم‪.‬‬
‫‪Z  94‬‬ ‫‪,‬‬ ‫مف مخطط الحصيمة ك بعد تفكؾ نكات البمكتكمنيكـ نجد‪N  146  1  145 :‬‬
‫‪0.25‬‬
‫ب‪ .‬أكتب معادلة تفاعؿ االنشطار النككم لمبمكتكنيكـ ‪ ،‬مع تحديد كؿ مف‪ Z ‬ك ‪. x‬‬
‫‪P  01n ‬‬
‫‪239‬‬
‫‪94 u‬‬ ‫‪T e  102‬‬
‫‪135‬‬
‫‪Z‬‬ ‫‪42 Mo  x 0 n‬‬
‫‪1‬‬

‫* تحديد كؿ مف ‪ Z ‬ك ‪. x‬‬


‫بتطبيؽ قانكني صكدم لدينا ‪:‬‬
‫‪0.25‬‬ ‫كمنو ‪x  3‬‬ ‫انحفاظ الكتمة (العدد الكتمي)‪239 1135 102  x :‬‬
‫‪0.25‬‬ ‫كمنو ‪Z   52‬‬ ‫انحفاظ الشحنة (انحفاظ العدد الشحني)‪94  0  Z  42  0 :‬‬

‫الصفحة ‪ 15‬من ‪17‬‬


‫‪ .3‬ماذا تمثؿ المقادير ‪ – . E 3 ، E 2 ، E 1‬أحسب قيمة ‪. E 2‬‬
‫* ‪ : E 1‬تمثؿ الطاقة الكتميةالسككنية لمنكاتج ‪.‬‬
‫‪0.25‬‬ ‫* ‪ : E 2‬تمثؿ الطاقة الكتميةالسككنية لممتفاعالت‬
‫* ‪ : E 3‬تمثؿ الطاقة الكتميةلنكميكنات المتفاعالت ك ىي منفردة ك ساكنة ‪.‬‬

‫* حساب قيمة ‪. E 2‬‬


‫‪0.25‬‬ ‫‪E 2  m  239‬‬
‫‪94 Pu   m ( 0 n) C   239,00060  1,00866   931,5  2,23568.10 Mev‬‬
‫‪1‬‬
‫‪ 2‬‬ ‫‪5‬‬

‫‪ .4‬استنتاج طاقة ربط نكاة البمكتكنيكـ ‪. Z Pu‬‬


‫‪A‬‬

‫‪0.25‬‬ ‫‪E l  239‬‬


‫مف مخطط الحصيمة الطاقكية لدينا ‪94 Pu   E 3  E 2 :‬‬

‫‪E l  239‬‬
‫ت ع‪94 Pu   (2,25375  2,23568).10  1807Mev :‬‬
‫‪5‬‬

‫‪ .‬مع التعميؿ‪.‬‬ ‫‪P ،‬‬ ‫‪Mo ،‬‬ ‫‪ .5‬حدد النكاة األكثر استقرار مف بيف األنكية‪T :‬‬
‫‪A‬‬ ‫‪102‬‬ ‫‪135‬‬
‫‪Z u‬‬ ‫‪42‬‬ ‫‪Z e‬‬

‫* حساب طاقة الربط لكؿ نكميكف في كؿ نكاة ‪:‬‬

‫‪A‬‬
‫‪P ‬‬
‫‪E l 1807Mev‬‬
‫‪0.25‬‬ ‫‪El‬‬ ‫‪239‬‬
‫‪94 u‬‬ ‫‪‬‬ ‫‪ 7,56Mev / nuc‬‬ ‫في نكاة البمكتكنيكـ ‪:‬‬
‫‪A‬‬ ‫‪239nuc‬‬
‫‪0.25‬‬ ‫‪El‬‬
‫‪( 102‬‬
‫‪E‬‬
‫‪Mo )  l ‬‬
‫‪873,773‬‬
‫في نكاة المكليبدنكـ ‪ 8,57Mev / nuc :‬‬
‫‪A‬‬ ‫‪42‬‬
‫‪A‬‬ ‫‪102‬‬
‫‪El‬‬
‫في نكاة التيمكريكـ ‪( 135Te )  8,48Mev / nuc :‬‬
‫‪A 52‬‬
‫‪0.25‬‬
‫‪El‬‬
‫أكبر ‪.‬‬ ‫ألنيا ذات طاقة ربط لكؿ نكميكف‬ ‫كمنو النكاة األكثر استقرار ىي نكاة ‪Mo‬‬
‫‪102‬‬
‫‪42‬‬
‫‪A‬‬

‫بكحدة ‪ Mev‬ثـ ب ػ ‪. joule‬‬ ‫‪239‬‬


‫‪94 u‬‬ ‫‪ .6‬أحسب قيمة الطاقة المحررة مف انشطار نكاة كاحدة مف البمكتكنيكـ ‪P‬‬
‫مف مخطط الحصيمة الطاقكية لدينا ‪E lib  E 2  E 1 :‬‬
‫‪0.25‬‬ ‫‪E lib  (2,23568  2,23370).105 198Mev‬‬ ‫ت ع‪:‬‬ ‫* بكحدة ‪Mev‬‬
‫‪E lib  198 1,6.1013  3,168.1011 joule‬‬ ‫ت ع‪:‬‬ ‫* بكحدة ‪joule‬‬

‫الصفحة ‪ 16‬من ‪17‬‬


‫‪ .7‬يستيمؾ ىذا المفاعؿ النككم كتمة قدرىا ‪ 97, 45Kg‬مف البمكتكنيكـ ‪ ZA Pu‬خالؿ مدة مف الزمف قدرىا ‪ t‬في‬
‫انتاج الكيرباء باستطاعة قيمتيا ‪ Pe  900MW‬ك بمردكد ‪. 30%‬‬
‫أ‪ .‬جد المدة ‪ t‬الشتغاؿ المفاعؿ النككم ‪.‬‬
‫‪Ee‬‬
‫‪0.25‬‬ ‫كمنو )‪E e  Pe  t .....(1‬‬ ‫‪Pe ‬‬ ‫لدينا ‪:‬‬
‫‪t‬‬
‫‪Ee‬‬
‫ك بتعكيض عبارة الطاقة الكيربائية (‪ )1‬في عبارة المردكد‬ ‫‪r% ‬‬ ‫بمردكد‪ 100 :‬‬
‫) ‪Elib (tot‬‬
‫‪Pe  t‬‬
‫‪r% ‬‬ ‫)‪ 100 .....(2‬‬ ‫نجد‪:‬‬
‫‪0.25‬‬ ‫) ‪Elib (tot‬‬
‫‪m‬‬
‫‪N ‬‬ ‫‪.N A‬‬ ‫ك‬ ‫المحرر‪E lib tot   N .E lib :‬‬
‫ة‬ ‫ك الطاقة االجمالية‬
‫‪M‬‬
‫‪m‬‬
‫‪E lib tot  ‬‬ ‫كمنو )‪.N A .E lib ....(3‬‬
‫‪0.25‬‬ ‫‪M‬‬
‫ك بتعكيض عبارة الطاقة المحررة الكمية (‪ )3‬في عبارة مردكد التحكيؿ الطاقكم (‪ )2‬نجد‪:‬‬
‫‪m .N A .E lib r %‬‬ ‫‪Pe  t‬‬
‫‪t ‬‬ ‫كمنو‪:‬‬ ‫‪r% ‬‬ ‫‪ 100‬‬
‫‪100.M . Pe‬‬ ‫‪m‬‬
‫‪.N A .E lib‬‬
‫‪0.25‬‬ ‫‪M‬‬
‫ت ع‪:‬‬
‫‪97,45.103  6,02.1023  3,168.1011  70‬‬
‫‪t ‬‬ ‫‪ 2,592.106 S  720h  30J  1moi‬‬
‫‪100  239  900.10‬‬ ‫‪6‬‬

‫مدة اشتغاؿ المفاعؿ النككم ىي شير كاحد‬

‫الصفحة ‪ 17‬من ‪17‬‬


‫اﻟﺘﺎرﻳــﺦ ‪2023/05/18 :‬‬ ‫ﻣﺪﻳﺮﻳــﺔ اﻟﱰﺑﻴــﺔ ﻟﻮﻻﻳﺔ ﺗﺒﺴــﺔ‬
‫د‬
‫اﳌـ ـ ـ ـ ــﺪة‪ 04 :‬ﺳﺎ‪30 ،‬‬ ‫اﻣﺘﺤ ـ ـ ــﺎن اﻟﺒﻜﺎﻟ ـ ــﻮر‪ $‬اﻟﺘﺠﺮﻳﺒـ ــﻲ ﻣــﺎي ‪2023‬‬ ‫اﳌﺴﺘﻮى‪ :‬اﻟﺜﺎﻟﺜـﺔ )ر‪$‬ﺿﻲ ‪ +‬ﺗﻘﲏ ر‪$‬ﺿﻲ(‬
‫اﳌﻘﺎﻃﻌـ ـﺔ اﻟﺘﻔﺘﻴﺸﻴﺔ ‪ :‬ﺗﺒﺴ ـ ــﺔ ‪02‬‬ ‫اﺧﺘﺒـ ــﺎر ﰲ ﻣـ ـ ــﺎدة ‪ :‬اﻟﻌﻠ ـ ـ ـ ــﻮم اﻟﻔﻴﺰ‪$‬ﺋﻴ ـ ـ ــﺔ‬

‫ﻋﺎﻟﺞ ﻣ ﺿ ﻋﺎ واﺣ ا ﻓﻘ ﻋﻠﻰ اﻟ ﺎر ‪:‬‬


‫اﻟ ﺿـــــ ع اﻷول‬
‫اﻟ ـــــ ء اﻷول ‪ :‬ﯾ ـــ ن ﻣـــ‪ $‬ﺛﻼﺛـــﺔ ﺗ ﺎر‪%‬ـــ‪. $‬‬
‫اﻟ )‪%‬ــــــ‪ $‬اﻷول ‪ 04.00 ) :‬ﻧﻘﺎ‪( .‬‬
‫ﻓ ﻧ ﺎ ﺎﻟﻠ ن اﻟ ﺗﻘﺎﻟﻲ إﺛ‬ ‫ﻣﺎ‬ ‫ﺑ ا ﺔ ﺷﻬ ﻓ اﯾ ﺳ‪%‬ﺎء وﺛﻠ ج ﻌ‬ ‫" ﺗﻠ ﻧ‬
‫ﻋﻠﻰ ﻋ‪ 4‬ﺔ ﻣ‪ 3‬اﻟ ﻣﻞ‬ ‫ﻋﺎﺻﻔﺔ رﻣﻠ'ﺔ‪ ،‬ﻟ‪ - .‬ﺟ‪'0%‬ﺔَ ‪ ARCO‬ﻧ‪.‬ﺎﺋﺞ دراﺳﺔ أﺟ ‪6‬‬
‫أن اﻟ>=ﺎر واﻟ ﻣﻞ‬ ‫ﯾ م ‪ 6‬ﻓ اﯾ ‪< % 2021‬ﻘﺔ ﺟ ار ‪ Jura‬اﻟﻔ ﻧ 'ﺔ ﺗ?‬ ‫ﺟ‪%‬ﻌ‬
‫‪6 .A‬ﺎن ﻋﻠﻰ اﻟ ‪ 6@4‬م ‪ 137‬اﻟ‪-%‬ﻊ‪ ،‬وأن اﻟ ‪-‬ﺎ‪ C‬اﻹﺷﻌﺎﻋﻲ اﻟ ﺎﺗﺞ ﻋ‪ 3‬ﻫ‪E‬ا اﻟ ‪4D‬‬
‫ﻘ ر ﺑـ ‪ ، 80kbq / km‬ﻟ‪ .4‬ﺎءل ُﻣ َ‪ A‬ر اﻟ راﺳﺔ ﻋ‪%'K 3‬ﺔ ﻫ‪E‬ا اﻟ ‪-‬ﺎ‪ C‬ﺎﻟ‪ %‬ﺎ‬
‫‪2‬‬

‫و‪ 6‬ﱠ‪E‬ﻛ اﻟﻔ ﻧ ‪ 344‬ﺑـ ‪ 13‬ﺗ‪S N‬ﺔ‬


‫اﻟ‪.‬ﻲ َوَردت ﻣ ﻬﺎ ﻫ‪E‬ﻩ اﻟ ﻣﺎل أ‪ P‬اﻟ‪ AO‬اء اﻟ‪@N‬اﺋ ‪6‬ﺔ ُ‬
‫ﻧ و‪6‬ﺔ ﻗﺎﻣ ﺑﻬﺎ ﻓ ﻧ ﺎ ﺳ ﺔ ‪ 1960‬ﻓﻲ ﻋ‪ %‬اﻷرض ﺑـإن إ‪ )01‬و‪ 3‬ﺗ‪N‬ﺎرب ﻋﻠﻰ ﺳ<ﺢ اﻷرض ﺑـرﻗﺎن "‬
‫ﻣﻘ ‪ 9:‬ﺑ ‪)7‬ف ﻋ‪ $‬ﺟ)‪ %‬ة ‪ Midi Libre‬ﺑ‪.‬ﺎر‪6‬ﺦ ‪28 / 02 / 2021‬‬
‫ﯾﻬ ف ﻫ=ا اﻟ )‪ $%‬إﻟﻰ دراﺳﺔ ﺗﻔ @ اﻟ‪ % CD‬م ‪ 137‬وﺗﻔﺎﻋﻞ اﻧ‪FG‬ﺎر اﻟ‪E‬ﻠ ﺗ ﻧ‪ C‬م ‪239‬‬
‫‪ -I‬اﻟ‪ % CD‬م ‪: 137‬‬
‫ﯾ‪ @4%.‬ﻋ ‪ O‬اﻟ ‪ 6@4‬م ‪ Cs‬ﺎﻟﻌ د اﻟ‪E‬ر‪ 55 P‬وﻟﻪ ‪ 31‬ﻧ‪ 4D‬ﯾ‪ .‬اوح ﻋ دﻫﺎ اﻟ[‪.‬ﻠﻲ ﻣ‪ 114 3‬إﻟﻰ ‪. 145‬‬
‫اﻟ ‪ 6@4‬م ‪ 133‬ﻧ‪ُ 4D‬ﻣ ‪.‬ﻘ أﻣﺎ اﻟ ‪ 6@4‬م ‪ 137‬ﻓ‪.4‬ﻔ[^ ﺑ@ﻣ‪ 3‬ﻧ‪O‬ﻒ ﻋ‪ %‬ﻗ رﻩ ‪ t1/2  30 ans‬ﻣﻌ<'ﺎ ﻧ اة‬
‫‪137‬‬
‫وﺟ '‪ X a‬و‪ 6‬اﻓ ذﻟ^ إﺷﻌﺎع ﺿ ﺋﻲ‪.‬‬ ‫‪56‬‬ ‫اﻟ=ﺎر‪ 6‬م ‪Ba‬‬
‫‪ -1‬ﻋ ف ‪h‬ﻞ ﻣ‪ :3‬اﻟﻌ د اﻟ‪E‬ر‪ ، P‬ﻧ‪ 4D‬ﻣ ‪.‬ﻘ ‪.‬‬
‫‪ -2‬ﻋﻠﻞ ﻣ ‪.‬ﻌ‪ 4‬ﺎ ‪%‬ﻌﺎدﻟﺔ ﺗﻔﺎﻋﻞ إﺻ ار إﺷﻌﺎع ﺿ ﺋﻲ ﻋ ﺗﻔ[^ اﻟ ‪ 6@4‬م ‪. 137‬‬
‫‪ -3‬أﻛ‪ l.‬ﻣﻌﺎدﻟﺔ ﺗﻔ[^ اﻟ ‪ 6@4‬م ‪ 137‬ﻣ ‪ 4‬ﺎ اﻟ ‪ n%‬اﻹﺷﻌﺎﻋﻲ اﻟ‪A‬ﺎدث‪.‬‬
‫‪ -4‬أﺣ ‪ l‬ﻋ د أﻧ ‪6‬ﺔ اﻟ ‪ 6@4‬م ‪ 137‬ﻓﻲ ‪h‬ﻞ ‪4h‬ﻠ ﻣ‪ .‬ﻣ ‪S‬ﻊ واﺣ ﻣ‪ 3‬ﻣ‪r‬ﺎن أﺧ‪ E‬اﻟﻌ‪ 4‬ﺔ اﻟ‪ %‬روﺳﺔ‪.‬‬
‫‪ -II‬اﻟ‪E‬ﻠ ﺗ ﻧ‪ C‬م ‪: 239‬‬

‫) ‪E (MeV‬‬ ‫ﻓﻲ ﺗ‪N‬ﺎرب رﻗﺎن‪ ،‬واﻟ‪PE‬‬


‫‪239‬‬
‫‪94‬‬ ‫اﺳ‪ s.‬م اﻟﻔ ﻧ ‪ 4‬ن اﻟ ﻠ ﺗ ﻧ‪ 4‬م ‪Pu‬‬
‫‪94 p  146 n‬‬
‫‪E2‬‬ ‫ﺑ ‪ .4‬ون‪ ،‬ﻓ‪ A.‬ث ﺳﻠ ﻠﺔ اﻟ‪.‬ﻔﺎﻋﻼت‬ ‫‪238‬‬
‫‪29‬‬ ‫‪ <O‬ﻊ ﻘ‪E‬ف ﻧ اة اﻟ‪ 4‬راﻧ‪ 4‬م ‪U‬‬
‫اﻟ‪.‬ﺎﻟ'ﺔ‪92U  93 Np  1 e  94 Pu  1 e :‬‬
‫‪U  01n  239‬‬
‫‪238‬‬ ‫‪239‬‬ ‫‪0‬‬ ‫‪239‬‬ ‫‪0‬‬
‫‪E1‬‬ ‫‪92‬‬

‫‪Pu  n‬‬ ‫‪E 2‬‬ ‫‪ -1‬ﻫﻞ ﺗ‪ A‬ل اﻟ‪ 4‬راﻧ‪ 4‬م ‪ 238‬إﻟﻰ اﻟ ﻠ ﺗ ﻧ‪ 4‬م ‪ 239‬ﻫ‬
‫‪239‬‬
‫‪94‬‬
‫‪E1‬‬
‫‪E‬‬ ‫ﺗﻔﺎﻋﻞ اﻧ‪<-‬ﺎر؟ ﻋﻠﻞ‪.‬‬
‫‪134‬‬
‫‪Xe  103‬‬
‫‪40 Zr  x.n‬‬
‫‪E3‬‬ ‫‪54‬‬
‫‪ -2‬ﻋ ف ﺎﻗﺔ اﻟ ‪ nS‬ﻟﻠ اة ‪. Eℓ‬‬
‫ﺍﻟﺸﻜﻞ‪-01-‬‬

‫ﺻﻔﺤﺔ ‪ 1‬ﻣﻦ ‪11‬‬


‫ﻋ ﻗ‪E‬ﻓﻬﺎ ﺑ ‪ .4‬ون وﻓ ﻣﺎ ﯾ ﺿﺢ ﻣ‪ n<s‬اﻟ‪4OA‬ﻠﺔ اﻟ<ﺎﻗ ‪6‬ﺔ ﻓﻲ اﻟ‪r-‬ﻞ ‪01-‬‬ ‫‪ -3‬ﺗ ‪ <-‬ﻧ اة ‪Pu‬‬
‫‪239‬‬
‫‪94‬‬

‫أ‪ -‬أﺣ ‪ l‬اﻟ‪%‬ﻘﺎدﯾ ‪ E1 :‬و ‪ E2‬و ‪. E3‬‬


‫ب‪ -‬اﺳ‪. .‬ﺞ ‪%'K‬ﺔ اﻟ‪%‬ﻘﺎدﯾ ‪ E1:‬و ‪. E 2‬‬
‫ﺟـ‪ -‬ﺣ د ‪%'K‬ﺔ اﻟ<ﺎﻗﺔ اﻟ‪ A%‬رة ﻋ‪ 3‬ﺗﻔﺎﻋﻞ اﻧ‪<-‬ﺎر اﻟ ﻠ ﺗ ﻧ‪ 4‬م ‪.239‬‬
‫‪ -III‬اﻟ‪ L)C‬ع اﻷزرق ‪: Gerboise bleue‬‬
‫‪ ( 3‬ﻣ‪ 3‬ﻣ‪.‬ﻔ‪ N‬ات اﻟـ ‪. TNT‬‬ ‫َﺣ ﱠ رت ﺗﻔ‪ Gerboise bleue 4N‬ﺑ ﻗﺎن ﺎﻗﺔ ﺗﻌﺎدل اﻧﻔ‪N‬ﺎر ‪4h 70 ) 70kT‬ﻠ‬
‫‪ -‬أﺣ ‪.h l‬ﻠﺔ اﻟ ﻠ ﺗ ﻧ‪ 4‬م ‪ 239‬اﻟ‪ s. %‬ﻣﺔ ﻓﻲ اﻟ‪.‬ﻔ‪ 4N‬ﻋﻠ‪%‬ﺎ أن ‪. 1kT (T N T )  4 , 1 8 4  10 J‬‬
‫‪12‬‬

‫‪239‬‬ ‫‪134‬‬ ‫‪103‬‬ ‫‪1‬‬ ‫‪1‬‬


‫‪1‬ﻌ‪F‬ﻰ ‪:‬‬
‫‪94‬‬ ‫‪Pu‬‬ ‫‪54‬‬ ‫‪Xe‬‬ ‫‪40‬‬ ‫‪Zr‬‬ ‫‪0‬‬‫‪n‬‬ ‫‪1‬‬ ‫‪p‬‬
‫‪u ‬‬ ‫‪239, 05276‬‬ ‫‪133, 90539‬‬ ‫‪102, 92720‬‬ ‫‪1, 00866‬‬ ‫‪1, 00 72 8‬‬

‫‪8‬‬ ‫‪23‬‬ ‫‪1‬‬


‫‪C  3  10 m / s; N A  6, 022  10 mol‬‬
‫‪1MeV  1, 6  10 13 J ;1u  931, 5MeV / C 2‬‬

‫اﻟ )‪ $%‬اﻟ‪Q‬ﺎﻧﻲ‪ 4 ):‬ﻧﻘﺎ‪(.‬‬


‫اﻹ <ﺎﻟﻲ ﻏﺎﻟ‪4‬ﻠﻲ ﻓﻲ دراﺳ‪.‬ﻪ ﻟ ﻘ ‪ C‬اﻷﺟ ﺎم ﻣ‪r-‬ﻠﺔ ‪'K‬ﺎس ﻣ ة‬ ‫اﻋ‪ .‬ﺿ‬
‫اﻟ ﻘ ‪ C‬ﺑ ﻗﺔ‪ ،‬ﻓﺎﺗ‪N‬ﻪ إﻟﻰ دراﺳﺔ ﺣ ‪h‬ﺔ اﻷﺟ ﺎم ﻋﻠﻰ ﻣ ‪ P .‬ﻣﺎﺋﻞ‪ ،‬ﻓ[ﺎﻧ‬
‫ﺗﻠ^ ﺑ ا ﺔ اﻟﻔ‪6@4‬ﺎء اﻟ‪' 6 N.‬ﺔ‪ ،‬وﺳﺎﻋ ت ﻓﻲ ﺗ‪%‬ﻬ‪ 4‬اﻟ< ‪ 6‬ﻟ ‪6 D‬ﺎت إﺳ‪A‬ﺎق‬
‫ﻧ‪ 4‬ﺗ‪ 3‬ﻋ‪ 3‬اﻟ‪r'%‬ﺎﻧ'^ واﻟ‪N‬ﺎذﺑ'ﺔ‪.‬‬
‫‪ -‬ﯾﻬ ف اﻟ )‪ $%‬إﻟﻰ دراﺳﺔ ﺣ)‪X‬ﺔ ﺟ‪ WD‬ﺻﻠ‪ U‬ﻋﻠﻰ ﻣ‪ T D‬ﻣﺎﺋﻞ‬
‫ﻣ ‪ P .‬ﻣﺎﺋﻞ اﺳ‪ s.‬ﻣﻪ ﻏﺎﻟ‪4‬ﻠﻲ‬
‫ﻧﻌ‪ .‬ﻩ ﻧﻘ<ﻲ ‪.h‬ﻠ‪.‬ﻪ ‪m‬‬ ‫) ‪(S‬‬ ‫ﻓﻲ اﻟﻠ‪DA‬ﺔ ‪ ، t  0‬ﯾ @ﻟ ﺟ ‪ a‬ﺻﻠ‪l‬‬
‫دون ﺳ ﻋﺔ اﺑ‪ .‬اﺋ'ﺔ و‪S‬ﺎﺣ‪[.‬ﺎﻛﺎت ﻣﻬ‪%‬ﻠﺔ ﻋﻠﻰ ﻣ ﺎر ﻣ ‪ a'•.‬ﻣ‪ 3‬اﻟ‪ %‬ﺿﻊ ‪ O‬أﻋﻠﻰ اﻟ‪ P . %‬اﻟ‪%‬ﺎﺋﻞ ‪ AB‬اﻟ‪4% PE‬ﻞ‬
‫ﻓﻲ اﻟ‪%‬ﻌﻠ‪  O, x, y  a‬اﻟ‪ PE‬ﻧﻌ‪ .‬ﻩ ﻋ<ﺎﻟ'ﺎ‪.‬‬ ‫‪(S ) a‬‬ ‫ﺎﻟ@او‪6‬ﺔ ‪ ، ‬ﻧ رس ﺣ ‪h‬ﺔ اﻟ‪N‬‬ ‫ﻋﻠﻰ اﻷﻓ‬
‫‪ -I‬اﻟ راﺳﺔ اﻟ ‪ 0%)Y‬ﺔ ‪:‬‬
‫‪.‬‬ ‫) ‪(S‬‬ ‫‪ -2‬ﻣ?ﻞ اﻟﻘ • اﻟ‪=<%‬ﻘﺔ ﻋﻠﻰ اﻟ‪a N‬‬ ‫‪ %‬أ إﻧ‪A‬ﻔﺎ‪ €‬اﻟ<ﺎﻗﺔ‪.‬‬ ‫‪ -1‬ذ‪h‬‬
‫ﺑ‪ 34‬اﻟ‪ %‬ﺿﻊ ‪ O‬وﻣ ﺿﻊ ‪'h‬ﻔﻲ ‪ M‬ﻋ أ‪ P‬ﻟ‪DA‬ﺔ ‪َ ،t‬ﺑِّ‪ 34‬أن‬ ‫) ‪(S‬‬ ‫‪ -3‬ﺑ‪ 4 <.‬ﻣ أ إﻧ‪A‬ﻔﺎ‪ €‬اﻟ<ﺎﻗﺔ ﻋﻠﻰ اﻟ‪a N‬‬
‫…=ﺎرة ﺗ ﺎرع اﻟ‪h A‬ﺔ ﺗ[‪ l.‬ﺑـ ـ ـ‪a  g sin  :‬‬
‫‪.‬‬ ‫) ‪(S‬‬ ‫'ﻌﺔ ﺣ ‪h‬ﺔ اﻟ‪a N‬‬ ‫‪ -4‬اﺳ‪. .‬ﺞ‬
‫‪ -II‬اﻟ راﺳﺔ اﻟ‪ X)Y‬ﺔ‪:‬‬
‫‪ x‬‬ ‫‪ vx ‬‬
‫‪ -1‬ﻋ ﺑ ﻻﻟﺔ اﻟ@ﻣ‪ 3‬ﻋ‪ 3‬ﻣ ‪=h‬ﺎت ﺷﻌﺎع اﻟ ﻋﺔ ‪ v  ‬وﻣ ‪=h‬ﺎت ﺷﻌﺎع اﻟ‪ %‬ﺿﻊ ‪. OM  ‬‬
‫‪ y‬‬ ‫‪ vy ‬‬
‫‪ -2‬ﻋ اﻧ‪.‬ﻘﺎل اﻟ‪ a N‬ﻋﻠﻰ اﻟ‪ A%‬ر ‪ O x‬ﺎﻟ‪ %‬ﺎﻓﺔ ‪ ، x  OM‬ﯾﻬ=‪ n‬ﺎﻻرﺗﻔﺎع ‪ h‬ﺎﻟ =ﺔ ﻟﻠ‪ P . %‬اﻷﻓﻘﻲ اﻟ‪%‬ﺎر‬
‫ﻣ‪ 3‬اﻟ‪ %‬ﺿﻊ ‪. O‬‬

‫ﺻﻔﺤﺔ ‪ 2‬ﻣﻦ ‪11‬‬


‫)‪vx (m / s‬‬
‫‪1‬‬
‫أ‪ -‬ﺑ‪ 34‬أن‪h   g   sin    t 2 :‬‬
‫‪2‬‬

‫‪a‬‬ ‫‪2‬‬
‫ب‪ -‬اﺣ ‪ l‬اﻟ@ﻣ‪ 3‬اﻟ‪. %‬ﻐ ق ﻟﻘ<ﻊ اﻻرﺗﻔﺎع ‪ h  1m‬ﻣ‪3‬‬
‫أﺟﻞ اﻟ@او‪6‬ﺔ ‪.   4 5 ‬‬
‫‪ -III‬زاو‪%‬ﺔ اﻟ ‪C‬ﻞ‪:‬‬
‫‪%‬ﻌﺎﻟ‪N‬ﺔ ﻣﻌﻠ ﻣﺎﺗ'ﺔ ﻟ‪%‬ﻌ<'ﺎت اﻟ‪h A‬ﺔ‪ ،‬ﺣ‪O‬ﻠ ﺎ ﻋﻠﻰ ﺑ'ﺎﻧﻲ ﻟ‪ <.‬ر‬
‫‪b‬‬ ‫اﻟ@ﻣ ﻲ ﻟﻠ ﻋﺔ )‪ vx  f (t‬اﻟ‪r-‬ﻞ‪ 02-‬وﻋﻠﻰ ﺑ'ﺎﻧﻲ اﻟ‪ <.‬ر اﻟ@ﻣ ﻲ‬
‫‪2,5‬‬ ‫ﻣ‪ 3‬أﺟﻞ اﻟ@او‪ 30 34.6‬و ‪ 45‬ﻓﻲ اﻟ‪r-‬ﻞ‪03-‬‬ ‫ﻟﻼرﺗﻔﺎع‬
‫) ‪h  f (t‬‬

‫‪0‬‬ ‫‪ -1‬ﻣ ‪.‬ﻌ‪ 4‬ﺎ ﺎﻟ راﺳﺔ اﻟ‪'h A‬ﺔ‪:‬‬


‫‪0, 4‬‬ ‫ﺍﻟﺸﻜﻞ‪02-‬‬ ‫)‪t (s‬‬

‫وﻋﻠﻰ‬ ‫ﻋﺔ ‪v x‬‬ ‫أ‪ -‬ﻣﺎ ﺗﺄﺛ‪ 4‬زاو‪6‬ﺔ ﻣ‪4‬ﻞ اﻟ‪ P . %‬اﻟ‪%‬ﺎﺋﻞ ‪ ‬ﻋﻠﻰ اﻟ‬
‫) ‪h( m‬‬ ‫اﻻرﺗﻔﺎع ‪ h‬؟‬
‫ب‪ -‬أرﻓ اﻟ‪' A %‬ﺎت اﻟ 'ﺎﻧ'ﺔ ‪  c  ،  b  ،  a ‬و ‪  d ‬ﻣﻊ زاو‪6‬ﺔ اﻟ‪4%‬ﻞ‬
‫‪c ‬‬
‫اﻟ‪ %‬اﻓﻘﺔ ﻟﻬﺎ‪.‬‬
‫‪ -2‬ﺟ …=ﺎرة اﻟ ﻋﺔ ‪ vx‬ﺑ ﻻﻟﺔ اﻻرﺗﻔﺎع ‪. h‬‬
‫‪ ‬ﻋﻠﻰ ﻋﻼﻗﺔ‬ ‫‪ -3‬ﻗ‪'O a‬ﺎﻏﺔ ﻧ‪N'.‬ﺔ ﺗ ﺿﺢ ﻓ‪4‬ﻬﺎ ﺗﺄﺛ‪ 4‬زاو‪6‬ﺔ اﻟ‪4%‬ﻞ‬
‫‪d ‬‬ ‫‪vx‬‬
‫ﺎﻻرﺗﻔﺎع ‪. h‬‬ ‫اﻟ ﻋﺔ‬
‫‪1‬‬ ‫اﻟ )‪%‬ــــــ‪ $‬اﻟ‪Q‬ﺎﻟــــ‪ 06.00 ) : Z‬ﻧﻘﺎ‪( .‬‬
‫اﻟ‪?r%‬ﻔﺎت ﻣ‪ 3‬اﻟﻌ ﺎﺻ اﻟ[ﻬ ‪S‬ﺎﺋ'ﺔ اﻟﻬﺎﻣﺔ ﻓﻲ ﺻ ﺎﻋﺔ اﻟ ارات اﻻﻟ[‪ .‬وﻧ'ﺔ وﻓﻲ‬
‫‪0‬‬
‫)‪t (s‬‬
‫‪0‬‬ ‫‪0, 4‬‬
‫ﺍﻟﺸﻜﻞ ‪03-‬‬ ‫اﻧ‪N‬ﺎز اﻟﻌ ﯾ ﻣ‪ 3‬دواﺋ اﻟﻘ • اﻟ[ﻬ ‪S‬ﺎﺋ'ﺔ ﻣ?ﻞ ﺑ ء ﺣ ‪h‬ﺔ اﻟ‪h A%‬ﺎت وﺗ‪34 A‬‬
‫ﻣﻌﺎﻣﻞ اﻻﺳ‪<.‬ﺎﻋﺔ‪.‬‬
‫‪ -‬ﯾﻬ ف ﻫ=ا اﻟ )‪ $%‬إﻟﻰ دراﺳﺔ ﺛ\ﺎﺋﻲ اﻟﻘ‪ RC UF‬وﺗ‪ Y‬ﯾ اﻟ‪ Q‬اﺑ] اﻟ ‪ C‬ة ﻟﻠ ارة‪.‬‬
‫ﺗ‪ [.‬ن دارة ‪h‬ﻬ ‪S‬ﺎﺋ'ﺔ ﻣ‪ 3‬ﻣ ﻟ ﻣ?ﺎﻟﻲ ﻟﻠ‪ .‬ﺗ ﻗ ﺗﻪ اﻟ‪h A%‬ﺔ اﻟ[ﻬ ‪S‬ﺎﺋ'ﺔ ‪ E‬وﻧﺎﻗﻞ أوﻣﻲ ﻣﻘﺎوﻣ‪.‬ﻪ ‪ R‬وﻣ‪?r‬ﻔﺔ ﻣﻔ ﻏﺔ‬
‫ﺳﻌ‪.‬ﻬﺎ ‪ C‬و‪S‬ﺎدﻟﺔ ‪%h K‬ﺎ ﻫ ﻣ ﺿﺢ ﺎﻟ‪r-‬ﻞ‪. 4-‬‬
‫‪1‬‬ ‫‪2‬‬
‫‪K‬‬
‫ﻋ اﻟﻠ‪DA‬ﺔ ‪ t  0‬ﺗ‪Š‬ﻊ اﻟ=ﺎدﻟﺔ ﻋﻠﻰ اﻟ ﺿﻊ ‪. 1 ‬‬

‫‪R‬‬ ‫‪ -I‬اﻟ^ﺎﻫ)ة اﻟﻔ‪% C‬ﺎﺋ ﺔ ‪:‬‬


‫‪E‬‬ ‫‪ -2‬ﻣﺎذا ‪ A‬ث ﻋﻠﻰ ﻣ ‪ • .‬ﻟ ﺳﻲ اﻟ‪?r%‬ﻔﺔ؟ اﺷ ح‪.‬‬ ‫‪ -1‬ﻋ ف اﻟ‪?r%‬ﻔﺔ‪.‬‬
‫‪C‬‬
‫‪ -II‬اﻟ ﺗ) اﻟ ﻬ)‪L‬ﺎﺋﻲ ﺑ‪)_ $C‬ﻓﻲ اﻟ ‪Q0‬ﻔﺔ‪:‬‬
‫ﻣ‪ r‬ﺎ ﺗ‪N‬ﻬ‪ ExAO @4‬ﻣ‪ 3‬ﻣ‪.‬ﺎ ﻌﺔ ﺗﻐ‪ 4‬ات اﻟ‪ .‬ﺗ ﺑ‪ 34‬ﻓﻲ اﻟ‪?r%‬ﻔﺔ ‪uC‬‬
‫ﺍﻟﺸﻜﻞ‪4-‬‬ ‫) ‪duC (t‬‬
‫ﻓﻲ اﻟ‪r-‬ﻞ‪5-‬‬ ‫ﺑ ﻻﻟﺔ اﻟ@ﻣ‪ t 3‬واﻟ‪ OA‬ل ﻋﻠﻰ اﻟ‪ A %‬ﻰ ) ‪ f (t‬‬
‫‪dt‬‬
‫ﻓﻲ اﻟ‪?r%‬ﻔﺔ ‪. uC‬‬ ‫‪ -1‬أﻋ رﺳ‪ a‬اﻟ ارة ﻣ ‪ 4‬ﺎ ‪'Œ'h‬ﺔ وﺻﻞ ﻻﻗ‪ n‬ﺟﻬﺎز ‪ ExAO‬ﻟ‪.%‬ﺎ ﻌﺔ اﻟ‪ .‬ﺗ ﺑ‪34‬‬
‫) ‪duC (t‬‬
‫‪ -2‬ﺑ‪ 4 <.‬ﻗﺎﻧ ن ﺟ‪%‬ﻊ اﻟ‪ .‬ﺗ ات ﺑ‪ 34‬أن ‪ A.u C (t )  A.B :‬‬
‫‪dt‬‬
‫ﺻﻔﺤﺔ ‪ 3‬ﻣﻦ ‪11‬‬
‫) ‪du C ( t‬‬
‫) ‪(V / s‬‬
‫‪dt‬‬ ‫<ﻠ‪ l‬ﻣ ^ اﻟ‪.‬ﻌ ‪ 4‬ﻋﻠ‪4‬ﻬﺎ ﺑ ﻻﻟﺔ‬ ‫ﺣ‪ A •4‬و ‪ B‬ﺛ اﺑ‬
‫ﻣ‪@4%‬ات اﻟ ارة وﺗ‪ A‬ﯾ اﻟ‪ %‬ﻟ ل اﻟﻔ‪6@4‬ﺎﺋﻲ ﻟ[ﻞ ﻣ ﻬ‪%‬ﺎ‪.‬‬
‫‪ -3‬ﺗ‪A‬ﻘ أن ﺣﻞ اﻟ‪%‬ﻌﺎدﻟﺔ اﻟ‪.‬ﻔﺎﺿﻠ'ﺔ ﻫ ‪:‬‬
‫) ‪u C ( t )  B (1  e  A t‬‬
‫اﻟ@ﻣ‪  3‬ﻟﻠ ارة‪.‬‬ ‫‪ -4‬ﺣ د ‪%'K‬ﺔ ﺛﺎﺑ‬
‫‪ -5‬اﺳ‪. .‬ﺞ ‪%'K‬ﺔ ‪ E‬اﻟﻘ ة اﻟ‪h A%‬ﺔ اﻟ[ﻬ ‪S‬ﺎﺋ'ﺔ ﻟﻠ‪ %‬ﻟ ‪.‬‬
‫‪30‬‬ ‫‪ -III‬اﻟ‪F‬ﺎﻗﺔ ‪:‬‬
‫‪0‬‬ ‫‪40‬‬ ‫'ﻌﺔ اﻟ<ﺎﻗﺔ اﻟ‪@s%‬ﻧﺔ ﺎﻟ‪?r%‬ﻔﺔ‪.‬‬ ‫‪ -1‬ﺣ د‬
‫) ‪t (m s‬‬
‫ﺍﻟﺸﻜﻞ‪5-‬‬ ‫‪ -2‬ﻌ ﻣ ة ﻗ رﻫﺎ ‪ 0,5s‬ﻣ‪ 3‬ﺑ ا ﺔ ﺷ‪ 3A‬اﻟ‪?r%‬ﻔﺔ‪ ،‬ﻧ‪Ž‬رﺟﺢ‬
‫اﻟ=ﺎدﻟﺔ إﻟﻰ اﻟ ﺿﻊ ‪  2 ‬ﻓﻲ ﻟ‪DA‬ﺔ ﻧﻌ‪ .‬ﻫﺎ ﻣ أ ﺟ ﯾ ا‬
‫ﻟﻸزﻣ ﺔ ‪. t  0‬‬
‫أ‪ -‬ﺣ د ﻧ‪ n%‬اﻟ‪6 A.‬ﻞ اﻟ<ﺎﻗ ‪ P‬اﻟ‪A‬ﺎدث ﻣ‪ 3‬اﻟ‪?r%‬ﻔﺔ إﻟﻰ اﻟ ﺎﻗﻞ اﻷوﻣﻲ‪.‬‬
‫) ‪du C (t‬‬
‫ب‪ -‬ﺑ‪ 4 <.‬ﻗﺎﻧ ن ﺟ‪%‬ﻊ اﻟ‪ .‬ﺗ ات ﺑ‪ 34‬أن‪ A  u C (t )  0 :‬‬
‫‪dt‬‬
‫‪u C (t )  B e  At‬‬ ‫ﺟـ‪ -‬ﺗ‪A‬ﻘ أن ﺣﻞ اﻟ‪%‬ﻌﺎدﻟﺔ اﻟ‪.‬ﻔﺎﺿﻠ'ﺔ ﻫ ‪:‬‬
‫‪ -3‬اﻻﺳ ‪F‬ﺎﻋﺔ‪:‬‬
‫اﻟ‪A‬ﺎدث ﺎﻟ ارة ﻋ‬ ‫) ‪P  f (t‬‬ ‫‪%‬ﻌﺎﻟ‪N‬ﺔ ﻣﻌﻠ ﻣﺎﺗ'ﺔ أﻣ‪ 3r‬ﻣ‪.‬ﺎ ﻌﺔ اﻟ‪ <.‬ر اﻟ@ﻣ ﻲ ﻻﺳ‪<.‬ﺎﻋﺔ اﻟ‪6 A.‬ﻞ اﻟ<ﺎﻗ ‪P‬‬
‫ﺗﻔ ‪6‬ﻎ اﻟ‪?r%‬ﻔﺔ ‪%h‬ﺎ ﺎﻟ‪r-‬ﻞ‪06-‬‬
‫أ‪ -‬اﻛ‪=… l.‬ﺎرة اﻟ<ﺎﻗﺔ اﻟ‪@s%‬ﻧﺔ ‪?r%‬ﻔﺔ ﺑ ﻻﻟﺔ ‪ C‬ﺳﻌﺔ اﻟ‪?r%‬ﻔﺔ واﻟ‪ A-‬ﺔ ) ‪. q(t‬‬
‫) ‪P ( mWatt‬‬ ‫ب‪ -‬ﺑ‪ 34‬أن اﺳ‪<.‬ﺎﻋﺔ اﻟ‪6 A.‬ﻞ اﻟ<ﺎﻗ ‪ P‬ﺗ[‪ l.‬ﺎﻟ‪r-‬ﻞ‪:‬‬
‫) ‪P (t )  i (t ) uC (t‬‬
‫‪ -IV‬اﺳ ﻐﻼل اﻟ\ ﺎﺋﺞ ‪:‬‬
‫ﺑ اﺳ<ﺔ ﺟﻬﺎز ‪ ExAO‬ﺣ‪O‬ﻠ ﺎ ﻋﻠﻰ ﻣ ‪ A‬ﻰ ﺗ< ر اﺳ‪<.‬ﺎﻋﺔ‬
‫اﻟ‪6 A.‬ﻞ‬
‫اﻟ<ﺎﻗ ‪ P‬اﻟ‪A‬ﺎدث ﻣ‪ 3‬اﻟ‪?r%‬ﻔﺔ إﻟﻰ اﻟ ﺎﻗﻞ اﻷوﻣﻲ‪.‬‬
‫‪8‬‬ ‫‪ -1‬ﺟ …=ﺎرة اﻻﺳ‪<.‬ﺎﻋﺔ اﻷﻋ‪'%D‬ﺔ ‪ P0‬ﺑ ﻻﻟﺔ ‪ E‬و ‪. R‬‬

‫) ‪t (m s‬‬
‫‪ -2‬اﺳ‪. .‬ﺞ ‪%'K‬ﺔ ‪ R‬ﻣﻘﺎوﻣﺔ اﻟ ﺎﻗﻞ اﻷوﻣﻲ ﺛ‪ C a‬ﺳﻌﺔ اﻟ‪?r%‬ﻔﺔ‪.‬‬
‫‪0‬‬ ‫‪25‬‬
‫ﺍﻟﺸﻜﻞ ‪06-‬‬ ‫‪ -3‬ﻣﺎ ‪%'K‬ﺔ اﻟ<ﺎﻗﺔ اﻟ‪ A‬ار‪6‬ﺔ اﻟ‪.‬ﻲ ﯾ ‪ -‬ﻫﺎ اﻟ ﺎﻗﻞ اﻷوﻣﻲ ﻓﻲ‬
‫ﻣ‪<'A‬ﻪ؟‬
‫ن ﻣ‪ $‬ﺗ )‪ $%‬واﺣ ﺗ )‪E%‬ﻲ ‪.‬‬ ‫اﻟ ـــــ ء اﻟ‪Q‬ﺎﻧــــــﻲ ‪ :‬ﯾ‬
‫اﻟ )‪%‬ــــــ‪ $‬اﻟ )‪E%‬ـــﻲ ‪ 06.00 ) :‬ﻧﻘﺎ‪( .‬‬
‫ﺎﺳ‪" a‬ﺗ ‪4?4%6‬ﻞ أﻣ‪ 34‬ﯾ ر‪6‬ﺎ ‪، tri methyl aminuria ‬‬ ‫ﺗٌﻌ ف ﻣ‪.‬ﻼزﻣﺔ راﺋ‪A‬ﺔ اﻟ ‪ Fish Odor Syndrome ^%‬‬

‫ﺻﻔﺤﺔ ‪ 4‬ﻣﻦ ‪11‬‬


‫ﺈﻓ از ﺛﻼﺛﻲ ﻣ‪4?4‬ﻞ أﻣ‪. tri methyl amine  34‬‬ ‫وﻫﻲ …=ﺎرة ﻋ‪ 3‬اﺿ< اب ﻣ‪.‬ﻌﻠ‬
‫‪ CH‬ﻓﻲ اﻟ[?‪ 4‬ﻣ‪ 3‬اﻷﻏ‪ E‬ﺔ و‪ @4%.6‬ﺑ اﺋ‪.A‬ﻪ اﻟ‪@4%%‬ة )راﺋ‪A‬ﺔ اﻟ ‪ ^%‬اﻟ‪.%‬ﻌﻔ‪.(3‬‬
‫ﯾ‪ .‬اﺟ ﺛﻼﺛﻲ ﻣ‪4?4‬ﻞ أﻣ‪3 9N 34‬‬

‫‪6‬ﻘﺔ اﻟ‪ ”'s-.‬ﻋﻠﻰ ‪'K‬ﺎس ‪'%h‬ﺔ "ﺛﻼﺛﻲ ﻣ‪4?4‬ﻞ أﻣ‪ "34‬و"أﻛ ‪ 4‬ﺛﻼﺛﻲ ﻣ‪4?4‬ﻞ أﻣ‪ " 34‬ﻓﻲ اﻟ ل‪ ،‬ﻌ‬ ‫ﺗﻌ‪%.‬‬
‫‪http://www.aljazeera.net/‬‬ ‫ﺗ ﺎول ‪ 300‬ﻏ ام ﻣ‪ 3‬اﻷﺳ‪%‬ﺎك اﻟ=‪6 A‬ﺔ‪ .‬ﻣﻘ ‪ 9:‬ﻋ‪ $‬ﻣ ﻗﻊ ﻗ\ﺎة اﻟ ‪)%‬ة اﻹﺧ‪:‬ﺎر‪%‬ﺔ‬
‫‪ -I‬اﻟ ‪: e G‬‬
‫اﻧ<ﻼﻗﺎ ﻋ‪ 4‬ﺔ ﻟ ل ﺷ‪ ”s‬ﺣ‪%N‬ﻬﺎ ‪ ، 50mL‬ﻧ‪ ŠA‬ﻣ‪A‬ﻠ ﻻ ‪ S1‬ﻣ‪ %‬دا ‪ 10‬ﻣ ات ﺣ‪%N‬ﻪ ‪ ، V1 10mL‬ﻧﻔ ض أن اﻟ ل‬
‫‪h‬ﻠ ر اﻟﻬ‪ 4‬روﺟ‪34‬‬ ‫‪ P .A‬ﻋﻠﻰ أﺳﺎس ﺛﻼﺛﻲ ﻣ?‪4‬ﻞ أﻣ‪ 34‬ﻓﻘ‪ ،n‬ﻧﻌﺎﯾ اﻟ‪A% V1 10mL aNA‬ﻠ ل ﻟ‪%A‬‬
‫) )‪ ( H3O (aq)  Cl  (aq‬ﺗ ‪@4h‬ﻩ ‪ ، Ca  0,084mol / L‬ﺑ‪N.‬ﻬ‪ @4‬ﻣ ﺎﺳ‪ l‬أﻣ‪ 3r‬اﻟ‪ OA‬ل ﻋﻠﻰ اﻟ‪ A %‬ﻰ ) ‪pH  f (Va‬‬
‫)اﻟ‪r-‬ﻞ‪ PH) (7-‬اﻟ‪6@%‬ﺞ اﻟ‪.‬ﻔﺎﻋﻠﻲ ﺑ ﻻﻟﺔ ﺣ‪ aN‬اﻟ‪A%‬ﻠ ل اﻟ‪Š%A‬ﻲ اﻟ‪ r %‬ب ‪.( Va‬‬
‫‪ -1‬اﻟ ‪Y‬ﻠ ل اﻷﺳﺎﺳﻲ‪:‬‬

‫‪c ‬‬ ‫ﻋﺔ‬ ‫ﻣ‬ ‫‪b ‬‬ ‫ﻋﺔ‬ ‫ﻣ‬ ‫‪a‬‬ ‫ﻋﺔ‬ ‫ﻣ‬ ‫أ‪ -‬ﻋ ف اﻷﺳﺎس‪.‬‬
‫‪ -‬ﺑ ‪)G‬‬ ‫‪ -‬ﺑ ‪)G‬‬ ‫‪ -‬ﺑ ‪)G‬‬ ‫ب‪ -‬أﻛ‪ l.‬ﻣﻌﺎدﻟﺔ اﻧ‪A‬ﻼل اﻷﺳﺎس ﻓﻲ اﻟ‪%‬ﺎء‪.‬‬
‫‪ -‬ﺣ ﺟﻠﺔ ‪ h‬ﺎر‪%‬ﺔ ‪10mL‬‬ ‫‪ -‬ﺣ ﺟﻠﺔ ‪ h‬ﺎر‪%‬ﺔ ‪50mL‬‬ ‫‪-‬ﺣ ﺟﻠﺔ ‪ h‬ﺎر‪%‬ﺔ ‪10mL‬‬ ‫ﯾ‪:‬‬ ‫‪ -2‬اﻟ‬
‫‪ -‬ﻣﺎﺻﺔ ‪ h‬ﺎر‪%‬ﺔ ‪1mL‬‬ ‫‪ -‬ﻣﺎﺻﺔ ﻣ رﺟﺔ ‪5 mL‬‬ ‫‪ -‬ﻣ ‪:‬ﺎر ﻣ رج ‪10mL‬‬
‫أ‪ -‬اﺧ‪ .‬اﻟ@ﺟﺎﺟ'ﺎت اﻟ‪ %‬ﺎﺳ=ﺔ ﻟﻌ‪%‬ﻠ'ﺔ‬
‫اﻟ‪ %.‬ﯾ ﻣ‪ 3‬ﺑ‪ 34‬اﻟ‪ %N%‬ﻋﺎت اﻟ‪%‬ﻘ‪ .‬ﺣﺔ‪.‬‬
‫ب‪ -‬أذ‪ h‬اﻟ وﺗ ‪ h‬ل اﻟ‪ 6 N.‬ﻲ ﻟﻌ‪%‬ﻠ'ﺔ اﻟ‪ %.‬ﯾ ‪.‬‬
‫‪ -3‬اﻟ ﻌﺎﯾ)ة‪.‬‬
‫‪pH‬‬
‫أ‪ -‬اذ‪ h‬ﻣ‪@4%‬ات ﺗﻔﺎﻋﻞ اﻟ‪%‬ﻌﺎﯾ ة‪.‬‬
‫ب‪ -‬اﻛ‪ l.‬ﻣﻌﺎدﻟﺔ ﺗﻔﺎﻋﻞ اﻟ‪%‬ﻌﺎﯾ ة‪.‬‬
‫ﺟـ‪ -‬ﻋ‪ 34‬اﺣ اﺛ'ﺎت ﻧﻘ<ﺔ اﻟ‪[.‬ﺎﻓ‪.Ž‬‬
‫ﻟﻠ? ﺎﺋ'ﺔ ‪. C3 H10 N  / C3 H9 N‬‬ ‫‪pKa‬‬ ‫اﻟ‪ %A‬ﺿﺔ‬ ‫ﺛ‪ a‬اﺳ‪. .‬ﺞ ﺛﺎﺑ‬
‫د‪ -‬ﺣ د ﺗ ‪ @4h‬اﻟ‪A%‬ﻠ ل اﻟ‪ %%‬د ‪Cb‬‬
‫واﺳ‪. .‬ﺞ ﺗ ‪ @4h‬ﺛﻼﺛﻲ ﻣ‪4?4‬ﻞ أﻣ‪ 34‬ﺎﻟ ل‪.‬‬
‫ﺔ‪:‬‬ ‫‪ -4‬اﻟ\‬
‫‪2‬‬
‫‪ r‬ن اﻟ‪ ”'s-.‬إ ‪N‬ﺎﺑ'ﺎ ﻣ‪ 3‬أﺟﻞ ﺗ ‪ @4h‬ﺛﻼﺛﻲ ﻣ‪4?4‬ﻞ أﻣ‪34‬‬
‫‪10‬‬
‫‪0‬‬ ‫‪1‬‬ ‫) ‪Va ( mL‬‬
‫ﻣ‪. 2,210 mol / L 3‬‬ ‫‪ CH‬ﻓﻲ اﻟ ل أﻛ‬
‫‪3 9N‬‬
‫اﻟ‪0G‬ﻞ‪7-‬‬
‫‪ -‬ﻫﻞ اﻟ‪ ”s-‬ﻣ‪O‬ﺎب ‪.%‬ﻼزﻣﺔ راﺋ‪A‬ﺔ اﻟ ‪^%‬؟ ﺑ ر‪.‬‬
‫‪ -II‬اﻟﻌﻼج ‪:‬‬
‫ﯾ‪.‬ﻔﺎﻋﻞ ﺛﻼﺛﻲ ﻣ‪4?4‬ﻞ أﻣ‪ 34‬ﻣﻊ ﺑ‪ 4‬و‪ 4 h‬اﻟﻬ‪ 4‬روﺟ‪ H2O2  aq 34‬ﻓﻲ ﺟ ‪ a‬اﻹﻧ ﺎن ﺑ ﺟ د إﻧ@‪ E a6‬ﻛ ﺳ'‪ n‬ﻓ‪. 4‬ﺞ‬
‫اﻟ‪%‬ﺎء وأو‪ 4 h‬ﺛﻼﺛﻲ ﻣ‪4?4‬ﻞ أﻣ‪ 34‬واﻟ‪ PE‬ﻧ ﻣ@ ﻟﻪ ﺑـ ‪ OTMA‬وﻓ اﻟ‪%‬ﻌﺎدﻟﺔ‪:‬‬
‫‪C3 H 9 N  aq   H 2O2  aq   C3 H 9 NO  aq   H 2 O  l ‬‬
‫‪ 3r%‬ﻟﻬ‪E‬ا اﻹﻧ@‪ a6‬ﻓﻲ – وف ﻣﻌ‪ 4‬ﺔ أن ﯾ‪ A.‬ر )ﯾ‪.‬ﻐ‪ 4‬ﺟ@ﺋ'ﺎ( ﻣ‪%‬ﺎ ﯾ‪Ž‬ﺛ ﻋﻠﻰ ﻓﻌﺎﻟ‪.4‬ﻪ‪ ،‬ﻟ‪E‬ا ﻌ‪%‬ﻞ اﻟﻌﻠ‪%‬ﺎء ﻋﻠﻰ دراﺳﺔ‬

‫ﺻﻔﺤﺔ ‪ 5‬ﻣﻦ ‪11‬‬


‫إﻧ@‪%6‬ﺎت ﻣ‪ A‬رة ﻟ‪ A.‬ﯾ اﻟ‪%‬ﻔ‪ 4‬ﻣ ﻬﺎ ﻟﻠﻌﻼج‪.‬‬
‫ﻟﻠ‪.‬ﻔﺎﻋﻞ اﻟ ﺎﺑ ﺑ ﻔ— اﻟ‪ .‬اﻛ‪ @4‬اﻻﺑ‪ .‬اﺋ'ﺔ وﻓﻲ ﻧﻔ— اﻟ‪ -‬و‪ C‬ﻟ[‪3‬‬ ‫ﻧ‪.‬ﺎﺋﺞ دراﺳﺔ ﺣ ‪'h‬ﺔ ﻟ?ﻼث ﺗ‪N‬ﺎرب أﺟ ‪6‬‬ ‫أﻋ<‬
‫ﺑ ﺟ د اﻹﻧ@‪ a6‬اﻷﺻﻠﻲ ‪ E‬ﺛ‪ a‬ﺑ ﺟ د اﻹﻧ@‪ a6‬اﻟ‪ A.%‬ر ‪ E1‬ﺛ‪ a‬ﺑ ﺟ د اﻹﻧ@‪ a6‬اﻟ‪ A.%‬ر ‪ E2‬ﻟ ‪OA‬ﻞ ﻋﻠﻰ اﻟ‪' A %‬ﺎت‬
‫اﻟ 'ﺎﻧ'ﺔ‪ .‬اﻟ‪r-‬ﻞ‪8-‬‬
‫‪E1‬‬ ‫‪ -1‬ﻋ ف اﻟ ﺳ'‪ .n‬وﻣﺎ ﻧ ع اﻟ ﺳﺎ ﺔ؟‬
‫‪ -2‬اﻛ‪=… l.‬ﺎرة اﻟ ﻋﺔ اﻟ‪'%NA‬ﺔ ﻟ‪r-.‬ﻞ أو‪4 h‬‬
‫‪E‬‬ ‫ﺛﻼﺛﻲ ﻣ‪4?4‬ﻞ أﻣ‪. OTMA 34‬‬
‫‪ -3‬اﺣ ‪ l‬اﻟ ﻋﺔ اﻟ‪'%NA‬ﺔ ﻟ‪r-.‬ﻞ أو‪ 4 h‬ﺛﻼﺛﻲ‬
‫ﻣ‪4?4‬ﻞ أﻣ‪ OTMA 34‬ﻋ اﻟﻠ‪DA‬ﺔ ‪ t  0‬ﻓﻲ ﺣﺎﻟﺔ ‪h‬ﻞ إﻧ@‪.a6‬‬
‫‪ -4‬اﻗ‪ .‬ح ﻣﻊ اﻟ‪ 6 .‬اﻷﻧ@‪ a6‬اﻟ‪ A.%‬ر اﻟ‪ %‬ﺎﺳ‪h l‬ﻌﻼج‬
‫ﻟ‪.%‬ﻼزﻣﺔ راﺋ‪A‬ﺔ اﻟ ‪.^%‬‬
‫‪E2‬‬

‫‪3‬‬ ‫ﺍﻟﺸﻜﻞ‪8-‬‬ ‫)‪t (min‬‬

‫أﺳﺎﺗﺬة اﳌ ــﺎدة ﲢﺖ إﺷﺮاف ﻣﻔﺘﺶ اﳌﺎدة‬ ‫‪C‬ﻟﺘﻮﻓﻴﻖ واﻟﻨﺠﺎح‬


‫ﺻﻔﺤﺔ ‪ 6‬ﻣﻦ ‪11‬‬
‫اﻟ ﺿـــــ ع اﻟ‪Q‬ﺎﻧــــﻲ‬
‫اﻟ ـــــ ء اﻷول ‪ :‬ﯾ ـــ ن ﻣـــ‪ $‬ﺛﻼﺛـــﺔ ﺗ ﺎر‪%‬ـــ‪. $‬‬
‫اﻟ )‪%‬ــــــ‪ $‬اﻷول ‪ 04.00 ) :‬ﻧﻘﺎ‪( .‬‬
‫‪6 h‬ﺔ ) ‪.h (S‬ﻠ‪.‬ﻬﺎ ‪ m‬ﻣ‪N‬ﻬ ﻟﺔ ﻟ‪ A.‬ﯾ ‪.%'K‬ﻬﺎ ﻗﺎم اﻷﺳ‪.‬ﺎذ ﺑ‪.‬ﻔ ‪6‬ﺞ اﻟ‪.‬ﻼﻣ‪ E4‬إﻟﻰ ﻣ‪ %N‬ﻋ‪. 34.‬‬
‫دراﺳﺔ ﺳﻘ ‪ C‬ﺷﺎﻗ ﻟﻲ ﻟﻠ[ ‪6‬ﺔ ﻓﻲ اﻟﻬ اء ‪.‬‬ ‫اﻟ‪ %N%‬ﻋﺔ اﻷوﻟﻰ ‪ :‬اﻗ‪ .‬ﺣ‬
‫ﺗ ﻘ‪ n‬اﻟ[ ‪6‬ﺔ ﺷﺎﻗ ﻟ'ﺎ ﺑ ءا ﻣ‪ 3‬ﻧﻘ<ﺔ ‪ O‬ﺎﻟ =ﺔ ﻟ‪%‬ﻌﻠ‪ a‬أرﺿﻲ دون ﺳ ﻋﺔ اﺑ‪ .‬اﺋ'ﺔ ﻓﻲ اﻟﻬ اء ‪ ،‬ﺗﻌ‪ 4‬ﺣ ‪h‬ﺔ‬

‫د ﺎ ﻣﻊ ﺳ ﻋ‪.‬ﻬﺎ‪.‬‬ ‫ﺗ‪ .‬ﺎﺳ‪ l‬ﺷ ﺗﻬﺎ‬ ‫ﺳﻘ ﻬﺎ ﻗ ة اﺣ‪[.‬ﺎك ‪f‬‬


‫) ‪v(m/s‬‬
‫‪?%‬ﻞ اﻟ 'ﺎن اﻟ‪r-‬ﻞ)‪ (1‬ﺗﻐ‪ 4‬ات اﻟ ﻋﺔ ﺑ ﻻﻟﺔ اﻟ@ﻣ‪.3‬‬
‫‪ -1‬ﻣﺎﻫ اﻟ‪ %‬ﺟﻊ اﻟ‪ %‬ﺎﺳ‪ l‬ﻟ راﺳﺔ ﺣ ‪h‬ﺔ ﻫ‪E‬ﻩ اﻟ[ ‪6‬ﺔ ؟‬
‫وﻣﺎﻫﻲ اﻟﻔ ﺿ'ﺔ اﻟ‪.%‬ﻌﻠﻘﺔ ﻪ واﻟ‪.‬ﻲ ﺗ ‪%‬ﺢ ﺑ‪4 <.‬‬
‫اﻟﻘﺎﻧ ن اﻟ?ﺎﻧﻲ ﻟ ‪ 4‬ﺗ‪ 3‬؟‬
‫‪ - 2‬أﻛ‪ l.‬ﻧ” اﻟﻘﺎﻧ ن اﻟ?ﺎﻧﻲ ﻟ ‪ 4‬ﺗ‪.3‬‬
‫‪2,8‬‬ ‫‪ -3‬اﺳ‪. .‬ﺞ ‪%'K‬ﺔ اﻟ ﻋﺔ اﻟ‪ A‬ﺔ ‪ ، vL‬ﺛ‪ a‬أﺣ ‪%'K l‬ﺔ‬
‫‪1,4‬‬ ‫اﻟ‪ .‬ﺎرع اﻻﺑ‪ .‬اﺋﻲ ‪ ، а0‬ﻣﺎذا ﺗ ‪. .‬ﺞ ؟‬
‫اﻟ‪0G‬ﻞ‪-1-‬‬ ‫) ‪t (s‬‬
‫‪ -4‬أﺛ أن اﻟ‪%‬ﻌﺎدﻟﺔ اﻟ‪.‬ﻔﺎﺿﻠ'ﺔ ﻟﻠ‪h A‬ﺔ ﺗ[‪l.‬‬
‫‪dv (t ) k‬‬
‫ﺎﻟ‪r-‬ﻞ‪ v (t )  g :‬‬
‫‪dt‬‬ ‫‪m‬‬
‫ﻌ<ﻰ‪ :‬ﻣﻌﺎﻣﻞ اﻻﺣ‪[.‬ﺎك ‪g =10 m/s ، K=3,57×10 kg/s :‬‬
‫‪2‬‬ ‫‪-‬‬ ‫‪2‬‬ ‫‪ -5‬أﺣ ‪%'K l‬ﺔ ‪.h‬ﻠﺔ اﻟ[ ‪6‬ﺔ ‪.m‬‬
‫‪6 h -‬ﺔ ) ﻧ اس ﻣ ن (‪.‬‬ ‫دراﺳﺔ ﺟ‪%‬ﻠﺔ ﻣﻬ‪@.‬ة ﻧﺎ‬ ‫اﻟ‪ %N%‬ﻋﺔ اﻟ?ﺎﻧ'ﺔ‪ :‬اﻗ‪ .‬ﺣ‬
‫‪S‬‬ ‫ﻣ ن ﺣﻠﻘﺎﺗﻪ ﻏ‪ 4‬ﻣ‪.‬ﻼﺻﻘﺔ ﺛﺎﺑ‬ ‫اﻟ[ ‪6‬ﺔ اﻟ ﺎ ﻘﺔ ﺑ ﺎ‬ ‫ﺗ?‬
‫’‪x‬‬
‫‪o‬‬ ‫‪x‬‬ ‫ﻣ وﻧ‪.‬ﻪ ‪%h ، K= 50 N /m‬ﺎ ﻫ ﻣ ﺿﺢ ﺎﻟ‪r-‬ﻞ)‪( 02‬‬
‫ﻧ@‪6‬ﺢ اﻟ[ ‪6‬ﺔ ) ‪ (S‬ﻋ‪ 3‬وﺿﻊ اﻟ‪ .‬ازن ‪%‬ﻘ ار ) ‪( +X0‬‬
‫اﻟ‪0G‬ﻞ ‪02 -‬‬ ‫وﻧ‪h .‬ﻬﺎ دون ﺳ ﻋﺔ اﺑ‪ .‬اﺋ'ﺔ ﻋ اﻟﻠ‪DA‬ﺔ ‪. t = 0‬‬
‫)‪x(cm‬‬ ‫‪%‬ﺢ ﺗ‪N‬ﻬ‪@4‬ﻣ ﺎﺳ‪ l‬ﺎﻟ‪ OA‬ل ﻋﻠﻰ ﺗ ‪4N‬ﻼ ﻟ‪<%‬ﺎل اﻟ‪h A‬ﺔ ) ‪ ( x‬ﻟ‪ @h %‬ﻋ<ﺎﻟﺔ‬
‫اﻟ[ ‪6‬ﺔ ﺑ ﻻﻟﺔ اﻟ@ﻣ‪%h (t ) 3‬ﺎ ﺎﻟ 'ﺎن اﻟ‪r-‬ﻞ)‪. (03‬‬
‫‪ –1‬ﻣ?ﻞ اﻟﻘ • اﻟ‪Ž%‬ﺛ ة ﻋﻠﻰ اﻟ[ ‪6‬ﺔ ﻋ ﻓﺎﺻﻠﺔ )‪. + x(t‬‬
‫‪3‬‬ ‫‪ –2‬ﻫﻞ ﺣ ‪h‬ﺔ اﻟﻬ@از ﻣ‪s.‬ﺎﻣ ة ؟ ﺑ ر اﺟﺎﺑ‪. ^.‬‬
‫‪0‬‬ ‫)‪t(s‬‬ ‫أوﺟ اﻟ‪%‬ﻘﺎدﯾ اﻟ‪@4%%‬ة اﻟ‪.‬ﺎﻟ'ﺔ ‪ :‬اﻟ ور اﻟ‪E‬اﺗﻲ ) ‪(T0‬‬
‫‪0.1‬‬
‫ﺳﻌﺔ اﻻﻫ‪ .‬ا@زات )‪ ، (X0‬اﻟ‪O‬ﻔ‪A‬ﺔ اﻻﺑ‪ .‬اﺋ'ﺔ) ‪. (φ‬‬
‫ﺷﻜﻞ‪-3-‬‬ ‫‪ –4‬اﻛ‪ l.‬اﻟ‪%‬ﻌﺎدﻟﺔ اﻟ@ﻣ 'ﺔ ﻟﻠ‪h A‬ﺔ ‪.‬‬
‫‪ –5‬اﺣ ‪.h l‬ﻠﺔ اﻟ[ ‪6‬ﺔ ‪ m‬ﺛ‪ a‬ﻗﺎرﻧﻬﺎ ﻣﻊ ﺗﻠ^ اﻟ‪S A%‬ﺔ ﺳﺎ ﻘﺎ‪.‬‬

‫ﺻﻔﺤﺔ ‪ 7‬ﻣﻦ ‪11‬‬


‫اﻟ )‪ $%‬اﻟ‪Q‬ﺎﻧﻲ ‪ 04.00 ) :‬ﻧﻘﺎ‪( .‬‬

‫ﺗ‪ [.‬ن اﻟ ﺷ ﻌﺔ ﻣ‪ 3‬ﺳﻠ^ ﻣﻌ ﻧﻲ ﻣﻠ ﻟ‪ l‬وﻣ<ﻠﻲ ﺎﻟ ﻧ‪ (vernier) 4‬ﻣ‪ 3‬اﺟﻞ ﻋ م ﻣﻼﻣ ﺔ اﻻﺳﻼك ﻟ=ﻌ‪Š‬ﻬﺎ‬

‫وﺣ وث ﺷ ارة ‪h‬ﻬ ‪S‬ﺎﺋ'ﺔ وﻫﻲ ﻣ‪ٍ 3‬اﺧ‪ .‬اع و‪6‬ﻠ'ﺎم ﺳ‪.‬ﺎﻧﻠﻲ ﺟ ﻧ‪ 4‬ر ﺗ ‪.‬ﻌ‪%‬ﻞ ﻓﻲ اﻟ[ﻬ ‪S‬ﺎء ﻟ‪ .‬ﻟ‪ 4‬ﺗ'ﺎر ‪h‬ﻬ و ﻣﻐ ﺎ ' ﻲ‬

‫‪ P N‬ﻣ ﺎﻓﺔ ‪ 4 h‬ة ﻓﻲ ﻣ ﺎﺣﺔ ﺻﻐ‪ 4‬ة ‪s'Ÿ‬ﻠ ﻣ‪N‬ﺎﻻ ﻣﻐ ﺎ ' 'ﺎ ﻗ ‪6‬ﺎ ‪ ،‬و‪h‬ﻠ‪%‬ﺎ ‪ ?h‬ﻋ د اﻟﻠﻔﺎت زادت ﻗ ة اﻟ‪N%‬ﺎل‬

‫اﻟ‪%‬ﻐ ﺎ ' ﻲ‪ ،‬ﻟﻠ ﺷ'ﻌﺔ اﺳ‪ s.‬اﻣﺎت ‪ 4?h‬ة ﺟ ا ﻓﻲ اﻟ[ﻬ ‪S‬ﺎء‪ ،‬ﻓ‪ %‬ﻬﺎ ﻣﺎ ﻌ‪%‬ﻞ ‪%h‬ﻔ‪.‬ﺎح ﻟﻔ‪.‬ﺢ أو ﻗﻔﻞ دارة ‪h‬ﻬ ‪S‬ﺎﺋ'ﺔ‪ ،‬وﻣ ﻬﺎ‬

‫ﻣﺎ ﻌ‪%‬ﻞ ﻓﻲ ﺟ س اﻟ=ﺎب‪ ،‬أﻣﺎ اﻟ[?‪ 4‬ﻣ ﻬﺎ ‪ s. 'Ÿ‬م ﻓﻲ اﻟ‪ %‬ﻟ ات اﻟ[ﻬ ‪S‬ﺎﺋ'ﺔ واﻟ‪ A%‬ﻻت‪.‬‬

‫‪ – I‬دراﺳﺔ ﻣ ‪ C‬ات وﺷ ﻌﺔ ‪:‬‬


‫‪ L,r‬‬
‫‪K‬‬ ‫ﺗ‪ [.‬ن دارة ‪h‬ﻬ ‪S‬ﺎﺋ'ﺔ ﻣ‪ 3‬ﻣ ﻟ ﻣ?ﺎﻟﻲ ﻟﻠ‪ .‬ﺗ ﻗ ﺗﻪ اﻟ‪h A%‬ﺔ اﻟ[ﻬ ‪S‬ﺎﺋ'ﺔ ‪ E‬وﻧﺎﻗﻞ أوﻣﻲ‬
‫‪E‬‬ ‫ﻣﻘﺎوﻣ‪.‬ﻪ ‪ R‬ﺣ‪ ، R  50 •4‬ووﺷ'ﻌﺔ ذاﺗ‪.4‬ﻬﺎ ‪ L‬وﻣﻘﺎوﻣ‪.‬ﻬﺎ ‪ ، r‬ﺎدﻟﺔ ‪K‬‬
‫‪R‬‬ ‫‪%h‬ﺎ ﻫ ﻣ ﺿﺢ ﺎﻟ‪r-‬ﻞ‪. 4-‬‬
‫ﻋ اﻟﻠ‪DA‬ﺔ ‪ t  0‬ﻧﻐﻠ اﻟﻘﺎ ﻌﺔ ‪ K‬و ﻧ ‪N‬ﻞ ﺗﻐ‪ 4‬ات اﻟ‪ .‬ﺗ ‪ u b t ‬اﻟ‪?%%‬ﻞ‬
‫ﺍﻟﺸﻜﻞ‪04-‬‬
‫ﺎﻟ‪r-‬ﻞ‪. 5-‬‬ ‫ﻓﻲ اﻟ 'ﺎن اﻟ‪ %‬ﻓ‬
‫‪u b V‬‬ ‫‪‬‬
‫‪ -1‬أﻋ رﺳ‪ a‬اﻟ ارة ﻣ ﺿ‪A‬ﺎ ﻋﻠ‪4‬ﻬﺎ ﺟﻬﺔ اﻟ‪ .‬ﺗ ات واﻟ‪'.‬ﺎر‪'Œ'h ،‬ﺔ‬
‫ﻓﻲ‬ ‫ر‪ nS‬راﺳ‪ a‬اﻻﻫ‪@.‬از اﻟ ﻗ‪%‬ﻲ ﻟ‪-%‬ﺎﻫ ة اﻟ‪ .‬ﺗ ﺑ‪34‬‬
‫اﻟ ﺷ'ﻌﺔ ‪.u b t ‬‬
‫‪ -2‬ﺎﻻﻋ‪%.‬ﺎد ﻋﻠﻰ اﻟ 'ﺎن ﺟ ‪:‬‬
‫أ‪ -‬اﻟﻘ ة اﻟ‪h A%‬ﺔ اﻟ[ﻬ ‪S‬ﺎﺋ'ﺔ ﻟﻠ‪ %‬ﻟ ﻣﻊ اﻟ‪.‬ﻌﻠ‪4‬ﻞ ‪.‬‬
‫ﻋ ﺑﻠ غ اﻟ ‪D‬ﺎم اﻟ اﺋ‪. a‬‬ ‫‪I‬‬ ‫‪0‬‬
‫ب‪%'K -‬ﺔ ﺷ ة اﻟ‪'.‬ﺎر‬
‫ﺟـ ‪ -‬ﻣﻘﺎوﻣﺔ اﻟ ﺷ'ﻌﺔ ‪. r‬‬
‫‪ -3‬دراﺳﺔ اﻟ ‪D‬ﺎم اﻻﻧ‪.‬ﻘﺎﻟﻲ ‪:‬‬
‫‪2‬‬
‫أ‪ -‬ﺑ‪ 34‬أن اﻟ‪%‬ﻌﺎدﻟﺔ اﻟ‪.‬ﻔﺎﺿﻠ'ﺔ اﻟ‪.‬ﻲ ‪A‬ﻘﻘﻬﺎ اﻟ‪ .‬ﺗ ‪u b t ‬‬
‫‪0‬‬ ‫‪10‬‬ ‫‪t  ms ‬‬ ‫ﻓﻲ اﻟ ﺷ'ﻌﺔ ﺗ[‪ l.‬ﻋﻠﻰ اﻟ‪r-‬ﻞ ‪:‬‬ ‫ﺑ‪34‬‬
‫ﺍﻟﺸﻜﻞ‪05-‬‬ ‫‪du b (t ) R  r‬‬ ‫‪r E‬‬
‫‪‬‬ ‫‪ u b (t ) ‬‬
‫‪dt‬‬ ‫‪L‬‬ ‫‪L‬‬
‫ب‪ -‬ﺣﻞ اﻟ‪%‬ﻌﺎدﻟﺔ اﻟ‪.‬ﻔﺎﺿﻠ'ﺔ ﻣ‪ 3‬اﻟ‪r-‬ﻞ ‪ub  t   A  B  e.t :‬‬
‫ﺣ‪ B ، A •4‬و ‪ ‬ﺛ اﺑ <ﻠ‪ l‬ﺗﻌ‪=… 344‬ﺎرة ‪h‬ﻞ ﻣ ﻬﺎ‪.‬‬
‫ﺟـ‪ -‬ﺑ‪ 34‬أن اﻟ‪%%‬ﺎس ﻋ اﻟﻠ‪DA‬ﺔ ‪ t  0‬ﻘ<ﻊ اﻟ‪ ns‬اﻟ‪%‬ﻘﺎرب ‪ ub  2V‬ﻋ اﻟﻠ‪DA‬ﺔ ‪ ،‬ﺛ‪ a‬اﺳ‪. .‬ﺞ ذاﺗ'ﺔ‬
‫اﻟ ﺷ'ﻌﺔ ‪. L‬‬
‫‪‬‬ ‫‪r ‬‬
‫د‪ -‬ﺑ‪ 34‬أن اﻟ‪%%‬ﺎس ﻋ اﻟﻠ‪DA‬ﺔ ‪ t  0‬ﻘ<ﻊ ﻣ‪ A‬ر اﻷزﻣ ﺔ ﻋ اﻟﻠ‪DA‬ﺔ ‪t    1  ‬‬
‫‪‬‬ ‫‪R‬‬

‫ﺻﻔﺤﺔ ‪ 8‬ﻣﻦ ‪11‬‬


‫)‪q(µC‬‬
‫‪ –II‬دراﺳﺔ اﻟ ارة ‪: LC‬‬
‫اﻟ ﺷ'ﻌﺔ اﻟ ﺎ ﻘﺔ ﻣ?ﺎﻟ'ﺔ ‪ ،‬ﻧ ‪<S‬ﻬﺎ ﻣﻊ ﻣ‪?r‬ﻔﺔ‬ ‫ﻧﻌ‪.‬‬
‫ﻣ‪ A-‬ﻧﺔ ﺳﻌ‪.‬ﻬﺎ ) ‪ ( c‬ﺛ‪ a‬ﻧﻐﻠ اﻟﻘﺎ ﻌﺔ ﻋ‬
‫‪2‬‬ ‫ﻟ‪DA‬ﺔ ﻧﻌ‪ .‬ﻫﺎ ﻣ أ ﻟﻸزﻣ ﺔ ‪. t  0‬‬
‫)‪t(ms‬‬
‫ﻓﻲ‬ ‫ﻣ‪.‬ﺎ ﻌﺔ ﺗﻐ‪ 4‬اﻟ‪ .‬ﺗ ﺑ‪34‬‬
‫‪0‬‬ ‫‪0.4‬‬
‫اﻟ‪?r%‬ﻔﺔ ) ‪ uc (t‬ﻣ‪. r‬ﻪ ﻣ‪ 3‬اﻟ‪ OA‬ل‬
‫اﻟ‪ %‬ﺿﺢ‬ ‫) ‪q (t‬‬ ‫ﻋﻠﻰ ﻣ ‪ A‬ﻰ ﺗﻐ‪ 4‬ات اﻟ‪ A-‬ﺔ‬
‫ﺎﻟ‪r-‬ﻞ‪. 06-‬‬
‫‪ –1‬ﻣﺎ ﻫ اﻟ ‪D‬ﺎم اﻟ‪ PE‬ﯾ ‪ 4‬ﻪ ﻫ‪E‬ا اﻟ‪ A %‬ﻰ؟‬
‫اﻟ‪0G‬ﻞ ‪06‬‬ ‫‪ –2‬أوﺟ اﻟ‪%‬ﻌﺎدﻟﺔ اﻟ‪.‬ﻔﺎﺿﻠ'ﺔ اﻟ‪.‬ﻲ ﺗ‪A‬ﻘﻘﻬﺎ اﻟ‪ A-‬ﺔ ) ‪. q (t‬‬
‫‪2‬‬
‫(‪. q (t )  Q 0 cos‬‬ ‫‪ –3‬ﻌ<ﻰ ﺣﻞ اﻟ‪%‬ﻌﺎدﻟﺔ اﻟ‪.‬ﻔﺎﺿﻠ'ﺔ اﻟ ﺎ ﻘﺔ ﺎﻟ‪r-‬ﻞ ‪t   ) :‬‬
‫‪T0‬‬
‫‪.T‬‬ ‫‪0‬‬ ‫أ– أﻛ‪ l.‬اﻟ‪=0‬ﺎرة اﻟ‪'Ÿ A‬ﺔ ﻟﻠ ور‬
‫ب– اﻋ‪%.‬ﺎدا ﻋﻠﻰ اﻟ 'ﺎن اوﺟ ‪.  ،T0 ، Q0 a'K‬‬
‫ﺟـ – أوﺟ ‪%'K‬ﺔ ﺳﻌﺔ اﻟ‪?r%‬ﻔﺔ ) ‪. ( c‬‬
‫د– اﺣ ‪ l‬اﻟ<ﺎﻗﺔ اﻟ‪@s%‬ﻧﺔ ﻓﻲ اﻟ ﺷ'ﻌﺔ ﻋ اﻟﻠ‪DA‬ﺔ ‪. t  0 .2 m s‬‬
‫اﻟ )‪ $%‬اﻟ‪Q‬ﺎﻟ‪ 06.00 ) : Z‬ﻧﻘﺎ‪( .‬‬
‫‪.‬ﻌ‪%‬ﻞ اﻟ[ ‪ S‬ن ‪ 14‬ﻟ‪.‬ﺄر‪6‬ﺦ ﻗ<ﻊ ﻣ‪ 3‬اﻟ =ﺎﺗﺎت أو اﻟ‪ 4A‬اﻧﺎت وﺟ ت ﻓﻲ أﻣﺎﻛ‪ 3‬أﺛ ‪6‬ﺔ ‪ ،‬إن اﻟ =ﺎﺗﺎت واﻟ‪ 4A‬اﻧﺎت ﺗ‪”.%‬‬
‫ﺛ ﺎﺋﻲ أﻛ ‪ 4‬اﻟ[ ‪ S‬ن اﻟ‪ P .A PE‬ﻋﻠﻰ ﻧ‪ 36 4D‬ﻣ‪ 3‬اﻟ[ ‪ S‬ن ‪ 12‬و ‪ 14‬أﺛ ﺎء ﺣ'ﺎﺗﻬﺎ‪،‬ﺗ[ ن اﻟ =ﺔ ﺑ‪ 34‬اﻟ ‪ 36 4D‬ﺛﺎﺑ‪.‬ﺔ‪.‬‬
‫أﻣﺎﻋ اﻟ ﻓﺎة ﺗ‪ .‬ﻗﻒ اﻟ =ﺎﺗﺎت واﻟ‪ 4A‬اﻧﺎت ﻋ‪ 3‬اﻣ‪O.‬ﺎص ﺛ ﺎﺋﻲ أﻛ ‪ 4‬اﻟ[ ‪ S‬ن‪ ،‬ﯾ أ ﻋ ﺋ‪ E‬اﻟ[ ‪ S‬ن ‪ 14‬اﻟ‪-%‬ﻊ اﻟ‪ .%‬اﺟ‬
‫ﻓﻲ أﻧ ‪.N‬ﻬﺎ ﺎﻟ‪.‬ﻔ[^ إﻟﻰ أزوت ‪ 14‬دون أن ﯾ‪ N.‬د‪ ،‬و‪6‬ﻘ ر زﻣ‪ 3‬ﻧ‪O‬ﻒ ﻋ‪ %‬اﻟ[ ‪ S‬ن ‪ 14‬ﺑ ـ ـ ـ ‪5730 ans :‬‬
‫‪ -‬اﻟ ‪-‬ﺎ‪ C‬اﻻﺷﻌﺎﻋﻲ –ﺎﻫ ة ﻋ‪ -‬اﺋ'ﺔ ؟‬ ‫‪ -1‬ﻣﺎﻟ‪%‬ﻘ‪ O‬د ﺑـ ـ ـ ‪ - :‬ﻧ‪ 4D‬ﻣ‪-‬ﻊ ؟‬
‫‪ -2‬أذ‪ h‬ﻗﺎﻧ ﻧﺎ اﻹﻧ‪A‬ﻔﺎ‪ €‬ﻟﻠﻌﺎﻟ‪ a‬ﺻ د‪.P‬‬
‫‪ -3‬أﻛ‪ l.‬ﻣﻌﺎدﻟﺔ اﻟ‪.‬ﻔ[^ ﻟﻠ[ ‪ S‬ن ‪ 14‬؟ ﻣ ‪N. .‬ﺎ ﻧ‪ n%‬اﻟ‪.‬ﻔ[^ اﻟ‪A‬ﺎدث ؟ وﻣ?ﻠﻪ ﻋﻠﻰ اﻟ‪. ( N-Z ) n<s%‬‬
‫ﻳﻌﻄﻰ ‪2He , 6C , 9 F , 7N , 8O :‬‬

‫‪ -4‬ﻣﺎذا ‪%‬ﻰ اﻟ@ﻣ‪ 3‬اﻟﻼزم ﻟ‪ ..‬ﺎﻗ” ﻋ د أﻧ ‪6‬ﺔ اﻟ[ ‪ S‬ن ‪ 14‬اﻻﺑ‪ .‬اﺋ'ﺔ ﻟﻠ ‪O‬ﻒ ﻌ ﻣ ت اﻟ[ﺎﺋ‪ 3‬؟‬
‫‪.‬‬ ‫اﻟ‪.‬ﻔ[^ اﻻﺷﻌﺎﻋﻲ ‪‬‬ ‫ﺛ‪ a‬أوﺟ …=ﺎرة ﻫ‪E‬ا اﻟ@ﻣ‪ 3‬ﺑ ﻻﻟﺔ ﺛﺎﺑ‬
‫) ‪N 0 ( 14C‬‬
‫‪12‬‬
‫ﺛﺎﺑ‪.‬ﺔ ﻓﻲ اﻟ[ﺎﺋ ﺎت اﻟ‪'A‬ﺔ‪ ،‬وﺗﻘ ر ﺑـ ـ ـ ‪ 1, 2  10 12 :‬‬ ‫‪14C‬‬
‫‪ -5‬ﺗ[ ن اﻟ =ﺔ ﺑ‪, 12C 34‬‬
‫)‪N ( C‬‬
‫‪ l‬ﺗ ﻗﻒ اﻻﻣ‪O.‬ﺎص ‪.‬‬ ‫و‪S‬ﻌ وﻓﺎة اﻟ[ﺎﺋ‪ 3‬ﺗ‪ .‬ﺎﻗ” ﻫ‪E‬ﻩ اﻟ =ﺔ ﻷن ‪ 14C‬ﯾ‪ .‬ﺎﻗ” أ‪ P‬ﻻ ﯾ‪ N.‬د‬
‫) ‪N (t )( 14C‬‬
‫‪12‬‬
‫‪ -‬ﻓﻲ ﺣﻔ ‪6‬ﺔ ﻟ[ﺎﺋ‪ 3‬ﺣﻲ وﺟ ت دراﺳﺔ أن ‪%'K‬ﺔ اﻟ =ﺔ اﻟ ﺎ ﻘﺔ ﻫﻲ‪ 0, 25  10 12 :‬‬
‫)‪N ( C‬‬

‫ﺻﻔﺤﺔ ‪ 9‬ﻣﻦ ‪11‬‬


‫‪ ah -‬ﻣ‪Š‬ﻰ ﻋﻠﻰ وﻓﺎة ﻫ‪E‬ا اﻟ[ﺎﺋ‪ 3‬؟‬
‫‪ -6‬ﻟ‪ A.‬ﯾ ﻋ‪ %‬ﻗ<ﻌﺔ ﺧ‪ l-‬ﻗ ‪ —'K ، a‬اﻟ ‪-‬ﺎ‪ C‬اﻻﺷﻌﺎﻋﻲ ﻟﻌ‪ 4‬ﺔ ﻣ ﻬﺎ ‪.h‬ﻠ‪.‬ﻬﺎ ‪ m=300 mg‬ﻋ ﻟ‪DA‬ﺔ ‪t‬‬
‫ﻓ ﺟ ‪ 1,38‬ﺗﻔ[^ ﻓﻲ اﻟ ‪'K‬ﻘﺔ ‪ ،‬وأﺧ‪E‬ت ﻋ‪ 4‬ﺔ ﻟﻬﺎ ﻧﻔ— اﻟ[‪.‬ﻠﺔ اﻟ ﺎ ﻘﺔ ﻣ‪ 3‬ﺷ‪ N‬ة ﺣ'ﺔ ﻓ ﺟ أن ‪.h‬ﻠﺔ اﻟ[ ‪ S‬ن‪12‬‬
‫‪.‬‬ ‫ﻓ‪4‬ﻬﺎ ‪150 mg‬‬
‫أ‪ -‬أﺣ ‪ l‬ﻋ د أﻧ ‪6‬ﺔ اﻟ[ ‪ S‬ن‪ ، 12‬واﺳ‪. .‬ﺞ ﻋ د أﻧ ‪6‬ﺔ اﻟ[ ‪ S‬ن ‪ 14‬اﻟ‪ %‬ﺟ دة ﻓﻲ اﻟﻌ‪ 4‬ﺔ اﻟ‪.‬ﻲ أﺧ‪E‬ت ﻣ‪3‬‬
‫اﻟ‪ N-‬ة اﻟ‪'A‬ﺔ ‪.‬‬
‫ب‪ -‬أﺣ ‪%'K l‬ﺔ اﻟ ‪-‬ﺎ‪ C‬اﻻﺷﻌﺎﻋﻲ اﻻﺑ‪ .‬اﺋﻲ ‪ ،‬ﺛ‪ a‬ﺣ د ﻋ‪ %‬ﻗ<ﻌﺔ اﻟ‪. l-s‬‬
‫اﻟ ء اﻟ‪Q‬ﺎﻧﻲ‪ 06) :‬ﻧﻘﺎ‪(.‬‬
‫اﻟ )‪%‬ـــ‪ $‬اﻟ )‪E%‬ـــﻲ ‪ 06.00 ) :‬ﻧﻘﺎ‪( .‬‬
‫‪.h‬ـﻠ‪.‬ﻬﺎ ‪ m  4,67g‬ﻓﻲ اﻟ‪%‬ﺎء اﻟ‪%‬ﻘ< ﻓ‪OA.‬ﻠ ا ﻋﻠﻰ‬ ‫ﻘﻲ ) ‪( A‬‬ ‫‪4 h S h‬ﻠﻲ اﻟ‬ ‫ﻗﺎم ﺗﻼﻣ‪ E4‬اﻟﻔ ج ﺑـ‪A‬ﻞ ‪'%h‬ﺔ ﻣ‪ 3‬اﻟ‪%A‬‬
‫ﻣ‪A‬ﻠ ل )‪ (S1‬ﺣ‪%N‬ﻪ ‪ V  200 mL‬وﻟﻪ ‪ pH1  2,7‬وﺗ ‪@4h‬ﻩ اﻟ‪ %‬ﻟﻲ ‪. C1‬‬
‫‪ ،‬ﻟﻪ ‪. pH 2  2,9‬‬ ‫‪C2 ‬‬
‫‪C1‬‬
‫اﻧ<ﻼﻗﺎ ﻣ‪ 3‬اﻟ‪A%‬ﻠ ل )‪ (S1‬ﻗﺎﻣ ا ﺑ‪ 4ŠA.‬ﻣ‪A‬ﻠ ل ) ‪ (S2‬ﺗ ‪@4h‬ﻩ اﻟ‪ %‬ﻟﻲ‬
‫‪10‬‬

‫) ‪ ( A‬ﺿ‪ ،£'0‬ﺛ‪ a‬اذ‪ h‬اﻟ وﺗ ‪ h‬ل اﻟ‪ 6 N.‬ﻲ ﻟ‪ 4ŠA.‬اﻟ‪A%‬ﻠ ل ) ‪. (S2‬‬ ‫‪ –1‬ﺑ‪ 34‬أن اﻟ‪%A‬‬
‫ﻣﻊ اﻟ‪%‬ﺎء ﻓﻲ اﻟ‪A%‬ﻠ ل )‪ (S1‬ﺗ‪ a‬اﺣ ‪ l‬اﻟ‪ @4h .‬اﻟ‪ %‬ﻟﻲ ﻟﻠ‪A%‬ﻠ ل )‪. (S1‬‬ ‫‪ –2‬اﻛ‪ l.‬ﻣﻌﺎدﻟﺔ ﺗﻔﺎﻋﻞ اﻟ‪%A‬‬
‫واﻛ‪ l.‬ﺻ'ﻐ‪.‬ﻪ ﻧ‪O‬ﻒ اﻟ‪%‬ﻔ‪O‬ﻠﺔ واذ‪ h‬اﺳ‪%‬ﻪ‪.‬‬ ‫)‪( A‬‬ ‫‪ –3‬أوﺟ اﻟ‪'O‬ﻐﺔ اﻟ‪%N%‬ﻠﺔ ﻟﻠ‪%A‬‬
‫) اﻟ‪A%‬ﺎﻟ‪4‬ﻞ ﻣﺄﺧ ذة ﻓﻲ اﻟ رﺟﺔ ‪.( 25C‬‬
‫‪  II‬اﻟﻔ ج اﻟ‪Q‬ﺎﻧﻲ‪X :‬ﻠﻒ ‪) p‬ا‪:o‬ﺔ ﺗﻔﺎﻋﻞ اﻟ‪ n Y‬اﻟ )‪CDX L‬ﻠﻲ ﻣﻊ ‪ YX‬ل‪.‬‬
‫ﻗﺎم ﺗﻼﻣ‪ E4‬اﻟﻔ ج ﺎﻟﻌ‪%‬ﻠ'ﺎت اﻟ‪.‬ﺎﻟ'ﺔ ‪:‬‬
‫ﺻ'ﻐ‪.‬ﻪ اﻟ‪%N%‬ﻠﺔ ‪ C3 H8O‬ﻓﻲ ﺣ ﺟﻠﺔ ﻣ@ودة‬ ‫)‪(B‬‬ ‫و ‪ 0,3mol‬ﻣ‪ Ah 3‬ل‬ ‫)‪( A‬‬ ‫– ﻣ@ج ‪ 0,2mol‬ﻣ‪ 3‬اﻟ‪%A‬‬
‫‪N‬ﻬﺎز اﻟ‪ 34s .‬اﻟ‪ %‬ﺗ ‪.‬‬
‫اﻟ‪ @h %‬ﻟﻠ‪6@%‬ﺞ‪.‬‬ ‫اﻟ[ ‪6‬‬ ‫– اﺿﺎﻓﺔ ‪Š‬ﻊ ﻗ< ات ﻣ‪ 3‬ﺣ‪%‬‬
‫– ﺗ ‪ 34s‬اﻟ‪6@%‬ﺞ ﻟ‪ %‬ة ‪h‬ﺎ‪'Ÿ‬ﺔ ﻟ ﺻ ل اﻟ‪.‬ﻔﺎﻋﻞ ﻟ‪A‬ﺎﻟﺔ اﻟ‪ .‬ازن‪.‬‬
‫– ﺗ ‪ 6‬اﻟ‪6@%‬ﺞ ﻣﻊ اﺿﺎﻓﺔ ‪'%h‬ﺔ ﻣ‪ 3‬ﻣ‪A‬ﻠ ل ‪h‬ﻠ ر اﻟ‪ O‬دﯾ م‪.‬‬
‫ﺑ اﺳ<ﺔ ﻫ‪ 4‬روﺟ‪ S h 34‬ﻧﺎت اﻟ‪ O‬دﯾ م ) ‪(Na  HCO3‬‬ ‫ﻌ ﻋ‪%‬ﻠ'ﺔ اﻟ ‪ lr‬وﺗ •'ﺔ اﻷﺳ‪ .‬ﻣ‪ 3‬اﻟ‪%A‬‬
‫اﻟ‪r-.%‬ﻞ ‪. m E  16, 47 g‬‬ ‫)‪(E‬‬ ‫وﺟ اﻟ‪.‬ﻼﻣ‪ E4‬أن ‪.h‬ﻠﺔ اﻷﺳ‪.‬‬
‫اﻟ‪@h %‬؟‬ ‫اﻟ[ ‪6‬‬ ‫‪ –1‬ﻣﺎ ﻫ دور اﻟ‪ 34s .‬اﻟ‪ %‬ﺗ ‪ ،‬وﻣﺎ اﻟﻔﺎﺋ ة ﻣ‪ 3‬إﺿﺎﻓﺔ ﻗ< ات ﻣ‪ 3‬ﺣ‪%‬‬
‫‪ –2‬ﻣﺎ اﻟﻔﺎﺋ ة ﻣ‪ 3‬إﺿﺎﻓﺔ ﻣ‪A‬ﻠ ل ‪h‬ﻠ ر اﻟ‪ O‬دﯾ م؟‬
‫‪ –3‬اﻛ‪ l.‬ﻣﻌﺎدﻟﺔ ﺗﻔﺎﻋﻞ اﻷﺳ‪ .‬ة‪ ،‬واذ‪ h‬ﺧ‪O‬ﺎﺋ” ﻫ‪E‬ا اﻟ‪.‬ﻔﺎﻋﻞ‪.‬‬
‫ﺗ ازن ﻫ‪E‬ا اﻟ‪.‬ﻔﺎﻋﻞ‪ ،‬واﺳ‪. .‬ﺞ ﺻ ﻒ اﻟ[‪ A‬ل اﻟ‪. %‬ﻌ‪%‬ﻞ واﻛ‪ l.‬ﺻ'ﻐ‪.‬ﻪ اﻟ‪%‬ﻔ‪O‬ﻠﺔ‪.‬‬ ‫‪ –4‬اﺣ ‪ l‬ﺛﺎﺑ‬
‫‪ –5‬اﺣ ‪ l‬ﻣ دود اﻟ‪.‬ﻔﺎﻋﻞ‪ ،‬واذ‪ h‬اﻟ< ‪6‬ﻘﺔ اﻟ‪.‬ﻲ ﻧ ﻓﻊ ﺑﻬﺎ اﻟ‪ %‬دود وﻧ‪OA‬ﻞ ﻋﻠﻰ اﺳ‪ .‬ﻧﻘﻲ‪.‬‬

‫ﺻﻔﺤﺔ ‪ 10‬ﻣﻦ ‪11‬‬


‫‪ III‬اﻟﻔ ج اﻟ‪Q‬ﺎﻟ‪X :Z‬ﻠﻒ ﺑ راﺳﺔ ﺗﻔﺎﻋﻞ اﻷﺳ ) اﻟ\ﺎﺗﺞ ﻣﻊ ﻣ‪Y‬ﻠ ل ﻫ‪ C‬رو‪ CDX‬اﻟ‪ 7‬دﯾ م‪.‬‬
‫‪‬‬ ‫‪‬‬
‫ﻗﺎم ﺗﻼﻣ‪ E4‬اﻟﻔ ج ‪@%‬ج ‪'%h‬ﺔ ) ‪ (n0‬ﻣ‪ 3‬اﻷﺳ‪ ( E ) .‬ﻣﻊ ‪'%h‬ﺔ ) ‪ (n0‬ﻣ‪ 3‬ﻣ‪A‬ﻠ ل ﻫ‪ 4‬رو‪ 4 h‬اﻟ‪ O‬دﯾ م ) ‪، (Na OH‬‬
‫ﻟ‪4r-.‬ﻞ وﺳ‪ n‬ﺗﻔﺎﻋﻠﻲ ﺣ‪%N‬ﻪ ‪. V  100mL‬‬
‫‪ –1‬ﻣﺎ اﺳ‪ a‬اﻟ‪.‬ﻔﺎﻋﻞ اﻟ‪A‬ﺎدث ﺑ‪ 34‬اﻷﺳ‪ .‬وﻫ‪ 4‬رو‪ 4 h‬اﻟ‪ O‬دﯾ م ؟ ﺛ‪ a‬اﻛ‪ l.‬ﻣﻌﺎدﻟﺔ اﻟ‪.‬ﻔﺎﻋﻞ اﻟ‪E% %‬ﺟﺔ ﻟﻠ‪ A.‬ل‬
‫ﺧ‪O‬ﺎﺋ‪O‬ﻪ‪.‬‬ ‫ﻌ‬ ‫اﻟ['‪'%‬ﺎﺋﻲ اﻟ‪A‬ﺎدث‪ ،‬اذ‪h‬‬
‫‪ –2‬أﻧ‪-‬ﺊ ﺟ وﻻ ﻟ‪.‬ﻘ م اﻟ‪.‬ﻔﺎﻋﻞ اﻟ‪A‬ﺎدث‪.‬‬
‫ﻣ‪ 3‬ﺗ‪4?%‬ﻞ ﺑ'ﺎن ﺗﻐ‪ 4‬ات ﺗﻘ م‬ ‫‪ –3‬ﻣ‪.‬ﺎ ﻌﺔ ﺗ< ر اﻟ‪.‬ﻔﺎﻋﻞ ﺑ اﺳ<ﺔ ﺟﻬﺎز ‪'K‬ﺎس اﻟ ﺎﻗﻠ'ﺔ اﻟ …'ﺔ ﻟﻠ‪6@%‬ﺞ ﻣ‪r‬‬
‫اﻟ‪.‬ﻔﺎﻋﻞ ﺑ ﻻﻟﺔ اﻟ ﺎﻗﻠ'ﺔ اﻟ …'ﺔ ) ‪. x  f (‬‬
‫اﻋ‪%.‬ﺎدا ﻋﻠﻰ اﻟ 'ﺎن أوﺟ ‪:‬‬
‫أ – ‪%'K‬ﺔ اﻟ ﺎﻗﻠ'ﺔ اﻟ …'ﺔ )‪ (0‬ﻟﻠ‪6@%‬ﺞ اﻟ‪.‬ﻔﺎﻋﻠﻲ‬
‫اﻻﺑ‪ .‬اﺋﻲ‪.‬‬
‫ب– ‪%'K‬ﺔ اﻟ‪.‬ﻘ م اﻷﻋ‪%D‬ﻲ ‪. xmax‬‬
‫ﺟـ‪%'K -‬ﺔ اﻟ ﺎﻗﻠ'ﺔ اﻟ …'ﺔ ﻓﻲ ﻧﻬﺎ ﺔ اﻟ‪.‬ﻔﺎﻋﻞ ‪،‬‬
‫‪. C H‬‬ ‫‪‬‬
‫ﺛ‪ a‬اﺣ ‪%'K l‬ﺔ‬
‫‪n‬‬ ‫‪2 n 1COO‬‬

‫اﻟ ﺎﻗﻠ'ﺔ اﻟ …'ﺔ‬ ‫‪ –4‬ﻓﻲ اﻟﻠ‪DA‬ﺔ ‪h t  8mn‬ﺎﻧ‬


‫‪.‬‬ ‫ﻟﻠ‪6@%‬ﺞ ‪ 1.68 S .m  1‬‬

‫‪ -‬ﺣ د ‪%'K‬ﺔ زﻣ‪ 3‬ﻧ‪O‬ﻒ اﻟ‪.‬ﻔﺎﻋﻞ ‪. t1 2‬‬


‫ﻣﻌ‪ F‬ﺎت‪:‬‬
‫‪Ka (Cn H2n1COOH / Cn H2n1COO )  1,26 105‬‬ ‫ﻓﻲ اﻟ رﺟﺔ ‪: 25C‬‬
‫‪M(H) 1g / mol ، M(C) 12g / mol ، M(O) 16g / mol‬‬ ‫‪، Na  5ms.m2 .mol 1‬‬
‫‪‬‬

‫أﺳﺎﺗ=ة اﻟ ـــﺎدة ﺗ‪ ]Y‬إﺷ)اف ﻣﻔ ‪ u‬اﻟ ﺎدة‬ ‫‪p‬ﺎﻟ ﻓ‪ tC‬واﻟ\ ﺎح‬

‫ﺻﻔﺤﺔ ‪ 11‬ﻣﻦ ‪11‬‬


‫اﻟ ﺎر ــﺦ ‪2023/05/18 :‬‬ ‫ــﺔ ﻟ ﻻ ﺔ ﺗ ــﺔ‬ ‫ﻣ ﯾ ــﺔ اﻟ‬
‫د‬ ‫ﺳــﺎ‬
‫‪30 ،‬‬ ‫اﻟ!ــــــ ـ ـــ ة‪03 :‬‬ ‫اﻣ (ـــــــﺎن اﻟ ‪,‬ﺎﻟـــــ ر ﺎ اﻟ ‪* +‬ــــﻲ ﻣــﺎ) ‪2023‬‬ ‫‪ :‬اﻟ ﺎﻟ ـﺔ ر ﺎﺿﻲ ‪ +‬ﺗﻘ‪$‬ﻲ ر ﺎﺿﻲ‬ ‫اﻟ!‬
‫اﻟ!ﻘﺎ;ﻌـــﺔ اﻟ ﻔ ‪9:9‬ﺔ‪ :‬ﺗ ـــــﺔ‪02‬‬ ‫اﺧ *ــــﺎر ﻓﻲ ﻣـ ــــﺎدة ‪ :‬اﻟﻌﻠـــــــ ـ م اﻟﻔ ‪ 2‬ﺎﺋ ـــــــﺔ‬
‫اﻟ ?(‪9‬ﺢ اﻟ‪ !$‬ذﺟﻲ ﻻﻣ (ﺎن اﻟ ‪,‬ﺎﻟ ر ﺎ اﻟ ‪* +‬ﻲ‬
‫اﻟ! ﺿـــــ ع اﻷول ‪:‬‬
‫ﺍﻟﻌﻼﻣﺔ‬
‫ﻋﻧﺎﺻــﺭ ﺍﻹﺟﺎﺑــﺔ‬
‫ﻣﺟﻣﻭﻉ‬ ‫ﻣﺟﺯﺃﺓ‬
‫ﺍﻟﺘﻤﺮﻳﻦ ﺍﻷﻭﻝ‪ 04) :‬ﻧﻘﺎﻁ(‬
‫‪ -I‬ﺍﻟﺴﻴﺰﻳﻮﻡ ‪137‬‬
‫‪ -1‬ﺍﻟﻌﺪﺩ ﺍﻟﺬﺭﻱ‪ :‬ﻫﻮ ﻋﺪﺩ ﻳﻤﻴﺰ ﺍﻟﻌﻨﺼﺮ ﺍﻟﻜﻴﻤﻴﺎﺋﻲ ‪ ،‬ﻭﻳﻤﺜﻞ ﻋﺪﺩ ﺍﻟﺒﺮﻭﺗﻮﻧﺎﺕ ﻓﻲ ﻧﻮﺍﺓ ﻫﺬﺍ‬
‫‪2x0,25‬‬ ‫ﺍﻟﻌﻨﺼﺮ‬
‫‪ -‬ﻧﻈﻴﺮ ﻣﺴﺘﻘﺮ‪ :‬ﻫﻮ ﻧﻮﺍﺓ ﺗﺤﺎﻓﻆ ﻋﻠﻰ ﻣﻜﻮﻧﺎﺗﻬﺎ ﺩﻭﻥ ﺗﻐﻴﺮ ﻭﻻ ﺗﺼﺪﺭ ﺃﻱ ﺇﺷﻌﺎﻋﺎﺕ‪.‬‬
‫‪ -2‬ﺍﻟﺘﻌﻠﻴﻞ ‪ :‬ﺗﻜﻮﻥ ﻧﻮﺍﺓ ﺍﻟﺒﺎﺭﻳﻮﻡ ‪ 137‬ﺍﻟﺒﻨﺖ ﺍﻟﻨﺎﺗﺠﺔ ﻋﻦ ﺗﻔﻜﻚ ﺍﻟﺴﻴﺰﻳﻮﻡ ‪ 137‬ﻓﻲ ﺣﺎﻟﺔ ﻣﺜﺎﺭﺓ‬
‫‪0,25‬‬ ‫ﺃﻱ ﻟﻬﺎ ﻓﺎﺋﺾ ﻣﻦ ﺍﻟﻄﺎﻗﺔ ﻓﺘﺼﺪﺭ ﺇﺷﻌﺎﻉ ﻏﺎﻣﺎ ‪‬‬

‫‪56 Ba  56 Ba  ‬‬
‫‪137‬‬ ‫*‬ ‫‪137‬‬

‫‪ -3‬ﻣﻌﺎﺩﻟﺔ ﺗﻔﻜﻚ ﺍﻟﺴﻴﺰﻳﻮﻡ ‪: 137‬‬


‫‪137‬‬ ‫‪137‬‬
‫‪55 Cs  56 B a  Z X  ‬‬
‫‪A‬‬

‫‪ A0‬‬
‫‪ ‬ﺍﻟﻨﻤﻂ ﺍﻟﺼﺎﺩﺭ ﻫﻮ‪  :‬‬ ‫ﺑﺘﻄﺒﻴﻖ ﻗﺎﻧﻮﻧﻲ ﺻﻮﺩﻱ‬
‫‪Z  1‬‬
‫‪0,50‬‬ ‫‪137‬‬ ‫‪137‬‬
‫‪55‬‬ ‫‪Ba  10e  ‬‬
‫‪Cs ‬‬ ‫‪56‬‬

‫‪ -4‬ﺣﺴﺎﺏ ﻋﺪﺩ ﺃﻧﻮﻳﺔ ﺍﻟﺴﻴﺰﻳﻮﻡ ‪ 137‬ﻓﻲ ﻛﻞ ﻛﻴﻠﻮﻣﺘﺮ ﻣﺘﺮ ﻣﺮﺑﻊ‪:‬‬


‫) ‪A(t )   N (t‬‬
‫‪0,25‬‬ ‫) ‪A(t‬‬ ‫‪80 103 bq / km2‬‬
‫‪N (t ) ‬‬ ‫‪‬‬ ‫‪ 365, 25  24  3600s  8, 415 1010 noyaux‬‬
‫‪‬‬ ‫‪30‬‬
‫‪ -II‬ﺍﻟﺒﻠﻮﺗﻮﻧﻴﻮﻡ ‪: 239‬‬
‫‪ -1‬ﺗﺤﻮﻝ ﺍﻟﻴﻮﺭﺍﻧﻴﻮﻡ ‪ 238‬ﺇﻟﻰ ﺍﻟﺒﻠﻮﺗﻮﻧﻴﻮﻡ ‪ 239‬ﻟﻴﺲ ﺗﻔﺎﻋﻞ ﺍﻧﺸﻄﺎﺭ ﻷﻥ ﺍﻻﻧﺸﻄﺎﺭ ﻳﻌﻄﻲ‬
‫‪0,25‬‬ ‫ﻧﻮﺍﺗﻴﻦ‪.‬‬
‫‪ -2‬ﺗﻌﺮﻳﻒ ﻁﺎﻗﺔ ﺍﻟﺮﺑﻂ ﻟﻠﻨﻮﺍﺓ ‪: Eℓ‬‬
‫‪0,25‬‬ ‫ﻫﻲ ﺍﻟﻄﺎﻗﺔ ﺍﻟﻮﺍﺟﺐ ﺇﻋﻄﺎﺋﻬﺎ ﻟﻠﻨﻮﺍﺓ ﻟﺘﺘﻔﻜﻚ ﺍﻟﻰ ﻣﻜﻮﻧﺎﺗﻬﺎ ﻣﻨﻔﺼﻠﺔ ﻭﺳﺎﻛﻨﺔ‪.‬‬
‫‪ -3‬ﺃ‪ -‬ﺣﺴﺎﺏ ‪ : E3 , E2 , E1‬ﻫﻲ ﻁﺎﻗﺔ ﺍﻟﻜﺘﻠﺔ )ﻋﻼﻗﺔ ﺃﻳﻨﺸﺘﺎﻳﻦ(‬
‫‪E1  ( m 239 Pu  mn ).C 2  (239, 05216  1, 00866)  931,5  223616, 65Mev‬‬
‫‪94‬‬

‫‪E2  (94.m p  146.mn ).C 2  (94 1, 00728  146  1, 00866)  931,5  225375,19 Mev‬‬
‫‪3x0,25‬‬
‫‪E3  (m134 Xe  m103 Zr  3mn ).C 2  (133,90539  102, 92720  3 1, 00866)  931, 5  223428, 26Mev‬‬
‫‪54‬‬ ‫‪40‬‬

‫ﺏ‪ -‬ﺍﺳﺘﻨﺘﺎﺝ ﻗﻴﻤﺔ ﺍﻟﻤﻘﺎﺩﻳﺮ‪ E1 :‬ﻭ ‪. E 2‬‬


‫‪239‬‬
‫‪94 Pu‬‬
‫‪ E1‬ﻁﺎﻗﺔ ﺍﻟﺮﺑﻂ ‪ Eℓ‬ﻟﻨﻮﺍﺓ‬
‫‪103‬‬
‫‪40‬‬ ‫‪ 134‬ﻭ ‪Zr‬‬
‫‪54 Xe‬‬
‫‪ E 2‬ﻫﻲ ﻁﺎﻗﺘﻲ ﺍﻟﺮﺑﻂ ﻟﻨﻮﺍﺗﻲ‬
‫‪0,25‬‬ ‫‪E 1  E 2  E1‬‬
‫‪E 1  225375,19  223616,65  1758.54MeV‬‬

‫‪1‬‬
‫‪0,25‬‬ ‫‪E 2  E 3  E 2‬‬
‫‪E 2  223428, 26  225375,19  1946,93MeV‬‬

‫ﺟـ‪ -‬ﺍﻟﻄﺎﻗﺔ ﺍﻟﻤﺤﺮﺭﺓ ﻋﻦ ﺗﻔﺎﻋﻞ ﺍﻧﺸﻄﺎﺭ ﺍﻟﺒﻠﻮﺗﻮﻧﻴﻮﻡ ‪.239‬‬


‫‪0,25‬‬
‫‪E lib  E 1  E 2  1758.54   1946,93  188,39Mev‬‬

‫‪ -III‬ﺍﻟﻴﺮﺑﻮﻉ ﺍﻷﺯﺭﻕ ‪: Gerboise bleue‬‬


‫ﻳﺠﺐ ﺣﺴﺎﺏ ﺍﻟﻄﺎﻗﺔ ﺍﻟﻤﺤﺮﺭﺓ ﺍﻟﻜﻠﻴﺔ ‪ ،‬ﺛﻢ ﺣﺴﺎﺏ ﺍﻟﻜﺘﻠﺔ ‪:‬‬
‫‪ETotal  70  4,184  1012 J  2, 92  1014 J‬‬
‫‪ETotal‬‬
‫‪2x0,25‬‬ ‫‪N‬‬ ‫‪E‬‬ ‫‪E‬‬ ‫‪M‬‬ ‫‪2,92 1014  239‬‬
‫‪m  n .M ‬‬ ‫‪.M  lib .M  Total‬‬ ‫‪‬‬ ‫‪ 3845.97 g‬‬
‫‪NA‬‬ ‫‪NA‬‬ ‫‪E lib  N A 188, 39 1, 6 1013  6, 02 1023‬‬

‫ﺍﻟﺘﻤﺮﻳﻦ ﺍﻟﺜﺎﻧﻲ‪ 04) :‬ﻧﻘﺎﻁ(‬


‫‪ -I‬اﻟ راﺳﺔ اﻟ ( ‪9,‬ﺔ‪:‬‬
‫‪ -1‬ﻣ أ إﻧ ﻔﺎ اﻟ ﺎﻗﺔ‪:‬‬
‫‪0,25‬‬ ‫" ﺍﻟﻄﺎﻗﺔ ﻻ ﺗﺴﺘﺤﺪﺙ ﻭﻻ ﺗﺰﻭﻝ ‪ ،‬ﻓﺈﺫﺍ ﺍﻛﺘﺴﺒﺖ ﺟﻤﻠﺔ ﻣﺎ ﻁﺎﻗﺔ ﺃﻭ ﻓﻘﺪﺗﻬﺎ ﻓﺈﻥ ﻫﺬﻩ ﺍﻟﻄﺎﻗﺔ ﺗﻜﻮﻥ‬
‫ﺑﺎﻟﻀﺮﻭﺭﺓ ﻗﺪ ﺃﺧﺬﺗﻬﺎ ﻣﻦ ﺟﻤﻠﺔ )ﺃﻭ ﺟﻤﻞ( ﺃﺧﺮﻯ ﺃﻭ ﻗﺪﻣﺘﻬﺎ ﻟﻬﺎ " ‪.‬‬
‫‪y‬‬ ‫ﺎرﺗﻪ‪E f  Ei  Erecue  Ecedee :‬‬
‫‪ -2‬ﺗﻣﺛﻳﻝ ﺍﻟﻘﻭﻯ ﺍﻟﻣﻁﺑﻘﺔ ﻋﻠﻰ‬
‫‪R‬‬ ‫ﻣﺭﻛﺯﻋﻁﺎﻟﺔ ﺍﻟﺟﺳﻡ ) ‪: (S‬‬
‫)‪(S‬‬
‫‪O‬‬
‫‪A‬‬

‫‪0,50‬‬ ‫ﺎرة ﺗ ﺎرع ﻣ‬ ‫‪ -3‬ا"!ﺎد‬


‫‪P‬‬ ‫‪‬‬ ‫‪x‬‬
‫‪B‬‬ ‫ﻋ ﺎﻟﺔ اﻟ! ‪* #‬ﺎﺳ(ﻌ&ﺎل ﻣ أ‬
‫اﻻﻧ ﻔﺎ ‪:‬‬
‫ﻲ اﻷرﺿﻲ ‪.‬‬ ‫اﻟ!&ﻠﺔ – ﺟ ‪ ، - #‬وﻣ ﺟﻊ اﻟ راﺳﺔ اﻟ‬ ‫‪ -‬ﻧﻌ(‬
‫ﺑ( ‪ ;:‬ﻣ أ اﻧ ﻔﺎ اﻟ ﺎﻗﺔ ﺑ‪ 9:‬اﻟ&‪8‬ﺿﻊ اﻻﺑ( اﺋﻲ ‪ O‬وﻣ‪8‬ﺿﻊ <ﻔﻲ ‪ M‬ﻧ! ‪:‬‬

‫‪E f  E i  E recue  E cedee‬‬


‫‪ ‬‬
‫‪E C  E CO W A M P W AM R‬‬ ‫‪ ‬‬
‫‪0,25‬‬ ‫‪EC‬‬ ‫‪ E W  P   W  P   W  R ‬‬
‫‪CO‬‬ ‫‪x‬‬ ‫‪y‬‬ ‫‪AM‬‬

‫‪EC‬‬ ‫‪W  P ‬‬ ‫‪x‬‬

‫‪1‬‬
‫‪ m v 2  m  g  x  sin ‬‬
‫‪2‬‬

‫‪2‬‬
‫*ﺎﻹﺷ(ﻘﺎق *ﺎﻟ= ﺔ ﻟﻠ ﻣ‪ 9‬ﻧ! ‪:‬‬
‫‪1‬‬ ‫‪dv‬‬ ‫‪dx‬‬
‫‪ m  2 v‬‬ ‫‪ m  g   sin ‬‬
‫‪0,25‬‬ ‫‪2‬‬ ‫‪dt‬‬ ‫‪dt‬‬
‫‪dv‬‬
‫‪ g  sin   a  g  sin ‬‬
‫‪dt‬‬
‫‪ -4‬اﺳ(=(ﺎج ‪< D‬ﻌﺔ ﺣ ﺔ اﻟ! ‪: ( S ) #‬‬
‫‪0,25‬‬ ‫ﺣ ﺔ ﻣ (‪&<F‬ﺔ ﻣ( ﺎرﻋﺔ *ﺎﻧ(‪H‬ﺎم *ﺎﻧ(‪H‬ﺎم ﻷن ‪a  Cte , a v  0 :‬‬
‫‪ -II‬اﻟ راﺳﺔ اﻟ( ‪9E‬ﺔ‪:‬‬
‫‪ -1‬ﻣﺮﻛﺒﺘﺎ ﺷﻌﺎﻋﻲ ﺍﻟﺴﺮﻋﺔ ﻭﺍﻟﻤﻮﺿﻊ ‪:‬‬
‫‪ vx  g .sin  .t ‬‬
‫‪v‬‬ ‫‪ -‬ﻣ ﺎت ﺷﻌﺎع اﻟ ﻋﺔ ‪ :‬‬
‫‪‬‬ ‫‪vy  0‬‬ ‫‪‬‬
‫‪2x0,25‬‬ ‫‪‬‬ ‫‪1 2‬‬
‫‪x‬‬ ‫‪‬‬ ‫‪at ‬‬
‫‪OM ‬‬ ‫‪2‬‬
‫‪‬‬ ‫‪ -‬ﻣ ﺎت ﺷﻌﺎع اﻟ&‪8‬ﺿﻊ ‪ :‬‬
‫‪ y0 ‬‬
‫‪1‬‬
‫‪ -2‬أ‪ -‬اﺛ ﺎت أن ‪ g   sin    t 2 :‬‬
‫‪2‬‬
‫‪h‬‬
‫‪2‬‬
‫‪h  x sin ‬‬
‫‪1‬‬
‫‪h   a  t 2  sin ‬‬
‫‪0,25‬‬ ‫‪2‬‬
‫‪1‬‬
‫‪h   g  sin   t 2  sin ‬‬
‫‪2‬‬
‫‪1‬‬
‫‪h   g   sin    t 2‬‬
‫‪2‬‬

‫‪2‬‬
‫ب‪ -‬ﺣ ﺎب اﻟ ﻣ‪ 9‬اﻟ& (ﻐ ق ﻟﻘ ﻊ اﻻرﺗﻔﺎع ‪ h  1m‬ﻣ‪ 9‬أﺟﻞ اﻟ او‪N‬ﺔ ‪:   45 ‬‬
‫‪1‬‬
‫‪g   sin    t 2‬‬
‫‪2‬‬
‫‪h‬‬
‫‪2‬‬
‫‪2h‬‬
‫‪t‬‬
‫‪g   sin  ‬‬
‫‪2‬‬
‫‪0,25‬‬
‫‪2 1‬‬
‫‪t‬‬ ‫‪ 0, 63s‬‬
‫‪10  0,5‬‬
‫‪ -III‬زاو ﺔ اﻟ! ﻞ‪:‬‬

‫وﻋﻠﻰ اﻻرﺗﻔﺎع ‪: h‬‬ ‫ﻋﺔ ‪vx‬‬ ‫‪ -1‬أ‪ -‬ﺗﺄﺛ‪ :‬زاو‪N‬ﺔ ﻣ‪:‬ﻞ اﻟ& (‪ P8‬اﻟ&ﺎﺋﻞ ‪ ‬ﻋﻠﻰ اﻟ‬
‫ﺎرة اﻟ ﻋﺔ ‪ : vx‬ﻠ&ﺎ زادت زاو‪N‬ﺔ اﻟ&‪:‬ﻞ زادت اﻟ ﻋﺔ ‪ vx‬ﻋ= ﻧﻔ‪ T‬اﻟﻠ ‪H‬ﺔ‪.‬‬ ‫* ﻣ‪9‬‬
‫‪0,25‬‬
‫‪0,25‬‬ ‫ﺎرة ‪ : h‬ﻠ&ﺎ زادت زاو‪N‬ﺔ اﻟ&‪:‬ﻞ ‪ ‬زادت ‪ h‬ﻋ= ﻧﻔ‪ T‬اﻟﻠ ‪H‬ﺔ‪.‬‬ ‫* ﻣ‪9‬‬
‫ب‪ -‬أرﻓﺎق اﻟ&= =<ﺎت اﻟ <ﺎﻧ<ﺔ ‪  c  ،  b  ،  a ‬و ‪  d ‬ﻣﻊ زاو‪N‬ﺔ اﻟ&‪:‬ﻞ اﻟ&‪8‬اﻓﻘﺔ ﻟﻬﺎ‪:‬‬
‫‪0,25‬‬
‫‪  45 ‬‬ ‫‪  30 ‬‬
‫‪ a ,  c‬‬ ‫‪ b ,  d ‬‬

‫‪3‬‬
‫ﺎرة اﻟ ﻋﺔ ‪ vx‬ﺑ ﻻﻟﺔ اﻻرﺗﻔﺎع ‪: h‬‬ ‫‪ -2‬ا"!ﺎد‬
‫‪v‬‬ ‫‪x‬‬ ‫‪ g  s in   t‬‬
‫‪  s in ‬‬ ‫‪‬‬
‫‪2‬‬ ‫‪2‬‬ ‫‪2‬‬ ‫‪2‬‬
‫‪v‬‬ ‫‪x‬‬ ‫‪ g‬‬ ‫‪t‬‬
‫‪vx2‬‬
‫‪ s in  ‬‬
‫‪2‬‬ ‫‪2‬‬
‫‪t‬‬ ‫‪‬‬
‫‪g 2‬‬
‫‪0,50‬‬ ‫‪1‬‬
‫‪ g   s in    t 2‬‬
‫‪2‬‬
‫‪h ‬‬
‫‪2‬‬
‫‪1‬‬ ‫‪vx2‬‬ ‫‪1vx2‬‬
‫‪h  g ‬‬ ‫‪‬‬
‫‪2‬‬ ‫‪g2‬‬ ‫‪2 g‬‬
‫‪2‬‬
‫‪2hg  v‬‬ ‫‪x‬‬ ‫‪ v‬‬ ‫‪x‬‬ ‫‪‬‬ ‫‪2 . g .h‬‬
‫‪ ‬ﻋﻠﻰ ﻋﻼﻗﺔ اﻟ ﻋﺔ ‪* vx‬ﺎﻻرﺗﻔﺎع ‪: h‬‬ ‫‪ -3‬ﺻ<ﺎﻏﺔ ﻧ(<!ﺔ ﺗ‪8‬ﺿﺢ ﺗﺄﺛ‪ :‬زاو‪N‬ﺔ اﻟ&‪:‬ﻞ‬
‫ﻣ‪ 9‬ﻋﻼﻗﺔ اﻟ ﻋﺔ ‪* vx‬ﺎﻻرﺗﻔﺎع ‪ h‬ﻓـﺈن ﻻ ﺗﺄﺛ‪ :‬ﻟ او‪N‬ﺔ اﻟ&‪:‬ﻞ ‪. ‬‬
‫‪0,25‬‬
‫أ‪ P‬أن اﻟ ﻋﺔ ‪ vx‬ﻋ= اﻻرﺗﻔﺎع ‪ h‬ﻫﻲ ﻧﻔ ﻬﺎ ﻣﻬ&ﺎ ﺎﻧ` زاو‪N‬ﺔ اﻟ&‪:‬ﻞ ‪.‬‬
‫ﺍﻟﺘﻤﺮﻳﻦ ﺍﻟﺜﺎﻟـــﺚ ‪ 06) :‬ﻧﻘﺎﻁ(‬
‫‪ -I‬اﻟ‪I‬ﺎﻫ ة اﻟﻔ ‪ 2‬ﺎﺋ‪9‬ﺔ ‪:‬‬
‫‪ -1‬ﻋ ف اﻟ&‪bc‬ﻔﺔ‪ :‬ﻋﻨﺼﺮ ﻛﻬﺮﺑﺎﺋﻲ ﻳﺘﻜﻮﻥ ﻣﻦ ﺻﻔﻴﺤﺘﻴﻦ ﻣﻌﺪﻧﻴﺘﻴﻦ ) ﻟﺒﻮﺳﻴﻦ ( ﻳﻔﺼﻞ ﺑﻴﻨﻬﻤﺎ ﻋﺎﺯﻝ‬
‫‪0,25‬‬ ‫) ﻫﻮﺍء ‪ ،‬ﻣﺎء ‪ ،‬ﺯﻳﺖ ‪ ،( ...‬ﺗﺴﺘﻌﻤﻞ ﺍﻟﻤﻜﻤﺜﻔﺔ ﻟﺘﺨﺰﻳﻦ ﻛﻤﻴﺔ ﻣﻦ ﺍﻟﻜﻬﺮﺑﺎء ‪ ،‬ﻳﺮﻣﺰ‬
‫ﻟﻠﻤﻜﺜﻔﺎﺕ ﻓﻲ ﺍﻟﺪﺍﺭﺍﺕ ﺍﻟﻜﻬﺮﺑﺎﺋﻴﺔ ﺑﺎﻟﺮﻣﺰ ‪┤├ :‬‬
‫‪ -2‬ﺷ ح ﻣﺎ " ث ﻋﻠﻰ ﻣ (‪ f8‬ﻟ ‪8‬ﺳﻲ اﻟ&‪bc‬ﻔﺔ ‪:‬‬
‫‪0,25‬‬ ‫" ث ﺗ‪ m<bk‬ﻟﻠ‪= l‬ﺎت ﻣ‪ 9‬ﺧﻼل اﻧ(ﻘﺎل اﻹﻟ‪ (k‬وﻧﺎت ﻣ‪ 9‬ﻟ ‪8‬س إﻟﻰ آﺧ ﻋ= ﺗ ‪;:‬‬
‫ﻬ ‪o‬ﺎﺋﻲ ﺑ‪<n D 9:‬ﻪ‪.‬‬ ‫ﺗ‪8‬ﺗ‬
‫‪ -II‬اﻟ ﺗ اﻟ‪M‬ﻬ ﺎﺋﻲ ﺑ ‪ ; J‬ﻓﻲ اﻟ!‪ ,‬ﻔﺔ‪:‬‬
‫‪ -1‬رﺳ‪ #‬اﻟ ارة‪<q< ،‬ﺔ وﺻﻞ ﻻﻗ‪ p‬ﺟﻬﺎز ‪ ExAO‬ﻟ&(ﺎ*ﻌﺔ اﻟ(‪8‬ﺗ ﺑ‪ D 9:‬ﻓﻲ اﻟ&‪bc‬ﻔﺔ ‪: uC‬‬

‫‪i‬‬

‫‪i‬‬ ‫‪1‬‬ ‫‪2‬‬

‫‪uR‬‬
‫‪R‬‬
‫‪E‬‬
‫‪0,50‬‬

‫‪uc‬‬
‫‪i‬‬

‫‪4‬‬
: ‫ اﺛ ﺎت اﻟ&ﻌﺎدﻟﺔ اﻟ(ﻔﺎﺿﻠ<ﺔ‬-2
: !‫ﺗ ات ﻧ‬8(‫ن ﺟ&ﻊ اﻟ‬8‫; ﻗﺎﻧ‬: (‫ﺑ‬
u C (t )  u R (t )  E
u C (t )  R .i (t )  E
du C (t )
u C (t )  R .C E
dt
0,50 du C (t ) 1 E
 u C (t )  .........(01)
dt R .C R .C
du C (t )
 A .uC (t )  A .B ..........(02)
dt
1
A ;B  E
R .C

.  9‫ب ﺛﺎﺑ` اﻟ ﻣ‬8‫ﻞ ﻣﻘﻠ‬b&" : A


0,50
‫ﻟ‬8&‫ﺎﺋ<ﺔ ﻟﻠ‬o ‫ﻬ‬k‫ﺔ اﻟ‬ &‫ة اﻟ‬8‫ﻞ اﻟﻘ‬b&" : B
:‫ اﻟ( ﻘ; أن ﺣﻞ اﻟ&ﻌﺎدﻟﺔ اﻟ(ﻔﺎﺿﻠ<ﺔ‬-3
u C (t )  B (1  e  A t )
u C (t )  B  Be  A t ..........(03)
du C (t )
0,50  A B e  A t .............(04)
dt
A Be  A t  A  B  Be  A t   A B
A Be  A t  A B  A Be  A t  A B
00
.‫ﻭﻣﻨﻪ ﺍﻟﺤﻞ ﻣﺤﻘﻖ‬
: ‫ ﻟﻠ ارة‬ 9‫<&ﺔ ﺛﺎﺑ` اﻟ ﻣ‬s ‫ ﺗ ﯾ‬-4

0,25 .   40ms : !‫ اﻟ <ﺎن ﻧ‬9‫ﻣ‬


: ‫ﻟ‬8&‫ﺎﺋ<ﺔ ﻟﻠ‬o ‫ﻬ‬k‫ﺔ اﻟ‬ &‫ة اﻟ‬8‫ اﻟﻘ‬E ‫<&ﺔ‬s ‫ اﺳ(=(ﺎج‬-5
du C (t )
 A Be  At
dt
du (t )
t 0  C  AB
dt
0,25
duC (t )
150
B  dt   6V  E  B  6V
A 1
40 103

5
:‫ﺎﻗﺔ‬O‫ اﻟ‬-III
0,25 ‫ﺎﺋ<ﺔ‬o ‫ﺎﻗﺔ ﻬ‬D :‫ﻔﺔ‬bc&‫ ﻧﺔ *ﺎﻟ‬u&‫ <ﻌﺔ اﻟ ﺎﻗﺔ اﻟ‬D ‫ ﺗ ﯾ‬-1
:‫ﻔﺔ إﻟﻰ اﻟ=ﺎﻗﻞ اﻷوﻣﻲ‬bc&‫ اﻟ‬9‫ اﻟ ﺎدث ﻣ‬P8‫ﻞ اﻟ ﺎﻗ‬N8 (‫ اﻟ‬p&‫ ﺗ ﯾ ﻧ‬-‫ أ‬-2
0,25
We ‫ﺎﺋﻲ‬o ‫ﻞ ﻬ‬N8 ‫ ﺗ‬p&‫ﻧ‬
du C (t )
 A  u C (t )  0 : ‫ اﺛ ﺎت اﻟ&ﻌﺎدﻟﺔ اﻟ(ﻔﺎﺿﻠ<ﺔ‬-‫ب‬
dt
: !‫ﺗ ات ﻧ‬8(‫ن ﺟ&ﻊ اﻟ‬8‫; ﻗﺎﻧ‬: (‫ﺑ‬

u C (t )  u R (t )  0 i

2
u C (t )  R  i (t )  0
uR
du (t ) R
u C (t )  R  C C 0
2x0,25 dt Y1 Y1
du C (t ) 1 CC
 uC (t )  0 uc
dt R C i
du C (t )
 A  uC (t )  0 .........(05) i
dt
:‫ اﻟ( ﻘ; أن ﺣﻞ اﻟ&ﻌﺎدﻟﺔ اﻟ(ﻔﺎﺿﻠ<ﺔ‬-‫ﺟـ‬
u C (t )  Be  A t .......(06)
du C (t )
  A Be  A t ........(07)
dt
0,25 : (05) ‫( ﻓ{{{{{{{{{{{{{{{{{{{{ﻲ‬07), (06)
A B e At  A B e At  0
A B e At  A B e At  0
00
:‫ﺎﻋﺔ‬O ‫ اﻻﺳ‬-3
: q (t ) ‫ =ﺔ‬l‫ﻔﺔ واﻟ‬bc&‫ ﺳﻌﺔ اﻟ‬C ‫ﻔﺔ ﺑ ﻻﻟﺔ‬bc&‫ ﻧﺔ *ﺎﻟ‬u&‫ﺎرة اﻟ ﺎﻗﺔ اﻟ‬ ‫ (ﺎ*ﺔ‬-‫أ‬
0,25 1
EC   q 2 (t )
2C
:‫ﻞ‬cl‫ *ﺎﻟ‬v(k‫ ﺗ‬P8‫ﻞ اﻟ ﺎﻗ‬N8 (‫ اﺛ ﺎت أن اﺳ( ﺎﻋﺔ اﻟ‬-‫ب‬
P (t )  i (t ) uC (t )
dE C (t )
P (t ) 
dt
 1 
d  q 2 (t ) 
2C : ‫ﻟ ﯾ=ﺎ‬
0,25 P (t )   
dt
1 dq (t )
P (t )   2  q (t ) 
2C dt
P (t )  uC (t )  i (t )

6
‫‪ -IV‬اﺳ ﻐﻼل اﻟ‪ $‬ﺎﺋﺞ‪:‬‬
‫ﺎرة اﻻﺳ( ﺎﻋﺔ اﻷﻋ‪<&H‬ﺔ ‪ P0‬ﺑ ﻻﻟﺔ ‪ E‬و ‪: R‬‬ ‫‪ -1‬ا"!ﺎد‬
‫ﻟ ﯾ=ﺎ ‪:‬‬
‫) ‪P (t )  uC (t )  i (t‬‬
‫‪0,25‬‬
‫‪E t /‬‬
‫‪P (t )  Ee t / ‬‬ ‫‪e‬‬
‫‪R‬‬

‫‪E 0 E2‬‬
‫‪0‬‬
‫‪t  0  P0  P (0)  Ee . e ‬‬
‫‪0,25‬‬ ‫‪R‬‬ ‫‪R‬‬
‫‪2‬‬
‫‪E‬‬
‫‪ P0 ‬‬
‫‪R‬‬
‫‪ -2‬اﺳ(=(ﺎج ‪&<s‬ﺔ ‪ R‬ﻣﻘﺎوﻣﺔ اﻟ=ﺎﻗﻞ اﻷوﻣﻲ ﺛ‪ C #‬ﺳﻌﺔ اﻟ&‪bc‬ﻔﺔ ‪:‬‬
‫‪0,25‬‬ ‫‪E2‬‬ ‫‪E2‬‬ ‫‪62‬‬
‫‪P0 ‬‬ ‫‪ R‬‬ ‫‪R ‬‬ ‫‪ 1000‬‬
‫‪R‬‬ ‫‪P0‬‬ ‫‪4,5  8 103‬‬
‫‪ -‬ﺗ ﯾ ‪ C‬ﺳﻌﺔ اﻟ&‪bc‬ﻔﺔ‪:‬‬
‫‪0,25‬‬
‫‪‬‬ ‫‪40  10 3‬‬
‫‪  RC‬‬ ‫‪C ‬‬ ‫‪‬‬ ‫‪ 40  10 6 F‬‬
‫‪R‬‬ ‫‪1000‬‬
‫‪ -3‬ﺣ ﺎب ‪&<s‬ﺔ اﻟ ﺎﻗﺔ اﻟ ار‪N‬ﺔ اﻟ(ﻲ ﯾ=‪ l‬ﻫﺎ اﻟ=ﺎﻗﻞ اﻷوﻣﻲ ﻓﻲ ﻣ < ﻪ‪:‬‬
‫‪1‬‬
‫‪E C 0  C  E 2‬‬
‫‪0,25‬‬ ‫‪2‬‬
‫‪1‬‬
‫‪E C 0   40  10 6  62  7, 2 104 J‬‬
‫‪2‬‬
‫ﺍﻟﺘﻤﺮﻳﻦ ﺍﻟﺘﺠﺮﻳﺒﻲ ‪ 06) :‬ﻧﻘﺎﻁ(‬
‫‪ -I‬اﻟ ‪:T9U:‬‬
‫ﻞ ﻓ د <&<ﺎﺋﻲ ﺟ ‪<xN‬ﺎ ﺎن أو ﺷﺎرد"ﺎ *ﺎﻣ‪c‬ﺎﻧﻪ اﻛ( ﺎب‬ ‫أ‪ -‬اﻟ!(ﻠ ل اﻷﺳﺎﺳﻲ‪:‬‬
‫‪0,50‬‬ ‫ﺑ وﺗ‪8‬ن ﻫ‪ :‬روﺟ‪ H  9:‬أو أﻛ‪ b‬ﺧﻼل ﺗﻔﺎﻋﻞ <&<ﺎﺋﻲ‪.‬‬
‫ب‪ -‬ﻣﻌﺎدﻟﺔ اﻧ ﻼل اﻷﺳﺎس ﻓﻲ اﻟ&ﺎء‪:‬‬
‫‪0,50‬‬ ‫‪‬‬
‫‪C 3 H 9 N aq   H 2O ( l )  C 3H 10 N‬‬ ‫) ‪( aq‬‬ ‫) ‪ OH  (aq‬‬
‫‪ -2‬اﻟ ! ﯾ ‪:‬‬
‫أ‪ -‬اﺧ(<ﺎر اﻟ ﺟﺎﺟ<ﺎت اﻟ&=ﺎﺳ ﺔ ﻟﻌ&ﻠ<ﺔ اﻟ(& ﯾ ‪:‬‬
‫‪0,25‬‬ ‫*&ﺎ أن ﻣﻌﺎﻣﻞ اﻟ(& ﯾ ‪ F  10‬وﺣ!‪ #‬اﻟ& ﻠ‪8‬ل اﻟ&& د ‪ V1  10mL‬ﻧ‪(u‬ﺎر‬
‫اﻟ&!&‪8‬ﻋﺔ ‪ c ‬‬
‫اﻟ وﺗ‪8 8‬ل اﻟ(! ‪ N‬ﻲ ﻟﻌ&ﻠ<ﺔ اﻟ(& ﯾ ‪:‬‬ ‫ب‪ -‬أذ‬
‫ﻓﻲ ﺑ<‪. l‬‬ ‫‪ -‬ﻧ ‪<& vc‬ﺔ ﻣ‪ 9‬اﻟ& ﻠ‪8‬ل اﻟ&‬

‫‪7‬‬
‫‪&* v‬ﺎﺻﺔ <ﺎر‪N‬ﺔ اﻟ !‪. V0  1mL #‬‬ ‫‪-‬ﻧ‬
‫‪ -‬ﻧ ‪ vc‬ﻣ (‪ f8‬اﻟ&ﺎﺻﺔ ﻓﻲ ﺣ‪8‬ﺟﻠﺔ <ﺎر‪N‬ﺔ ﺳﻌ(ﻬﺎ ‪. V1  10mL‬‬
‫‪0,50‬‬
‫‪ -‬ﻧ{<‪ m‬اﻟ&ﺎء إﻟﻰ ﻣ (‪ f8‬اﻟ ‪8‬ﺟﻠﺔ ﻋﻠﻰ ﻋ ة ﻣ ات ﻣﻊ رج ﻣ(‪8‬اﺻﻞ‪.‬‬
‫‪ -‬ﻧ‪&k‬ﻞ *ﺎﻟ&ﺎء إﻟﻰ ﺧ‪ p‬اﻟ|<ﺎر‪.‬‬
‫‪ -3‬اﻟ!ﻌﺎﯾ ة ‪:‬‬

‫‪0,25‬‬ ‫أ‪ -‬ﻣ&‪ :‬ات ﺗﻔﺎﻋﻞ اﻟ&ﻌﺎﯾ ة ‪ :‬ﺳ ‪N‬ﻊ ‪ ،‬ﺗﺎم ‪ ،‬ﺣ ار‪. P‬‬
‫ب‪( -‬ﺎ*ﺔ ﻣﻌﺎدﻟﺔ ﺗﻔﺎﻋﻞ اﻟ&ﻌﺎﯾ ة ‪:‬‬
‫‪0,50‬‬ ‫) ‪C 3H 9 N (aq )  H 3O  (aq )  C 3 H 10 N  aq   H 2O( l‬‬
‫ﺟـ‪ -‬اﺣ اﺛ<ﺎت ﻧﻘ ﺔ اﻟ(‪k‬ﺎﻓ}‪:‬‬
‫‪0,50‬‬ ‫)‪E (Va E  4, 6 mL, pH E  5,8‬‬
‫* اﺳ(=(ﺎج ﺛﺎﺑ` اﻟ &‪8‬ﺿﺔ ‪ pKa‬ﻟﻠ‪=b‬ﺎﺋ<ﺔ ) ‪: (C 3 H 10 N  / C 3 H 9 N‬‬
‫ﻣ‪ 9‬ﻧﻘ ﺔ ﻧ•ﻒ اﻟ(‪k‬ﺎﻓ} ﻧ! ‪:‬‬
‫‪4, 6‬‬
‫‪0,25‬‬ ‫‪E ' : V E ' ‬‬ ‫‪ 2,3 mL‬‬
‫‪2‬‬
‫' ‪ pH E‬‬ ‫‪ pKa  4,9  2  9,8‬‬
‫د‪ -‬ﺗ ﯾ اﻟ( ‪ :‬اﻟ&‪8‬ﻟﻲ ﻟﻠ& ﻠ‪8‬ل اﻟ&& د ‪: Cb‬‬
‫ﻣ‪ 9‬ﻋﻼﻗﺔ اﻟ(‪k‬ﺎﻓ} ﻧ! ‪:‬‬
‫‪C b V b  C a V aE‬‬
‫‪0,50‬‬ ‫‪C a V aE‬‬
‫‪Cb ‬‬
‫‪Vb‬‬
‫‪0, 084  4, 6‬‬
‫‪Cb ‬‬ ‫‪ 0, 03864mol / L‬‬
‫‪10‬‬
‫* اﺳ(=(ﺎج ﺗ ‪ :‬ﺛﻼﺛﻲ ﻣ‪:b:‬ﻞ أﻣ‪* 9:‬ﺎﻟ ‪8‬ل‪:‬‬
‫‪0,25‬‬ ‫‪C0  10Cb  0, 3864mol / L‬‬
‫‪ -4‬اﻟ‪+9 $‬ﺔ‪ :‬اﻟ‪ €ul‬ﻣ•ﺎب *&(ﻼزﻣﺔ راﺋ ﺔ اﻟ &• ﻷن‪:‬‬
‫‪0,50‬‬ ‫‪C 0  0, 3864 mol / L  0, 22  10 9 mol / L‬‬
‫‪ - II‬اﻟﻌﻼج ‪:‬‬
‫‪ -1‬ﺗﻌ ‪ mN‬اﻟ‪8‬ﺳ<‪: p‬‬
‫ﻧ‪8‬ع <&<ﺎﺋﻲ ﯾ}د‪ P‬وﺟ‪8‬دﻩ ﻓﻲ اﻟ‪8‬ﺳ‪ p‬اﻟ(ﻔﺎﻋﻠﻲ إﻟﻰ ﺗ ‪N‬ﻊ اﻟ( ‪8‬ل اﻟ‪<&<k‬ﺎﺋﻲ‪.‬‬
‫‪0,25‬‬
‫* ﻧ‪8‬ع اﻟ‪8‬ﺳﺎ‪D‬ﺔ‪ :‬إﻧ ‪<&N‬ﺔ‬

‫‪0,25‬‬ ‫ﺎرة اﻟ ﻋﺔ اﻟ !&<ﺔ ﻟ(‪cl‬ﻞ أو ‪ :‬ﺛﻼﺛﻲ ﻣ‪:b:‬ﻞ أﻣ‪: OTMA 9:‬‬ ‫‪( -2‬ﺎ*ﺔ‬
‫‪d OTMA‬‬
‫‪vvol ‬‬
‫‪dt‬‬

‫‪8‬‬
=‫ ﻋ‬OTMA 9:‫ﻞ أﻣ‬:b:‫ ﺛﻼﺛﻲ ﻣ‬: ‫ﻞ أو‬cl(‫ ﺣ ﺎب اﻟ ﻋﺔ اﻟ !&<ﺔ ﻟ‬-3
: #N ‫ ﻓﻲ ﺣﺎﻟﺔ ﻞ إﻧ‬t  0 ‫ﺔ‬H ‫اﻟﻠ‬

E vvol 
 3  0   3 1010  3, 0 1010 mol.L1.min1
1  0   3
3x0,25
E1 vvol 
 5  0   3 1010  10, 0 1010 mol.L1.min1
 0,5  0  3
E2
vvol 
 0,5  0   3 1010
 7,14 1012 mol.L1.min 1
 7  0  3

p<‫ﺳ‬8‫ اﻟ‬8‫ ﻷﻧﻪ ﻫ‬E1 8‫ ﻌﻼج ﻟ&(ﻼزﻣﺔ راﺋ ﺔ اﻟ &• ﻫ‬v‫ر اﻟ&=ﺎﺳ‬8 (&‫ اﻟ‬#N ‫ اﻷﻧ‬-4
0,25 .9:‫ﻞ أﻣ‬:b:‫ ﺛﻼﺛﻲ ﻣ‬#‫ ﻓﻌﺎﻟ<ﺔ )اﻷﺳ ع( وﻣ=ﻪ "&=ﻊ ﺗ اﻛ‬b‫اﻷﻛ‬

9
‫اﻟ! ﺿـــــ ع اﻟ ﺎﻧﻲ ‪:‬‬

‫ﺍﻟﻌﻼﻣﺔ‬
‫ﻋﻧﺎﺻــﺭ ﺍﻹﺟﺎﺑــﺔ‬
‫ﻣﺟﻣﻭﻉ‬ ‫ﻣﺟﺯﺃﺓ‬
‫ﺍﻟﺘﻤﺮﻳﻦ ﺍﻷﻭﻝ‪ 04) :‬ﻧﻘﺎﻁ(‬
‫اﻟ&!&‪8‬ﻋﺔ اﻷوﻟﻰ ‪ :‬دراﺳﺔ اﻟ ﻘ‪ …8‬اﻟ‪l‬ﺎﻗ‪8‬ﻟﻲ ﻟﻠ‪N k‬ﺔ ﻓﻲ ﻣﺎﺋﻊ ‪.‬‬
‫ﻲ اﻷرﺿﻲ ‪.‬‬ ‫‪ –1‬اﻟ& ﺟﻊ اﻟ&=ﺎﺳ‪ v‬ﻟ راﺳﺔ ﺣ ﺔ اﻟ‪N k‬ﺔ ﻫ‪ 8‬اﻟ& ﺟﻊ اﻟ‬
‫اﻟﻔ ﺿ<ﺔ اﻟ&(ﻌﻠﻘﺔ *ﻪ واﻟ(ﻲ ﺗ &ﺢ ﺑ( ‪ ;:‬اﻟﻘﺎﻧ‪8‬ن اﻟ‪b‬ﺎﻧﻲ ﻟ=‪8:‬ﺗ‪ 9‬ﻻ ﺑ ان "‪8c‬ن ﻏﺎﻟ‪:‬ﻠ<ﺎ‬
‫‪2x0,25‬‬
‫ﺔ اﻟ& روﺳﺔ اﻗﻞ *‪ :bc‬ﻣ‪ 9‬دور‬ ‫وﻟ‪k‬ﻲ ﯾ( ﻘ; ذﻟ• "!‪ v‬ان ﺗ‪8k‬ن اﻟ& ة اﻟ ﻣ=<ﺔ ﻟﻠ‬
‫اﻷرض ﺣ‪8‬ل ﻧﻔ ﻬﺎ ‪.‬‬
‫‪ –2‬ﻧ‪ €‬اﻟﻘﺎﻧ‪8‬ن اﻟ‪b‬ﺎﻧﻲ ﻟ=‪8:‬ﺗ‪: 9‬‬
‫‪0,25‬‬ ‫ﻋ ﺎﻟ(ﻪ "‪v (c‬‬ ‫ﻓﻲ ﻣﻌﻠ‪ #‬ﻏﺎﻟ‪:‬ﻠﻲ اذا أﺛ ت ﻗ‪8‬ة ﺛﺎﺑ(ﺔ ﻓﻲ ﺟ ‪( #‬ﻠ(ﻪ ‪ ، m‬ﻓﺎن ﻣ‬
‫‪F  m .a‬‬ ‫ﺗ ﺎرﻋﺎ ﺛﺎﺑ(ﺎ ‪ a‬ﺣ‪: ‰:‬‬
‫‪0,25‬‬ ‫‪vL =14m/s‬‬ ‫‪ –3‬ﺗ ﯾ ‪&<s‬ﺔ اﻟ ﻋﺔ اﻟ "ﺔ ‪ :vL‬ﻣ‪ 9‬اﻟ <ﺎن ﻧ!‬
‫‪dv v L 14‬‬
‫‪a0 ‬‬ ‫‪‬‬ ‫‪‬‬ ‫‪ 10 m / s 2‬‬ ‫‪&<s‬ﺔ اﻟ( ﺎرع اﻻﺑ( اﺋﻲ ‪:‬‬
‫‪0,25‬‬ ‫‪dt‬‬ ‫‪ 1, 4‬‬
‫‪0,25‬‬ ‫ﻧ (=(ﺞ ان داﻓﻌﺔ اﺗ ﺧ&‪ :‬س ﻣﻬ&ﻠﺔ ‪.‬‬ ‫*&ﺎان ‪a0 = g = 10 m/s2 :‬‬
‫‪dv‬‬ ‫‪k‬‬
‫‪ –4‬اﺛ ﺎت ان اﻟ&ﻌﺎدﻟﺔ اﻟ(ﻔﺎﺿﻠ<ﺔ ‪  v  g :‬‬
‫'‪z‬‬ ‫‪dt‬‬ ‫‪m‬‬
‫‪f‬‬
‫‪0,25‬‬ ‫ﺗ&‪:b‬ﻞ اﻟﻘ‪ f8‬اﻟ‪u‬ﺎرﺟ<ﺔ اﻟ& ﻘﺔ ﻋﻠﻰ اﻟ! ‪: (S ) #‬‬
‫ﺑ( ‪ ;:‬اﻟﻘﺎﻧ‪8‬ن اﻟ‪b‬ﺎﻧﻲ ﻟ=‪8:‬ﺗ‪ 9‬ﻓﻲ ﻣ ﺟﻊ ﺳ ﻲ ارﺿﻲ ﻧﻌ( ﻩ ﻏﺎﻟ‪:‬ﻠ<ﺎ‬
‫‪∑ ⃗.‬‬ ‫⃗‬
‫‪P‬‬
‫‪P  f  m .a‬‬
‫‪z‬‬ ‫‪mg  kv  ma‬‬
‫‪0,25‬‬ ‫‪dv‬‬ ‫‪k‬‬
‫‪ vg‬‬ ‫وﻣ=ﻪ ‪:‬‬
‫‪dt‬‬ ‫‪m‬‬
‫‪ – 5‬ﺣ ﺎب (ﻠﺔ اﻟ‪N k‬ﺔ ‪: m‬‬
‫‪k v L‬‬ ‫‪k‬‬ ‫‪dv‬‬
‫‪m‬‬ ‫‪← 0‬‬ ‫‪ ،‬وﻣ=ﻪ ‪v L  g :‬‬ ‫ﻓﻲ اﻟ=‪H‬ﺎم اﻟ اﺋ‪8c" #‬ن ‪ 0 :‬‬
‫‪g‬‬ ‫‪m‬‬ ‫‪dt‬‬
‫‪0,25‬‬
‫‪3, 57 102 14‬‬
‫‪m‬‬ ‫‪ 0, 05 kg  50 g‬‬
‫‪10‬‬
‫اﻟ&!&‪8‬ﻋﺔ اﻟ‪b‬ﺎﻧ<ﺔ ‪ :‬دراﺳﺔ اﻟ!&ﻠﺔ اﻟ&ﻬ( ة‬
‫‪ –1‬ﺗ&‪:b‬ﻞ اﻟﻘ‪: f8‬‬

‫‪0,25‬‬

‫‪10‬‬
‫‪0,25‬‬ ‫ﺔ ﻟ< ` ﻣ(‪u‬ﺎﻣ ة ﻻن اﻟ ﻌﺔ ﺛﺎﺑ(ﺔ ‪.‬‬ ‫‪ – 2‬اﻟ‬
‫‪ – 3‬اﻟ&ﻘﺎدﯾ اﻟ&&‪ :‬ة ‪:‬‬
‫‪0,25‬‬ ‫اﻟ ور اﻟ‪Œ‬اﺗﻲ ‪T0 = 0,1×2 =0,2 s :‬‬
‫‪0,25‬‬
‫ﺳﻌﺔ اﻻﻫ( ازات ‪Xm = 6cm ← x(0) = Xm :‬‬
‫اﻟ•ﻔ ﺔ اﻻﺑ( اﺋ<ﺔ ‪ :‬اﻟ&ﻌﺎدﻟﺔ اﻟ ﻣ=<ﺔ )‪x(t) = Xmcos(ω0t + φ‬‬
‫ﻟ ﯾ=ﺎ ‪ x(0) = Xm :‬ﻧﻌ‪8‬ض ﻓﻲ اﻟ&ﻌﺎدﻟﺔ اﻟ(ﻔﺎﺿﻠ<ﺔ ﻓ=! ‪:‬‬
‫)‪ cosφ = 1←Xm = Xmcos(φ‬ﻭﻣﻨﻪ ‪φ=0‬‬
‫‪0,25‬‬
‫‪( – 4‬ﺎ*ﺔ اﻟ&ﻌﺎدﻟﺔ اﻟ ﻣ=<ﺔ ‪x(t) = Xmcos(ω0t + φ) :‬‬
‫‪x(t) = 0.06cos(2π/0.2)t‬‬
‫‪0,25‬‬
‫)‪x(t) = 0.06cos(10π.t)……..(m‬‬
‫‪ – 5‬ﻟ ﯾ=ﺎ ‪:‬‬
‫‪m‬‬
‫‪T‬‬ ‫‪0‬‬ ‫‪‬‬ ‫‪2 ‬‬
‫‪k‬‬
‫‪0,25‬‬
‫ﻭﻣﻨﻪ ‪m = 50g← m = k.T02/4π2 :‬‬
‫ﺍﻟﺘﻤﺮﻳﻦ ﺍﻟﺜﺎﻧﻲ‪ 04) :‬ﻧﻘﺎﻁ(‬
‫‪ – I‬دراﺳﺔ ﻣ&‪ :‬ات وﺷ<ﻌﺔ ‪:‬‬
‫‪ -1‬رﺳ‪ #‬اﻟ ارة وﺗ ﯾ ﺟﻬﺔ اﻟ(‪8‬ﺗ ات واﻟ(<ﺎر‪ ،‬ر‪ po‬راﺳ‪ #‬اﻻﻫ( از اﻟ ﻗ&ﻲ ﻟ&‪l‬ﺎﻫ ة‬
‫‪i‬‬ ‫‪:u b‬‬ ‫ﺑ‪ D 9:‬ﻓﻲ اﻟ‪8‬ﺷ<ﻌﺔ ‪t ‬‬ ‫اﻟ(‪8‬ﺗ‬
‫‪i‬‬
‫‪0,25‬‬ ‫‪K‬‬ ‫‪ L,r ‬‬ ‫‪ub‬‬

‫‪E‬‬
‫‪R‬‬ ‫‪uR‬‬ ‫‪ -2‬ﻣ‪ 9‬اﻟ <ﺎن ﻧ! ‪:‬‬
‫‪i‬‬
‫ﺔ اﻟ‪k‬ﻬ ‪o‬ﺎﺋ<ﺔ ﻟﻠ&‪8‬ﻟ ‪:‬‬ ‫أ‪ -‬اﻟﻘ‪8‬ة اﻟ&‬
‫‪0,25‬‬ ‫‪i‬‬
‫*&ﺎ أﻧﻪ ﻋ= ‪8c" t=0‬ن ‪ i=0‬و‪o‬ﺎﻟ(ﺎﻟﻲ ‪ ، uR=0‬وﻣ‪ 9‬ﻗﺎﻧ‪8‬ن ﺟ&ﻊ اﻟ(‪8‬ﺗ ات ﻧ! ‪:‬‬
‫‪u b  E  12V‬‬
‫ب‪&<s -‬ﺔ ﺷ ة اﻟ(<ﺎر ‪ I 0‬ﻋ= ﺑﻠ‪8‬غ اﻟ=‪H‬ﺎم اﻟ اﺋ‪: #‬‬
‫ﺗ‪8k‬ن ‪&<s‬ﺔ ﺷ ة اﻟ(<ﺎر ﻓﻲ اﻟ=‪H‬ﺎم اﻟ اﺋ‪ #‬ﻋ‪&H‬ﻰ ‪ ،‬وﻣ=ﻪ ‪:‬‬
‫‪E‬‬ ‫‪UR‬‬ ‫‪Ur‬‬
‫‪I0 ‬‬ ‫‪ max  max‬‬
‫‪R r‬‬ ‫‪R‬‬ ‫‪r‬‬
‫‪0,25‬‬
‫‪UR‬‬ ‫‪12  2‬‬
‫‪ I 0  max ‬‬ ‫‪ 0, 2 A‬‬
‫‪R‬‬ ‫‪50‬‬
‫ﺟـ ‪ -‬ﻣﻘﺎوﻣﺔ اﻟ‪8‬ﺷ<ﻌﺔ ‪: r‬‬
‫‪0,25‬‬ ‫‪U rmax‬‬ ‫‪U rmax‬‬ ‫‪2‬‬
‫‪I0 ‬‬ ‫‪r ‬‬ ‫‪‬‬ ‫‪ 10 ‬‬
‫‪r‬‬ ‫‪I0‬‬ ‫‪0, 2‬‬

‫‪11‬‬
: ‫ﺎم اﻻﻧ(ﻘﺎﻟﻲ‬H=‫ دراﺳﺔ اﻟ‬-3
: u b t  ‫ﺗ‬8(‫ اﺛ ﺎت اﻟ&ﻌﺎدﻟﺔ اﻟ(ﻔﺎﺿﻠ<ﺔ اﻟ(ﻲ " ﻘﻘﻬﺎ اﻟ‬-‫أ‬
: !‫ﺗ ات ﻧ‬8(‫ن ﺟ&ﻊ اﻟ‬8‫; ﻗﺎﻧ‬: (‫ﺑ‬
u b (t )  u R (t )  E
 u b (t )  ri (t )  E .......(01)
: !=‫( ﻓ‬01) ‫ ﻓﻲ اﻟ&ﻌﺎدﻟﺔ‬D ;(l‫ﻧ‬
du b (t ) di (t ) di (t ) 1
R 0 /   (u b (t )  ri (t )
dt dt dt L
du (t ) R u (t )
 b   (u b (t )  ri (t )  0 / i (t )  R
0,25 dt L R
du (t ) R  u (t ) 
 b  (u b (t )  r R   0 / u R (t )  E  u b (t )
dt L R 
du (t ) R  E  u b (t ) 
 b  (u b (t )  r   0
dt L R
du b (t ) R  r E r
  u b (t )  ........(02)
dt L L
: `‫اﺑ‬8b‫ﺎرة اﻟ‬ 9::‫ ﺗﻌ‬-‫ب‬
u b t   A  B  e  .t ........(03)
du b (t )
   B  e  .t .......(04)
dt
: !‫( ﻧ‬02) ‫( ﻓﻲ‬04) ‫( و‬03) ‫ض‬8‫ﻧﻌ‬
R r r E
  B  e  .t   (A  B  e  .t ) 
L L
R r R r r E
   B  e  .t  A   B  e  .t 
L L L
R r R r r E
2x0,25  B  e  .t (  ) A 
L L L
 R r
   L  0  R r 1 r E
     , A  rI 0  U rmax , B  U R max
 R  r r  E  L  R  r
A 
 L L
: ‫ﺔ‬H ‫ ﻋ= اﻟﻠ‬ub  2V ‫ اﻟ&ﻘﺎرب‬pu‫ "ﻘ ﻊ اﻟ‬t  0 ‫ﺔ‬H ‫ اﺛ ﺎت أن اﻟ&&ﺎس ﻋ= اﻟﻠ‬-‫ﺟـ‬
: ‫ "& *ﺎﻟ& أﻣﻌﺎدﻟ(ﻪ‬#<F( ‫ ﻣ‬9‫ﺎرة ﻋ‬ t=0 =‫اﻟ&&ﺎس ﻋ‬
U rmax  E
ub t   at  b / a , b E
t1  0
dub t  U rmax  E
 a  .........(05)
dt t1  0

12
: ‫ﺎ ﻟ ﯾ=ﺎ‬N H‫ﻧ‬
du b (t )
   B  e  .t ......(06)
dt
du (t ) R  r R E R E
t 0  b    B     ......(06)
dt L R r L
: !‫( ﻧ‬06) ، (05) 9‫ﻣ‬
U rmax  E R E

t1 L
0,25
r E
( E  U rmax )  L (E  )L
t1   R r
R E R E
L
 t1  
R r
: L ‫ﺷ<ﻌﺔ‬8‫* اﺳ(=(ﺎج ذاﺗ<ﺔ اﻟ‬
  10 ms : !‫ اﻟ <ﺎن ﻧ‬9‫ﻣ‬
: ‫وﻣ=ﻪ‬
0,25 L
  L   ( R  r )  0, 01(50  10)  0,6H
R r
: t   1  r  ‫ﺔ‬H ‫ر اﻷزﻣ=ﺔ ﻋ= اﻟﻠ‬8 ‫ "ﻘ ﻊ ﻣ‬t  0 ‫ﺔ‬H ‫اﺛ ﺎت أن اﻟ&&ﺎس ﻋ= اﻟﻠ‬-‫د‬
 R
: ‫ "& *ﺎﻟ& أﻣﻌﺎدﻟ(ﻪ‬#<F( ‫ ﻣ‬9‫ﺎرة ﻋ‬ t=0 =‫اﻟ&&ﺎس ﻋ‬
E
u b t   at  b / a , b E
t2
du b t  E
 a  .........(07)
dt t2
: ‫ﺎ ﻟ ﯾ=ﺎ‬N H‫ﻧ‬
du b (t )
   B  e  .t ......(08)
dt
du (t ) R  r R E R E
t 0  b    B     ......(09)
0,25 dt L R r L
: !‫( ﻧ‬09) ، (07) 9‫ﻣ‬
E R E

t2 L
L
t 2  / L   (R  r )
R
 (R  r ) r
 t2    (1  )
R R

13
: LC ‫– دراﺳﺔ اﻟ ارة‬II
0,25
. ‫ﺎﻣ‬u(‫ ﻣ‬:‫ ﺣ ﻏ‬P‫ﺎم اﻻﻫ( از دور‬H‫ – ﻧ‬1
: ‫ =ﺔ‬l‫ – اﻟ&ﻌﺎدﻟﺔ اﻟ(ﻔﺎﺿﻠ<ﺔ ﻟﻠ‬2
: !‫ﺗ ات ﻧ‬8(‫ن ﺟ&ﻊ اﻟ‬8‫; ﻗﺎﻧ‬: (‫ﺑ‬
u C (t )  u b (t )  0
q (t ) di (t ) d 2q (t )
uC (t )  ‫ و‬u b (t )  L L : ‰:‫ﺣ‬
C dt dt 2
d 2q (t ) q (t )
: !‫ ﻧ‬L ‫ *ﺎﻟﻘ &ﺔ ﻋﻠﻰ‬. L   0 : !‫• ﻧ‬N8‫*ﻌ اﻟ(ﻌ‬
dt 2 C
0,25 d 2q (t ) 1
2
 q (t )  0
dt LC
2
q (t )  Q 0 cos( t   ) : ‫ – أ – ﻟ ﯾ=ﺎ‬3
T0
dq (t ) 2 2
  Q0 sin( t   ) : !‫*ﺎﻻﺷ(ﻘﺎق ﻧ‬
dt T0 T0
d 2q (t ) 2 2
2
 ( )2Q 0 cos( t   ) : !‫*ﺎﻻﺷ(ﻘﺎق ﻧ‬
dt T0 T0
: ‫• ﻓﻲ اﻟ&ﻌﺎدﻟﺔ اﻟ(ﻔﺎﺿﻠ<ﺔ‬N8‫*ﺎﻟ(ﻌ‬
4 2 1 2 2 1 2
2
 :‫ وﻣ=ﻪ‬( )2Q0 cos( t   )  Q0 cos( t   )  0
T0 LC T0 T0 LC T0
0,25
.T 0  2 LC : ‫إذن‬
Q0  6 C  6106 C :‫ب – ﺑ<ﺎﻧ<ﺎ‬
0,25
T0  0,8 ms
0,25
q (0) Q0 : !‫ ﻧ‬t  0 : ‫ﻟ&ﺎ‬
2
Q0  Q0 cos( (0)   ) : ‫ﺎرة اﻟ ﻞ‬ ‫• ﻓﻲ‬N8‫*ﺎﻟ(ﻌ‬
T0
0,25
.   0 : ‫إذن‬ cos( )  1 : ‫وﻣ=ﻪ‬
T 0  2 LC :‫ﺟـ – ﻟ ﯾ=ﺎ‬
2 4 2
T (8 10 )
0,25 .c  0
 2.67 107 F : ‫وﻣ=ﻪ‬
4 L 4 10  0, 06
2

. t  0, 2 ms ‫ﺔ‬H ‫ﻔﺔ ﻓﻲ اﻟﻠ‬bc&‫ ﻧﺔ ﻓﻲ اﻟ‬u&‫ اﻟ ﺎﻗﺔ اﻟ‬- ‫د‬


q (0, 2 ms )  0 : !‫ﻧ‬ t  0.2ms :‫ﻟ&ﺎ‬
. ‫ﺷ<ﻌﺔ‬8‫ﻔﺔ ﻠ<ﺎ إﻟﻰ اﻟ‬bc&‫ ﻧﺔ ﻓﻲ اﻟ‬u&‫ﻞ اﻟ ﺎﻗﺔ اﻟ‬N8 ‫ ﺗ‬#‫ ﺗ‬P‫أ‬
1 1
0,25 E L (0.2 m s )   10 6  6 2 : ‫وﻣ=ﻪ‬ E L (0.2 m s )  E C (0)  CU C2 (0) : ‫وﻣ=ﻪ‬
2 2
. E L (0, 2 ms )  1.8  10 5 J

14
‫ﺍﻟﺘﻤﺮﻳﻦ ﺍﻟﺜﺎﻟﺚ‪ 06) :‬ﻧﻘﺎﻁ(‬
‫اﻟ!‪:‬ﻊ ‪ :‬ﻧ‪8‬اة ﻏ‪ :‬ﻣ (ﻘ ة ﻟﻬﺎ أﻧ‪N8‬ﺔ ﻣ‪l‬ﺎﺑﻬﺔ ﻟﻬﺎ ﻓﻲ اﻟﻌ د اﻟ‪= l‬ﻲ وﺗ‪(u‬ﻠﻒ ﻋ=ﻬﺎ‬ ‫‪-1‬اﻟ‪I$‬‬
‫‪0,25‬‬
‫ﻓﻲ اﻟﻌ د اﻟ‪(k‬ﻠﻲ ‪ ،‬و‪=c&N‬ﻬﺎ أن ﺗ(ﻔ‪ •k‬ﺗﻠﻘﺎﺋ<ﺎ ﻟ(ﻌ ﻲ ﻧ‪8‬اة ﺑ=` أﻛ‪ b‬اﺳ(ﻘ ا ار‬
‫واﺻ ار ﺟ <&ﺎت أو اﺷﻌﺎﻋﺎت ‪.‬‬
‫‪ -‬اﻟ=‪l‬ﺎ… اﻻﺷﻌﺎﻋﻲ –ﺎﻫ ة ﻋ‪8l‬اﺋ<ﺔ ا‪– P‬ﺎﻫ ة ﻏ‪ :‬ﻣ=(‪&H‬ﺔ ‪.‬‬
‫‪0,25‬‬
‫‪ -2‬ﻗﺎﻧ ﻧﺎ اﻻﻧ(ﻔﺎ\ ﻟ? د) ‪:‬‬
‫‪0,50‬‬ ‫ﻥ ‪   A‬ﻡ‪ A‬‬ ‫*اﻟﻘﺎﻧ‪8‬ن اﻷول‪ :‬ﻗﺎﻧ‪8‬ن اﻧ ﻔﺎ اﻟﻌ د اﻟ‪(k‬ﻠﻲ ‪A‬‬
‫ﻥ ‪  Z‬ﻡ‪Z‬‬ ‫*اﻟﻘﺎﻧ‪8‬ن اﻷول‪ :‬ﻗﺎﻧ‪8‬ن اﻧ ﻔﺎ اﻟﻌ د اﻟ‪(k‬ﻠﻲ ‪Z‬‬
‫‪ E -3‬ﺎ]ﺔ ﻣﻌﺎدﻟﺔ اﻟ ﻔ‪ ^M‬ﻟﻠ‪ M‬ن‪: 14‬‬
‫‪0,50‬‬ ‫‪14‬‬
‫‪C  147 N  10e‬‬
‫‪6‬‬
‫‪0,25‬‬
‫* ﻧ&‪ p‬اﻟ(ﻔ‪ •k‬ﻣ‪ 9‬ﻧ‪8‬ع ‪ ‬‬
‫‪N‬‬ ‫*ﺗ&‪:b‬ﻞ اﻟ(ﻔ‪ •k‬اﻟ ﺎدث ﻋﻠﻰ اﻟ&‪: ( N-Z ) p u‬‬

‫‪14‬‬
‫‪0,50‬‬ ‫‪8‬‬ ‫‪C‬‬
‫‪6‬‬

‫‪7‬‬ ‫‪14‬‬
‫‪7‬‬ ‫‪N‬‬

‫‪5‬‬ ‫‪6‬‬ ‫‪7‬‬ ‫‪Z‬‬

‫‪& " -4‬ﻰ اﻟ ﻣ‪ 9‬اﻟﻼزم ﻟ((=ﺎﻗ‪ €‬ﻋ د أﻧ‪N8‬ﺔ اﻟ‪8o k‬ن ‪ 14‬اﻻﺑ( اﺋ<ﺔ ﻟﻠ=•ﻒ *ﻌ ﻣ‪8‬ت اﻟ‪k‬ﺎﺋ‪9‬‬
‫‪0,25‬‬
‫ﺑ ﻣ‪ 9‬ﻧ•ﻒ اﻟﻌ& ‪.‬‬
‫ﺎرة زﻣ‪ 9‬ﻧ•ﻒ اﻟﻌ& ﺑ ﻻﻟﺔ ﺛﺎﺑ` اﻟ(ﻔ‪ •k‬اﻻﺷﻌﺎﻋﻲ ‪: ‬‬ ‫* ا"!ﺎد‬
‫ﻟ ﯾ=ﺎ ﻋﻼﻗﺔ اﻟ(=ﺎﻗ‪ €‬اﻻﺷﻌﺎﻋﻲ ‪:‬‬
‫)‪N (t )  N 0e  t .........(01‬‬
‫‪N0‬‬
‫‪t  t 1/2‬‬ ‫‪ N (t 1/2 ) ‬‬ ‫)‪.........(02‬‬
‫‪2‬‬
‫ﻧﻌ‪8‬ض )‪ (02‬ﻓﻲ )‪ (01‬ﻓ=! ‪:‬‬

‫‪N0‬‬ ‫‪1‬‬
‫‪ N 0 e  t1/2‬‬ ‫‪‬‬ ‫‪ e  t1/2‬‬
‫‪2‬‬ ‫‪2‬‬
‫‪0,50‬‬
‫‪ln 2‬‬
‫‪  ln 2  t1/ 2‬‬ ‫‪ t1/ 2 ‬‬
‫‪‬‬

‫‪15‬‬
: 9‫ﺎﺋ‬k‫ت اﻟ‬8‫ ﺣ ﺎب اﻟ& ة اﻟ ﻣ=<ﺔ اﻟ(ﻲ ﻣ{` ﻋﻠﻰ ﻣ‬-5
N 0 ( 14C )
12
 1, 2  10 12 ........(03) : ‫ﻟ ﯾ=ﺎ‬
N ( C)
N (t )( 14C )
12
 0, 25  10 12 .......(04)
N ( C)
: !‫( ﻧ‬03) ‫( ﻋﻠﻰ‬04) ‫*ﻘ &ﺔ‬
N 0 ( 14C ) N 0 ( 14C )
 4,8   4,8
N (t )( 14C ) N 0 ( 14C )  e  t
0,50
1 4,8 4,8  t 1/2
  t
 4,8  t  4,8  t  
e  ln 2
4,8  5730
t   39680 ans
ln 2
: 12‫ن‬8o k‫ﺔ اﻟ‬N8‫ ﺣ ﺎب ﻋ د أﻧ‬-‫ أ‬-6

12 m ( 12C )  N A 0,15  6, 02  10 23
0,50 N ( C) 12
  7,525  10 21 noyaux
M ( C) 12
: ‫! ة اﻟ <ﺔ‬l‫ اﻟ‬9‫ت ﻣ‬Œ‫=ﺔ اﻟ(ﻲ أﺧ‬:‫دة ﻓﻲ اﻟﻌ‬8‫ﺟ‬8&‫ اﻟ‬14 ‫ن‬8o k‫ﺔ اﻟ‬N8‫*اﺳ(=(ﺎج ﻋ د أﻧ‬
: ‫ﻟ ﯾ=ﺎ‬
N 0 ( 14C )
12
 1, 2  10 12  N 0 ( 14C )  1, 2  1012  N ( 12C )
N ( C)
0,50
 N 0 ( 14C )  1, 2  1012  7, 525  1021  9, 03  109 noyaux
: ‫ﺎ… اﻻﺷﻌﺎﻋﻲ اﻻﺑ( اﺋﻲ‬l=‫ ﺣ ﺎب اﻟ‬-‫ب‬
N 0 (14C )  ln 2
A0    N 0 ( C )  14

t 1/ 2
0,50 9, 03  109  ln 2
 A0   0, 0346 Bq
5730  365, 25  24  3600
: vlu‫* ﺗ ﯾ ﻋ& ﻗ ﻌﺔ اﻟ‬
1, 38
0,50 A (t )   0, 023 Bq : ‫ اﻟ&ﻌ <ﺎت‬9‫ﻣ‬
60
: !‫ﺎ… اﻻﺷﻌﺎﻋﻲ ﻧ‬l=‫ ﻋﻼﻗﺔ اﻟ‬9‫ﻣ‬
t1/2 A 5730 0,0346
0,50 t  ln 0    12436 ans
ln 2 A (t ) ln 2 0,023

16
‫ﺍﻟﺘﻤﺮﻳﻦ ﺍﻟﺘﺠﺮﻳﺒﻲ ‪ 06) :‬ﻧﻘﺎﻁ(‬
‫‪ –1‬اﺛ ﺎت أن اﻟ &• ) ‪ ( A‬ﺿ|<‪: m‬‬
‫‪− log‬‬ ‫ﺇﺫﺍ ﻛﺎﻥ ﺍﻟﺣﻣﺽ ﻗﻭﻳﺎ ﻳﻛﻭﻥ ﻟﺩﻳﻧﺎ ‪:‬‬

‫‪0,25‬‬ ‫‪− log‬‬ ‫)‪−(log − log 10‬‬


‫‪10‬‬
‫‪− log + 1 2.7 + 1 3.7‬‬
‫‪3.7 ≠ 2.9‬‬
‫ﻭ ﺑﺎﻟﺗﺎﻟﻲ‪ :‬ﺍﻟـﺣﻣﺽ ﺿﻌﻳﻑ‬
‫* اﻟ وﺗ‪8 8‬ل اﻟ(! ‪ N‬ﻲ ﻟ( {‪ :‬اﻟ& ﻠ‪8‬ل ) ‪: (S2‬‬

‫اﻷدوات واﻟ ﺟﺎﺟ<ﺎت‬ ‫‪-‬ذ‬


‫اﻟ&‪8‬اد اﻟ‪<&<k‬ﺎﺋ<ﺔ اﻟﻼزﻣﺔ‬ ‫‪-‬ذ‬
‫‪0,25‬‬ ‫‪N D -‬ﻘﺔ اﻟﻌ&ﻞ ‪:‬‬
‫ﻟﺩﻳﻧﺎ ﻣﻌﺎﻣﻝ ﺍﻟﺗـﻣﺩﻳﺩ ‪10‬‬
‫ﻧﺄﺧﺫ ﺑﻭﺍﺳﻁﺔ ﻣﺎﺻﺔ ﺣـﺟﻣﺎ ﻣﻥ ﺍﻟـﻣﺣﻠﻭﻝ ) ( ﻭﻧﺿﻌﻪ ﻓﻲ‬
‫ﺣﻭﺟﻠﺔ ﻋﻳﺎﺭﻳﺔ‬
‫ﻭﻧﺿﻳﻑ ﻟﻪ ﺍﻟـﻣﺎء ﺍﻟـﻣﻘﻁﺭ ﺇﻟﻰ ﺃﻥ ﻳﺻﺑﺢ ﺣـﺟﻣﻪ ‪ ,10‬ﺛﻡ ﻧـﺭﺝ ﻟﻧﺟﻌﻝ‬
‫ﺍﻟـﻣﺣﻠﻭﻝ ) ( ﻣﺗﺟﺎﻧﺳﺎ‪.‬‬
‫‪( – 2‬ﺎ*ﺔ ﻣﻌﺎدﻟﺔ ﺗﻔﺎﻋﻞ اﻟ &• ﻣﻊ اﻟ&ﺎء ﻓﻲ اﻟ& ﻠ‪8‬ل )‪: (S1‬‬
‫‪0,25‬‬
‫) ‪C n H 2n 1  COOH (aq )  H 2O(l )  C n H 2n 1 COO (aq )  H 3O (aq‬‬

‫* ﺣ ﺎب اﻟ( ‪ :‬اﻟ&‪8‬ﻟﻲ ﻟﻠ& ﻠ‪8‬ل )‪: (S1‬‬


‫‪2‬‬
‫‪H 3O  f  Cn H 2n 1COO  f‬‬ ‫‪H 3O  f‬‬
‫‪Ka  ‬‬ ‫‪ ‬‬
‫‪C n H 2n 1COOH f‬‬ ‫‪c1  H 3O  f‬‬
‫‪2‬‬
‫‪ H 3O  f  K a  H 3O  f  c1  K a  0‬‬

‫) ‪H 3O  f ( K a  H 3O  f ) 10 pH (K a 10 pH‬‬


‫‪c1  ‬‬ ‫‪‬‬
‫‪Ka‬‬ ‫‪Ka‬‬
‫‪0,25‬‬
‫) ‪102,7 (1,26 105 102,7‬‬
‫‪c1 ‬‬ ‫‪ 0,316 mol / L‬‬
‫‪1,26 105‬‬

‫‪:‬‬ ‫)‪( A‬‬ ‫‪ –3‬ا"!ﺎد اﻟ•<ﻐﺔ اﻟ&!&ﻠﺔ ﻟﻠ &•‬


‫‪C n H 2 nO 2‬‬ ‫* اﻟ•<ﻐﺔ اﻟﻌﺎﻣﺔ ﻟﻠ &• ‪:‬‬
‫* ﺣ ﺎب اﻟ‪(k‬ﻠﺔ اﻟ&‪8‬ﻟ<ﺔ ﻟﻠ &• ‪:‬‬
‫‪m‬‬ ‫‪m‬‬ ‫‪4,67‬‬
‫‪n  c1 V ‬‬ ‫‪M ‬‬ ‫‪‬‬ ‫‪ 74 g / mol‬‬
‫‪M‬‬ ‫‪c1 V 0,316  0,2‬‬
‫‪0,25‬‬ ‫‪74  32‬‬
‫‪M  14n  32  74  n ‬‬ ‫وﻣ=ﻪ ﻧ! ‪ 3 :‬‬
‫‪14‬‬
‫‪C 3H 6O2‬‬ ‫وﻣ=ﻪ اﻟ•<ﻐﺔ اﻟ&!&ﻠﺔ ﻫﻲ‪:‬‬

‫‪17‬‬
‫اﻟ•<ﻐﺔ ﻧ•ﻒ اﻟ&ﻔ•ﻠﺔ ﻟﻠ &• ‪:‬‬
‫‪0,25‬‬ ‫‪CH 3 CH 2 COOH‬‬
‫‪acide propanoic‬‬
‫‪  II‬اﻟﻔ ج اﻟ ﺎﻧﻲ‪ :‬ﻣ ا` ﺔ ﺗﻔﺎﻋﻞ اﻟ(!_ اﻟ‪ E M‬ﻠﻲ ﻣﻊ ‪ (E‬ل‪.‬‬

‫‪0,25‬‬ ‫‪ –1‬دور اﻟ( ‪ 9:u‬اﻟ& ﺗ ‪ :‬ﺗ ‪N‬ﻊ اﻟ(ﻔﺎﻋﻞ ‪ ،‬ﺣ‪ ‰:‬ﺗ(‪bk‬ﻒ اﻷ*‪ u‬ة اﻟ&(‪cl‬ﻠﺔ وارﺟﺎﻋﻬﺎ‬
‫ﻟﻠ& ‪N‬ﺞ و‪o‬ﺎﻟ(ﺎﻟﻲ اﻟ ﻔﺎ ﻋﻠﻰ &<ﺔ ﻣﺎدة اﻷﻧ‪8‬اع اﻟ‪<&<k‬ﺎﺋ<ﺔ ﻓﻲ اﻟ& ‪N‬ﺞ ‪.‬‬
‫‪0,25‬‬ ‫ﺗ ‪N‬ﻊ اﻟ(ﻔﺎﻋﻞ ‪.‬‬ ‫* اﻟﻔﺎﺋ ة ﻣ‪ 9‬إﺿﺎﻓﺔ ﻗ ات ﻣ‪ 9‬ﺣ&• اﻟ‪ `N k‬اﻟ&‬
‫‪ –2‬اﻟﻔﺎﺋ ة ﻣ‪ 9‬إﺿﺎﻓﺔ ﻣ ﻠ‪8‬ل ﻠ‪8‬ر اﻟ•‪8‬دﯾ‪8‬م ﻫ‪ 8‬ﻋ ل اﻷﺳ( اﻟ=ﺎﺗﺞ ﻷﻧﻪ ﻻ ﯾ= ﻞ ﻓﻲ‬
‫‪0,25‬‬ ‫اﻟ&ﺎء اﻟ&ﺎﻟﺢ ‪cl<n‬ﻞ ‪ D‬ﻘﺔ "&‪ 9c‬ﻓ•ﻠﻬﺎ * ﻬ‪8‬ﻟﺔ ‪.‬‬
‫‪( –3‬ﺎ*ﺔ ﻣﻌﺎدﻟﺔ ﺗﻔﺎﻋﻞ اﻷﺳ( ة ‪:‬‬
‫‪0,25‬‬
‫‪CH 3  CH 2  COOH  C 3H 7OH  CH 3  CH 2  COOC 3H 7  H 2O‬‬
‫‪0,25‬‬
‫*ﺧ•ﺎﺋ‪ €‬اﻟ(ﻔﺎﻋﻞ ‪ :‬ﻣ ود ‪ ،‬ﻻ ﺣ ار‪ ، P‬ﻋ‪8c‬س ‪ * ،‬ﻲء‬
‫‪ –4‬ﺣ ﺎب ﺛﺎﺑ` ﺗ‪8‬ازن اﻟ(ﻔﺎﻋﻞ ‪:‬‬

‫‪K‬‬
‫‪‬‬‫‪C 6H12O2   H 2O ‬‬
‫‪‬‬
‫‪X f2‬‬
‫)‪.........(01‬‬
‫) ‪C3H 6O2  C3H 8O  (0,2  X f )  (0,3  X f‬‬
‫اﻟ ‪ ‰‬ﻋ‪&<s 9‬ﺔ اﻟ(ﻘ م اﻟ=ﻬﺎﺋﻲ ‪:‬‬
‫) ‪m f (aster‬‬
‫‪0,25‬‬ ‫‪X f  n f (aster ) ‬‬ ‫‪/ M  116 g / mol‬‬
‫‪M‬‬
‫‪16, 47‬‬
‫‪Xf ‬‬ ‫‪ 0,142 mol‬‬
‫‪116‬‬
‫*ﺎﻟ(ﻌ‪ •N8‬ﻓﻲ )‪: (01‬‬
‫‪(0,142)2‬‬
‫‪K‬‬ ‫‪ 2,25‬‬
‫‪0,25‬‬ ‫)‪(0,2  0,142)  (0,3  0,142‬‬
‫‪K  2,25‬‬ ‫* ﻧ (=(ﺞ أن ﺻ=ﻒ اﻟ‪8 k‬ل اﻟ& (ﻌ&ﻞ ﺛﺎﻧ‪ P8‬ﻷن‪:‬‬
‫* ﺻ<ﻐ(ﻪ ﻧ•ﻒ اﻟ&ﻔ•ﻠﺔ ‪:‬‬
‫‪0,25‬‬ ‫‪CH 3 CHOH CH 3‬‬
‫‪propane 2 ol‬‬
‫‪ –5‬ﺣ ﺎب ﻣ دود اﻟ(ﻔﺎﻋﻞ ‪:‬‬
‫‪0,25‬‬ ‫‪Xf‬‬ ‫‪0,142‬‬
‫‪r% ‬‬ ‫‪‬‬ ‫‪ 71%‬‬
‫‪X max 0,2‬‬
‫* "&‪ 9c‬رﻓﻊ اﻟ& دود *ﻌ ة ‪ D‬ق أﻫ&ﻬﺎ اﻟ& (ﻌ&ﻠﺔ ﻓﻲ ﻫ‪Œ‬ا اﻟ(ﻔﺎﻋﻞ واﻟ&(&‪b‬ﻠﺔ ﻓﻲ ‪:‬‬
‫…‪ - 1‬ﻣﻔﺎﻋﻠﺔ ﻣ ‪N‬ﺞ اﺑ( اﺋﻲ ﻏ‪ :‬ﻣ( ﺎو‪ P‬ﻓﻲ &<ﺔ اﻟ&ﺎدة ‪.‬‬
‫‪0,25‬‬ ‫…‪ - 2‬ﻧ ع اﻷﺳ( اﻟ=ﺎﺗﺞ *ﺎﻟ(ﻘ ‪ :‬ﺍﻟﻣﺟﺯﺃ ﻳﺷﺭﻁ ﺃﻥ ﻳﻛﻭﻥ ﻟﻸﺳﺗﺭ ﺃﺻﻐﺭ ﺩﺭﺟﺔ‬
‫ﻏﻠﻳﺎﻥ ﺑﺎﻟﻧﺳﺑﺔ ﻟﻸﻧﻭﺍﻉ ﺍﻟﻛﻳﻣﻳﺎﺋﻳﺔ ﺍﻷﺧﺭﻯ )ﻛـﺣﻭﻝ – ﺣﻣﺽ – ﻣﺎء(‪.‬‬

‫‪18‬‬
‫اﻟ? دﯾ م‪.‬‬ ‫اﻟ‪$‬ﺎﺗﺞ ﻣﻊ ﻣ(ﻠ ل ﻫ رو‪E‬‬ ‫‪ III‬اﻟﻔ ج اﻟ ﺎﻟ‪ :a‬دراﺳﺔ ﺗﻔﺎﻋﻞ اﻷﺳ‬
‫‪ –1‬اﺳ‪ #‬اﻟ(ﻔﺎﻋﻞ اﻟ ﺎدث ﺑ‪ 9:‬اﻷﺳ( وﻫ‪ :‬رو ‪ :‬اﻟ•‪8‬دﯾ‪8‬م ‪:‬‬
‫‪0,25‬‬
‫ﺗﻔﺎﻋﻞ اﻟ(• ‪9‬‬
‫* (ﺎ*ﺔ ﻣﻌﺎدﻟﺔ اﻟ(ﻔﺎﻋﻞ اﻟ ﺎدث ‪:‬‬
‫) ‪CH 3  CH 2  COOC 3H 7 ( l )  (Na   OH  )(aq )  ( Na  (aq )  CH 3  CH 2  COO  (aq ) )  C 3H 7OH (aq‬‬
‫‪0,25‬‬
‫* ﺧ•ﺎﺋ‪ €‬ﺗﻔﺎﻋﻞ اﻟ(• ‪ : 9‬ﺗﺎم ‪ * ،‬ﻲء ‪ ،‬ﺣ ار‪P‬‬
‫‪–2‬ﺟ ول ﺗﻘ م اﻟ(ﻔﺎﻋﻞ ‪:‬‬
‫ﺍﻟﻤﻌﺎﺩﻟﺔ‬
‫‪‬‬ ‫‪‬‬ ‫‪‬‬
‫‪CH 3  CH 2  COOC 3 H 7 ( l )  (Na  OH )( aq )  (Na‬‬ ‫) ‪(aq‬‬ ‫) ‪ CH 3  CH 2  COO  (aq ) )  C 3 H 7OH (aq‬‬

‫‪0,25‬‬ ‫ﺍﻟﺤﺎﻟﺔ‬ ‫)‪x(mol‬‬


‫ﻛﻤﻴـــــﺎﺕ ﺍﻟﻤـــــﺎﺩﺓ ﺑﺎﻟﻤـــــﻮﻝ‬
‫‪t=0‬‬ ‫‪0‬‬ ‫‪n0‬‬ ‫‪n0‬‬ ‫‪0‬‬ ‫‪0‬‬
‫‪t‬‬ ‫‪x‬‬
‫‪n0 - x‬‬ ‫‪n0 - x‬‬ ‫‪x‬‬ ‫‪x‬‬
‫‪tf‬‬ ‫‪x‬‬ ‫‪n0 - Xf‬‬ ‫‪C0V- Xf‬‬ ‫‪Xf‬‬ ‫‪Xf‬‬
‫‪ –3‬ﻣ(ﺎ*ﻌﺔ ﺗ ‪8‬ر اﻟ(ﻔﺎﻋﻞ *‪<F‬ﺎس اﻟ=ﺎﻗﻠ<ﺔ اﻟ=‪< 8‬ﺔ ﻟﻠ& ‪N‬ﺞ ‪:‬‬

‫‪0,25‬‬ ‫ﺃ– ﻗﻳﻣﺔ ﺍﻟﻧﺎﻗﻠﻳﺔ ﺍﻟﻧﻭﻋﻳﺔ ) ‪ ( 0‬ﻟﻠﻣﺯﻳﺞ ﺍﻟﺗﻔﺎﻋﻠﻲ ﺍﻻﺑﺗﺩﺍﺋﻲ ‪:‬‬


‫‪%‬‬
‫‪σ#‬‬ ‫‪2.5 .‬‬ ‫ﻭ ﺑﺎﻟﺗﺎﻟﻲ ‪:‬‬ ‫ﻗﺑﻝ ﺑﺩء ﺍﻟﺗﻔﺎﻋﻝ ﻳﻛﻭﻥ ‪! 0 :‬‬
‫ﺏ – ﻗﻳﻣﺔ ﺍﻟﺗﻘﺩﻡ ﺍﻷﻋﻅﻣﻲ ‪: X max‬‬
‫‪0,25‬‬
‫('&‬
‫‪10‬‬ ‫*)‬
‫ﺟـ– ﻗﻳﻣﺔ ﺍﻟﻧﺎﻗﻠﻳﺔ ﺍﻟﻧﻭﻋﻳﺔ ﻓﻲ ﻧﻬﺎﻳﺔ ﺍﻟﺗﻔﺎﻋﻝ‪:‬‬
‫‪0,25‬‬
‫‪+, 1.7 × 0.5 0.85 . %‬‬
‫ﺣﺳﺎﺏ ‪ : C H COO‬ﻧﺧﺗﺻﺭ ‪ / /0 11%‬ﺑــ ‪2%‬‬ ‫‪n‬‬ ‫‪2 n1‬‬
‫‪‬‬

‫‪08‬‬
‫‪+,‬‬ ‫‪34(5 67‬‬ ‫‪+ 39: 62% 8‬‬
‫ﺍﻟﺗﻔﺎﻋﻝ ﺗﺎﻡ ﻣﻌﻧﺎﻩ ﺍﺧﺗﻔﺎء ‪ 1 %‬ﻭﺍﻷﺳﺗﺭ‪:‬‬
‫‪;#‬‬ ‫‪;#‬‬
‫‪+,‬‬ ‫‪34(5 + 39:‬‬
‫‪;#‬‬
‫‪+,‬‬ ‫) ‪(34(5 + 39:‬‬
‫>=×<‬
‫‪34(5 + 39:‬‬ ‫ﻓﺈﻥ ‪:‬‬ ‫‪;#‬‬ ‫('!‬ ‫ﺑﻣﺎ ﺃﻥ ‪:‬‬
‫?‪/‬‬
‫‪100 × 10%@ × 0.85‬‬ ‫‪%A‬‬
‫‪3‬‬ ‫‪9‬‬ ‫‪: + 5 × 10‬‬
‫‪0,25‬‬ ‫‪10 × 10%A‬‬
‫‪39: 8.5 × 10%A − 5 × 10%A‬‬
‫‪39: 3.5 × 10%A . . )*%‬‬
‫‪ – 4‬ﻓﻲ ﺍﻟﻠﺣﻅﺔ ‪ t  8mn‬ﻛﺎﻧﺕ ﺍﻟﻧﺎﻗﻠﻳﺔ ﺍﻟﻧﻭﻋﻳﺔ ﻟﻠﻣﺯﻳﺞ ‪.   1.68S .m 1‬‬
‫>=‪=? 0‬‬
‫‪σ‬‬ ‫ﻗﻳﻣﺔ ﺯﻣﻥ ﻧﺻﻑ ﺍﻟﺗﻔﺎﻋﻝ ‪ : t1 2‬ﻗﻳﻣﺔ ﺍﻟﻧﺎﻗﻠﻳﺔ ﺍﻟﻧﻭﻋﻳﺔ ﻋﻧﺩ ‪ t1 2‬ﺗﻛﺗﺏ ﺑﺎﻟﺷﻛﻝ‪:‬‬
‫‪2.5 + 0.85‬‬
‫‪0.25‬‬ ‫‪σ‬‬ ‫‪1.68 . %‬‬
‫‪2‬‬
‫‪CD‬‬ ‫;‪8 E‬‬ ‫ﺑﺎﻻﺳﻘﺎﻁ ﻋﻠﻰ ﺍﻟﺑﻳﺎﻥ ﻧﺟﺩ ‪::‬‬

‫‪19‬‬
‫الجميورية الجزائرية الديمقراطية الشعبية‬
‫و ازرة التربية الوطنية‬
‫مديرية التربية لوالية ورڤـــمة‬ ‫امتحان بكالوريا التجريبي التعميم الثانوي‬
‫دورة ‪ :‬ماي ‪2023‬‬ ‫الشعبة ‪ :‬رياضيات ‪ ،‬تقني رياضي‬
‫المدة ‪ 04 :‬سا و ‪ 30‬د‬ ‫اختبار في مادة ‪ :‬العـموم الفيزيائية‬

‫عمى المترشح ان يختار أحد الموضوعين اآلتيين ‪:‬‬


‫الموضوع األول‬
‫يحتوي الموضوع األول عمى ‪ 44‬صفحات (من الصفحة ‪ 1‬من ‪ 8‬إلى الصفحة ‪ 4‬من ‪)8‬‬
‫الجزء األول‪ 13( :‬نقاط)‬
‫التمرين األول‪ 44( :‬نقاط)‬
‫‪ -I‬دراسة قوانين كيبمر‪ :‬مونديال قطر ‪ 2222‬والذي شاىده الماليين عبر شاشات التمفاز واالنترنت بتقنيات عالية الجودة‬
‫وذلك بفضل القمر االصطناعي سهيل سات ‪ ،2‬الذي نعتبره نقطة مادية ) ( يتحرك وفق مدار إىميميجي حول األرض‬
‫‪،‬و‬ ‫) ( بعده عن سطح األرض يتغير بين القيمتين‬
‫و األوج‬ ‫‪ -1‬مثل بمخطط مدار) ( حول ) ( موضحا عميو كل من ) ( و) ( و النقطتين‪ :‬الحضيض‬
‫بالنسبة ليذا المدار ؟‬ ‫‪ -2‬ماذا يمثل مركز األرض‬
‫لمدار) (‬ ‫‪ -3‬استنتج طول المحور الكبير‬
‫‪ -4‬بين أن حركة ) ( غير منتظمة‪.‬‬
‫دراسة الحركة الدائرية المنتظمة ‪ :‬نعتبر مدار القمر ) ( حول االرض) ( دائري نصف قطره‬ ‫‪-II‬‬
‫‪ -1‬حدد المرجع الغاليمي المناسب لدراسة حركة ) (‪.‬‬
‫‪.‬‬ ‫و‬ ‫و‬ ‫و‬ ‫⃗ التي تجذب بيا االرض) ( القمر ) ( ثم أكتب عبارتيا بداللة‬ ‫‪⁄‬‬ ‫‪ -2‬مثل شعاع القوة‬
‫‪ -3‬بتطبيق القانون الثاني لنيوتن ‪:‬‬
‫أ‪ -‬بين أن حركة ) ( دائرية منتظمة حول ) ( ‪.‬‬
‫‪.‬‬ ‫‪،‬‬ ‫لـ ) ( بداللة كتمة االرض‬ ‫ب‪ -‬جد عبارة مربع السرعة‬
‫عن سطح األرض اوجد ‪:‬‬ ‫‪ -4‬عندما يدور ) ( عمى ارتفاع قدره‬
‫‪.‬‬ ‫أ‪ -‬نصف القطر‬
‫لـمقمر) (‪.‬‬ ‫ب‪ -‬السرعة‬
‫‪.‬‬ ‫ج‪ -‬الدور المداري‬
‫د‪ -‬ما ىي طبيعة ىذا القمر االصطناعي ‪ ،‬عمل‬
‫كتمة األرض‬ ‫ثابت الجذب العام‬ ‫نصف لقطر األرض‬ ‫دور االرض‬
‫المعطيات‬

‫صفحة ‪ 1‬من ‪8‬‬


‫امتحان بكالوريا التجريبي في مادة ‪ :‬العموم الفيزيائية ‪ //‬الشعبة ‪ :‬رياضيات ‪ ،‬تقني رياضي ‪2423 //‬‬

‫التمرين الثاني‪ 46( :‬نقاط)‬


‫المعطيات‪:‬‬
‫من الماء المقطر‬ ‫في‬ ‫نحل منو‬ ‫) (‬
‫‪ -I‬بغرض تحضير محمول ) ( لغاز الميثيل أمين‬
‫لممحمول ) (‪ ،‬عمما أن الحجم المولي في شروط التجربة ىو‬ ‫‪ -1‬احسب التركيز المولي‬
‫‪ -2‬اكتب المعادلة الكيميائية لمتفاعل المنمذج لمتحول الكيميائي الحاصل‬
‫‪.‬‬ ‫أعطى القيمة‬ ‫المحمول ) ( في الدرجة‬ ‫‪ -3‬ان قياس‬
‫أ‪ -‬انشئ جدوال لتقدم التفاعل‬
‫ب‪-‬احسب النسبة النيائية لمتقدم ‪ ، τ‬ماذا تستنتج ؟‬
‫وتركيزه المولي‬ ‫كمف األستاذ في حصة لألعمال المخبرية فوج من التالميذ لتحضير محمول ) ( حجمو‬
‫انطالقا من المحمول ) (‬
‫‪ -4‬ما ىي الخطوات العممية المتبعة لتحضير المحمول ) (‬
‫𝜏 لتقدم التفاعل‪.‬‬ ‫‪ ،‬احسب قيمة النسبة النيائية‬ ‫‪ -5‬ان قيمة المحمول( ) المحضر تساوي‬

‫‪ -6‬ما تأثير الحالة االبتدائية لمجممة عمى النسبة النيائية لتقدم التفاعل ‪τ‬‬
‫(‬ ‫لمثنائية )‬ ‫‪ -7‬احسب قيمة ثابت الحموضة‬
‫تركيزه المولي‬ ‫من محمول مائي لـ ميثيل أمين‬ ‫مترية حجم‬ ‫‪ -II‬نحقق معايرة‬
‫‪.‬يمثل الشكل (‪)1‬‬ ‫( تركيزه المولي‬ ‫)‬ ‫بواسطة محمول لحمض كمور الماء‬
‫لمحمض المضاف ‪.‬‬ ‫الوسط التفاعمي بدالل الحجم‬ ‫منحنى تطور‬
‫‪ -1‬اكتب معادلة تفاعل المعايرة‪.‬‬
‫الشكل (‪)1‬‬ ‫‪.‬‬ ‫‪ -2‬عين احداثيات نقطة التكافؤ‪ ،‬واستنتج التركيز‬
‫‪ -3‬باالعتماد عمى البيان تأكد من قيمة‬
‫(‪ ،‬مع التوضيح‪.‬‬ ‫لمثنائية )‬
‫‪ -4‬انشئ جدول تقدم التفاعل‪.‬‬
‫[‬ ‫]‬
‫و‬ ‫‪,‬‬ ‫بداللة‪:‬‬ ‫‪ -5‬عبر عن النسبة‬
‫[‬ ‫]‬

‫‪.‬‬ ‫و الـ‬ ‫ثم عبر عنيا بداللة الـ‬


‫عند التكافؤ ‪ ،‬واحسب نسبة‬ ‫‪ -6‬استنتج قيمة‬
‫التقدم النيائي 𝜏 لتفاعل المعايرة عند التكافؤ ‪ ،‬ماذا تستنتج فيما يخص تفاعل المعايرة‪.‬‬

‫صفحة ‪ 2‬من ‪8‬‬


‫امتحان بكالوريا التجريبي في مادة ‪ :‬العموم الفيزيائية ‪ //‬الشعبة ‪ :‬رياضيات ‪ ،‬تقني رياضي ‪2423 //‬‬

‫التمرين الثالث‪ 44( :‬نقاط)‬


‫أو النجم العظيم الشأن‪ ،‬يستمد طاقتو اليائمة من سمسمة تفاعالت اندماج العناصر‬ ‫ستَِع ُر األعظم‬
‫الم ْ‬
‫ُ‬
‫والتي تتم في درجات ح اررة عالية جدا نحو‬ ‫الخفيفة فيما يعرف بدورة ‪ :‬الكربون‪ -‬نيتروجين‪ -‬أكسجين‬
‫(‪ 622‬مميون درجة مئوية)‪ ،‬نمخص التفاعالت في المعادلة التالية‪:‬‬

‫)𝑽𝑬𝑴( 𝑬‬ ‫الشكل (‪)3‬‬ ‫الشكل (‪ )3‬مخطط الحصيمة الطاقوية لمتفاعل السابق‬


‫‪ -1‬لماذا نحتاج طاقة كبيرة لدمج األنوية‬
‫‪ -2‬جد قيمة كل من‬
‫𝑝𝑥‬ ‫𝑛𝑦‬
‫𝐸‬ ‫‪ ،‬احسب قيمتيا‬ ‫‪ -3‬أ‪ -‬ماذا تمثل الطاقة‬
‫‪ ،‬ماذا تمثل‬ ‫ب‪ -‬استنتح‬
‫𝐸‬ ‫𝐸‬ ‫ت ‪ -‬احسة كتلة نواة‬
‫‪ -4‬أ‪ -‬احسب‬
‫‪ ،‬ماذا تمثل‬ ‫ب‪ -‬استنتح‬
‫𝑁‬ ‫𝐻‬ ‫‪ -5‬اوخد الطاقة المحررة من تفاعل االندماج‬
‫𝐸‬
‫‪ -6‬خالل مدة زمنية تتحرر من تفاعل االندماج طاقة‬
‫𝐸‬ ‫𝐴‬ ‫) (‬
‫قدرىا‬
‫𝐶‬ ‫𝑋𝑍‬
‫𝐸‬ ‫‪ -‬احسب عدد تفاعالت االندماج الالزمة لذلك‬
‫وكتمة النيتروجين‬ ‫‪ -7‬اوجد كتمة الييدروجين‬
‫المستيمكتين خالل ىذه التفاعالت‬

‫(‬ ‫)‬ ‫(‬ ‫)‬ ‫المعطيات‬

‫(‬ ‫)‬ ‫(‬ ‫)‬

‫الجزء الثاني ‪ 46( :‬نقاط)‬


‫الشكل (‪)3‬‬ ‫التمرين التجريبي‪ 46( :‬نقاط)‬
‫نحقق التركيب التجريبي الموضح في الشكل (‪ )3‬المتكون من‪:‬‬
‫‪.‬‬ ‫‪ -‬مولد توتر ثابت قوتو المحركة الكيربائية‬

‫و مقاومتيا الداخمية ‪.‬‬ ‫‪ -‬وشيعة حقيقية ) ( ذاتيتيا‬

‫‪.‬‬ ‫‪ -‬ناقل أومي مقاومتو‬

‫و أسالك التوصيل ‪ ،‬صمام ثنائي‬ ‫‪ -‬قاطعة كيربائية‬

‫صفحة ‪ 3‬من ‪8‬‬


‫امتحان بكالوريا التجريبي في مادة ‪ :‬العموم الفيزيائية ‪ //‬الشعبة ‪ :‬رياضيات ‪ ،‬تقني رياضي ‪2423 //‬‬

‫نغمق القاطعة‬ ‫‪ -I‬عند المحظة‬


‫‪ -1‬بتطبيق قانون جمع التوترات بين أن المعادلة التفاضمية لتطور شدة التيار الكيربائي ) ( تكتب من الشكل‬
‫) (‬
‫) (‬
‫ثابتان تطمب عبارة كل منيما بداللة مميزات الدارة ‪.‬‬ ‫و‬ ‫حيث‬
‫بداللة شدة التيار ‪ .‬اعتمادا عمى‬ ‫‪ -2‬نمثل في الشكل (‪ )4‬تغيرات‬
‫الشكل (‪)4‬‬
‫البيان جد ‪:‬‬
‫وقيمة ثابت الزمن 𝜏‪.‬‬ ‫أ‪ -‬قيمة ذاتية الوشيعة‬
‫ب‪ -‬مقدار مقاومة الوشيعة ‪.‬‬
‫‪ ،‬ثم تأكد من قيمتو حسابيا‪.‬‬ ‫ج‪ -‬شدة التيار األعظمي‬
‫في الوشيعة‬ ‫د‪ -‬احسب قيمة الطاقة األعظمية‬
‫‪ ،‬نغمق القاطعة ثم‬ ‫‪ -II‬نعيد نفس التجربة السابقة ونستبدل الوشيعة الحقيقية ( ) بوشيعة مثالية ( ) ذاتيتيا‬
‫نفتحيا ونعتبره مبدأ جديد لألزمنة ‪:‬‬ ‫بعد مدة وفي لحظة‬
‫بين طرفي الوشيعة من الشكل ‪:‬‬ ‫‪ -1‬بين أن المعادلة التفاضمية لتطور التوتر ) (‬
‫) (‬
‫) (‬

‫حال ليا ‪.‬‬ ‫) (‬ ‫‪ -2‬بين أن المعادلة التفاضمية السابقة تقبل العبارة الزمنية التالية‪:‬‬
‫‪.‬‬ ‫‪ -3‬ما دور الصمام الثنائي‬
‫‪ -4‬بواسطة راسم االىتزاز الميبطي ذي الذاكرة تمكنا من مشاىدة المنحنى البياني الموضح في الشكل (‪)5‬‬
‫أ‪ -‬اوجد سمم الرسم لمحور التراتيب‬
‫ثم استنتج قيمتو‬ ‫ب‪ -‬جد عبارة شدة التيار األعظمي‬
‫ج‪ -‬استنتج قيمة ثابت الزمن 𝜏 ‪ ،‬قارنيا مع 𝜏 ماذا تستنتج‬

‫الشكل (‪)5‬‬

‫انتهى الموضوع األول‬

‫صفحة ‪ 4‬من ‪8‬‬


‫امتحان بكالوريا التجريبي في مادة ‪ :‬العموم الفيزيائية ‪ //‬الشعبة ‪ :‬رياضيات ‪ ،‬تقني رياضي ‪2423 //‬‬

‫الموضوع الثاني‬
‫يحتوي الموضوع الثاني عمى ‪ 4‬صفحات (من الصفحة ‪ 5‬من ‪ 8‬إلى الصفحة ‪ 8‬من ‪)8‬‬
‫الجزء األول‪ 16( :‬نقاط)‬
‫التمرين األول‪ 44( :‬نقاط)‬
‫‪ ،‬نترك الكرية (‪ )1‬تسقط سقوطا شاقوليا في‬ ‫كريتان متماثمتان (‪ )1‬و (‪ )2‬كتمة كل منيما‬
‫اليواء بدون سرعة ابتدائية‪ ،‬وعند نفس المستوي نقذف الكرية (‪ )2‬نحو األسفل بسرعة ابتدائية ⃗⃗⃗⃗⃗ ‪،‬‬
‫‪ ،‬الدراسة التجريبية مكنتنا من رسم‬ ‫تخضع الكريتان إلى قوة احتكاك مع اليواء شدتيا‬
‫الخاص بالكرية‬ ‫الخاص بالكرية (‪ )1‬في الشكل (‪ )1‬والمنحنى ) (‬ ‫المنحنى ) (‬
‫(‪ )2‬و المبين في الشكل (‪)2‬‬
‫‪ -1‬دراسة حركة الكرية (‪)1‬‬
‫أ‪ -‬بتطبيق القانون الثاني لنيوتن‪ ،‬بين أن المعادلة التفاضمية لتطور شدة قوة االحتكاك المؤثرة عمى الكرية (‪)1‬‬
‫ثابتين يطمب تعيين عبارتييما‬ ‫و‬ ‫‪ ،‬حيث‬ ‫تكتب عمى الشكل التالي‬
‫ب‪ -‬إعتمادا عمى بيان الشكل (‪ )1‬جد قيمة كل من ‪ :‬ثابت الزمن 𝜏‪ ،‬ثابت االحتكاك ‪ ،‬السرعة الحدية‬
‫‪ -2‬دراسة حركة الكرية (‪)2‬‬
‫لمكرية (‪)2‬‬ ‫أ‪ -‬بتطبيق القانون الثاني لنيوتن أوجد المعادلة التفاضمية التي تحققيا السرعة‬
‫‪ ،‬ماذا تستنتج ؟‬ ‫ب‪ -‬حدد عبارة السرعة الحدية‬
‫ت‪ -‬ضع سمما لمحور تراتيب بيان الشكل (‪)2‬‬
‫؟ أوجد قيمتيا‬ ‫ث‪ -‬ماذا تمثل القيمة‬

‫الشكل (‪)2‬‬ ‫الشكل (‪)1‬‬

‫‪ -3‬دراسة حركة كرية أخرى (‪)3‬‬


‫من نفس المستوي السابق نقذف نحو األسفل كرية (‪ )3‬مماثمة لمكريتين السابقتين بسرعة إبتدائية قيمتيا‬
‫أ‪ -‬مثل منحنى تطور سرعة الكرية (‪ )3‬بداللة الزمن ) (‬
‫معطيات‪:‬‬

‫صفحة ‪ 5‬من ‪8‬‬


‫امتحان بكالوريا التجريبي في مادة ‪ :‬العموم الفيزيائية ‪ //‬الشعبة ‪ :‬رياضيات ‪ ،‬تقني رياضي ‪2423 //‬‬

‫التمرين الثاني‪ 46( :‬نقاط)‬


‫نحقق التركيب الموضح في الشكل (‪ )3‬و المكون من ‪:‬‬ ‫‪ -I‬لتحديد سعة مكثفة‬
‫‪.‬‬ ‫‪ -‬مولد تيار مثالي يعطي تيا ار كيربائيا ثابتا‬
‫الشكل (‪)3‬‬ ‫‪ -‬مكثفة فارغة سعتيا ‪.‬‬
‫‪.‬‬ ‫‪،‬‬ ‫‪ -‬ناقالن أوميان‬
‫و أسالك التوصيل ‪.‬‬ ‫‪ -‬بادلة‬
‫‪ -‬جياز فولط متر رقمي‬
‫نضع البادلة في الوضع (‪ ، )1‬المتابعة الزمنية لتطور التوتر الكيربائي بين طرفي المكثفة مكنتنا‬ ‫عند المحظة‬
‫لتغيرات الطاقة المخزنة في الوشيعة بداللة مربع الزمن الموضح في الشكل (‪، )4‬‬ ‫من رسم البيان ) (‬
‫وعمى جياز الفولط متر الرقمي سجمنا القيمة الثابتة‬
‫الذي يعطيو المولد ‪.‬‬ ‫‪ -1‬جد قيمة التيار الكيربائي‬
‫و ‪.‬‬ ‫و‬ ‫بداللة‬ ‫‪ -2‬اوجد عبارة الطاقة المخزنة في المكثفة ) (‬
‫‪:‬‬ ‫‪ -3‬اعتمادا عمى البيان ) (‬
‫لممكثفة‬ ‫أ‪ -‬جد قيمة السعة‬
‫الشكل (‪)4‬‬ ‫ب‪ -‬حدد الزمن النيائي 𝜏 لشحن المكثفة والطاقة العظمى‬
‫بين طرفي المكثفة عند نياية الشحن ‪.‬‬ ‫ت‪ -‬احسب التوتر الكيربائي‬
‫)‬ ‫نؤرجح البادلة الوضع (‪ )2‬في لحظة (‬ ‫‪ -II‬عندما يشير الفولط متر لمقيمة‬
‫‪.‬‬ ‫بين طرفي الناقل األومي‬ ‫‪ -1‬اكتب المعادلة التفاضمية لمتوتر ) (‬
‫)‬ ‫‪ 𝛼( .‬ثابت يطمب تعيين عبارتو بداللة‬ ‫) (‬ ‫‪ -2‬حل المعادلة يكتب بالشكل‬
‫بين طرفي المكثفة ‪.‬‬ ‫‪ -3‬استنتج العبارة الزمنية لمتوتر‬
‫الشكل (‪)5‬‬ ‫‪ -4‬البيان الموضح في الشكل (‪ )5‬يمثل تغيرات الطاقة‬
‫المخزنة بداللة الزمن ) (‬
‫أ‪ -‬بين أن العبارة المحظية لمطاقة المخزنة في المكثفة تكتب‬

‫) (‬ ‫من الشكل ‪:‬‬


‫ب‪ -‬عين قيمة ثابت الزمن ‪τ‬‬
‫‪.‬‬ ‫ت‪ -‬احسب مقاومة الناقل األومي‬
‫ث‪ -‬احسب الطاقة المستيمكة بفعل جول عند الناقل‬
‫عند اللحظة 𝜏‬ ‫األومي‬

‫صفحة ‪ 6‬من ‪8‬‬


‫امتحان بكالوريا التجريبي في مادة ‪ :‬العموم الفيزيائية ‪ //‬الشعبة ‪ :‬رياضيات ‪ ،‬تقني رياضي ‪2423 //‬‬

‫التمرين الثالث ‪ 46( :‬نقاط)‬


‫كتمتو المـ ـ ـ ـ ـ ـ ـ ـ ـ ــولية الجزيئية‬ ‫أستر سائل كتمتو الحجمية‬ ‫ايثانوات االيثيل‬
‫يستعمل في صناعة الصابون من خالل مفاعمتو مع محمول ىيدروكسيد الصـ ـ ـ ـ ـ ـ ـ ـ ـ ـ ـ ـ ـ ــوديوم‬
‫(‪ ،‬ينمذج التحول الكيميائي البطيئ و التام الحادث بمعادلة التفاعل التالية‪:‬‬ ‫)‬
‫→‬
‫من ىيدروكسيد الصوديوم تركيزه‬ ‫نضع بيشر يحوي عمى حجم‬ ‫في المحظة‬
‫ثم نضيف لو ‪ 3‬قطرات من‬ ‫ف ــوق مخالط مغناطيسي ونغمس فيو مسبار جياز قياس الناقمية ثابت خميتو‬
‫بالمنحنى (‪ )1‬في الشكل (‪)6‬‬ ‫إيثانوات االيثيل‪ ،‬و نمثل تطور الناقمية بداللة الزمن ) (‬
‫‪ -1‬أنجز جدوال لتقدم التفاعل الحادث‬
‫‪ ،‬مع تعيين المتفاعل المحد‬ ‫‪ ،‬أحسب التقدم األعظمي‬ ‫‪ -2‬باعتبار حجم القطرة‬
‫‪ -3‬إقترح طريقة أخرى تصمح لمتابعة ىذا التحول زمنيا‬
‫ثابتين يطمب تعيين‬ ‫و‬ ‫‪ ،‬حيث‬ ‫‪ -4‬بين أن ناقمية المزيج التفاعمي تكتب من الشكل‪:‬‬
‫( ‪.‬‬ ‫( ‪)،‬‬ ‫( ‪)،‬‬ ‫و الناقميات المولية )‬ ‫‪،‬‬ ‫‪،‬‬ ‫عبارتييما بداللة المقادير‪:‬‬
‫و ‪.‬‬ ‫‪ -5‬أوجد من البيان قيمة كل من‬
‫‪ ،‬وعين قيمتو بيانيا‬ ‫‪ -6‬عرف زمن نصف التفاعل‬
‫وأحسب قيمتيا عند المحظة‬ ‫‪ -7‬بين أن سرعة التفاعل تكتب من الشكل‬
‫‪ -8‬كيف تتطور سرعة التفاعل بمرور الزمن ؟ فسر ذلك مجيريا‬
‫‪ -9‬نعيد التجربة السابقة و ذلك بتغيير أحد العوامل الحركية فنحصل عمى المنحنى (‪ )2‬من الشكل السابق‪،‬‬
‫ماىو ىذا العامل الحركي؟‬

‫الشكل (‪)6‬‬

‫𝟓𝟎‬

‫صفحة ‪ 7‬من ‪8‬‬


‫امتحان بكالوريا التجريبي في مادة ‪ :‬العموم الفيزيائية ‪ //‬الشعبة ‪ :‬رياضيات ‪ ،‬تقني رياضي ‪2423 //‬‬

‫الجزء الثاني‪ 44( :‬نقاط)‬


‫الشكل (‪)7‬‬
‫التمرين التجريبي‪ 44( :‬نقاط)‬
‫من أجل اإلجابة عمى السؤالين التاليين‪ :‬من أين تأتي الطاقة الكيربائية‬
‫التي تقدميا األعمدة ؟ و كيف تشتغل ؟‪ ،‬قام فوج من التالميذ بدراسة‬
‫تجريبية لمبدأ اشتغال عمود دانيال انطالقا من الوسائل و المواد التالية‪:‬‬
‫من محمول مائي‬ ‫‪ -‬بيشر يحتوي عمى حجم‬
‫( تركيزه المولي‬ ‫لنترات الفضة )‬
‫( تركيزه‬ ‫من محمول مائي لكمور الحديد الثنائي )‬ ‫‪ -‬بيشر يحتوي عمى نفس الحجم‬
‫‪ -‬صفيحة من الزنك ‪ -‬صفيحة من الحديد ‪ -‬إضافة إلى جسر ممحي‬ ‫المولي‬
‫ربط التالميذ قطبي العمود بجياز الفولطمتر كما ىو موضح في الشكل (‪ ،)7‬فأشار الجياز إلى توتر كيربائي قيمتو‬

‫‪ -1‬ماذا تمثل القيمة التي يشير إلييا الفولط متر‬


‫‪ -2‬أكتب الرمز االصطالحي لمعمود المدروس‬
‫‪ -3‬أكتب المعادلتين النصفيتين لألكسدة و لالرجاع الحادثتين عند المسريين‪ ،‬ثم إستنتج منيما معادلة التحول‬
‫الذي يحدث عند اشتغال العمود‬
‫[ بداللة الزمن‬ ‫يمثل بيان الشكل (‪ )8‬تطور تركيز شوارد الفضة ]‬
‫و التركيز المولي‬ ‫‪ -4‬حدد قيمة شدة التيار الكيربائي‬
‫[ عند الحظة‬ ‫‪ -5‬أحسب التركيز المولي لشوارد الحديد ]‬

‫الشكل (‪)8‬‬

‫اإلجابة النموذجية‬

‫انتهى الموضوع الثاني‬

‫صفحة ‪ 8‬من ‪8‬‬


‫اٌعّ‪ٛٙ‬س‪٠‬ح اٌعضائش‪٠‬ح اٌذ‪ّ٠‬مشاط‪١‬ح اٌشؼث‪١‬ح‬
‫‪ٚ‬صاسج اٌرشت‪١‬ح اٌ‪ٛ‬طٕ‪١‬ح‬
‫ِذ‪٠‬ش‪٠‬ح اٌرشت‪١‬ح ٌ‪ٛ‬ال‪٠‬ح ورڤـــلت‬ ‫اِرحاْ تىاٌ‪ٛ‬س‪٠‬ا اٌرعش‪٠‬ث‪ ٟ‬اٌرؼٍ‪ ُ١‬اٌصأ‪ٞٛ‬‬
‫د‪ٚ‬سج ‪ِ :‬ا‪2023 ٞ‬‬ ‫اٌشؼثح ‪ :‬س‪٠‬اػ‪١‬اخ ‪ ،‬ذمٕ‪ ٟ‬س‪٠‬اػ‪ٟ‬‬
‫الموضوع األول‬ ‫االظاتح إٌّ‪ٛ‬رظ‪١‬ح التخرثاس اٌثاواٌ‪ٛ‬س‪٠‬ا اٌرعش‪٠‬ث‪ ٟ‬ف‪ِ ٟ‬ادج ‪ :‬العـلوم الفيزيائيت‬
‫الموضوع األول‬
‫العالمت‬ ‫عناصر اإلجابت‬
‫ِعضأج ِعّ‪ٛ‬ع‬
‫التمرين األول‪ 04( :‬نقاط)‬
‫‪.I‬‬
‫‪ّ٠‬صً ِشوض األسع تإٌغثح ٌٍّذاس أحذ‬ ‫‪ 1‬ذّص‪ً١‬‬
‫(‪)T‬‬
‫‪P‬‬ ‫‪A‬‬
‫اٌّحشل‪ ( ٓ١‬اٌثؤسج )‬ ‫‪ 2‬اٌّخطط‬
‫‪O‬‬
‫‪A‬‬ ‫‪ 3‬ط‪ٛ‬ي‬
‫)‪(S‬‬ ‫(‬ ‫)‬ ‫اٌّح‪ٛ‬س‬
‫𝑷𝒉‬ ‫𝑨𝒉‬ ‫(‬ ‫)‬ ‫‪.‬‬ ‫اٌىث‪١‬ش‬
‫‪2a‬‬ ‫‪.‬‬
‫) ( ⃗‬ ‫) ( ⃗‬ ‫) ( ⃗‬ ‫‪ 4‬ذث‪ ٓ١١‬أْ‬
‫‪⁄‬‬ ‫‪⁄‬‬ ‫‪⁄‬‬
‫{‪.‬‬ ‫حشوح اٌمّش‬
‫) ( ⃗‬ ‫ارْ اٌغشػح غ‪١‬ش شاترح ‪ ِٕٗٚ‬حشوح اٌمّش غ‪١‬ش‬
‫غ‪١‬ش ِٕرظّح‬
‫‪⁄‬‬ ‫ِٕرظّح‬
‫‪.II‬‬
‫‪ 1‬اٌّشظغ اٌغاٌ‪ ٍٟ١‬إٌّاعة ٘‪ ٛ‬اٌّشظغ اٌّشوض‪ ٞ‬األسػ‪ ( ٟ‬اٌع‪ِٛ١‬شوض‪) ٞ‬‬
‫)𝑆(‬ ‫⃗‬ ‫‪.‬‬ ‫‪ 2‬ذّص‪ ً١‬اٌم‪ٜٛ‬‬
‫‪⁄‬‬

‫‪𝐹⃗𝑇⁄‬‬ ‫ترطث‪١‬ك لأ‪ٛ١ٔ ْٛ‬ذٓ اٌصأ‪:ٟ‬‬ ‫‪ 3‬أ ـ حشوح‬


‫𝑆‬ ‫⃗∑‬ ‫⃗⃗⃗⃗‬ ‫‪.‬‬ ‫دائش‪٠‬ح‬
‫)𝑻(‬ ‫ِٕرظّح‬
‫‪.‬‬
‫تُ أْ اٌّغاس دائش‪ ٚ ٞ‬اٌعغُ تخاػغ ٌم‪ٛ‬ج‬
‫ِشوض‪٠‬ح فئْ اٌمّش) ( ف‪ ٟ‬حشوح دائش‪٠‬ح ِٕرظّح‬
‫ب‪ /‬ػثاسج‬
‫{‪.‬‬ ‫اٌغشػح‬

‫‪ 4‬أ‪ /‬ل‪ّ١‬ح‬
‫ٔظف لطش‬
‫ب‪ /‬حغاب‬
‫‪.‬‬ ‫√‬ ‫√‬ ‫‪⁄‬‬ ‫اٌغشػح‬
‫ض‪ /‬حغاب‬
‫) (‬ ‫د‪ٚ‬س اٌّذاس‬
‫) (‬ ‫) (‬ ‫) (‬ ‫) (‬ ‫د‪ /‬طث‪١‬ؼح‬
‫لّش ظ‪ِٛ١‬غرمش‬ ‫(‬ ‫)‬ ‫‪ ٚ‬ػٍ‪ ٗ١‬فئْ اٌمّش‬ ‫اٌمّش‬

‫طفحح ‪8 ِٓ 1‬‬
‫االجابت النموذجيت الختبار الباكالوريا التجريبي في مادة ‪ :‬العلوم الفيزيائيت ‪ /‬الشعبت ‪ :‬رياضياث ‪ ،‬تقني رياضي‬
‫التمرين الثاني‪ 06( :‬نقاط)‬
‫‪.I‬‬
‫‪⁄ .‬‬ ‫‪ 1‬حغاب‬
‫اٌرشو‪١‬ض‬
‫) (‬ ‫)(‬ ‫(‬ ‫)‬ ‫(‬ ‫)‬ ‫‪ِ 2‬ؼادٌح‬
‫اٌّؼادٌح‬ ‫) (‬ ‫)(‬ ‫(‬ ‫)‬ ‫) (‬ ‫أ‪ /‬ظذ‪ٚ‬ي‬
‫اٌحاٌح‬ ‫اٌرمذَ‬ ‫وّ‪١‬ح اٌّادج تاٌّ‪ٛ‬ي‬ ‫اٌرمذَ‬
‫اإلترذائ‪١‬ح‬ ‫‪0‬‬ ‫‪0‬‬ ‫‪0‬‬
‫االٔرماٌ‪١‬ح‬ ‫ت‪ٛ‬فشج‬
‫إٌ‪ٙ‬ائ‪١‬ح‬
‫[‬ ‫]‬
‫‪.‬‬ ‫‪ 3‬ب‪ /‬حغاب‬
‫[‬ ‫[]‬ ‫]‬ ‫‪.‬‬ ‫‪.‬‬
‫[‬ ‫]‬ ‫ٔغرٕرط أْ اٌرفاػً غ‪١‬ش ذاَ ‪ٚ‬‬
‫[‬ ‫]‬
‫أعاط ذفىىٗ ف‪ ٟ‬اٌّاء ذفىه ظضئ‪.ٟ‬‬
‫‪.‬‬
‫اٌ‪ٛ‬اظة أتخزٖ ترطث‪١‬ك لأ‪ ْٛ‬اٌرّذ‪٠‬ذ‪:‬‬ ‫‪ -1‬حغاب حعُ اٌّحٍ‪ٛ‬ي‬ ‫‪ 4‬اٌخط‪ٛ‬اخ‬
‫اٌؼٍّ‪١‬ح‬

‫‪ٔ ٚ‬غىثٗ‬ ‫ِٓ اٌّحٍ‪ٛ‬ي‬ ‫ٔأتخز حعُ‬ ‫‪ -2‬ت‪ٛ‬اعطح ِاطح ػ‪١‬اس‪٠‬ح‬


‫ت‪ٙ‬ا وّ‪١‬ح ِٓ اٌّاء اٌّمطش‪.‬‬ ‫ف‪ ٟ‬ح‪ٛ‬ظٍح ػ‪١‬اس‪٠‬ح‬
‫‪ٔ -3‬ىًّ تاٌّاء اٌّمطش اٌ‪ ٝ‬غا‪٠‬ح تخط اٌؼ‪١‬اس ِغ اٌشض ‪ٚ‬اٌرحش‪٠‬ه‪.‬‬

‫‪.‬‬ ‫‪ 5‬حغاب‬

‫‪ٚ‬‬ ‫‪ ٚ‬اٌعٍّح ذرط‪ٛ‬س تاذعاٖ ذشىً‬ ‫ػٍّ‪١‬ح اٌرّذ‪٠‬ذ ذشفغ ِٓ ل‪ّ١‬ح‬ ‫‪ 6‬ذأش‪١‬ش اٌحاٌح‬
‫اإلترذائ‪١‬ح‬
‫ػٍ‪ٝ‬‬
‫[‬ ‫[ ]‬ ‫]‬
‫‪.‬‬
‫[‬ ‫[ ]‬ ‫]‬
‫‪.‬‬ ‫‪ 7‬حغاب‬
‫[‬ ‫]‬ ‫[‬ ‫]‬
‫(‬ ‫)‬
‫[‬ ‫]‬ ‫‪.‬‬ ‫‪.‬‬
‫[‬ ‫]‬ ‫(‬ ‫)‬
‫‪.‬‬ ‫‪.‬‬
‫‪.‬‬ ‫[‬ ‫]‬
‫[‬ ‫]‬
‫[‬ ‫]‬ ‫[‬ ‫]‬ ‫(‬ ‫)‬
‫‪.‬‬
‫‪.II‬‬
‫(‬ ‫)‬ ‫(‬ ‫)‬ ‫(‬ ‫)‬ ‫)(‬ ‫‪ِ 1‬ؼادٌح‬
‫(‬ ‫)‬ ‫‪ 2‬إحذاش‪١‬اخ‬
‫[‬ ‫]‬
‫ٌذ‪ٕ٠‬ا ‪.‬‬ ‫تاإلعماط ػٍ‪ٝ‬‬ ‫‪.‬‬ ‫‪ 3‬اٌرأوذ ِٓ‬
‫[‬ ‫]‬ ‫ل‪ّ١‬ح‬
‫ٔعذ‬ ‫ِح‪ٛ‬س اي‬
‫‪٠‬ى‪ْٛ‬‬ ‫ػٕذ ٔمطح ٔظف اٌرىافؤ‪:‬‬ ‫ت‪١‬أ‪١‬ا‬
‫ٔفظ اٌم‪ّ١‬ح اٌغاتمح‬
‫[‬ ‫[ ]‬ ‫]‬
‫‪ ٚ‬ػٍ‪ ٗ١‬فئْ‬

‫طفحح ‪8 ِٓ 2‬‬
‫االجابت النموذجيت الختبار الباكالوريا التجريبي في مادة ‪ :‬العلوم الفيزيائيت ‪ /‬الشعبت ‪ :‬رياضياث ‪ ،‬تقني رياضي‬
‫اٌّؼادٌح‬ ‫(‬ ‫)‬ ‫(‬ ‫)‬ ‫(‬ ‫)‬ ‫)(‬ ‫‪ 4‬ظذ‪ٚ‬ي ذمذَ‬
‫اٌرمذَ اٌحاٌح‬ ‫وّ‪١‬ح اٌّادج تاٌّ‪ٛ‬ي‬ ‫اٌّؼا‪٠‬شج‬
‫اإلترذائ‪١‬ح‬ ‫‪0‬‬ ‫‪0‬‬ ‫ت‪ٛ‬فشج‬
‫اٌرىافؤ‬

‫[‬ ‫]‬
‫‪ 5‬اٌرؼث‪١‬ش ػٓ‬
‫‪( ).‬‬ ‫[‬ ‫]‬
‫[‬ ‫]‬
‫[‬ ‫]‬
‫[‬ ‫]‬ ‫[‬ ‫]‬
‫‪.‬‬ ‫اٌرؼث‪١‬ش ػٓ‬
‫[‬ ‫]‬ ‫[‬ ‫]‬ ‫[‬ ‫]‬
‫[‬ ‫]‬ ‫[‬ ‫]‬
‫‪( ).‬‬
‫[‬ ‫]‬

‫‪ 6‬إعرٕراض ل‪ّ١‬ح ِٓ ) ( ‪ٔ ( )ٚ‬عذ‪.:‬‬


‫ػٕذ‬
‫‪.‬‬ ‫اٌرىافؤ‬
‫‪.‬‬ ‫حغاب‬
‫ػٕذ اٌرىافؤ‬
‫ٔغرٕرط أْ ذفاػً اٌّؼا‪٠‬شج ذفاػً ذاَ‪.‬‬ ‫اإلعرٕراض‬

‫اٌّ‪ٛ‬ػ‪ٛ‬ع األ‪ٚ‬ي‬
‫اٌؼالِح‬
‫ػٕاطش اإلظاتح‬
‫ِعضأج ِعّ‪ٛ‬ع‬
‫التمرين الثالث‪ 04( :‬نقاط) ‪:‬‬
‫‪0.20‬‬ ‫‪0.20‬‬ ‫ٌٍرغٍة ػٍ‪ ٝ‬طالح اٌرٕافش ت‪ ٓ١‬إٌ‪ٛ‬اذ‪ٓ١‬‬ ‫‪1‬‬
‫‪ .1‬ل‪ّ١‬ح وً ِٓ ‪X ,Y ,Z,A‬‬
‫‪0.20‬‬ ‫بخقص‪:‬‬
‫‪0.0‬‬ ‫‪2‬‬
‫‪0.20‬‬
‫‪:ِٕٗٚ‬‬
‫طالح ورٍح اٌثش‪ٚ‬ذ‪ٔٛ‬اخ ‪ٚ‬إٌ‪١‬رش‪ٔٚ‬اخ ‪ِ ٟ٘ٚ‬رفشلح ‪ٚ‬عاوٕح‬
‫‪0.20‬‬
‫‪0.0‬‬ ‫(‬ ‫)‬ ‫أ‬
‫‪0.20‬‬
‫( [‬ ‫)‬ ‫(‬ ‫])‬
‫‪0.20‬‬
‫‪0.0‬‬ ‫ذّصً طالح اٌشتط ٌٕ‪ٛ‬اج‬ ‫ب‬
‫‪0.20‬‬
‫‪3‬‬
‫‪.‬‬ ‫) (‬
‫‪0.20‬‬
‫‪0.0‬‬ ‫خ‬
‫‪0.20 .‬‬ ‫(‬ ‫)‬ ‫(‬ ‫)‬
‫‪.‬‬
‫(‬
‫‪.‬‬ ‫)) (‬

‫(‬ ‫(‬ ‫)‬ ‫(‬ ‫)‬


‫)‬ ‫‪4‬‬
‫‪0.25‬‬ ‫‪0.25‬‬ ‫أ‬
‫(‬ ‫)‬

‫طفحح ‪8 ِٓ 3‬‬
‫االجابت النموذجيت الختبار الباكالوريا التجريبي في مادة ‪ :‬العلوم الفيزيائيت ‪ /‬الشعبت ‪ :‬رياضياث ‪ ،‬تقني رياضي‬
‫‪0.25‬‬ ‫‪E 3  E 2  E 2  15022.48  120.198  14902.282MeV‬‬
‫‪0. 5‬‬ ‫ب‬
‫‪0.20‬‬ ‫طالح ورٍح إٌ‪ٛ‬اذط ‪ ٟ٘ٚ‬عاوٕح‬
‫‪0.25‬‬ ‫‪0.25‬‬ ‫‪E Lib  E  E 3  E 1  14902.282  14907.634  5.352MeV‬‬ ‫‪5‬‬
‫‪ET‬‬ ‫‪4.78 10‬‬ ‫‪23‬‬

‫‪0.25‬‬ ‫‪0.25‬‬ ‫‪N ‬‬ ‫‪‬‬ ‫‪13‬‬


‫‪ 5.58 1035 Noyau‬‬ ‫‪6‬‬
‫‪E lib 5.352 1.6 10‬‬
‫‪N  M 5.58 1035 15‬‬
‫‪m N  ‬‬ ‫‪‬‬ ‫‪ 1.38 1013 g‬‬
‫‪0.25‬‬ ‫‪NA‬‬ ‫‪6.023 10‬‬ ‫‪23‬‬

‫‪0.5‬‬ ‫‪7‬‬
‫‪0.25‬‬ ‫‪N  M 5.58 1035 1‬‬
‫‪m H  ‬‬ ‫‪‬‬ ‫‪ 9.26 1011 g‬‬
‫‪NA‬‬ ‫‪6.023 10‬‬ ‫‪23‬‬

‫التمرين التجريبي‪ 06( :‬نقاط) ‪:‬‬


‫‪0.25‬‬ ‫‪di R  r‬‬ ‫‪E‬‬
‫‪U b U R  E ‬‬ ‫‪‬‬ ‫‪i ‬‬
‫‪0.25‬‬ ‫‪dt‬‬ ‫‪L‬‬ ‫‪L‬‬
‫‪1‬‬ ‫‪1‬‬
‫‪0.25‬‬ ‫‪R r‬‬ ‫‪E‬‬
‫‪A‬‬ ‫‪B ٚ‬‬
‫‪0.25‬‬ ‫‪L‬‬ ‫‪L‬‬
‫‪E‬‬ ‫‪12‬‬
‫‪0. 5‬‬ ‫‪ 20  L ‬‬ ‫‪ 0.6H‬‬ ‫اٌث‪١‬اْ ػثاسج ػٍ‪ ٝ‬تخط ِغرم‪ِ ُ١‬ؼادٌرٗ ِٓ‬
‫‪L‬‬ ‫‪20‬‬
‫‪1‬‬ ‫‪0.25‬‬ ‫‪di‬‬ ‫أ‬
‫‪1‬‬ ‫اٌشىً ‪  400i  20‬تاٌّطاتمح ٔعذ‪:‬‬
‫‪0.25‬‬ ‫‪‬‬ ‫‪ 2.5ms‬‬ ‫‪dt‬‬
‫‪400‬‬
‫‪L‬‬ ‫‪L‬‬ ‫‪0.6‬‬
‫‪0.0‬‬ ‫‪0. 5‬‬ ‫‪‬‬ ‫‪ r  R  r ‬‬ ‫ب ‪ 200  40‬‬
‫‪R r‬‬ ‫‪‬‬ ‫‪2.5 103‬‬ ‫‪2‬‬
‫‪I 0  50 103  0.05A‬‬
‫‪0.25‬‬
‫‪0.0‬‬ ‫‪E‬‬ ‫‪12‬‬ ‫ض‬
‫‪0.25‬‬ ‫‪I0 ‬‬ ‫‪‬‬ ‫‪ 0.05A‬‬
‫‪R  r 240‬‬
‫‪1‬‬
‫‪0.5‬‬ ‫‪0.5‬‬ ‫‪E bMax  LI 02  0.5  0.6  (0.05)2  7.5 104 J‬‬ ‫د‬
‫‪2‬‬
‫‪II‬‬
‫‪di R‬‬ ‫‪dU b R‬‬
‫‪0.0‬‬ ‫‪0.5‬‬ ‫‪U b U R  0 ‬‬ ‫‪ i 0‬‬ ‫‪ Ub  0‬‬ ‫‪1‬‬
‫' ‪dt L‬‬ ‫‪dt‬‬ ‫'‪L‬‬
‫‪0.25‬‬
‫‪0.5‬‬ ‫‪ِ 0=0‬حممح‬ ‫‪ 2‬اٌّؼادٌح خ ذمثً حال‪ :‬اشرماق ‪ +‬ذؼ‪٠ٛ‬غ‬
‫‪0.25‬‬
‫‪0.25‬‬ ‫‪-‬حّا‪٠‬ح اٌذاسج ِٓ فشط اٌر‪ٛ‬ذش‬
‫‪0.0‬‬ ‫‪3‬‬
‫‪0.20‬‬ ‫‪٠ -‬غّح تّش‪ٚ‬س اٌر‪١‬اس ف‪ ٟ‬ظ‪ٙ‬ح ‪ٚ‬احذج‬
‫‪6cm  12V‬‬
‫‪0.25‬‬ ‫‪0.25‬‬ ‫أ‬
‫‪1cm  2V‬‬
‫‪E 12‬‬
‫‪0.20‬‬ ‫‪0.20‬‬ ‫‪I0  ‬‬ ‫‪ 0.06A‬‬ ‫‪4‬‬
‫‪R 200‬‬
‫‪0.20‬‬ ‫‪L ' 0.6‬‬
‫‪ ‬‬ ‫‪ 3ms‬‬
‫‪0.0‬‬ ‫‪  ' ‬ذ‪ٛ‬ظذ ػاللح ػىغ‪١‬ح ت‪ ٓ١‬شاتس اٌضِٓ ‪ِٚ‬ما‪ِٚ‬ح اٌذاسج‪.‬‬
‫‪0.20‬‬ ‫‪R 200‬‬

‫طفحح ‪8 ِٓ 4‬‬
‫االجابت النموذجيت الختبار الباكالوريا التجريبي في مادة ‪ :‬العلوم الفيزيائيت ‪ /‬الشعبت ‪ :‬رياضياث ‪ ،‬تقني رياضي‬
‫اٌعّ‪ٛٙ‬س‪٠‬ح اٌعضائش‪٠‬ح اٌذ‪ّ٠‬مشاط‪١‬ح اٌشؼث‪١‬ح‬
‫‪ٚ‬صاسج اٌرشت‪١‬ح اٌ‪ٛ‬طٕ‪١‬ح‬
‫ِذ‪٠‬ش‪٠‬ح اٌرشت‪١‬ح ٌ‪ٛ‬ال‪٠‬ح ورڤـــلت‬ ‫اِرحاْ تىاٌ‪ٛ‬س‪٠‬ا اٌرعش‪٠‬ث‪ ٟ‬اٌرؼٍ‪ ُ١‬اٌصأ‪ٞٛ‬‬
‫د‪ٚ‬سج ‪ِ :‬ا‪2023 ٞ‬‬ ‫اٌشؼثح ‪ :‬س‪٠‬اػ‪١‬اخ ‪ ،‬ذمٕ‪ ٟ‬س‪٠‬اػ‪ٟ‬‬
‫الموضوع الثاني‬ ‫االظاتح إٌّ‪ٛ‬رظ‪١‬ح التخرثاس اٌثاواٌ‪ٛ‬س‪٠‬ا اٌرعش‪٠‬ث‪ ٟ‬ف‪ِ ٟ‬ادج ‪ :‬العـلوم الفيزيائيت‬
‫اٌّ‪ٛ‬ػ‪ٛ‬ع اٌصأ‪ٟ‬‬
‫اٌؼالِح‬
‫ػٕاطش اإلظاتح‬
‫ِعضأج ِعّ‪ٛ‬ع‬
‫التمرين األول ‪ 04(:‬نقاط) ‪:‬‬
‫‪I‬‬
‫⃗∑‬ ‫⃗‬ ‫⃗⃗‬ ‫⃗⃗‬ ‫ترطث‪١‬ك اٌمأ‪ ْٛ‬اٌصأ‪ٛ١ٌٕ ٟ‬ذٓ‪⃗ :‬‬
‫⃗‬
‫تاالعماط ػٍىّح‪ٛ‬س اٌحشوح ٔعذ ‪:‬‬
‫) (‬ ‫أ‪ -‬اٌّؼادٌح‬
‫اٌرفاػٍ‪١‬ح‬
‫(‬ ‫)‬

‫(‬ ‫تاٌّطاتمح ٔعذ‪) :‬‬ ‫‪1‬‬

‫شاتد اٌضِٓ ‪:‬‬


‫)اٌّ‪(ً١‬‬

‫‪K‬‬ ‫⃪‬ ‫شاتد االحرىان‪ٌ :K‬ذ‪ٕ٠‬ا‬


‫⃪ ِٓ‬ ‫⃪‬ ‫‪ :‬ف‪ ٟ‬إٌظاَ اٌذائُ‬ ‫اٌغشػح اٌحذ‪٠‬ح‬ ‫ب‪-‬ا‪٠‬عاد اٌم‪ُ١‬‬

‫اٌث‪١‬اْ ٔعذ ‪⃪ =0.04N‬‬


‫→‬
‫⃗∑‬ ‫⃗‬ ‫⃗⃗‬ ‫⃗⃗‬ ‫ترطث‪١‬ك اٌمأ‪ ْٛ‬اٌصأ‪ٛ١ٌٕ ٟ‬ذٓ‪⃗ :‬‬
‫⃗‬ ‫أ‪-‬اٌّؼادٌح‬
‫تاالعماط ػٍ‪ِ ٝ‬ح‪ٛ‬س اٌحشوح ٔعذ ‪:‬‬ ‫اٌرفاػٍ‪١‬ح‬
‫ٌٍغشػح‬
‫→‬ ‫(‬ ‫)‬
‫تاٌرؼ‪٠ٛ‬غ ف‪ ٟ‬اٌّؼادٌح اٌرفاػٍ‪١‬ح‬ ‫⃪‬ ‫ف‪ ٟ‬إٌظاَ اٌذائُ ‪:‬‬
‫ٔعذ‪:‬‬ ‫‪ 2‬ب‪-‬اٌغشػح‬
‫ٔغرٕرط أْ‬ ‫(‬ ‫⃪)‬ ‫(‬ ‫)‬ ‫اٌحذ‪٠‬ح‬
‫اٌغشػح اٌحذ‪٠‬ح ال ذرؼٍك تاٌغشػح االترذائ‪١‬ح‬
‫اٌغٍُ‬ ‫خ – عٍُ‬
‫)ذذس‪٠‬عاخ(‬
‫ِح‪ٛ‬س اٌرشذ‪١‬ة‬
‫ذّصً اٌم‪ّ١‬ح ‪ t1‬شاتد اٌضِٓ ‪ِّ ‬ا عثك ‪ٚ‬ظذٔا‬ ‫ز‪ -‬اٌم‪ّ١‬ح ‪t1‬‬

‫‪ 3‬إٌّحٕ‪ٝ‬‬

‫طفحح ‪8 ِٓ 5‬‬
‫االجابت النموذجيت الختبار الباكالوريا التجريبي في مادة ‪ :‬العلوم الفيزيائيت ‪ /‬الشعبت ‪ :‬رياضياث ‪ ،‬تقني رياضي‬

‫اٌّ‪ٛ‬ػ‪ٛ‬ع اٌصأ‪ٟ‬‬
‫اٌؼالِح‬
‫ػٕاطش اإلظاتح‬
‫ِعضأج ِعّ‪ٛ‬ع‬
‫التمرين الثاني ‪ 60(:‬نقاط) ‪:‬‬
‫‪I‬‬
‫⃪‬ ‫‪ 1‬ل‪ّ١‬ح شذج ‪I0‬‬
‫ػثاسج اٌطالح‬
‫) (‬ ‫(‬ ‫)‬ ‫‪2‬‬
‫اٌّخضٔح‬
‫تاٌّطثمح ِغ اٌؼاللح إٌظش‪٠‬ح‬ ‫ِٓ اٌث‪١‬اْ‬ ‫ل‪ّ١‬ح عؼح‬
‫‪,‬‬
‫(‬ ‫)‬ ‫اٌّىصفح‬
‫(‬ ‫)‬
‫⃪‬ ‫ِٓ اٌث‪١‬اْ ‪4‬‬ ‫‪ 3‬اٌضِٓ ‪tf‬‬
‫‪.‬‬ ‫اٌر‪ٛ‬ذش‬
‫اٌى‪ٙ‬شتائ‪ ٟ‬ػٕذ‬
‫√‬ ‫√‬ ‫‪.‬‬ ‫ٔ‪ٙ‬ا‪٠‬ح اٌشحٓ‬
‫‪II‬‬
‫⃪ تاالشرماق ٔعذ‬ ‫ترطث‪١‬ك لأ‪ ْٛ‬ظّغ اٌر‪ٛ‬ذشاخ‬
‫اٌّؼادٌح‬
‫‪1‬‬
‫اٌرفاػٍ‪١‬ح‬
‫⃪‬ ‫⃪‬
‫تاٌرؼ‪٠ٛ‬غ ف‪ ٟ‬اٌّؼادٌح اٌرفاػٍ‪١‬ح ٔعذ ‪:‬‬ ‫ا‪٠‬عاد ػثاسج‬
‫‪2‬‬
‫⃪‬ ‫‪=0‬‬ ‫اٌصاتد ‪α‬‬
‫اٌؼثاسج اٌضِٕ‪١‬ح‬
‫‪3‬‬
‫ٌـ‬
‫(‬ ‫)‬ ‫أ‪ -‬ػثاسج اٌطالح‬
‫⃪‬ ‫ِٓ اٌث‪١‬اْ‬ ‫ب‪ -‬ل‪ّ١‬ح شاتد‬
‫اٌضِٓ‬
‫⃪‬ ‫‪ 4‬خ‪ -‬ل‪ّ١‬ح ِم‪ِٛ‬ح‬
‫إٌالً األ‪ِٟٚ‬‬
‫(‬ ‫) ) (‬ ‫(‬ ‫ز‪ -‬اٌطالح‬
‫(‪.‬‬ ‫) )‬ ‫اٌّغر‪ٍٙ‬ىح‬

‫طفحح ‪8 ِٓ 6‬‬
‫االجابت النموذجيت الختبار الباكالوريا التجريبي في مادة ‪ :‬العلوم الفيزيائيت ‪ /‬الشعبت ‪ :‬رياضياث ‪ ،‬تقني رياضي‬

‫التمرين الثالث ‪ 06( :‬نقاط) ‪:‬‬

‫‪0‬‬ ‫‪0‬‬ ‫ظذ‪ٚ‬ي‬


‫‪1‬‬
‫اٌرمذَ‬

‫اٌرمذَ‬
‫االػظّ‪ٟ‬‬
‫‪ٚ 2‬‬
‫{‬ ‫اٌّرفاػً‬
‫اٌّحذ‬
‫‪ ٚ‬اٌّرفاػً اٌّحذ ٘‪ٛ‬‬ ‫‪ ِٕٗ ٚ‬اٌرمذَ االػظّ‪ٟ‬‬
‫‪ّ٠‬ىٓ ِراتؼح ٘زا اٌرح‪ٛ‬ي ػٓ طش‪٠‬ك ل‪١‬اط اٌّؼا‪٠‬شج اٌٍ‪١ٔٛ‬ح‬ ‫‪3‬‬

‫(‬ ‫[‬ ‫]‬ ‫[‬ ‫]‬ ‫[‬ ‫)]‬


‫[‬ ‫]‬ ‫‪.‬‬ ‫[‬ ‫]‬ ‫[ ‪.‬‬ ‫]‬ ‫‪.‬‬
‫(‬ ‫(‬ ‫)‬ ‫‪).‬‬
‫(‬ ‫‪).‬‬ ‫اٌؼاللح‬
‫‪4‬‬

‫(‬ ‫)‬ ‫(‬ ‫)‬

‫(‬ ‫)‬
‫{‬
‫(‬ ‫)‬
‫ِٓ اٌث‪١‬اْ‬
‫ل‪ُ١‬‬
‫(‬ ‫)‬ ‫(‬ ‫)‬ ‫‪5‬‬
‫‪.‬‬ ‫‪ٚ‬‬

‫٘‪ ٛ‬اٌضِٓ اٌالصَ ٌثٍ‪ٛ‬ؽ ذمذَ اٌرفاػً ٔظف ل‪ّ١‬رٗ االػظّ‪١‬ح‬ ‫صِٓ‬
‫‪ٔ 6‬ظف‬
‫‪⁄‬‬ ‫‪⁄‬‬ ‫اٌرفاػً‬

‫ِٓ ػثاسج إٌالٍ‪١‬ح ٔعذ ‪:‬‬


‫‪: ِٕٗ ٚ‬‬
‫ل‪ّ١‬ر‪ٙ‬ا ػٕذ اٌٍحظح‬ ‫عشػح‬
‫‪7‬‬
‫(‬ ‫)‬ ‫اٌرفاػً‬

‫(‬ ‫)‬
‫(‬ ‫)‬
‫‪⁄‬‬

‫طفحح ‪8 ِٓ 7‬‬
‫االجابت النموذجيت الختبار الباكالوريا التجريبي في مادة ‪ :‬العلوم الفيزيائيت ‪ /‬الشعبت ‪ :‬رياضياث ‪ ،‬تقني رياضي‬

‫ذمً عشػح اٌرفاػً تّش‪ٚ‬س اٌضِٓ ‪٘ٚ‬زا ساظغ اٌ‪ٔ ٝ‬مظاْ اٌرظادِاخ اٌفؼاٌح ٔر‪١‬عح ذٕالض‬
‫‪8‬‬
‫ذشاو‪١‬ض اٌّرفاػالخ أشٕاء اٌرفاػً‬

‫اٌؼاًِ اٌحشو‪ ٛ٘ ٟ‬دسظح اٌحشاسج‬ ‫‪9‬‬

‫اٌّ‪ٛ‬ػ‪ٛ‬ع اٌصأ‪ٟ‬‬
‫اٌؼالِح‬
‫ػٕاطش اإلظاتح‬
‫ِعضأج ِعّ‪ٛ‬ع‬
‫التمرين التجريبي ‪ 04(:‬نقاط) ‪:‬‬

‫ذشو‪١‬ض اٌّ‪ٌ ٌٟٛ‬ش‪ٛ‬اسد اٌحذ‪٠‬ذ ػٕذ اٌٍحظح‬

‫[‬ ‫]‬
‫[‬ ‫]‬
‫[‬ ‫]‬

‫طفحح ‪8 ِٓ 8‬‬
‫اجلمهورية اجلزائرية الدميقراطية الشعبية‬
‫وزارة الرتبية الوطنية‬
‫اثنوية حاشي عبد الرمحان ‪-‬مسعد‪-‬‬
‫دورة‪2023 :‬‬ ‫امتحان البكالوراي التجريبية‬
‫الشعبة‪ :‬رايضيات‪ ،‬تقين رايضي‬
‫املدة‪ 04 :‬سا و‪ 30‬د‬ ‫اختبار يف مادة‪ :‬العلوم الفيزايئية‬
‫على المترشح أن يختار أحد الموضوعين اآلتيين‪:‬‬
‫الموضوع األول‬
‫يحتوي الموضوع على (‪ )05‬صفحات (من الصفحة ‪ 01‬من ‪ 10‬إلى الصفحة ‪ 05‬من ‪)10‬‬
‫الجزء األول‪ 14( :‬نقطة)‬
‫التمرين األول‪ 04( :‬نقاط)‬
‫مثؿ كؿ الكواكب في نظامنا الشمسي‪ ،‬تدور األرض والمريخ‬
‫حوؿ الشمس‪ .‬لكف األرض أقرب إلى الشمس‪ ،‬وبالتالي‬
‫األرض‬
‫تتسابؽ عمى طوؿ مدارىا بسرعة أكبر‪ " .‬تقوم األرض‬
‫الشمس‬ ‫تقريبا الذي‬
‫ً‬ ‫برحمتين حول الشمس في نفس الوقت‬
‫المريخ‬ ‫يستغرقه المريخ لمقيام برحمة واحدة "‪ .‬لذلؾ في بعض‬
‫الشكل‪ :1 .‬رسم يوضح مسارات بعض الكواكب حول الشمس‪.‬‬ ‫األحياف يكوف الكوكباف عمى جانبيف متقابميف مف الشمس‪،‬‬
‫جدا‪ ،‬وفي أحياف أخرى‪ ،‬تمحؽ األرض بجارتيا‬ ‫متباعداف ً‬
‫نسبيا في ظاىرة تدعى بػ"التقابؿ"‪.‬‬
‫وتمر بالقرب منيا ً‬
‫يهدف التمرين إلى دراسة بعض مميزات كوكبي األرض والمريخ‪ ،‬والتعرف عمى ظاهرة "التقابل"‪.‬‬
‫‪ -‬المعطيات‪:‬‬
‫‪ -‬كتمة الشمس‪M S  2 10 kg 7‬‬
‫‪30‬‬
‫‪G  6,67  10‬‬ ‫‪11‬‬
‫‪ -‬ثابت التجاذب الكوني‪SI 7‬‬
‫‪ -‬الوحدة الفمكية ‪1U .A  1,5 108 km 7U . A‬‬

‫يوضح الشكؿ‪ 1.‬نظرة العالـ كبمر لحركة الكواكب حوؿ الشمس في بداية دراستو‪ .‬اشرح ذلؾ‬
‫ّ‬ ‫‪.1‬‬
‫‪ .0‬نعتبر أف حركة كؿ مف األرض والمريخ حوؿ الشمس دائرية منتظمة‪( .‬نيمؿ باقي القوى المؤثرة عمى الكوكبيف‬
‫أماـ تأثير قوة الجذب العاـ التي تطبقيا الشمس)‪.‬‬
‫وعرفو‪.‬‬
‫حدد المرجع المناسب لمدراسة‪ّ ،‬‬‫‪ّ .1.0‬‬
‫‪ّ ،  P ‬بيف ّ‬
‫أف عبارة سرعتو المدارية ‪ v P‬تكتب‬ ‫‪ .0.0‬بتطبيؽ القانوف الثاني لنيوتف عمى مركز عطالة كوكب‬
‫‪G M S‬‬
‫‪vP ‬‬ ‫عمى الشكؿ التالي‪7‬‬
‫‪rP‬‬
‫حيث ‪ M S‬كتمة الشمس ‪  S ‬و ‪ rP‬نصؼ قطر مدار الكوكب ‪.  P ‬‬
‫‪ .0.0‬استنتج عبارة القانوف الثالث لكبمر‪.‬‬

‫صفحة ‪ 1‬من ‪10‬‬


‫اختبار يف مادة‪ :‬العلوم الفيزايئية‪ .‬الشعبة‪ :‬رايضيات‪ ،‬تقين رايضي بكالوراي جتريبية ‪2023‬‬

‫‪ .1.0‬الجدوؿ التالي يوضح بعض‬


‫المريخ‬ ‫األرض‬ ‫الكوكب‬ ‫خصائص الكواكب المدروسة‪7‬‬
‫الدور‬ ‫‪ -‬أنقؿ الجدوؿ عمى ورقة اإلجابة وأكممو‪.‬‬
‫نصف قطر المدار‬ ‫‪ .0‬خالؿ سنوات مضت وقعت ظاىرة فمكية‬
‫تدعى "التقابؿ"‪ ،‬بحيث يكوف المريخ‪ ،‬األرض‬
‫والشمس عمى استقامة واحدة بحيث يمكف‬
‫مشاىدتو باستعماؿ تيميسكوب أو حتى بعض‬
‫المرات بواسطة العيف المجردة‪.‬‬
‫‪TT‬‬
‫بيف دور األرض ودور المريخ حوؿ الشمس‪.‬‬ ‫‪ .1.0‬أحسب النسبة‬
‫‪TM‬‬
‫تقريبا الذي يستغرقه المريخ‬
‫ً‬ ‫صحة العبارة " تقوم األرض برحمتين حول الشمس في نفس الوقت‬
‫‪ .0.0‬ناقش ّ‬
‫لمقيام برحمة واحدة "‪.‬‬

‫التمرين الثاني‪ 04( :‬نقاط)‬


‫"تيي" ىي ممكة مصرية قديمة ُعثر عمى مومياءىا في‬
‫مقبرة بوادي المموؾ سنة ‪1565‬ـ؛ وتـ ﻣﻌرﻓﺔ عف أنيا‬
‫المومياء الممقبة بػ "السيدة العظيمة" وذلؾ في عاـ ‪.0212‬‬
‫الدراسات األولية التي تمت عمى المومياء بينت مبدئيا أنيا توفيت قبؿ ‪ 0222‬إلى ‪ 1222‬سنة‪.‬‬
‫يهدف التمرين إلى دراسة تفكك الكربون المشع وتحديد تاريخ وفاة الممكة تي‪.‬‬
‫يمثؿ الشكؿ‪ 2.‬جزء مف مخطط ‪  N  Z ‬حيث تمثؿ المنطقة المظممة‬
‫وادي االستقرار الذي يشمؿ األنوية المستقرة‪.‬‬
‫‪ .1‬عرؼ ما يمي‪ 7‬ﻧوﺍﺓ ﻣﺸﻌﺔ‪ ،‬تفكؾ ‪.  ‬‬
‫‪ .0‬اعتمادا عمى الشكؿ‪ ، 2.‬اكتب معادلة تفكؾ النواة ‪ 146C‬مع تحديد النواة‬
‫والجسيـ الصادر‪.‬‬ ‫‪A‬‬
‫‪Z‬‬ ‫البنت الناتجة ‪X‬‬
‫‪ .0‬دراسة النشاط االشعاعي لعينة مشعة مف الكربوف ‪ 11‬مكنتنا‬

‫الشكل‪ 2 .‬جزء من المخطط‬


‫مف الحصوؿ عمى الشكؿ‪ 3.‬يمثؿ تغيرات ‪ ln A t ‬لعينة مشعة‬
‫مف الكربوف ‪ 14‬بداللة الزمف‪.‬‬
‫‪ .1.0‬أعط عبارة قانوف النشاط االشعاعي ‪، A t ‬‬
‫وبيف ّأنو يكتب عمى الش ػػكؿ‪ln A t   .t  ln A0 7‬‬
‫ّ‬
‫‪ .0.0‬استنتج بيانيا ثابت النشاط االشعاع ‪، ‬‬

‫صفحة ‪ 2‬من ‪10‬‬


‫اختبار يف مادة‪ :‬العلوم الفيزايئية‪ .‬الشعبة‪ :‬رايضيات‪ ،‬تقين رايضي بكالوراي جتريبية ‪2023‬‬

‫أف قيمة زمف نصؼ العمر ‪ t 1/2‬لمكربوف ‪ 11‬ىي ‪. 5730ans‬‬


‫وبيف ّ‬
‫ّ‬
‫عينة مف المومياء‪ ،‬وتـ قياس نشاطيا االشعاعي‬
‫‪ .0.0‬أخذت ّ‬
‫ﺳﻨﺔ ‪ 2010‬فأعطى القيمة ‪ 0,154 Bq‬في حيف أف نشاط عينة حية‬
‫مماثمة ليا في الكتمة ىو ‪. 0,230Bq‬‬
‫‪ -‬حدد تاريخ وفاة "الممكة تي"‪.‬‬
‫‪ .1.0‬حسب النتائج المحسوبة سابقا‪ ،‬وضح إف كانت النتائج األولية صحيحة‪.‬‬
‫بداللة الزمن‬ ‫الشكل‪ 3 .‬تغيرات‬
‫التمرين الثالث‪ 06( :‬نقاط)‬
‫ٗسون اُزشً‪٤‬ت اُزدش‪٣‬ج‪ ٢‬أُج‪ ٖ٤‬ك‪ ٢‬اُشٌَ (‪ ٝ ) 4‬أٌُ‪ ٖٓ ٕٞ‬اُؼ٘بطش اٌُ‪ٜ‬شثبئ‪٤‬خ اُزبُ‪٤‬خ ‪:‬‬
‫‪.‬‬ ‫‪ُٞٓ -‬ذ ر‪ٞ‬رش ثبثذ ه‪ٞ‬ر‪ ٚ‬أُسشًخ اٌُ‪ٜ‬شثبئ‪٤‬خ‬
‫‪ٓ ٝ‬وب‪ٓٝ‬ز‪ٜ‬ب ‪.‬‬ ‫‪ٝ -‬ش‪٤‬ؼخ رار‪٤‬ز‪ٜ‬ب‬
‫‪.‬‬ ‫‪ٝ‬‬ ‫‪ٗ -‬بهالٕ أ‪٤ٓٝ‬بٕ ٓو‪ٓٞ‬ز‪ٜٔ‬ب‬
‫‪ ٝ‬أعالى اُز‪ٞ‬ط‪. َ٤‬‬ ‫‪ -‬هبؽؼخ ً‪ٜ‬شثبئ‪٤‬خ‬
‫ٗـِن اُوبؽؼخ ‪ ٝ‬ثبالػزٔبد ػِ‪ٗ ٠‬زبئح اُذساعخ اُزدش‪٣‬ج‪٤‬خ ‪ ٝ‬ثشٓد‪٤‬خ إػالّ آُ‪ ٢‬رٌٔ٘ب ٖٓ سعْ‬ ‫ػ٘ذ اُِسظخ‬
‫‪ًٔ ،‬ب ‪ٓ ٞٛ‬ج‪ ٖ٤‬ك‪ ٢‬اُشٌَ (‪. )5‬‬ ‫‪( )،‬‬ ‫‪( )،‬‬ ‫أُ٘س٘‪٤‬بد اُج‪٤‬بٗ‪٤‬خ ‪( ) :‬‬
‫‪ٝ -1‬ػر ػِ‪ ٠‬اُذاسح ثأع‪ ْٜ‬خ‪ٜ‬خ اُز‪ٞ‬رش ث‪ ٖ٤‬ؽشك‪ ًَ ٢‬ث٘بئ‪ ٢‬هطت ‪ ٝ‬خ‪ٜ‬خ اُز‪٤‬بس أُبس ك‪ ٢‬اُذاسح ‪.‬‬
‫‪ -2‬أ‪ -‬ثزطج‪٤‬ن هبٗ‪ ٕٞ‬خٔغ اُز‪ٞ‬رشاد خذ أُؼبدُخ اُزلبػِ‪٤‬خ ثذالُخ شذح اُز‪٤‬بس ) (‬
‫‪، ،‬‬ ‫‪،‬‬ ‫ك‪ ٢‬اُ٘ظبّ اُذائْ ثذالُخ‬ ‫ب ‪ -‬اعز٘زح ػجبسح شذح اُز‪٤‬بس اٌُ‪ٜ‬شثبئ‪٢‬‬
‫‪⁄‬‬
‫) (‬ ‫(‬ ‫)‬ ‫خ – ث‪ ٖ٤‬إٔ اُؼجبسح‬

‫ثبثذ اُضٖٓ ‪٣‬طِت رؼ‪ ٖ٤٤‬ػجبسر‪ٚ‬‬ ‫زَ ُِٔؼبدُخ اُزلبػِ‪٤‬خ ز‪٤‬ث‬


‫‪( )،‬‬ ‫‪( )،‬‬ ‫‪ -3‬أًزت اُؼجبساد اُضٓ٘‪٤‬خ ُِز‪ٞ‬رشاد ) (‬
‫‪ -4‬أٗغت ًَ ٓ٘س٘‪ ٠‬ث‪٤‬بٗ‪ ( ) ، ( ) ، ( ) ٢‬إُ‪ ٠‬اُز‪ٞ‬رش أُ‪ٞ‬اكن‬
‫ٓغ اُزؼِ‪. َ٤‬‬ ‫ُ‪ ٚ‬ػِٔب إٔ‬
‫‪ -5‬ثبالػزٔبد ػِ‪ ٠‬أُ٘س٘‪٤‬بد اُثالثخ خذ ه‪ٔ٤‬خ ًَ ٖٓ ‪:‬‬
‫اُشٌَ ( ‪)4‬‬ ‫ػِٔب إٔ شذح اُز‪٤‬بس اٌُ‪ٜ‬شثبئ‪٢‬‬ ‫‪،‬‬ ‫‪، ،‬‬ ‫‪،‬‬
‫ك‪ ٢‬اُ٘ظبّ اُذائْ‬
‫‪.‬‬ ‫‪ -6‬أزغت ه‪ٔ٤‬خ اُطبهخ أُخضٗخ ك‪ ٢‬اُ‪ٞ‬ش‪٤‬ؼخ ك‪ ٢‬اُِسظخ‬

‫صفحة ‪ 3‬من ‪10‬‬


‫اختبار يف مادة‪ :‬العلوم الفيزايئية‪ .‬الشعبة‪ :‬رايضيات‪ ،‬تقين رايضي بكالوراي جتريبية ‪2023‬‬
‫)𝑽(𝒖‬
‫اىشنو (‪) 5‬‬

‫) (‬

‫) (‬

‫) (‬
‫𝟏‬

‫‪0‬‬ ‫𝟎𝟏‬ ‫)𝒔𝒎(𝒕‬


‫الجزء الثاني‪ 06( :‬نقاط)‬
‫التمرين التجريبي‪:‬‬
‫حمض الميثانويك 𝐻𝑂𝑂𝐶𝐻 و يس ّمى أيضا حمض الفورميك‪ ،‬هو أبسط حمض كربوكسيلي‪ ،‬تفرزه النّملة لتتبّع أثرها إلى‬
‫تتعرض للخطر‪ ،‬و يتواجد هذا الحمض أيضا في سم النّحلة‪...‬‬
‫ّ‬ ‫جحرها كما تفرزه أيضا عندما‬
‫يكون هذا الحمض في الحالة السائلة عند درجة الحرارة العادية ) 𝐶‪.(25°‬‬
‫يهدف هذا التمرين إلى التحقق من درجة النقاوة 𝑷 المكتوبة على لصيقة قارورة محلول تجاري لحمض الميثانويك‬
‫بطريقتين (تجربتين)‪.‬‬
‫تحمل لصيقة محلول تجاري ) ‪ (𝑆0‬لحمض الميثانويك المعلومات التالية‪:‬‬
‫‪ -‬الكتلة المولية الجزيئية‪𝑀(𝐶𝐻2 𝑂2 ) = 46 𝑔/𝑚𝑜𝑙 :‬‬
‫‪ -‬الكثافة‪𝑑 = 1,15 :‬‬
‫‪ -‬النسبة المئوية الكتلية (درجة النقاوة)‪𝑃 = 80% :‬‬
‫نأخذ حجما 𝐿𝑚 ‪ 𝑉0 = 2‬من المحلول ) ‪ (𝑆0‬الذي تركيزه المولي ‪ 𝐶0‬و نضيف إليه الماء المقطر لنحصل على محلول‬
‫)𝑆( حجمه 𝐿 ‪ 𝑉𝑆 = 1‬تركيزه المولي 𝐶‪.‬‬
‫التجربة األولى‪:‬‬
‫نأخذ حجما 𝐿𝑚 ‪ 𝑉𝑎 = 50‬من المحلول )𝑆( و نعايره بمحلول هيدروكسيد الصوديوم )) 𝑞𝑎( ‪(𝑁𝑎+ (𝑎𝑞 ) + 𝑂𝐻−‬‬
‫تركيزه المولي 𝐿‪ 𝐶𝑏 = 0,1 𝑚𝑜𝑙/‬و نتابع تغيرات 𝐻𝑝 المزيج بداللة حجم األساس المضاف 𝑏𝑉‪ ،‬فتمكننا من رسم‬
‫]𝐻𝑂𝑂𝐶𝐻[‬ ‫‪1‬‬
‫(الشكل‪.)7-‬‬ ‫البيان ) 𝑏𝑉(𝑓 = 𝐻𝑝 (الشكل‪ ،)6-‬ومكنتنا برمجية خاصة من رسم البيان ) 𝑉( 𝑔 =‬
‫] ‪[𝐻𝐶𝑂𝑂−‬‬ ‫𝑏‬

‫صفحة ‪ 4‬من ‪10‬‬


‫اختبار يف مادة‪ :‬العلوم الفيزايئية‪ .‬الشعبة‪ :‬رايضيات‪ ،‬تقين رايضي بكالوراي جتريبية ‪2023‬‬
‫اكتب المعادلة الكيميائية المنمذجة للتحول الحادث أثناء المعايرة‪.‬‬ ‫‪.1‬‬
‫اعتمادا على بيان الشكل‪ ، 7-‬تأكد أن 𝐿𝑚 ‪.𝑉𝑏𝐸 = 20‬‬ ‫‪.0‬‬
‫احسب التركيز المولي 𝐶 للمحلول )𝑆( ثم استنتج التركيز المولي ‪ 𝐶0‬للمحلول ) ‪.(𝑆0‬‬ ‫‪.0‬‬
‫احسب درجة النقاوة 𝑃 للمحلول ) ‪ .(𝑆0‬هل تتوافق مع ما هو مكتوب على اللصيقة ؟‬ ‫‪.1‬‬
‫‪.5‬‬
‫]𝐻𝑂𝑂𝐶𝐻[‬
‫بيّن أن ] ‪ 𝑝𝐻 = 𝑝𝐾𝑎 − log [𝐻𝐶𝑂𝑂−‬حيث 𝑎𝐾𝑝 هو ثابت الحموضة للثنائية ) ‪.(𝐻𝐶𝑂𝑂𝐻/𝐻𝐶𝑂𝑂−‬‬
‫‪ .6‬جد سلم محور الفواصل لبيان الشكل‪ ،6-‬ثم استنتج قيمة 𝑎𝐾𝑝 للثنائية ) ‪.(𝐻𝐶𝑂𝑂𝐻/𝐻𝐶𝑂𝑂−‬‬
‫𝐻𝑝‬ ‫]𝑯𝑶𝑶𝑪𝑯[‬
‫الشكل‪6 -‬‬ ‫]‪[𝑯𝑪𝑶𝑶−‬‬
‫الشكل‪7 -‬‬

‫‪2,9‬‬
‫𝟏‬
‫) 𝟏‪(𝒎𝑳−‬‬
‫𝒃𝑽‬

‫)𝑳𝒎( 𝒃𝑽‬
‫التجربة الثانية‪:‬‬
‫يتفاعل حمض النمل مع ثنائي البروم وفق المعادلة التالية‪:‬‬
‫)𝑔( ‪𝐻𝐶𝑂𝑂𝐻 (𝑎𝑞 ) + 𝐵𝑟2 (𝑎𝑞 ) = 2𝐵𝑟 − (𝑎𝑞 ) + 2𝐻+ (𝑎𝑞 ) + 𝐶𝑂2‬‬
‫نأخذ حجما 𝐿𝑚 ‪ 𝑉1 = 50‬من المحلول )𝑆( السابق الذي تركيزه المولي 𝐶 (مجهول) و نمزجه في لحظة ‪ 𝑡 = 0‬مع‬
‫حجم 𝐿𝑚 ‪ 𝑉2 = 50‬من محلول ثنائي البروم ‪ 𝐵𝑟2‬تركيزه المولي 𝐿‪.𝐶2 = 6 × 10−2 𝑚𝑜𝑙/‬‬
‫المتابعة الزمنية لتطور كمية مادة الشوارد ‪ 𝐵𝑟 −‬مكنتنا من رسم البيان )𝑡(𝑓 = ) ‪( 𝑛(𝐵𝑟 −‬الشكل‪.)8-‬‬
‫)𝒍𝒐𝒎𝒎( ‪𝒏𝑩𝒓−‬‬
‫‪ .1‬جد كمية المادة االبتدائية لثنائي البروم ) ‪ ، 𝑛0 (𝐵𝑟2‬ثم أنشئ‬
‫جدول تقدّم التفاعل‪.‬‬
‫الشكل‪8 -‬‬ ‫‪ .0‬حدّد قيمة التقدّم األعظمي 𝑥𝑎𝑚𝑥‪ ،‬ثم استنتج المتفاعل المحدّ‪.‬‬
‫‪ .0‬احسب التركيز المولي 𝐶 للمحلول )𝑆(‪.‬‬
‫‪ .1‬استنتج التركيز المولي ‪ 𝐶0‬للمحلول ) ‪.(𝑆0‬‬
‫‪ .5‬تأكد من قيمة درجة النقاوة 𝑃 المحسوبة في التجربة األولى‪.‬‬
‫‪ّ .6‬‬
‫عرف زمن نصف التفاعل ‪ 𝑡1/2‬ثم حدّد قيمته‪.‬‬
‫‪ .7‬احسب السرعة الحجمية للتفاعل عند اللحظة ‪. 𝑡 = 0‬‬
‫)𝒔(𝒕‬
‫انتهى الموضوع األول‬
‫صفحة ‪ 5‬من ‪10‬‬
‫اختبار يف مادة‪ :‬العلوم الفيزايئية‪ .‬الشعبة‪ :‬رايضيات‪ ،‬تقين رايضي بكالوراي جتريبية ‪2023‬‬
‫الموضوع الثاني‬
‫يحتوي الموضوع على (‪ )05‬صفحات (من الصفحة ‪ 06‬من ‪ 10‬إلى الصفحة ‪ 10‬من ‪)10‬‬

‫الجزء األول‪ 14( :‬نقطة)‬


‫التمرين األول‪ 04( :‬نقاط)‬

‫يرتكز إنتاج الطاقة في المفاعالت النووية على االنشطار النووي لليورانيوم ‪ 235‬إال أنه تجرى حاليا أبحاث حول كيفية‬
‫تطوير إنتاج الطاقة النووية وذلك باالعتماد على دمج نوى عنصر الهيدروجين‪.‬‬
‫بنترون بطيء إلى إنتاج‬ ‫‪235‬‬
‫𝑈‪92‬‬ ‫يؤدي تفاعل االنشطار النووي الذي يحدث في قلب مفاعل نووي‪ ،‬إثر تصادم نواة اليورانيوم‬
‫وعدد 𝑥 من النيترونات‪.‬‬ ‫‪94‬‬
‫𝑟𝑆‪38‬‬ ‫و‬ ‫‪140‬‬
‫𝑒𝑋‪54‬‬ ‫أنوية أكثر استقرار‬
‫‪ .1‬أكتب معادلة تفاعل االنشطار النووي‪.‬‬
‫‪ .2‬أحسب بالـ 𝑉𝑒𝑀 الطاقة المحررة ‪ 𝐸𝐿𝑖𝑏1‬عن االنشطار النووي لنواة واحدة من اليورانيوم ‪.235‬‬
‫‪ .3‬علما أن المفاعل النووي يستهلك في اليوم الواحدة كتلة من اليورانيوم ‪ 235‬مقدارها ‪ ،𝑚 = 2,6 𝑘g‬فينتج بذلك طاقة‬
‫كهربائية باستطاعة تحويل متوسطة مقدارها 𝑊𝑀 ‪.𝑃𝑒 = 900‬‬
‫أ‪ -‬أحسب الطاقة الكهربائية 𝑒𝐸 الناتجة خالل يوم واحد‪.‬‬
‫ب‪ -‬أحسب الطاقة المحررة 𝑇𝐸 من المفاعل النووي خالل يوم واحد‪.‬‬
‫جـ‪ -‬باعتبار 𝑟 مردود تحول الطاقة الح اررية إلى الطاقة الكهربائية‪ ،‬أحسب قيمته‪.‬‬
‫‪ .1‬أ‪ -‬أكتب معادلة تفاعل اندماج نظيري الهيدروجين 𝐻‪ 21‬و𝐻‪ 31‬الذي ينتج عنهما نواة الهيليوم 𝑒𝐻‪ 42‬وجسيم 𝑋𝑍𝐴 ‪.‬‬
‫ب‪ -‬جد طاقة ربط نواة الهيليوم ‪ ،4‬ثم استنتج كتلتها بوحدة الكتلة الذرية 𝑢‪.‬‬
‫جـ‪ -‬أحسب بالـ 𝑉𝑒𝑀 الطاقة ‪ 𝐸𝐿𝑖𝑏2‬المحررة عن التفاعل الحادث‪.‬‬
‫د‪ -‬أعط مبررين العتماد االندماج النووي عوض االنشطار النووي في إنتاج الطاقة‪ ،‬مع التعليل‪.‬‬
‫المعطيات‪:‬‬
‫‪−13‬‬ ‫‪−2‬‬
‫‪1 𝑀𝑒𝑉 = 1,6 × 10‬‬ ‫𝐽‬ ‫𝑐 ‪1 𝑢 = 931,5 𝑀𝑒𝑉.‬‬
‫𝑡𝑡𝑎𝑊 ‪1 𝑀𝑊 = 10‬‬ ‫‪6‬‬
‫𝑢 ‪= 3,016049‬‬ ‫) 𝐻‪𝑚( 31‬‬
‫‪𝐸𝑙 4‬‬
‫𝑢 ‪𝑚( 10𝑛) = 1,00866 𝑢 𝑚( 11𝑝) = 1,00728 𝑢 𝑚( 21𝐻 ) = 2,014102‬‬ ‫𝑛‪( 𝐻𝑒) = 7,07 𝑀𝑒𝑉/‬‬
‫‪𝐴 2‬‬
‫‪𝑚( 140‬‬ ‫‪94‬‬ ‫‪235‬‬
‫𝑢 ‪54𝑋𝑒 ) = 139,92164 𝑢 𝑚( 38𝑆𝑟 ) = 93,91154 𝑢 𝑚( 92𝑈) = 235,04393‬‬

‫صفحة ‪ 6‬من ‪10‬‬


‫اختبار يف مادة‪ :‬العلوم الفيزايئية‪ .‬الشعبة‪ :‬رايضيات‪ ،‬تقين رايضي بكالوراي جتريبية ‪2023‬‬
‫التمرين الثاني‪ 04( :‬نقاط)‬
‫اىتىش أ‪ً ٝ‬شح أُؼشة ٗ‪ٞ‬ع ٖٓ س‪٣‬بػبد اُشاذ ‪ٝ‬اُز‪٣ ٢‬ز٘بكظ ك‪ٜ٤‬ب الػجبٕ ‪٣‬سَٔ‬
‫ًَ ٓ٘‪ٜٔ‬ب ٓؼشثب ُؼشة اٌُشح ٗس‪٘ٓ ٞ‬طوخ اُخظْ ‪ِٓ ،‬ؼت اُز٘ظ ػجبسح ػٖ ٓغزط‪َ٤‬‬
‫‪ٝ‬ػؼذ ك‪٘ٓ ٢‬زظل‪ ٚ‬شجٌخ اسرلبػ‪ٜ‬ب‬ ‫‪ٝ‬ػشػ‪ٚ‬‬ ‫ؽ‪ُٚٞ‬‬
‫‪ٝ‬خؾ ‪ٞ٣‬خذ‬
‫ٍ‬ ‫ػ٘ذٓب ‪٣‬شعَ اُالػت اٌُشح ‪٣‬دت إٔ رغوؾ ك‪٘ٓ ٢‬طوخ ٓسظ‪ٞ‬سح ث‪ ٖ٤‬اُشجٌخ‬
‫ٖٓ اُشجٌخ ًٔب ‪ٞٓ ٞٛ‬ػر ك‪ ٢‬اُشٌَ (‪)1‬‬ ‫ػِ‪ ٠‬ثؼذ‬

‫𝑶‬ ‫⃗𝟎𝒗‬ ‫𝒙‬

‫اُشٌَ (‪)1‬‬

‫الشبكـة‬
‫𝑚‪12‬‬ ‫𝑚‪6.4‬‬
‫𝒚‬

‫أُلز‪ٞ‬زخ ‪ٝ‬ازذح ٖٓ ثط‪ٞ‬الد اٌُجش‪( ٟ‬اُـشاٗذ عالّ)‪.‬‬ ‫ك‪ ٢‬ثط‪ُٞ‬خ روالن غاروس‬
‫‪٣‬ش‪٣‬ذ اُالػت رافاٌٍو واداه ‪ٝ‬أُؼش‪ٝ‬ف ثـ ‪ :‬ميل اىمالعة اىتراتٍح اعوبؽ اٌُشح ك‪٘ٓ ٢‬طوخ اُخظْ‬
‫إلٗدبص اإلسعبٍ ‪٣‬وزف واداه اٌُشح ث‪٤‬ذ‪ ٙ‬شبه‪٤ُٞ‬ب ٗس‪ ٞ‬األػِ‪ ٠‬ثْ ‪٣‬ؼشث‪ٜ‬ب ثٔؼشث‪ٗ ٖٓ ٚ‬وطخ (‪ )O‬اسرلبػ‪ٜ‬ب‬
‫⃗‬ ‫ػٖ عطر األسع كز٘طِن ثغشػخ اثزذائ‪٤‬خ أفـقٍح‬
‫ًٔب ‪ٞٓ ٞٛ‬ػر ك‪ ٢‬اُشٌَ (‪)1‬‬
‫‪ٓ .1‬ثَ اُو‪ ٟٞ‬أُؤثشح ػِ‪ٓ ٠‬شًض ػطبُخ اٌُشح ثؼذ ػشة اٌُشح ؟‬
‫‪ .0‬ثزطج‪٤‬ن اُوبٗ‪ ٕٞ‬اُثبٗ‪ٞ٤ُ٘ ٢‬رٖ ‪:‬‬
‫(؟‬ ‫(‪)ٝ‬‬ ‫أ‪ -‬أدسط ؽج‪٤‬ؼخ زشًخ ًشح اُز٘ظ ‪ٝ‬كن أُس‪ٞ‬س‪) ٖ٣‬‬
‫؟‬ ‫‪( )ٝ‬‬ ‫ب‪ -‬أ‪ٝ‬خذ أُؼبدُز‪ ٖ٤‬اُضٓ٘‪٤‬ز‪ُِ ٖ٤‬غشػخ ‪( ) :‬‬
‫؟‬ ‫جـ‪ -‬أ‪ٝ‬خذ أُؼبدُز‪ ٖ٤‬اُضٓ٘‪٤‬ز‪ُِٞٔ ٖ٤‬ػغ ‪( ) ٝ ( ) :‬‬
‫؟‬ ‫د‪ -‬اعز٘زح ٓؼبدُخ أُغبس ) (‬
‫‪ .0‬زذد اُِسظخ اُضٓ٘‪٤‬خ اُز‪ ٢‬رٔش ك‪ٜ٤‬ب اٌُشح ك‪ٞ‬م اُشجٌخ ‪ ،‬أزغت اسرلبػ‪ٜ‬ب ػ٘ذئز ؟‬
‫‪ .1‬ازغت عشػخ اٌُشح ُسظخ ٓش‪ٝ‬س‪ٛ‬ب ك‪ٞ‬م اُشجٌخ ‪ًٝ‬زا اُضا‪٣ٝ‬خ ‪ β‬اُز‪٣ ٢‬ظ٘ؼ‪ٜ‬ب شؼبع اُغشػخ ٓغ األكن ؟‬
‫‪ٗ َٛ .5‬در رافاٌٍو واداه ك‪ ٢‬االسعبٍ ‪ ،‬ﻋﻠﻞ؟‬
‫ٌعطى ‪:‬‬

‫صفحة ‪ 7‬من ‪10‬‬


‫اختبار يف مادة‪ :‬العلوم الفيزايئية‪ .‬الشعبة‪ :‬رايضيات‪ ،‬تقين رايضي بكالوراي جتريبية ‪2023‬‬

‫التمرين الثالث‪ 06( :‬نقاط)‬

‫أ‪ ٝ‬األٓ‪٤ٗٞ‬بى ‪٣‬غزخذّ ُؼالج اُشؼش ز‪٤‬ث رؼبٗ‪ ٢‬اُ٘غبء ٖٓ ٓشٌِخ رغبهؾ اُشؼش ‪ٝ ،‬اُز‪ ٢‬رؼ‪ٞ‬د ُؼذح أعجبة‬ ‫األمىوٍا‬
‫ٓ٘‪ٜ‬ب اُظش‪ٝ‬ف اُج‪٤‬ئ‪٤‬خ أ‪ ٝ‬اعزخذاّ ٓظللبد اُشؼش اُسشاس‪٣‬خ ثئكشاؽ ‪ .‬ز‪٤‬ث أثجذ ٓسِ‪ ٍٞ‬األٓ‪٤ٗٞ‬بى كبػِ‪٤‬خ كبئوخ ك‪ ٢‬اٗجبد‬
‫اُشؼش ثبإلػبكخ إُ‪٘ٓ ٠‬س‪ ٚ‬اٌُثبكخ اُالصٓخ ُظ‪ٜٞ‬س‪ ٙ‬ثشٌَ طس‪ٓٝ ٢‬زأُن ‪ٝ ،‬رُي ػٖ ؽش‪٣‬ن سش‪ ٚ‬ػِ‪ ٠‬اُشؼش ثشٌَ ‪. ٢ٓٞ٣‬‬
‫دراصح اىمحيىه اىمائً ىألمىوٍاك ‪:‬‬ ‫‪-I‬‬
‫‪ .‬أػط‪ ٠‬ه‪٤‬بط‬ ‫‪ ٝ‬رشً‪٤‬ض‪⁄ ٙ‬‬ ‫( زدٔ‪ٚ‬‬ ‫()‬ ‫( ُألٓ‪٤ٗٞ‬بى )‬ ‫ٗؼزجش ٓسِ‪ٞ‬ال ٓبئ‪٤‬ب )‬
‫ٌعطى ‪ :‬اُدذاء اُشبسد‪ُِٔ ١‬بء‬ ‫‪.‬‬ ‫‪ٛ‬زا أُسِ‪ ٍٞ‬اُو‪ٔ٤‬خ‬ ‫اُـ‬
‫‪ -1‬أًزت ٓؼبدُخ رلبػَ األٓ‪٤ٗٞ‬بى ٓغ أُبء ‪.‬‬
‫‪ -2‬أٗشئ خذ‪ ٍٝ‬روذّ اُزلبػَ ‪.‬‬
‫‪ -3‬أ‪ -‬ث‪ ٖ٤‬إٔ ػجبسح ٗغجخ اُزوذّ اُ٘‪ٜ‬بئ‪ُٜ ( )٢‬زا اُزلبػَ رٌزت ٖٓ اُشٌَ‬

‫ة‪ -‬أزغت ه‪ٔ٤‬ز‪ٓ ، ٚ‬برا رغز٘زح ؟‬


‫‪ ،‬أزغت ه‪ٔ٤‬ز‪. ٚ‬‬ ‫‪ٝ‬‬ ‫ػ٘ذ اُز‪ٞ‬اصٕ ثذالُخ‬ ‫‪ -4‬ػجش ػٖ ػجبسح ًغش اُزلبػَ‬
‫(‬ ‫‪⁄‬‬ ‫ُِث٘بئ‪٤‬خ )‬ ‫‪ -5‬أ‪ٝ‬خذ ه‪ٔ٤‬خ اُـ‬
‫(‬ ‫(‬ ‫)‬ ‫(‬ ‫معاٌرج محيىه األمىوٍاك تىاصطح محيىه حمض ميىر اىماء ‪)) :‬‬ ‫‪-II‬‬
‫( زدٔ‪ٚ‬‬ ‫ٗو‪ ّٞ‬ثٔؼب‪٣‬شح ٓسِ‪ٓ ٍٞ‬بئ‪ُ ٢‬ألٓ‪٤ٗٞ‬بى )‬
‫اُشٌَ (‪)2‬‬ ‫ر‪ ١‬اُزشً‪٤‬ض‬ ‫ث‪ٞ‬اعطخ ٓسِ‪ ٍٞ‬زٔغ ًِ‪ٞ‬س أُبء‬ ‫‪ ٝ‬رشً‪٤‬ض‪ٙ‬‬
‫ثو‪٤‬بط اُـ‬ ‫أُ‪⁄ ٢ُٞ‬‬
‫‪ -1‬أًزت ٓؼبدُخ رلبػَ أُؼب‪٣‬شح ‪.‬‬
‫اُخِ‪٤‬ؾ ثذالُخ‬ ‫‪ٔ٣ -2‬ثَ أُ٘س٘‪ ٠‬ك‪ ٢‬اُشٌَ (‪ )2‬رـ‪٤‬ش‬
‫ُسٔغ ًِ‪ٞ‬س أُبء أُؼبف ‪.‬‬ ‫ُِٔسِ‪ٍٞ‬‬ ‫اُسدْ‬
‫) ٗوطخ اُزٌبكؤ‬ ‫أ‪ -‬زذد إزذاث‪٤‬بد (‬
‫ب‪ -‬أزغت اُزشً‪٤‬ض أُ‪٢ُٞ‬‬
‫خ‪ -‬ػ‪ٓ ٖ٤‬ؼِال خ‪ٞ‬اثي ‪ ،‬اٌُبشق أُ٘بعت إلٗدبص‬
‫ٓزش ‪.‬‬ ‫‪ٛ‬ز‪ ٙ‬أُؼب‪٣‬شح ك‪ ٢‬ؿ‪٤‬بة خ‪ٜ‬بص اُـ‬

‫اُـل‪ ٍٞ٘٤‬كـثبُ‪ٖ٤‬‬ ‫أصسم اُجش‪ٞٓٝ‬ر‪ٍٞٔ٤‬‬ ‫أزٔش اٌُِ‪ٞ‬س‪ٝ‬كـ‪ٍٞ٘٤‬‬ ‫اُ‪ٜ‬ـ‪ِ٤‬ـ‪٤‬بٗـز‪ٖ٤‬‬ ‫اىناشف اىميىن‬


‫‪8.0 - 10‬‬ ‫‪6 - 7.6‬‬ ‫‪5.2 - 6.8‬‬ ‫‪3.1 - 4.4‬‬ ‫مجاه اىتغٍر اىيىوً‬

‫صفحة ‪ 8‬من ‪10‬‬


‫اختبار يف مادة‪ :‬العلوم الفيزايئية‪ .‬الشعبة‪ :‬رايضيات‪ ،‬تقين رايضي بكالوراي جتريبية ‪2023‬‬

‫الجزء الثاني‪ 06( :‬نقاط)‬


‫التمرين التجريبي‪:‬‬

‫قاـ سيؼ الديف بتفكيؾ شاحف ىاتفو بعد تعطمو فالحظ وجود دارة إلكترونية‬
‫تـ التطرؽ ليا في وحدة الظواىر الكيربائية مف بينيا‪:‬‬
‫تحتوي عناصر كيربائية ّ‬
‫‪ ‬العنصر (‪ :)1‬أسطوانة سوداء تحمؿ كتابة غير واضحة ‪. (2,2  F ) :‬‬
‫‪ ‬العنصر (‪ :)2‬سمؾ نحاسي معزوؿ وممفوؼ حوؿ شرائح مف الحديد‪.‬‬
‫العنصر (‪)2‬‬ ‫العنصر (‪)1‬‬ ‫الهدف هو التعرف عمى بعض العناصر الكهربائية وايجاد الثوابت‬
‫صورة لمدارة االلكترونية الموجودة في شاحن الهاتف‬ ‫المميزة لها‪.‬‬
‫ّ‬
‫مكونة مف مولد مثالي لمتوتر قوتو المحركة الكيربائية ‪ ، E  6V‬قاطعة ‪، k‬‬
‫أ ُْنجزت الدارة الكيربائية المقابمة ال ّ‬
‫القط التيار لجياز ‪ ، ExAO‬ناقؿ أومي مقاومتو ‪. R‬‬
‫‪ .I‬دراسة العنصر (‪:)1‬‬
‫تعرؼ عمى العنصر(‪ ،)1‬و اذكر مدلوؿ الكتابة غير الواضحة‪.‬‬
‫‪ّ .1‬‬
‫تـ ربط جياز فولطمتر بيف طرفي العنصر‪ 1‬فأشار الى القيمة صفر‬ ‫‪ّ .2‬‬
‫القط التيار‬

‫‪ -‬اعط تفسي ار ليذه النتيجة‪.‬‬

‫ثـ ُنغمؽ القاطعة ‪. k‬‬ ‫‪ .3‬نربط العنصر (‪ )1‬بيف النقطتيف ‪ A‬و ‪B‬‬
‫ّ‬
‫‪di (t ) 1‬‬
‫ثـ استنتج عبارة ‪. 1‬‬
‫‪ّ ،‬‬ ‫أف المعادلة التفاضمية لشدة التيار المار في الدارة ىي‪  i (t )  0 :‬‬
‫‪ّ -‬بيف ّ‬
‫‪dt‬‬ ‫‪1‬‬

‫المتحصؿ عمييا م ّكنت مف رسـ البياف الممثّؿ بالشكؿ (‪. )3‬‬


‫ّ‬ ‫‪ .4‬النتائج‬
‫‪ .1.4‬احسب معامؿ توجيو البياف‪ ،‬و استنتج قيمة ‪. 1‬‬
‫عيف مف البياف قيمة شدة التيار األعظمية ‪. I 0‬‬
‫‪ّ .2.4‬‬
‫أف‪. R  100  :‬‬ ‫‪ .3.4‬تأ ّكد حسابيا ّ‬
‫‪ .4.4‬اختر مف بيف القيمتيف‪ 2,2  F :‬أو ‪ 2 2  F‬القيمة‬
‫الصحيحة لمكتابة غير الواضحة عمى العنصر (‪ )1‬مع التبرير‪.‬‬
‫‪60‬‬

‫‪0‬‬
‫‪0,01‬‬
‫بداللة‬ ‫تغيرات‬
‫الشكل (‪ )3‬يمثل ّ‬

‫صفحة ‪ 9‬من ‪10‬‬


‫اختبار يف مادة‪ :‬العلوم الفيزايئية‪ .‬الشعبة‪ :‬رايضيات‪ ،‬تقين رايضي بكالوراي جتريبية ‪2023‬‬

‫‪.II‬دراسة العنصر (‪:)2‬‬


‫مجددا‪.‬‬
‫ً‬ ‫ثـ نغمؽ القاطعة‬
‫غير العنصر(‪ )1‬بالعنصر(‪ّ )2‬‬‫ون ّ‬
‫نفتح القاطعة ُ‬
‫ِّ‬
‫المميزة لو‪.‬‬ ‫تعرؼ عمى العنصر (‪ ، )2‬و اذكر المقادير‬
‫‪ّ .1‬‬
‫‪ .2‬ارسـ الدارة الكيربائية في ىذه الحالة مع توجيييا( تمثيؿ جية التيار و التوترات الكيربائية)‪.‬‬
‫‪di (t ) R  r‬‬ ‫‪E‬‬
‫‪.‬‬ ‫‪‬‬ ‫‪ i (t ) ‬‬ ‫لتطور شدة التيار المار في الدارة ىي ‪:‬‬
‫أف المعادلة التفاضمية ّ‬ ‫‪ّ .3‬بيف ّ‬
‫‪dt‬‬ ‫‪L‬‬ ‫‪L‬‬
‫‪‬‬ ‫‪t‬‬

‫‪‬‬
‫‪ .4‬حؿ المعادلة التفاضمية السابقة ىو‪i (t )  I 0  1  e  :‬‬
‫‪2‬‬

‫‪-‬جد عبارة الثابتيف ‪ I 0‬و ‪  2‬بداللة مميزات الدارة‪.‬‬


‫‪ .5‬تحصمنا عمى البياف الممثّؿ بالشكؿ (‪.)4‬‬
‫‪ .1.5‬جد بيانيا قيمة كؿ مف ‪  2‬و ‪. I 0‬‬
‫‪ .2.5‬احسب قيمة ‪ ، r‬ماذا يمكنؾ القوؿ بخصوص العنصر(‪)2‬؟‬
‫‪ .3.5‬احسب قيمة المقدار ‪. L‬‬

‫بداللة الزمن‬ ‫تغيرات‬


‫الشكل ‪ّ :4‬‬

‫انتهى الموضوع الثاني‬

‫صفحة ‪ 10‬من ‪10‬‬


‫اإلجابة النموذجية‪ .‬مادة‪ :‬العلوم الفيزايئية‪ .‬الشعبة‪ :‬رايضيات ‪ +‬تقين رايضي ﺍﻟﺒﻜﺎﻟﻮﺭﻳﺎ ﺍﻟﺘﺠﺮﻳﺒﻴﺔ‪2023 :‬‬

‫العالمة‬
‫عناصر اإلجابة ( الموضوع األول)‬
‫مجموع‬ ‫مجزأة‬
‫التمرين األول‪ 04( :‬نقاط)‬
‫‪1‬‬

‫‪0,25‬‬ ‫‪0,25‬‬ ‫‪1‬‬

‫‪0,25‬‬
‫‪0,25‬‬

‫‪0,25‬‬

‫‪0,25‬‬

‫‪0,25‬‬

‫‪3,25‬‬ ‫‪0,25‬‬

‫‪0,5‬‬

‫‪0,25‬‬

‫‪0,5‬‬

‫ﺻﻔﺤﺔ ‪ 1‬ﻣﻦ ‪10‬‬


‫اإلجابة النموذجية‪ .‬مادة‪ :‬العلوم الفيزايئية‪ .‬الشعبة‪ :‬رايضيات ‪ +‬تقين رايضي ﺍﻟﺒﻜﺎﻟﻮﺭﻳﺎ ﺍﻟﺘﺠﺮﻳﺒﻴﺔ‪2023 :‬‬

‫‪0,5‬‬

‫‪0,25‬‬

‫‪0, 5‬‬
‫‪0,25‬‬

‫التمرين الثاني‪ 04( :‬نقاط)‬

‫‪0, 5‬‬

‫‪0,5‬‬

‫‪2‬‬
‫‪0,5‬‬ ‫‪3‬‬
‫‪-1‬‬
‫‪ans‬‬

‫‪ans‬‬

‫‪2010-3342= -1332‬‬
‫‪0,5‬‬ ‫ﺍﺫﻥ ﺗﺎﺭﻳﺦ ﺍﻟوﻓﺎﺓ ‪ 1332‬ﻕﻡ‬ ‫‪3342‬‬ ‫‪ans‬‬
‫‪4‬‬

‫ﺻﻔﺤﺔ ‪ 2‬ﻣﻦ ‪10‬‬


‫اإلجابة النموذجية‪ .‬مادة‪ :‬العلوم الفيزايئية‪ .‬الشعبة‪ :‬رايضيات ‪ +‬تقين رايضي ﺍﻟﺒﻜﺎﻟﻮﺭﻳﺎ ﺍﻟﺘﺠﺮﻳﺒﻴﺔ‪2023 :‬‬

‫التمرين الثالث‪ 06( :‬نقاط)‬


‫‪I‬‬
‫‪ 1‬انرًث‪ٛ‬م‬

‫أ ـ انًؼادنح‬
‫انرفاظه‪ٛ‬ح‬
‫‪.‬‬

‫ٔيُّ ‪:‬‬ ‫ٔ‬ ‫ف‪ ٙ‬انُظاو انذائى ‪:‬‬ ‫ب ـ ػثاسج‬


‫‪2‬‬

‫َؼٕض (‪ )2( ٔ )1‬ف‪ ٙ‬انًؼادنح انرفاظه‪ٛ‬ح‬ ‫خ ـ زم‬


‫‪.‬‬
‫‪.‬‬ ‫انًؼادنح‬

‫(‬ ‫)‬ ‫‪.‬‬ ‫‪.‬‬

‫‪.‬‬
‫‪.‬‬ ‫انؼثاساخ‬
‫‪3‬‬
‫انضيُ‪ٛ‬ح ‪:‬‬

‫انًُسُٗ‬
‫انًُسُٗ‬ ‫‪4‬‬
‫انًُسُٗ‬
‫‪.‬‬
‫‪.‬‬
‫‪.‬‬ ‫‪5‬‬

‫‪.‬‬ ‫‪.‬‬
‫(‬ ‫‪) .‬‬ ‫‪.‬‬ ‫‪ 6‬انطاقح انًخضَح‬
‫‪.‬‬

‫ﺻﻔﺤﺔ ‪ 3‬ﻣﻦ ‪10‬‬


‫اإلجابة النموذجية‪ .‬مادة‪ :‬العلوم الفيزايئية‪ .‬الشعبة‪ :‬رايضيات ‪ +‬تقين رايضي ﺍﻟﺒﻜﺎﻟﻮﺭﻳﺎ ﺍﻟﺘﺠﺮﻳﺒﻴﺔ‪2023 :‬‬

‫الجزء الثاني‪ 06( :‬نقاط)‬


‫التمرين التجريبي‪:‬‬
‫التجربة األولى‪:‬‬
‫‪ -1‬المعادلة الكيميائية المنمذجة للتحول الحادث أثناء المعايرة‬
‫𝟓‪𝟎,2‬‬ ‫‪0,25‬‬ ‫) 𝑙( 𝑂 ‪𝑂𝐻 − (𝑎𝑞) + 𝐻𝐶𝑂𝑂𝐻 (𝑎𝑞 ) = 𝐻𝐶𝑂𝑂 − (𝑎𝑞 ) + 𝐻2‬‬
‫‪ -2‬التأكد من أن 𝑳𝒎 𝟎𝟐 = 𝑬𝒃𝑽‬
‫]𝐻𝑂𝑂𝐶𝐻[‬
‫‪0,25‬‬ ‫عند التكافؤ تكون كمية مادة الحمض معدومة‪ ،‬أي أن ‪= 0‬‬
‫] ‪[𝐻𝐶𝑂𝑂−‬‬
‫𝟓‪𝟎,‬‬ ‫باإلسقاط على محور الفواصل نجد‪:‬‬
‫‪1‬‬ ‫‪1‬‬
‫‪0,25‬‬ ‫= 𝐸𝑏𝑉 ⇒ ‪= 0,05‬‬ ‫𝐿𝑚 ‪= 20‬‬
‫𝐸𝑏𝑉‬ ‫‪0,05‬‬
‫‪ -3‬حساب التركيز المولي 𝑪 للمحلول )𝑺(‬
‫𝐸𝑏𝑉 ‪𝐶. 𝑉𝑎 = 𝐶𝑏 .‬‬ ‫عند التكافؤ يكون‪:‬‬
‫‪𝐶𝑏 . 𝑉𝑏𝐸 0,1 × 20‬‬
‫‪0,25‬‬ ‫=𝐶‬ ‫=‬ ‫𝐿‪= 0,04 𝑚𝑜𝑙/‬‬
‫𝟓‪𝟎,‬‬ ‫𝑎𝑉‬ ‫‪50‬‬
‫استنتاج التركيز المولي ‪ 𝐶0‬للمحلول ) ‪(𝑆0‬‬
‫‪𝐶0 𝑉𝑆 1000‬‬
‫=𝐹‬ ‫= =‬ ‫‪= 500‬‬
‫𝐶‬ ‫‪𝑉0‬‬ ‫‪2‬‬
‫‪0,25‬‬ ‫𝐿‪𝐶0 = 500 × 𝐶 = 500 × 0,04 = 20 𝑚𝑜𝑙/‬‬ ‫و منه‬
‫‪ -4‬حساب درجة النقاوة 𝑷 للمحلول ) 𝟎𝑺(‬
‫𝑀 × ‪𝐶0‬‬ ‫‪20 × 46‬‬
‫‪0,25‬‬ ‫‪0,25‬‬ ‫=𝑃‬ ‫=‬ ‫‪= 80%‬‬
‫‪10 × 𝑑 10 × 1,15‬‬
‫نعم‪ ،‬تتوافق مع ما هو مكتوب على اللصيقة‪.‬‬
‫]𝑯𝑶𝑶𝑪𝑯[‬
‫𝐠𝐨𝐥 ‪𝒑𝑯 = 𝒑𝑲𝒂 −‬‬ ‫‪−‬‬
‫‪ -5‬بيان أن‬
‫] 𝑶𝑶𝑪𝑯[‬
‫] ‪[𝐻𝐶𝑂𝑂− ] × [𝐻3 𝑂 +‬‬
‫‪0,25‬‬ ‫= 𝑎𝐾‬
‫] ‪[𝐻𝐶𝑂𝑂 −‬‬
‫] ‪[𝐻𝐶𝑂𝑂 − ] × [𝐻3 𝑂 +‬‬
‫𝟓𝟕 ‪𝟎,‬‬ ‫‪0,25‬‬ ‫‪𝑝𝐾𝑎 = − log 𝐾𝑎 = − log‬‬
‫] ‪[𝐻𝐶𝑂𝑂 −‬‬
‫] 𝑂𝑂𝐶𝐻[‬‫‪−‬‬ ‫] ‪[𝐻𝐶𝑂𝑂 −‬‬
‫]‪+‬‬
‫]𝐻𝑂𝑂𝐶𝐻[‬
‫[‬
‫‪𝑝𝐾𝑎 = − log 𝐻3 𝑂 − log‬‬ ‫=‬ ‫𝐻𝑝‬ ‫‪−‬‬ ‫‪log‬‬ ‫=‬ ‫𝐻𝑝‬ ‫‪+‬‬ ‫‪log‬‬
‫] ‪[𝐻𝐶𝑂𝑂 −‬‬ ‫] ‪[𝐻𝐶𝑂𝑂 −‬‬ ‫] ‪[𝐻𝐶𝑂𝑂 −‬‬
‫]𝐻𝑂𝑂𝐶𝐻[‬
‫‪0,25‬‬ ‫‪𝑝𝐻 = 𝑝𝐾𝑎 − log‬‬
‫] ‪[𝐻𝐶𝑂𝑂 −‬‬
‫‪ -6‬إيجاد سلّم محور الفواصل لبيان الشكل‪6 -‬‬
‫نعلم أن فاصلة النقطة 𝐸 هي 𝐸𝑏𝑉 إذن 𝐿𝑚 ‪4 𝐶𝑚 → 20‬‬
‫‪0,25‬‬ ‫𝐿𝑚 ‪1 𝐶𝑚 → 5‬‬ ‫و منه فالسلم هو‪:‬‬

‫ﺻﻔﺤﺔ ‪ 4‬ﻣﻦ ‪10‬‬


‫اإلجابة النموذجية‪ .‬مادة‪ :‬العلوم الفيزايئية‪ .‬الشعبة‪ :‬رايضيات ‪ +‬تقين رايضي ﺍﻟﺒﻜﺎﻟﻮﺭﻳﺎ ﺍﻟﺘﺠﺮﻳﺒﻴﺔ‪2023 :‬‬

‫𝑯𝑶𝑶𝑪𝑯‬
‫𝟓𝟕 ‪𝟎,‬‬ ‫(‬ ‫استنتاج قيمة 𝒂𝑲𝒑 للثنائية )‬
‫‪𝑯𝑪𝑶𝑶−‬‬
‫𝐸𝑏𝑉‬ ‫]𝐻𝑂𝑂𝐶𝐻[‬
‫‪0,25‬‬ ‫= 𝐿𝑚 ‪𝑉𝑏 = 10‬‬ ‫من أجل‬ ‫من الشكل ‪ : 7‬يكون ‪= 1‬‬
‫‪2‬‬ ‫] ‪[𝐻𝐶𝑂𝑂−‬‬
‫و عندئذ يكون‪𝑝𝐻 = 𝑝𝐾𝑎 :‬‬
‫بإسقاط قيمة 𝑚 ‪ 𝑉𝑏 = 10‬على بيان الشكل ‪ 6‬نجد‪:‬‬
‫‪0,25‬‬ ‫‪𝑝𝐾𝑎 ≈ 3,8‬‬
‫التجربة الثانية‪:‬‬
‫‪ -1‬إيجاد كمية المادة االبتدائية لثنائي البروم ) 𝟐𝒓𝑩( 𝟎𝒏‬
‫‪0,25‬‬ ‫𝑙𝑜𝑚 ‪𝑛0 (𝐵𝑟2 ) = 𝐶2 × 𝑉2 = 6 × 10−2 × 50 × 10−3 = 3 × 10−3‬‬
‫جدول تقدم التفاعل‬
‫𝟓𝟕 ‪𝟎,‬‬ ‫المعادلة‬ ‫‪−‬‬ ‫‪+‬‬
‫)𝑔( ‪𝐻𝐶𝑂𝑂𝐻(𝑎𝑞 ) + 𝐵𝑟2 (𝑎𝑞 ) = 2𝐵𝑟 (𝑎𝑞 ) + 2𝐻 (𝑎𝑞 ) + 𝐶𝑂2‬‬
‫الحالة‬ ‫التقدم‬ ‫كميات المادة )𝑙𝑜𝑚𝑚(‬
‫ابتدائية‬ ‫‪0‬‬ ‫‪𝐶. 𝑉1‬‬ ‫‪3‬‬ ‫‪0‬‬ ‫‪0‬‬ ‫‪0‬‬
‫‪0,5‬‬
‫انتقالية‬ ‫)𝑡(𝑥‬ ‫)𝑡(𝑥 ‪𝐶. 𝑉1 −‬‬ ‫)𝑡(𝑥 ‪3 −‬‬ ‫)𝑡(𝑥‪2‬‬ ‫)𝑡(𝑥‪2‬‬ ‫)𝑡(𝑥‬
‫نهائية‬ ‫𝑥𝑎𝑚𝑥‬ ‫𝑥𝑎𝑚𝑥 ‪𝐶. 𝑉1 −‬‬ ‫𝑥𝑎𝑚𝑥 ‪3 −‬‬ ‫𝑥𝑎𝑚𝑥‪2‬‬ ‫𝑥𝑎𝑚𝑥‪2‬‬ ‫𝑥𝑎𝑚𝑥‬
‫‪ -2‬تحديد قيمة 𝒙𝒂𝒎𝒙‬
‫من جدول التقدم و من بيان الشكل‪ 8 -‬نجد‪:‬‬
‫𝑙𝑜𝑚𝑚 ‪𝑛𝑓 (𝐵𝑟− ) = 2𝑥𝑚𝑎𝑥 = 4‬‬
‫𝟓‪𝟎,‬‬ ‫‪0,25‬‬ ‫𝑙𝑜𝑚 ‪𝑥𝑚𝑎𝑥 = 2 𝑚𝑚𝑜𝑙 = 2 × 10−3‬‬
‫استنتاج المتفاعل المحد‪:‬‬
‫لدينا ‪𝑛𝑓 (𝐵𝑟2 ) = 3 − 𝑥𝑚𝑎𝑥 = 3 − 2 = 1 𝑚𝑚𝑜𝑙 ≠ 0‬‬
‫‪0,25‬‬ ‫إذن ‪ 𝐵𝑟2‬ليس متفاعال محدّا‬
‫فالمتفاعل المحد هو 𝐻𝑂𝑂𝐶𝐻‪.‬‬
‫‪ -3‬حساب التركيز المولي 𝑪 للمحلول )𝑺(‬
‫بما أن 𝐻𝑂𝑂𝐶𝐻 متفاعل محد‪ ،‬فنجد‪:‬‬
‫‪0,25‬‬ ‫𝑥𝑎𝑚𝑥‬ ‫‪2 × 10‬‬ ‫‪−3‬‬
‫‪0,25‬‬ ‫= 𝐶 ⇒ ‪𝐶. 𝑉1 − 𝑥𝑚𝑎𝑥 = 0‬‬ ‫=‬ ‫𝐿‪= 0,04 𝑚𝑜𝑙/‬‬
‫‪𝑉1‬‬ ‫‪50 × 10−3‬‬
‫‪ -4‬استنتاج التركيز المولي 𝟎𝑪 للمحلول ) 𝟎𝑺(‬
‫𝟓‪𝟎,2‬‬ ‫‪0,25‬‬ ‫𝐿‪𝐶0 = 𝐹 × 𝐶 = 500 × 0,04 = 20 𝑚𝑜𝑙/‬‬
‫‪ -5‬التأكد من قيمة درجة النقاوة 𝑷 المحسوبة سابقا‬
‫‪0,25‬‬ ‫‪0,25‬‬ ‫𝑀 × ‪𝐶0‬‬ ‫‪20 × 46‬‬
‫=𝑃‬ ‫=‬ ‫‪= 80%‬‬
‫‪10 × 𝑑 10 × 1,15‬‬
‫‪ -6‬تعريف زمن نصف التفاعل‬
‫𝑓𝑥‬
‫𝟓‪𝟎,‬‬ ‫‪0,25‬‬ ‫هو الزمن الالزم لبلوغ التفاعل نصف تقدمه النهائي‪ .‬و نكتب = ) ‪𝑥(𝑡1/2‬‬
‫‪2‬‬
‫تحديد قيمة 𝟐‪𝒕𝟏/‬‬
‫‪0,25‬‬ ‫𝑠 ‪𝑡1/2 = 200‬‬
‫‪ -7‬حساب السرعة الحجمية للتفاعل عند اللحظة 𝟎 = 𝒕‬
‫) ‪𝑣𝑣𝑜𝑙 (𝐵𝑟−‬‬
‫‪0,25‬‬ ‫= 𝑙𝑜𝑣𝑣‬
‫‪2‬‬
‫𝟓‪𝟎,‬‬ ‫‪1 1 𝑑𝑛𝐵𝑟−‬‬
‫× = 𝑙𝑜𝑣𝑣‬
‫𝑡𝑑 𝑇𝑉 ‪2‬‬
‫‪0,25‬‬ ‫‪1‬‬ ‫‪1‬‬ ‫‪1−0‬‬
‫× = 𝑙𝑜𝑣𝑣‬ ‫×‬ ‫‪= 6,25 × 10−2 𝑚𝑚𝑜𝑙. 𝑠 −1 . 𝐿−1‬‬
‫‪2 0,1 80 − 0‬‬

‫ﺻﻔﺤﺔ ‪ 5‬ﻣﻦ ‪10‬‬


‫اإلجابة النموذجية‪ .‬مادة‪ :‬العلوم الفيزايئية‪ .‬الشعبة‪ :‬رايضيات ‪ +‬تقين رايضي ﺍﻟﺒﻜﺎﻟﻮﺭﻳﺎ ﺍﻟﺘﺠﺮﻳﺒﻴﺔ‪2023 :‬‬

‫العالمة‬
‫عناصر اإلجابة ( الموضوع الثاني)‬
‫مجموع‬ ‫مجزأة‬
‫الجزء األول‪ 14( :‬نقطة)‬
‫التمرين األول‪ 04( :‬نقاط)‬
‫‪ .1‬معادلة تفاعل االنشطار‪:‬‬
‫‪235‬‬
‫‪0,25‬‬ ‫‪0,25‬‬ ‫𝑈‪92‬‬ ‫‪+ 10𝑛 → 38‬‬
‫‪94‬‬
‫‪𝑆𝑟 + 140‬‬ ‫‪1‬‬
‫𝑛‪54𝑋𝑒 + 𝑥 0‬‬

‫بتطبيق قانوني االنحفاظ‪𝒙 = 𝟐 :‬‬


‫‪ .2‬حساب الطاقة املحررة 𝟏𝒃𝒊𝑳 𝑬 من تفاعل االنشطار‪:‬‬
‫‪𝐸𝐿𝑖𝑏1 = ∆𝑚. 𝑐 2‬‬ ‫نعلم أن‪:‬‬
‫𝟓‪𝟎,‬‬ ‫‪0,5‬‬
‫ومنه‪ 𝐸𝐿𝑖𝑏1 = [𝑚(𝑈) + 𝑚(𝑛) − 𝑚(𝑆𝑟) − 𝑚(𝑋𝑒) − 2𝑚(𝑛)]. 931,5 :‬وعليه‪𝑬𝑳𝒊𝒃𝟏 = 𝟏𝟖𝟖, 𝟐𝟒 𝑴𝒆𝑽 :‬‬
‫‪ .3‬أ‪ -‬حساب الطاقة الكهربائية 𝒆𝑬‪:‬‬
‫‪0,5‬‬ ‫𝐸‬
‫نعلم أن‪ 𝑃𝑒 = 𝑒 → 𝐸𝑒 = 𝑃𝑒 . ∆𝑡 = 900 × 106 × 24 × 3600 :‬وعليه‪𝑬𝒆 = 𝟕, 𝟕𝟕 × 𝟏𝟎𝟏𝟑 𝑱 :‬‬
‫𝑡∆‬
‫ب‪ -‬حساب الطاقة الكلية املحررة 𝑻𝑬‪:‬‬
‫𝟓‪1,2‬‬ ‫‪0,5‬‬ ‫‪2,6 × 103 × 6,02 × 1023‬‬ ‫𝐴𝑁 ‪𝑚.‬‬
‫= 𝑇𝐸‬ ‫= 𝑇𝐸 تطبيق عددي‪× 188,24 × 1,6 × 10−13:‬‬ ‫لدينا‪. 𝐸𝐿𝑖𝑏1 :‬‬
‫‪235‬‬ ‫𝑀‬
‫إذن‪𝑬𝑻 = 𝟐 × 𝟏𝟎𝟏𝟒 𝑱 :‬‬
‫جـ‪ -‬حساب املردود 𝒓‪:‬‬
‫‪0,25‬‬ ‫𝑒𝐸‬ ‫‪100 × 7,77 × 1013‬‬
‫إذن‪𝒓 = 𝟑𝟖, 𝟖𝟓 % :‬‬ ‫=𝑟‬ ‫= ‪× 100‬‬ ‫‪= 38,85 %‬‬
‫𝑇𝐸‬ ‫‪2 × 1014‬‬
‫‪2‬‬
‫𝐻‪1‬‬ ‫‪ .4‬أ‪ -‬معادلة تفاعل االندماج‪+ 31𝐻 → 42𝐻𝑒 + 𝐴𝑍𝑋 :‬‬
‫‪0,25‬‬ ‫‪2‬‬
‫𝐻‪1‬‬ ‫𝑛‪+ 31𝐻 → 42𝐻𝑒 + 10‬‬ ‫‪ 𝐴 = 1‬إذن‪:‬‬ ‫بتطبيق قانوني االنحفاظ‪𝑍 = 0 :‬‬
‫ب‪ -‬حساب طاقة الربط لـ )𝒆𝑯𝟐𝟒 ( 𝒍𝑬 و)𝒆𝑯𝟐𝟒 (𝒎‪:‬‬
‫‪0,75‬‬ ‫𝑉𝑒𝑀 ‪𝐸𝑙 ( 42𝐻𝑒) = 4 × 7,07 = 28,28‬‬
‫‪𝐸𝑙 ( 42𝐻𝑒) 28,28‬‬
‫= 𝑚∆‬
‫‪931,5‬‬
‫=‬
‫‪931,5‬‬
‫𝒖 𝟑𝟎 ‪= 𝟎,‬‬ ‫ونعلم أن‪:‬‬
‫‪2 ,0‬‬ ‫𝑢 ‪𝑚 ( 42𝐻𝑒) = 2. 𝑚𝑝 + 2. 𝑚𝑛 − ∆𝑚 = 4,00155‬‬ ‫منه‪:‬‬
‫جـ‪ -‬حساب الطاقة املحررة 𝟐𝒃𝒊𝑳𝑬‪:‬‬
‫‪0,5‬‬ ‫𝑽𝒆𝑴 ‪𝑬𝑳𝒊𝒃2 = 𝟏𝟖, 58‬‬ ‫وعليه‪:‬‬ ‫‪2‬‬ ‫‪3‬‬ ‫‪4‬‬
‫‪𝐸𝐿𝑖𝑏2 = [𝑚 ( 1𝐻) + 𝑚 ( 1𝐻) − 𝑚 ( 2𝐻𝑒) − 𝑚( 10𝑛)] . 931,5‬‬

‫د‪ -‬املقارنة بين تفاعلي االنشطار واالندماج‪:‬‬


‫‪0,5‬‬ ‫‪𝐸𝐿𝑖𝑏2 𝐸𝐿𝑖𝑏1‬‬
‫>‬ ‫‪ -‬االندماج يحرر طاقة أكبر منه في االنشطار‪.‬‬
‫𝐴‬ ‫𝐴‬
‫‪ -‬االندماج صديق للبيئة‪.‬‬
‫‪ -‬وفرة املادة األولية في الطبيعة‪.‬‬

‫ﺻﻔﺤﺔ ‪ 6‬ﻣﻦ ‪10‬‬


‫اإلجابة النموذجية‪ .‬مادة‪ :‬العلوم الفيزايئية‪ .‬الشعبة‪ :‬رايضيات ‪ +‬تقين رايضي ﺍﻟﺒﻜﺎﻟﻮﺭﻳﺎ ﺍﻟﺘﺠﺮﻳﺒﻴﺔ‪2023 :‬‬

‫التمرين الثاني‪ 04( :‬نقاط)‬


‫‪I‬‬
‫⃗⃗⃗⃗⃗‬
‫𝟎𝒗‬ ‫𝒙‬ ‫⃗∑‬ ‫‪⃗.‬‬
‫𝑶‬ ‫‪ 1‬انرًث‪ٛ‬م‬
‫⃗⃗‬ ‫⃗‬
‫تاإلسقاط ػهٗ انًسٕس‪:ٍٚ‬‬
‫⃗⃗‬
‫𝒑‬
‫دساسح‬
‫طث‪ٛ‬ؼح‬
‫) يسرق‪ًٛ‬ح يُرظًح‬ ‫انسشكح ٔفق (‬
‫𝒚‬ ‫انسشكح‬
‫) يسرق‪ًٛ‬ح يرساسػح‬ ‫انسشكح ٔفق (‬
‫تاَرظاو‬
‫‪.‬‬ ‫تانركايم‬ ‫‪.‬‬ ‫تانركايم‬
‫𝑡‬ ‫𝑡‬ ‫انًؼادنر‪ٍٛ‬‬
‫انضيُ‪ٛ‬ر‪ٍٛ‬‬
‫𝟓 ‪2 ,2‬‬ ‫‪ 2‬نهسشػح‬

‫تانركايم‬
‫تانركايم‬
‫انًؼادنر‪ٍٛ‬‬
‫𝑡‬ ‫𝑡 ‪.‬‬
‫انضيُ‪ٛ‬ر‪ٍٛ‬‬
‫نهًٕظغ‬
‫‪.‬‬
‫‪.‬‬ ‫‪.‬‬ ‫يؼادنح انًساس‬

‫انهسظح‬

‫𝟓‪𝟎,‬‬ ‫‪3‬‬
‫‪.‬‬
‫االسذفاع‬

‫√‬ ‫‪.‬‬
‫انسشػح‬
‫𝟓‪𝟎,‬‬
‫√‬ ‫‪.‬‬ ‫‪4‬‬
‫‪.‬‬ ‫‪.‬‬ ‫انضأ‪ٚ‬ح‬

‫‪.‬‬ ‫√‬ ‫√‬ ‫‪.‬‬


‫𝟓‪𝟎,‬‬ ‫‪ 5‬زساب انًذٖ‬
‫ن‪ٛ‬كٌٕ االسسال َاخر ‪ٚ‬دة أٌ ذقغ انكشج ف‪ ٙ‬انًُطقٕ انًسصٕسج ت‪ ٍٛ‬انشثكح‬
‫يٍ انشثكح أ٘ ‪:‬‬ ‫ٔخط ‪ٕٚ‬خذ ػهٗ تؼذ‬
‫ٔيُّ نى ‪ُٚ‬در سافا‪ٛٚ‬م َادال ف‪ ٙ‬االسسال ألٌ ‪:‬‬

‫ﺻﻔﺤﺔ ‪ 7‬ﻣﻦ ‪10‬‬


‫اإلجابة النموذجية‪ .‬مادة‪ :‬العلوم الفيزايئية‪ .‬الشعبة‪ :‬رايضيات ‪ +‬تقين رايضي ﺍﻟﺒﻜﺎﻟﻮﺭﻳﺎ ﺍﻟﺘﺠﺮﻳﺒﻴﺔ‪2023 :‬‬

‫التمرين الثالث‪ 06( :‬نقاط)‬


‫‪I‬‬
‫‪ 1‬انًؼادنح‬
‫‪0‬‬ ‫‪0‬‬ ‫زانح إترذائ‪ٛ‬ح‬
‫تٕفشج‬ ‫زانح إَرقان‪ٛ‬ح‬ ‫‪ 2‬خذٔل انرقذو‬
‫زانح َٓائ‪ٛ‬ح‬
‫[‬ ‫]‬ ‫[‬ ‫]‬
‫انؼثاسج ‪:‬‬
‫[‬ ‫]‬
‫‪τ‬‬ ‫‪. τ‬‬ ‫‪.‬‬
‫‪2‬‬ ‫‪3‬‬
‫‪τ‬‬ ‫‪.‬‬ ‫ق‪ًٛ‬رّ‬
‫َسرُرح أٌ انرفاػم غ‪ٛ‬ش ذاو‬
‫االسرُراج‬
‫أساط ظؼ‪ٛ‬ف‬ ‫ٔاألساط‬
‫[‬ ‫]‬ ‫‪τ‬‬ ‫[‬ ‫]‬ ‫[‬ ‫]‬
‫[‬ ‫[ ]‬ ‫]‬ ‫[‬ ‫]‬ ‫‪τ‬‬ ‫‪τ‬‬
‫‪0‬‬ ‫[‬ ‫]‬ ‫[‬ ‫]‬ ‫[‬ ‫]‬ ‫‪τ‬‬
‫‪.‬‬ ‫‪ 4‬كسش انرفاػم‬
‫‪τ‬‬
‫‪.‬‬
‫‪τ‬‬
‫[‬ ‫]‬ ‫[‬ ‫]‬ ‫[‬ ‫]‬
‫[‬ ‫]‬ ‫[‬ ‫]‬
‫‪pKa 5‬‬

‫‪II‬‬
‫‪ 1‬انًؼادنح‬
‫االزذاث‪ٛ‬اخ‬

‫‪.‬‬ ‫انرشك‪ٛ‬ض انًٕن‪ٙ‬‬

‫‪2‬‬

‫انكاشف ْٕ أزًش انكهٕسٔف‪ُٕٛ‬ل‬


‫ذُرً‪ ٙ‬إنٗ يدال‬ ‫ألٌ ق‪ًٛ‬ح‬ ‫انكاشف‬
‫ذغ‪ٛ‬شِ انهَٕ‪ٙ‬‬

‫ﺻﻔﺤﺔ ‪ 8‬ﻣﻦ ‪10‬‬


‫اإلجابة النموذجية‪ .‬مادة‪ :‬العلوم الفيزايئية‪ .‬الشعبة‪ :‬رايضيات ‪ +‬تقين رايضي ﺍﻟﺒﻜﺎﻟﻮﺭﻳﺎ ﺍﻟﺘﺠﺮﻳﺒﻴﺔ‪2023 :‬‬

‫الجزء الثاني‪ 06( :‬نقاط)‬


‫التمرين التجريبي‪:‬‬
‫‪0,25‬‬ ‫‪0,25‬‬
‫‪01‬‬

‫‪ .2‬تفسير النتيجة المتحصل عليها‪:‬‬


‫‪0,25‬‬ ‫‪0,25‬‬ ‫الفولط متر اعطى القيمة صفر بين طرفي المكثفة يعني ان التوتر بين طرفيها معدوم‬
‫إذن المكثفة غير مشحونة ألن 𝐶𝑈 × ‪𝑞 = C‬‬

‫‪di (t ) 1‬‬
‫‪  i (t )  0‬‬
‫‪dt‬‬ ‫‪1‬‬
‫) ‪q(t‬‬
‫‪0,25‬‬ ‫‪ R  i (t )  E‬‬ ‫‪uC (t )  u R (t )  E‬‬
‫‪C‬‬
‫‪0,75‬‬ ‫) ‪di (t‬‬ ‫‪1‬‬ ‫) ‪1 dq(t‬‬ ‫) ‪di (t‬‬
‫‪0,25‬‬ ‫‪‬‬ ‫‪ i (t )  0‬‬ ‫‪‬‬ ‫‪R ‬‬ ‫‪0‬‬
‫‪dt‬‬ ‫‪RC‬‬ ‫‪C dt‬‬ ‫‪dt‬‬

‫‪0,25‬‬ ‫‪ 1  RC‬‬ ‫‪1‬‬

‫‪ .4.4‬حساب معامل تىجيه البيان واستنتاج قيمة ‪ 1‬‬


‫‪0,25‬‬
‫‪1‬‬ ‫‪1‬‬
‫‪0,25‬‬ ‫‪a‬‬ ‫‪‬‬
‫‪RC  1‬‬
‫‪1‬‬ ‫‪1‬‬
‫‪0,25‬‬ ‫‪1 ‬‬ ‫‪ 2, 2  104 s‬‬ ‫‪1 ‬‬
‫‪4500‬‬ ‫‪a‬‬

‫‪0,25‬‬
‫‪R  100 ‬‬
‫‪0,25‬‬ ‫‪E‬‬ ‫‪E‬‬
‫‪R  100 ‬‬ ‫‪R ‬‬ ‫‪I0 ‬‬
‫‪I0‬‬ ‫‪R‬‬
‫‪2,2  F‬‬
‫‪0,25‬‬ ‫‪1‬‬
‫‪C  2,2  106 F  2,2  F‬‬ ‫‪C ‬‬ ‫‪1  R  C‬‬
‫‪R‬‬

‫ﺻﻔﺤﺔ ‪ 9‬ﻣﻦ ‪10‬‬


2023 :‫ تقين رايضي ﺍﻟﺒﻜﺎﻟﻮﺭﻳﺎ ﺍﻟﺘﺠﺮﻳﺒﻴﺔ‬+ ‫ رايضيات‬:‫ الشعبة‬.‫ العلوم الفيزايئية‬:‫ مادة‬.‫اإلجابة النموذجية‬

0,25 0,25 02

0,25 0,25

di (t ) R  r E
  i (t ) 
dt L L
di (t )
0,5
0,25 R  i (t )  r  i (t )  L E u R (t )  u B (t )  E
dt
di (t ) R  r E
0,25   i (t ) 
dt L L
 2 I 0

0,25
di (t ) I 0  t 
dt

2
e 2

i (t )  I 0  1  e
t
2

0,75

0,25  2 E L
I 0  2 
R r R r
.5
I 0 2
0,25  2  0, 43 ms  4,3  104 s
di (t ) I di (t ) I 0  t 
0,25  0 t 0  e 2

dt ¨t 0  2 dt 2
di (t )
0,25 I 0  4,3  104  140  0,06 A I 0   2 
1,5 dt ¨t 0
r
E E
0,25 r 0 r R I 0 
I 0 R r
0,25

L L
0,25 L  4,3  102  43 mH 2  2 
R r R r

10 ‫ ﻣﻦ‬10 ‫ﺻﻔﺤﺔ‬
‫اﻝﺠﻤﻬورﻴﺔ اﻝﺠزاﺌرﻴـــﺔ اﻝدﻴﻤﻘراطﻴﺔ اﻝﺸﻌﺒﻴــﺔ‬
‫ﻤدﻴرﻴـــﺎت اﻝﺘرﺒﻴــــﺔ ﻝوﻻﻴــــﺎت‪ :‬ﺘﻴزي وزو‪،‬ﺴطﻴف ‪،‬أم اﻝﺒواﻗﻲ واﻝﺠزاﺌرﻏرب‪.‬‬
‫ﺜﺎﻨوﻴﺎت‪ :‬أﻋﻤر أوﻋﻤران ﻓرﻴﻘﺎت ‪ ،‬ﻤﻔدي زﻜرﻴﺎ ﺴطﻴف ‪ ،‬ﻤﻌﻨﺼرأوﻨﻴس ﻋﻴن ﻜرﺸﺔ ‪ ،‬روﺒﺎل اﻝﻌرﺒﻲ ‪ ،‬ﺜﺎﻨوﻴﺔ ﺨﺎﺼﺔ اﻝﺠزاﺌر‪.‬‬
‫اﻝﺘﺎرﻴـــــﺦ‪2023/05/18 :‬‬ ‫اﻤﺘﺤﺎن ﺒﻜــﺎﻝورﻴﺎ اﻝﺘﺠرﻴﺒـــﻲ اﻝﺘﻌﻠﻴـــــم اﻝﺜﺎﻨوي‬
‫اﻝﻤـــــدة ‪ 04‬ﺴـــﺎ و ‪ 30‬د‬ ‫اﻝﺸﻌﺒﺔ‪ :‬رﻴﺎﻀــﻴـــــــــــــــﺎت ‪،‬ﺘﻘﻨــﻲ رﻴﺎﻀـﻲ‬ ‫اﺨﺘﺒـــﺎر ﻓﻲ ﻤﺎدة ‪ :‬اﻝﻌﻠـــوم اﻝﻔﻴزﻴﺎﺌﻴـــﺔ‬
‫ﻋﻠﻰ اﻝﺘﻠﻤﻴذ أن ﻴﺨﺘﺎر أﺤد اﻝﻤوﻀوﻋﻴن اﻵﺘﻴﻴن‪:‬‬
‫اﻝﻤوﻀوع اﻷول‬
‫ﻴﺤﺘوي اﻝﻤوﻀوع اﻷول ﻋﻠﻰ )‪ (04‬ﺼﻔﺤﺎت) ﻤن اﻝﺼﻔﺤﺔ ‪ 01‬ﻤن ‪ 04‬إﻝﻰ ‪ 04‬ﻤن ‪(09‬‬
‫اﻝﺠزء اﻷول‪ 14 ):‬ﻨﻘطـﺔ (‬
‫اﻝﺘـﻤرﻴن اﻷول‪ 05):‬ﻨﻘــﺎط(‬
‫" اﻝﻤﻔﺎﻋل اﻝﻨووي اﻝﺴﻼم ﻫو ﻤﻔﺎﻋل ﻨووي أﺒﺤﺎث ﺠزاﺌري ﻤﺨﺼص ﻹﻨﺘﺎج اﻝﻤواد اﻝﺼﻴدﻻﻨﻴﺔ اﻹﺸﻌﺎﻋﻴﺔ‪ ،‬ﻴﻤﻜﻨﻪ إﻨﺘﺎج ﻗوة‬
‫ﻗدرﻫﺎ ‪ ،15MW‬ﺘم ﺒﻨﺎؤﻩ ﺒﺎﻝﺘﻨﺴﻴق ﻤﻊ اﻝﺼﻴن ﺒﻤﻨطﻘﺔ ﻋﻴن وﺴﺎرة وﻻﻴﺔ اﻝﺠﻠﻔﺔ‪ ،‬ودﺨل اﻝﺨدﻤﺔ ﻤﻨذ ﻋﺎم ‪ 1993‬وﻴﻌﻤل ﺒﺎﻝﻤﺎء‬
‫اﻝﺜﻘﻴل وﻫو ﺨﺎﻀﻊ ﻝﻤراﻗﺒﺔ اﻝوﻜﺎﻝﺔ اﻝدوﻝﻴﺔ ﻝﻠطﺎﻗﺔ اﻝذرﻴﺔ" *** وﻴﻜﻴﺒﻴدﻴﺎ***‬
‫‪ .I‬ﻓﻲ ﻗﻠب ﻤﻔﺎﻋل ﻨووي ﻴﺘم ﻗذف أﻨوﻴﺔ ﻤن اﻝﻴوراﻨﻴوم ‪ 235‬ﺒواﺴطﺔ ﻨﻴﺘروﻨﺎت ﺤ اررﻴﺔ ﻴﻨﻤذج أﺤد اﻝﺘﺤوﻻت اﻝﻨووﻴﺔ ﺒﺎﻝﻤﻌﺎدﻝﺔ‪:‬‬
‫‪.‬‬ ‫‪U + 01n → 134‬‬
‫‪235‬‬
‫‪92‬‬ ‫‪52Te + z Zr + k 0 n‬‬
‫‪99‬‬ ‫‪1‬‬

‫‪ -1‬أﻋط اﺴم ﻫذا اﻝﺘﺤول ؟ ﻫل ﻫو طﺒﻴﻌﻲ أم ﻤﻔﺘﻌل ؟ ﻋرﻓﻪ‪.‬‬


‫‪ -2‬ﻋﻴن ﻜل ﻤن ‪ z‬و ‪ k‬ﻤﺒﻴﻨﺎ اﻝﻘواﻨﻴن اﻝﻤطﺒﻘﺔ؟‬
‫‪ -3‬أﺤﺴب اﻝطﺎﻗﺔ اﻝﻤﺤررة ) ‪ (Elib‬ﻋن اﻨﺸطﺎر ﻨواة واﺤدة ﻤن اﻝﻴوراﻨﻴوم ‪235‬‬
‫ﻤﻘدرة ﺒﺎﻝـ ‪ MeV‬وﺒﺎﻝﺠول ) ‪. ( J‬‬
‫‪= 15‬‬ ‫‪ -4‬ﻋﻠﻤﺎ أن اﻝﻤﻔﺎﻋل اﻝﻨووي ﻴﻨﺘﺞ اﺴﺘطﺎﻋﺔ ﻜﻬرﺒﺎﺌﻴﺔ ﻤﺘوﺴطﺔ ﻤﻘدارﻫﺎ‬
‫إﻋداد اﻷﺴﺎﺘذة ‪-:‬ﺴﺎﻋﻲ ﻤﺴﻌود –ﺒوﻗطﺎﻴﺔ ﻤﺤﻤد –ﻏوﺒﺎل رﻤزي‬ ‫ﺒﻤردود طﺎﻗوي ‪. = 30%‬‬
‫ﺨﻼل ﻴوم‪.‬‬ ‫‪ .1.5‬اﺤﺴب اﻝطﺎﻗﺔ اﻝﻜﻬرﺒﺎﺌﻴﺔ اﻝﻨﺎﺘﺠﺔ‬
‫‪ ′‬ﻋﻨدﺌذ‪.‬‬ ‫‪ .2.5‬اﺤﺴب اﻝطﺎﻗﺔ اﻝﻤﺤررة ﻤن اﻝﻤﻔﺎﻋل اﻝﻨووي‬
‫ﻝﻠﻴوراﻨﻴوم ‪ 235‬اﻝﻤﺴﺘﻬﻠﻜﺔ ﻤن طرف ﻫذا اﻝﻤﻔﺎﻋل اﻝﻨووي ﺨﻼل ﻴوم واﺤد‪.‬‬ ‫‪ .3.5‬اﺴﺘﻨﺘﺞ ﻤﻘدار اﻝﻜﺘﻠﺔ‬
‫ﻤن أﺸﺒﺎﻩ اﻝﻤﻌﺎدن ذو ﻝون رﻤﺎدي ﻓﻀﻲ ‪.‬وﻫو ﻤن اﻝﻌﻨﺎﺼر اﻝﻤﺸﻌﺔ ﻝـ ‪ β −‬وﻴراﻓﻘﻪ إﺸﻌﺎع ‪. γ‬‬ ‫‪134‬‬
‫‪52‬‬‫‪ .II‬اﻝﺘﻴﻠورﻴوم ‪Te‬‬
‫‪ .1‬أﻜﺘب ﻤﻌﺎدﻝﺔ ﺘﻔﻜك ﻨواة اﻝﺘﻴﻠورﻴوم ‪ . 13452Te‬ﻤوﻀﺤﺎ ﻤﺼدري ﻜل ﻤن ‪ β −‬و ‪. γ‬‬
‫)‪A(×1013 Bq‬‬
‫‪ .2‬ﻋرف طﺎﻗﺔ ﺘﻤﺎﺴك اﻝﻨواة‪ .‬وأﻜﺘب ﻋﺒﺎرﺘﻬﺎ‪.‬‬
‫‪.‬‬ ‫‪ .3‬أﺤﺴب طﺎﻗﺔ اﻝﺘﻤﺎﺴك ﻝﻠﻨواة اﻝﺘﻴﻠورﻴوم‬
‫اﻝﺸﻜل‪01‬‬
‫‪ .4‬ﻗﺎرن ﺒﻴن اﺴﺘﻘرار ﺘواة اﻷم وﻨواة اﻝﺒﻨت ﻓﻲ اﻝﺘﻔﻜك اﻝﺴﺎﺒق ﻋﻠﻤﺎ‬
‫‪.‬‬ ‫‪ = 1127,8602‬اﻝﺒﻨت‬ ‫أن!‬
‫‪1‬‬ ‫‪ .5‬ﻹﻴﺠﺎد زﻤن ﻨﺼف اﻝﻌﻤر ﻝﻨواة اﻝﺘﻴﻠورﻴوم ‪ 13452Te‬ﺘﺤﺼﻠﻨﺎ ﺒواﺴطﺔ‬
‫)‪t (min‬‬
‫‪20,9‬‬ ‫ﺒرﻨﺎﻤﺞ ﻤﻌﻠوﻤﺎﺘﻲ ﻋﻠﻰ اﻝﻤﻨﺤﻨﻰ اﻝﺒﻴﺎﻨﻲ ) ‪) A(t ) = f (t‬اﻝﺸﻜل‪(01‬‬
‫أ‪-‬اﺴﺘﻨﺘﺞ ﻤن اﻝﻤﻨﺤﻨﻰ ﻗﻴﻤﺔ اﻝﻨﺸﺎط ‪ A0‬ﻓﻲ اﻝﻠﺤظﺔ ‪. t = 0s‬‬

‫ﺼﻔﺤﺔ ‪ 1‬ﻤن ‪9‬‬


‫‪ .‬ﺒﻜﺎﻝورﻴﺎ ﺘﺠرﻴﺒﻲ ‪2023‬‬ ‫اﻝﺸﻌﺒﺔ‪ :‬رﻴﺎﻀﻴﺎت‪ ،‬ﺘﻘﻨﻲ رﻴﺎﻀﻲ‬ ‫اﺨﺘﺒﺎر ﻓﻲ ﻤﺎدة اﻝﻌﻠوم اﻝﻔﻴزﻴﺎﺌﻴﺔ‪.‬‬
‫ب‪-‬أﻜﺘب ﻋﺒﺎرة ) ‪ A(t‬ﺒدﻻﻝﺔ ‪ λ ، A0‬و ‪ t‬ﺜم اﺴﺘﻨﺘﺞ ﻗﻴﻤﺔ اﻝﻨﺸﺎط اﻻﺸﻌﺎﻋﻲ ﻓﻲ اﻝﻠﺤظﺔ ‪ t = τ‬؟اﺴﺘﻨﺘﺞ ﺜﺎﺒت اﻝزﻤن ‪ τ‬؟‬
‫ج‪-‬ﺒﻴن أن ‪ t1/2‬زﻤن ﻨﺼف اﻝﻌﻤر ﻝـﻨواة اﻝﺘﻴﻠورﻴوم ‪ 13452Te‬ﻴﻌطﻰ ﺒﺎﻝﻌﻼﻗﺔ ‪ t1/2 = τ ln 2‬وأﺤﺴب ﻗﻴﻤﺘﻪ؟ ﺘﺄﻜد ﻤن ذﻝك ﺒﻴﺎﻨﻴﺎ‬
‫‪1an = 365, 25 jours‬‬ ‫اﻝﻤﻌطﻴﺎت‪، 1M W = 10 6W :‬‬
‫‪، N A = 6,023×1023.mol −1 ، c = 3 × 108 m / s ، 1eV = 1,6 ×10−19 J ،‬‬ ‫‪1u = 1,66 ×10−27 kg = 931,5MeV / c2‬‬

‫ا اة أو ا‬ ‫ا‬ ‫ا را م‬ ‫ا د‬ ‫ر م‬ ‫ا‬ ‫م‬ ‫ا ر‬ ‫ا و ن‬ ‫ون‬ ‫ا‬


‫‪235‬‬ ‫‪134‬‬ ‫‪134‬‬ ‫‪99‬‬ ‫‪1‬‬ ‫‪1‬‬
‫ا‬ ‫‪U‬‬
‫‪92‬‬ ‫‪53‬‬ ‫‪I‬‬ ‫‪52‬‬ ‫‪Te‬‬ ‫‪z‬‬ ‫‪Zr‬‬ ‫‪1‬‬ ‫‪p‬‬ ‫‪0‬‬ ‫‪n‬‬
‫ا ‪! " #‬ـ ‪u‬‬ ‫‪234, 9935‬‬ ‫‪133,8808‬‬ ‫‪133,8830‬‬ ‫‪98, 8946‬‬ ‫‪1, 0073‬‬ ‫‪1, 0087‬‬
‫اﻝﺘـﻤرﻴــــــن اﻝﺜﺎﻨــــــــﻲ‪ 04):‬ﻨﻘﺎط(‬
‫ﺘﻌﺘﺒر رﻤﺎﻴﺔ اﻝﻘوس واﻝرﻤﺢ ﻤن أﻗدم اﻝرﻴﺎﻀﺎت اﻝﺘﻲ ﻋرﻓﻬﺎ اﻹﻨﺴﺎن‪ ،‬وﻤﺎ زاﻝت ﺘﻤﺎرس ﺤﺘﻰ ﻴوﻤﻨﺎ ﻫذا‪ .‬وﻤﻨذ ﺒداﻴﺔ ﻤﻌرﻓﺔ اﻹﻨﺴﺎن‬
‫ﺒﻪ ﺤﺘﻰ اﻝﻘرن اﻝﺨﺎﻤس ﻋﺸر ﻜﺎن اﻝرﻤﺢ اﻝرﻓﻴق اﻝداﺌم ﻝﻺﻨﺴﺎن اﻝذي ﻴوﻓر ﻝﻪ اﻷﻤﺎن واﻝطﻌﺎم‪ ،‬وﻤﻨذ ذﻝك اﻝوﻗت ﻝﻌﺒت ﻫذﻩ‬
‫دور ﻤﻬﻤﺎ ﻋﻨد ﺸﻌوب اﻝﻌﺎﻝم اﻝﻤﺨﺘﻠﻔﺔ‪.‬‬
‫اﻝرﻴﺎﻀﺔ ا‬
‫ﻨرﺴل رﻤﺤﺎ اﻨطﻼﻗﺎ ﻤن ﻤن اﻝﻨﻘطﺔ ‪ O‬ﺒﺴرﻋﺔ اﺒﺘداﺌﻴﺔ ‪ v0‬ﺘﺼﻨﻊ زاوﻴﺔ ‪ α‬ﻤﻊ اﻝﻤﺤور اﻷﻓﻘﻲ ) ‪) (Ox‬اﻝﺸﻜل‪.(02‬‬
‫ﻨﻨﻤذج اﻝرﻤﺢ ﺒﻨﻘطﺔ ﻤﺎدﻴﺔ ﻜﺘﻠﺘﻬﺎ ‪ m‬وﻤرﻜز ﻋطﺎﻝﺘﻬﺎ ‪. G‬‬
‫ﻨدرس ﺤرﻜﺔ ﻤرﻜز اﻝﻌطﺎﻝﺔ ‪ G‬ﻓﻲ ﻤﻌﻠم ﻤﺘﻌﺎﻤد وﻤﺘﺠﺎﻨس ) ‪ (o, i , k‬ﻤرﺘﺒط ﺒﺴطﺢ اﻷرض ‪.‬ﻓﻲ ﻜﺎﻤل اﻝﺘﻤرﻴن ﻨﻬﻤل ﻜل ﺘﺄﺜﻴرات‬
‫اﻝﻬواء ﻋﻠﻰ اﻝرﻤﺢ‪.‬‬
‫‪OC = 70m‬‬ ‫اﻝﻤﻌطﻴـــــﺎت‪ α = 4 o ، g = 10m / s 2 :‬و‬
‫‪ .1‬ﻓﻲ أي ﻤرﺠﻊ ﻨدرس ﺤرﻜﺔ اﻝرﻤﺢ؟ وﻤﺎﻫو اﻝﺸرط ﻝﻜﻲ ﻴﻜون ﻫذا اﻝﻤرﺠﻊ ﻏﺎﻝﻴﻠﻴﺎ؟‬
‫‪ .2‬ﺒﺘطﺒﻴق اﻝﻘﺎﻨون اﻝﺜﺎﻨﻲ ﻝﻨﻴوﺘن أوﺠد ﻋﺒﺎرة ﺘﺴﺎرع ﻤرﻜز ﻋطﺎﻝﺔ اﻝرﻤﺢ ﻓﻲ اﻝﻤﻌﻠم ) ‪. (o , i , k‬‬
‫‪$‬‬ ‫‪ .3‬اﺴﺘﻨﺘﺞ طﺒﻴﻌﺔ ﺤرﻜﺔ اﻝرﻤﺢ اﻝﻤﻨﺴوﺒﺔ‬

‫‪$#‬‬ ‫‪#‬‬ ‫ﻝﻠﻤﻌﻠم‪ ،‬وأﻜﺘب اﻝﻤﻌﺎدﻝﺘﻴن اﻝزﻤﻨﺘﻴن‬


‫ﻝﻜل ﻤن اﻝﺴرﻋﺔ ) ‪ v (t‬و اﻝﻤوﻀﻊ‬
‫‪ℎ‬‬ ‫ا دف ‪C‬‬
‫) ‪ oG(t‬ﻋﻠﻰ ﻜل ﻤﺤور‪.‬‬
‫‪-*+‬‬ ‫)(‬
‫‪****+‬‬ ‫‪ .4‬أﻜﺘب ﻤﻌﺎدﻝﺔ اﻝﻤﺴﺎر ) ‪. z = f ( x‬‬
‫&‬ ‫‪ .5‬ﻨﻌﺘﺒر أن اﻝرﻤﺢ ﻴﻨطﻠق ﻤن اﻝﻨﻘطﺔ‬
‫'‬
‫‪,+‬‬ ‫‪%‬‬
‫اا ل‬
‫ل‪-1-‬‬
‫‪02‬‬ ‫اﻝﺘﻲ ﻨﻌﺘﺒرﻫﺎ ﻤﺒدأ ﻝﻸزﻤﻨﺔ ﺒﺴرﻋﺔ‬
‫إﺒﺘداﺌﻴﺔ ‪ ، v0 = 70m / s‬ﺤﻴث‬
‫ﻴﺼﻴب اﻝﻬدف ﻋﻨد اﻝﻨﻘطﺔ )‪ C ( xc = 70m, zc = 0‬ﻋﻨد اﻝﻠﺤظﺔ ‪. tc‬‬
‫أ‪ .‬أوﺠد ﻋﺒﺎرة ‪ tc‬ﺒدﻻﻝﺔ ﻜل ﻤن ‪ α ، v0‬و ‪ ، xc‬ﺜم أﺤﺴب ﻗﻴﻤﺘﻬﺎ‪.‬‬
‫ب‪ .‬ﻝﺘﻜن ‪ A‬ﻫدﻓﺎ آﺨرا‪ ،‬أﻨظر اﻝﺸﻜل‪ .‬ﻋﻨد ﺒﻠوغ اﻝرﻤﺢ اﻝﻬدف ‪ ، A‬ﻴﺴﻘط ﻤن ارﺘﻔﺎع ‪. ( h = AC ) h‬‬
‫ج‪ .‬ﻤﺎﻫﻲ اﻝﻔرﻀﻴﺔ اﻝﺘﻲ ﺘﺴﻤﺢ ﺒﺄن ﻴﺒﻠﻎ اﻝرﻤﺢ اﻝﻨﻘطﺔ ‪ ، A‬وﻓق اﻝﻤﺴﺎر ) ‪ (OA‬ﻤﻊ اﻝﺤﻔﺎظ ﻋﻠﻰ ﻨﻔس اﻝﺸروط‬
‫اﻻﺒﺘداﺌﻴﺔ؟ﻋﻠل ﺠواﺒك؟‬
‫إﻋداد اﻷﺴﺎﺘذة ‪-:‬ﺴﺎﻋﻲ ﻤﺴﻌود –ﺒوﻗطﺎﻴﺔ ﻤﺤﻤد –ﻏوﺒﺎل رﻤزي‬

‫ﺼﻔﺤﺔ ‪ 2‬ﻤن ‪9‬‬


‫‪ .‬ﺒﻜﺎﻝورﻴﺎ ﺘﺠرﻴﺒﻲ ‪2023‬‬ ‫اﻝﺸﻌﺒﺔ‪ :‬رﻴﺎﻀﻴﺎت‪ ،‬ﺘﻘﻨﻲ رﻴﺎﻀﻲ‬ ‫اﺨﺘﺒﺎر ﻓﻲ ﻤﺎدة اﻝﻌﻠوم اﻝﻔﻴزﻴﺎﺌﻴﺔ‪.‬‬
‫د‪ .‬ﻨﻌﺘﺒر أن ﻤدة ﺒﻠوغ اﻝرﻤﺢ اﻝﻨﻘطﺔ ‪ A‬وﺒﻠوﻏﻪ اﻝﻨﻘطﺔ ‪ C‬ﻫﻲ ﻨﻔﺴﻬﺎ‪.‬ﻋﺒر ﻓﻲ ﻫذﻩ اﻝﺸروط ﻋن ﻤﺴﺎﻓﺔ اﻝﺴﻘوط ‪ h‬ﺒدﻻﻝﺔ‬
‫‪ α ، v0‬و ‪. tc‬‬
‫‪1‬‬
‫ه‪ .‬ﺒﺎﺴﺘﻌﻤﺎل اﻝﻤﻌﺎدﻝﺔ اﻝزﻤﻨﻴﺔ ) ‪ ، z ( t‬ﺒﻴن أن ﻋﺒﺎرة اﻻرﺘﻔﺎع ‪ h‬ﻓﻲ ﻫذﻩ اﻝﺤﺎﻝﺔ ﻫو‪ h = gtc2 :‬ﺜم أﺤﺴب ﻗﻴﻤﺘﻪ‪.‬‬
‫‪2‬‬
‫و‪ .‬ﺘﺤﻘق ﻤن ﻗﻴﻤﺔ ‪ h‬ﻫﻨدﺴﻴﺎ ﺒﺎﻻﻋﺘﻤﺎد ﻋﻠﻰ اﻝﺸﻜل‪.02‬‬

‫اﻝﺘـﻤرﻴــن اﻝﺜﺎﻝث‪ 05):‬ﻨﻘﺎط(‬


‫‪E‬‬ ‫‪K‬‬ ‫‪ ،‬ﻨﺎﻗﻠﻴن أوﻤﻴﻴن‬ ‫ﻨﺤﻘق دارة ﻜﻬرﺒﺎﺌﻴﺔ ﻤﻜوﻨﺔ ﻤن اﻝﻌﻨﺎﺼر اﻝﺘﺎﻝﻴﺔ‪ :‬ﻤوﻝد ﻤﺜﺎﻝﻲ ﻗوﺘﻪ اﻝﻜﻬرﺒﺎﺌﻴﺔ‬
‫‪ .‬ﻗﻴﻤﺘﻪ ﻤﺠﻬوﻝﺔ وﻤﻜﺜﻔﺔ ﻓﺎرﻏﺔ ﺴﻌﺘﻬﺎ ‪ /‬ﻜﻤﺎ ﻓﻲ اﻝﺸﻜل‪.03‬‬ ‫‪ . = 20Ω‬و‬
‫‪E‬‬ ‫ﻓﻲ اﻝﻠﺤظﺔ ‪ 0 = 0‬ﻨﻐﻠق اﻝﻘﺎطﻌﺔ وﻨﺸﺎﻫد ﻋﻠﻰ ﺸﺎﺸﺔ راﺴم اﻻﻫﺘزاز اﻝﻤﻬﺒطﻲ ذو ذاﻜرة‬
‫اﻝﺒﻴﺎﻨﻴن‪ 2‬و‪ 3‬اﻝﻤوﻀﺤﻴن ﻓﻲ اﻝﺸﻜل‪.04‬‬
‫‪C‬‬ ‫‪R1‬‬
‫‪R2‬‬
‫‪M‬‬ ‫‪B‬‬
‫‪A‬‬

‫‪U2‬‬ ‫‪U1‬‬ ‫‪ .1‬ﺤدد اﻝﺒﻴﺎن اﻝذي ﻴواﻓق ﻜل ﻤدﺨل ﻤﻊ اﻝﺘﻌﻠﻴل‪.‬‬


‫‪Y2‬‬
‫‪U3‬‬ ‫‪Y1‬‬ ‫‪ .2‬اﻜﺘب اﻝﻤﻌﺎدﻝﺔ اﻝﺘﻔﺎﻀﻠﻴﺔ ﺒدﻻﻝﺔ )‪ 4 (0‬اﻝﺘوﺘر اﻝﻜﻬرﺒﺎﺌﻲ ﺒﻴن طرﻓﻲ اﻝﻤﻜﺜﻔﺔ‪.‬‬
‫ل‪03‬‬ ‫ا‬ ‫‪ .3‬ﺘﻌطﻰ ﻋﺒﺎرة اﻝﺘوﺘر اﻝﻠﺤظﻲ ﺒﻴن طرﻓﻲ اﻝﻨﺎﻗل اﻷوﻤﻲ ‪ .‬ﺒﺎﻝﻌﻼﻗﺔ‪:‬‬
‫;‪9:.‬‬
‫) ‪u (V‬‬ ‫‪a‬‬ ‫‪ 47 (0) = #.‬ﺤﻴث ‪ #‬و & ﺜﺎﺒﺘﻴن ﻏﻴر ﻤﻌدوﻤﻴن‪،‬‬
‫ﺒﺎﻻﻋﺘﻤﺎد ﻋﻠﻰ ﻗﺎﻨون ﺠﻤﻊ اﻝﺘوﺘرات ﺒﻴن أن‪:‬‬
‫) ?‪(7= >7‬‬ ‫;‪9:.‬‬
‫= )‪.4 (0‬‬ ‫‪−‬‬ ‫‪.‬‬
‫=‪7‬‬
‫‪b‬‬
‫‪ .4‬ﻤن اﻝﺸروط اﻻﺒﺘداﺌﻴﺔ واﻝﻤﻌﺎدﻝﺔ اﻝﺘﻔﺎﻀﻠﻴﺔ اوﺠد ﻗﻴﻤﺔ ‪ #‬و &‪.‬‬
‫‪ .5‬اﻜﺘب اﻝﻌﺒﺎرة اﻝﻠﺤظﻴﺔ ﻝـ )‪.4 (0‬‬

‫ل‪2-‬‬ ‫ا‬
‫‪ .6‬ﺒﺎﺴﺘﻐﻼل اﻝﺒﻴﺎﻨﻴن ﻓﻲ اﻝﺸﻜل‪.04‬‬
‫‪04‬‬
‫‪1‬‬ ‫‪ .1.6‬ﺤدد ﻗﻴﻤﺔ ﻜل ﻤن ‪ . ،‬و @ ﻋﻠﻤﺎ أن ﻤدة اﻝﻨظﺎم اﻻﻨﺘﻘﺎﻝﻲ ﻫﻲ‬
‫‪0,1‬‬
‫)‪t (ms‬‬ ‫‪ 0,75 A‬ﺜم اﺴﺘﻨﺘﺞ ﺴﻌﺔ اﻝﻤﻜﺜﻔﺔ ‪. /‬‬
‫) ‪u (V‬‬ ‫‪ .2.6‬اوﺠد ﻋﺒﺎرة ﺸدة اﻝﺘﻴﺎر اﻷﻋظﻤﻲ ﺜم اﺤﺴب ﻗﻴﻤﺘﻪ‪.‬‬
‫‪ .3.6‬ﻤﺜل ﺒﺸﻜل ﻜﻴﻔﻲ ﻤﺤددا اﻝﻘﻴم اﻝﻤﻤﻴزة ﻝﻠﺘوﺘر )‪.47? (0‬‬
‫‪B‬‬
‫= ‪ 4‬اﺤﺴب ‪ 4C‬و ‪. 4C‬‬ ‫‪ .4.6‬ﻤن أﺠل‬
‫‪ .7‬اﺤﺴب اﻝطﺎﻗﺔ اﻝﺘﻲ ﺘﺨزﻨﻬﺎ اﻝﻤﻜﺜﻔﺔ ﻋﻨد اﻝﻠﺤظﺔ @‪. 0 = 5‬‬
‫‪2‬‬

‫‪3‬‬
‫‪ .8‬ﻨﻌﻴد اﻝﺘﺠرﺒﺔ اﻝﺴﺎﺒﻘﺔ وذﻝك ﺒﺘﻐﻴﻴر ﻗﻴﻤﺔ‬
‫‪1‬‬ ‫‪1‬‬ ‫‪D‬‬ ‫‪3‬‬ ‫‪2‬‬ ‫اﻝﺘﺠرﺒﺔ‬ ‫‪ .‬وﻨﻜرر اﻝﻌﻤﻠﻴﺔ ﻤن أﺠل ﻜل ﺘﺠرﺒﺔ‬
‫)‪t (ms‬‬ ‫‪20‬‬ ‫‪10‬‬ ‫‪0‬‬ ‫)‪. (Ω‬‬ ‫ﻓﻨﺤﺼل ﻋﻠﻰ ﺒﻴﺎﻨﺎت اﻝﺸﻜل‪:05‬‬
‫‪0,1‬‬
‫ارﻓق اﻝﺒﻴﺎن اﻝذي ﻴواﻓق ﻜل ﺘﺠرﺒﺔ ﻤﻊ اﻝﺘﺒرﻴر‪.‬‬
‫اﻝﺠزء اﻝﺜﺎﻨﻲ‪ 06 ):‬ﻨﻘــﺎط(‬
‫اﻝﺘﻤرﻴـــن اﻝﺘﺠرﻴﺒــــﻲ)‪06‬ﻨﻘﺎط(‬
‫إﻋداد اﻷﺴﺎﺘذة ‪-:‬ﺴﺎﻋﻲ ﻤﺴﻌود –ﺒوﻗطﺎﻴﺔ ﻤﺤﻤد –ﻏوﺒﺎل رﻤزي‬

‫ﺼﻔﺤﺔ ‪ 3‬ﻤن ‪9‬‬


‫‪ .‬ﺒﻜﺎﻝورﻴﺎ ﺘﺠرﻴﺒﻲ ‪2023‬‬ ‫اﻝﺸﻌﺒﺔ‪ :‬رﻴﺎﻀﻴﺎت‪ ،‬ﺘﻘﻨﻲ رﻴﺎﻀﻲ‬ ‫اﺨﺘﺒﺎر ﻓﻲ ﻤﺎدة اﻝﻌﻠوم اﻝﻔﻴزﻴﺎﺌﻴﺔ‪.‬‬
‫وﺸرﻴط ﻤن‬ ‫أﺜﻨﺎء ﺤﺼﺔ اﻷﻋﻤﺎل اﻝﺘطﺒﻴﻘﻴﺔ أراد ﺘﻼﻤﻴذ اﻝﺴﻨﺔ ‪ 3‬رﻴﺎﻀﻲ اﻨﺠﺎز ﻋﻤود ﻜﻬرﺒﺎﺌﻲ ﺒﺎﺴﺘﻌﻤﺎل ﺸوارد اﻝﻤﻐﻨﻴزﻴوم > ‪E‬‬
‫>‬
‫‪ /4‬وﺼﻔﻴﺤﺔ ﻤن اﻝﻤﻌدن‪ ./4‬ﻓﺎﻨﻘﺴم اﻝﻔوج إﻝﻰ ﻤﺠﻤوﻋﺘﻴن ‪.‬‬ ‫ﻤﻊ ﺸوارد اﻝﻨﺤﺎس‬ ‫ﻤﻌدن اﻝﻤﻐﻨﻴزﻴم‪E‬‬
‫اﻝﻤﺠﻤوﻋـــﺔ اﻷوﻝـــــﻰ ‪:‬‬
‫وﻤﺤﻠول ﺤﻤض ﻜﻠور اﻝﻤﺎء‬ ‫ﻷﺠل ﺘﺤﻀﻴر ﺸوارد اﻝﻤﻐﻨﻴزﻴوم ﻗﺎﻤوا ﺒﺘﺤﻘﻴق اﻝﺘﻔﺎﻋل اﻝﻜﻴﻤﻴﺎﺌﻲ اﻝﺤﺎﺼل ﺒﻴن اﻝﻤﻐﻨﻴزﻴوم ‪E‬‬
‫))‪ (F '> (2G) + /I 9 (2G‬وﻓق اﻝﻤﻌﺎدﻝﺔ ‪:‬‬
‫)‪E(A) + 2F '> (2G) = E > (2G) + F (E) + 2F '(I‬‬
‫‪ .1‬ﻋﻨد اﻝﻠﺤظﺔ‪ 0 = 0A‬ﺘوﻀﻊ ﻗطﻌﺔ ﻜﺘﻠﻬﺎ ‪ = 1E‬ﻤن ﻤﻌدن اﻝﻤﻐﻨﻴزﻴوم ﻓﻲ ﻜﺄس ﺒﻴﺸر ﺒﻪ ﻤﺤﻠول ﻤن ﺤﻤض ﻜﻠور‬
‫اﻝﻤﺎء ﺤﺠﻤﻪ ‪ ! = 30 J‬وﺘرﻜﻴزﻩ اﻝﻤوﻝﻲ ‪.D = 0,1 KI/J‬‬
‫‪+2‬‬ ‫‪−2‬‬
‫)‪ Mg  (×10 mol / L‬‬
‫أ‪ -‬ﺤدد اﻝﺜﻨﺎﺌﻴﺘﻴن)‪ ('M/. N‬اﻝداﺨﻠﺘﻴن ﻓﻲ اﻝﺘﻔﺎﻋل ﻤﻊ ﻜﺘﺎﺒﺔ‬
‫اﻝﻤﻌﺎدﻝﺘﻴن اﻝﻨﺼﻔﻴﺘﻴن‪.‬‬
‫ب‪ -‬ﻫل اﻝﻤزﻴﺞ اﻻﺒﺘداﺌﻲ ﻓﻲ اﻝﺸروط اﻝﺴﺘوﻜﻴوﻤﺘرﻴﺔ‪.‬‬

‫ل‪06‬‬ ‫ا‬ ‫ج‪ -‬أﻨﺠز ﺠدوﻻ ﻝﺘﻘدم اﻝﺘﻔﺎﻋل‪ ،‬وﺤدد اﻝﻤﺘﻔﺎﻋل اﻝﻤﺤد‪.‬‬
‫>‬
‫‪. E‬‬ ‫د‪ -‬اﺤﺴب اﻝﺘرﻜﻴز اﻷﻋظﻤﻲ ﻝﻠﺸوارد‬
‫‪1‬‬ ‫‪ .2‬ﺒﻤﺘﺎﺒﻌﺔ ﺘطور ﺘرﻜﻴز ‪ O E > P‬ﺨﻼل اﻝزﻤن ﺘﺤﺼﻠت ﻫذﻩ اﻝﻤﺠﻤوﻋﺔ‬
‫)‪t (min‬‬ ‫ﻋﻠﻰ اﻝﺒﻴﺎن اﻝﻤوﻀﺢ ﻓﻲ اﻝﺸﻜل‪.06‬‬
‫‪0‬‬ ‫‪2‬‬
‫أ‪ .‬ﻋرف زﻤن ﻨﺼف اﻝﺘﻔﺎﻋل ‪ ، t1/2‬واﺴﺘﻨﺘﺞ ﻗﻴﻤﺘﻪ؟‬
‫ب‪ .‬اﺴﺘﻨﺘﺞ ﻋﺒﺎرة اﻝﺴرﻋﺔ اﻝﺤﺠﻤﻴﺔ ﻝﻠﺘﻔﺎﻋل ﺒدﻻﻝﺔ ﺘرﻜﻴز ﺸوارد اﻝﻤﻐﻨﻴزﻴوم ‪ O E > P‬ﺜم اﺤﺴب ﻗﻴﻤﺘﻬﺎ ﻋﻨد اﻝﻠﺤظﺔ‬
‫‪ 0 = 0 QR‬و ‪.0 = 8 QR‬ﻜﻴف ﺘﺘطور ﻫذﻩ اﻝﺴرﻋﺔ؟ ﻋﻠل‪.‬‬
‫ج‪ .‬ﻴﻤﻜن ﻤﺘﺎﺒﻌﺔ ﻫذا اﻝﺘﺤول ﻋن طرﻴق ﻗﻴﺎس اﻝﻨﺎﻗﻠﻴﺔ ﻝﻤﺎذا؟ ﺒﻴن دون ﺤﺴﺎب أن اﻝﻨﺎﻗﻠﻴﺔ ﺘﺘﻨﺎﻗص ﺒﻤرور اﻝزﻤن؟‬
‫‪S‬‬ ‫‪?T‬‬ ‫‪= 10,6‬‬ ‫‪A.‬‬ ‫‪.‬‬ ‫‪KI 9 ; SV W = 7,6‬‬ ‫‪A.‬‬ ‫‪.‬‬ ‫‪KI 9 ; SX‬‬ ‫‪T‬‬ ‫‪= 35,0‬‬ ‫‪A.‬‬ ‫‪.‬‬ ‫اﻝﻤﻌطﻴـــﺎت‪KI 9 :‬‬
‫‪YZ‬‬

‫اﻝﻤﺠﻤوﻋــﺔ اﻝﺜﺎﻨﻴـــﺔ‪:‬‬
‫ﺒﻐرض ﺘﺤﻘﻴق اﻝﻌﻤود أﻀﺎﻓت ﻫذﻩ اﻝﻤﺠﻤوﻋﺔ إﻝﻰ اﻝﺨﻠﻴط اﻝﺴﺎﺒق ﺸرﻴط ﻤن اﻝﻤﻐﻨﻴزﻴوم ﺜم ﺤﻀرت ﻜﺄس ﺒﻴﺸر آﺨر ﻴﺤﺘوي‬
‫ﻋﻠﻰ ﺸوارد اﻝﻨﺤﺎس))‪(/4 > (2G) + [' 9 (aq‬ﺤﺠﻤﻪ ‪ ! = 30 J‬وﺘرﻜﻴزﻩ ‪ c1 = 0,05 mol/L‬ﺒﻪ ﺼﻔﻴﺤﺔ ﻤن اﻝﻨﺤﺎس‬
‫‪ ، /4‬ﻴرﺒط اﻝﻤﺴرﻴﻴن ﺒﺠﺴر ﻤﻠﺤﻲ ﻝﻤﺤﻠول ﻨﺘرات اﻷﻤوﻨﻴوم))‪(^F> (2G) + ^'9 (2G‬ﺜم ﻴوﺼل اﻝﻌﻤود ﺒدارة ﺘﺸﻤل‬
‫ﻗﺎطﻌﺔ _ وﻨﺎﻗل أوﻤﻲ ﻤﻘﺎوﻤﺘﻪ ‪..‬اﻝﺘﺤول اﻝﻜﻴﻤﻴﺎﺌﻲ اﻝﺤﺎﺼل ﻴﻨﻤذج ﺒﺎﻝﺘﻔﺎﻋل ذي اﻝﻤﻌﺎدﻝﺔ اﻝﺘﺎﻝﻴﺔ ‪:‬‬
‫) ‪Mg (s ) + Cu 2 + (aq ) = Mg 2+ (aq ) + Cu (s‬‬
‫اﺴﺘﺨدم أﺤد اﻝﺘﻼﻤﻴذ ﺠﻬﺎز اﻝﻔوﻝط ﻤﺘر ﻤن أﺠل ﺘﺤدﻴد ﻗطﺒﻲ اﻝﻌﻤود ﻓﺘﺒﻴن أن ‪.`Va ˃`bc‬‬
‫أ‪ -‬ﺤدد ﻗطﺒﻲ اﻝﻌﻤود واﻜﺘب اﻝﻤﻌﺎدﻝﺘﻴن اﻝﻨﺼﻔﻴﺘﻴن ‪.‬‬
‫ب‪ -‬أرﺴم ﺸﻜﻼ ﺘﺨطﻴطﻴﺎ ﻝﻠﻌﻤود اﻝﻤﺤﻘق ﻤﻊ ﻜﺘﺎﺒﺔ اﻝﺒﻴﺎﻨﺎت اﻝﻼزﻤﺔ و رﻤزﻩ اﻻﺼطﻼﺤﻲ)رﻤز اﻝﻌﻤود( ‪.‬‬
‫ﺠـ ‪ -‬ﻴﺸﺘﻐل اﻝﻌﻤود ﻝﻤدة زﻤﻨﻴﺔ ﻗدرﻫﺎ ‪ 1ℎ30 QR‬ﺒﺸدة ﺘﻴﺎر ﺜﺎﺒﺘﺔ ‪.d = 40 #‬‬
‫إﻋداد اﻷﺴﺎﺘذة ‪-:‬ﺴﺎﻋﻲ ﻤﺴﻌود –ﺒوﻗطﺎﻴﺔ ﻤﺤﻤد –ﻏوﺒﺎل رﻤزي‬ ‫‪ -1‬اﺤﺴب ﻤﻘدار اﻝﺘﻘدم ‪. %‬‬
‫∆ ﻝﻤﺴرى اﻝﻤﻐﻨﻴزﻴوم‪. E‬‬ ‫‪ -2‬اﺤﺴب ﻤﻘدا ر اﻝﻨﻘص اﻝﻜﺘﻠﻲ‬
‫اﻨﺘﻬﻰ اﻝﻤوﻀوع اﻻول‬ ‫‪1g = 96500D ،‬‬ ‫ﻴﻌطﻰ‪( E) = 24E/ KI :‬‬

‫ﺼﻔﺤﺔ ‪ 4‬ﻤن ‪9‬‬


‫‪ .‬ﺒﻜﺎﻝورﻴﺎ ﺘﺠرﻴﺒﻲ ‪2023‬‬ ‫اﻝﺸﻌﺒﺔ‪ :‬رﻴﺎﻀﻴﺎت‪ ،‬ﺘﻘﻨﻲ رﻴﺎﻀﻲ‬ ‫اﺨﺘﺒﺎر ﻓﻲ ﻤﺎدة اﻝﻌﻠوم اﻝﻔﻴزﻴﺎﺌﻴﺔ‪.‬‬
‫اﻝﻤوﻀـــــــــوع اﻝﺜﺎﻨـــــــــﻲ‪:‬‬
‫ﻴﺤﺘوي اﻝﻤوﻀوع اﻝﺜﺎﻨﻲ ﻋﻠﻰ )‪ (05‬ﺼﻔﺤﺎت) ﻤن اﻝﺼﻔﺤﺔ ‪ 05‬ﻤن ‪ 09‬إﻝﻰ ‪ 09‬ﻤن ‪(09‬‬
‫اﻝﺠـــــزء اﻷول‪ 14 ) :‬ﻨﻘطﺔ(‬
‫اﻝﺘﻤرﻴن اﻷول‪ 05 ) :‬ﻨﻘﺎط(‬
‫‪-I‬دراﺴﺔ ﺤرﻜﺔ ﺠﺴم ﻋﻠﻰ ﻤﺴﺘوي ﻤﺎﺌل‬
‫ﻴﻨﺘﻘل ﺠﺴم ﻜﺘﻠﺘﻪ ‪ = 0,5 _E‬ﻤن اﻝﺴﻜون ﻋﻠﻰ ﻤﺴﺘوي ‪ #i‬ﻤﺎﺌل وأﻤﻠس ﻴﻤﻴل ﻋن اﻷﻓق ﺒزاوﻴﺔ ‪α = 30°‬‬
‫ﺤﻴث ﻴﺨﻀﻊ اﻝﺠﺴم أﺜﻨﺎء ﺤرﻜﺘﻪ ﻋﻠﻰ اﻝﺠزء ‪ #i‬ﻝﻠﻘوة ‪ g+‬ﺤﺎﻤﻠﻬﺎ ﻴوازي ﺨط اﻝﻤﻴل اﻷﻋظم ﻝﻠﻤﺴﺘوي اﻝﻤﺎﺌل ﻜﻤﺎ ﻓﻲ‬
‫اﻝﺸﻜل‪ ،-1-‬ﺘم ﻗﻴﺎس ﺴرﻋﺔ ﻤرﻜز ﻋطﺎﻝﺔ اﻝﺠﺴم ﻋﻠﻰ اﻝﺠزء ‪ #i‬ﺨﻼل أزﻤﻨﺔ ﻤﺨﺘﻠﻔﺔ وﻤﻨﻪ ﺘم رﺴم ﺒﻴﺎن اﻝﺴرﻋﺔ ﺒدﻻﻝﺔ‬
‫اﻝزﻤن )‪ ( = l(0‬ﻜﻤﺎ ﻓﻲ اﻝﺸﻜل‪-2-‬‬
‫)‪v(m / s‬‬
‫ل‪2-‬‬ ‫ا‬
‫‪،E = 10 . A 9‬‬ ‫ﻴﻌطﻰ‪:‬‬

‫‪n‬‬
‫ل‪-1-‬‬ ‫ا‬
‫‪i‬‬
‫‪g+‬‬

‫&‬
‫‪#‬‬
‫‪2‬‬ ‫‪%‬‬
‫'‬ ‫‪/‬‬ ‫‪o‬‬
‫‪0‬‬ ‫)‪t (s‬‬
‫‪1‬‬ ‫‪ .1‬ذﻜر ﺒﻨص اﻝﻘﺎﻨون اﻝﺜﺎﻨﻲ ﻝﻨﻴوﺘن‪.‬‬
‫‪ .2‬ﺒﺘطﺒﻴق اﻝﻘﺎﻨون اﻝﺜﺎﻨﻲ ﻝﻨﻴوﺘن ﻋﻠﻰ اﻝﺠﻤﻠﺔ )ﺠﺴم( اوﺠد ﻋﺒﺎرة ﺘﺴﺎرع ﻤرﻜز ﻋطﺎﻝﺘﻪ‪ ،‬ﺜم اﺴﺘﻨﺘﺞ طﺒﻴﻌﺔ اﻝﺤرﻜﺔ‪.‬‬
‫‪ .3‬ﺒﺎﺴﺘﻐﻼل ﺒﻴﺎن اﻝﺸﻜل‪ -2-‬اﺤﺴب ﻗﻴﻤﺔ اﻝﺘﺴﺎرع ‪ 2‬ﺜم اﺴﺘﻨﺘﺞ ﻗﻴﻤﺔ ‪.g‬‬
‫‪ .4‬ﻫل ﺸﻌﺎع اﻝﻘوة ‪ g+‬ﺜﺎﺒت ﺨﻼل اﻝﺤرﻜﺔ؟ ﻋﻠل‪.‬‬
‫‪ .5‬اﺤﺴب طول اﻝﻤﺴﺎر ‪.#i‬‬
‫‪ .6‬ﺒﺘطﺒﻴق ﻤﺒدأ اﻨﺤﻔﺎظ اﻝطﺎﻗﺔ ﻋﻠﻰ اﻝﺠﻤﻠﺔ )ﺠﺴم( ﺒﻴن اﻝﻤوﻀﻌﻴن ‪ #‬و ‪.i‬‬
‫‪ E ،‬و &‪.‬‬ ‫أ‪ -‬أوﺠد ﻋﺒﺎرة اﻝﺴرﻋﺔ ‪ (m‬ﺒدﻻﻝﺔ ‪، #i ، g‬‬
‫ب‪ -‬اﺤﺴب ﻗﻴﻤﺔ ‪ (m‬وﺘﺄﻜد ﻤﻨﻬﺎ ﺒﻴﺎﻨﻴﺎ‪.‬‬
‫‪-II‬دراﺴﺔ ﺠﺴم ﺨﺎﻀﻊ ﻝﻘوة ﺜﻘﻠﻪ ﻓﻲ ﺤﻘل اﻝﺠﺎذﺒﻴﺔ اﻷرﻀﻴﺔ‪.‬‬
‫ﻋﻨد وﺼول اﻝﺠﺴم إﻝﻰ اﻝﻨﻘطﺔ ‪ i‬ﻴواﺼل ﺤرﻜﺘﻪ ﻓﻲ اﻝﻬواء)ﺒﺈﻫﻤﺎل ﺘﺄﺜﻴرات اﻝﻬواء( ‪.‬ﻨﻌﺘﺒر ﻤﺒدأ اﻷزﻤﻨﺔ ﻝﺤظﺔ ﻤﻐﺎدرة اﻝﺠﺴم‬
‫اﻝﻨﻘطﺔ ‪.i‬‬
‫‪ .1‬ادرس ﺤرﻜﺔ اﻝﺠﺴم وﻓق اﻝﻤﺤورﻴن )‪ (K%‬و)‪.(Kn‬‬
‫‪ .2‬اﻜﺘب ﻋﺒﺎرة ﻜل ﻤن )‪ % = l(0‬و )‪.n = E(0‬‬
‫‪ .3‬اوﺠد ﻤﻌﺎدﻝﺔ اﻝﻤﺴﺎر‪.‬‬
‫إﻋداد اﻷﺴﺎﺘذة ‪-:‬ﺴﺎﻋﻲ ﻤﺴﻌود –ﺒوﻗطﺎﻴﺔ ﻤﺤﻤد –ﻏوﺒﺎل رﻤزي‬
‫‪ .4‬اﺤﺴب ﻗﻴﻤﺔ ‪.%‬‬
‫‪ .5‬اﺤﺴب ﻗﻴﻤﺔ اﻝﺴرﻋﺔ ﻋﻨد اﻝﻤوﻀﻊ ‪ /‬ﺒطرﻴﻘﺘﻴن ﻤﺨﺘﻔﺘﻴن‪.‬‬
‫‪ ،/o = 5‬وﻤن أﺠل ذﻝك ﻨﻐﻴر ﻤن ﻗﻴﻤﺔ ‪. g‬‬ ‫‪ .6‬ﻨرﻴد اﻝزﻴﺎدة ﻓﻲ ﻗﻴﻤﺔ اﻝﻤدى ﺤﺘﻰ ﻴﺒﻠﻎ اﻝﺠﺴم اﻝﻨﻘطﺔ ‪ D‬ﺤﻴث‬
‫ﺼﻔﺤﺔ ‪ 5‬ﻤن ‪9‬‬
‫‪ .‬ﺒﻜﺎﻝورﻴﺎ ﺘﺠرﻴﺒﻲ ‪2023‬‬ ‫اﻝﺸﻌﺒﺔ‪ :‬رﻴﺎﻀﻴﺎت‪ ،‬ﺘﻘﻨﻲ رﻴﺎﻀﻲ‬ ‫اﺨﺘﺒﺎر ﻓﻲ ﻤﺎدة اﻝﻌﻠوم اﻝﻔﻴزﻴﺎﺌﻴﺔ‪.‬‬
‫ﻤﺎﻫﻲ ﻗﻴﻤﺔ ‪ g‬اﻝواﺠب ﺘطﺒﻴﻘﻬﺎ ﻋﻠﻰ اﻝﺠﺴم ﺤﺘﻰ ﻴﺒﻠﻎ اﻝﻨﻘطﺔ ‪.o‬‬
‫اﻝﺘﻤرﻴن اﻝﺜﺎﻨﻲ‪ 06) :‬ﻨﻘﺎط(‬
‫‪ .I‬دراﺴﺔ ﺤرﻜﻴﺔ اﻝﺘﺤول اﻝﻜﻴﻤﻴﺎﺌﻲ اﻝﺤﺎدث ﺒﻴن ﺜﻨﺎﺌﻲ اﻝﺒروم وﺤﻤض اﻝﻤﻴﺜﺎﻨوﻴك‬
‫ﻴﻌطﻰ‪.!b = 25J/ KI :‬‬
‫ﺤﻤض اﻝﻨﻤل ﺴﺎﺌل ﻴﻔرزﻩ اﻝﻨﻤل وﻴﺴﺘﺨدم ﻓﻲ ﺼﻨﺎﻋﺔ اﻝورق واﻝﻨﺴﻴﺞ‬
‫ﻤﺤﻠول ﺜﻨﺎﺌﻲ اﻝﺒروم ﻝوﻨﻪ أﺤﻤر ﺒﻨﻲ وﻝﻜن ﻤﺤﻠول ﺤﻤض اﻝﺒروم ﻋدﻴم اﻝﻠون‬
‫ﻓﻲ اﻝﻠﺤظﺔ ‪ t = 0s‬ﻨﻤزج ‪ 50ml‬ﻤن ﻤﺤﻠول ﺜﻨﺎﺌﻲ اﻝﺒروم ‪ Br2‬ﺘرﻜﻴزﻩ‬
‫‪ 50ml‬ﻤن ﻤﺤﻠول ﺤﻤض اﻝﻤﻴﺜﺎﻨوﻴك ﺘرﻜﻴزﻩ ‪. C2 = 0, 03mol / L‬‬ ‫‪ C1 = 0,024mol / L‬ﻤﻊ‬
‫ﻴﺤدث اﻝﺘﻔﺎﻋل ﺒﻴن ﺤﻤض اﻝﻤﻴﺜﺎﻨوﻴك وﺜﻨﺎﺌﻲ اﻝﺒروم وﻓق اﻝﻤﻌﺎدﻝﺔ اﻝﺘﺎﻝﻴﺔ‪:‬‬
‫‪9‬‬
‫‪F/''F(pq) + i‬‬ ‫)‪(pq‬‬
‫‪= 2i‬‬ ‫)‪(pq‬‬ ‫'‪+ 2F> (pq) + /‬‬ ‫)‪(c‬‬

‫‪ .1‬ﺤدد ﻤﺨﺘﻠف طرق اﻝﻤﺘﺎﺒﻌﺔ اﻝزﻤﻨﻴﺔ ﻝﻬذا اﻝﺘﺤول اﻝﻜﻴﻤﻴﺎﺌﻲ‪.‬‬


‫‪ .2‬ﻝﻤﺎذا ﻴﺘﻌذر ﻤﺘﺎﺒﻌﺔ ﻫذا اﻝﺘﺤول ﺒواﺴطﺔ اﻝﻤﻌﺎﻴرة اﻝﻠوﻨﻴﺔ ﻝﻠﺤﻤض ﻓﻲ اﻝﻤزﻴﺞ اﻝﺘﻔﺎﻋﻠﻲ ﺒواﺴطﺔ ﻤﺤﻠول أﺴﺎﺴﻲ‪.‬‬
‫‪ .3‬ﻫل اﻝﻤزﻴﺞ اﻻﺒﺘداﺌﻲ ﻴﺤﻘق اﻝﻨﺴب اﻝﺴﺘوﻜﻴوﻤﺘرﻴﺔ‪.‬‬
‫ﻫو ﺤﺠم ﻏﺎز '‪/‬‬ ‫?‪rs‬‬ ‫‪ [ Br2 ]t = 0,012 − 4 ×10−4VCO‬ﺤﻴث‬ ‫‪2‬‬
‫‪ .4‬أﻨﺠز ﺠدوﻻ ﻝﺘﻘدم اﻝﺘﻔﺎﻋل ﺜم اﺜﺒت اﻝﻌﻼﻗﺔ اﻝﺘﺎﻝﻴﺔ‪:‬‬
‫اﻝﻤﻨطﻠق ﻋﻨد اﻝﻠﺤظﺔ ‪ 0‬ﻤﻘدر ﺒـ ‪. J‬‬
‫‪ .5‬ﻗﻤﻨﺎ ﺒﻤﺘﺎﺒﻌﺔ ﻫذا اﻝﺘﺤول ﻓﺘﺤﺼﻠﻨﺎ ﻋﻠﻰ اﻝﻨﺘﺎﺌﺞ اﻝﺘﺎﻝﻴﺔ‪:‬‬

‫)‪t (s‬‬ ‫‪0‬‬ ‫‪50‬‬ ‫‪100‬‬ ‫‪150‬‬ ‫‪200‬‬ ‫‪250‬‬ ‫‪300‬‬ ‫‪350‬‬ ‫‪400‬‬
‫) ‪VCO2 (ml‬‬ ‫‪0‬‬ ‫‪04,56‬‬ ‫‪08,50‬‬ ‫‪11,76‬‬ ‫‪14,50‬‬ ‫‪16,80‬‬ ‫‪18,72‬‬ ‫‪20,40‬‬ ‫‪21,70‬‬

‫)‪[ Br2 ] (mmol / L‬‬


‫‪-1-5‬ﻤﺎ ﻫﻲ اﻝطرﻴﻘﺔ اﻝﺘﻲ ﺘﻤت ﺒﻬﺎ ﻤﺘﺎﺒﻌﺔ ﻫذا اﻝﺘﺤول ؟ أرﺴم اﻝﺘﺠﻬﻴز اﻝﺘﺠرﻴﺒﻲ اﻝذي ﺘراﻩ ﻤﻨﺎﺴﺒﺎ‪.‬‬
‫‪[ HCOOH ] f‬‬
‫‪ HCOO − ‬‬
‫‪f‬‬
‫= ‪[ Br2 ]t‬‬ ‫‪-2-5‬أﻜﻤل اﻝﺠدول اﻝﺴﺎﺒق وارﺴم اﻝﺒﻴﺎن ) ‪f (t‬‬

‫‪6,94‬‬
‫ل‪03‬‬ ‫ا‬
‫‪ .6‬أوﺠد ﻋﺒﺎرة اﻝﺴرﻋﺔ اﻝﺤﺠﻤﻴﺔ ﻝﻠﺘﻔﺎﻋل ﺒدﻻﻝﺔ ‪ [ Br2 ]t‬ﺜم أﺤﺴب ﻗﻴﻤﺘﻬﺎ‬
‫ﻓﻲ اﻝﻠﺤظﺔ ‪. 0 = 200A‬‬
‫‪ .7‬ﻓﻲ اﻝﻠﺤظﺔ ‪ t = 450s‬ﻜﺎن ﺤﺠم ﻏﺎز ‪ CO2‬ﻫو ‪ 22, 60ml‬ﻫل اﺨﺘﻔﻰ‬
‫اﻝﻠون اﻷﺤﻤر اﻝﺒﻨﻲ‪ .‬ﺒرر ذﻝك؟‬
‫دراﺴﺔ ﺘﻔﺎﻋل ﻤﻌﺎﻴرة ﺤﻤض ﻀﻌﻴف ﺒﺄﺴﺎس ﻗوي‪:‬‬ ‫‪.II‬‬
‫ﻨﺤﻀر ﺤﺠﻤﺎ ‪!′p = 500 J‬ﻝﻤﺤﻠول ﺤﻤض اﻝﻤﻴﺜﺎﻨوﻴك‬
‫‪pH‬‬ ‫‪ HCOOH‬ﺘرﻜﻴزﻩ اﻝﻤوﻝﻲ ‪ C = 9, 9 × 10−3 mol / L‬ﻗﻴﺎس اﻝـ‬
‫‪1‬‬
‫اﻝﻤﺤﻠول أﻋطﻰ اﻝﻘﻴﻤﺔ ‪ 2, 9‬ﻋﻨد درﺠﺔ اﻝﺤ اررة ‪. 250 C‬‬
‫‪0‬‬ ‫‪2‬‬
‫)‪Vb (mL‬‬ ‫‪ .1‬أﻜﺘب ﻤﻌﺎدﻝﺔ ﺘﻔﺎﻋل ﺤﻤض اﻝﻤﻴﺜﺎﻨوﻴك ﻤﻊ اﻝﻤﺎء؟‬

‫ﺼﻔﺤﺔ ‪ 6‬ﻤن ‪9‬‬


‫‪ .‬ﺒﻜﺎﻝورﻴﺎ ﺘﺠرﻴﺒﻲ ‪2023‬‬ ‫اﻝﺸﻌﺒﺔ‪ :‬رﻴﺎﻀﻴﺎت‪ ،‬ﺘﻘﻨﻲ رﻴﺎﻀﻲ‬ ‫اﺨﺘﺒﺎر ﻓﻲ ﻤﺎدة اﻝﻌﻠوم اﻝﻔﻴزﻴﺎﺌﻴﺔ‪.‬‬
‫‪ .2‬أﺤﺴب ﻨﺴﺒﺔ ﺘﻘدم اﻝﻨﻬﺎﺌﻲ ﻝﻠﺘﻔﺎﻋل ؟ ﻤﺎذا ﺘﺴﺘﻨﺘﺞ؟‬
‫إﻋداد اﻷﺴﺎﺘذة ‪-:‬ﺴﺎﻋﻲ ﻤﺴﻌود –ﺒوﻗطﺎﻴﺔ ﻤﺤﻤد –ﻏوﺒﺎل رﻤزي‬
‫‪ .3‬أﺤﺴب ﺜﺎﺒت اﻝﺘوازن ﻝﻬذا اﻝﺘﺤول؟‬
‫‪ .4‬ﻨﺄﺨذ ﺤﺠﻤﺎ ‪ Va‬ﻤن ﻤﺤﻠول ﺤﻤض اﻝﻤﻴﺜﺎﻨوﻴك وﻨﻌﺎﻴرﻩ ﺒواﺴطﺔ ﻤﺤﻠول ﻫﻴدروﻜﺴﻴد اﻝﺼودﻴوم ) ‪ ( Na + + HO −‬ﺘرﻜﻴزﻩ اﻝﻤوﻝﻲ‬
‫‪ ./ = 109‬ﻤﻜﻨت اﻝﻨﺘﺎﺌﺞ اﻝﻤﺤﺼل ﻋﻠﻴﻬﺎ ﻤن اﻝﺤﺼول ﻋﻠﻰ اﻝﻤﻨﺤﻨﻲ اﻝﺒﻴﺎﻨﻲ اﻝﺸﻜل‪.03‬‬ ‫‪KI/J‬‬
‫أ‪ -‬أﻜﺘب ﻤﻌﺎدﻝﺔ ﺘﻔﺎﻋل اﻝﻤﻌﺎﻴرة اﻝﺤﺎدث؟‬
‫ب‪ -‬ﺤدد ﺒﻴﺎﻨﻴﺎ ‪ Vbe‬اﻝﺤﺠم اﻝﻼزم ﻝﺒﻠوغ اﻝﺘﻜﺎﻓؤ؟‬
‫ج‪ -‬ﺘﺤﻘق ﻤن ﻗﻴﻤﺔ اﻝﺘرﻜﻴز ‪ C‬ﻝﻤﺤﻠول ﺤﻤض اﻝﻤﻴﺜﺎﻨوﻴك ﺜم اﺴﺘﻨﺘﺞ ﻗﻴﻤﺔ اﻝﺤﺠم ‪ !p‬؟‬
‫دراﺴﺔ ﺘﻔﺎﻋل اﻷﺴﺘرة‪:‬‬ ‫‪.III‬‬
‫ﻨﻘوم ﺒﺎﻝﺘﺴﺤﻴن ﻝﻤزﻴﺞ ﻤﻜون ﻤن ‪ 0, 4mol‬ﻝﺤﻤض اﻝﻤﻴﺜﺎﻨوﻴك و ‪ 0, 4mol‬ﻝﻠﺒروﺒﺎن‪-2-‬ول ﻤﻊ إﻀﺎﻓﺔ ﻗطرات ﻤن ﺤﻤض‬
‫اﻝﻜﺒرﻴت اﻝﻤرﻜز‪ .‬ﺒﻌد ﻤرور ﺴﺎﻋﺔ )‪ (60 min‬ﻤن ﺒداﻴﺔ اﻝﺘﺠرﺒﺔ‪ ،‬ﻨوﻗف اﻝﺘﻔﺎﻋل وﻨﺤدد ﻜﻤﻴﺔ اﻝﺤﻤض اﻝﻤﺘﺒﻘﻴﺔ وذﻝك ﺒﻤﻌﺎﻴرﺘﻬﺎ‬
‫ﺒﺄﺴﺎس ) ‪ ( Na + + HO −‬ﻓﻨﺠد ‪.Rp = 0,16 KI‬‬
‫‪ .1‬أﻜﺘب ﻤﻌﺎدﻝﺔ اﻝﺘﻔﺎﻋل ﺒﻴن ﺤﻤض اﻝﻤﻴﺜﺎﻨوﻴك واﻝﺒروﺒﺎن‪-2-‬ول؟‬
‫‪ .2‬أﻋط اﺴم اﻝﻤرﻜب اﻝﻌﻀوي اﻝﻨﺎﺘﺞ؟‬
‫‪ .3‬أﻨﺸﺊ ﺠدوﻻ ﻝﺘﻘدم اﻝﺘﻔﺎﻋل اﻝﻤدروس؟‬
‫‪ .4‬ﺤدد ﻗﻴﻤﺔ اﻝﺘﻘدم ﻋﻨد اﻝﺘوازن ‪ xéq‬؟ اﺴﺘﻨﺘﺞ ﻗﻴﻤﺔ ﺜﺎﺒت اﻝﺘوازن ‪ K‬؟‬
‫‪ .5‬أﺤﺴب ﻤردود اﻝﺘﻔﺎﻋل؟‬
‫إﻋداد اﻷﺴﺎﺘذة ‪-:‬ﺴﺎﻋﻲ ﻤﺴﻌود –ﺒوﻗطﺎﻴﺔ ﻤﺤﻤد –ﻏوﺒﺎل رﻤزي‬
‫اﻝﺘﻤرﻴن اﻝﺜﺎﻝث‪ 03,5):‬ﻨﻘﺎط(‬
‫ﻤﺸروع ‪ ITER‬ﻫو أﻜﺒر ﻤﺸروع ﺘﺠرﻴﺒﻲ ﻹﻨﺘﺎج اﻝطﺎﻗﺔ اﻝذي ﻴﻌﺘﻤد ﻋﻠﻰ اﻻﻨدﻤﺎج اﻝﻨووي ‪ .‬ﻓﻜرة اﻝﻤﺸروع ﻤﺄﺨوذة ﻤن اﻻﻨدﻤﺎج‬
‫اﻝﻨووي اﻝذي ﻴﺤدث ﻓﻲ ﻗﻠب اﻝﻨﺠوم وﻓق اﻝﺘﺤول اﻝﻨووي اﻝﻤﻨﻤذج ﺒﺎﻝﻤﻌﺎدﻝﺔ اﻝﺘﺎﻝﻴﺔ‪H + 13H → 24H + 01n :‬‬
‫‪2‬‬
‫‪1‬‬

‫اﻝﻤﻌطﻴﺎت‪. M ( 126C ) = 12 g / mol ، N A = 6.023 ×1023 mol −1 ،D = 3 × 10v /A:‬‬


‫‪c‬‬
‫) (اﻝﻜﺘﻠﺔ اﻝﻤوﻝﻴﺔ ﻝﻠﻌﻨﺼر‬ ‫) ‪(_E‬اﻝﻜﺘﻠﺔ‬ ‫رﻤز ﻨواة اﻝﻌﻨﺼر أو اﻝﺠﺴﻴم‬
‫‪wx‬‬
‫‪2,0‬‬ ‫‪3,3435 × 109 z‬‬ ‫‪H‬‬
‫‪3,0‬‬ ‫‪5,00737 × 109 z‬‬ ‫‪H‬‬
‫‪4,0‬‬ ‫‪6,64466 × 109 z‬‬ ‫‪He‬‬
‫‪//‬‬ ‫‪1,67493 × 109 z‬‬ ‫‪)R‬‬

‫‪//‬‬ ‫‪1,6726 × 109‬‬ ‫‪z‬‬

‫)‪(t‬‬ ‫‪ .1‬ﻋرف اﻻﻨدﻤﺎج اﻝﻨووي‪ ،‬ﻝﻤﺎذا ﻴﺘطﻠب ﺤ اررة ﻋﺎﻝﻴﺔ ﺠدا ﻝﺤدوﺜﻪ‪.‬‬
‫‪2 11 p + 3 01n‬‬ ‫‪.‬‬ ‫~}|‬ ‫‪ .2‬ﺒﻴن أن اﻝطﺎﻗﺔ اﻝﻤﺤررة ﻤن اﻨدﻤﺎج ‪ F‬و ‪ F‬ﻗدرﻫﺎ ‪= 2,8152 × 109 t‬‬
‫‪.‬‬ ‫‪ .3‬اﺤﺴب اﻝطﺎﻗﺔ اﻝﻤﺤررة ﻋﻨدﻤﺎ ﺘﻨﺘﺞ ﻜﺘﻠﺔ ﻤن اﻝﻬﻠﻴوم ﻗدرﻫﺎ ‪= 1_E‬‬
‫∆‬ ‫•‬
‫∆‬ ‫‪.‬‬ ‫‪ .4‬إذا ﻜﺎن ﻝدﻴﻨﺎ ﻤزﻴﺞ ﻤﺘﺴﺎوي اﻷﻨوﻴﺔ ﻤن ‪ F‬و ‪ F‬ﻜﺘﻠﺘﻪ ‪= 1_E‬‬
‫‪H+ H‬‬
‫‪2‬‬ ‫‪3‬‬
‫‪1‬‬ ‫‪1‬‬
‫أ‪ .‬ﺒﻴن أن اﻝطﺎﻗﺔ اﻝﻜﻠﻴﺔ اﻝﻤﺤررة ﻤن اﻝﻜﺘﻠﺔ اﻝﺴﺎﺒﻘﺔ ﺘﻌطﻰ ﺒﺎﻝﻌﻼﻗﺔ‪:‬‬
‫∆‬ ‫‪He + 01n‬‬ ‫†…„‪w• .‚ƒ .B‬‬
‫=‬
‫‪4‬‬
‫‪2‬‬ ‫‪ ،‬ﺜم اﺤﺴب ﻗﻴﻤﺘﻬﺎ‪.‬‬ ‫‪€‬‬ ‫ˆ ‪b‡ ?=X ˆ>b‡ Y=X‬‬
‫ل‪04‬‬ ‫ا‬ ‫•‬
‫ﻋﻠﻤﺎ أن اﺤﺘراق‬ ‫ب‪ .‬اﺤﺴب ﻜﺘﻠﺔ اﻝﻜرﺒون اﻝﻤﻜﺎﻓﺌﺔ ﻝﻨﻔس اﻝطﺎﻗﺔ اﻝﺘﻲ ﺤررﺘﻬﺎ اﻝﻜﺘﻠﺔ‬
‫ﺼﻔﺤﺔ ‪ 7‬ﻤن ‪9‬‬
‫‪ .‬ﺒﻜﺎﻝورﻴﺎ ﺘﺠرﻴﺒﻲ ‪2023‬‬ ‫اﻝﺸﻌﺒﺔ‪ :‬رﻴﺎﻀﻴﺎت‪ ،‬ﺘﻘﻨﻲ رﻴﺎﻀﻲ‬ ‫اﺨﺘﺒﺎر ﻓﻲ ﻤﺎدة اﻝﻌﻠوم اﻝﻔﻴزﻴﺎﺌﻴﺔ‪.‬‬
‫‪ 1mol‬ﻤن اﻝﻜرﺒون ﻴﺤرر طﺎﻗﺔ ﻗدرﻫﺎ ‪. 240KJ‬‬
‫‪ .5‬إذا ﻜﺎن اﻝﻤﻔﺎﻋل اﻝﻨووي ﻴﻨﺘﺞ طﺎﻗﺔ ﻜﻬرﺒﺎﺌﻴﺔ ﺒﻤردود ‪. = 30%‬‬
‫•‬
‫• ﻤﺎﻫﻲ ﻗﻴﻤﺔ اﻝطﺎﻗﺔ اﻝﻜﻬرﺒﺎﺌﻴﺔ اﻝﺘﻲ ﻴﻨﺘﺠﻬﺎ اﻝﻤﻔﺎﻋل اﻝﻨووي ﺨﻼل ‪ 30‬ﻴوم‪ ،‬ﻋﻠﻤﺎ اﻨﻪ اﺴﺘﻬﻠك ﻜﺘﻠﺔ ﻗدرﻫﺎ ‪= 1_E‬‬
‫ﺨﻼل ﻫذﻩ اﻝﻤدة‪.‬‬
‫‪ .6‬ﻴﻤﺜل اﻝﺸﻜل‪ 04‬ﻤﺨطط اﻝﺤﺼﻴﻠﺔ اﻝطﺎﻗوﻴﺔ ﻝﺘﻔﺎﻋل اﻻﻨدﻤﺎج اﻝﺴﺎﺒق؟‬
‫∆‪.‬‬ ‫∆و‬ ‫‪،‬‬ ‫‪،‬‬ ‫‪،‬‬ ‫أ‪ .‬ﻤﺎذا ﻴﻤﺜل ﻜل ﻤن‬
‫∆‪.‬‬ ‫∆و‬ ‫ب‪ .‬أﺤﺴب ﻜل ﻤن‬

‫اﻝﺠزء اﻝﺜﺎﻨﻲ‪ 06) :‬ﻨﻘﺎط(‬


‫‪E‬‬ ‫اﻝﺘﻤرﻴن اﻝﺘﺠرﻴﺒﻲ‪ 05,5):‬ﻨﻘﺎط(‬
‫‪K‬‬
‫دﺨل أﺴﺘﺎذ اﻝﻌﻠوم اﻝﻔﻴزﻴﺎﺌﻴﺔ إﻝﻰ اﻝﻤﺨﺒر ﻓوﺠد اﻷﺠﻬزة اﻝﻤﺨﺒرﻴﺔ اﻝﺘﺎﻝﻴﺔ‪ :‬ﻤوﻝد‬
‫‪E‬‬ ‫ﻝﻠﺘوﺘر اﻝﻜﻬرﺒﺎﺌﻲ ﻗوﺘﻪ اﻝﻤﺤرﻜﺔ اﻝﻜﻬرﺒﺎﺌﻴﺔ ‪ ، E‬ﻨﺎﻗل أوﻤﻲ ﻤﻘﺎوﻤﺘﻪ ‪، R = 60Ω‬‬
‫)‪(L ,r‬‬
‫‪R‬‬ ‫وﻤﻘﺎوﻤﺘﻬﺎ اﻝداﺨﻠﻴﺔ ‪ ، r‬ﻗﺎطﻌﺔ ‪ k‬وﺼﻤﺎم ﺜﻨﺎﺌﻲ ‪ ، D‬ﻓوﻝطﻤﺘر‬ ‫‪L‬‬ ‫وﺸﻴﻌﺔ ذاﺘﻴﺘﻬﺎ‬
‫‪A‬‬ ‫‪C‬‬
‫‪B‬‬ ‫وأﻤﺒﻴر ﻤﺘر رﻗﻤﻴﻴن‪ ،‬ﺠﻬﺎز ‪. EX A O‬‬
‫‪D‬‬ ‫أراد اﻷﺴﺘﺎذ اﻝﺘﺤﻘق ﻤن ﺒﻌض ﻤﻤﻴزات اﻷﺠﻬزة اﻝﺴﺎﺒﻘﺔ‪ ،‬ﺤﻘق ﻤﻊ ﺘﻼﻤذﺘﻪ اﻝﺘرﻜﻴﺒﺔ‬
‫اﻝﺘﺠرﻴﺒﻴﺔ اﻝﻤوﻀﺤﺔ ﺒﺎﻝﺸﻜل اﻝﻤﻘﺎﺒل‪.‬‬
‫‪-I‬ﻏﻠق اﻝﻘﺎطﻌﺔ‪:‬‬
‫ﻋﻨد اﻝﻠﺤظﺔ ‪ 0 = 0‬ﻨﻌﺘﺒرﻫﺎ ﻤﺒدأ ﻝﻸزﻤﻨﺔ ‪ ،‬ﻨﻐﻠق اﻝﻘﺎطﻌﺔ ‪.-‬‬
‫إﻋداد اﻷﺴﺎﺘذة ‪-:‬ﺴﺎﻋﻲ ﻤﺴﻌود –ﺒوﻗطﺎﻴﺔ ﻤﺤﻤد –ﻏوﺒﺎل رﻤزي‬ ‫‪ .1‬ﻤﺎﻫﻲ اﻝظﺎﻫرة اﻝﺘﻲ ﺘﺤدث ﻓﻲ اﻝوﺸﻴﻌﺔ؟‬
‫‪ .2‬أﻋد رﺴم اﻝدارة اﻝﻜﻬرﺒﺎﺌﻴﺔ ﻤﻊ ﺘﺤدﻴد ﻜل ﻤن ﺠﻬﺔ اﻝﺘﻴﺎر اﻝﻜﻬرﺒﺎﺌﻲ اﻝﻤﺎر ﻓﻲ اﻝدارة وأﺴﻬم اﻝﺘوﺘرات ﺒﻴن طرﻓﻲ ﻜل ﻤن اﻝﻨﺎﻗل‬
‫اﻷوﻤﻲ واﻝوﺸﻴﻌﺔ‪.‬‬
‫‪E‬‬
‫= ‪. I0‬‬ ‫‪ .3‬ﺒﺘطﺒﻴق ﻗﺎﻨون ﺠﻤﻊ اﻝﺘوﺘرات ﺒﻴن أن ﺸدة اﻝﺘﻴﺎر اﻝﻜﻬرﺒﺎﺌﻲ ﻓﻲ اﻝﻨظﺎم اﻝداﺌم‬
‫‪R+r‬‬

‫‪-II‬ﻓﺘﺢ اﻝﻘﺎطﻌﺔ‪:‬ﻋﻨد اﻝﻠﺤظﺔ ‪ t = 0s‬ﻨﻌﺘﺒرﻫﺎ ﻤﺒدأ ﻝﻸزﻤﻨﺔ ‪ ،‬ﻨﻔﺘﺢ اﻝﻘﺎطﻌﺔ ‪. k‬‬

‫أ‪ .‬ﺠد اﻝﻤﻌﺎدﻝﺔ اﻝﺘﻔﺎﻀﻠﻴﺔ ﻝﺘطور ﺸدة اﻝﺘﻴﺎر ) ‪ i (t‬اﻝﻤﺎر ﺒﺎﻝدارة‪.‬‬


‫ب‪ .‬ﻴﻌطﻰ ﺤل اﻝﻤﻌﺎدﻝﺔ اﻝﺘﻔﺎﻀﻠﻴﺔ اﻝﺴﺎﺒﻘﺔ ‪، i (t ) = A e −t /α‬ﺤﻴث ‪ α‬و ‪ #‬ﺜﺎﺒﺘﻴن ﻏﻴر ﻤﻌدوﻤﻴن ﻴطﻠب اﻴﺠﺎد ﻋﺒﺎرﺘﻬﺎ‬
‫ﺒدﻻﻝﺔ ﻤﻤﻴزات اﻝدارة ‪.‬‬
‫ج‪ .‬ﻤﺎذا ﻴﻤﺜل ﻜل ﻤن ‪ α‬و ‪ #‬وﻤﺎﻫو ﻤدﻝوﻝﻬﻤﺎ اﻝﻔﻴزﻴﺎﺌﻲ‪.‬‬
‫د‪ .‬ﻤﺎ ﻫو اﻝﺠﻬﺎز اﻝذي ﻨﺴﺘﻌﻤﻠﻪ ﻝﺘﺤﻘﻴق ﻝﻘﻴﺎس اﻝﻤﻘدار ‪ .#‬وﻜﻴف ﻴﺘم ﺘوﺼﻴﻠﻪ ﻓﻲ اﻝدارة ) ﻋﻠﻰ اﻝﺘﻔرع أم ﻋﻠﻰ اﻝﺘﺴﻠﺴل (‪.‬‬
‫‪ .1‬أﻋﺎد اﻷﺴﺘﺎذ اﻝﺘرﻜﻴب اﻝﺴﺎﺒق ﺒﺤﻴث ﻓﺘﺢ اﻝﻘﺎطﻌﺔ ‪ k‬ﻤن ﺠدﻴد ﻓﻲ اﻝﻠﺤظﺔ ‪ 0 = 0‬ﺒﻌدﻤﺎ ﺒﻠﻐت اﻝدارة اﻝﻨظﺎم اﻝداﺌم‪ ،‬طﺎرﺤﺎ‬
‫اﻷﺴﺌﻠﺔ اﻝﺘﺎﻝﻴﺔ ﻋﻠﻰ ﺘﻼﻤذﺘﻪ‪.‬‬

‫ﺼﻔﺤﺔ ‪ 8‬ﻤن ‪9‬‬


‫‪ .‬ﺒﻜﺎﻝورﻴﺎ ﺘﺠرﻴﺒﻲ ‪2023‬‬ ‫اﻝﺸﻌﺒﺔ‪ :‬رﻴﺎﻀﻴﺎت‪ ،‬ﺘﻘﻨﻲ رﻴﺎﻀﻲ‬ ‫اﺨﺘﺒﺎر ﻓﻲ ﻤﺎدة اﻝﻌﻠوم اﻝﻔﻴزﻴﺎﺌﻴﺔ‪.‬‬
‫أ‪ .‬ﻋﻨد ﺘوﺼﻴل ﺠﻬﺎز ‪ EX A O‬ﻝﻤﺸﺎﻫدة اﻝﺘوﺘر ) ‪ u AB (t‬ﺘﺒﻴن أﻨﻪ ﻏﻴر ﺼﺎﻝﺢ ﻝﻼﺴﺘﻌﻤﺎل‪ ،‬ﻓطﻠب ﻤن أﺤد ﺘﻼﻤﻴذﻩ إﺤﻀﺎر‬
‫ﺠﻬﺎز أﺨر ﻴؤدي ﻨﻔس اﻝﻐرض‪ .‬ﻤﺎ اﺴم ﻫذا اﻝﺠﻬﺎز‪ ،‬ﺒﻴن ﻜﻴف ﻴﻤﻜن ﺘوﺼﻴﻠﻪ ﻝﻤﺸﺎﻫدة اﻝﺘوﺘر ) ‪. u AB (t‬‬
‫ب‪ .‬ﻤن ﺒﻴن اﻝﻤﻨﺤﻨﻴﺎت اﻝﺘﺎﻝﻴﺔ اﺨﺘر ﻤﻊ اﻝﺘﻌﻠﻴل اﻝﻤﻨﺤﻨﻰ اﻝﻤﺘﺤﺼل ﻋﻠﻴﻪ ﺒواﺴطﺔ اﻝﺠﻬﺎز اﻝذي أﺤﻀرﻩ اﻝﺘﻠﻤﻴذ وﻫذا ﻝﻤﺸﺎﻫدة‬
‫اﻝﺘوﺘر ) ‪. u AB (t‬‬

‫‪II‬‬ ‫‪III‬‬ ‫‪IV‬‬


‫‪I‬‬

‫‪ .2‬أ‪ .‬أﻜﺘب ﻋﺒﺎرة اﻝطﺎﻗﺔ اﻝﻤﺨزﻨﺔ ﻓﻲ اﻝوﺸﻴﻌﺔ ) ‪ Eb (t‬ﺒدﻻﻝﺔ ذاﺘﻴﺔ اﻝوﺸﻴﻌﺔ ) ‪ ( L‬وﺸدة اﻝﺘﻴﺎر اﻝﻤﺎر ﺒﻬﺎ ) ‪. i (t‬‬
‫‪t‬‬
‫‪−2‬‬
‫‪ Eb (t ) = E0e‬ﺤﻴث ‪ E0‬ﺜﺎﺒت ﻴطﻠب ﺘﻌﻴﻴﻨﻪ‪.‬‬ ‫‪τ‬‬
‫ب‪ .‬ﺒﻴن أن ﻋﺒﺎرة ) ‪ Eb (t‬ﻴﻤﻜن ﻜﺘﺎﺒﺘﻬﺎ ﻋﻠﻰ اﻝﺸﻜل‬
‫‪ .4‬ﺘﺤﺼﻠﻨﺎ ﻋﻠﻰ اﻝوﺜﻴﻘﺔ اﻝﺘﺎﻝﻴﺔ واﻝﺘﻲ ﺘﻤﺜل اﻝﺸﻜﻠﻴن‪ :‬اﻝﺸﻜل )‪-5‬أ( وﻫو ﺘﻤﺜﻴل ﻝﻠﻤﻨﺤﻨﻰ اﻝﺒﻴﺎﻨﻲ ) ‪ Eb = f (i 2‬واﻝﺸﻜل)‪-5‬ب( وﻫو‬
‫ﺘﻤﺜﻴل ﻝﻠﻤﻨﺤﻨﻰ اﻝﺒﻴﺎﻨﻲ ) ‪ . Eb = g (t‬ﺒﺎﺴﺘﻐﻼل اﻝﺒﻴﺎﻨﺎت‪:‬‬
‫) ‪Eb (×10−3 J‬‬
‫) ‪Eb (×10 −3 J‬‬

‫‪1‬‬
‫ل‪5-‬أ‬ ‫ا‬
‫‪1‬‬ ‫) ‪t (×10 −2 s‬‬
‫‪1‬‬
‫) ‪i 2 (×10−2 A2‬‬
‫ل‪5-‬ب‬ ‫ا‬
‫‪0‬‬ ‫‪0, 25‬‬
‫‪ .1.4‬اوﺠد ﻗﻴﻤﺔ ﺸدة اﻝﺘﻴﺎر ‪ I 0‬وﻗﻴﻤﺔ اﻝطﺎﻗﺔ ‪. E0‬‬
‫‪ .2.4‬اﺴﺘﻨﺘﺞ ذاﺘﻴﺔ اﻝوﺸﻴﻌﺔ ‪. L‬‬
‫‪ .3‬أﺤﺴب ﻗﻴﻤﺔ ‪ Eb‬ﻋﻨد اﻝﻠﺤظﺔ ‪ t = τ‬وﻤن اﻝﻤﻨﺤﻨﻰ اﻝﺸﻜل‪5-‬ب أوﺠد ﻗﻴﻤﺔ ‪. τ‬‬
‫‪ .4‬اﺴﺘﻨﺘﺞ اﻝﻤﻘﺎوﻤﺔ اﻝداﺨﻠﻴﺔ ﻝﻠوﺸﻴﻌﺔ ‪ r‬واﻝﻘوة اﻝﻤﺤرﻜﺔ اﻝﻜﻬرﺒﺎﺌﻴﺔ ﻝﻠﻤوﻝد ‪. E‬‬

‫إﻋداد اﻷﺴﺎﺘذة ‪-:‬ﺴﺎﻋﻲ ﻤﺴﻌود –ﺒوﻗطﺎﻴﺔ ﻤﺤﻤد –ﻏوﺒﺎل رﻤزي‬

‫اﻨﺘﻬﻰ اﻝﻤوﻀوع اﻝﺜﺎﻨﻲ‬

‫ﺼﻔﺤﺔ ‪ 9‬ﻤن ‪9‬‬


‫ﻣﻊ ﺳﻠﻢ ﺍﻟﺘﻨﻘﻴﻂ‬ ‫ﻟﻠﻤﻮﺿﻮﻉ ﺍﻷﻭﻝ‬ ‫ﺍﻹﺟﺎﺑﺔ ﺍﻟﻨﻤﻮﺫﺟﻴﺔ‬
‫ﺍﻟﺠﺰﺀ ﺍﻷﻭﻝ )‪ 14‬ن(‬
‫ﺍﻟﺘﻤﺮﻳﻦ ﺍﻷﻭﻝ ‪ 05) :‬ﻥ(‬
‫ﺍﺳﻢ ﺍﻟﺘﺤﻮﻝ ﻫﻮ‪ :‬ﺍﻻﻧﺸﻄﺎﺭ‪.‬‬ ‫‪-1 -I‬‬
‫‪0.25‬‬ ‫ﺗﻌﺮﻳﻔﻪ‪ :‬ﻫﻮ ﺗﻔﺎﻋﻞ ﻣﻔﺘﻌﻞ ﻳﺘﻢ ﻓﻴﻪ ﻗﺬﻑ ﻧﻮﺍﺓ ﺛﻘﻴﻠﺔ ﺑﻨﺘﺮﻭﻥ ﺑﻄﻲﺀ ﻓﺘﻨﻘﺴﻢ ﺇﻟﻰ ﻧﻮﺍﺗﻴﻦ ﺃﺧﻒ‬
‫ﻭﺃﻛﺜﺮ ﺍﺳﺘﻘﺮﺍﺭﺍ ﻣﻊ ﺗﺤﺮﻳﺮ ﻃﺎﻗﺔ ﻭﻧﺘﺮﻭﻧﺎﺕ‪.‬‬
‫ﺗﻌﻴﻴﻦ ‪ Z‬ﻭ ‪ : A‬ﺑﺘﻄﺒﻴﻖ ﻗﻮﺍﻧﻴﻦ ﺍﻻﻧﺤﻔﺎﻅ‪:‬‬ ‫‪-2‬‬
‫‪0.25‬‬ ‫‪235 1134  99  k  k  3‬‬
‫‪92  52  Z  Z  40‬‬
‫‪:‬‬ ‫ﺣﺴﺎﺏ ﺍﻟﻄﺎﻗﺔ ﺍﻟﻤﺤﺮﺭﺓ‬ ‫‪-3‬‬
‫‪0.25‬‬ ‫=‬ ‫‪+‬‬ ‫(‬ ‫‪)−‬‬ ‫‪−‬‬ ‫‪−3‬‬ ‫‪.‬‬
‫‪0.25‬‬ ‫‪= 184,9‬‬
‫‪0.25‬‬ ‫‪= 2,958 × 10‬‬
‫ﺧﻼﻝ ﻳﻮﻡ‪:‬‬ ‫‪ -1‬ﺣﺴﺎﺏ ﺍﻟﻄﺎﻗﺔ ﺍﻟﻜﻬﺮﺑﺎﺋﻴﺔ ﺍﻟﻨﺎﺗﺠﺔ‬
‫‪0.25‬‬
‫=‬ ‫→‬ ‫=‬ ‫‪. ∆ = 15 × 10 × 24 × 3600 = 1,296 × 10‬‬
‫∆‬
‫‪ -2‬ﺣﺴﺎﺏ ﺍﻟﻄﺎﻗﺔ ﺍﻟﻤﺤﺮﺭﺓ ﻣﻦ ﺍﻟﻤﻔﺎﻋﻞ ﺍﻟﻨﻮﻭﻱ ‪ ′‬ﻋﻨﺪﺋﺬ‪:‬‬
‫‪0.25‬‬ ‫‪1,296 × 10‬‬
‫‪′‬‬ ‫=‬ ‫= ‪× 100‬‬ ‫‪× 100 = 4,32 × 10‬‬
‫‪30‬‬
‫‪ -3‬ﺍﺳﺘﻨﺘﺞ ﻣﻘﺪﺍﺭ ﺍﻟﻜﺘﻠﺔ ﻟﻠﻴﻮﺭﺍﻧﻴﻮﻡ ‪ 235‬ﺍﻟﻤﺴﺘﻬﻠﻜﺔ‪:‬‬
‫‪′‬‬
‫‪0.25‬‬ ‫‪′‬‬ ‫‪= .‬‬ ‫= →‬ ‫‪= 146 × 10‬‬
‫‪.‬‬
‫=‬ ‫‪= 56,98‬‬
‫‪-II‬‬
‫‪134‬‬
‫‪Te  134‬‬ ‫‪0‬‬ ‫‪134‬‬ ‫‪-1‬‬
‫‪0.25‬‬ ‫‪52‬‬ ‫ﻣﻌﺎﺩﻟﺔ ﺗﻔﻜﻚ ‪53 X  1e   : 52Te‬‬

‫ﻟﻬﺎ ﻓﺎﺋﺾ ﻓﻲ ﺍﻟﻨﺘﺮﻭﻧﺎﺕ ﻓﻴﺘﺤﻮﻝ ﻧﻴﺘﺮﻭﻥ ﻹﻟﻰ ﺑﺮﻭﺗﻮﻥ ﻋﻠﻰ ﻣﺴﺘﻮﻯ‬ ‫ﻣﺼﺪﺭ ‪ :  ‬ﻧﻮﺍﺓ ‪Te‬‬
‫‪134‬‬
‫‪52‬‬

‫‪0.25‬‬ ‫‪1‬‬
‫‪0‬‬ ‫ﺍﻟﻨﻮﺍﺓ ﻭﻓﻖ ﺍﻟﻤﻌﺎﺩﻟﺔ‪n  11P  10e :‬‬

‫ﻣﺼﺪﺭ ‪ : ‬ﻧﻮﺍﺓ ﺍﻟﺒﻨﺖ ﺍﻟﻨﺎﺗﺠﺔ ﻓﻲ ﺣﺎﻟﺔ ﻣﺜﺎﺭﺓ ﺃﻱ ﻟﻬﺎ ﺯﻳﺎﺩﺓ ﻓﻲ ﺍﻟﻄﺎﻗﺔ ﻓﺘﺘﺨﻠﺺ ﻣﻨﻬﺎ ﺑﺎﺻﺪﺍﺭ‬
‫‪0.25‬‬
‫ﺍﺷﻌﺎﻋﺎﺕ ‪. ‬‬
‫ﺗﻌﺮﻳﻒ ﻃﺎﻗﺔ ﺗﻤﺎﺳﻚ ﺍﻟﻨﻮﺍﺓ‪ :‬ﻫﻲ ﺍﻟﻄﺎﻗﺔ ﺍﻟﻼﺯﻣﺔ ﺗﻮﻓﻴﺮﻫﺎ ﻣﻦ ﺍﻟﻮﺳﻂ ﺍﻟﺨﺎﺭﺟﻲ ﻟﺘﻔﻜﻴﻚ‬ ‫‪-2‬‬
‫‪0.25‬‬ ‫ﻧﻮﺍﺓ ﻣﺘﻤﺎﺳﻜﺔ ﻭﺳﺎﻛﻨﺔ ﺇﻟﻰ ﻧﻜﻠﻴﻮﻧﺎﺗﻬﺎ ﺍﻟﺤﺮﺓ ﻭﺍﻟﺴﺎﻛﻨﺔ‪.‬‬
‫‪. El‬‬ ‫‪‬‬ ‫‪A‬‬
‫‪Z‬‬ ‫‪‬‬ ‫‪‬‬
‫‪X   Z .m p  (A  Z )m p  m ZA X  .c 2‬‬ ‫‪‬‬
‫‪:‬‬ ‫ﺣﺴﺎﺏ ﻃﺎﻗﺔ ﺍﻟﺘﻤﺎﺳﻚ ﻟﻠﻨﻮﺍﺓ ﺍﻟﺘﻴﻠﻮﺭﻳﻮﻡ‬ ‫‪-3‬‬
‫‪0.25‬‬
‫‪El‬‬ ‫‪‬‬ ‫‪‬‬ ‫‪‬‬
‫‪Te  52.m p  82m p  m ZA X  .c 2  1127,115MeV‬‬
‫‪134‬‬
‫‪52‬‬ ‫‪‬‬
‫ﻗﺎﺭﻥ ﺑﻴﻦ ﺍﺳﺘﻘﺮﺍﺭ ﻧﻮﺍﺓ ﺍﻷﻡ ﻭﻧﻮﺍﺓ ﺍﻟﺒﻨﺖ‪:‬‬ ‫‪-4‬‬
‫ﺍﻟﺒﻨﺖ‬
‫‪0.25‬‬ ‫‪= 8,411306‬‬ ‫‪/‬‬ ‫‪،‬‬ ‫‪= 8,41687‬‬ ‫‪/‬‬
‫ﺍﻟﺒﻨﺖ‬
‫ﻓﺎﻥ ﺍﻟﻨﻮﺍﺓ ﺍﻟﺒﻨﺖ ﺃﻛﺜﺮ ﺍﺳﺘﻘﺮﺍﺭﺍ‪.‬‬ ‫<‬ ‫ﺑﻤﺎ ﺃﻥ‬
‫‪-5‬‬
‫‪1‬‬
‫‪0.25‬‬ ‫ﺃ‪ -‬ﻗﻴﻤﺔ ﺍﻟﻨﺸﺎﻁ ‪A 0  4.2  1013 Bq : A0‬‬
‫‪0.25‬‬ ‫ﺏ‪ -‬ﻗﻴﻤﺔ ﺍﻟﻨﺸﺎﻁ ﺍﻹﺷﻌﺎﻋﻲ ﻓﻲ ﺍﻟﻠﺤﻈﺔ ‪A ( )  0.37 A 0  1.554  1013 Bq : t  ‬‬
‫‪0.25‬‬ ‫ﺍﺳﺘﻨﺘﺎﺝ ﻗﻴﻤﺔ ‪ :‬ﺑﺎﺳﻘﺎﻁ ﺍﻟﻘﻴﻤﺔ ﺍﻟﺴﺎﺑﻘﺔ ﻋﻠﻰ ﺍﻟﺒﻴﺎﻥ ﻧﺠﺪ ‪  60,6min :‬‬
‫‪0.25‬‬ ‫‪A (t 1/ 2 )  A 0 2  A 0e   .t1/2  t 1/ 2 ‬‬ ‫)‪ln(2‬‬
‫‪‬‬
‫ﺝ‪ -‬ﺇﺛﺒﺎﺕ ﺃﻥ ‪  ln(2) : t1/2   ln 2‬‬
‫‪0.25‬‬ ‫‪t1/2   ln(2)  42 min‬‬
‫‪0.25‬‬ ‫ﺑﻴﺎﻧﻴﺎ ‪t1/2  2  20,9  41,8min :‬‬

‫ﺣﻞ ﺍﻟﺘﻤﺮﻳﻦ ﺍﻟﺜﺎﻧﻲ‪ 04) :‬ﻥ(‬


‫‪0.25‬‬ ‫ﻧﺪﺭﺱ ﺣﺮﻛﺔ ﺍﻟﺮﻣﺢ ﻓﻲ ﺍﻟﻤﺮﺟﻊ ﺍﻟﺴﻄﺤﻲ ﺍﻷﺭﺿﻲ‪.‬‬ ‫‪-1‬‬
‫ﺍﻟﺸﺮﻁ ﻛﻲ ﻳﻜﻮﻥ ﻫﺬﺍ ﺍﻟﻤﺮﺟﻊ ﻏﺎﻟﻴﻠﻴﺎ ﻫﻮ ﺃﻥ ﺗﻜﻮﻥ ﻣﺪﺓ ﺍﻟﺪﺭﺍﺳﺔ ﺻﻐﻴﺮﺓ ﺟﺪﺍ ﺑﺎﻟﻨﺴﺒﺔ‬
‫‪0.25‬‬
‫ﻟﻤﺪﺓ ﺩﻭﺭﺍﻥ ﺍﻷﺭﺽ ﺣﻮﻝ ﻧﻔﺴﻬﺎ‪.‬‬
‫ﻋﺒﺎﺭﺓ ﺗﺴﺎﺭﻉ ﻣﺮﻛﺰ ﻋﻄﺎﻟﺔ ﺍﻟﺮﻣﺢ‪:‬‬ ‫‪-2‬‬
‫ﺍﻟﺠﻤﻠﺔ‪ :‬ﺍﻟﺮﻣﺢ‬
‫ﺍﻟﻤﺮﺟﻊ ‪ :‬ﺳﻄﺤﻲ ﺃﺭﺿﻲ ﻧﻌﺘﺒﺮﻩ ﻋﻄﺎﻟﻲ‬
‫‪0.25‬‬
‫∑‬ ‫=⃗‬ ‫=⃗ →⃗ ‪.‬‬ ‫ﺑﺘﻄﺒﻴﻖ ﺍﻟﻘﺎﻧﻮﻥ ﺍﻟﺜﺎﻧﻲ ﻟﻨﻴﻮﺗﻦ ‪. ⃗ :‬‬
‫=‪0‬‬ ‫‪.‬‬ ‫→‬ ‫(‪=0:‬‬ ‫ﺑﺎﻹﺳﻘﺎﻁ ﻋﻠﻰ ﺍﻟﻤﺤﻮﺭ)‬
‫‪− .‬‬ ‫=‬ ‫‪.‬‬ ‫→‬ ‫(‪=− :‬‬ ‫ﺑﺎﻹﺳﻘﺎﻁ ﻋﻠﻰ ﺍﻟﻤﺤﻮﺭ)‬
‫ﻃﺒﻴﻌﺔ ﺍﻟﺤﺮﻛﺔ‪:‬‬ ‫‪-3‬‬
‫‪0.25‬‬ ‫ﻓﺎﻥ ﺍﻟﺤﺮﻛﺔ ﻣﺴﺘﻘﻴﻤﺔ ﻣﻨﺘﻈﻤﺔ‪.‬‬ ‫(‪ :‬ﺑﻤﺎ ﺃﻥ ‪= 0 :‬‬ ‫ﻋﻠﻰ ﺍﻟﻤﺤﻮﺭ)‬ ‫‪‬‬
‫‪0.25‬‬ ‫ﻓﺎﻥ ﺍﻟﺤﺮﻛﺔ ﻣﺴﺘﻘﻴﻤﺔ ﻣﺘﻐﻴﺮﺓ ﺑﺎﻧﺘﻈﺎﻡ‪.‬‬ ‫(‪ :‬ﺑﻤﺎ ﺃﻥ ‪= − :‬‬ ‫ﻋﻠﻰ ﺍﻟﻤﺤﻮﺭ)‬ ‫‪‬‬
‫ﺍﻟﻤﻌﺎﺩﻻﺕ ﺍﻟﺰﻣﻨﻴﺔ‪:‬‬ ‫‪‬‬
‫‪0.25‬‬ ‫)‪= − … … … … … … … … . . . . (4‬‬ ‫)‪= 0 … … … … … . … (1‬‬
‫‪0.25‬‬ ‫(‬ ‫)‬ ‫‪=− +‬‬ ‫( ‪… … … … … (5) ،‬‬ ‫)‬ ‫=‬ ‫)‪… … … . . (2‬‬
‫‪0.25‬‬ ‫‪=−‬‬ ‫‪+‬‬ ‫)‪. … … . (6‬‬ ‫=‬ ‫)‪. … … … . (3‬‬
‫‪0.25‬‬
‫ﻣﻌﺎﺩﻟﺔ ﺍﻟﻤﺴﺎﺭ‪:‬‬ ‫‪-4‬‬

‫‪0.25‬‬ ‫= → )‪(3‬‬

‫‪=−‬‬ ‫‪+ .‬‬ ‫ﺑﺎﻟﺘﻌﻮﻳﺾ ﻓﻲ )‪ (6‬ﻧﺠﺪ‪:‬‬


‫‪.‬‬
‫‪-5‬‬

‫‪0.25‬‬ ‫=‬ ‫‪.‬‬ ‫→‬ ‫=‬ ‫‪ :‬ﻟﺪﻳﻨﺎ ‪= 1‬‬ ‫ﺃ‪ -‬ﺇﻳﺠﺎﺩ‬


‫ﺏ‪ -‬ﺍﻟﻔﺮﺿﻴﺔ ﺍﻟﺘﻲ ﺗﺴﻤﺢ ﺑﺄﻥ ﻳﺒﻠﻎ ﺍﻟﺮﻣﺢ ﺍﻟﻨﻘﻄﺔ ‪ ، A‬ﻭﻓﻖ ﺍﻟﻤﺴﺎﺭ )‪ (OA‬ﻣﻊ ﺍﻟﺤﻔﺎﻅ ﻋﻠﻰ ﻧﻔﺲ‬
‫ﺍﻟﺸﺮﻭﻁ ﺍﻻﺑﺘﺪﺍﺋﻴﺔ ﻫﻲ ﺇﻫﻤﺎﻝ ﺗﺄﺛﻴﺮ ﺍﻟﺠﺎﺫﺑﻴﺔ ﻋﻠﻰ ﺍﻟﺠﺴﻢ )ﺇﻫﻤﺎﻝ ﻗﻮﺓ ﺍﻟﺜﻘﻞ( ﻷﻧﻪ ﻋﻨﺪ‬
‫‪0.25‬‬
‫ﺇﻫﻤﺎﻝ ﻗﻮﺓ ﺍﻟﺜﻘﻞ ﺗﺼﺒﺢ ﻣﺤﺼﻠﺔ ﺍﻟﻘﻮﻯ ﺍﻟﺨﺎﺟﻴﺔ ﺍﻟﻤﺆﺛﺮﺓ ﻋﻠﻰ ﺍﻟﺮﻣﺢ ﻣﻌﺪﻭﻣﺔ ﺃﻱ ﺗﺼﺒﺢ‬
‫‪.‬‬ ‫ﺍﻟﺤﺮﻛﺔ ﻣﺴﺘﻘﻴﻤﺔ ﻣﻨﺘﻈﻤﺔ ﻭﻓﻖ‬
‫ﺝ‪ -‬ﺇﻳﺠﺎﺩ ﻋﺒﺎﺭﺓ ‪ h‬ﺑﺪﻻﻟﺔ ‪  ، v0‬ﻭ ‪: tc‬‬
‫‪0.25‬‬ ‫=‬ ‫‪.‬‬ ‫ﺑﻤﺎ ﺃﻥ ﺍﻟﺤﺮﻛﺔ ﻣﺴﺘﻘﻴﻤﺔ ﻣﻨﺘﻈﻤﺔ ﻋﻠﻰ ‪ :OA‬ﻓﺎﻥ‪:‬‬
‫= ‪.ℎ‬‬ ‫‪.‬‬ ‫=‪→ℎ‬‬ ‫‪. .‬‬ ‫ﻭﻟﺪﻳﻨﺎ ﻣﻦ ﺍﻟﺸﻜﻞ‪:‬‬
‫‪1 2‬‬
‫‪0.25‬‬ ‫‪h‬‬ ‫ﺩ‪ -‬ﺇﺛﺒﺎﺕ ﺃﻥ ﻋﺒﺎﺭﺓ ﺍﻻﺭﺗﻔﺎﻉ ‪ h‬ﻓﻲ ﻫﺬﻩ ﺍﻟﺤﺎﻟﺔ ﻫﻮ‪gtc :‬‬
‫‪2‬‬

‫‪2‬‬
‫‪=−‬‬ ‫‪+‬‬ ‫‪.‬‬ ‫‪=0→−‬‬ ‫=‪+ℎ=0→ℎ‬‬ ‫ﻟﺪﻳﻨﺎ‪:‬‬
‫‪0.25‬‬ ‫=‪ℎ‬‬ ‫‪= × 10 × 1 = 5‬‬ ‫ﻗﻴﻤﺔ ‪:ℎ‬‬ ‫‪‬‬
‫ﻩ‪ -‬ﺍﻟﺘﺄﻛﺪ ﻣﻦ ﺍﻟﺸﻜﻞ‪:‬‬
‫‪0.25‬‬
‫=‬ ‫=‬ ‫→‬ ‫=‬ ‫=‪→ℎ‬‬ ‫‪.‬‬ ‫‪.‬‬ ‫‪≃5‬‬ ‫ﻣﻦ ﺍﻟﻤﺜﻠﺚ ‪: OAC‬‬
‫‪.‬‬

‫ﺣﻞ ﺍﻟﺘﻤﺮﻳﻦ ﺍﻟﺜﺎﻟﺚ‪ 05) :‬ﻥ(‬


‫ﺇﺭﻓﺎﻕ ﻛﻞ ﺑﻴﺎﻥ ﺑﺎﻟﻤﺪﺧﻞ ﺍﻟﻤﻮﺍﻓﻖ‪:‬‬ ‫‪-1‬‬
‫ﻭﻣﻨﻪ‪:‬‬ ‫ﻫﻮ ﺍﻟﺘﻮﺗﺮ ﺍﻟﻤﻮﺍﻓﻖ ﻟﻠﻤﻮﻟﺪ ﺃﻱ‪= E :‬‬ ‫ﺍﻟﺘﻮﺗﺮ‬
‫‪0.25‬‬
‫ﻓﻲ ﺍﻟﻤﺪﺧﻞ‬ ‫‪ ‬ﺍﻟﺒﻴﺎﻥ ‪ a‬ﻳﻮﺍﻓﻖ ‪= E‬‬
‫ﻓﻲ ﺍﻟﻤﺪﺧﻞ ‪.‬‬ ‫‪ ‬ﺍﻟﺒﻴﺎﻥ ‪ b‬ﻳﻮﺍﻓﻖ‬
‫ﺍﻟﻤﻌﺎﺩﻟﺔ ﺍﻟﺘﻔﺎﺿﻠﻴﺔ ﺑﺪﻻﻟﺔ ) ( ‪:‬‬ ‫‪-2‬‬

‫‪0.25‬‬ ‫‪+‬‬ ‫‪+‬‬ ‫=‬ ‫→‬ ‫(‪+‬‬ ‫‪+‬‬ ‫ﺣﺴﺐ ﻗﺎﻧﻮﻥ ﺟﻤﻊ ﺍﻟﺘﻮﺗﺮﺍﺕ‪) = :‬‬

‫(→‬ ‫‪+‬‬ ‫)‬ ‫‪+‬‬ ‫=‬


‫(‬ ‫)‬
‫=) ( ‪.‬‬ ‫‪−‬‬ ‫‪.‬‬ ‫‪.‬‬
‫ﺇﺛﺒﺎﺕ ﺃﻥ‪:‬‬ ‫‪-3‬‬
‫‪0.25‬‬
‫‪( )= .‬‬ ‫‪.‬‬
‫=‬ ‫= →‬ ‫‪.‬‬
‫ﻟﺪﻳﻨﺎ‪:‬‬
‫‪0.25‬‬ ‫(‪+‬‬ ‫‪+‬‬ ‫= )‬ ‫→‬ ‫=‬ ‫(‪−‬‬ ‫‪+‬‬ ‫)‬ ‫‪.‬‬
‫ﻣﻦ ﻗﺎﻧﻮﻥ ﺟﻤﻊ ﺍﻟﺘﻮﺗﺮﺍﺕ‪:‬‬
‫ﻭ ‪:‬‬ ‫ﺇﻳﺠﺎﺩ ﻗﻴﻤﺔ‬ ‫‪-4‬‬
‫‪.‬‬
‫‪0.25‬‬ ‫→ ‪(0) = 0‬‬ ‫(‪−‬‬ ‫‪+‬‬ ‫)‬ ‫→‪=0‬‬ ‫(=‬ ‫)‬
‫ﻣﻦ ﺍﻟﺸﺮﻭﻁ ﺍﻻﺑﺘﺪﺍﺋﻴﺔ‪:‬‬
‫=‬ ‫‪−‬‬ ‫‪.‬‬
‫→‬ ‫=‬ ‫‪.‬‬
‫ﻭﻣﻨﻪ‪:‬‬
‫(‬ ‫‪+‬‬ ‫‪) .‬‬ ‫‪.‬‬
‫‪+‬‬ ‫‪−‬‬ ‫‪.‬‬
‫=‬ ‫→‬ ‫ﺑﺎﻟﺘﻌﻮﻳﺾ ﻓﻲ ﺍﻟﻤﻌﺎﺩﻟﺔ ﺍﻟﺘﻔﺎﺿﻠﻴﺔ‪= :‬‬
‫‪0.25‬‬ ‫(‬ ‫)‬
‫=‬
‫‪= (1 −‬‬ ‫‪/‬‬
‫ﺃﻱ‪) :‬‬
‫‪:‬‬
‫ﺍﻟﻌﺒﺎﺭﺓ ﺍﻟﻠﺤﻈﻴﺔ ﻟـ ) (‬ ‫‪-5‬‬
‫‪.‬‬ ‫‪.‬‬ ‫‪.‬‬
‫‪.‬‬ ‫‪/‬‬
‫‪0.25‬‬ ‫=) (‬ ‫‪. +‬‬ ‫=‬ ‫‪.‬‬ ‫‪.‬‬ ‫‪+‬‬ ‫‪−‬‬ ‫‪= −‬‬ ‫‪.‬‬
‫(‬ ‫‪+‬‬ ‫)‬ ‫) ‪( +‬‬
‫‪0.25‬‬ ‫‪ -1‬ﻣﻦ ﺍﻟﻤﻨﺤﻨﻰ ‪ a‬ﻧﺠﺪ‪= 6 :‬‬ ‫‪-6‬‬
‫‪ ‬ﻣﻦ ﺍﻟﺒﻴﺎﻥ ‪ b‬ﻧﺠﺪ‪= 10Ω :‬‬
‫‪.‬‬
‫‪0.25‬‬ ‫= )‪(0‬‬ ‫→‪=2‬‬
‫(‬ ‫)‬
‫‪0.25‬‬ ‫‪5 = 0,75‬‬ ‫→‬ ‫‪= 0,15‬‬ ‫‪ ‬ﻟﺪﻳﻨﺎ‪:‬‬
‫‪0.25‬‬ ‫(= ‪.‬‬ ‫‪+‬‬ ‫→ )‬ ‫(=‬ ‫)‬
‫‪= 5 × 10‬‬ ‫‪ ‬ﺍﺳﺘﻨﺘﺎﺝ ﺳﻌﺔ ﺍﻟﻤﻜﺜﻔﺔ ‪:‬‬

‫‪:‬‬ ‫‪ -2‬اﻟﻣﻌﺎدﻟﺔ اﻟﺗﻔﺎﺿﻠﯾﺔ ﺑدﻻﻟﺔ ) (‬


‫‪+‬‬ ‫‪+‬‬ ‫=‬ ‫→‬ ‫(‪+‬‬ ‫‪+‬‬ ‫= )‬ ‫ﺣﺳب ﻗﺎﻧون ﺟﻣﻊ اﻟﺗوﺗرات‪:‬‬
‫‪0.25‬‬ ‫(→‬ ‫‪+‬‬ ‫)‬ ‫‪+‬‬ ‫=‬

‫‪0.25‬‬ ‫‪:‬‬ ‫‪ -3‬ﺑﻴﺎﻥ ﺍﻟﺘﻮﺗﺮ ) (‬


‫‪.‬‬ ‫‪/‬‬
‫‪0.25‬‬ ‫=) (‬
‫(‬ ‫‪+‬‬ ‫)‬
‫‪3‬‬
‫) (‬ ‫∞→‬ ‫=‬ ‫‪=0‬‬
‫‪0‬‬ ‫‪0,74‬‬ ‫‪2‬‬ ‫) (‬

‫‪0‬‬ ‫(‬ ‫)‬

‫‪:‬‬ ‫=‬ ‫ﻣﻦ ﺃﺟﻞ‬ ‫ﻭ‬ ‫‪ -4‬ﺣﺴﺎﺏ‬


‫‪0.25‬‬ ‫=‬ ‫= ‪.‬‬ ‫‪.‬‬ ‫ﻟﺪﻳﻨﺎ‪:‬‬
‫‪+‬‬ ‫‪+‬‬ ‫=‬ ‫→‬ ‫‪+‬‬ ‫‪.‬‬ ‫‪+‬‬ ‫=‬ ‫→‬ ‫ﻭﻟﺪﻳﻨﺎ ‪= 0,5‬‬
‫‪0.25‬‬ ‫‪.‬‬ ‫=‬ ‫= ‪.‬‬ ‫‪.‬‬ ‫‪=1‬‬ ‫ﻭﻣﻨﻪ‪:‬‬
‫ﺍﻟﻄﺎﻗﺔ ﺍﻟﺘﻲ ﺗﺨﺰﻧﻬﺎ ﺍﻟﻤﻜﺜﻔﺔ ﻋﻨﺪ ﺍﻟﻠﺤﻈﺔ ‪: = 5‬‬ ‫‪-7‬‬
‫‪0.25‬‬ ‫=‬ ‫ﻋﻨﺪ ﺍﻟﻠﺤﻈﺔ ‪ : = 5‬ﻳﻜﻮﻥ‬
‫=‬ ‫→‬ ‫=) ‪( =5‬‬ ‫‪= 90 × 10‬‬ ‫ﻭﻣﻨﻪ‪:‬‬
‫ﺍﻟﻤﻨﺤﻨﻰ )‪ (2‬ﻳﻮﺍﻓﻖ ﺍﻟﺘﺠﺮﺑﺔ ﺍﻟﻤﺪﺭﻭﺳﺔ ﻓﻲ ﺍﻟﺘﻤﺮﻳﻦ ﺃﻱ ﺍﻟﺘﺠﺮﺑﺔ ‪b‬‬ ‫‪-8‬‬
‫‪0.25‬‬
‫ﻋﻨﺪﻣﺎ ﺗﺰﺩﺍﺩ ﻗﻴﻤﺔ ‪ 2‬ﺗﺰﺩﺍﺩﺍ ﻗﻴﻤﺔ ﻭﻣﻨﻪ ﻣﺪﺓ ﺍﻟﻨﻈﺎﻡ ﺍﻻﻧﺘﻘﺎﻟﻲ ﺗﺰﺩﺍﺩ‪.‬‬ ‫‪‬‬
‫ﻓﻲ ﺍﻟﻤﻨﺤﻨﻰ )‪ (1‬ﺗﻜﻮﻥ ﻣﺪﺓ ﺍﻟﻨﻈﺎﻡ ﺍﻻﻧﺘﻘﺎﻟﻲ ﺃﻗﻞ ﻭﻣﻨﻪ ﺗﻮﺍﻓﻖ ﻗﻴﻤﺔ ﺃﻗﻞ ﺃﻱ ﻗﻴﻤﺔ‬ ‫‪‬‬
‫‪0.25‬‬
‫‪ 2‬ﺃﻗﻞ ﺃﻱ ﺍﻟﺘﺠﺮﺑﺔ ‪.a‬‬
‫ﻓﻲ ﺍﻟﻤﻨﺤﻨﻰ )‪ (3‬ﺗﻜﻮﻥ ﻣﺪﺓ ﺍﻟﻨﻈﺎﻡ ﺍﻻﻧﺘﻘﺎﻟﻲ ﺃﻛﺒﺮ ﻭﻣﻨﻪ ﺗﻮﺍﻓﻖ ﻗﻴﻤﺔ ﺃﻛﺒﺮ ﺃﻱ‬ ‫‪‬‬
‫‪0.25‬‬
‫ﻗﻴﻤﺔ ‪ 2‬ﺃﻛﺒﺮ ﺃﻱ ﺍﻟﺘﺠﺮﺑﺔ ‪.C‬‬

‫ﺍﻟﺠﺰﺀ ﺍﻟﺜﺎﻧﻲ‬
‫ﺣﻞ ﺍﻟﺘﻤﺮﻳﻦ ﺍﻟﺘﺠﺮﻳﺒﻲ‪ 06) :‬ﻥ(‬
‫ﺍﻟﻤﺠﻤﻮﻋﺔ ﺍﻷﻭﻟﻰ‪:‬‬ ‫‪-1‬‬
‫‪:( /‬‬ ‫ﺗﺤﺪﻳﺪ ﺍﻟﺜﻨﺎﺋﻴﺘﻴﻦ )‬ ‫ﺃ‪.‬‬
‫‪0.5‬‬ ‫=) (‬ ‫(‬ ‫‪)+2‬‬ ‫(‪………….‬‬ ‫) ‪/‬‬
‫‪0.5‬‬ ‫‪2‬‬ ‫‪+2‬‬ ‫=‬ ‫‪+2‬‬ ‫(…………‬ ‫) ‪/‬‬
‫ﺏ‪ .‬ﻫﻞ ﺍﻟﻤﺰﻳﺞ ﺍﻻﺑﺘﺪﺍﺋﻲ ﺳﺘﻜﻴﻮﻣﺘﺮﻱ‪:‬‬
‫(‬ ‫)‬
‫‪0.25‬‬ ‫ﻭﻣﻨﻪ ﺍﻟﻤﺰﻳﺞ ﻻ ﻳﺤﻘﻖ ﺍﻟﺸﺮﻭﻁ ﺍﻟﺴﺘﻮﻛﻴﻮﻣﺘﺮﻳﺔ‪.‬‬ ‫≠‬ ‫ﻷﻥ‬ ‫(‬ ‫≠)‬ ‫ﺑﻤﺎ ﺃﻥ ‪:‬‬

‫ﺝ‪ .‬ﺟﺪﻭﻝ ﺍﻟﺘﻘﺪﻡ‪:‬‬


‫) (‬ ‫‪+‬‬ ‫(‬ ‫)‬ ‫=‬ ‫(‬ ‫)‬ ‫‪+‬‬ ‫) (‬ ‫‪+‬‬ ‫)(‬
‫‪0,04‬‬ ‫‪3 × 10‬‬ ‫‪0‬‬ ‫‪0‬‬ ‫‪//‬‬
‫‪0.25‬‬
‫‪0,04 −‬‬ ‫‪3 × 10‬‬ ‫‪−2‬‬ ‫‪//‬‬
‫‪0,04 −‬‬ ‫‪3 × 10‬‬ ‫‪−2‬‬ ‫‪//‬‬
‫‪ ‬ﺍﻳﺠﺎﺩ ﺍﻟﻤﺘﻔﺎﻋﻞ ﺍﻟﻤﺤﺪ‪:‬‬
‫(‬ ‫)‬ ‫(‬ ‫)‬
‫‪0.25‬‬ ‫=‬ ‫ﻭ × ‪= 1,5‬‬ ‫ﻓﺎﻥ ﺍﻟﻤﺘﻔﺎﻋﻞ ﺍﻟﻤﺤﺪ ﻫﻮ‪:‬‬ ‫(‬ ‫>)‬ ‫ﺑﻤﺎ ﺃﻥ‬
‫‪0.25‬‬ ‫‪10‬‬
‫‪0.25‬‬ ‫‪.‬‬ ‫ﺩ‪ .‬ﺍﻟﺘﺮﻛﻴﺰ ﺍﻷﻋﻈﻤﻲ ﻟﻠﺸﻮﺍﺭﺩ‬
‫‪4‬‬
‫× ‪,‬‬
‫[‪.‬‬ ‫= ]‬ ‫=‬ ‫‪= 0,05‬‬ ‫ﻣﻦ ﺟﺪﻭﻝ ﺍﻟﺘﻘﺪﻡ‪/ :‬‬
‫×‬

‫‪0.25‬‬ ‫‪ :‬ﻫﻮ ﺍﻟﺰﻣﻦ ﺍﻟﻼﺯﻡ ﻟﺒﻠﻮﻍ ﺗﻘﺪﻡ ﺍﻟﺘﻔﺎﻋﻞ ﻧﺼﻒ ﻗﻴﻤﺘﻪ ﺍﻷﻋﻈﻤﻴﺔ‪.‬‬ ‫‪/‬‬ ‫ﺃ‪ .‬ﺯﻣﻦ ﻧﺼﻒ ﺍﻟﺘﻔﺎﻋﻞ‬ ‫‪-2‬‬
‫‪0.25‬‬ ‫‪/ = 2,3‬‬ ‫[ ﺑﺎﻻﺳﻘﺎﻁ ﻧﺠﺪ‪:‬‬ ‫‪] / = 2,5 × 10‬‬ ‫‪/‬‬
‫ﺏ‪ .‬ﻋﺒﺎﺭﺓ ﺍﻟﺴﺮﻋﺔ ﺍﻟﺤﺠﻤﻴﺔ ﻟﻠﺘﻔﺎﻋﻞ‪:‬‬
‫‪0.25‬‬
‫=‬ ‫=‬ ‫ﻟﺪﻳﻨﺎ‬
‫‪0.25‬‬ ‫= )‪( = 0‬‬ ‫‪= 2,5 × 10‬‬ ‫‪/ .‬‬ ‫ﻣﻦ ﺍﻟﺒﻴﺎﻥ‪:‬‬
‫‪0.25‬‬ ‫= )‪( = 8‬‬ ‫‪= 0,143 × 10‬‬ ‫‪/ .‬‬ ‫ﻭ‪:‬‬
‫‪0.25‬‬ ‫ﺝ‪ .‬ﻳﻤﻜﻦ ﻣﺘﺎﺑﻌﺔ ﻫﺬﺍ ﺍﻟﺘﺤﻮﻝ ﻋﻦ ﻃﺮﻳﻖ ﻗﻴﺎﺱ ﺍﻟﻨﺎﻗﻠﻴﺔ ﻷﻧﻪ ﻣﺤﻠﻮﻝ ﺷﺎﺭﺩﻱ‬
‫‪ ‬ﺗﻨﺎﻗﺺ ﺍﻟﻨﺎﻗﻠﻴﺔ ﺑﻤﺮﻭﺭ ﺍﻟﺰﻣﻦ ﺭﺍﺟﻊ ﻟﻠﻨﺎﻗﻠﻴﺔ ﺍﻟﻨﻮﻋﻴﺔ ﺍﻟﻤﻮﻟﻴﺔ ﺍﻟﺸﺎﺭﺩﻳﺔ ﻟﻠﺸﻮﺍﺭﺩ‬
‫‪0.25‬‬ ‫ﺃﻛﺒﺮ ﻣﻦ ﺍﻟﻨﺎﻗﻠﻴﺔ ﺍﻟﻨﻮﻋﻴﺔ ﺍﻟﻤﻮﻟﻴﺔ ﺍﻟﺸﺎﺭﺩﻳﺔ ﻟﻠﺸﻮﺍﺭﺩ ﺍﻟﻤﺘﺸﻜﻠﺔ‬ ‫ﺍﻟﻤﺨﺘﻔﻴﺔ‬
‫‪.‬‬
‫ﺍﻟﻤﺠﻤﻮﻋﺔ ﺍﻟﺜﺎﻧﻴﺔ‪:‬‬
‫ﻭ ﺍﻟﻘﻄﺐ ﺍﻟﺴﺎﻟﺐ ﻫﻮ‬ ‫ﻓﺎﻥ ﺍﻟﻘﻄﺐ ﺍﻟﻤﻮﺟﺐ ﻳﻤﺜﻞ‬ ‫˃‬ ‫ﺃ‪ .‬ﻗﻄﺒﻲ ﺍﻟﻌﻤﻮﺩ‪ :‬ﺑﻤﺎ ﺃﻥ‬
‫‪0.25‬‬

‫‪.‬‬ ‫=‬ ‫‪+ 2é ،‬‬ ‫= ‪+ 2é‬‬ ‫‪ ‬ﺍﻟﻤﻌﺎﺩﻟﺘﻴﻦ ﺍﻟﻨﺼﻔﻴﺘﻴﻦ‪:‬‬

‫‪0.25‬‬

‫‪0.25‬‬

‫‪0.25‬‬ ‫‪−‬‬ ‫‪/‬‬ ‫‪//‬‬ ‫‪/‬‬ ‫ﺏ‪ .‬ﺷﻜﻞ ﺗﺨﻄﻴﻄﻲ ﻟﻠﻌﻤﻮﺩ ﺍﻟﻤﺤﻘﻖ‪+ :‬‬
‫ﺝ‪.‬‬
‫‪:‬‬ ‫‪ .1‬ﻣﻘﺪﺍﺭ ﺍﻟﺘﻘﺪﻡ‬
‫∆‪.‬‬
‫‪0.25‬‬ ‫‪= . .‬‬ ‫→‬ ‫=‬ ‫‪= 1,12 × 10‬‬ ‫ﻟﺪﻳﻨﺎ ‪:‬‬
‫‪.‬‬
‫‪0.25‬‬
‫‪:‬‬ ‫‪ .2‬ﺍﻟﻨﻘﺺ ﺍﻟﻜﺘﻠﻲ ∆ ﻟﻤﺴﺮﻯ ﺍﻟﻤﻐﻨﻴﺰﻳﻮﻡ‬
‫‪0.25‬‬
‫(‬ ‫=)‬ ‫‪−‬‬ ‫→‬ ‫=‬ ‫( ‪−‬‬ ‫( ∆=)‬ ‫ﺑﺎﻻﻋﺘﻤﺎﺩ ﻋﻠﻰ ﺟﺪﻭﻝ ﺍﻟﺘﻘﺪﻡ‪) :‬‬
‫( ∆‬ ‫)‬
‫‪0.25‬‬ ‫( ∆‬ ‫=)‬ ‫=‬ ‫( ∆→‬ ‫‪)= .‬‬ ‫‪= 26,86 × 10‬‬ ‫ﻭﻣﻨﻪ‪:‬‬

‫‪5‬‬
‫الجمهورية الجزائرية الديمقراطية الشعبية‬
‫وزارة التربية الوطنية‬
‫ثانوية شرايطية يوسف – العوانة ‪-‬‬ ‫مديرية التربية لوالية جيجل‬
‫ثانوية ثرخاش محمد –بوراوي بلهادف‪-‬‬ ‫امتحان بكالوريا تجريبي ماي ‪2023‬‬
‫شعبة‪ :‬تقني رياضي‬
‫المدة‪ 04 :‬سا و‪ 30‬د‬ ‫اختبار في مادة‪ :‬العلوم الفيزيائية‬
‫على المترشح أن يختار أحد الموضوعين اآلتيين‪:‬‬
‫الموضوع األول‬
‫يحتوي الموضوع على (‪ )05‬صفحات (من الصفحة ‪ 01‬من ‪ 10‬إلى الصفحة ‪ 05‬من ‪)10‬‬
‫التمرين األول‪ 05( :‬نقاط)‬

‫مرت عدة سنوات على التفجيرات والتجارب النووية للمستعمر الفرنسي التي ما تزال تخلف ضحايا في الجنوب الجزائري‬
‫الشاسع‪ ،‬وذلك من يوم االنفجار الموافق لتاريخ ‪ 13‬فيفري ‪ 1960‬أين سجل المستعمر الفرنسي دخوله المدوي إلى نادي القوى‬
‫النووية مخلفا وراءه بالحمودية ضواحي رقان نفايات نووية إثر االنفجار المريع والتي الزالت بعد نصف قرن تلحق أض ار ار‬
‫بالبيئة والسكان بسبب اإلشعاعات المخلفة‪.‬‬
‫أهم النظائر المؤثرة على الصحة بعد االنفجار النووي‪.‬‬ ‫‪131‬‬
‫‪53‬‬ ‫ونظير اليود ‪I‬‬ ‫يعتبر كل من نظير السيزيوم ‪Cs‬‬
‫‪137‬‬
‫‪55‬‬

‫‪. 137‬‬
‫والسيزيوم ‪55 Cs‬‬
‫‪131‬‬
‫‪53‬‬ ‫يهدف التمرين لدراسة النشاط اإلشعاعي لكل من نظيري اليود ‪I‬‬
‫في حالة إثارة وفق نمط تفكك ‪.  ‬‬ ‫‪A‬‬
‫‪Z‬‬ ‫مشعة ينتج عن تفككها نواة الكزينون ‪Xe‬‬ ‫‪131‬‬
‫‪53‬‬ ‫‪ .1‬نواة اليود ‪I‬‬

‫أ‪ -‬اكتب معادلة التفاعل النووي الحادث‪ّ ،‬‬


‫وعين قيمتي كل من ‪ A‬و ‪. Z‬‬
‫ب‪ -‬عند رجوع النواة الناتجة المثارة إلى حالتها األساسية تبعث إشعاعات شديدة النفاذية‪.‬‬
‫‪ -‬اكتب معادلة تحولها النووي مع تحديد نوع النشاط اإلشعاعي‪.‬‬
‫الممثل في الشكل(‪.)1‬‬ ‫نستعمل المنحنى البياني ) ‪ f (t‬‬ ‫‪ .2‬من أجل تحديد زمن نصف العمر لنواة ‪I‬‬
‫)‪N d (t‬‬ ‫‪131‬‬
‫‪53‬‬
‫) ‪N (t‬‬
‫) ‪N d (t‬‬
‫) ‪N (t‬‬
‫أ‪ -‬اكتب عبارة التناقص اإلشعاعي ) ‪ N (t‬بداللة ‪  ، N0‬و ‪. t‬‬
‫ب‪ -‬استنتج عبارة األنوية المتفككة ) ‪ Nd (t‬بداللة ‪  ، N0‬و ‪. t‬‬
‫‪.‬‬ ‫ج‪ -‬أثبت من العالقات السابقة أن‪ e  1 :‬‬
‫‪N d (t ) t‬‬
‫الشكل (‪)1‬‬ ‫) ‪N (t‬‬

‫د‪ -‬اعتمادا على الشكل (‪ )1‬حدد زمن نصف العمر ‪ t 12‬لنواة ‪. 53 I‬‬
‫‪131‬‬

‫‪1‬‬
‫فقط بحيث كان النشاط‬ ‫‪131‬‬
‫‪53‬‬ ‫‪ .3‬نعتبر أن المنطقة تلوثت إشعاعيا باليود ‪I‬‬
‫‪0‬‬ ‫) ‪t ( Jours‬‬ ‫اإلشعاعي آنذاك ‪. A0  1,76 109 GBq‬‬
‫‪4‬‬
‫حدد التاريخ الذي نعتبر فيه أن هذه المنطقة أصبحت غير ملوثة باليود المشع إذا اعتبرنا أن المنبع غير فعال‬
‫أ‪ّ -‬‬
‫عندما يبلغ نشاطه اإلشعاعي ‪. 1Bq‬‬

‫صفحة ‪ 1‬من ‪10‬‬


‫بكالوريا تجريبي ماي ‪2023‬‬ ‫‪/‬‬ ‫شعبة تقني رياضي‬ ‫‪/‬‬ ‫اختبار في مادة‪ :‬العلوم الفيزيائية‬
‫‪ 137‬مع إصدار إشعاع ‪ ، ZA X‬زمن نصف عمرها يساوي‪. t 1  30,1 ans :‬‬
‫إلى نواة الباريوم ‪56 Ba‬‬ ‫‪ .4‬تتفكك نواة السيزيوم ‪Cs‬‬
‫‪137‬‬
‫‪55‬‬
‫‪2‬‬

‫يصل التلوث النووي لمنطقة مساحتها ‪ ، 10000 Km2‬حيث كان النشاط اإلشعاعي يساوي ‪ 555 KBq‬لكل ‪. 1 m2‬‬
‫مع تحديد نمط اإلشعاع‪.‬‬ ‫‪137‬‬
‫‪55‬‬ ‫أ‪ -‬اكتب معادلة تفكك نواة ‪Cs‬‬
‫لكل ‪. 1 m2‬‬ ‫ب ‪ -‬احسب عدد األنوية ‪Cs‬‬
‫‪137‬‬
‫‪55‬‬

‫ج ‪ -‬احسب كتلة السيزيوم ‪ 137‬بالكيلوغرام ) ‪ ( Kg‬الموجودة في المساحة ‪. 10000 Km2‬‬


‫‪ .5‬إذا علمت أن منظمة األمم المتحدة العلمية المعنية بآثار اإلشعاع الذري )‪ (UNSCEAR‬تشترط أال يتعدى النشاط‬
‫اإلشعاعي الحد األدنى‪ ،‬والذي يساوي ‪. 37 KBq.m2‬‬
‫حدد التاريخ الذي تصبح فيه المنطقة قابلة لالستخدام‪.‬‬
‫ّ‬ ‫‪-‬‬
‫المعطيات‪. 1GBq 109 Bq ، N A  6, 023 1023 mol 1 ، 1an  365 jours :‬‬

‫التمرين الثاني‪ 04( :‬نقاط)‬


‫كثير من التجهيزات االلكترونية مثل التليفزيون‪ ،‬الحاسوب‪ ،‬الهاتف‪ ،‬آلة التصوير‪ ...‬تحتوي في دارتها على مكثفات‪.‬‬
‫يهدف هذا التمرين إلى تحديد سعة المكثفة تجريبيا ثم دراسة تفريغها في ناقل أومي‪.‬‬
‫الشكل (‪)2‬‬
‫لهذا الغرض نقوم بتحقيق التركيب التجريبي الممثل في الشكل (‪ )2‬الذي يحتوي على‪:‬‬
‫‪ -‬مولد للتيار يعطي تيا ار ثابتا شدته ‪. I0  0,5 mA‬‬
‫‪ -‬مكثفة فارغة سعتها ‪. C‬‬
‫‪ -‬ناقل أومي مقاومته ‪ ، R‬أمبير متر‪ ،‬فولط متر‪ ،‬كرونومتر وبادلة ‪. K‬‬

‫في اللحظة ‪ t  0‬وضعنا البادلة في الوضع (‪ ،)1‬وبعد مدة زمنية غيرنا وضع‬
‫البادلة إلى(‪، )2‬بواسطة جهاز فولط متر‪ ،‬كرونومتر وبرمجية إعالم آلي تمكنا من رسم المنحنى البياني ) ‪uc  f (t‬‬
‫الممثل لتغيرات التوتر بين طرفي المكثفة بداللة الزمن ‪) t‬الشكل (‪.()3‬‬
‫البادلة في الوضع (‪:)1‬‬
‫‪ .1‬أتمم الشكل (‪ )2‬مبينا عليه موضع كل من جهاز الفولط متر واألمبير متر والتوتر ‪. uc‬‬
‫‪ .2‬أعط العالقة بين ‪ C ، t ، I 0‬و ‪. uc‬‬
‫‪ .3‬اعتمادا على الشكل (‪ )3‬حدد‪:‬‬
‫أ ‪ -‬قيمة سعة المكثفة ‪. C‬‬
‫ب‪ -‬قيمة التوتر األعظمي ‪ U 0‬وأحسب الطاقة األعظمية المخزنة في المكثفة‪.‬‬
‫البادلة في الوضع (‪:)2‬‬
‫‪ .1‬حدد اللحظة الزمنية ' ‪ t‬التي نغير فيها وضع البادلة إلى الوضع(‪.)2‬‬
‫‪ .2‬بتطبيق قانون جمع التوترات‪ ،‬جد المعادلة التفاضلية للتوتر بين طرفي المكثفة ‪. uc‬‬

‫صفحة ‪ 2‬من ‪10‬‬


‫بكالوريا تجريبي ماي ‪2023‬‬ ‫‪/‬‬ ‫شعبة تقني رياضي‬ ‫‪/‬‬ ‫اختبار في مادة‪ :‬العلوم الفيزيائية‬
‫‪ .3‬تقبل المعادلة التفاضلية حال من الشكل‪.  (t t ') :‬‬
‫‪ ، uc (t )  Ae‬حيث ‪ A‬و ‪ ‬ثابتين يطلب تعيين عبارتيهما بداللة ثوابت‬
‫الدارة الكهربائية المدروسة‪.‬‬
‫‪ .4‬حدد ثابت الزمن ‪ ، ‬ثم استنتج قيمة ‪ . R‬و احسب قيمة الطاقة الضائعة بفعل جول عند اللحظة ‪. t  ‬‬
‫) ‪uc (V‬‬
‫)‪Uc(V‬‬

‫‪2‬‬

‫‪0‬‬ ‫)‪t ( s‬‬


‫‪20‬‬ ‫الشكل (‪)3‬‬
‫التمرين الثالث‪ 05( :‬نقاط)‬
‫غاز الهيدروجين شفاف‪ ،‬عديم اللون والرائحة‪ ،‬ال يذوب في الماء ويحضر مخبريا عن طريق تفاعل بين محلول‬
‫حمضي ومعدن‪.‬‬
‫يهدف هذا التمرين إلى المتابعة الحركية لتفاعل عن طريق الضغط واستنتاج الشروط التجريبية‪.‬‬
‫المعطيات‪:‬‬
‫‪ -‬تجرى القياسات عند درجة ح اررة ثابتة حيث‪ T ( K )   C  273 :‬و اعتبار كل الغازات مثالية‪.‬‬
‫‪ - . M (Zn)  65, 4 g / mol -‬ثابت الغازات المثالية ‪. R  8,31 SI‬‬
‫لدراسة التحول الحادث بين معدن الزنك ‪ Zn s ‬ومحلول مائي لحمض الكبريت ‪ (2 H 3O   SO4 2 ) aq ‬عن طريق قياس الضغط‬
‫نضع في حوجلة حجمها ‪ V  1L‬ومجهزة بسدادة كتلة ‪ m  2,94 g‬من مسحوق الزنك ثم ندخل القط قياس الضغط عبر‬
‫السدادة فيشير إلى القيمة ‪. P0‬‬
‫‪ .1‬في اللحظة ‪ t  0‬وعند درجة ح اررة ‪ ، T‬نصب في الحوجلة بواسطة سحاحة حجما ‪ V0  200mL‬من محلول‬
‫مائي لحمض الكبريت تركيزه بشوارد ‪ H 3O ‬يساوي ‪ .  H 3O    C0‬ينمذج التحول الحادث بالمعادلة‪:‬‬
‫) ‪Zn( s )  2H 3O  ( aq )  Zn 2 ( aq )  H 2( g )  2H 2O(l‬‬
‫أ‪ -‬أنجز جدوال لتقدم التفاعل‪.‬‬
‫‪، P‬‬ ‫ب‪ -‬بين أن الضغط الكلي داخل الحوجلة يعرف بداللة التقدم ‪ x‬بالعبارة‪. x   :‬‬
‫‪RT‬‬
‫‪Vg‬‬
‫حيث ‪ ‬ثابت يطلب تعيين عبارته ومدلوله العلمي‪.‬‬
‫‪ .2‬النتائج التجريبية المسجلة مكنتنا من رسم البيانين ‪ 1‬و‪ 2‬الموضحين في الشكلين (‪ )4‬و(‪ )5‬على التوالي‪:‬‬
‫البيان ‪ 1‬يمثل )‪ P  f ( x‬تغير الضغط الكلي داخل الحوجلة بداللة تقدم التفاعل‪.‬‬
‫البيان ‪ 2‬يمثل ) ‪ n ( H3O )  g (t‬تغير كمية مادة شوارد الهيدرونيوم بداللة الزمن‪.‬‬
‫صفحة ‪ 3‬من ‪10‬‬
‫بكالوريا تجريبي ماي ‪2023‬‬ ‫‪/‬‬ ‫شعبة تقني رياضي‬ ‫‪/‬‬ ‫اختبار في مادة‪ :‬العلوم الفيزيائية‬
‫‪ - 1.2‬اعتمادا على البيان (‪:)1‬‬
‫أ‪ -‬جد قيمة كل من‪ :‬قيمة الضغط عند نهاية التفاعل ‪ ، Pf‬قيمة التقدم األعظمي ‪ xmax‬واستنتج المتفاعل المحد‪.‬‬
‫ب‪ -‬أكتب العبارة البيانية للبيان (‪ )1‬ثم استنتج قيمة كل من‪ :‬الثابت ‪ ‬ودرجة ح اررة الوسط ‪ T‬بـ ‪. C‬‬
‫‪ - 2.2‬انطالقا من جدول التقدم واعتمادا على البيان (‪:)2‬‬
‫أ‪ -‬بين أن‪ n0 ( H3O )  3xmax :‬ثم أحسب قيمته واستنتج قيمة ‪. C0‬‬
‫ب‪ -‬احسب السرعة الحجمية األعظمية الختفاء شوارد الهيدريونيوم ثم استنتج السرعة الحجمية للتفاعل عند نفس اللحظة‪.‬‬
‫‪n0  n f‬‬
‫‪ ، nt ( H 3O  ) ‬جد قيمة ‪. t 1‬‬ ‫ج‪ -‬عرف زمن نصف التفاعل ‪ t 1‬ثم برهن أن‪:‬‬
‫‪2‬‬
‫‪1‬‬
‫‪2‬‬
‫‪2‬‬ ‫‪2‬‬
‫) ‪n ( H 3O  )(m ol‬‬
‫)‪P(105 Pa‬‬
‫‪2‬‬
‫‪1‬‬
‫‪n0‬‬

‫‪1, 013‬‬

‫‪0‬‬ ‫) ‪x(mmol‬‬
‫)‪t (min‬‬ ‫‪15‬‬
‫‪0‬‬ ‫الشكل (‪)4‬‬
‫‪10‬‬ ‫الشكل (‪)5‬‬

‫التمرين التجريبي‪ 06( :‬نقاط)‬

‫اهتم العالم اإليطالي غاليلي بدراسة حركة سقوط أجسام مختلفة‪ ،‬وقد تمت هذه الدراسات بتحرير أجسام من ارتفاعات مختلفة‪.‬‬

‫يهدف هذا التمرين إلى دراسة السقوط الحر لكرة على سطح القمر والسقوط الشاقولي الحقيقي لها على سطح األرض‪.‬‬

‫) ‪v(m / s‬‬ ‫تم تصوير شريط فيديو لحركة السقوط الشاقولي الحر لكرة على سطح القمر‪،‬‬ ‫‪.I‬‬
‫وعند تحليل الشريط حصلنا على مخطط سرعة الكرة )‪ ، v  f (t‬الشكل (‪،)6‬‬
‫بحيث ننسب حركتها لمرجع مرتبط بسطح القمر ونعتبره غاليليا ومرتبط بمحور‬
‫𝟏‬
‫شاقولي ) ‪ ( z ' z‬موجه نحو سطح القمر‪.‬‬

‫‪0‬‬
‫‪ .1‬في أية جهة قذفت الكرة؟ عّلل‪.‬‬
‫)‪t ( s‬‬
‫‪0,5‬‬
‫‪ .2‬بتطبيق القانون الثاني لنيوتن في المرجع السابق‪:‬‬
‫‪ -‬عبر عن تسارع الكرة بداللة تسارع الثقالة ‪ g L‬بجوار سطح القمر‪.‬‬
‫للسرعة ) ‪. v  f (t‬‬
‫‪ .3‬اكتب المعادلة الزمنية ّ‬
‫‪ .4‬احسب قيمة ‪ g L‬وقارنها مع شدة تسارع الثقالة على سطح األرض ‪. g‬‬
‫الشكل (‪)6‬‬

‫صفحة ‪ 4‬من ‪10‬‬


‫بكالوريا تجريبي ماي ‪2023‬‬ ‫‪/‬‬ ‫شعبة تقني رياضي‬ ‫‪/‬‬ ‫اختبار في مادة‪ :‬العلوم الفيزيائية‬

‫‪ .5‬قذفت الكرة من المبدأ ‪ O‬للمحور الذي يبعد عن سطح القمر بالمسافة ‪ z0  0,5m‬عند اللحظة ‪. t  0‬‬
‫أ‪ -‬جد المعادلة الزمنية للحركة ) ‪ z  f (t‬منسوبة للمحور )' ‪. ( zz‬‬
‫ب‪ -‬جد بطريقتين مختلفتين المسافة الكلية التي قطعتها الكرة من لحظة قذفها حتى لحظة وصولها لسطح القمر‪.‬‬

‫الكرة السابقة عبارة عن كرة مطاطية مملوءة بغاز ثاني أكسيد الكربون كتلتها ‪ m‬ونصف قطرها ‪، r  10cm‬‬ ‫‪.II‬‬
‫بإهمال كتلة المطاط أمام كتلة الغاز‪.‬‬
‫نترك هذه الكرة تسقط بدون سرعة ابتدائية شاقوليا من ارتفاع ‪ h‬عن سطح األرض‪ ،‬تخضع الكرة أثناء سقوطها لتأثير‬
‫الهواء الذي ننمذجه في قوة احتكاك مائع شدتها ‪ f  kv2‬وشعاعها معاكس لشعاع السرعة‪ ،‬ودافعة أرخميدس ‪، FA  m0 g‬‬
‫حيث ‪ m0‬هي كتلة الهواء المنزاح من طرف الكرة‪.‬‬
‫ننسب حركة الكرة لمرجع سطحي أرضي نعتبره غاليليا مرتبط بمحور شاقولي موجه نحو األسفل ) ‪. ( z ' z‬‬
‫‪ -1‬تكتسب الكرة بعد مدة زمنية سرعة حدية ‪: vL‬‬
‫‪ -‬بتطبيق القانون الثاني لنيوتن بين أن المعادلة التفاضلية تكتب من الشكل‪ v  vL :‬‬
‫‪dv k 2 k 2‬‬
‫‪dt m‬‬ ‫‪m‬‬
‫‪ -2‬بواسطة تجهيز خاص وبرنامج معلوماتي تمكنا من تحديد سرعة الكرة في لحظات مختلفة وقيمة مشتق السرعة‬
‫بالنسبة للزمن في تلك اللحظات‪ ،‬ثم مثلنا بيانيا ) ‪ ، a  f (vL 2  v 2‬حيث ‪ a‬هو التسارع اللحظي للكرة‪.‬‬
‫) ‪a (m / s 2‬‬ ‫أ‪ -‬احسب كتلة الكرة‪.‬‬
‫ب‪ -‬اعتمادا على بيان الشكل (‪:)7‬‬
‫‪ -‬احسب ثابت االحتكاك ‪. k‬‬
‫‪ -‬احسب التسارع االبتدائي للكرة‪ ،‬ثم استنتج الكتلة الحجمية للهواء ‪ air‬في شروط‬
‫التجربة واحسب السرعة الحدية للكرة‪.‬‬

‫‪1‬‬
‫‪ -3‬نعيد نفس التجربة في نفس الشروط بكرة لها نفس الحجم مملوءة بغاز‬
‫الهيليوم ‪. He‬‬
‫) ‪(vL 2  v 2 )(m2 / s 2‬‬
‫‪ -‬احسب شدة دافعة أرخميدس المؤثرة على الكرة‪.‬‬
‫‪0‬‬
‫‪1‬‬ ‫الشكل (‪)7‬‬
‫‪ -‬مثل القوى المؤثرة على الكرة عند اللحظة ‪ ، t  0‬ثم بعد انطالقها‪.‬‬
‫جد المعادلة التفاضلية لسرعة الكرة ثم احسب السرعة الحدية للكرة‪.‬‬ ‫‪-‬‬
‫المعطيات‪ :‬حجم الكرة‪ ، V   r 3 :‬تسارع الثقالة على سطح األرض ‪g 10 m / s 2‬‬
‫‪4‬‬
‫‪3‬‬
‫في شروط التجربة‪ :‬الكتلة الحجمية للغازين‪. He  0,17 Kg / m3 ، CO  1,87 Kg / m3 :‬‬
‫‪2‬‬

‫انتهى الموضوع األول‬

‫صفحة ‪ 5‬من ‪10‬‬


‫بكالوريا تجريبي ماي ‪2023‬‬ ‫‪/‬‬ ‫الشعبة تقني رياضي‬ ‫‪/‬‬ ‫اختبار في مادة‪ :‬العلوم الفيزيائية‬
‫الموضوع الثاني‬
‫يحتوي الموضوع الثاني على ‪ 05‬صفحات (من الصفحة ‪ 06‬إلى الصفحة ‪)10‬‬
‫التمرين األول‪ 05( :‬نقاط)‬
‫خصيصا ليسقط بعدها على‬
‫ً‬ ‫مصمم‬
‫ّ‬ ‫المدفع البشري هو أداة ُّ‬
‫يتم فيها إخراج الشخص المجازف على شكل "قذيفة "من مدفع‬
‫شبكة أفقية انظر الشكل (‪.)1‬‬
‫يهدف التمرين إلى دراسة حركة مركز عطالة المجازف باستعمال قوانين نيوتن ومبدأ انحفاظ الطاقة‪.‬‬
‫‪ -‬كتلة المجازف ‪ m  70 Kg‬ونأخذ ‪. g  9,8 m  s 2‬‬
‫الشكل (‪)1‬‬
‫تُهمل تأثيرات الهواء على المجازف‪.‬‬ ‫‪-‬‬
‫نعتبر سطح األرض كمستوى مرجعي لقياس االرتفاعات‪.‬‬ ‫‪-‬‬
‫عند اللحظة ‪ُ t  0s‬يغادر المجازف المدفع من الموضع ‪ A‬الذي ارتفاعه‬
‫‪ hA  6m‬عن سطح األرض بسرعة ابتدائية ‪ v A‬ليسقط على الشبكة في موضع‬
‫)𝑱𝑲(𝒄𝑬‬
‫‪ B‬الذي ارتفاعه ‪ ، hB‬تتم الدراسة في المعلم ) ‪ (o , i , j‬المرتبط بمرجع غاليلي‪.‬‬
‫𝟓𝟏‬
‫‪ .1‬حدد المرجع المناسب لدراسة حركة مركز عطالة المجازف‪.‬‬
‫𝟗 ‪𝟏𝟑,‬‬
‫‪ .2‬اشرح العبارة التالية‪" :‬تهمل تأثيرات الهواء على المجازف"‪.‬‬
‫‪ .3‬الدراسة التجريبية مكنت من رسم البيان الممثل لتغيرات‬
‫الشكل (‪)2‬‬ ‫الطاقة الحركية بداللة الزمن ) ‪(. Ec  f (t‬الشكل (‪.))2‬‬
‫‪ .1.3‬جد قيمة السرعة االبتدائية ‪ v A‬لمركز عطالة المجازف‪.‬‬
‫‪ .2.3‬الموضع ‪ S‬هو أقصى ارتفاع يصله المجازف‪.‬‬
‫𝟖 ‪𝟔,‬‬

‫‪ -‬احسب قيمة السرعة ‪ v S‬ثم استنتج زاوية القذف ‪. ‬‬


‫‪ .3.3‬بتطبيق مبدأ انحفاظ الطاقة على الجملة (مجازف ‪ +‬أرض)‪:‬‬
‫𝟐‬
‫‪ -‬أوجد ارتفاع نقطة السقوط ‪. hB‬‬
‫𝟎‬ ‫)𝒔(𝒕‬ ‫‪ .4‬للتأكد من الدراسة الطاقوية نلجأ إلى تطبيق القانون الثاني لنيوتن‪.‬‬
‫‪ .1.4‬بتطبيق القانون الثاني لنيوتن على المجازف جد المعادلتين الزمنيتين ) ‪ x (t‬و ) ‪ y (t‬ثم استنتج معادلة المسار‪.‬‬
‫‪ .2.4‬إذا علمت أن فاصلة نقطة السقوط هي ‪ ، xB  42 m‬تحقق من قيمة ‪ hB‬المحسوبة سابقا‪.‬‬
‫‪ .5‬تأكد أن سلم الرسم على محور األزمنة يعطى بالشكل‪1cm  0,5 s :‬‬

‫التمرين الثاني‪ 04( :‬نقاط)‬


‫يتم تحرير الطاقة النووية من خالل تفاعالت نووية مفتعلة‪ ،‬وقد تم اكتشافها في الثالثينات من القرن الماضي‪ ،‬أما اليوم فهي‬
‫مسببة لألضرار على البيئة واإلنسان‪.‬‬
‫تعد مصد ار موثوقا لتوليد الطاقة الكهربائية والح اررية فرغم كل مزياها إال ّأنها تبقى ّ‬
‫صفحة ‪ 6‬من ‪10‬‬
‫بكالوريا تجريبي ماي ‪2023‬‬ ‫‪/‬‬ ‫الشعبة تقني رياضي‬ ‫‪/‬‬ ‫اختبار في مادة‪ :‬العلوم الفيزيائية‬
‫يهدف هذا التمرين لدراسة مثالين عن استعماالت الطاقة المحررة للتفاعالت النووية المفتعلة‪.‬‬

‫المعطيات‪، 1   931,5 MeV / C 2 ، m ( 01n)  1,00866  ، m ( 11 p)  1,00728  ، 1 MeV 1,6 1013 J :‬‬
‫‪. 1 MW 106 W ، N A  6,02 1023 mol 1 ، El‬‬ ‫‪ H   2, 23 MeV ، E  H   8,57 MeV ، m ‬‬
‫‪2‬‬
‫‪1‬‬ ‫‪l‬‬
‫‪3‬‬
‫‪1‬‬
‫‪A‬‬
‫‪Z‬‬ ‫‪He   4, 00138 ‬‬

‫عام ‪1889‬م من قبل العالمين بيار وماري كوري‪ ،‬وهو نادر الوجود في الطبيعة‬ ‫‪210‬‬
‫‪84‬‬ ‫‪ -I‬تم اكتشاف عنصر البولونيوم ‪Po‬‬

‫للحصول على نيترونات‪ ،‬حيث تصطدم الجسيمات ‪‬‬ ‫‪9‬‬


‫‪4‬‬ ‫جدا‪ ،‬يمزج البولونيوم ‪ 210‬مع البيريليوم ‪Be‬‬
‫نشاطه اإلشعاعي كبير ّ‬
‫المحررة من هذا التفاعل‬
‫ّ‬ ‫‪ ، 235‬الطاقة‬
‫انيوم ‪92U‬‬
‫مع أنوية ‪ 49 Be‬فتنطلق نيترونات‪ .‬هذه النيترونات يتم استعمالها لقذف أنوية اليور‬
‫األخير يتم استغاللها لتزويد الغواصة النووية النتقالها في عمق البحار‪ .‬أحد هذه التفاعالت يمكن كتابته على الشكل التالي‪:‬‬
‫‪U ‬‬
‫‪235‬‬
‫‪92‬‬
‫‪1‬‬
‫‪0‬‬ ‫‪n ‬‬ ‫‪139‬‬
‫‪54‬‬ ‫‪Xe ‬‬ ‫‪94‬‬
‫‪38‬‬ ‫‪Sr  3 01n‬‬

‫) ‪E ( MeV‬‬
‫‪ -1‬يدعى تفاعل االنشطار النووي تفاعل تسلسلي مغذى ذاتيا‪ ،‬اشرح‪.‬‬
‫‪92 11 p  144 01n‬‬
‫طط الحصيلة الطاقوية للتفاعل السابق‪:‬‬
‫‪ -2‬يمّثل الشكل (‪ )3‬مخ ّ‬

‫‪E1‬‬
‫‪. E3‬‬ ‫أ‪ -‬ماذا يمّثل كل مقدار من المقادير ‪ E2،E 1‬و‬
‫‪U  01n‬‬ ‫المحررة من هذا التّفاعل‪.‬‬ ‫طاقة‬
‫ب‪ -‬استنتج قيمة ال ّ‬
‫‪235‬‬
‫‪219835,9‬‬ ‫‪92‬‬ ‫‪E2‬‬ ‫ّ‬

‫‪E3‬‬
‫طاقة الح اررية‬
‫كل ال ّ‬ ‫النووي للغواصة ّ‬
‫يحول ّ‬ ‫أن المفاعل ّ‬ ‫ج‪ -‬إذا فرضنا ّ‬
‫‪139‬‬
‫‪Xe ‬‬ ‫‪94‬‬
‫‪Sr  3 n‬‬
‫‪1‬‬ ‫أن استطاعة المفاعل ‪: P  15 MW‬‬ ‫إلى طاقة كهربائية‪ ،‬و ّ‬
‫‪219656, 69‬‬ ‫‪54‬‬ ‫‪38‬‬ ‫‪0‬‬

‫الشكل (‪)3‬‬ ‫‪. 235‬‬


‫الزمنية التي تستهلك خاللها الغواصة كتلة قدرها ‪ m  27 g‬من ‪92U‬‬
‫المدة ّ‬
‫‪-‬جد باليوم ّ‬

‫أن وقود المفاعالت النووية المستقبلية في تفاعالت االندماج هو خليط مكون من الديتريوم ‪ D ‬‬
‫‪ -II‬يتنبأ علماء ال ّذرة حاليا ّ‬
‫نواته ‪ 12 H‬و التريتيوم ‪ T ‬نواته ‪ 13H‬وفق معادلة التفاعل النووي‪:‬‬
‫‪2‬‬
‫‪1‬‬ ‫‪H  13H ‬‬ ‫‪A‬‬
‫‪Z‬‬ ‫‪He ‬‬ ‫‪1‬‬
‫‪0‬‬‫‪n‬‬
‫‪ -1‬باستعمال قوانين االنحفاظ أوجد قيمة العددين ‪ A‬و ‪. Z‬‬
‫عرف تفاعل االندماج النووي‪.‬‬
‫‪ّ -2‬‬
‫‪ -3‬احسب طاقة الربط للنواة ‪. ZA He‬‬
‫األقل إلى األكثر استقرار‪ .‬عّلل‪.‬‬
‫‪ -4‬رتّب األنوية‪ 1 H ، Z He :‬و ‪ 1 H‬من ّ‬
‫‪2‬‬ ‫‪3‬‬ ‫‪A‬‬

‫المحررة عند اندماج نواتي ‪ 12 H‬و ‪. 13 H‬‬


‫ّ‬ ‫طاقة‬
‫‪ -5‬احسب ب ـ ‪ MeV‬ال ّ‬
‫‪ -6‬احسب الطاقة المحررة لكل نكليون لتفاعلي االنشطار واالندماج النووين السابقين‪.‬‬
‫فأيهما أفضل؟ عّلل‪.‬‬
‫‪ -7‬يتم التفكير حاليا في انتاج الطاقة من تفاعل االندماج بدل االنشطار ّ‬
‫صفحة ‪ 7‬من ‪10‬‬
‫بكالوريا تجريبي ماي ‪2023‬‬ ‫‪/‬‬ ‫الشعبة تقني رياضي‬ ‫‪/‬‬ ‫اختبار في مادة‪ :‬العلوم الفيزيائية‬
‫التمرين الثالث‪ 05( :‬نقاط)‬
‫تمكن الفيزيائي األمريكي جوزيف هنري من التعرف على الظواهر المرتبطة بوجود الوشيعة في دارة كهربائية ّ‬
‫وتمكن من‬ ‫ّ‬
‫اكتشاف ذاتيتها في سنة ‪1932‬م ولتشريفه أعطي اسم هنري )‪ (Henry‬لوحدتها‪.‬‬
‫يهدف هذا التمرين لدراسة خصائص ثنائي القطب ‪ RL‬أثناء غلق وفتح القاطعة‪.‬‬
‫الموضحة في الشكل (‪ )4‬لدراسة استجابة‬
‫ّ‬ ‫في حصة األنشطة التجريبية طلب أستاذ من أفواج التالميذ تركيب الدارة الكهربائية‬
‫ثنائي القطب ‪ RL‬للرتبة توتر أثناء غلق وفتح القاطعة (‪ (K‬وذلك بواسطة راسم اهتزاز مهبطي‪.‬‬

‫الشكل (‪)4‬‬

‫غلق القاطعة‪:‬‬ ‫‪.I‬‬


‫‪ -1‬أثناء متابعة األستاذ لعمل األفواج الحظ على شاشة راسم االهتزاز المهبطي الشكل (‪:)5‬‬
‫أ‪ -‬ماذا يمثل كل منحنى؟‬
‫)𝑽(𝒖‬
‫ثم ّبين ما يجب على أعضاء‬‫عينه ّ‬
‫يؤكد الظاهرة المالحظة‪ّ ،‬‬
‫ب‪ -‬أحد المنحنيين ال ّ‬
‫الفوج القيام به لتصحيح الخلل‪ ،‬كيف يصبح الشكل (‪ )5‬بعدئذ؟ مّثله بشكل كيفي‪.‬‬
‫‪ -2‬بتطبيق قانون جمع التّوترات‪،‬‬
‫‪1‬‬ ‫‪ -‬استخرج المعادلة التفاضلية بداللة التّوتر بين طرفي الوشيعة ‪. uB‬‬
‫𝟒‬
‫الشكل‪uB (t )  A.(r  R.e .t ) :‬‬
‫حال من ّ‬
‫‪ -3‬المعادلة التفاضلية السابقة‪ ،‬تقبل ّ‬
‫الدارة‪.‬‬
‫مميزات ّ‬
‫حيث ‪ A‬و ‪ ‬ثوابت موجبة يطلب تعيين عبارتها بداللة ّ‬
‫𝟎‬ ‫)𝒔𝒎(𝒕‬
‫𝟎𝟐‬

‫‪ -4‬باالعتماد على المنحنيات البيانية الممّثلة في الشكل (‪ ،)5‬استنتج القيم‪:‬‬


‫‪2‬‬
‫‪ E -‬القوة الكهربائية للموّلد‬
‫الداخلية للوشيعة‪.‬‬
‫‪ r -‬المقاومة ّ‬
‫‪ L -‬ذاتية الوشيعة‪.‬‬
‫الشكل (‪)5‬‬

‫فتح القاطعة‪:‬‬ ‫‪.II‬‬


‫تفاجأ أعضاء أحد األفواج بحدوث ش اررة كهربائية لحظة فتح القاطعة‪.‬‬
‫وضح سبب ذلك‪.‬‬‫‪ّ -1‬‬
‫السابقة‪.‬‬
‫الدارة ّ‬
‫موضحا ذلك على ّ‬
‫عمليا؟ ّ‬
‫‪ -2‬كيف يمكن تفادي حدوث ذلك ّ‬
‫صفحة ‪ 8‬من ‪10‬‬
‫بكالوريا تجريبي ماي ‪2023‬‬ ‫‪/‬‬ ‫الشعبة تقني رياضي‬ ‫‪/‬‬ ‫اختبار في مادة‪ :‬العلوم الفيزيائية‬
‫‪ -3‬بواسطة راسم االهتزاز المهبطي وبرمجية إعالم آلي تحصلنا على المنحنى البياني الممّثل في الشكل (‪.)6‬‬
‫‪. i (t )  I 0 .e‬‬
‫شدة التّيار اللحظية بالعبارة التالية‪:‬‬
‫تعطى عبارة ّ‬
‫‪t‬‬
‫𝒊 𝒏𝒍‬ ‫‪‬‬

‫𝟎𝟐‬
‫وشدة التّيار األعظمية ‪. I 0‬‬
‫الزمن ‪ّ ‬‬ ‫كل من ثابت ّ‬ ‫أ‪ -‬باالعتماد على البيان جد قيمة ّ‬
‫𝟎‬ ‫)𝒔𝒎(𝒕‬
‫طاقة المخزنة في الوشيعة لحظة فتح القاطعة‪.‬‬ ‫ب‪ -‬احسب قيمة ال ّ‬
‫𝟐‪−‬‬
‫ج‪ -‬كم تصبح قيمة هذه الطاقة بعد مرور ‪ 80 ms‬من فتح القاطعة؟ عّلل‪.‬‬

‫الشكل (‪)6‬‬

‫التمرين التجريبي‪ 06( :‬نقاط)‬


‫تستعمل بعض األحماض الكربوكسيلية كمواد حافظة لألغذية مثل األجبان‪ ،‬المشروبات والمعّلبات‪ ،...‬كما تستعمل في‬
‫تحضير بعض العطور‪ ،‬مستحضرات التّجميل وبعض األدوية‪.‬‬
‫المرحلة ‪ :01‬دراسة تفاعل محلول حمض البروبانويك مع محلول هيدروكسيد الصوديوم‪.‬‬
‫ررة ‪25 C‬‬
‫المعطيات‪ :‬تمت القياسات عند درجة ح ا‬
‫‪ -‬الجداء الشاردي للماء‪Ke  1014 :‬‬
‫‪ -‬ثابت الحموضة للثنائية ‪pKa  C2 H 5COOH / C2 H 5COO    4,9‬‬

‫بهدف إيجاد التركيز المولي ‪ Ca‬لمحلول مائي ‪  Sa ‬لحمض البروبانويك ) ‪ C2 H 5COOH ( aq‬نعاير حجما ‪ Va  5 mL‬منه بواسطة‬
‫الصوديوم ‪  Na   HO   aq ‬تركيزه المولي ‪. Cb  5,0 102 mol / L‬‬
‫محلول مائي ‪  Sb ‬لهيدروكسيد ّ‬
‫𝑯𝒑‬ ‫يوضح الشكل (‪ )7‬تغيرات قيم ‪ pH‬المزيج التفاعلي بداللة الحجم ‪ Vb‬المضاف‪:‬‬
‫ّ‬

‫𝟐‬

‫𝟎‬ ‫)𝑳𝒎( 𝒃𝑽‬


‫𝟐‬
‫الشكل (‪)7‬‬

‫صفحة ‪ 9‬من ‪10‬‬


‫بكالوريا تجريبي ماي ‪2023‬‬ ‫‪/‬‬ ‫الشعبة تقني رياضي‬ ‫‪/‬‬ ‫اختبار في مادة‪ :‬العلوم الفيزيائية‬
‫طط المعايرة مع تحديد البيانات‪.‬‬
‫‪ -1‬ارسم مخ ّ‬
‫‪ -2‬اكتب معادلة تفاعل المعايرة‪ ،‬ثم أنجز جدول التقدم‪.‬‬
‫‪ -3‬احسب التركيز المولي ‪ Ca‬للمحلول ‪.  Sa ‬‬
‫) ‪10 pH E 14 (Va  VbE‬‬
‫‪  f 1 ‬ثم احسب قيمتها‪ .‬ماذا تستنتج؟‬ ‫أن نسبة التّقدم النهائي تعطى بالعبارة‪:‬‬
‫‪ -4‬أثبت ّ‬
‫‪Cb .VbE‬‬
‫النوع الكيميائي الغالب للثنائية ‪  C2 H 5COOH / C2 H 5COO  ‬عند نقطة التكافؤ‪.‬‬
‫‪ -5‬استنتج ّ‬
‫المرحلة ‪ :02‬تصنيع إيثانوات اإليزوميل المستعملة في صناعة العطور (تفاعل حمض اإليثانويك مع ‪-3‬ميثيل بوتان‪-1-‬ول)‬
‫‪CH3‬‬ ‫المعطيات‪ :‬الصيغة الجزيئية نصف المفصلة للكحول المستعمل موضحة في الشكل (‪:)7‬‬
‫‪H2‬‬ ‫‪ -‬كثافة حمض اإليثانويك (حمض الخل) المستعمل ‪d1 1,05‬‬
‫‪CH‬‬ ‫‪C‬‬
‫‪H3C‬‬ ‫‪C‬‬ ‫‪OH‬‬ ‫‪ -‬كثافة الكحول (‪-3‬ميثيل بوتان‪-1-‬ول) المستعمل ‪d2  0,81‬‬
‫‪H2‬‬ ‫‪‬‬
‫‪ -‬الكتلة الحجمية للماء ‪ eau 1 g / mL‬و ‪. d  l‬‬
‫الشكل (‪)7‬‬ ‫‪eau‬‬
‫‪ -‬الكتلة المولية الجزيئية لحمض اإليثانويك ‪M1  60 g / mol‬‬
‫‪ -‬الكتلة المولية الجزيئية للكحول المستعمل ‪M 2  88 g / mol‬‬
‫المتشكل ‪M E 130 g / mol‬‬
‫ّ‬ ‫‪ -‬الكتلة المولية الجزيئية لألستر‬
‫نمزج حجما ‪ V1  30 mL‬من حمض اإليثانويك ‪ CH3COOH‬مع حجم ‪ V2  20 mL‬من الكحول وقطرات من حمض الكبريت‬
‫لمدة كافية باالرتداد‪.‬‬
‫ونسخن المزيج ّ‬
‫ّ‬ ‫المركز‪ ،‬ونضيف للمزيج قطع من حجر الخفان‪،‬‬
‫ّ‬
‫المركز‪.‬‬
‫ّ‬ ‫‪ -1‬ما هو الهدف من التّسخين باالرتداد ومن إضافة حمض الكبريت‬
‫‪ -2‬احسب كمية المادة االبتدائية لكل من الحمض ‪ n01‬والكحول ‪. n02‬‬
‫ثم أعط التّسمية النظامية لألستر الناتج‪.‬‬
‫‪ -3‬اكتب معادلة التفاعل باستخدام الصيغ نصف المفصلة ّ‬
‫ثم استنتج التّقدم األعظمي ‪. xmax‬‬
‫لتقدم التفاعل‪ّ ،‬‬
‫‪ -4‬أنجز جدوال ّ‬
‫الناتج ‪. d E  0,87‬‬
‫أن كثافة االستر ّ‬
‫الناتج تجريبيا هو ‪ ، VE  27 mL‬استنتج مردود التّفاعل علما ّ‬
‫إن حجم االستر ّ‬
‫‪ّ -5‬‬
‫تمكننا من تحسين مردود التّفاعل‪.‬‬
‫‪ -6‬اذكر طريقة ّ‬

‫بالتوفيق في شهادة البكالوريا‬

‫صفحة ‪ 10‬من ‪10‬‬


‫اإلجابة النموذجية لموضوع اختبار في مادة‪ :‬العلوم الفيزيائية ‪ /‬الشعبة تقني رياضي ‪ /‬بكالوريا تجريبي ماي ‪2023‬‬

‫العالمة‬
‫عناصر اإلجابة (الموضوع األول)‬
‫المجموع‬ ‫مجزأة‬
‫التمرين األول (‪ 05‬نقاط)‬
‫‪0,5‬‬ ‫‪ -1‬أ‪ -‬معادلة التفاعل النووي الحادث‪53 I  Z Xe  1e :‬‬
‫‪. 131‬‬ ‫‪A‬‬ ‫*‬ ‫‪0‬‬

‫بتطبيق قوانين صودي لالنحفاظ نجد‪ A  131 :‬و ‪ Z  54‬فتصبح المعادلة‪54 Xe  1e :‬‬
‫‪I  131‬‬
‫‪131‬‬ ‫*‬ ‫‪0‬‬
‫‪53‬‬

‫‪0,5‬‬ ‫نمط االشعاع ‪( ‬اشعاع كهرومغناطيسي)‬ ‫‪131‬‬


‫‪54‬‬ ‫‪Xe* ‬‬ ‫‪131‬‬
‫‪54‬‬ ‫ب‪ -‬معادلة التحول النووي‪Xe  00 :‬‬
‫‪0,25‬‬ ‫‪ -2‬أ‪ -‬عبارة التناقص اإلشعاعي‪N (t )  N0 .e t :‬‬
‫‪0,25‬‬ ‫ب‪ -‬عبارة األنوية المتفككة‪N d (t )  N 0  N 0 .e  t  N 0 . 1  e  t  :‬‬

‫ج‪ -‬اثبات العالقة‪ et  1 :‬‬


‫) ‪N d (t‬‬
‫‪:‬‬
‫) ‪N (t‬‬
‫‪0,25‬‬
‫‪N d (t ) N 0 . 1  e  1  e t et‬‬
‫‪ t‬‬

‫وهو المطلوب‪.‬‬ ‫ومنه‬ ‫من العالقات السابقة‪  t  t :‬‬


‫) ‪N d (t‬‬ ‫‪t‬‬
‫‪ e 1‬‬ ‫‪‬‬
‫) ‪N (t‬‬ ‫) ‪N (t‬‬ ‫‪N 0 .e t‬‬ ‫‪e‬‬ ‫‪e‬‬

‫د‪ -‬تحديد نصف عمر النواة ‪: t 1‬‬


‫‪2‬‬

‫‪0,5‬‬ ‫) ‪N d (t 1‬‬
‫‪ln 2‬‬
‫‪. t1‬‬

‫‪ .‬باإلسقاط على المنحنى نجد‪t 1  8 Jours :‬‬


‫‪t 1‬‬ ‫‪t1‬‬ ‫‪2‬‬
‫‪2‬‬
‫‪e‬‬ ‫‪2‬‬
‫‪1  e‬‬ ‫‪2‬‬
‫‪1  2  1  1‬‬
‫‪2‬‬ ‫) ‪N (t 1‬‬
‫‪2‬‬

‫‪ -3‬أ‪ -‬التاريخ الذي نعتبر فيه أن هذه المنطقة أصبحت غير ملوثة‪:‬‬
‫‪ln 2‬‬
‫‪t1‬‬ ‫‪‬‬ ‫‪t‬‬

‫اذن‪ ln 0 :‬‬ ‫النشاط اإلشعاعي يعطى بالعالقة‪ A(t )  A0 .et :‬ومنه‬


‫‪A‬‬ ‫) ‪A(t‬‬ ‫‪t1‬‬
‫‪t‬‬ ‫‪2‬‬
‫‪ e  t  e‬‬ ‫‪2‬‬

‫‪0,5‬‬ ‫‪ln 2‬‬ ‫) ‪A(t‬‬ ‫‪A0‬‬


‫‪ 1, 76 109 109  8‬‬
‫‪ t  ln ‬اذن‪t 1 an , 4 mois :‬‬ ‫‪‬‬ ‫بالتعويض‪ 484,88 Jours :‬‬
‫‪‬‬ ‫‪1‬‬ ‫‪ ln 2‬‬
‫ومنه يمكن اعتبار المنطقة غير ملوثة بتاريخ‪ 13 :‬جوان ‪.1961‬‬
‫‪137‬‬
‫‪55‬‬‫‪Cs ‬‬ ‫‪137‬‬
‫‪56‬‬ ‫‪Ba  ZA X‬‬ ‫‪: 137‬‬
‫‪ -4‬أ‪ -‬معادلة تفكك نواة السيزيوم ‪55 Cs‬‬

‫‪0,75‬‬ ‫‪137‬‬
‫‪55‬‬ ‫بتطبيق قوانين صودي لالنحفاظ نجد‪ A  0 :‬و ‪ Z  1‬فتصبح المعادلة‪56 Ba  1e :‬‬
‫‪Cs  137‬‬ ‫‪0‬‬

‫‪ -‬نمط االشعاع هو‪  :‬‬


‫ب‪ -‬حساب عدد األنوية لكل ‪: 1m2‬‬
‫‪0,5‬‬ ‫‪A t1‬‬
‫‪ N  ‬بالتعويض‪:‬‬ ‫‪  ‬ومنه‪:‬‬ ‫لدينا‪ A    N :‬حيث‪:‬‬
‫‪A‬‬ ‫‪2‬‬ ‫‪ln 2‬‬
‫‪‬‬ ‫‪ln 2‬‬ ‫‪t1‬‬
‫‪2‬‬

‫‪555 103  30,1 365  24  3600‬‬


‫‪ N ‬اذن‪ :‬عدد األنوية لكل ‪ 1m2‬هو‪. N  7,6 1014 noyaux :‬‬
‫‪ln 2‬‬
‫ج‪ -‬حساب كتلة السيزيوم ‪ 137‬الموجودة في المساحة ‪: 10000 Km2‬‬
‫نحسب أوال كتلة السيزيوم ‪ 137‬الموجودة في المساحة ‪ : 1 m2‬نعلم أن ‪ M  137 Cs ‬‬
‫‪N‬‬
‫‪m'‬‬
‫‪NA‬‬
‫‪0,5‬‬
‫‪7, 6 1014‬‬
‫‪ m ' ‬ومنه ‪. m ' 1,73 107 g  1,73 1010 Kg‬‬ ‫بالتعويض‪137 :‬‬
‫‪6, 02 1023‬‬
‫نحسب كتلة السيزيوم ‪ 137‬الموجودة في المساحة ‪: 10000 Km2‬‬
‫‪1 m  m '  1, 73 10 Kg‬‬
‫‪10‬‬
‫‪10000 106 1, 73 1010‬‬
‫‪2‬‬

‫‪ m ‬أي‪. m 1, 73 Kg :‬‬ ‫‪ ‬ومنه‬


‫‪10000 10 m  m  Kg ‬‬
‫‪6‬‬ ‫‪2‬‬
‫‪1‬‬

‫‪ -5‬تحديد التاريخ الذي تصبح فيه المنطقة قابلة لالستخدام‪:‬‬


‫‪ 555 103 ‬‬ ‫‪t1‬‬
‫بالتعويض‪:‬‬ ‫لدينا‪ A(t )  A0 .e t :‬ومنه ‪ ln 0‬‬
‫‪30,1‬‬ ‫‪A‬‬
‫‪t‬‬ ‫‪ ln ‬‬ ‫‪3 ‬‬
‫‪‬‬ ‫‪117,‬‬ ‫‪6‬‬ ‫‪ans‬‬ ‫‪t‬‬ ‫‪‬‬ ‫‪2‬‬

‫‪0,5‬‬ ‫‪ln 2‬‬ ‫‪ 37 10 ‬‬ ‫‪ln 2‬‬ ‫) ‪A(t‬‬


‫اذن‪t 117 ans , 7 mois , 6 Jours :‬‬
‫ومنه تصبح المنطقة صالحة لالستخدام بتاريخ‪ 19 :‬سبتمبر ‪.2077‬‬
‫التمرين الثاني (‪ 04‬نقاط)‬
‫البادلة في الوضع (‪:)01‬‬
‫‪ -1‬إتمام الشكل‪:‬‬

‫‪0,5‬‬

‫‪ -2‬العالقة بين ‪ C ، t ، I 0‬و ‪: uc‬‬


‫‪0,25‬‬ ‫‪q I t‬‬
‫‪. uc ‬‬ ‫‪ uc   0‬و بالتالي‪.t....(1) :‬‬ ‫نعلم أن‪ q  I 0  t :‬و ‪ q  C.uc‬ومنه‪:‬‬
‫‪I0‬‬
‫‪C‬‬ ‫‪C‬‬ ‫‪C‬‬

‫‪ -3‬أ‪-‬إيجاد قيمة سعة المكثفة ‪: C‬‬


‫‪0,5‬‬ ‫العبارة البيانية‪ :‬البيان عبارة عن خط مستقيم يمر من المبدأ معادلته من الشكل‪uc  a.t :‬‬
‫حيث ‪ a  0,1‬ومنه نكتب‪. uc  0,1.t...(2) :‬‬
‫‪I 0 0,5 103‬‬
‫‪ C ‬ومنه ‪. C  5 103 F‬‬ ‫بالمطابقة بين )‪ (1‬و )‪ (2‬نجد‪ 0  0,1 :‬وبالتالي نجد‪:‬‬
‫‪I‬‬
‫‪‬‬
‫‪0,1‬‬ ‫‪0,1‬‬ ‫‪C‬‬
‫األعظمي ‪: U 0‬‬ ‫ج‪ -‬قيمة التوتر‬
‫‪0,5‬‬ ‫‪ -‬باإلسقاط على البيان نجد‪. U 0 10V :‬‬
‫حساب الطاقة األعظمية المخزنة في المكثفة‪:‬‬
‫لدينا‪ E0  . C.U 02 :‬بالتعويض‪ E0  .  5 103  .102 :‬ومنه ‪E0  0, 25 J‬‬ ‫‪-‬‬
‫‪1‬‬ ‫‪1‬‬
‫‪2‬‬ ‫‪2‬‬
‫البادلة في الوضع (‪:)2‬‬
‫‪0,25‬‬ ‫‪ -1‬تحديد اللحظة الزمنية ' ‪ t‬التي نغير فيها وضع البادلة إلى الوضع(‪ :)2‬من البيان‪t ' 100 s :‬‬

‫‪ -2‬المعادلة التفاضلية للتوتر بين طرفي المكثفة ‪: uc‬‬


‫‪0,5‬‬ ‫‪dq d  C.uc ‬‬
‫بتطبيق قانون جمع التوترات‪ uc  uR  0 :‬أي‪ uc  R.i  0 :‬علما أن ‪ C. c‬‬
‫‪du‬‬
‫‪i‬‬ ‫‪‬‬
‫‪dt‬‬ ‫‪dt‬‬ ‫‪dt‬‬
‫‪ 1 ‬‬
‫‪ ‬نجد المعادلة التفاضلية المطلوبة‪. uc  0 :‬‬ ‫بالتعويض والضرب في ‪‬‬
‫‪duc‬‬ ‫‪1‬‬
‫‪‬‬
‫‪dt RC‬‬ ‫‪ RC ‬‬
‫‪ -3‬تعيين عبارة الثابتين ‪ A‬و ‪ ‬بداللة ثوابت الدارة الكهربائية المدروسة‪:‬‬
‫‪duc‬‬
‫‪ uc (t )  Ae‬ومنه باالشتقاق‪  . A.e (t t ') ....(**) :‬‬ ‫لدينا‪.  (t t ') ....(*) :‬‬
‫‪dt‬‬
‫‪0,75‬‬
‫بتعويض )*( و )**( في المعادلة التفاضلية نجد‪. A.e  (t t ')  0 :‬‬
‫‪1‬‬
‫‪ . A.e (t t ') ‬‬
‫‪RC‬‬
‫‪ A.e (t t ')   ‬اذن‪:‬‬
‫‪1 ‬‬
‫ومنه ‪  0‬‬
‫‪1‬‬
‫‪‬‬
‫‪RC‬‬ ‫‪‬‬ ‫‪RC ‬‬
‫وعند اللحظة‪ uc (t ')  A.e0 U 0 : t  t ' :‬ومنه ‪. A U0‬‬
‫‪ -4‬تحديد ثابت الزمن ‪: ‬‬
‫عند اللحظة ‪ uc ( )  0,37 10  3,7V : t  ‬باإلسقاط على المنحنى نجد‪.   20 s :‬‬
‫‪0,75‬‬ ‫‪‬‬
‫‪.R ‬‬ ‫‪ R ‬بالتعويض نجد‪ 4 K  :‬‬ ‫‪ -‬استنتاج قيمة ‪ : R‬نعلم أن ‪   R.C‬ومنه‬
‫‪20‬‬
‫‪3‬‬
‫‪5 10‬‬ ‫‪C‬‬
‫‪ -‬حساب قيمة الطاقة الضائعة بفعل جول عند اللحظة ‪Elib  Emax  E    E0  .C.uc ( ) : t  ‬‬
‫‪1‬‬
‫‪2‬‬
‫‪5 103  3, 62‬‬
‫‪ Elib  0, 25 ‬اذن‪Elib  0, 2 J :‬‬ ‫لدينا من البيان‪ uc ( )  3,6V :‬ومنه بالتعويض نجد‪:‬‬
‫‪2‬‬
‫التمرين الثالث‪ 05( :‬نقاط)‬
‫‪ -1‬أ‪ -‬جدول تقدم التفاعل‪:‬‬
‫معادلة التفاعل‬ ‫) ‪Zn( s )  2 H 3O  ( aq )  Zn2aq   H 2 g   2 H 2O(l‬‬

‫حج‬ ‫التقدم ‪ mol ‬‬ ‫كـ ـم ـ ـيـ ــات الم ـ ـ ــادة‬


‫‪0,25‬‬ ‫‪t0‬‬ ‫‪x 0‬‬ ‫‪n0  Zn ‬‬ ‫‪n0  H 3O  ‬‬ ‫‪0‬‬ ‫‪0‬‬ ‫بوفرة‬
‫‪t‬‬ ‫‪x‬‬ ‫‪n0  Zn   x‬‬ ‫‪n0  H 3O    2 x‬‬ ‫‪x‬‬ ‫‪x‬‬ ‫بوفرة‬
‫‪t‬‬ ‫‪f‬‬ ‫‪xm‬‬ ‫‪n0  Zn   xm‬‬ ‫‪n0  H 3O    2 xm‬‬ ‫‪xm‬‬ ‫‪xm‬‬ ‫بوفرة‬
‫‪: P‬‬ ‫ب‪ -‬إثبات أن‪. x   :‬‬
‫‪RT‬‬
‫‪Vg‬‬
‫الضغط الكلي داخل الحوجلة هو‪P  Pg  P0 :‬‬
‫‪0,75‬‬
‫‪nH 2 .R.T‬‬
‫‪ Pg ‬ومن جدول التقدم‪nH 2  x :‬‬ ‫لدينا من قانون الغازات المثالية‪ PH .VH  nH .R.T :‬ومنه‪:‬‬
‫‪Vg‬‬ ‫‪2‬‬ ‫‪2‬‬ ‫‪2‬‬
‫‪. P‬‬ ‫‪ Pg ‬ومنه‪. x  P0 ...(1) :‬‬ ‫بالتعويض نجد‪. x :‬‬
‫‪R.T‬‬ ‫‪R.T‬‬
‫‪Vg‬‬ ‫‪Vg‬‬
‫بالمطابقة بين ‪ 1‬والعالقة المعطاة نجد ‪P0   :‬‬
‫وهي تمثل الضغط الجوي في الحوجلة قبل بداية التفاعل ‪. Patm‬‬
‫‪ -1-2‬اعتمادا على البيان (‪:)1‬‬
‫أ‪ -‬قيمة الضغط عند نهاية التفاعل ‪ Pf  2, 4 1, 013 105 : Pf‬ومنه ‪. Pf  2, 43 105 Pa‬‬
‫قيمة التقدم األعظمي ‪ xm  3 15 : xmax‬ومنه‪. xm  45 mmol  4,5 102 mol :‬‬
‫‪0,75‬‬ ‫استنتاج المتفاعل المحد‪:‬‬
‫اذا كان ‪ Zn‬متفاعل محد يكون‪ n0  Zn   xm  0 :‬يعني أن ‪n0  Zn   xm‬‬

‫‪ n0  Zn  ‬اذن‪n0  Zn   xm  0, 0045 mol :‬‬ ‫ولدينا‪ 0, 00449 mol :‬‬


‫‪m 2,94‬‬
‫‪‬‬
‫‪M 65, 4‬‬
‫ومنه المتفاعل المحد هو ‪. Zn‬‬
‫ب‪ -‬العبارة البيانية للبيان (‪:)1‬‬
‫البيان عبارة عن خط مستقيم يمر من المبدأ معادلته من الشكل‪ P  a.x  b :‬حيث ‪ a‬ميل البيان‪.‬‬

‫‪ a ‬و ‪b 1,013 105 Pa‬‬


‫حسابه‪ 2, 4312  1, 013 105  31,51105 Pa / mol :‬‬
‫‪0, 045  0‬‬
‫ومنه معادلة البيان‪P  31,51105.x 1,013 105....(2) :‬‬
‫‪0,75‬‬
‫استنتاج قيمة كل من‪ :‬الثابت ‪ ‬ودرجة ح اررة الوسط ‪: T‬‬
‫بالمطابقة بين العبارة النظرية ‪ 1‬والعبارة البيانية ‪  2 ‬نجد‪ 1,013105 Pa :‬‬
‫‪31,51105  1  0, 2  103‬‬ ‫‪a Vg‬‬
‫‪ T ‬بالتعويض نجد‪:‬‬ ‫اذن‬ ‫و ‪a‬‬
‫‪R.T‬‬
‫‪T‬‬
‫‪8,31‬‬ ‫‪R‬‬ ‫‪Vg‬‬
‫ومنه ‪T  303,34K  30,34 C‬‬
‫‪ - 2.2‬انطالقا من جدول التقدم واعتمادا على البيان (‪:)2‬‬
‫أ‪ -‬اثبات أن‪: n0 ( H3O )  3xmax :‬‬
‫‪n0  n f‬‬
‫‪xm ‬‬ ‫لدينا من جدول التقدم‪ n f  H 3O    n0  H 3O    2 xm :‬ومنه‪....(*) :‬‬
‫‪2‬‬
‫‪1 ‬‬ ‫‪n  1 2 .n0‬‬
‫ومن البيان نالحظ أن ‪ . n f  0‬بالتعويض في )*( نجد‪:‬‬
‫‪n‬‬
‫‪xm    n0  0   ‬‬
‫‪0,75‬‬ ‫‪2 ‬‬ ‫‪3 2‬‬ ‫‪3‬‬ ‫‪3‬‬
‫ومنه‪ n0  H 3O    3 xm :‬وهو المطلوب‪.‬‬
‫حساب قيمته‪ n0  H 3O    3.xm  3 0, 045 :‬ومنه ‪. n0  H 3O   1,35 102 mol‬‬
‫‪n0  H 3O   1,35 102‬‬
‫‪C0 ‬‬ ‫‪‬‬ ‫استنتاج قيمة ‪ n0  H 3O     H 3O    V  C0 V : C0‬ومنه‬
‫‪V‬‬ ‫‪0, 2‬‬
‫اذن‪C0  0,675 mol / L :‬‬
‫ب‪ -‬حساب السرعة الحجمية األعظمية الختفاء شوارد الهيدريونيوم أي عند ‪: t  0 min‬‬
‫‪1  dn  H 3O  ‬‬
‫‪‬‬
‫‪1  0  0,135 ‬‬
‫‪vVol‬‬
‫‪H O ‬‬
‫‪(t  0)  ‬‬ ‫‪.‬‬ ‫‪‬‬ ‫ومنه‪:‬‬ ‫‪v‬‬
‫‪ 3 ‬‬
‫(‬‫‪t‬‬ ‫‪‬‬ ‫)‪0‬‬ ‫‪‬‬ ‫‪‬‬ ‫‪.‬‬ ‫‪‬‬ ‫لدينا‪ :‬‬
‫‪V ‬‬ ‫‪‬‬
‫‪‬‬ ‫‪Vol H O ‬‬
‫‪0, 2  15  0 ‬‬ ‫‪dt‬‬
‫‪‬‬ ‫‪t  0‬‬
‫‪3‬‬

‫‪0,75‬‬ ‫‪vVol‬‬ ‫اذن‪(t  0)  4,5 102 mol / L.min :‬‬


‫‪H O ‬‬
‫‪3‬‬
‫‪‬‬

‫استنتاج السرعة الحجمية للتفاعل عند نفس اللحظة‪:‬‬


‫) ‪(4,5 102‬‬
‫لدينا‪ vVol  t  0   .vVol  H O  (t  0) :‬ومنه‬
‫‪1‬‬
‫‪vVo  t  0  ‬‬
‫‪2‬‬ ‫‪2‬‬ ‫‪3‬‬

‫اذن‪. vVo  t  0   2, 25 102 mol / L.min :‬‬


‫ج‪ -‬تعريف زمن نصف التفاعل ‪ : t 1‬هو الزمن الالزم لبلوغ التقدم التفاعل نصف قيمته األعظمية‪.‬‬
‫‪2‬‬

‫‪n0  n f‬‬
‫‪: nt ( H 3O  ) ‬‬ ‫‪ -‬اثبات العالقة‬
‫‪1‬‬
‫‪2‬‬ ‫‪2‬‬
‫‪n0  n f‬‬
‫‪1‬‬ ‫‪xm ‬‬ ‫لدينا من جدول التقدم‪ n f  H 3O    n0  2 xm :‬ومنه‪:‬‬
‫‪2‬‬
‫‪n  n f 2n0  n0  n f‬‬
‫‪nt 1  H 3O    n0  2  m  n0  xm  n0  0‬‬ ‫لما ‪: t  t 1‬‬
‫‪x‬‬
‫‪‬‬
‫‪2‬‬ ‫‪2‬‬ ‫‪2‬‬ ‫‪2‬‬ ‫‪2‬‬

‫‪n n‬‬
‫ومنه ‪. nt 1  H 3O    0 f‬وهو المطلوب‪.‬‬
‫‪2‬‬ ‫‪2‬‬
‫‪0,135  0, 045‬‬
‫‪ nt 1  H 3O   ‬باإلسقاط على البيان نجد‪. t 1  7 min :‬‬ ‫قيمته‪ 0, 09 mol :‬‬
‫‪2‬‬ ‫‪2‬‬ ‫‪2‬‬
‫التمرين التجريبي‪ 06( :‬نقاط)‬
‫‪0,5‬‬ ‫‪ -1 -I‬بما أن عند اللحظة ‪ t  0s‬إشارة سرعة الكرة سالبة (البيان)‪ ،‬إذن قذفت الكرة نحو األعلى‪ .‬ألن‬
‫)' ‪ ( zz‬موجه نحو األسفل‪.‬‬
‫‪ -2‬التعبير عن تسارع الكرة بداللة تسارع الثقالة ‪ g L‬بجوار سطح القمر‪:‬‬
‫‪0,5‬‬ ‫ومنه ‪P  m. a‬‬ ‫‪F‬‬ ‫‪ext‬‬ ‫بتطبيق القانون الثاني لنيوتن في المرجع السابق‪ m. a :‬‬
‫باإلسقاط على المحور )' ‪ ( zz‬نجد‪ P  m. a :‬ومنه‪m .g L  m .a :‬‬
‫ومنه‪. a  g L :‬‬
‫‪0,25‬‬ ‫‪ -3‬كتابة المعادلة الزمنية للسرعة ) ‪: v  f (t‬‬
‫‪ v(t )  g L .t  v0‬ومن الشروط االبتدائية ‪  v0  2 m / s : t  0 ‬ومنه ‪v(t )  g L .t  2‬‬
‫‪ -4‬حساب قيمة ‪: g L‬‬
‫البيان )‪ v  f (t‬عبارة عن خط مستقيم ال يمر من المبدأ معادلته من الشكل ‪. v   .t  ‬‬
‫‪0,25‬‬ ‫‪0   2 ‬‬
‫‪  ‬و ‪    2‬ومنه تصبح المعادلة‪v 1, 6. t  2 :‬‬ ‫حيث ‪ ‬هو الميل‪ ،‬حسابه‪1, 6 :‬‬
‫‪2,5  0,5  0‬‬
‫بالمطابقة مع العبارة النظرية نجد‪g L 1,6 :‬‬
‫‪0,25‬‬ ‫ومنه‪g  g L :‬‬
‫‪g 10‬‬
‫‪‬‬ ‫المقارنة بين ‪ g‬و ‪ 6,3 : g L‬‬
‫‪g L 1, 6‬‬
‫‪-5‬أ‪ -‬المعادلة الزمنية للحركة ) ‪: z  f (t‬‬
‫‪0,25‬‬ ‫‪ z (t )  g L .t 2  2.t  z0‬ومن الشروط االبتدائية ‪  z0  0 m : t  0 ‬ومنه ‪z (t )  g L .t 2  2.t‬‬
‫‪1‬‬ ‫‪1‬‬
‫‪2‬‬ ‫‪2‬‬
‫بالتعويض نجد‪z (t )  0,8.t 2  2.t :‬‬
‫ب‪ -‬المسافة الكلية التي قطعتها الكرة من لحظة قذفها حتى لحظة وصولها لسطح القمر‪:‬‬
‫الطريقة ‪ :01‬المسافة تمثل المساحة المحصورة بين محور الزمن والمستقيم ) ‪: v  f (t‬‬
‫‪2   2,5  2  2, 4   3  0,5‬‬
‫‪ d ‬ومنه ‪. d  3, 05 m‬‬ ‫‪‬‬ ‫‪ 1, 25  1,8‬‬
‫‪2‬‬ ‫‪2‬‬
‫الطريقة ‪ :02‬الكرة تصعد نحو األعلى ثم تتوقف لتعود من جديد إلى سطح القمر‪.‬‬
‫‪0,5‬‬
‫حساب مسافة الصعود‪ :‬من البيان زمن الصعود هو‪( t 1, 25 s :‬تنعدم السرعة)‬
‫بالتعويض في المعادلة الزمنية للحركة‪Z  0,8. 1, 25   2. 1, 25  :‬‬
‫‪2‬‬
‫‪Z‬‬ ‫‪Z‬‬
‫‪O‬‬ ‫اذن ‪( Z  1, 25 m‬عكس جهة المحور)‪.‬‬
‫‪z0‬‬ ‫ومنه ‪ d  2.Z  z0 :‬اذن ‪ d  2 1, 25  0,5‬فنجد ‪d  3 m :‬‬
‫‪z‬‬ ‫سطح القمر‬

‫‪ -1 -II‬إثبات أن المعادلة التفاضلية تكتب من الشكل‪ v  vL :‬‬


‫‪dv k 2 k 2‬‬
‫‪dt m‬‬ ‫‪m‬‬
‫بتطبيق القانون الثاني لنيوتن على الجملة (كرة) في مرجع سطحي أرضي نعتبره غاليليا مرتبط بمحور‬
‫ومنه‪P  FA  f  m. a :‬‬ ‫‪F‬‬ ‫‪ext‬‬ ‫شاقولي موجه نحو األسفل )' ‪ m. a : ( zz‬‬
‫باإلسقاط وفق المحور ‪  zz '‬نجد‪P  FA  f  m.a :‬‬
‫‪0,5‬‬ ‫‪ m ‬‬
‫‪ mg  m0 .g  kv 2  m.‬اذن نكتب‪ .v  g 1  0  ...(1) :‬‬ ‫ومنه‬
‫‪dv k 2‬‬ ‫‪dv‬‬
‫‪dt m‬‬ ‫‪‬‬ ‫‪m‬‬ ‫‪dt‬‬
‫‪m  m ‬‬ ‫‪ m ‬‬
‫ومنه ‪ .v 2L  g 1  0 ‬اذن‪v 2L  .g 1  0  :‬‬ ‫لما ‪ v  vL‬يكون ‪ 0‬‬
‫‪k‬‬ ‫‪dv‬‬
‫‪k ‬‬ ‫‪m‬‬ ‫‪m‬‬ ‫‪‬‬ ‫‪m‬‬ ‫‪dt‬‬
‫وهو المطلوب‪.‬‬ ‫بالتعويض في ‪ 1‬نجد‪ .v  .v L :‬‬
‫‪dv k 2 k 2‬‬
‫‪dt m‬‬ ‫‪m‬‬
‫‪0,25‬‬ ‫‪ -2‬أ‪ -‬حساب كتلة الكرة‪ m  CO2 .V 1,87   3,14   0,1 :‬ومنه ‪m  7,83 103 Kg‬‬
‫‪4‬‬ ‫‪3‬‬

‫‪3‬‬
‫ب‪ -‬حساب ثابت االحتكاك ‪: k‬‬
‫العبارة البيانية‪ :‬البيان عبارة عن خط مستقيم يمر من المبدأ معادلته من الشكل ‪. a   .  vL2  v 2 ‬‬
‫حيث ‪ ‬هو الميل‪ ،‬حسابه‪.  1 :‬‬
‫‪1,25‬‬ ‫‪.‬‬
‫‪dv‬‬
‫العبارة النظرية‪ :‬لدينا من ‪ a  .  v 2L  v 2  : 1‬‬
‫‪k‬‬
‫‪dt‬‬ ‫‪m‬‬
‫بالمطابقة بين العبارتين نجد‪ 1 :‬ومنه ‪. k  7,83 103 Kg / m‬‬
‫‪k‬‬
‫‪m‬‬
‫‪-‬حساب التسارع االبتدائي للكرة‪ :‬أكبر قيمة للتسارع هي التسارع االبتدائي‪.‬‬
‫‪ a0 ‬من البيان‪. a0  4 m / s 2 :‬‬
‫‪7,83 103‬‬
‫‪7,83 10‬‬ ‫‪3‬‬
‫‪ a0 ‬ومنه ‪  4 ‬‬
‫‪dv‬‬ ‫‪k‬‬
‫‪dt t 0 m‬‬
‫‪‬‬ ‫‪‬‬
‫حيث‪ . v 2L  v 2 0  .v 2L :‬‬
‫‪k‬‬
‫‪m‬‬
‫‪‬‬ ‫‪ .V‬‬ ‫‪‬‬ ‫‪‬‬ ‫‪‬‬ ‫‪‬‬
‫‪a0  g 1  air‬‬ ‫‪  g 1  air‬‬
‫‪ CO‬‬ ‫‪ -‬استنتاج الكتلة الحجمية للهواء ‪ air‬في شروط التجربة‪ :‬‬
‫‪ CO . V‬‬ ‫‪‬‬
‫‪‬‬ ‫‪2‬‬ ‫‪‬‬ ‫‪‬‬ ‫‪2‬‬ ‫‪‬‬
‫‪ a ‬‬ ‫‪‬‬
‫ومنه ‪ air 1  0‬اذن ‪  air  CO2 . 1  0  1,87  0, 6‬وبالتالي‪. air  1,12 Kg / m3 :‬‬
‫‪a‬‬
‫‪‬‬ ‫‪g‬‬ ‫‪CO2‬‬ ‫‪g‬‬

‫‪ vL ‬اذن‪vL  2 m / s :‬‬ ‫‪ -‬حساب السرعة الحدية للكرة‪ :‬وجدنا سابقا‪ a0  .v 2L :‬ومنه ‪ 4‬‬
‫‪a0 .m‬‬ ‫‪k‬‬
‫‪k‬‬ ‫‪m‬‬
‫‪-3‬‬
‫‪0,25‬‬ ‫حساب شدة دافعة أرخميدس المؤثرة على الكرة‪ FA  air Vg  g :‬اذن‪FA 1,12  4,18 102 :‬‬ ‫‪-‬‬
‫ومنه ‪. FA  4,68 102 N‬‬
‫‪ -‬تمثيل القوى المؤثرة على الكرة عند اللحظة ‪ ، t  0‬ثم بعد انطالقها‪:‬‬
‫‪ P '  m '.g  He .V .g 0,17  4,18 103 10  7,1103 N‬بما أن ' ‪ : FA  P‬الكرة تصعد نحو األعلى‪.‬‬

‫‪0,5‬‬

‫‪ -‬ايجاد المعادلة التفاضلية لسرعة الكرة‪:‬‬


‫تطبيق القانون الثاني لنيوتن على الجملة (كرة) في مرجع سطحي أرضي نعتبره غاليليا مرتبط بمحور‬
‫ومنه‪P  FA  f  m. a :‬‬ ‫‪F‬‬ ‫‪ext‬‬ ‫شاقولي موجه نحو األسفل )' ‪ m. a : ( zz‬‬
‫باإلسقاط وفق المحور ‪  zz '‬نجد‪ P  FA  f  m '.a :‬حيث ' ‪ m‬هي كتلة الكرة وهي مملوءة بالهيليوم‪0,5 .‬‬
‫‪ m ‬‬
‫‪ m ' g  m0 .g  kv 2  m '.‬اذن المعادلة التفاضلية هي‪ .v  g 1  0  :‬‬ ‫ومنه‬
‫‪dv k 2‬‬ ‫‪dv‬‬
‫' ‪dt m‬‬ ‫‪ m' ‬‬ ‫‪dt‬‬

‫حساب السرعة الحدية للكرة‪:‬‬ ‫‪-‬‬


‫‪0,25‬‬ ‫‪‬‬
‫‪k 2‬‬ ‫‪ m ‬‬
‫أي‪.vL  g 1  0  :‬‬
‫‪dv‬‬
‫لما ‪ v  vL‬تكون‪ 0 :‬‬
‫'‪m‬‬ ‫‪ m' ‬‬ ‫‪dt‬‬
‫‪m '.g  m0 ‬‬ ‫‪7,1103  1,12 ‬‬
‫ومنه‪vL  2, 23 m / s :‬‬ ‫‪vL  ‬‬
‫‪2‬‬
‫‪1 ‬‬ ‫‪ ‬‬ ‫‪ 1‬‬ ‫اذن ‪ 5‬‬
‫‪k  m' ‬‬ ‫‪7,83 103  0,17 ‬‬
‫اإلجابة النموذجية لموضوع اختبار في مادة‪ :‬العلوم الفيزيائية ‪ /‬الشعبة تقني رياضي ‪ /‬بكالوريا تجريبي ماي ‪2023‬‬

‫العالمة‬
‫عناصر اإلجابة (الموضوع الثاني)‬
‫المجموع‬ ‫مجزأة‬
‫‪0,25‬‬ ‫التمرين األول (‪ 05‬نقاط)‬
‫‪ -1‬المرجع المناسب لدراسة حركة مركز عطالة المجازف‪ :‬المرجع السطحي األرضي الذي نعتبره غاليليا‪.‬‬
‫‪0,25‬‬ ‫‪ -2‬شرح العبارة‪" :‬تهمل تأثيرات الهواء على المجازف"‪ :‬المجازف يخضع لتأثير ثقله فقط (سقوط حر)‪.‬‬
‫‪ -1-3‬ايجاد قيمة السرعة االبتدائية ‪ v A‬لمركز عطالة المجازف‪:‬‬
‫‪0,5‬‬ ‫‪ v A ‬و من البيان نجد‪EcA 13,9 KJ :‬‬ ‫أن‪:‬‬
‫أي ّ‬ ‫‪ EcA  . m . vA2‬ومنه‪:‬‬
‫‪2 Ec A‬‬ ‫‪2 EcA‬‬ ‫‪1‬‬
‫‪v A2 ‬‬
‫‪m‬‬ ‫‪m‬‬ ‫‪2‬‬
‫‪2 13,9 103‬‬
‫وبالتالي‪vA 19,93 m / s :‬‬ ‫‪vA ‬‬ ‫إذن‪:‬‬
‫‪70‬‬
‫‪ -2-3‬حساب قيمة السرعة ‪: v S‬‬
‫‪0,25‬‬ ‫‪ vS ‬و من البيان نجد‪EcA  6,8 KJ :‬‬ ‫أن‪:‬‬
‫أي ّ‬ ‫‪ EcS  . m . vS2‬ومنه‪:‬‬
‫‪2 EcS‬‬ ‫‪2 EcS‬‬ ‫‪1‬‬
‫‪vS2 ‬‬
‫‪m‬‬ ‫‪m‬‬ ‫‪2‬‬
‫‪2  6,8 103‬‬
‫وبالتالي‪vA 13,94 m / s :‬‬ ‫‪vA ‬‬ ‫إذن‪:‬‬
‫‪70‬‬
‫‪ -‬استنتاج قيمة زاوية القذف ‪: ‬‬
‫‪ cos   Ax‬حيث‪ vAx  vS :‬إلن عند الموضع ‪ S‬يكون ‪ v y  0‬و السرعة وفق المحور ‪  Ox ‬ثابتة‬
‫‪v‬‬
‫‪vA‬‬
‫‪0,25‬‬
‫‪ vS ‬‬ ‫‪1  13,94 ‬‬
‫أن ‪  45,62‬‬
‫‪   cos ‬أي ّ‬
‫‪1‬‬
‫‪  cos ‬‬ ‫إذن‪ :‬‬
‫‪ vA ‬‬ ‫‪ 19,93 ‬‬
‫‪ -3-3‬ايجاد ارتفاع نقطة السقوط ‪: hB‬‬
‫بتطبيق مبدأ انحفاظ الطاقة على الجملة (مجازف ‪ +‬أرض) بين الموضعين ‪ A‬و ‪: B‬‬
‫‪0,5‬‬ ‫‪EcA  EcB‬‬
‫‪hB  hA ‬‬ ‫‪ EppB  EcB  EppA  EcA‬ومنه‪ m.g.hB  m.g.hA  EcA  EcB :‬وعليه‪:‬‬
‫‪m.g‬‬

‫‪ hB  6 ‬ومنه‪. hB  4,39 m :‬‬


‫إذن‪13,9 15 103 :‬‬
‫‪70  9,8‬‬
‫‪ -1-4‬ايجاد المعادلتين الزمنيتين ) ‪ x (t‬و ) ‪: y (t‬‬
‫‪ -‬المرجع‪ :‬سطحي أرضي نعتبره عطاليا‪.‬‬ ‫‪ -‬الجملة‪ :‬مجازف‬
‫‪0,5‬‬ ‫‪vAx  vA .cos ‬‬
‫‪‬‬
‫‪vA  ‬‬ ‫و‬
‫‪x  0‬‬
‫عند اللحظة ‪: t  0‬‬
‫‪‬‬
‫‪vAy  vA .sin ‬‬
‫‪‬‬ ‫‪ y  hA‬‬
‫ومنه‪P  m. a :‬‬ ‫‪F‬‬ ‫‪ext‬‬ ‫بتطبيق القانون الثاني لنيوتن نجد‪ m. a :‬‬

‫‪0,5‬‬ ‫باإلسقاط وفق المحور ‪  Ox ‬نجد‪ m . ax  0 :‬ومنه‪. ax  0 :‬‬


‫باإلسقاط وفق المحور ‪  Oy ‬نجد‪ m . ay   g :‬ومنه‪. a y   g :‬‬
‫‪0,5‬‬ ‫‪vx  t   v A .cos ‬‬
‫‪‬‬ ‫بمكاملة عبارتي ‪ ax‬و ‪ a y‬نجد‪:‬‬
‫‪v y  t    g .t  v A .sin ‬‬
‫‪ x  t   vA .cos  .t ................ 1‬‬
‫‪0,5‬‬ ‫‪‬‬
‫‪‬‬ ‫‪1 2‬‬ ‫بمكاملة عبارتي ‪ vx  t ‬و ‪ v y  t ‬نجد‪:‬‬
‫‪ y  t    g.t  vA .sin  .t  hA . ..........  2 ‬‬
‫‪‬‬ ‫‪2‬‬
‫‪ -‬استنتاج معادلة المسار‪:‬‬
‫‪0,5‬‬ ‫‪ t ‬و بالتعويض في العالقة ‪  2 ‬نجد معادلة المسار‪:‬‬
‫‪x‬‬
‫من العالقة ‪: 1‬‬
‫‪vA .cos ‬‬
‫‪g‬‬
‫‪y‬‬ ‫‪. x 2   tan   x  hA‬‬
‫‪2.v .cos 2 ‬‬
‫‪2‬‬
‫‪A‬‬

‫‪ -2-4‬التحقق من قيمة ‪: hB‬‬


‫‪0,25‬‬ ‫لدينا معادلة المسار‪ ، y   0,025. x2 1,022 x  6 :‬المجازف في الموضع ‪B  xB  42 , yB ‬‬
‫بالتعويض نجد‪ yB   0, 025.  42   1, 022  42   6 :‬ومنه‪yB  4,82 m :‬‬
‫‪2‬‬

‫‪ -5‬التأكد أن سلم الرسم على محور األزمنة يعطى بالشكل‪1cm  0,5 s :‬‬
‫‪0,25‬‬ ‫‪tB ‬‬
‫‪xB‬‬
‫‪‬‬
‫‪42‬‬
‫زمن وصول المجازف إلى الموضع ‪ B‬هو‪ 3 s :‬‬
‫)‪vA .cos  19,93  cos (45, 62‬‬
‫أن‪ ، 6 cm  3 s :‬إذن‪1cm  0,5 s :‬‬
‫وهي ممثلة بـ ‪ 6 cm‬يعني ّ‬
‫التمرين الثاني (‪ 04‬نقاط)‬
‫‪0,25‬‬ ‫‪ -1-I‬يطلق عليه تفاعل تسلسلي مغذى ذاتيا ألن النيترونات الناتجة عن االنشطار األول تُحدث‬
‫انشطارات أخرى‪.‬‬
‫‪ -2‬أ‪ -‬المقادير ‪ E2،E 1‬و ‪ E3‬هي‪:‬‬
‫‪ : E1‬تمثل طاقة الربط لنواة اليورانيوم ‪U‬‬
‫‪235‬‬
‫‪92‬‬

‫‪0,75‬‬ ‫‪139‬‬
‫‪54‬‬ ‫‪Xe،Sr38‬‬
‫‪94‬‬
‫‪ : E2‬تمثل مجموع طاقتي الربط للنواتين‬
‫‪ : E3‬تمثل الطاقة المحررة من تفاعل االنشطار‪.‬‬
‫‪0,25‬‬ ‫المحررة من هذا التّفاعل‪ :‬من المخطط ‪Elib  E1  E2  219835,9  219656,69‬‬
‫ّ‬ ‫طاقة‬
‫ب‪ -‬استنتاج قيمة ال ّ‬
‫ومنه‪Elib 179, 21 MeV :‬‬
‫‪: 235‬‬
‫انيوم ‪92U‬‬
‫الزمنية التي تستهلك خاللها الغواصة كتلة قدرها ‪ m  27 g‬من اليور‬
‫المدة ّ‬
‫ج‪ -‬إيجاد ّ‬
‫‪0,25‬‬ ‫ة من انشطار كتلة ‪ m  27 g‬من ‪92U‬‬
‫‪: 235‬‬ ‫نحسب أوال الطاقة المحرر‬
‫‪‬‬ ‫‪‬‬
‫‪  ‬‬ ‫‪‬‬
‫‪ Elib‬اذن نجد‪ 6, 02 1023  179, 21 MeV :‬‬
‫‪27‬‬ ‫‪m‬‬
‫‪Elib‬‬ ‫‪  N  Elib  ‬‬ ‫‪‬‬ ‫‪N‬‬ ‫‪‬‬
‫‪A  Elib‬‬
‫‪ 235‬‬ ‫‪‬‬ ‫‪ M  235U ‬‬ ‫‪‬‬
‫‪‬‬ ‫‪‬‬
‫‪ 1, 24 1025 MeV 1,984 1012 J‬‬
‫‪Elib‬‬ ‫ومنه‪:‬‬
‫‪0,25‬‬ ‫‪1,984 1012‬‬
‫‪ t ‬بالتعويض نجد‪1,32 105 s  1,51 J :‬‬
‫‪E‬‬
‫أن‪ P  :‬ومنه‪:‬‬
‫‪E‬‬
‫نعلم‬
‫ّ ‪t‬‬
‫‪t‬‬
‫‪15 10‬‬ ‫‪6‬‬
‫‪P‬‬
‫‪0,25‬‬ ‫‪2  3  A  1‬‬
‫‪‬‬ ‫‪ ‬‬
‫‪A  4‬‬
‫‪ -1-II‬إيجاد قيمة العددين ‪ A‬و ‪: Z‬‬
‫‪1  1  Z‬‬ ‫‪Z  2‬‬
‫‪0.25‬‬ ‫‪ -2‬تفاعل االندماج هو تفاعل نووي مفتعل يتم فيه التحام نواتين خفيفتين لتنتج نواة أثقل و أكثر استق ار ار‬
‫مع تحرير طاقة‪.‬‬
‫‪0,25‬‬ ‫‪El  42 He    A.m p  ( A  Z ).mn  m( 42 He   C 2‬‬ ‫‪ -3‬حساب طاقة الربط للنواة ‪: ZA He‬‬
‫‪. El‬‬ ‫‪‬‬ ‫‪4‬‬
‫‪2‬‬ ‫أي ‪ El  42 He    2 1, 00728  2 1, 00866  4, 00138   931,5‬ومنه ‪He   28, 41 MeV‬‬

‫‪ -4‬ترتيب األنوية‪ :‬نحسب طاقة الربط لكل نوية‪:‬‬


‫‪0,5‬‬ ‫‪El  42 He ‬‬ ‫‪El  13 H  8,57‬‬ ‫‪El  12 H  2, 23‬‬
‫‪ 2,857 MeV ،‬‬
‫‪28, 41‬‬
‫‪‬‬ ‫‪ 7,103 MeV ،‬‬ ‫‪‬‬ ‫‪‬‬ ‫‪1,115 MeV‬‬
‫‪A‬‬ ‫‪4‬‬ ‫‪A‬‬ ‫‪3‬‬ ‫‪A‬‬ ‫‪2‬‬
‫ومنه النواة األقل استقرار هي ‪ 12 H‬تليها نواة ‪ 13 H‬ثم نواة ‪ 42 He‬األكثر استق اررا‪.‬‬
‫‪0,25‬‬ ‫‪ -5‬حساب الطاقة المحررة عن اندماج نواتي ‪ 12 H‬و ‪Elib  El (12 H )  El (13 H )  El ( 24 He) : 13 H‬‬

‫ومنه ‪Elib  17, 6 MeV‬‬ ‫‪Elib  2, 23  8,57  28, 41‬‬


‫‪ -6‬حساب الطاقة المحررة لكل نكليون لتفاعلي االنشطار واالندماج‪:‬‬
‫‪0,5‬‬ ‫‪Elib 179, 21‬‬
‫‪‬‬ ‫االنشطار‪ 0, 76 MeV / nuc :‬‬ ‫‪،‬‬
‫‪Elib 17, 6‬‬
‫‪‬‬ ‫االندماج‪ 3, 25 MeV / nuc :‬‬
‫‪ A 235‬‬ ‫‪ A 23‬‬
‫‪0,25‬‬ ‫ألن تفاعل االندماج يحرر طاقة أكبر بمشاركة عدد‬
‫‪ -7‬تفاعل االندماج أفضل من تفاعل االنشطار‪ّ :‬‬
‫نويات أقل‪ ،‬له مخلفات إشعاعية أقل ووفرة في الوقود النووي‪.‬‬
‫التمرين الثالث‪ 05( :‬نقاط)‬
‫‪0,5‬‬ ‫‪-I‬غلق القاطعة‪:‬‬
‫‪ -1‬أ‪ -‬المنحنيات‪ :)1( :‬التوتر بين طرفي الوشيعة ‪ :)2( ، ub‬التوتر بين طرفي الناقل األومي ‪. uR‬‬
‫عند غلق القاطعة‪ ،‬ينشأ التيار مع مرور الزمن مما يؤدي إلى تزايد التوتر بين طرفي الناقل األومي‬
‫‪ ، uR  R.i‬وحسب قانون جمع التوترات‪ ،‬التوتر بين طرفي الوشيعة ‪ ub‬يتناقص‪.‬‬
‫)𝑽(𝒖‬
‫ب‪ -‬المشكلة الموجودة في التجربة‪:‬‬
‫‪2‬‬ ‫حسب ربط جهاز راسم االهتزاز المهبطي‪ ،‬المدخل ‪ y2‬مربوط‬
‫‪0,5‬‬ ‫من جهة الكمون المنخفض للمولد‪ ،‬وعليه التوتر بين طرفي الناقل‬
‫األومي يظهر على الشاشة معكوس ومن أجل تصحيحه يجب‬
‫𝟒‬
‫الضغط على زر ‪  INV ‬للمدخل ‪. y2‬‬
‫‪1‬‬

‫فيصبح البيان‪:‬‬
‫)𝒔𝒎(𝒕‬
‫𝟎‬ ‫𝟎𝟐‬

‫‪-2‬المعادلة التفاضلية بداللة التّوتر بين طرفي الوشيعة ‪: uB‬‬


‫‪0,5‬‬ ‫‪di‬‬
‫‪L.‬‬ ‫بتطبيق قانون جمع التوترات نجد ‪ ub  uR  E :‬ومنه ‪  R  r  .i  E.... *‬‬
‫‪dt‬‬
‫‪E  ub‬‬
‫‪ i ‬باالشتقاق نجد ‪،   b‬‬ ‫من جهة أخرى لدينا ‪ ub  R.i  E‬ومنه‬
‫‪di E du‬‬
‫‪dt R dt‬‬ ‫‪R‬‬
‫‪ 1 du ‬‬ ‫‪E 1 ‬‬
‫بتعويض ‪ i‬و ‪ di‬في ‪ *‬نجد‪L.  . b    R  r  .   .ub   E :‬‬
‫‪ R dt ‬‬ ‫‪R R ‬‬ ‫‪dt‬‬
‫‪dub  R  r ‬‬ ‫نضرب طرفي المعادلة في ‪ R  r  .E  R .E :  R‬‬
‫‪dt‬‬
‫‪‬‬
‫‪L‬‬
‫‪.ub ‬‬
‫‪L‬‬ ‫‪L‬‬
‫‪‬‬ ‫‪L‬‬ ‫‪‬‬
‫‪dub  R  r ‬‬
‫ومنه نجد المعادلة التفاضلية ‪:‬‬
‫‪r‬‬
‫‪‬‬ ‫‪.ub  .E.... **‬‬
‫‪dt‬‬ ‫‪L‬‬ ‫‪L‬‬
‫‪ -3‬تعيين عبارتي ‪ A‬و ‪: ‬‬
‫لدينا‪ uB (t )  A.(r  R.e .t ) :‬باالشتقاق نجد‪ A.R. .e .t :‬‬
‫‪duB‬‬
‫‪dt‬‬
‫‪0,5‬‬ ‫بتعويض عبارتي ‪ u‬و ‪ duB‬في ** نجد‪ R  r  .( A.r  A.R.e .t )  r .E :‬‬
‫‪A.R. .e .t ‬‬ ‫‪ ‬‬ ‫‪b‬‬
‫‪L‬‬ ‫‪L‬‬ ‫‪dt‬‬
‫‪‬‬ ‫‪R  r  R  r‬‬
‫بعد عملتي النشر والتبسيط نجد‪. A.r  .E :‬‬
‫‪r‬‬
‫‪A.R..e .t   ‬‬ ‫‪‬‬
‫‪‬‬ ‫‪L ‬‬ ‫‪L‬‬ ‫‪L‬‬
‫‪Rr‬‬
‫‪.A‬‬ ‫‪  ‬و‬ ‫ومنه‪:‬‬
‫‪E‬‬
‫‪Rr‬‬ ‫‪L‬‬
‫‪ -4‬استنتاج قيم‪ r ، E :‬و ‪: L‬‬
‫‪E 12V‬‬ ‫قيمة ‪: E‬‬
‫‪0,75‬‬ ‫‪52 1, 6‬‬ ‫‪u    r .I 0‬‬ ‫‪u     r.I 0 1, 6 V‬‬
‫‪ r ‬ومنه‪r  8  :‬‬ ‫‪ b‬نجد‪:‬‬ ‫‪  b‬ومنه‬ ‫قيمة ‪: r‬‬
‫‪1, 6‬‬
‫‪‬‬ ‫‪‬‬
‫‪10, 4‬‬ ‫‪uR    R.I 0 10, 4‬‬ ‫‪uR     R.I 0 10, 4V‬‬

‫‪  ‬ومنه ‪L   R  r  ‬‬ ‫قيمة ‪ : L‬من البيان نجد‪  10 ms 10 103 s :‬ومن جهة أخرى‪:‬‬
‫‪L‬‬
‫‪Rr‬‬
‫‪. L  0, 6 H‬‬ ‫بالتعويض نجد‪ L   52  8 10 103 :‬ومنه‪:‬‬
‫‪-II‬فتح القاطعة‪:‬‬
‫‪0,25‬‬ ‫‪ -1‬التوضيح‪ :‬حدوث ش اررة كهربائية‪.‬‬
‫‪ -2‬تحديد حل المشكلة تجريبيا‪ :‬يجب توصيل صمام ثنائي‪.‬‬

‫‪0,5‬‬

‫‪-3‬أ‪ -‬إيجاد قيم كل من ثابت الزمن ‪ ‬وشدة التيار األعظمية ‪: I 0‬‬


‫‪-‬العبارة البيانية‪ :‬البيان عبارة عن خط مستقيم ال يمر من المبدأ معادلته من الشكل‪ln i  a . t  b :‬‬
‫‪0,75‬‬ ‫حيث ‪ a‬هو الميل‪ .‬نجد‪ a   0,1 :‬و ‪ b  1, 6‬ومنه‪ln i   0,1. t 1, 6.... 1 :‬‬

‫‪‬‬
‫‪ln i  ln I 0 . e‬‬
‫‪t‬‬
‫‪‬‬
‫‪‬‬ ‫‪-‬العبارة النظرية‪ :‬لدينا‪ i  t   I 0 . e  :‬ومنه‪  .t  ln I 0 ....  2  :‬‬
‫‪1‬‬
‫‪‬‬
‫‪t‬‬

‫‪ 1‬‬
‫‪   0,1‬‬ ‫‪ 10 ms‬‬
‫‪ ‬‬ ‫‪‬‬ ‫‪‬‬ ‫‪1,6‬‬
‫بالمطابقة بين ‪ 1‬و ‪  2 ‬نجد‪:‬‬
‫‪ln I 0   1, 6‬‬ ‫‪ I 0  e  0, 2 A‬‬

‫المخزنة في الوشيعة‪:‬‬
‫ّ‬ ‫ب‪ -‬حساب قيمة الطاقة‬
‫‪0,25‬‬ ‫نعلم أن عند فتح القاطعة‪ i  I 0 :‬ومنه‪ Em  . L. I 02  0,5  0, 6  0, 22 :‬اذن‪. Em  0, 012 J :‬‬
‫‪1‬‬
‫‪0‬‬ ‫‪0‬‬
‫‪2‬‬
‫ج‪ -‬حساب قيمة الطاقة عند ‪: t  80 ms‬‬
‫لما ‪ ln i   9, 6 : t  80 ms‬ومنه ‪ i  e 9,6  6,77 105 A‬و منه‪Em t  0,5  0, 6   6, 77 105  :‬‬
‫‪2‬‬

‫‪0,5‬‬ ‫إذن‪Em t 1,37 109 J :‬‬


‫‪ -‬تخزن الوشيعة طاقة كهرومغناطيسية عند مرور التيار الكهربائي و عند انقطاعه تستهلك تللك الطاقة‬
‫في الناقل األومي وفي مقاومتها الداخلية ( فعل جول)‪.‬‬
‫التمرين التجريبي‪ 06( :‬نقاط)‬
‫المرحلة ‪:01‬‬
‫‪ -1‬مخطط المعايرة‪:‬‬
‫‪0,25‬‬

‫‪0,25‬‬ ‫‪ -2‬معادلة تفاعل المعايرة‪C2 H 5COOH ( aq )  HO  ( aq )  C2 H 5COO ( aq )  H 2O(l ) :‬‬


‫‪ -‬جدول التقدم‪:‬‬
‫معادلة التفاعل‬ ‫) ‪C2 H 5COOH ( aq )  HO  ( aq )  C2 H 5COO ( aq )  H 2O(l‬‬
‫حج‬ ‫التقدم ‪ mol ‬‬ ‫كـ ـم ـ ـيـ ــات الم ـ ـ ــادة‬
‫‪0,5‬‬ ‫‪t0‬‬ ‫‪x 0‬‬ ‫‪Ca .Va‬‬ ‫‪Cb .Vb‬‬ ‫‪0‬‬ ‫بوفرة‬
‫‪t‬‬ ‫‪x‬‬ ‫‪Ca .Va  x‬‬ ‫‪Cb .Vb  x‬‬ ‫‪x‬‬ ‫بوفرة‬
‫‪t‬‬ ‫‪f‬‬ ‫‪xE‬‬ ‫‪Ca .Va  xE‬‬ ‫‪Cb .VbE  xE‬‬ ‫‪xE‬‬ ‫بوفرة‬
‫‪ -3‬حساب التركيز المولي ‪ Ca‬للمحلول ‪:  Sa ‬‬
‫‪ -‬عند التكافؤ يكون‪ Ca .Va  Cb .VbE :‬ومنه ‪.... *‬‬
‫‪Cb .VbE‬‬
‫‪Ca ‬‬
‫‪Va‬‬
‫‪0,5‬‬
‫ومن البيان في الشكل(‪ )7‬وباإلسقاط نجد احداثيات نقطة التكافؤ‪E VbE  6 mL, pH E  6,8 :‬‬
‫‪5, 0 102  6 103‬‬
‫‪ Ca ‬ومنه ‪. Ca  6 102 mol / L‬‬ ‫بالتعويض في ‪ *‬نجد‪:‬‬
‫‪5 103‬‬
‫‪xf‬‬ ‫) ‪10 pH E 14 (Va  VbE‬‬
‫أن‪ E .... ** :‬‬ ‫نعلم‬ ‫‪:‬‬ ‫‪ -4‬اثبات العالقة‬
‫‪x‬‬
‫‪f ‬‬ ‫ّ‬ ‫‪‬‬ ‫‪f‬‬ ‫‪‬‬ ‫‪1‬‬ ‫‪‬‬
‫‪xmax‬‬ ‫‪xmax‬‬ ‫‪Cb .VbE‬‬
‫‪0,75‬‬
‫‪n ( HO  ) Cb .VbE  xE‬‬
‫‪ HO    f‬‬ ‫‪‬‬ ‫من جدول التقدم لدينا‪:‬‬
‫‪f‬‬ ‫‪Va  VbE‬‬ ‫‪Va  VbE‬‬
‫‪1014‬‬
‫ومن الجداء الشاردي للماء‪ Ke   HO  f .  H 3O  f :‬اذن‬
‫‪Ke‬‬
‫‪ HO   ‬‬ ‫‪‬‬ ‫‪ pH E‬‬
‫‪f‬‬ ‫‪ H 3O ‬‬
‫‪‬‬
‫‪10‬‬
‫‪E‬‬

‫‪1014‬‬ ‫‪Cb .VbE  xE‬‬ ‫‪1014‬‬


‫‪ a bE   b bE E ‬‬ ‫وبالتالي‬ ‫أي أن‪:‬‬
‫ّ ‪10 pH E‬‬
‫‪.‬‬ ‫‪V‬‬ ‫‪‬‬ ‫‪V‬‬ ‫‪‬‬ ‫‪C‬‬ ‫‪.‬‬
‫‪V‬‬ ‫‪‬‬ ‫‪x‬‬ ‫‪‬‬
‫‪10 pH E‬‬ ‫‪Va  VbE‬‬
‫نجد‪xE  Cb .VbE 10 pH E 14. Va  VbE  ... 1 :‬‬
‫أن‪xmax  CbVbE ....  2  :‬‬
‫عند التكافؤ المزيج في شروط ستوكيومترية يعني ّ‬
‫‪C .V 10 pH E 14. Va  VbE ‬‬
‫بتعويض ‪ 1‬و ‪  2 ‬في ‪ **‬نجد‪:‬‬
‫‪x‬‬
‫‪ f  E  b bE‬‬
‫‪xmax‬‬ ‫‪CbVbE‬‬
‫‪10 pH E 14. Va  VbE ‬‬
‫‪  f 1 ‬وهو المطلوب‪.‬‬ ‫ومنه‬
‫‪0,25‬‬
‫‪CbVbE‬‬
‫‪108,614.  5  6 ‬‬
‫‪  f 1 ‬ومنه تفاعل المعايرة تام‪.‬‬ ‫‪ -‬حساب قيمتها‪ 0,999  1 :‬‬
‫‪5, 0 102  6‬‬
‫‪ -5‬النوع الكيميائي الغالب عند التكافؤ‪:‬‬
‫‪C2 H 5COO  ‬‬ ‫‪C2 H 5COO  ‬‬
‫‪E‬‬
‫‪ 10‬‬ ‫‪pHE  pKa‬‬
‫‪ pH E  pKa  log‬ومنه‬ ‫‪E‬‬
‫لدينا‪:‬‬
‫‪0,5‬‬ ‫‪C2 H 5COOH E‬‬ ‫‪C2 H 5COOH E‬‬
‫‪C2 H 5COO  ‬‬
‫‪C2 H 5COO   C2 H 5COOH E‬‬
‫‪‬‬
‫أي‬ ‫‪E‬‬
‫بالتعويض نجد‪ 108,6 4,9  5011,87 :‬‬
‫‪E‬‬
‫‪ 2 5‬‬
‫‪C‬‬ ‫‪H‬‬ ‫‪COOH‬‬ ‫‪E‬‬
‫ومنه عند التكافؤ تكون الصفة األساسية هي الغالبة‪.‬‬
‫المرحلة ‪:02‬‬
‫‪0,25‬‬ ‫‪ -1‬الهدف من التسخين باالرتداد‪ :‬تسريع التفاعل والحفاظ على كميات مادة المتفاعالت والنواتج‪.‬‬
‫والهدف من إضافة قطرات من حمض الكبريت المركز‪ :‬تسريع التفاعل‪.‬‬
‫‪ -2‬حساب ‪ n01‬و ‪: n02‬‬
‫‪0,5‬‬ ‫‪m1 1.V1 d1.eau .V1 1, 05 1 30‬‬
‫‪ n01 ‬ومنه ‪n01  0,525 mol‬‬ ‫‪‬‬ ‫‪‬‬ ‫‪‬‬ ‫كمية المادة االبتدائية للحمض‪:‬‬
‫‪M1‬‬ ‫‪M1‬‬ ‫‪M1‬‬ ‫‪60‬‬
‫‪ .V d . .V 0,811 20‬‬
‫‪ n02  2  2 2  2 eau 2 ‬ومنه ‪n02  0,184 mol‬‬ ‫كمية المادة االبتدائية للكحول‪:‬‬
‫‪m‬‬
‫‪M2‬‬ ‫‪M2‬‬ ‫‪M2‬‬ ‫‪88‬‬
‫‪ -3‬معادلة التفاعل باستخدام الصيغ نصف المفصلة‪:‬‬
‫‪OH‬‬ ‫‪CH3‬‬ ‫‪O‬‬ ‫‪CH3‬‬

‫‪+‬‬ ‫‪H2‬‬ ‫‪H2‬‬


‫‪+‬‬
‫‪0,5‬‬
‫‪H3C‬‬ ‫‪C‬‬ ‫‪CH‬‬ ‫‪C‬‬
‫‪C‬‬ ‫‪CH‬‬ ‫‪H2O‬‬
‫‪C‬‬
‫‪H3C‬‬ ‫‪C‬‬ ‫‪OH‬‬
‫‪H2‬‬ ‫‪H3C‬‬ ‫‪O‬‬ ‫‪C‬‬ ‫‪CH3‬‬
‫‪O‬‬ ‫‪H2‬‬

‫‪0,25‬‬ ‫تسمية االستر الناتج‪ :‬ايثانوات ‪-3‬ميثيل البوتيل‬


‫‪ -4‬جدول التقدم للتفاعل‪:‬‬
‫معادلة التفاعل‬ ‫) ‪CH 3COOH ( aq )  C5 H11OH ( aq )  CH 3CO2C5 H11( aq )  H 2O(l‬‬
‫‪0,5‬‬ ‫حج‬ ‫التقدم ‪ mol ‬‬ ‫كـ ـم ـ ـيـ ــات الم ـ ـ ــادة‬
‫‪t0‬‬ ‫‪x 0‬‬ ‫‪n01‬‬ ‫‪n02‬‬ ‫‪0‬‬ ‫‪0‬‬
‫‪t‬‬ ‫‪x‬‬ ‫‪n01  x‬‬ ‫‪n02  x‬‬ ‫‪x‬‬ ‫‪x‬‬
‫‪t‬‬ ‫‪f‬‬ ‫‪x‬‬ ‫‪f‬‬ ‫‪n01  x f‬‬ ‫‪n02  x f‬‬ ‫‪xf‬‬ ‫‪xf‬‬
‫‪0,25‬‬ ‫فإن ‪. xmax  0,184 mol :‬‬
‫أن ‪ّ n02  n01‬‬
‫‪ -‬إيجاد التقدم األعظمي ‪ : xmax‬بما ّ‬

‫‪ -5‬استنتاج قيمة مردود التفاعل‪:‬‬


‫‪0,5‬‬ ‫‪ r % ‬ومنه ‪r %  98, 2%‬‬
‫‪xf‬‬
‫‪‬‬
‫‪nE‬‬ ‫‪d . .V‬‬
‫‪ E eau E ‬‬
‫‪0,87 1 27‬‬
‫لدينا‪100 :‬‬
‫‪xmax‬‬ ‫‪xmax‬‬ ‫‪M E .xmax‬‬ ‫‪130  0,184‬‬
‫‪0,25‬‬ ‫‪ -6‬طريقة لتحسين مردود تفاعل األسترة‪:‬‬
‫‪ -‬عملية التقطير المج أز (نزع االستر أثناء تشكله)‪.‬‬
‫‪ -‬جهاز دين ستارك (نزع الماء أثناء تشكله)‪.‬‬
‫‪ -‬استعمال كلور األسيل‪.‬‬
‫اٌغّ‪ٛٙ‬س‪٠‬خ اٌغضائش‪٠‬خ اٌذ‪ّ٠‬مشاغ‪١‬خ اٌشؼج‪١‬خ‬
‫اٌّذسعخ اٌؼٍ‪١‬ب ٌألعبرزح – ‪ٚ‬سڤـٍخ‬ ‫‪ٚ‬صاسح اٌزؼٍ‪ ُ١‬اٌؼبٌ‪ٚ ٟ‬اٌجؾش اٌؼٍّ‪ٟ‬‬
‫د‪ٚ‬سح ‪ :‬أفش‪2023 ً٠‬‬ ‫امتحبن انبكبنىرٌب انتجرٌبً‬
‫إػذاد األعزبر ‪ِ :‬ذ‪ٚ‬س ع‪١‬ف اٌذ‪ٓ٠‬‬ ‫اٌشؼجخ ‪ :‬س‪٠‬بظ‪١‬بد ‪ ،‬رمٕ‪ ٟ‬س‪٠‬بظ‪ٟ‬‬
‫اٌّذح ‪ 04 :‬عبػخ ‪ 30 ٚ‬دل‪١‬مخ‬ ‫اخزجبس ف‪ِ ٟ‬بدح ‪ :‬انعـهىو انفٍسٌبئٍت‬
‫عهى انمترشح ان ٌختبر أحذ انمىضىعٍه اَتٍٍه ‪:‬‬
‫انمىضىع األول‬
‫ٌحتىي انمىضىع األول عهى ‪ 04‬صفحبث (مه انصفحت ‪ 1‬مه ‪ 8‬إنى انصفحت ‪ 4‬مه ‪)8‬‬
‫انتمرٌه األول ‪ 04( :‬وقبط)‬
‫خالي ِ‪ٛ‬عُ لطف رّ‪ٛ‬س دقهت وىر ‪ٚ‬اٌٍّمجخ ثـ سٍذة انتمىر ِٓ ‪ٚ‬اؽبد ثغىشح ‪ ،‬أساد أؽذ اٌزالِ‪١‬ز اعزؼّبي ِب دسعٗ ف‪ٟ‬‬
‫فمبَ ثزص‪٠ٛ‬ش ؽشوخ ِشوض ػطبٌخ ؽجخ اٌزّش ‪ ٟ٘ٚ‬رغمػ عم‪ٛ‬غب شبل‪١ٌٛ‬ب ِٓ‬ ‫اٌّ‪١‬ىبٔ‪١‬ه ٌزؾذ‪٠‬ذ ل‪ّ١‬خ ِؼبًِ االؽزىبن‬
‫إٌخٍخ ‪ ،‬صُ لبَ ثّؼبٌغخ شش‪٠‬ػ اٌف‪١‬ذ‪ ٛ٠‬ثجشِغ‪١‬خ اػالَ آٌ‪ ٟ‬فزؾصً ػٍ‪ ٝ‬اٌج‪١‬بْ اٌّ‪ٛ‬ظؼ ف‪ ٟ‬اٌشىً (‪)1‬‬

‫𝒔 𝒎 𝑮𝑽‬ ‫انشكم (‪)1‬‬

‫𝐬 𝒕‬

‫‪ -1‬صف غج‪١‬ؼخ ؽشوخ ؽجخ اٌزّش ِٓ خالي إٌّؾٕ‪ ٝ‬اٌج‪١‬بٔ‪ٟ‬‬


‫‪ِ ، 𝛱 ٚ‬برا رغزٕظ‬ ‫‪ -2‬اؽغت إٌغجخ ث‪ٓ١‬‬
‫‪ِ -3‬ضً اٌم‪ ٜٛ‬اٌّؤصشح ػٍ‪ ٝ‬ؽجخ اٌزّش‬
‫رىزت ِٓ اٌشىً ‪:‬‬ ‫‪ -4‬ثزطج‪١‬ك اٌمبٔ‪ ْٛ‬اٌضبٔ‪ٛ١ٌٕ ٟ‬رٓ أصجذ أْ اٌّؼبدٌخ اٌزفبظٍ‪١‬خ ٌغشػخ ِشوض ػطبٌخ ؽجخ اٌزّش‬

‫صبثذ ‪٠‬طٍت رؾذ‪٠‬ذ ػجبسرٗ‬ ‫‪ ،‬ؽ‪١‬ش‬

‫‪ ،‬صُ اعزٕزظ ل‪ّ١‬ز‪ٙ‬ب ث‪١‬بٔ‪١‬ب‬ ‫‪ -5‬ا‪ٚ‬عذ ػجبسح اٌغشػخ اٌؾذ‪٠‬خ‬


‫‪ -6‬اعزٕزظ ل‪ّ١‬خ اٌضِٓ اٌّّ‪١‬ض ٌٍؾشوخ ‪ τ‬ث‪١‬بٔ‪١‬ب‬
‫‪ٚ‬‬ ‫‪،‬‬ ‫ثذالٌخ ‪:‬‬ ‫‪ -7‬أ‪ -‬ا‪ٚ‬عذ ػجبسح اٌضبثذ‬
‫صُ اؽغت ل‪ّ١‬زٗ اٌؼذد‪٠‬خ‬ ‫ة ‪ -‬ثبعزؼّبي اٌزؾٍ‪ ً١‬اٌجؼذ‪ ٞ‬ؽذد ‪ٚ‬ؽذح‬
‫صُ رؤوذ ِٓ ل‪ّ١‬زٗ ث‪١‬بٔ‪١‬ب‬ ‫‪ -8‬عذ ػجبسح اٌزغبسع االثزذائ‪ٟ‬‬

‫رّـش‬ ‫𝜌‬ ‫انمعطٍبث ‪:‬‬

‫صفؾخ ‪8 ِٓ 1‬‬
‫اختببر انبكبنىرٌب انتجرٌبً فً مبدة ‪ :‬انعهىو انفٍسٌبئٍت ‪ //‬انشعبت ‪ :‬رٌبضٍبث ‪ ،‬تقىً رٌبضً ‪ //‬دورة ‪ :‬أفرٌم ‪2023‬‬
‫انتمرٌه انثبوً ‪ 04( :‬وقبط)‬
‫ٔؾمك اٌزشو‪١‬ت اٌزغش‪٠‬ج‪ ٟ‬اٌّج‪ ٓ١‬ف‪ ٟ‬اٌشىً )‪ٚ (2‬اٌّى‪ٔٛ‬خ ِٓ ‪:‬‬
‫‪ٌِٛ -‬ذ ِضبٌ‪ٌٍ ٟ‬ز‪١‬بس اٌى‪ٙ‬شثبئ‪ ٟ‬ل‪ّ١‬زٗ صبثزخ‬
‫‪ٔ -‬بلً أ‪ِ ِٟٚ‬مب‪ِٚ‬زٗ ‪.‬‬
‫‪ِ -‬ىضفخ غ‪١‬ش ِشؾ‪ٔٛ‬خ عؼز‪ٙ‬ب ‪.‬‬
‫انشكم (‪)2‬‬ ‫‪ -‬ثبدٌخ و‪ٙ‬شثبئ‪١‬خ ‪ ،K‬ف‪ٌٛ‬ػ ِزش‪ ،‬أِج‪١‬ش ِزش ‪.‬‬
‫ٔعغ اٌجبدٌخ ف‪ ٟ‬اٌ‪ٛ‬ظغ )‪ (1‬ف‪١‬ش‪١‬ش األِج‪١‬ش ِزش إٌ‪ٝ‬‬ ‫ػٕذ اٌٍؾظخ‬ ‫‪-I‬‬
‫‪ ،‬ث‪ّٕ١‬ب ‪٠‬زغ‪١‬ش ِؤشش اٌف‪ٌٛ‬ػ ِزش رذس‪٠‬غ‪١‬ب خالي اٌضِٓ‪ٚ .‬إٌزبئظ ِذ‪ٔٚ‬خ ف‪ ٟ‬اٌغذ‪ٚ‬ي اٌزبٌ‪:ٟ‬‬ ‫ل‪ّ١‬خ صبثزخ‬
‫)‪t(s‬‬ ‫‪0‬‬ ‫‪50‬‬ ‫‪100‬‬ ‫‪150‬‬ ‫‪200‬‬ ‫‪250‬‬
‫‪0‬‬ ‫‪2‬‬ ‫‪4‬‬ ‫‪6‬‬ ‫‪8‬‬ ‫‪10‬‬

‫‪.‬‬ ‫‪ -1‬اػزّبدا ػٍ‪ ٝ‬عٍُ سعُ ِٕبعت‪ ،‬اسعُ ػٍ‪ٚ ٝ‬سلخ ٍِ‪ّ١‬زش‪٠‬خ إٌّؾٕ‪ ٝ‬اٌج‪١‬بٔ‪:ٟ‬‬
‫‪ -2‬ث‪ ٓ١‬أْ ػجبسح اٌز‪ٛ‬رش اٌى‪ٙ‬شثبئ‪ ٟ‬ث‪ ٓ١‬غشف‪ ٟ‬اٌّىضفخ رىزت ثبٌشىً‪:‬‬

‫‪ -3‬اؽغت عؼخ اٌّىضفخ‬


‫‪ -4‬ؽذد وً ِٓ اٌٍج‪ٛ‬ط اٌّ‪ٛ‬عت ‪ٚ‬اٌٍج‪ٛ‬ط اٌغبٌت ٌٍّىضفخ‪.‬‬
‫ؽ‪ٕٙ١‬ب ٔؤسعؼ اٌجبدٌخ ‪ K‬إٌ‪ ٝ‬اٌ‪ٛ‬ظغ )‪(2‬‬ ‫‪ -II‬ثؼذ شؾٓ اٌّىضفخ وٍ‪١‬ب ‪٠‬ش‪١‬ش اٌف‪ٌٛ‬ػ ِزش إٌ‪ ٝ‬اٌم‪ّ١‬خ‬
‫‪:‬‬ ‫‪ٔٚ‬ؼزجشٖ ِجذأ عذ‪٠‬ذ ٌألصِٕخ‬
‫‪ِ -1‬ب٘‪ ٟ‬اٌظب٘شح اٌز‪ ٟ‬رؾذس ٌٍّىضفخ ؟ فغش٘ب ِغ‪ٙ‬ش‪٠‬ب‪.‬‬
‫‪ -2‬ث‪ٛ‬اعطخ ثشٔبِظ ِٕبعت رّىٕب ِٓ سعُ إٌّؾٕ‪ ٝ‬اٌج‪١‬بٔ‪ٌ ٟ‬زغ‪١‬شاد اٌطبلخ اٌّخضٔخ ف‪ ٟ‬اٌّىضفخ ثذالٌخ اٌضِٓ‬
‫وّب ٘‪ِٛ ٛ‬ظؼ ف‪ ٟ‬اٌشىً (‪: )3‬‬
‫‪ ،‬ػٍّب أْ اٌؼجبسح اٌضِٕ‪١‬خ ٌٍز‪ٛ‬رش ث‪ ٓ١‬غشف‪ ٟ‬اٌّىضفخ‬ ‫أ‪ -‬اوزت اٌؼجبسح اٌضِٕ‪١‬خ ٌٍطبلخ اٌّخضٔخ ف‪ ٟ‬اٌّىضفخ‬

‫رؼط‪ ٝ‬ثبٌشىً‪:‬‬
‫انشكم (‪)3‬‬ ‫ة‪ -‬عذ ل‪ّ١‬خ صبثذ اٌضِٓ ‪ ، τ‬صُ اعزٕزظ ل‪ّ١‬خ اٌّمب‪ِٚ‬خ ‪.R‬‬
‫اٌّ‪ٛ‬افك ٌزٕبلص اٌطبلخ اٌّخضٔخ ف‪ٟ‬‬ ‫ث‪ -‬عذ ل‪ّ١‬خ اٌضِٓ‬
‫ِٓ ل‪ّ١‬ز‪ٙ‬ب االثزذائ‪١‬خ‪.‬‬ ‫اٌّىضفخ إٌ‪ٝ‬‬
‫اٌّؾ‪ٌٛ‬خ ف‪ ٟ‬إٌبلً األ‪ِٟٚ‬‬ ‫ث‪ -‬اعزٕزظ ل‪ّ١‬خ اٌطبلخ‬
‫‪.‬‬ ‫ثفؼً ع‪ٛ‬ي ف‪ ٟ‬اٌٍؾظخ‬
‫‪ٔ -III‬ؼ‪١‬ذ ٔفظ اٌزغشثخ اٌغبثمخ ‪ٚ‬لجً ‪ٚ‬ظغ اٌجبدٌخ ف‪ ٟ‬اٌ‪ٛ‬ظغ )‪(2‬‬
‫ٔشثػ ٔبلال أ‪١ِٚ‬ب آخش ’‪ R‬ػٍ‪ ٝ‬اٌز‪ٛ‬اص‪ِ ٞ‬غ إٌبلً األ‪.R ِٟٚ‬‬
‫‪ِ ً٘ -1‬ذح رفش‪٠‬غ اٌّىضفخ رضداد أَ رٕمص؟ ػًٍ‪.‬‬
‫‪.‬‬ ‫ػٍّب أْ ل‪ّ١‬خ صبثذ اٌضِٓ‬ ‫‪ -2‬اؽغت ل‪ّ١‬خ‬

‫صفؾخ ‪8 ِٓ 2‬‬
‫اختببر انبكبنىرٌب انتجرٌبً فً مبدة ‪ :‬انعهىو انفٍسٌبئٍت ‪ //‬انشعبت ‪ :‬رٌبضٍبث ‪ ،‬تقىً رٌبضً ‪ //‬دورة ‪ :‬أفرٌم ‪2023‬‬
‫انتمرٌه انثبنج ‪ 06( :‬وقبط)‬
‫‪٠‬صٕغ شؼبػ‪ٙ‬ب ِغ‬ ‫‪٠‬ش‪٠‬ذ اْ ‪٠‬مزف وشح ثغشػخ اثزذائ‪١‬خ‬ ‫ِٓ أسظ‪١‬خ ٍِؼت وشح اٌغٍخ ‪ٛ٠‬عذ الػت‬ ‫ِٓ إٌمطخ‬
‫‪ِٛٚ ،‬ع‪ٛ‬دح ػٍ‪ ٝ‬اسرفبع‬ ‫‪ α‬ثبرغبٖ اٌغٍخ اٌز‪ٔ ٟ‬ؼزجش٘ب ؽٍمخ دائش‪٠‬خ ِشوض٘ب‬ ‫األفك اٌضا‪٠ٚ‬خ‬
‫ِٓ عطؼ األسض‪ ،‬ػٕذِب رغبدس اٌىشح ‪٠‬ذ اٌالػت ف‪ٔ ٟ‬مطخ ِٓ اٌٍّؼت ‪٠‬ى‪ِ ْٛ‬شوض ػطبٌز‪ٙ‬ب ػٍ‪ ٝ‬اسرفبع ِٓ عطؼ األسض‬
‫اٌشىً (‪ٔ ، )4‬ؼزجش اْ اٌ‪ٙ‬ذف ‪٠‬غغً ػٕذِب ‪ّ٠‬ش ِشوض اٌىشح ثّشوض اٌغٍخ‪.‬‬

‫اٌشىً (‪)4‬‬

‫ػٍ‪ ٝ‬أسظ‪١‬خ‬ ‫ِ‪ٛ‬ظغ اٌالػت‬ ‫‪ -1‬ثبػزجبس ِجذأ األصِٕخ ٌؾظخ لزف اٌالػت ٌٍىشح‪ِٚ ،‬جذأ اإلؽذاص‪١‬بد ػٕذ إٌمطخ‬
‫ِٕطجك ػٍ‪ ٝ‬األسض ‪ِٚ‬زغٗ ٔؾ‪ ٛ‬اٌشبل‪ٛ‬ي اٌّبس ِٓ ِشوض اٌغٍخ‪،‬‬ ‫اٌٍّؼت ‪ ،‬ثؾ‪١‬ش ‪٠‬ى‪ ْٛ‬اٌّؾ‪ٛ‬س‬
‫‪.‬‬ ‫‪٠‬ى‪ ْٛ‬ػّ‪ٛ‬د‪ ٞ‬ػٍ‪ ٝ‬أسظ‪١‬خ اٌٍّؼت ‪ِٚ‬زغٗ ٔؾ‪ ٛ‬األػٍ‪ٔ .ٝ‬ؼزجش‬ ‫‪ٚ‬اٌّؾ‪ٛ‬س‬
‫أ‪ -‬ادسط غج‪١‬ؼخ ؽشوخ اٌىشح ف‪ ٟ‬اٌٍّؼت ؟‬
‫ة‪ -‬اوزت اٌّؼبدالد اٌضِٕ‪١‬خ ٌٍؾشوخ ‪ٚ‬وزا ِؼبدٌخ اٌّغبس ِج‪ٕ١‬ب غج‪١‬ؼزٗ ؟‬
‫ِز‪ٛ‬لف ٌؾظخ لزفٗ ٌٍىشح ‪٠ ٛ٘ٚ ،‬جؼذ ػٓ اٌشبل‪ٛ‬ي اٌّبس ِٓ ِشوض اٌغٍخ ثّمذاس‬ ‫‪ -2‬إرا وبْ اٌالػت‬
‫‪٠‬غت أْ ‪٠‬مزف اٌالػت اٌىشح ؽز‪٠ ٝ‬غغً اٌ‪ٙ‬ذف ؟‬ ‫أ‪ -‬ثؤ‪ ٞ‬عشػخ‬
‫ة‪ِ -‬ب٘‪ ٟ‬اٌّذح اٌضِٕ‪١‬خ اٌز‪ ٟ‬رغزغشل‪ٙ‬ب اٌىشح ِٕز ٌؾظخ لزف‪ٙ‬ب ِٓ غشف اٌالػت اٌ‪ ٝ‬غب‪٠‬خ دخ‪ٌٙٛ‬ب اٌغٍخ ؟‬
‫ط‪ -‬اؽغت عشػخ اٌىشح ٌؾظخ ِش‪ٚ‬س٘ب ثّشوض اٌغٍخ ‪ٚ‬وزا اٌضا‪٠ٚ‬خ ‪ β‬اٌز‪٠ ٟ‬صٕؼ‪ٙ‬ب ِغ األفك ؟‬
‫ِٓ اٌالػت‬ ‫‪ٚ‬اٌغٍخ ػٍ‪ ٝ‬ثؼذ‬ ‫ِٓ اٌفش‪٠‬ك إٌّبفظ ‪٠‬مف ث‪ ٓ١‬اٌالػت‬ ‫‪ٔ -3‬فشض اْ اٌالػت‬
‫‪٠ٚ‬ؾب‪ٚ‬ي اػزشاض ِغبس اٌىشح ثبٌمفض شبل‪١ٌٛ‬ب سافؼب ‪٠‬ذ‪ ٗ٠‬إٌ‪ ٝ‬األػٍ‪ ٝ‬ؽز‪ ٝ‬رجٍغ أغشاف أصبثؼٗ‬
‫ف‪٠ ًٙ‬زّىٓ ِٓ رغغ‪ ً١‬اٌ‪ٙ‬ذف ٘زٖ اٌّشح ‪ ،‬اششػ‬ ‫اٌىشح ثٕفظ اٌغشػخ اٌغبثمخ‬ ‫فبرا لزف اٌالػت‬

‫صفؾخ ‪8 ِٓ 3‬‬
‫اختببر انبكبنىرٌب انتجرٌبً فً مبدة ‪ :‬انعهىو انفٍسٌبئٍت ‪ //‬انشعبت ‪ :‬رٌبضٍبث ‪ ،‬تقىً رٌبضً ‪ //‬دورة ‪ :‬أفرٌم ‪2023‬‬
‫انتمرٌه انتجرٌبً ‪ 06( :‬وقبط)‬
‫غ‪١‬ش إٌم‪٠( ٟ‬ؾز‪ ٞٛ‬ػٍ‪ ٝ‬ش‪ٛ‬ائت ال رزفبػً) دسعخ ٔمب‪ٚ‬رٗ‬ ‫ر‪ٛ‬عذ ف‪ ٟ‬اٌّخجش لبس‪ٚ‬سح وزت ػٍ‪ٙ١‬ب ِغؾ‪ٛ‬ق اٌّغٕ‪١‬ض‪َٛ٠‬‬
‫ّٕٔزط اٌزؾ‪ٛ‬ي اٌى‪١ّ١‬بئ‪ ٟ‬اٌزبَ ‪ٚ‬اٌجط‪ٟ‬ء ثبٌّؼبدٌخ ‪:‬‬

‫ِٓ ِؾٍ‪ٛ‬ي ؽّط وٍ‪ٛ‬س‬ ‫‪٠‬ؾز‪ٞٛ‬‬ ‫ف‪ ٟ‬ثبٌ‪ ْٛ‬صعبع‪ ٟ‬ؽغّٗ‬ ‫ِٓ ِغؾ‪ٛ‬ق‬ ‫ٔعغ وزٍخ‬
‫‪ ،‬رزُ ِزبثؼخ ٘زا اٌزؾ‪ٛ‬ي ػٓ غش‪٠‬ك ل‪١‬بط ظغػ غبص‬ ‫ػٕذ دسعخ ؽشاسح صبثزخ‬ ‫اٌ‪١ٙ‬ذس‪ٚ‬ع‪ ٓ١‬رشو‪١‬ضٖ‬
‫إٌّطٍك ‪ٚ‬اٌز‪ٔ ٞ‬ؼزجشٖ غبصا ِضبٌ‪١‬ب ‪ٚ‬رٌه اػزّبدا ػٍ‪ ٝ‬أؽذ اٌزشو‪١‬ج‪ ٓ١‬اٌزغش‪٠‬ج‪ ٓ١١‬اٌّ‪ٛ‬ظؾ‪ ٓ١‬ف‪ ٟ‬اٌشىً(‪:)5‬‬ ‫اٌ‪١ٙ‬ذس‪ٚ‬ع‪ٓ١‬‬

‫اٌشىً (‪)5‬‬

‫‪ -1‬اخزش اٌزشو‪١‬ت اٌزغش‪٠‬ج‪ ٟ‬اٌّغزؼًّ ف‪٘ ٟ‬زٖ اٌّزبثؼخ ‪ ،‬صُ عّ‪ ٟ‬اٌؼٕبصش اٌّشلّخ ف‪ٗ١‬‬
‫ثذالٌخ‬ ‫‪ -2‬أٔشئ عذ‪ٚ‬ال ٌزمذَ اٌزفبػً ‪ٚ‬اعزخشط ػجبسح رمذَ اٌزفبػً‬
‫٘‪ ٛ‬ظغػ اٌ‪ٛٙ‬اء داخً اٌجبٌ‪ْٛ‬‬ ‫ؽ‪١‬ش‬ ‫ف‪ ٟ‬اٌجبٌ‪ ْٛ‬ثذالٌخ‬ ‫‪ -3‬ػجش اٌعغػ‬
‫ؽغُ اٌجبٌ‪ ْٛ‬اٌضعبع‪ ،ٟ‬اٌزمذَ األػظّ‪ٟ‬‬ ‫‪،‬‬ ‫‪ -4‬اػزّبدا ػٍ‪ ٝ‬اٌشىً(‪ ، )6‬اؽغت‪ :‬ؽغُ اٌغبص إٌّطٍك‬
‫ٌّغؾ‪ٛ‬ق اٌّغٕ‪١‬ض‪َٛ٠‬‬ ‫‪ -5‬اعزٕزظ اٌّزفبػً اٌّؾذ ‪ٚ ،‬اؽغت إٌغجخ اٌىزٍ‪١‬خ‬
‫‪ -6‬ػشف اٌغشػخ اٌؾغّ‪١‬خ ٌٍزفبػً ‪ٚ‬اوزت ػجبسر‪ٙ‬ب ثذالٌخ اٌعغػ اٌجبٌ‪ ، ْٛ‬اؽغت ل‪ّ١‬ز‪ٙ‬ب األػظّ‪١‬خ‬
‫‪ ،‬ؽذدٖ ث‪١‬بٔ‪١‬ب‬ ‫‪ -7‬ػشف صِٓ ٔصف اٌزفبػً‬
‫‪ٔ -8‬غزؼًّ ٔفظ اٌىزٍخ اٌغبثمخ ٌىٓ ػٍ‪ ٝ‬شىً لطؼخ ‪ ،‬أػذ سعُ ِٕؾٕ‪ ٝ‬اٌج‪١‬بْ (ة) ف‪ٔ ٟ‬فظ إٌّؾٕ‪ ٝ‬اٌغبثك ِغ اٌزؼٍ‪ً١‬‬

‫اٌشىً (‪)6‬‬

‫اٌج‪١‬بْ (ة)‬ ‫اٌج‪١‬بْ (أ)‬

‫اوتهى انمىضىع األول‬ ‫انمعطٍبث ‪:‬‬


‫صفؾخ ‪8 ِٓ 4‬‬
‫اختببر انبكبنىرٌب انتجرٌبً فً مبدة ‪ :‬انعهىو انفٍسٌبئٍت ‪ //‬انشعبت ‪ :‬رٌبضٍبث ‪ ،‬تقىً رٌبضً ‪ //‬دورة ‪ :‬أفرٌم ‪2023‬‬
‫انمىضىع انثبوً‬

‫ٌحتىي انمىضىع انثبوً عهى ‪ 04‬صفحبث (مه انصفحت ‪ 5‬مه ‪ 8‬إنى انصفحت ‪ 8‬مه ‪)8‬‬
‫انتمرٌه األول ‪ 04( :‬وقبط)‬

‫‪ٚ‬اٌز‪ ٞ‬صِٓ ٔصف ػّشٖ‬ ‫‪ٔ ِٓ -I‬ظبئش ػٕصش اٌىشث‪ٔ ) ( ْٛ‬غذ إٌظ‪١‬ش اٌضبٌش‬
‫‪ٚ‬رٕزظ ٔ‪ٛ‬اح ثٕذ‬ ‫رزفىه ٔ‪ٛ‬ارٗ اٌّشؼخ ‪ٚ‬فك إٌّػ‬
‫‪ -1‬اػػ رؼش‪٠‬ف وً ِٓ ‪ :‬صِٓ ٔصف اٌؼّش ‪ ،‬اٌزفىه‬
‫‪،‬‬ ‫‪،‬‬ ‫‪،‬‬ ‫‪،‬‬ ‫ٌعطى ‪:‬‬ ‫‪.‬‬ ‫‪ -2‬اوزت ِؼبدٌخ اٌزفىه إٌ‪ ٞٚٛ‬صُ ؽذد إٌ‪ٛ‬اح‬
‫‪.‬‬ ‫‪ٚ‬‬ ‫) صُ ؽذد ػٍ‪ِٛ ٗ١‬لغ إٌ‪ٛ‬ار‪ٓ١‬‬ ‫‪ -3‬اسعُ ف‪ِ ٟ‬ؼٍُ ِزؼبِذ ِزغبٔظ ِمزطغ ِٓ اٌّخطػ (‬
‫‪ -II‬اعزؼبدد اٌغضائش ف‪ ٟ‬ع‪١ٍ٠ٛ‬خ ‪ 2020‬عّبعُ ش‪ٙ‬ذاء اٌض‪ٛ‬سح‬
‫اٌشؼج‪١‬خ ِٓ اٌّزؾف اٌفشٔغ‪ٌٍ ٟ‬زبس‪٠‬خ اٌطج‪١‬ؼ‪ ٟ‬ثؼذ اؽزغبص٘ب ألوضش‬
‫ِٓ لشْ ‪ ،‬لبَ اٌخجشاء ثزبس‪٠‬خ ‪ 2020/07/17‬ثؤخز ػ‪ٕ١‬خ ِٓ اؽذ‪ٜ‬‬
‫اٌغّبعُ ‪ٚ‬ثؼذ ل‪١‬بط ٔشبغ‪ٙ‬ب االشؼبػ‪ ٟ‬أػط‪ٝ‬‬
‫‪ِ -1‬ب ٘‪ ٛ‬اٌغ‪ٙ‬بص اٌّغزؼًّ ف‪ ٟ‬ل‪١‬بط إٌشبغ االشؼبػ‪ٟ‬‬
‫‪ -2‬ث‪ ٓ١‬أْ وزٍخ اٌؼ‪ٕ١‬خ اٌّؤخ‪ٛ‬رح ٘‪: ٟ‬‬
‫‪ -3‬ث‪ ٓ١‬أٔٗ ‪ّ٠‬ىٓ وزبثخ لبٔ‪ ْٛ‬اٌزٕبلص االشؼبػ‪ ٟ‬ثبٌشىً ‪:‬‬
‫𝒎 𝒏𝒍‪−‬‬
‫‪26,921‬‬
‫‪ -4‬ثبعزغالي ث‪١‬بْ اٌشىً (‪ )1‬ؽذد ‪:‬‬
‫أ‪ -‬ل‪ّ١‬خ صبثذ إٌشبغ االشؼبػ‪λ ٟ‬‬
‫ة‪ -‬وزٍخ اٌؼ‪ٕ١‬خ االثزذائ‪١‬خ‬
‫انشكم (‪)1‬‬
‫‪26,87‬‬ ‫‪ -5‬ؽذد ػّش اٌؼ‪ٕ١‬خ اٌّؤخ‪ٛ‬رح ِٓ اٌغّغّخ‬
‫(ِٓ ٌؾظخ االعزش‪ٙ‬بد إٌ‪ٌ ٝ‬ؾظخ أخز اٌم‪١‬بط)‬
‫𝒔𝒏𝒂 𝒕‬ ‫‪ ِٓ -6‬خالي اٌغذ‪ٚ‬ي رؼشف ػٍ‪ ٝ‬اٌش‪١ٙ‬ذ صبؽت اٌغّغّخ‬
‫ربس‪٠‬خ االعزش‪ٙ‬بد‬ ‫إٌّصت‬ ‫اٌش‪١ٙ‬ذ‬
‫‪1849‬‬ ‫لبئذ ص‪ٛ‬سح اٌضػبغشخ‬ ‫اٌش‪١‬خ أؽّذ ث‪ٛ‬ص‪٠‬بْ‬
‫‪1854‬‬ ‫لبئذ ص‪ٛ‬سح اٌمجبئً‬ ‫ِؾّذ األِغذ (اٌشش‪٠‬ف ث‪ٛ‬ثغٍخ)‬
‫انمعطٍبث ‪:‬‬

‫صفؾخ ‪8 ِٓ 5‬‬
‫اختببر انبكبنىرٌب انتجرٌبً فً مبدة ‪ :‬انعهىو انفٍسٌبئٍت ‪ //‬انشعبت ‪ :‬رٌبضٍبث ‪ ،‬تقىً رٌبضً ‪ //‬دورة ‪ :‬أفرٌم ‪2023‬‬
‫انتمرٌه انثبوً ‪ 04( :‬وقبط)‬
‫)‬ ‫‪٠‬غزؼًّ ف‪ِٕ ٟ‬غ رآوً اٌغخبٔبد اٌّبئ‪١‬خ (‬ ‫اٌ‪١ٙ‬ذساص‪ٛٔ ٓ٠‬ع و‪١ّ١‬بئ‪ ٟ‬عبئً ص‪١‬غزٗ اٌى‪١ّ١‬بئ‪١‬خ‬
‫‪ٚ‬أٔبث‪١‬ت اٌزذفئخ اٌّشوض‪٠‬خ ‪ٚ‬رٌه ثئظبفخ وّ‪١‬خ ِٕٗ إٌ‪ ٝ‬اٌّبء اٌّ‪ٛ‬ع‪ٛ‬د داخً اٌغخبْ ‪.‬‬
‫ٌٍّؾٍ‪ٛ‬ي‬ ‫اٌّبء اٌّمطش أػط‪ ٝ‬ل‪١‬بط اٌـ‬ ‫ِٓ اٌ‪١ٙ‬ذساص‪ ٓ٠‬إٌم‪١‬خ ف‪ٟ‬‬ ‫‪ٔ -I‬ؾعش ِؾٍ‪ٛ‬ال ِبئ‪١‬ب ثئراثخ‬
‫‪ّٕ٠ ،‬زط اٌزؾ‪ٛ‬ي اٌى‪١ّ١‬بئ‪ ٟ‬اٌؾبدس ثبٌّؼبدٌخ ‪:‬‬ ‫اٌم‪ّ١‬خ‬

‫‪٠‬غٍه عٍ‪ٛ‬ن أعبط ؽغت ثش‪ٔٚ‬شزذ‬ ‫‪ -1‬ث‪ ٓ١‬أْ اٌ‪١ٙ‬ذساص‪ٓ٠‬‬


‫‪ -2‬أٔشئ عذ‪ٚ‬ي اٌزمذَ ٌٍزفبػً‬
‫‪ِ ،‬برا رغزٕزظ‬ ‫‪ -3‬اؽغت إٌغجخ إٌ‪ٙ‬بئ‪١‬خ ٌزمذَ اٌزفبػً‬
‫[‬ ‫]‬
‫صُ ػ‪ ٓ١‬اٌصفخ اٌغبٌجخ ف‪ ٟ‬اٌّؾٍ‪ٛ‬ي‬ ‫‪ -4‬اؽغت إٌغجخ‬
‫[‬ ‫]‬

‫[‬ ‫‪٠‬ؼط‪ ٝ‬ثبٌؼجبسح ]‬ ‫‪ -5‬أصجذ أْ صبثذ اٌز‪ٛ‬اصْ‬

‫اؽغت ل‪ّ١‬زٗ‬
‫ػٍّب أْ ‪:‬‬ ‫‪ٚ‬‬ ‫‪ -6‬لبسْ ث‪ ٓ١‬ل‪ٛ‬ر‪ ٟ‬األعبع‪ٓ١‬‬
‫صُ ٔؼب‪٠‬ش٘ب ث‪ٛ‬اعطخ ِؾٍ‪ٛ‬ي ؽّط وٍ‪ٛ‬س اٌّبء‬ ‫‪ٔ -II‬ؤخز ػ‪ٕ١‬خ ِٓ ِبء عخبْ اٌزذفئخ اٌّشوض‪٠‬خ ؽغّ‪ٙ‬ب‬
‫‪ ،‬إٌزبئظ اٌزغش‪٠‬ج‪١‬خ اٌّؾصً ػٍ‪ٙ١‬ب ِىٕذ ِٓ سعُ‬ ‫رشو‪١‬ضٖ اٌّ‪ٌٟٛ‬‬
‫اٌشىً (‪)2‬‬ ‫إٌّؾٕ‪ ٝ‬اٌج‪١‬بٔ‪ٟ‬‬
‫ِزش‪٠‬خ‬ ‫‪ -1‬اسعُ اٌزشو‪١‬ت اٌزغش‪٠‬ج‪ٌٍّ ٟ‬ؼب‪٠‬شح اٌـ‬

‫انشكم (‪)2‬‬ ‫‪ -2‬اوزت ِؼبدٌخ رفبػً اٌّؼب‪٠‬شح‬


‫‪ -3‬ػ‪ ٓ١‬اؽذاص‪١‬بد ٔمطخ اٌزىبفؤ‬
‫‪ -4‬اؽغت وّ‪١‬خ ِبدح اٌ‪١ٙ‬ذساص‪ ٓ٠‬ف‪ِ ٟ‬بء اٌغخبْ‬
‫ِٓ اٌّؾٍ‪ٛ‬ي اٌؾّع‪: ٟ‬‬ ‫‪ -5‬ػٕذ إظبفخ ؽغُ‬
‫أ‪ -‬ػ‪ ٓ١‬اٌصفخ اٌغبٌجخ‬
‫ة‪ -‬اؽغت إٌغجخ إٌ‪ٙ‬بئ‪١‬خ ٌزمذَ اٌزفبػً‬
‫ِبرا رغزٕزظ‬
‫‪ٌ" -6‬مىع انهٍذرازٌه كم مه انسخبن انمبئً‬
‫وشبكت انتذفئت انمركسٌت مه انتآكم"‬
‫اششػ ٘زٖ اٌؼجبسح ِٓ ‪ٚ‬ع‪ٙ‬خ إٌظش اٌى‪١ّ١‬بئ‪١‬خ‬
‫انمعطٍبث ‪:‬‬

‫صفؾخ ‪8 ِٓ 6‬‬
‫اختببر انبكبنىرٌب انتجرٌبً فً مبدة ‪ :‬انعهىو انفٍسٌبئٍت ‪ //‬انشعبت ‪ :‬رٌبضٍبث ‪ ،‬تقىً رٌبضً ‪ //‬دورة ‪ :‬أفرٌم ‪2023‬‬
‫انتمرٌه انثبنج ‪ 06( :‬وقبط)‬
‫ثؼذ ‪ٚ‬لذ لص‪١‬ش ِٓ غش‪ٚ‬ة شّظ ‪ِ 28 َٛ٠‬بسط ‪2023‬‬ ‫‪-I‬‬
‫سصذ ثغّبء اٌ‪ٛ‬غٓ اٌؼشث‪ ٟ‬ؽذس فٍى‪ٔ ٟ‬بدس (ػشط اٌغّبء)‬
‫ؽ‪١‬ش اصطفذ خّغخ و‪ٛ‬اوت ‪ِٚ‬غّ‪ٛ‬ػخ ٔغّ‪١‬خ ش‪١ٙ‬شح ‪ٚ‬اٌمّش‬
‫ِؼب ‪ ،‬شىً ػطبسد ‪ٚ‬اٌّشزش‪ٚ ٞ‬اٌض٘شح ‪ ٚ‬أ‪ٚ‬سأ‪ٛ‬ط ‪ٚ‬اٌّش‪٠‬خ‬
‫ل‪ٛ‬عًب ثغبٔت اٌمّش ‪ٚ‬وبْ اٌؾذس ِشئ‪١‬ب ثبٌؼ‪ ٓ١‬اٌّغشدح ِّب عؼً‬
‫ً‬
‫عّ‪١‬ال ف‪ ٟ‬عّبء اٌٍ‪. ً١‬‬ ‫اٌّش‪ٙ‬ذ‬
‫‪ِ -1‬ب ٘‪ ٛ‬اٌّشعغ إٌّبعت ٌذساعخ ؽشوخ و‪ٛ‬اوت اٌّغّ‪ٛ‬ػخ اٌشّغ‪١‬خ ‪ ،‬ػشفٗ‬
‫‪ -2‬اْ ِشالجخ ؽشوخ اٌى‪ٛ‬اوت ِىٕزٕب ِٓ عذ‪ٚ‬ي اٌم‪١‬بعبد اٌزبٌ‪: ٟ‬‬
‫اٌض٘شح‬ ‫اٌّشزش‪ٞ‬‬ ‫ػطبسد‬ ‫اٌى‪ٛ‬وت‬
‫اٌذ‪ٚ‬س‬
‫ٔصف لطش اٌذ‪ٚ‬ساْ‬
‫‪،‬‬ ‫٘‪ ٟ‬اٌ‪ٛ‬ؽذح اٌفٍى‪١‬خ ؽ‪١‬ش ‪:‬‬
‫أ‪ -‬ثزطج‪١‬ك اٌمبٔ‪ ْٛ‬اٌضبٔ‪ٛ١ٌٕ ٟ‬رٓ أوزت ػجبسح اٌغشػخ اٌّذاس‪٠‬خ ٌى‪ٛ‬وت ِٓ اٌّغّ‪ٛ‬ػخ اٌشّغ‪١‬خ‬
‫انشكم (‪)3‬‬
‫ة‪ -‬ث‪ ٓ١‬أْ لبٔ‪ ْٛ‬وجٍش اٌضبٌش ‪٠‬ؼط‪ ٝ‬ثبٌؼاللخ ‪:‬‬

‫ث‪ -‬أوًّ اٌغذ‪ٚ‬ي أػالٖ‬


‫ث‪ -‬أؽغت وزٍخ اٌشّظ‬
‫اٍ٘‪١ٍ١‬غ‪١‬ب وّب‬ ‫‪٠‬ى‪ِ ْٛ‬غبس ؽشوخ ِشوض ػطبٌخ و‪ٛ‬وت‬ ‫‪-II‬‬
‫إٌ‪ٝ‬‬ ‫‪ٛ٠‬ظؾٗ اٌشىً (‪ ، )3‬ؽ‪١‬ش ‪ٕ٠‬زمً اٌى‪ٛ‬وت ِٓ إٌمطخ‬
‫خالي ٔفظ اٌّذح اٌضِٕ‪١‬خ‬ ‫إٌ‪ٝ‬‬ ‫صُ ِٓ إٌمطخ‬

‫انشكم (‪)4‬‬ ‫‪ -1‬اػزّبدا ػٍ‪ ٝ‬لبٔ‪ ْٛ‬وجٍش األ‪ٚ‬ي فغش ‪ٚ‬ع‪ٛ‬د ِ‪ٛ‬لغ اٌشّظ ف‪ ٟ‬إٌمطخ‬
‫‪ٚ‬‬ ‫‪ -2‬ؽغت لبٔ‪ ْٛ‬وجٍش اٌضبٔ‪ِ ٟ‬ب٘‪ ٟ‬اٌؼاللخ ث‪ ٓ١‬اٌّغبؽز‪ٓ١‬‬
‫‪ٚ‬‬ ‫ألً ِٓ اٌغشػخ اٌّز‪ٛ‬عطخ ث‪ ٓ١‬اٌّ‪ٛ‬ظؼ‪ٓ١‬‬ ‫‪ٚ‬‬ ‫‪ -3‬ث‪ ٓ١‬أْ اٌغشػخ اٌّز‪ٛ‬عطخ ث‪ ٓ١‬اٌّ‪ٛ‬ظؼ‪ٓ١‬‬
‫‪ ِٓ -III‬أعً رغ‪ ً١ٙ‬اٌذساعخ ٔمجً أْ ؽشوخ اٌى‪ٛ‬وت ؽ‪ٛ‬ي اٌشّظ دائش‪٠‬خ ‪ٚ‬أٔ‪ٙ‬ب ال رخعغ إال ٌزؤص‪١‬ش٘ب فمػ اٌشىً (‪)4‬‬
‫𝟐‬
‫𝑮𝒂‬ ‫𝟎𝟏‬ ‫𝟐𝒔 𝒎‬ ‫‪ِ -1‬ضً اٌم‪ٛ‬ح اٌز‪ ٟ‬رؤصش ث‪ٙ‬ب اٌشّظ ػٍ‪ ٝ‬اٌى‪ٛ‬وت صُ اػػ ػجبسر‪ٙ‬ب‬
‫اٌى‪ٛ‬وت‬ ‫‪ -2‬ثزطج‪١‬ك اٌمبٔ‪ ْٛ‬اٌضبٔ‪ٛ١ٌٕ ٟ‬رٓ ث‪ ٓ١‬أْ ػجبسح رغبسع‬

‫‪ ،‬ػ‪ ٓ١‬ػجبسح ‪α‬‬ ‫ؽ‪ٛ‬ي اٌشّظ رؼط‪ ٝ‬ثبٌؼاللخ‬


‫انشكم (‪)5‬‬
‫‪ -3‬أػػ اٌؼجبسح اٌز‪٠ ٟ‬زشعّ‪ٙ‬ب ث‪١‬بْ اٌشىً (‪)5‬‬
‫‪ -4‬ثبالػزّبد ػٍ‪ ٝ‬اٌؼاللز‪ ٓ١‬إٌظش‪٠‬خ ‪ٚ‬اٌؼٍّ‪١‬خ ‪ ،‬اعزٕزظ وزٍخ اٌشّظ‬
‫‪2‬‬ ‫𝟏‬ ‫𝟐𝟐‬ ‫𝟐‬
‫𝟎𝟏‬ ‫𝒎‬
‫𝟐𝒓‬

‫صفؾخ ‪8 ِٓ 7‬‬
‫اختببر انبكبنىرٌب انتجرٌبً فً مبدة ‪ :‬انعهىو انفٍسٌبئٍت ‪ //‬انشعبت ‪ :‬رٌبضٍبث ‪ ،‬تقىً رٌبضً ‪ //‬دورة ‪ :‬أفرٌم ‪2023‬‬
‫انتمرٌه انتجرٌبً ‪ 06( :‬وقبط)‬
‫رؾز‪ ٞٛ‬األع‪ٙ‬ضح اٌى‪ٙ‬شثبئ‪١‬خ ػٍ‪ِ ٝ‬ىضفبد ‪ٛٔٚ ٚ‬الً أ‪١ِٚ‬خ ‪ٚٚ‬شبئغ ‪ ،‬رخزٍف ‪ٚ‬ظ‪١‬فخ وً ِٕ‪ٙ‬ب ؽغت و‪١‬ف‪١‬خ رشو‪١‬ج‪ٙ‬ب ‪ِٚ‬غبي‬
‫اعزؼّبٌ‪ٙ‬ب ‪ ِٓ ،‬أعً رؾذ‪٠‬ذ ِّ‪١‬ضاد ثؼط اٌؼٕبصش اٌى‪ٙ‬شثبئ‪١‬خ ٕٔغض اٌذاسح اٌى‪ٙ‬شثبئ‪١‬خ اٌشىً (‪ ٚ )6‬اٌّى‪ٔٛ‬خ ِٓ ‪:‬‬
‫‪ٌِٛ -‬ذ ر‪ٛ‬رش صبثذ ل‪ٛ‬رٗ اٌّؾشوخ اٌى‪ٙ‬شثبئ‪١‬خ‬
‫‪ٛٔ -‬الً أ‪١ِٚ‬خ ِمب‪ِٚ‬ز‪ٙ‬ب ‪:‬‬

‫‪ِ -‬ىضفخ فبسغخ عؼز‪ٙ‬ب‬


‫‪ٚ -‬ش‪١‬ؼخ صشفخ ( ) رار‪١‬ز‪ٙ‬ب‬
‫انشكم (‪)6‬‬
‫‪ -‬ثبدٌخ‬
‫اٌّج‪ ٓ١‬ف‪ ٟ‬اٌشىً (‪)7‬‬ ‫ف‪ ٟ‬اٌ‪ٛ‬ظغ (‪ )1‬فٕؾصً ػٍ‪ ٝ‬إٌّؾٕ‪ٝ‬‬ ‫) ٔعغ اٌجبدٌخ‬ ‫‪ -I‬ف‪ ٟ‬اٌٍؾظخ (‬
‫‪ -1‬ثزطج‪١‬ك لبٔ‪ ْٛ‬عّغ اٌز‪ٛ‬رشاد ث‪ ٓ١‬أْ اٌّؼبدٌخ اٌزفبظٍ‪١‬خ ٌٍز‪ٛ‬رش‬

‫انشكم (‪)7‬‬ ‫رىزت ػٍ‪ ٝ‬اٌشىً ‪:‬‬

‫ؽال ٌ‪ٙ‬ب‬ ‫‪ -2‬رمجً اٌّؼبدٌخ اٌزفبظٍ‪١‬خ اٌؼجبسح‬


‫صبثذ ‪٠‬طٍت ا‪٠‬غبد ػجبسرٗ ثذالٌخ ص‪ٛ‬اثذ اٌذاسح‬ ‫ؽ‪١‬ش‬
‫‪ -3‬اعزٕزظ وً ِٓ ‪:‬‬
‫أ‪ -‬اٌز‪ٛ‬رش ث‪ ٓ١‬غشف‪ ٟ‬اٌّ‪ٌٛ‬ذ‬
‫ة‪ -‬صبثذ اٌضِٓ‬
‫ث‪ -‬عؼخ اٌّىضفخ‬
‫ث‪ -‬شذح اٌز‪١‬بس األػظّ‪ ٟ‬اٌّبس ف‪ ٟ‬اٌذاسح‬
‫إٌ‪ ٝ‬اٌ‪ٛ‬ظغ (‪)2‬‬ ‫) ٔؤسعؼ اٌجبدٌخ‬ ‫ف‪ٌ ٟ‬ؾظخ صِٕ‪١‬خ ٔؼزجش٘ب وّجذأ عذ‪٠‬ذ ٌألصِٕخ (‬ ‫‪-II‬‬
‫‪ -1‬ثزطج‪١‬ك لبٔ‪ ْٛ‬عّغ اٌز‪ٛ‬رشاد عذ اٌّؼبدٌخ اٌزفبظٍ‪١‬خ ٌٍز‪١‬بس‬
‫‪−‬‬ ‫‪ -2‬رمجً اٌّؼبدٌخ اٌزفبظٍ‪١‬خ اٌؼجبسح‬
‫وؾً ٌ‪ٙ‬ب ‪ ،‬ؽ‪١‬ش ‪ ٚ‬صبثز‪٠ ٓ١‬طٍت ا‪٠‬غبد ػجبسر‪ّٙ١‬ب‬
‫انشكم (‪)8‬‬ ‫‪ّٔ -3‬ضً ف‪ ٟ‬اٌشىً (‪ )8‬رغ‪١‬شاد ثذالٌخ اٌضِٓ‬
‫ثذالٌخ اٌضِٓ‬ ‫اوزت ػجبسح‬

‫عذ ‪:‬‬ ‫‪ -4‬اػزّبدا ػٍ‪ ٝ‬إٌّؾٕ‪ ٝ‬اٌج‪١‬بٔ‪ٟ‬‬

‫أ‪ -‬ل‪ّ١‬خ رار‪١‬خ اٌ‪ٛ‬ش‪١‬ؼخ‬


‫ة‪ -‬صبثذ اٌضِٓ‬
‫د‪ -‬اٌطبلخ االػظّ‪١‬خ‬
‫اٌّخضٔخ ف‪ ٟ‬اٌ‪ٛ‬ش‪١‬ؼخ‬
‫اوتهى انمىضىع انثبوً‬ ‫اإلجببت انىمىرجٍت‬

‫صفؾخ ‪8 ِٓ 8‬‬
‫انغًٕٓس‪ٚ‬خ انغضائش‪ٚ‬خ انذ‪ًٚ‬وشاغ‪ٛ‬خ انشؼج‪ٛ‬خ‬
‫انًذسعخ انؼه‪ٛ‬ب نألعبرزح – ٔسڤـهخ‬ ‫ٔصاسح انزؼه‪ٛ‬ى انؼبن‪ٔ ٙ‬انجحش انؼهً‪ٙ‬‬
‫دٔسح ‪ :‬أكش‪ٚ‬م ‪0202‬‬ ‫امتحان انبكانوريا انتجريبي‬
‫إػذاد األعزبر ‪ :‬يذٔس ع‪ٛ‬ق انذ‪ٍٚ‬‬ ‫انشؼجخ ‪ :‬س‪ٚ‬بظ‪ٛ‬بد ‪ ،‬روُ‪ ٙ‬س‪ٚ‬بظ‪ٙ‬‬
‫انًذح ‪ 24 :‬عبػخ ٔ ‪ 22‬ده‪ٛ‬وخ‬ ‫االعبثخ انًُٕرع‪ٛ‬خ التخزجبس ك‪ ٙ‬يبدح ‪ :‬انعـهوو انفيزيائيت‬

‫انموضوع األول‬
‫انؼاليخ‬
‫ػُبصش اإلعبثخ‬
‫يغضأح يغًٕع‬
‫انتمريه األول ‪ 44( :‬وقاط) ‪:‬‬
‫انًُحُٗ ‪ٚ‬زكٌٕ يٍ َظبي‪: ٍٛ‬‬
‫‪ 1‬غج‪ٛ‬ؼخ انحشكخ ك‪ ٙ‬انُظبو االَزوبن‪ : ٙ‬ركٌٕ حشكخ يشكض ػطبنخ حجخ انزًش يغزو‪ًٛ‬خ يزغبسػخ‬
‫ك‪ ٙ‬انُظبو انذائى ‪ :‬ركٌٕ حشكخ يشكض ػطبنخ حجخ انزًش يغزو‪ًٛ‬خ يُزظًخ‬
‫‪ .‬ظؼق يشح‬
‫‪ 2‬انُغجخ‬
‫ارٌ رًٓم داكؼخ استخً‪ٛ‬ذط أيبو انضوم‬
‫⃗‬ ‫⃗⃗⃗⃗⃗‬ ‫‪ 3‬رًض‪ٛ‬م انوٕٖ‬
‫⃗ ⃗⃗‬ ‫⃗⃗⃗⃗⃗‬
‫ثبالعوبغ ػهٗ انًحٕس(‪) OZ‬‬

‫‪.‬‬ ‫‪ 4‬انًؼبدنخ‬
‫‪.‬‬
‫‪.‬‬
‫‪.‬‬ ‫ك‪ ٙ‬انُظبو انذائى ‪:‬‬
‫‪ 5‬انغشػخ انحذ‪ٚ‬خ‬
‫‪.‬‬ ‫‪.‬‬
‫‪τ‬‬ ‫‪ 6‬انضيٍ انًً‪ٛ‬ض‬
‫ك‪ ٙ‬انُظبو‬
‫‪.‬‬ ‫‪.‬‬ ‫‪ 7‬أ ـ انضبثذ ‪K‬‬
‫انذائى ‪:‬‬
‫‪.‬‬ ‫[] [] [‬ ‫]‬ ‫ة ـ انزحه‪ٛ‬م‬
‫] [‬ ‫‪.‬‬ ‫] [‬ ‫[] [‬ ‫]‬
‫[] [‬ ‫]‬ ‫انجؼذ٘‬
‫‪.‬‬ ‫‪.‬‬ ‫‪ 8‬انزغبسع‬

‫صلحخ ‪ 1‬يٍ ‪10‬‬


‫االجابت الختبار انبكانوريا انتجريبي في مادة‪ :‬انعهوو انفيزيائيت‪//‬انشعبت ‪ :‬رياضياث‪ ،‬تقىي رياضي‪ //‬دورة ‪ :‬أفريم ‪2423‬‬
‫انؼاليخ‬
‫ػُبصش اإلعبثخ‬
‫يغضأح يغًٕع‬
‫انتمريه انثاوي ‪ 44( :‬وقاط) ‪:‬‬
‫‪I‬‬
‫انج‪ٛ‬بٌ تخػ يغزو‪ٛ‬ى يؼبدنزّ ‪:‬‬
‫‪.‬‬ ‫‪ 1‬انًُحُٗ‬

‫‪.‬‬ ‫‪.‬‬
‫‪ 2‬ػجبسح انزٕرش‬
‫‪.‬‬ ‫‪.‬‬

‫ثبنًطبثوخ‬
‫‪ 3‬عؼخ انًكضلخ‬
‫‪.‬‬
‫انهجٕط انغبنت ْٕ ‪B‬‬ ‫انهجٕط انًٕعت ْٕ ‪A‬‬ ‫‪ 4‬انهجٕع‪ٍٛ‬‬
‫‪II‬‬
‫رلش‪ٚ‬ؾ انًكضلخ‬ ‫انظبْشح‬
‫االنكزشَٔبد انًزشاكًخ ك‪ ٙ‬انهجٕط ‪ B‬رؼٕد رذس‪ٚ‬غ‪ٛ‬ب إنٗ انهجٕط ‪ A‬حزٗ ‪ٚ‬صجح‬ ‫‪1‬‬
‫انزلغ‪ٛ‬ش‬
‫انهجٕع‪ ٍٛ‬يزؼبدن‪ ٍٛ‬كٓشثبئ‪ٛ‬ب ػُذْب رزلشؽ انًكضلخ كه‪ٛ‬ب‬
‫‪.‬‬ ‫‪.‬‬ ‫انؼجبسح انضيُ‪ٛ‬خ‬
‫ة ـ صبثذ‬
‫‪τ‬‬ ‫‪τ‬‬ ‫ثبإلعوبغ َغذ‬ ‫انضيٍ ‪τ‬‬
‫‪τ‬‬ ‫‪.‬‬ ‫‪ 2‬انًوبٔيخ‬

‫د ـ انضيٍ‬
‫ثبإلعوبغ َغذ‬
‫س ـ انطبهخ‬

‫‪III‬‬
‫صبثذ انضيٍ ‪ٚ‬زُبعت غشدا يغ انًوبٔيخ انًكبكئخ‬
‫سثػ انًوبٔيبد ػهٗ انزٕاص٘ ‪ُٚ‬وص يٍ ه‪ًٛ‬خ انًوبٔيخ انًكبكئخ‬ ‫‪ 1‬يذح انزلش‪ٚ‬ؾ‬
‫ارٌ ه‪ًٛ‬خ صبثذ انضيٍ رُوص ‪ ،‬ارٌ يذح انزلش‪ٚ‬ؾ رُوص‬
‫‪τ‬‬
‫‪τ‬‬ ‫‪.‬‬
‫‪ 2‬حغبة‬
‫‪.‬‬

‫صلحخ ‪ 2‬يٍ ‪10‬‬


‫االجابت الختبار انبكانوريا انتجريبي في مادة‪ :‬انعهوو انفيزيائيت‪//‬انشعبت ‪ :‬رياضياث‪ ،‬تقىي رياضي‪ //‬دورة ‪ :‬أفريم ‪2423‬‬
‫انؼاليخ‬
‫ػُبصش اإلعبثخ‬
‫يغضأح يغًٕع‬
‫انتمريه انثانث ‪ 46( :‬وقاط) ‪:‬‬
‫⃗‬ ‫⃗⃗⃗⃗⃗‬ ‫أ ـ دساعخ‬
‫⃗⃗⃗⃗⃗‬ ‫غج‪ٛ‬ؼخ انحشكخ‬
‫ثبالعوبغ ػهٗ انًحٕس‪)OX, OY(ٍٚ‬‬
‫انحشكخ ػهٗ (‪ )OY‬يغزو‪ًٛ‬خ يزـ‪ٛ‬شح ثبَزظبو‬ ‫انحشكخ ػهٗ (‪ )OX‬يغزو‪ًٛ‬خ يُزظًخ‬
‫انششٔغ‬ ‫‪1‬‬
‫‪.‬‬
‫ثبنزكبيم‬ ‫ة ـ انًؼبدالد االثزذائ‪ٛ‬خ‬
‫‪.‬‬
‫انضيُ‪ٛ‬خ‬
‫نهغشػخ‬

‫ثبنزكبيم‬
‫‪.‬‬
‫‪.‬‬ ‫يٍ انششٔغ‬
‫االثزذائ‪ٛ‬خ‬
‫‪.‬‬
‫انًؼبدالد انضيُ‪ٛ‬خ‬
‫‪.‬‬
‫نهًٕظغ‬
‫َؼٕض ك‪ ٙ‬انًؼبدنخ انضيُ‪ٛ‬خ‬
‫‪.‬‬ ‫‪1‬‬
‫يؼبدنخ‬
‫(‬ ‫)‬ ‫(‬ ‫)‬ ‫‪.‬‬ ‫انًغبس‬
‫‪.‬‬
‫ْٔ‪ ٙ‬يؼبدنخ هطغ يكبكئ إرٌ‬ ‫يؼبدنخ انًغبس يٍ انشكم‬ ‫غج‪ٛ‬ؼخ‬
‫يغبس انكشح ػجبسح ػٍ هطغ يكبكئ‬ ‫انًغبس‬
‫‪.‬‬
‫√‬ ‫أ ـ انغشػخ‬
‫‪.‬‬

‫‪.‬‬
‫ة ـ انًذح‬
‫‪.‬‬ ‫انضيُ‪ٛ‬خ‬
‫‪2‬‬
‫ط ـ عشػخ‬
‫انكشح‬
‫√‬ ‫√‬ ‫‪.‬‬

‫|‬ ‫|‬ ‫|‬ ‫|‬


‫‪.‬‬
‫انضأ‪ٚ‬خ ‪β‬‬

‫صلحخ ‪ 3‬يٍ ‪10‬‬


‫االجابت الختبار انبكانوريا انتجريبي في مادة‪ :‬انعهوو انفيزيائيت‪//‬انشعبت ‪ :‬رياضياث‪ ،‬تقىي رياضي‪ //‬دورة ‪ :‬أفريم ‪2423‬‬
‫نًؼشكخ ايكبَ‪ٛ‬خ رغغ‪ٛ‬م انٓذف َحغت اسرلبع انكشح ك‪ ٙ‬انُوطخ انز‪ٚ ٙ‬وق ك‪ٓٛ‬ب‬
‫ك‪ ٙ‬يؼبدنخ انًغبس‬ ‫انالػت (‪ٔ )B‬رنك ثزؼٕ‪ٚ‬ط كبصهزٓب‬
‫‪.‬‬ ‫‪3‬‬
‫‪.‬‬
‫انٓذف ال ‪ٚ‬غغم ألٌ اسرلبع انكشح أهم يٍ هلضح انالػت‬

‫انؼاليخ‬
‫ػُبصش اإلعبثخ‬
‫يغضأح يغًٕع‬
‫انتمريه انتجريبي ‪ 46( :‬وقاط) ‪:‬‬
‫انزشك‪ٛ‬ت انًغزؼًم ْٕ ‪ :‬انزشك‪ٛ‬ت‬ ‫انزشك‪ٛ‬ت‬
‫‪ -2‬هًغ صعبع‪ٙ‬‬ ‫‪ -1‬إسنًُب‪ٚ‬ش‬ ‫‪1‬‬
‫انج‪ٛ‬بَبد‬
‫‪ -4‬عٓبص ه‪ٛ‬بط انعـػ‬ ‫‪ -0‬يض‪ٚ‬ظ رلبػه‪ٔ( ٙ‬عػ)‬

‫‪0‬‬ ‫‪0‬‬
‫عذٔل انزوذو‬

‫ثض‪ٚ‬بدح‬
‫‪2‬‬

‫‪.‬‬ ‫‪.‬‬ ‫انؼجبسح‬


‫‪.‬‬ ‫‪ 3‬انعـػ ‪P‬‬
‫انج‪ٛ‬بٌ تخػ يغزو‪ٛ‬ى يؼبدنزّ‬
‫‪.‬‬

‫ثبنًطبثوخ‬ ‫‪.‬‬
‫‪4‬‬
‫‪.‬‬

‫‪.‬‬
‫ارٌ حًط كهٕس انٓ‪ٛ‬ذسٔع‪ ٍٛ‬يزلبػم ؿ‪ٛ‬ش يحذ ٔثًب أٌ‬ ‫انًزلبػم انًحذ‬
‫انزلبػم ربو كإٌ انًـُ‪ٛ‬ض‪ٕٚ‬و ْٕ انًزلبػم انًحذ‬ ‫‪5‬‬
‫‪.‬‬
‫انُغجخ انكزه‪ٛ‬خ‬
‫‪.‬‬ ‫‪.‬‬
‫ْ‪ ٙ‬عشػخ انزلبػم ك‪ٔ ٙ‬حذح انحغى‬ ‫انزؼش‪ٚ‬ق‬

‫انغشػخ‬
‫‪6‬‬
‫‪.‬‬ ‫انحغً‪ٛ‬خ‬
‫‪.‬‬
‫‪.‬‬ ‫انو‪ًٛ‬خ انؼظًٗ‬
‫صلحخ ‪ 4‬يٍ ‪10‬‬
‫االجابت الختبار انبكانوريا انتجريبي في مادة‪ :‬انعهوو انفيزيائيت‪//‬انشعبت ‪ :‬رياضياث‪ ،‬تقىي رياضي‪ //‬دورة ‪ :‬أفريم ‪2423‬‬

‫ْٕ انضيٍ انالصو نجهٕؽ انزلبػم َصق روذيّ انُٓبئ‪ٙ‬‬ ‫صيٍ َصق‬
‫) (‬ ‫‪.‬‬ ‫‪7‬‬
‫ا‪ٚ‬غبدِ ث‪ٛ‬بَ‪ٛ‬ب‬
‫ثبإلعوبغ َغذ ‪:‬‬
‫𝑷‬ ‫𝒂𝑷 𝟓𝟎𝟏‬
‫𝒈𝑴 مسحوق‬
‫انؼبيم انحشك‪ ٙ‬انًذسٔط ْٕ يغبحخ‬
‫عطح انزاليظ ‪ ،‬ح‪ٛ‬ش كهًب صادد‬
‫𝒈𝑴 قطعة‬ ‫يغبحخ انزاليظ صادد عشػخ انزلبػم‬ ‫‪ 8‬سعى انًُحُٗ‬
‫(يغبحخ عطح راليظ انًغحٕم أكجش‬
‫يٍ انوطؼخ)‬
‫𝟓𝟎‬
‫𝒏𝒊𝒎 𝒕‬
‫𝟎‬ ‫𝟓‬

‫صلحخ ‪ 5‬يٍ ‪10‬‬


‫االجابت الختبار انبكانوريا انتجريبي في مادة‪ :‬انعهوو انفيزيائيت‪//‬انشعبت ‪ :‬رياضياث‪ ،‬تقىي رياضي‪ //‬دورة ‪ :‬أفريم ‪2423‬‬
‫انموضوع انثاوي‬
‫انؼاليخ‬
‫ػُبصش اإلعبثخ‬
‫يغضأح يغًٕع‬
‫انتمريه األول ‪ 44( :‬وقاط) ‪:‬‬
‫‪I‬‬
‫انزلكك ‪ ْٕ : β‬تخشٔط عغ‪ٛ‬ى عبنت‬ ‫صيٍ َصق انؼًش ‪ ْٕ :‬انضيٍ انالصو‬
‫ػٍ غش‪ٚ‬ن‬ ‫انشحُخ يٍ انُٕاح‬ ‫نزلكك َصق ػذد األَٕ‪ٚ‬خ االثزذائ‪ٛ‬خ‬
‫‪ 1‬انزؼش‪ٚ‬ق‬
‫رحٕل انُٕرشٌٔ إنٗ ثشٔرٌٕ‬

‫حغت هبََٕ‪ ٙ‬االَحلبظ نـ صٕد٘ ‪:‬‬


‫‪ 2‬انًؼبدنخ‬

‫‪ - 1‬انغٓبص انًغزؼًم ك‪ ٙ‬ه‪ٛ‬بط‬


‫انُشبغ االشؼبػ‪: ٙ‬‬
‫‪ 3‬انًخطػ‬
‫ػذاد ؿـ‪ٛ‬ــش (عـ‪ٛ‬غـش)‬

‫‪II‬‬
‫‪.‬‬ ‫𝜆‬ ‫‪.‬‬ ‫‪.‬‬
‫‪ 2‬كزهخ انؼ‪ُٛ‬خ‬
‫‪.‬‬
‫‪.‬‬ ‫‪ 3‬هبٌَٕ انزُبهص‬
‫انج‪ٛ‬بٌ تخػ يغزو‪ٛ‬ى يؼبدنزّ يٍ انشكم‬
‫‪.‬‬
‫𝜆‬ ‫أـ صبثذ ‪λ‬‬
‫‪.‬‬
‫𝜆‬
‫‪4‬‬
‫ثبنًطبثوخ ث‪ ٍٛ‬انؼجبسح‬
‫انش‪ٚ‬بظ‪ٛ‬خ‬
‫ة ـ انكزهخ‬
‫ٔ هبٌَٕ انزُبهص َغذ‬
‫‪λ‬‬
‫‪𝜆 .‬‬
‫‪.‬‬ ‫‪.‬‬ ‫‪ 5‬ػًش انؼ‪ُٛ‬خ‬
‫‪𝜆 .‬‬ ‫(‬ ‫)‬ ‫‪.‬‬
‫ْ‪ ٙ‬نهشٓ‪ٛ‬ذ أحًذ ثٕص‪ٚ‬بٌ‬ ‫‪ 6‬انشٓ‪ٛ‬ذ‬

‫صلحخ ‪ 6‬يٍ ‪10‬‬


‫االجابت الختبار انبكانوريا انتجريبي في مادة‪ :‬انعهوو انفيزيائيت‪//‬انشعبت ‪ :‬رياضياث‪ ،‬تقىي رياضي‪ //‬دورة ‪ :‬أفريم ‪2423‬‬

‫انؼاليخ‬
‫ػُبصش اإلعبثخ‬
‫يغضأح يغًٕع‬
‫انتمريه انثاوي ‪ 45( :‬وقاط) ‪:‬‬
‫‪I‬‬
‫‪ 1‬األعبط‬
‫حبنخ انغًهخ‬
‫‪.‬‬ ‫االثزذائ‪ٛ‬خ‬
‫االَزوبن‪ٛ‬خ‬ ‫‪ 2‬عذٔل انزوذو‬

‫ثٕكشح‬ ‫انُٓبئ‪ٛ‬خ‬

‫‪.‬‬
‫[‬ ‫]‬ ‫‪.‬‬ ‫َغجخ انزوذو‬
‫[‬ ‫]‬
‫‪3‬‬
‫‪.‬‬ ‫انُٓبئ‪ٛ‬خ‬
‫انزلبػم ؿ‪ٛ‬ش ربو ٔاألعبط‬
‫‪τ‬‬ ‫‪.‬‬
‫ظؼ‪ٛ‬ق‬
‫[‬ ‫]‬ ‫[‬ ‫]‬
‫‪.‬‬ ‫‪.‬‬
‫[‬ ‫]‬ ‫[‬ ‫]‬
‫[‬ ‫]‬ ‫[‬ ‫]‬
‫[‬ ‫]‬
‫‪.‬‬
‫[‬ ‫]‬
‫‪.‬‬ ‫‪ 4‬انُغجخ‬
‫[‬ ‫]‬
‫‪.‬‬ ‫[‬ ‫]‬ ‫[‬ ‫‪] .‬‬ ‫انصلخ األعبع‪ٛ‬خ‬
‫[‬ ‫]‬ ‫ْ‪ ٙ‬انـبنجخ‬
‫[‬ ‫[ ]‬ ‫]‬
‫[‬ ‫]‬ ‫‪.‬‬ ‫‪.‬‬
‫[‬ ‫]‬
‫ػجبسح ‪K‬‬
‫[‬ ‫]‬ ‫‪.‬‬ ‫‪τ‬‬ ‫‪τ‬‬ ‫‪.‬‬ ‫‪5‬‬
‫[‬ ‫]‬ ‫[ ‪τ‬‬ ‫]‬
‫‪.‬‬ ‫‪.‬‬ ‫‪.‬‬ ‫حغبة ‪K‬‬
‫‪τ‬‬

‫ألٌ‬ ‫أهٕٖ يٍ األعبط‬ ‫األعبط‬ ‫‪ 6‬هٕح األعبع‪ٍٛ‬‬


‫)‬ ‫(‬
‫‪II‬‬
‫‪ 0‬ـ ث‪ٛ‬شش‬ ‫‪ 1‬ـ عحبحخ‬ ‫انشعى‬
‫‪ 4‬ـ حبيم‬ ‫‪ 2‬ـ يخالغ‬ ‫‪1‬‬
‫انج‪ٛ‬بَبد‬
‫يزش‬ ‫‪ 5‬ـ عٓبص‬
‫‪ 2‬انًؼبدنخ‬
‫‪ 3‬االحذاص‪ٛ‬بد‬
‫‪ 4‬كً‪ٛ‬خ انًبدح‬

‫أ ـ انصلخ‬
‫ْ‪ ٙ‬انـبنجخ ألٌ‬ ‫انصلخ انحًع‪ٛ‬خ‬ ‫‪5‬‬
‫انـبنجخ‬

‫صلحخ ‪ 7‬يٍ ‪10‬‬


‫االجابت الختبار انبكانوريا انتجريبي في مادة‪ :‬انعهوو انفيزيائيت‪//‬انشعبت ‪ :‬رياضياث‪ ،‬تقىي رياضي‪ //‬دورة ‪ :‬أفريم ‪2423‬‬

‫‪τ‬‬ ‫‪.‬‬
‫ْٕ انًزلبػم انًحذ‬

‫[‬ ‫]‬ ‫ة ـ حغبة ‪τ‬‬

‫‪.τ‬‬ ‫َغزُزظ أٌ رلبػم انًؼب‪ٚ‬شح رلبػم ربو‬


‫انًغججخ نزآكم انغخبٌ انًبئ‪ٙ‬‬ ‫انٓ‪ٛ‬ذساص‪ٚ ٍٚ‬زلبػم يغ‬ ‫‪6‬‬

‫انؼاليخ‬
‫ػُبصش اإلعبثخ‬
‫يغضأح يغًٕع‬
‫انتمريه انثانث ‪ 45( :‬وقاط) ‪:‬‬
‫‪I‬‬
‫انًشعغ انٓ‪ٛ‬ه‪ٕٛ‬يشكض٘ ‪ ْٕ :‬يشعغ يجذأِ يشكض انشًظ يحبٔسِ انضالصخ رزغّ‬
‫‪ 1‬انًشعغ‬
‫َحٕ َغٕو ثؼ‪ٛ‬ذح َؼزجشْب عبكُخ ‪.‬‬
‫⃗‬ ‫‪⃗.‬‬ ‫‪.‬‬
‫ثبالعوبغ ػهٗ انًحٕس ‪on‬‬ ‫أ ـ ػجبسح‬
‫‪.‬‬ ‫انغشػخ‬
‫‪.‬‬ ‫صٕاثذ ارٌ‬ ‫ٔ‬
‫ة ـ هبٌَٕ كجهش‬
‫‪.‬‬ ‫انضبنش‬
‫√‬ ‫‪.‬‬
‫ْٕٔ هبٌَٕ كجهش انضبنش ‪.‬‬
‫حغبة ه‪ًٛ‬خ ‪ k‬يٍ‬
‫‪.‬‬
‫كٕكت ػطبسد‬ ‫‪2‬‬

‫√‬ ‫√‬ ‫‪.‬‬ ‫د ـ اكًبل‬


‫انغذٔل‬

‫√‬

‫ط ـ حغبة كزهخ‬
‫‪.‬‬ ‫‪.‬‬
‫انشًظ‬
‫‪II‬‬
‫حغت هبٌَٕ كجهش األٔل يغبس انكٕكت اْه‪ٛ‬ه‪ٛ‬غ‪ٔ ٙ‬انز٘ ركٌٕ انشًظ ك‪ ٙ‬أحذ‬
‫‪ 1‬انزلغ‪ٛ‬ش‬
‫انًحشه‪ٔ ٍٛ‬‬
‫حغت هبٌَٕ كجهش انضبَ‪= ٙ‬‬ ‫‪ 2‬انؼالهخ‬
‫يٍ انشكم (‪)0‬‬
‫‪ 3‬انغشػخ‬

‫صلحخ ‪ 8‬يٍ ‪10‬‬


‫االجابت الختبار انبكانوريا انتجريبي في مادة‪ :‬انعهوو انفيزيائيت‪//‬انشعبت ‪ :‬رياضياث‪ ،‬تقىي رياضي‪ //‬دورة ‪ :‬أفريم ‪2423‬‬
‫‪III‬‬
‫‪ 1‬انزًض‪ٛ‬م‬
‫‪.‬‬
‫‪ 2‬انؼجبسح انؼهً‪ٛ‬خ‬
‫‪.‬‬

‫‪.‬‬
‫‪ 3‬انؼجبسح انج‪ٛ‬بَ‪ٛ‬خ‬

‫‪.‬‬ ‫ثبنًطبثوخ‬ ‫‪ 4‬كزهخ انشًظ‬

‫انؼاليخ‬
‫ػُبصش اإلعبثخ‬
‫يغضأح يغًٕع‬
‫انتمريه انتجريبي ‪ 46( :‬وقاط) ‪:‬‬
‫‪I‬‬
‫ثزطج‪ٛ‬ن هبٌَٕ عًغ انزٕرشاد‬ ‫𝑅‬
‫‪.‬‬ ‫𝑅‬ ‫𝑅‬
‫‪.‬‬ ‫انًؼبدنخ‬
‫‪.‬‬ ‫‪1‬‬
‫نشتق‬
‫انزلبظه‪ٛ‬خ‬
‫‪.‬‬
‫‪.‬‬
‫‪.‬‬

‫‪.‬‬
‫‪ 2‬ػجبسح‬
‫‪.‬‬
‫‪.‬‬ ‫‪.‬‬ ‫أ ـ انزٕرش‬
‫𝜏‬ ‫ثبإلعوبغ‬ ‫𝜏‬ ‫ة ـ انضبثذ 𝜏‬
‫𝜏‬ ‫‪.‬‬ ‫‪ 3‬د ـ انغؼخ‬

‫س ـ انز‪ٛ‬بس‬
‫‪II‬‬
‫ثزطج‪ٛ‬ن هبٌَٕ عًغ انزٕرشاد‬ ‫𝐿÷‬ ‫انًؼبدنخ‬
‫‪1‬‬
‫‪.‬‬ ‫انزلبظه‪ٛ‬خ‬

‫َؼٕض (‪ )0(ٔ )1‬ك‪ ٙ‬انًؼبدنخ‬ ‫‪.‬‬


‫‪.‬‬ ‫ٔ‬ ‫‪ 2‬انضبثز‪ٍٛ‬‬
‫‪.‬‬ ‫‪.‬‬

‫(‬ ‫)‬ ‫(‬ ‫)‬ ‫‪.‬‬

‫صلحخ ‪ 9‬يٍ ‪10‬‬


‫االجابت الختبار انبكانوريا انتجريبي في مادة‪ :‬انعهوو انفيزيائيت‪//‬انشعبت ‪ :‬رياضياث‪ ،‬تقىي رياضي‪ //‬دورة ‪ :‬أفريم ‪2423‬‬

‫‪.‬‬ ‫‪.‬‬ ‫‪ 3‬انؼجبسح‬


‫‪.‬‬ ‫‪.‬‬
‫أ ـ انزار‪ٛ‬خ‬
‫‪.‬‬

‫‪.‬‬ ‫𝜏‬ ‫‪ 4‬ة ـ انضبثذ 𝜏‬

‫) (‬ ‫(‬ ‫‪) .‬‬ ‫د ـ انطبهخ‬

‫صلحخ ‪ 14‬يٍ ‪10‬‬


‫شعار العمل في هذا الموسم ‪:‬‬
‫ِ‬

You might also like